Всі тести з Крок 1 - Медицина
1 / 6854
Хворий на гіпертонічну хворобу, який лікувався гипотиазидом, скаржиться на загальну слабкість, втрату апетиту, серцебиття. Спостерігається гіпотонія м'язів, мляві паралічі, ослаблення перистальтики кишечника. Що може бути причиною такого стану?

Гіперкаліємія

Гіперурикемія

Гіперкальціємія

Гипокалиемия

Гипонатриемия

2 / 6854
При диспансерному обстеженні хлопчику 7 років встановлено діагноз - дальтонізм. Батьки здорові, кольорове зір нормальний. Але у дідуся по материнській лінії така ж аномалія. Який тип успадкування цієї аномалії?

Домінантний, зчеплений з статтю

Рецесивний, зчеплений з статтю

Аутосомно - рецесивний

Неповне домінування

Аутосомно - домінантний

3 / 6854
Методом непрямої калориметрії встановлено, що основний обмін досліджуваного на 40% нижче належного. Порушення діяльності якої ендокринної залози є причиною?

Щитовидна залоза

Надпочечники

Епіфіз

Тимус

Підшлункова залоза

4 / 6854
Проводячи хірургічне втручання на жовчному міхурі лікарю необхідно визначити загальний міхурцевий проток. В якому анатомічному утворенні очеревини він знаходиться?

Lig. Hepatorenale

Lig. Gastrocolicum

Lig. Duodenorenale

Lig. Hepatogastricum

Lig. Hepatoduodenal

5 / 6854
Під час розтину померлого в комі молодого людини виявлені великий тромбоемболічний інфаркт лівого півкулі мозку, велика септична селезінка, імунокомплексний гломерулонефрит, виразки на стулках аортального клапана, покриті поліпоподобнимі тромбами з колоніями стафілококів. Яке забол Еван викликало церебральну тромбоемболію?

Гострий ревматичний вальвулит

Септикопіємія

Ревматичний тромбендокардіт

Септичний бактеріальний ендокардит

Септицемія

6 / 6854
Провідником наукової експедиції по Індії був місцевий житель, який ніколи НЕ розлучався зі своєю собакою. Якими інвазійними захворюваннями можуть бути заражені члени експедиції при контакті з цією собакою, якщо вона є джерелом інвазії?

парагонімоз

дикроцеліоз

фасциолезом

теніозом

ехінококоз

7 / 6854
У хворого камінь загального жовчного протока припинив надходження жовчі в кишечник. Порушення якого процесу травлення при цьому спостерігається?

Всмоктування білків

Переваривание жирів

Всмоктування вуглеводів

Переваривание білків

Переваривание вуглеводів

8 / 6854
Больног про з явищами енцефалопатії госпіталізували в неврологічний стаціонар і виявили кореляцію між наростанням енцефалопатії і речовинами, які надходять з кишечника в загальний кровотік. Які сполуки, що утворюються в кишечнику, можуть викликати ендотоксемії?

Орнитин

Біотин

Індол

Ацетоацетат

Бутират

9 / 6854
При реєстрації ЕКГ хворого з гіперфункцією щитовидної залози зареєстровано збільшення частоти серцевих скорочень. Скорочення якого елемента ЕКГ про це свідчить?

Сегмента PQ

Комплексу QRS

Інтервалу RR

Інтервалу PQ

Інтервалу Р-Т

10 / 6854
Хворий 62 років поступив в неврологічне відділення в зв'язку з мозковим крововиливом. Стан важкий. Спостерігається наростання глибини і частоти дихання, а потім його зменшення до апное, сел ле чого цикл дихальних рухів відновлюється. Який тип дихання виник у хворого?

Гаспінг - дихання

Кусмауля

Апнейстичне

Біота

Чейна - Стокса

11 / 6854
Внаслідок блокади іонних каналів мембрани клітини її потенціал спокою зменшився з -90 мВ до -70 мВ. Які канали заблоковані?

Калієві

Натрієві

кальцієві

Хлорні

Магнієві

12 / 6854
У чоловіка 45 років після значної психоемоційної навантаження несподівано з'явилася стискає біль в області серця з іррадіацією в ліву руку, шию, під ліву лопатку. Особа стало блідим, покрилося холодним потім. Нітрогліцерин зняв напад болю. Який процес розвинувся у хворого?

Психогенний шок

Стенокардія

Перфорація виразки шлунка

Інсульт

Інфаркт міокарда

13 / 6854
На час профогляду у людини, НЕ має скарг на стан здоров'я, виявили лейкоцитоз. Причиною цього може бути те, що кров для аналізу здана після:

Вживання алкоголю

Відпочинку на курорті

Розумової роботи

Значного споживання води

Фізичної навантаження

14 / 6854
У препараті черв ого кісткового мозку людини визначаються скупчення гігантських клітин, що знаходяться в тісному контакті з синусоїдними капілярами. Які формені елементи крові утворюються з цих клітин?

Моноцити

Кров'яні пластинки

Лімфоцити

Лейкоцити

Еритроцити

15 / 6854
У крові пацієнта вміст глюкози натще було 5,65 ммоль / л, через 1 годину після цукрового навантаження становило 8,55 ммоль / л, а через 2 години - 4,95 ммоль / л. Такі показники характерні для:

Хворого з прихованим цукровим діабетом

Здорової людини

Хворого з тиреотоксикозом

Хворого з інсуліннезалежний цукровий діабет

Хворого з інсулінозалежним цукровим діабетом

16 / 6854
Жінка 30 років хворіє близько року, коли вперше з'явилися болі в області суглобів, їх припухлість, почервоніння шкіри над ними. Попередній діагноз - ревматоїдний артрит. Однією з ймовірних причин цього захворювання є зміна в структурі білка сполучної тканини:

овальбумін

муцин

міозин

тропонин

Коллагена

17 / 6854
У результаті безконтрольного прийому вітамінного препарату у дитини з'явилися анорексія, нудота, блювота, пронос, гіпертермія, з'явилися крововиливи на шкірі та слизових, а також явища менингизма. Який препарат приймав дитина?

Соматотропин

Ретинолу ацетат

Рутин

Нікотинамід

Токоферолу ацетат

18 / 6854
У породіллі через 24 години після пологів діагностовано емболія легеневої артерії. З'явилася задишка з частим глибоким диханням. АТ - 90/40 мм рт. ст, частота серцевих скорочень - 99 / хв. Газовий аналіз: рН = 7,48; pC Про 2 = 32 мм рт. ст.; [ НСО 3 ] = 20 ммоль / л ; [BE] = +1 ммоль / л. Яке з перерахованих нижче розладів кислотно - лужного стану має місце в цьому випадку?

Компенсований метаболічний ацидоз

Де компенсований метаболічний ацидоз

Респіраторний алкалоз + метаболічний ацидоз

Компенсований респіраторний алкалоз

Декомпенсований респіраторний алкалоз

19 / 6854
Хворому при подагрі лікар призначив алопуринол. Яке фармакологічне властивість алопуринолу забезпечує терапевтичний ефект в даному випадку?

Прискорення синтезу нуклеїнових кислот

Прискорення катаболізму піримідинових нуклеотидів

Уповільнення реутилізацію піримідинових нуклеотидів

Збільшення швидкості виведення азотовмісних речовин

Конкурентне інгібування ксантиноксидази

20 / 6854
Є одноклітинний організм, що характеризується набором хромосом 2 п = 8, що розмножується безстатевим шляхом. Генетичне різноманітність особин в популяції складе (без урахування мутацій):

1 тип

32 типу

256 типів

128 типів

8 типів

21 / 6854
Під час розтину померлого від поширеного перит ВНІТ в дистальних відділах тонкої кишки виявлено численні виразки овальної форми, розташовані уздовж кишки. Дно виразок чисте, гладке, утворене м'язовою або серозною оболонкою, краї виразок рівні, закруглені. У двох виразках є перфоративні отвори діаметром до 0,5 см. Яке захворювання можна припустити?

Холера

Туберкульоз

Черевний тиф

Висипний тиф

Дизентерія

22 / 6854
З метою серологічної діагностики коклюшу поставлена розгорнута реакція з кашлюковим і паракоклюшная діагностикумами. На дні пробірок, в які було внесено діагностикум з Bordetella parapertussis, утворився зернистий осад. Які антитіла виявила ця реакція?

Антитоксини

Опсоніни

преціпітіни

бактеріолізини

Аглютиніни

23 / 6854
У дитини після перенесеної ангіни різко збільшилися лімфатичні вузли: паратрахеальние, біфуркаційні, шийні. При мікроскопії шийного лімфатичного вузла виявлені вогнища некрозу, обмежені лімфоцитами, епітеліоїдними клітинами і клітинами Пирогова - Лангханса. Яка найбільш ймовірна патологія?

Саркоидоз

риносклероми

Туберкульоз

Сифіліс

Сап

24 / 6854
У працівника хімчистки виявлена жирова дистрофія печінки. Порушення синтезу якого речовини в печінці може привести до даної патології?

холевой кислоти

фосфатидними кислоти

тристеарин

Сечовини

фосфатидилхолін

25 / 6854
У інфекційне відділення поступив дитина 9 років з скаргами на біль в животі, пронос, втрата апетиту. У випорожненнях - еритроцити. Аналіз показав наявність дизентерійної флори. Який препарат краще призначити біль ному?

Норсульфазол

Фталазол

Етазол

Бісептол

фенілсаліцілат

26 / 6854
На розтині жінки 33 років виявлено потовщення стінки шлунка в пілоричному відділі (на розрізі шари стінки помітні) з розростанням щільної білуватої тканини в підслизовому шарі і дрібними тяжами її в м'язовому шарі. Рельєф слизової оболонки збережений, складки ригідні, нерухомі. Яка макроскопічна форма пухлини в даному випадку?

Інфільтративно-виразкова форма

Кіста

Вузол

Виразка

Інфільтрат

27 / 6854
У людини зменшений діурез, гіпернатріємія, гіпокаліємія. Гиперсекреция якого гормону може бути причиною таких змін?

Адреналін

Альдостерон

Передсердний натрійуретичний фактор

Паратгормон

Вазопрессин

28 / 6854
У хворого виявлено порушення прохідності дихальних шляхів на рівні дрібних і середніх бронхів. Які порушення кислотно - лужного рівноваги можна визначити в крові в даному випадку?

Метаболічний алкалоз

Респіраторний ацидоз

Респіраторний алкалоз

Метаболічний ацидоз

29 / 6854
При розтині жінки 40 років, стра дає ревматоїдний артрит, виявлено збільшену щільну селезінку. На розрізі її тканина коричнево - червоного кольору з збільшеними фолікулами, які мають вигляд напівпрозорих сірувато - білуватих зерен. Який з перерахованих патологічних процесів найбільш вірогідний?

глазурного селезінка

Гіаліноз селезінки

порфірний селезінка

Сагова селезінка

Сальна селезінка

30 / 6854
Жінка 58 років. Стан важкий, помутніння свідомості, шкіра суха, очі запалі, ціаноз, запах гнилих яблук з рота. Об'єктивно: глюкоза крові 15,1 ммоль / л, глюкоза сечі 3,5%. Яка найбільш ймовірна причина цього стану?

Гіповалеміческая кома

Анафілактичний шок

Уремічна кома

Гипергликемическая кома

Гипогликемическая кома

31 / 6854
У травматологічний пункт доставлений хворий з пошкодженням м'язів нижніх кінцівок. За рахунок яких клітин можлива репаративна регенерація м'язових волокон і відновлення функції м'язів?

Клітин - сателітів

міофібробласти

міобласти

міоепітеліальние клітин

Фібробластів

32 / 6854
До лікаря - інфекціоніста на прийом прийшов хворий з скаргами на лихоманку, що триває три дні, загальну слабкість, безсоння, погіршення апетиту. Лікар запідозрив черевної тиф. Який метод лабораторної діагностики найбільш доцільно призначити для підтвердження діагнозу?

Виділення гемокультури

Виділення білікультури

Виділення копрокультури

Виділення міелокультури

Виділення урінокультури

33 / 6854
У хворого з хронічно протікає запальним процесом шкіри і підшкірної клітковини виявлено переважання процесів проліф ераціі. Брак якого гормону може призвести до цього?

Кортизон

СТГ

Тироксин

Альдостерон

Інсулін

34 / 6854
Хворий на цукровий діабет вранці натщесерце отримав призначену дозу інсуліну пролонгованої дії. Пропустив черговий прийом їжі і незабаром відчув ознаки гіпоглікемії. Застосування глюкози стан НЕ полегшило. Який препарат необхідно ввести для купірування даного стану?

Гидрокортизон

Преднізолон

Норадреналин

Адреналін

Триамцинолон

35 / 6854
При мікроскопічному дослідженні прямої кишки виявлені великі осередки некрозу слизової оболонки, некротичні маси просякнуті фібрином, утворюючи плівку. Слизова і підслизова оболонка по периферії ділянок некрозу повнокровні, набряклі з крововиливами і лейкоцитарної інфільтрацією. Яке захворювання можна припустити?

Амебіаз

Черевний тиф

Сальмонельоз

Холера

Дизентерія

36 / 6854
Пацієнт звернувся в клініку з скаргами на загальну слабкість, ниючі болі в животі, поганий апетит, з підозрою на жовтяницю. У сироватці крові виявлено 77,3 мкмоль / л загального білірубіну і 70,76 мкмоль / л кон'югованого білірубіну. Який найбільш ймовірний вид жовтяниці?

Механічна жовтяниця

Цироз печінки

Гемолітична жовтяниця

Гострий гепатит

Обтурационная жовтяниця

37 / 6854
При гепатиті, інфаркті міокарда в плазмі крові хворих різко зростає активність аланін - і аспартатамінотрансферази. Які причини зростання активності цих ферментів в крові?

Нестача піридоксину

Збільшення активності ферментів гормонами

Зростання швидкості синтезу амінокислот у тканинах

Збільшення швидкості розпаду амінокислот в тканинах

Пошкодження мембран клітин і вихід ферментів у кров

38 / 6854
Хворому 50 років з метою лікування черевного тифу призначений левоміцетин, але на наступний день стан хворого погіршився, температура піднялася до 39,6 ° С. Чим пояснити погіршення стану хворого?

Алергічної реакцією

Приєднання вторинної інфекції

Дією ендотоксинів збудника

нечутливість збудника до левоміцетину

реінфекцій

39 / 6854
При обстеженні хворого виявлено наступні клінічні прояви: шкірні покриви рожеві, теплі на дотик, сухі, ЧСС - 92 / хв., ЧД - 22 / хв., Температура тіла - 39,2 ° С. Яке співвідношення процесів утворення і віддачі тепла в описаному періоді лихоманки?

Зниження тепловіддачі на фоні незміненої теплопродукції

Теплопродукция нижче тепловіддачі

Посилення теплопродукції без зміни тепловіддачі

Теплопродукция перевищує тепловіддачу

Теплопродукция равн а тепловіддачі

40 / 6854
У спортсмена на старті перед змаганнями відзначається підвищення артеріального тиску і частоти серцевих скорочень. Впливом яких відділів ЦНС можливо пояснити зазначені зміни?

Середнього мозку

довгаста мозку

Кори великих півкуль

Проміжного мозку

Гіпоталамуса

41 / 6854
У експерименті на кролі через 2 тижні після звуження ниркової артерії виявлено збільшення кількості еритроцитів і гемоглобіну в крові внаслідок стимуляції еритропоезу еритропоетинами. Що підсилює утворення еритропоетинів?

Гипоксемия

Гиповолемия

гіпоосмія

Гиперосмия

Гіперкапнія

42 / 6854
Хворий, який три місяці тому отримав травму голови, чує мова, розуміє її, але НЕ може правильно назвати предмет. В який частці кори великих півкуль є ушкодження?

Нижня лобова

Середня лобова

Середня скронева

Верхня скронева

Верхня лобова

43 / 6854
При розподілі клітини досліднику вдалося спостерігати фазу, при якій були відсутні мембрани ядра і ядерце, а центриоли перебували на полюсах клітини. Хромосоми мали вигляд клубка ниток, вільно розташованих в цитоплазмі. Для якої фази це характерно?

Інтерфаз

телофаза

профазі

анафазу

метафазі

44 / 6854
Під час патронажу лікар виявив у дитини симетричну шорсткість щік, діарею, порушення нервової діяльності. Недолік будь харчових факторів є причиною такого стану?

Лізин, аскорбінова кислота

Метіонін, ліпоєва кислота

Нікотинова кислота, триптофан

Треонін, пантотенова кислота

Фенілаланін, пангамовая кислота

45 / 6854
Хворий гіпотонією, відчувши погіршення загального стану прийняв без контролю лікаря кілька таблеток ефедрину через короткі проміжки часу. Але деякий поліпшення спостерігав тільки після прийому першої таблетки. Яким явищем обумовлено таке дію?

Сенсибілізація

Звикання

Ідіосинкразія

Тахіфілаксія

Кумуляция

46 / 6854
У чоловіка 60 років діагностовано інсульт в області латеральних ядер гіпоталамуса. Які зміни поведінки слід очікувати при цьому?

Ненаситність

Відмова від їжі

Агресивність

Депресія

Жага

47 / 6854
У хворого на ЕКГ виявлено збільшення тривалості інтервалу QT. Це може бути наслідком зменшення в шлуночках швидкості:

Деполяризації і реполяризації

деполяризація

Скорочення

Розслаблення

реполяризації

48 / 6854
Хворий скаржиться на болі в шлунку, печію. При огляді виявлено про підвищення кислотності шлункового соку. Що необхідно призначити хворому для нейтралізації кислотності шлункового соку?

Папаверина гідрохлорид

Прозерін

Атропіну сульфат

Бензогексоний

Алмагель

49 / 6854
У хворих на алкоголізм часто спостерігається гіповітаміноз Вх, який є наслідком порушення харчування. Симптомами гіповітамінозу В 1 є розлади нервової системи, психози, втрата пам'яті. Чому до дефіциту вітаміну Вх особливо чутливі клітини нервової тканини?

Порушується окислення жирних кислот

Порушується аеробний розпад глюкози

Знижується інтенсивність гліколізу

Підвищується інтенсивність гліколізу

Посилюється ліполіз жирової тканини

50 / 6854
Чоловік 37 років доставлений в хірургічне відділення з явищами гострого панкреатиту: блювота, пронос, брадикардія, гіпотензія, слабкість, явища зневоднення організму. Який препарат найбільш доцільно використовувати в першу чергу?

Ефедрин

Етаперазін

Але - шпу

Платифиллин

Контрикал

51 / 6854
У тимусі призупинено освіту Т - лімфоцитів - хелперів. Які процеси імуногенезу, що відбуваються в сполучної тканини, будуть порушуватися в першу чергу?

Перетворення В-лімфоцитів в плазматичні клітини

фагоцітірованія чужорідних тіл

опсонізації

фагоцітірованія антигенів макрофагами

Освіти попередників Т-лімфоцитів

52 / 6854
Жінка 52 років, хворіє на цукровий діабет, скаржиться на головний біль, стомлюваність, безсоння. Під час огляду встановлено високий рівень артеріального тиску - 200/100 мм рт. ст. Який препарат найбільш доцільно використовувати д ля швидкого зниження артеріального тиску?

Анаприлин

Празозин

Резерпін

Каптоприл

Папаверин

53 / 6854
Хворому з закритим переломом плечової кістки накладена гіпсова пов'язка. На наступний день з'явилася припухлість, синюшність і похолодання кисті травмованої руки. Про якому розладі периферичного кровопостачання свідчать ці ознаки?

Ішемія

Емболія

Тромбоз

Венозна гіперемія

Артеріальна гіперемія

54 / 6854
До отоларинголога звернувся хворий, у якого при огляді гортані виявлено неповне змикання голосових зв'язок при фонації. Голосова щілина при цьому приймає форму овалу. Функція який м'язи гортані порушена у хворого?

М. thyroaryttenoideus

M. vocalis

M. cricoar yttenoideus posterior

М. cricoaryttenoideus lateralis

М. aryttenoideus transversus

55 / 6854
Лікування дитини, хворої на рахіт, з допомогою вітаміну D 3 НЕ дало позитивного результату. Яка найбільш ймовірна причина неефективності лікування?

Недостатність ліпідів в їжі

Порушення включення вітаміну D3 в фермент

Порушення гідроксилювання вітаміну D3

Підвищений використання вітаміну D3 мікрофлорою кишечника

Порушення транспорту вітаміну D3 білками крові

56 / 6854
У жінки виявлено пухлину яєчника. Показана операція. Яку зв'язку повинен перерізати хірург, щоб відділити яєчник від матки?

Бічну пупкову зв'язку

Власну зв'язку яєчника

Широку зв'язку матки

Круглу зв'язку матки

Зв'язку, підвішуючу яєчник

57 / 6854
У хворого за добу виділяється води з організму менше, ніж надходить. Яке захворювання може привести до такого стану?

Гепатит

Інфекційні хвороби

Панкреатит

Серцева недостатність

Цистит

58 / 6854
У хворого в 2 рази збільшена щитовидна залоза. При пальпації заліза щільна, поверхня нерівномірна, горбиста. При гістологічному дослідженні - дифузна інфільтрація тканин залози лімфоцитами, плазматичними клітинами з утворенням фолікул і посилене розростання сполучної тканини. Який найбільш ймовірний діагноз?

Спорадичний зоб

Зоб Хасімото

Зоб Ріделя

Дифузний токсичний зоб

Ендемічний зоб

59 / 6854
У пацієнтки з постійною гіпоглікемією аналіз крові після введення адреналіну суттєво НЕ змінився. Лікар припустив порушення в печінці. Про зміну який функції печінки може йти мова?

Екскреторної

Глікогендепонірующей

гликолитическому

Холестерінообразующей

Кетогенной

60 / 6854
У результаті черепно - мозкової травми у хворого були виявлені наступні симптоми: інтенційний тремор, дисметрія, адиадохокинез, дизартрія. Яка структура голів ного мозку пошкоджена?

Стріатум

Рухова кора

Мозочок

Блідий куля

Чорне речовина

61 / 6854
У хворого М., 45 років, при аналізі ЕКГ виявлено: ритм синусовий, число передсердних комплексів більше числа шлуночкових комплексів ; прогресуюче подовження інтервалу Р -Q від комплексу до комплексу ; випадання окремих шлуночкових комплексів ; зубці Р і комплекси QRST без змін. Назвіть тип порушень серцевого ритму?

Сіноаурікуляр ная блокада

внутрішньопередсердну блокада

Повна атріовентрикулярна блокада

Атріовентрикулярна блокада I ступеня

Атриовентрикулярная блокада II ступеня

62 / 6854
У лікарню доставлений дворічний дитина з уповільненим розумовим і фізичним розвитком, що страждає частими рвотами після прийому їжі. У сечі визначена фенилпировиноградная кислота. Наслідком порушення якого обміну є дана патологія?

Ліпідного обміну

Водно - сольового обміну

Обміну амінокислот

Фосфорно - кальцієвого обміну

Вуглеводного обміну

63 / 6854
У жінки 42 років, яка перенесла операцію на нирці, після наркозу розвинулися явища рекурарізаціі і припинилося дихання. В якості миорелаксанта був використаний дитилін. Яке засіб найбільш доцільно використовувати для відновлення тонусу м'язів? - Стрихніну нітрат

Прозерін

Галантоміна гідробромід

Плазму крові

кофеі н

64 / 6854
У клініку доставлений дитина 8 років з різаною раною підошви правої ноги. При хірургічної обробці виявлена глибока рана з розтином сухожилля м'язи на підошовної поверхні, ближче до латерального краю стопи. У хворого обмежене підняття латерального краю стопи. Функція який м'язи швидше за все порушена?

M. triceps surae

М. peroneus longus

M. tibialis anterior

М. extensor digitorum longus

М. quadriceps femoris

65 / 6854
Хворий 40 років госпіталізований з скаргами на загальну слабкість, судоми верхніх і нижніх кінцівок, АТ - 160/100 мм рт. ст. Результати дослідження: глюкоза крові - 6,5 ммоль / л, холестерин - 6 ммоль / л, кальцій - 2 ммоль / л, фосфор - 1 ммоль / л, натрій -160 ммоль / л. Сечовиділення - 700 мл в добу. Як перша патологія найбільш імовірно буде причиною такого стану?

гіперпаратиреоїдизмом

Гіперальдостеронізм

Тиреотоксикоз

Гіпоальдостеронізм

Рахіт

66 / 6854
Хворому на туберкульоз, в анамнезі якого була відкрита легенева форма захворювання, проведено мікроскопічне дослідження мокротиння з метою визначення збудника. Який метод фарбування доцільно при цьому використовувати?

Метод Ціля - Нільсона

Метод Нейссера

Метод Бурри - Гінса

Метод Романовського - Гімза

Метод Грама

67 / 6854
При томографії у хворого в середостінні виявлені збільшені лімфатичні вузли. При гістологічному дослідженні в лімфовузлі виявлені циркулярні розростання сполучної тканини, навколишнього гранулемоподобная освіти з лімфоцитів, плазмоцитів і великих двоядерних их клітин Березовського - Штернберга. Який найбільш ймовірний діагноз?

Лімфосаркома

Саркоидоз

Лімфогранулематоз

Туберкульоз

Лімфолейкоз

68 / 6854
У експерименті на постсинаптическую мембрану нейрона подіяли речовиною, яке викликало її гіперполяризацію. Проникність для яких іонів на постсинаптичні мембрані збільшилася в даній ситуації?

Калію

Натрію

Магнію

Кальцію

Марганця

69 / 6854
Вивчення відбитків виступів епідермісу пальців рук (т. Н. Дактилоскопія) використовується в криміналістиці для ідентифікації особи, а також для діагностики генетичних аномалій, в зокрема хвороби Дауна. Який шар шкіри визначає індивідуальність відбитків?

Сосочковий

Сітчастий

Базальний

Блискучий

Роговий

70 / 6854
У інфекційне відділений ие госпіталізовано хворого з скаргами на багаторазовий пронос і блювоту, біль в м'язах ніг, слабкість, запаморочення, ірач поставив попередній діагноз - 'холера'. Як необхідно досліджувати матеріал від хворого для експрес - діагнозу?

РА

Бактеріологічним методом

Біологічним методом

Пряма і непряма РІФ

71 / 6854
Після впливу мутагена в метафазної платівці людини виявлено на три хромосоми менше норми. Зазначена мутація відноситься до:

Політенія

Інверсія

Транслокация

Анеуплоїдія

Полиплоидия

72 / 6854
У лабораторії при мікроскопії мокротиння хворого на пневмонію випадково виявлені личинки. При аналізі крові виявлено еозинофілія. Який гельмінтоз можна припустити?

Трихоцефальоз

Парагонімоз

Аскаридоз

Описторхоз

Ентеробіоз

73 / 6854
Клітку лабораторного тваринного піддали надлишкового рентгенівському опроміненню. В результаті утворилися білкові фрагменти в цитоплазмі. Який органоид клітини прийме участь в їх утилізації?

Ендоплазматичний Ретик лум

Комплекс Гольджі

Лізосоми

Клітинний центр

Рибосоми

74 / 6854
У дівчинки 12 років при розтині виявлені множинні крововиливи в шкірі (переважно сідниць, нижніх кінцівок), серозних і слизових оболонках, в головному мозку. У надниркових залозах - вогнищевий некроз і масивні крововиливи, в нирках - некротичний нефроз, гнійний артрит, іридоцикліт, васкуліт. Який найбільш ймовірний діагноз?

Висипний тиф

Променева хвороба

Вузликовий періартеріїт

Системна червона вовчак

менінгококцемія

75 / 6854
На базарі громадянин А. продавав ковбасу під назвою ' свиняча домашня '. У Держсанінспекцією виникло підозра фальсифікації ковбаси. З допомогою якої серологічної реакції імунітету можна ідентифікувати харчовий продукт?

преципітацією

імунофлуоресценція

РСК

Аглютинації

РНГА

76 / 6854
У породіллі 35 років визначається больовий синдром, пов'язаний з затримкою першого періоду пологів. Який препарат краще за все застосувати для зменшення болю?

Кодеїн

Морфін

Парацетамол

Анальгін

Промедол

77 / 6854
У добовому раціоні дорослої здорової людини повинні бути жири, білки, вуглеводи, вітаміни, мінеральні солі і вода. Вкажіть кількість білка, що забезпечує нормальну життєдіяльність організму.

40-50

70-80

100-120

50-60

10-20

78 / 6854
У хворого виник спазм гладкої мускулатури бронхів. Використання будь активаторів для зняття нападу буде фізіологічно обгрунтованим?

α - адренорецепторів

Н - холінорецепторів

М - холінорецепторів

Β - адреноблокатори

α - і β - адреноблокатори

79 / 6854
Хворому з значними опік ами зробили пересадку донорської шкіри, але на 8- перші добу трансплантат набряк, змінився його колір і на 11 добу він почав відторгатися. Які клітини беруть в цьому участь?

Т - лімфоцити

В - лімфоцити

Еозинофіли

Еритроцити

Базофіли

80 / 6854
У клініку доставлений пацієнт з ознаками гострого алкогольного отруєння. Які зміни вуглеводного обміну характерні для цього стану?

У печінці посилюється глюконеогенез

У м'язах переважає анаеробний розпад глюкози

У печінці посилюється розпад глікогену

В печінці знижується швидкість глюконеогенезу

У м'язах посилюється аеробний розпад глюкози

81 / 6854
При дослідженні гнійних виділень з шийки матки бактериоскопически виявлено наявність грамнегативних бобовидних диплококков, що знаходяться як в середині, так і поза лейкоцитів. Назвіть збудника гнійного запалення шийки матки. -Calymmatobacterium granulomatis

Chlamydia trachomatis

Neisseria gonorrhoeae

Trichomonas vaginalis

Haemophilus vaginalis

82 / 6854
У дитини 7 років на шкірі розгинальних поверхонь Локтєв их і колінних суглобів з'явилися щільні, безболісні вузлики розміром 1-2 мм. У біоптаті вузликів - великий осередок фибриноидного некрозу сполучної тканини з лімфоцитами і макрофагами по периферії. При якому захворюванні спостерігаються такі вузлики?

Ревматоїдний артрит

Системна червона вовчак

Ревматизм

Вузликовий періартеріїт

Склеродермия

83 / 6854
У тварин в експерименті реєструють електричну активність нейронів спірального вузла, що дозволяє аналізувати афферентную імпульсацію від рецепторів:

напівкружними каналів

пристеночно

Кортиєва органу

Вестибулярних

Вестибулярних і Кортиєва органу

84 / 6854
Хвора 25 років звернулася з скаргами на погіршення зору. При огляді виявлено порушення акомодації, зіниця розширена, НЕ реагує на світло. Функція яких м'язів порушена?

звужує і розширює зіницю м'язи

Латеральна пряма м'яз, що звужує зіницю

Верхня коса, ресничная

М'яз, що звужує зіницю, ресничная

М'яз, що розширює зіницю, ресничная

85 / 6854
У клініку доставили пацієнта 32 років з масивною крововтратою внаслідок автодорожньої травми. Пульс 110 уд / хв., Частота дихання - 22 за 2 хв., АТ - 100/60 мм рт. ст. Яке зміна крові з перерахованих буде найбільш характерним через 1 годину після крововтрати?

Гипопротеинемия

Лейкопенія

Гіпохромія еритроцитів

Еритропенія

Гиповолемия

86 / 6854
Під час хірургічної операції пацієнту проведено переливання крові. На антигени якого збудника необхідно перевірити цю кров?

Вірусу гепатиту А

ентеровірус

Вірусу гепатиту Е

Вірусу гепатиту В

аденовіруси

87 / 6854
Пацієнт скаржиться на задишку після фізичної навантаження. Об'єктивно: анемія, наявність парапротеина в зоні гамма - глобулінів. Який показник в сечі необхідно визначити для підтвердження діагнозу мієломи?

Білірубін

Гемоглобін

Церулоплазмін

Білок Бене - Джонса

антитрипсину

88 / 6854
У хворого 45 років на правій нозі спостерігається блідість шкіри гомілки і стопи, визначаться відсутність пульсації тильної артерії стопи та задньої великогомілкової артерії. Пульсація стегнової артерії збережена. Поразка який артерії сталося?

Глибокої артерії стегна

Зовнішньої клубової

Підколінної

Малогомілкової

Низхідній колінної

89 / 6854
Хворий тривалий час приймав глюкокортикоїди. Після різкої відміни препарату скаржиться на міалгію ію, підвищену стомлюваність, емоційну нестабільність, головний біль, безсоння, втрату апетиту, нудоту. Розвинувся синдром відміни глюкокортикоїдів. Призначення яких препаратів показано для корекції даного стану?

Кортикостероїди

Адреналін

АКТГ

Мінералокортикоїди

Глюкокортикоїди

90 / 6854
При обстеженні міміки хворого виявлено, що він НЕ може скласти губи трубочкою, НЕ може свистіти, під час сміху куточки рота НЕ піднімаються вгору, ротова щілина розтягується в боки (попе чная посмішка). Пацієнт страждає на міопатію - спадкове захворювання з дистрофічних пошкодженням м'язів. На атрофію який м'язи вказують дані симптоми?

Великий виличної м'язи

Шийної м'язи

Круговий м'язи рота

М'язи сміху

Жувальною м'язи

91 / 6854
У експерименті на тварині видалення ділянки кори півкуль мозку усунуло раніше вироблені умовні рефлекси на світлове роздратування. Який ділянку кори був видалений?

Потилична кора

Постцентральна звивина

прецентральная звивина

Скронева частка

Лімбічна кора

92 / 6854
У чоловіка 36 років з черепно - мозкової травмою дихання слабке, пульс ніткообразний, рефлекси відсутні. Який шлях введення пірацетаму найбільш доцільний в даному випадку?

Інгаляційний

Підшкірний

Внутрішньовенний

Пероральний

Ректальний

93 / 6854
У хворого інфаркт передньої стінки лівого шлуночка. У басейні якого судини виникло порушення кровообігу?

огинає гілок і лівої вінцевої артерії

Предсердно - шлуночкових гілок правої вінцевої артерії

Передньої міжшлуночкової гілки лівої вінцевої артерії

Лівою крайової гілки лівої вінцевої артерії

Задній міжшлуночкової гілки правої вінцевої артерії

94 / 6854
Юнак 16 років, що страждає на цукровий діабет з 10 років, почув про можливості замінити ін'єкції інсуліну таблетками глібенкламіду. Однак лікар, до якого він звернувся, категорично йому відмовив. Чому глібенкламід не можна призначити в даному випадку?

Швидко деградує в печінці

Стимулює альфа - клітини

Викликає гіперпродукцію гідрокортизону

Підсилює виділення адреналіну

Чи не стимулює бета - клітини

95 / 6854
У хворого на туберкульоз після тривалого лікування з'явилися шум і дзвін у вухах, зниження слуху, висипання на шкірі, набряк слизових оболонок і порушилася координація рухів. Після скасування препарату стан хворого значно покращився. Який препарат приймав хворий?

Бепаск

Рифампіцин

Ізоніазид

Етамбутол

Стрептомицина сульфат

96 / 6854
При обстеженні у хворого виявлено підвищений вміст ліпопротеїнів низької щільності в сироватці крові. Яке захворювання можна очікувати у цього хворого?

Гострий панкреатит

Атеросклероз

Пошкодження нирок

Гастрит

Запалення легенів

97 / 6854
При черговому нападі бронхіальної астми хворий з допомогою інгалятора вдихнув препарат, який в вигляді інгаляцій використовується як бронхолитик, а в ін'єкціях - в акушерській практиці для попередження викиднів. Який препарат був використаний хворим?

Еуфілін

Ефедрин

Сальбутамол

Адреналін

Фенотерол

98 / 6854
На електронній мікрофотографії біопсійного матеріалу представлено легке недоношеної дитини. Виявлено спадання стінки альвеол з - за відсутності сурфактанту. Порушення функції яких клітин стінки альвеоли обумовлюють дану картину?

Секр еторних клітин

альвеолоцитами I типу

Альвеолярних макрофагів

Альвеолоцитами II типу

Фібробластів

99 / 6854
У здорового дорослого людини проводять зондування порожнин серця і великих судин. Де знаходиться зонд, якщо на протязі серцевого циклу зареєстровані зміни тиску від 0 до 120 мм рт. ст?

Аорта

Передсердя

Легенева артерія

Правий шлуночок

Лівий шлуночок

100 / 6854
Пацієнту, який проживає на специфічній геохимической території, поставлений діагноз - ендемічний зоб. Який вид посттрансляционной модифікації тиреоглобуліну порушений в організмі хворого?

Глікозилювання

Фосфорилування

Йодування

Метилирование

Ацетилювання

101 / 6854
У хворого діагностовано гострий інфаркт міокарда, що супроводжується стійкими болями за грудиною. Неефективність раніше введених препаратів дала підставу вр Ачу провести нейролептанальгезию. Який нейролептик треба використовувати?

Резерпін

метеразін

Аміназин

Галоперидол

Дроперидол

102 / 6854
Дитина 5 років поступив в ЛОР - відділення з діагнозом гнійне запалення середнього вуха. Захворювання почалося з запалення носоглотки. Через якийсь канал скроневої кістки інфекція потрапила в барабанну порожнину?

Мишечнотрубний канал

канадець барабанної струни

Сонний канал

Барабанний каналець

Сонно - барабанні канальці

103 / 6854
У дівчинки 5 років спостерігається висока температура і біль в горлі. Об'єктивно: набряк м'якого піднебіння, на мигдалинах сірі плівки, які важко відділяються, залишаючи глибокі кровоточать дефекти тканини. Яке з нижчеперелічених захворювань найбільш ймовірно?

Ангіна Симонівського - Венсана

Лакунарна ангіна

Дифтерія зіва

Некротична ангіна

Інфекційний мононуклеоз

104 / 6854
При дисбактеріозах, що супроводжуються процесом гниття (протея, псевдомонад) і підвищенням рН фекалій, необхідно призначати біологічні препарати, Подкисляющие середу і проявляють антагоністичну дію. Які мікроорганізми для цього підходять?

серрацій

Клебсієли

Біфідумбактеріі

азотобактер

ентеробактерій

105 / 6854
Хворий скаржиться на загальну слабкість, головний біль, нудоту, блювоту, рідкі випорожнення з домішкою слизу і крові. При мікроскопії дуоденального вмісту і при дослідженні свіжих фекалій виявлено рухомі личинки. Поставте діагноз.

Стронгілоїдоз

Ентеробіоз

Анкілостомоз

дракункульоз

Трихоцефальоз

106 / 6854
В клініку потрапив чоловік 54 років з скаргами на болі в правій підреберній області, блювоту з кров'ю. Об'єктивно: збільшення розмірів печінки, варикозне розширення вен стравоходу і шлунка, кровотечі з них. Порушення функції якого судини найімовірніше за все мало місце?

Aorta abdominalis

Vena hepatica

Vena cava superior

Vena porta,

Vena cava inferior

107 / 6854
Хворий помер від інфаркту міокарда. При патогистологическом дослідженні міокарда виявлені значні контрактурной зміни в кардіоміоцитах. Це обумовлено нагромадженням в кардіоміоцитах іонів:

Хлора

Кальцію

Натрію

Водню

Магнію

108 / 6854
Однорічний дитина відстає в розумовому розвитку від своїх однолітків. За вранці: блювота, судоми, втрата свідомості. У крові - гіпоглікемія натще. З дефектом якого ферменту це пов'язано?

аргінази

лактози

Сахарози

глікогенсінтази

фосфорилазу

109 / 6854
З метою аналгезії можуть бути використані речовини, що імітують ефекти морфіну, але продукуються в ЦНС. Вкажіть їх.

Окситоцин

Вазопрессин

Соматоліберин

Β - ендорфін

Кальцитонин

110 / 6854
Хворий звернувся з скаргами на порушення зору, що супроводжується опущеними століття, неможливістю повернути око вгору і до середини. Об'єктивно: очей відведений назовні, зіниця розширена і НЕ реагує на світло, хворий НЕ бачить близько. Який нерв пошкоджений?

Окоруховий

Зоровий

блоковий

Трійчастого

Відвідний

111 / 6854
Через кілька хвилин після введення препарату хворому на правець з'явилася задишка, частий пульс, зниження артеріального тиску. Який препарат міг бути найбільш імовірною причиною виниклого ускладнення?

Донорський гамма-глобулін

Антитоксична сироватка

Анатоксин

Антибіотик

Сульфаніламід

112 / 6854
При розтині померлого, що страждав від атеросклерозу, в головному мозку виявлено тромбоз гілки внутрішньої сонної артерії і сірого кольору вогнище вологого розм'якшення тканин. Який патологічний процес виявлений в головному мозку?

Геморагічна інфільтрація

Ішемічний інфаркт

Енцефаліт

Пухлина мозку

Гематома

113 / 6854
При моделюванні запалення нижньої кінцівки у тварини підвищилася температура тіла, збільшилася зміст антитіл та лейкоцитів у крові. Які речовини зумовили розвиток цих загальних реакцій організму при запаленні?

Мінералокортикоїди

Глюкокортикоїди

Інтерлейкіни

соматомедину

Лейкотрієни

114 / 6854
Потерпілий доставлений в клініку з відкритим переломом гілки нижньої щелепи, з великим кровотечею в області перелому. Пошкодження який артерії найімовірніше за все мало місце?

Висхідною піднебінної

Осередковою нижньої

Мовне

Середній скроневій

Особовий

115 / 6854
У потерпілого - різана рана шиї, розташована уздовж заднього краю m. sternocleidomastoideus, ослаблення чутливості шкіри задньої поверхні вушної раковини. Який нерв може бути пошкодженим?

N. auricularis magnus

nn. auriculares anteriores

п. auricularis posterior

г. auricularis nervi vagi

n. auriculotemporalis

116 / 6854
У хворого з гострим ринітом виявлена гіперемія і підвищене утворення слизу в носовій порожнині. Активність яких клітин епітелію слизової оболонки підвищена?

Бокаловидних

Мікроворсінчатих

Базальних

ресничном

Ендокринних

117 / 6854
Хвора звернулася в травмпункт по приводу нагноєння різаної рани. Лікар для очищення рани від гнійних виділень промив її 3% розчином перекису водню. При цьому піна НЕ утворилася. З чим пов'язано відсутність дії препарату?

Спадкова недостатність фосфатдегідрогенази еритроцитів

Спадкова недостатність каталази

Неглибока рана

Наявність в рані гнійного вмісту

Низька концентрація Н 202

118 / 6854
В результаті виснажливої м'язової роботи у робочого значно зменшилася буферна ємність крові. Надходженням якого кислого речовини в кров можна пояснити це явище?

1,3- бісфосфогліцерата

α - кетоглутарата

Лактату

3- фосфогліцерата

піруват

119 / 6854
Хворий безконтрольно приймав дігоксин, що викликало порушення діяльності серця: зниження скоротливості, брадикардію, шлуночкову екстрасистолію. Яке з перерахованих речовин є препаратом вибору для купірування виникли порушень?

Анаприлин

Новокаинамид

Калію хлорид

етмозін

Верапаміл

120 / 6854
У дитини, яка народилася в пізньому шлюбі, малий зріст, відставання в розумовому розвитку, товстий ' географічний ' язик, вузькі очні щілини, плоске обличчя з широкими вилицями. Яка найбільш ймовірна причина розвитку описаного синдрому?

Внутрішньоутробна інтоксикація

Внутрішньоутробна інфекція

Родова травма

Спадкова хромосомна патологія

Внутрішньоутробний імунний конфлікт

121 / 6854
В приймальне відділення доставлений хворий з скаргами на сухість у роті, світлобоязнь та порушення зо ня. Шкіра гіперемована, суха, зіниці розширені, тахікардія. При подальшому обстеженні був встановлений діагноз: отруєння алкалоїдами беладони. Яке з лікарських засобів доцільно застосувати?

Армін

дипироксим

Прозерін

Пілокарпін

Діазепам

122 / 6854
Хворий 50 років скаржиться на спрагу, п'є багато води, виражена поліурія. Глюкоза крові 4,8 ммоль / л, в сечі глюкози і ацетонових тіл немає, сеча безбарвна, питома вага 1,002-1,004. Яка причина полиурии?

Альдостеронізм

Інсулінова недостатність

Недолік АДГ

Тиреотоксикоз

Гіпотиреоз

123 / 6854
У хворого визначена хворобливість по ходу великих нервових стовбурів і збільшене зміст пірувату в крові. Недолік якого вітаміну може викликати такі зміни?

РР

Пантотенова кислота

B1

Біотин

В2

124 / 6854
Юнак 16 років скаржиться на свербіж між пальцями рук і на животі, який посилюється вночі. При огляді на шкірі були виявлені тоненькі смуги сірого кольору і дрібна висипка. Який найбільш ймовірний збудник цієї хвороби?

Ixodes persulcatus

Ornithodorus papillipes

Sarcoptes scabiei

Dermacentor pictus

Ixodes ricinus

125 / 6854
Проводять дослідження на ізольованій порушуємо клітці. Встановлено, що поріг сили подразнення клітини суттєво зменшився. Що з зазначеного може бути причиною цього?

Інактивація натрієвих каналів мембрани

Інактивація кальцієвих каналів мембрани

Блокада енергоутворення в клітині

Активація калієвих каналів мембрани

Активація натрієвих каналів мембрани

126 / 6854
Пацієнту з гострим інфарктом міокарда внутрішньовенно крапельно введено 1500 мл різних розчинів в протягом 8 годин, кисень інтраназально. Смерть наступила від набряку легенів. Що стало причиною набряку легенів?

Інгаляція кисню

Алергічна реакція

Нейрогенная реакція

Зменшення онкотичного тиску за рахунок гемодилюції

Перевантаження лівого шлуночка об'ємом

127 / 6854
У хворого через 12 годин після гострого нападу загрудинної болю визначено різке підвищення активності АсАТ в сироватці крові. Яка з перерахованих патологій буде найбільш імовірною?

Цукровий діабет

Інфаркт міокарда

Нецукровий діабет

колагенози

Вірусний гепатит

128 / 6854
Хворому з прогресуючою м'язовою дистрофією було проведено біохімічне дослідження сечі. Поява якого речовини в великому кількості в сечі може підтвердити захворювання м'язів у даного хворого?

Гіппурова кислота

Креатин

Сечовина

Креатинін

Порфірини

129 / 6854
У жінки з первинним гіперпаратиреоїдизмом періодично повторюються напади ниркової коліки. Ультразвукове обстеження показало наявність дрібних каменів в нирках. Яка найбільш ймовірна причина освіти цих каменів?

Гіперфосфатемія

Гіперурикемія

Гиперкальциемия

Гіперкаліємія

Гіперхолестеринемія

130 / 6854
У дитини з розумовою відсталістю виявлено зелена забарвлення сечі після додавання 5% розчину FeCl 3. Про порушення обміну якої амінокислоти свідчить позитивний результат цієї діагностичної проби?

Тирозину

Глутаміну

фенілаланін

Аргініну

Триптофану

131 / 6854
При бактеріологічному дослідженні випорожнень чотиримісячної дитини з симптомами гострої кишкової інфекції в середовищі Ендо виросли в великому кількості червоні колонії. Які це можуть бути мікроорганізми?

Шигели

Сальмонели

Ешерихії

Стафілококи

Стрептококи

132 / 6854
Велика частина навчаючи стніков експедиції Магеллана в Америку загинула від захворювання авітамінозом, яке виявлялося загальної слабкістю, підшкірними крововиливами, випаданнями зубів, кровотечами з ясен. Вкажіть назву цього авітамінозу.

Анемія Бирмера

Рахіт

Скорбут (цинга).

Поліневрит (бері - бері)

Пелагра

133 / 6854
У 40- річного чоловіка з стенозирующим (без метастазів) раком стравоходу виявляються наступні зміни: атрофія скелетних м'язів, жирової клітковини. Шкіра землисто - коричневої забарвлення, епідерміс витончений, серце зменшено по розмірам. Міокард і печінку бурого кольору. Яка найбільш ймовірна причина такого стану?

Алиментарная кахексія

Міастенія

Ракова кахексія

Хвороба Аддісона

Бура атрофія

134 / 6854
При дослідженні каріотипу у пацієнта були виявлені два типи клітин в однаковому кількості з хромосомними наборами 46XY і 47XXY. Який найбільш ймовірний діагноз?

Синдром Патау

Синдром Дауна

Синдром Клайнфельтера

Нормальний каріотип

Моносомія - Х

135 / 6854
У чоловіка 44 років, мисливця на ондатр, підвищилася температура тіла до 38 ° С, з'явилися головний біль, набряк повік, гіперемія кон'юнктив, на шкірі в області шиї утворилася неглибока виразка. Вкажіть ймовірного збудника захворювання.

Francisella tularensis

Leptospira interrogans

Yersinia pseudotuberculosis

Bacillus anthracis

Brucella suis

136 / 6854
Чоловікові 35 років з виразковою хворобою зроблено резекцію антрального відділу шлунка. Секреція якого гастроінтестинального гормону внаслідок операції буде порушена найбільш за все?

Нейротензин

Холецистокінін

Гістамін

Гастрін

Сек ретин

137 / 6854
Під час розтину в верхній частці правого легкого виявлений великий клиноподібний осередок темно - червоною щільною тканини. Під час гістологічного дослідження в ній виявлений некроз стінок альвеол, просвіт альвеол щільно заповнений еритроцитами. Який процес розвинувся в легенях?

Ателектаз легенів

Геморагічний інфаркт легень

Крововилив в легені

карніфікація легких

Гангрена легенів

138 / 6854
Хворий 2 роки тому переніс операцію резекції пілоричного відділу шлунка. Спостерігається слабкість, періодичне поява темних кіл під очима, задишка. В аналізі крові: НЬ - 70 г / л, ер. - 3,0 * 1012 / л, ц. п. - 0,7. Які зміни еритроцитів в мазках крові найбільш характерні для даного стану?

мікроцітов

шізоціти

Овалоціти

мегалоціти

Мак РОЦИТ

139 / 6854
При обстеженні чоловіка 45 років, який перебував тривалий час на рослинній дієті, виявлено негативний азотистий баланс. Яка особливість раціону стала причиною цього явища? - Недостатнє кількість жирів і білків

Надмірне кількість води

Недостатнє кількість білків

Надмірне кількість вуглеводів

Недостатнє кількість жирів

140 / 6854
Куди потрібно провести катетер для забору лімфи з грудного лімфатичного протока?

В нижню порожнисту вену

В лівий венозний кут

У правий венозний кут

У ліву пахову вену

У верхню порожнисту вену

141 / 6854
У хворого 27 років виявлені патологічні зміни печінки і головного мозку. У плазмі крові виявлено різке зниження, а в сечі збільшення вмісту міді. Поставлено діагноз - хвороба Вільсона. Активність якого ферменту в сироватці крові необхідно дослідити для підтвердження діагнозу?

лейцінамінопептідази

Карбоангідрази

ксантиноксидазу

Церулоплазміну

Алкогольдегідрогенази

142 / 6854
У собаки в експериментальних НТЕ дратували на шиї периферичний відрізок блукаючого нерва, при цьому спостерігали такі зміни серцевої діяльності:

Збільшення збудливості міокарда

Зменшення частоти скорочень

Збільшення атріовентрикулярного проведення

Збільшення сили скорочень

Збільшення частоти і сили скорочень

143 / 6854
Хворому виконана трансплантація рогівки. Які особливості будови рогівки дозволяють сподіватися на її приживлення, а НЕ відторгнення?

Надмірна іннервація

Наявність сполучної тканини

Відсутність кровоносних і типових лімфатичних судин

Наявність багатошарового переднього епітелію

Наявність одношарового плоского епітелію

144 / 6854
У хворої на бронхіальну астму вірусне інфікування спровокувало астматичний статус зі смертельним результатом. Під час г істологіческого дослідження легких виявлено спазм і набряк бронхіол, в їх стінках виражена інфільтрація лімфоцитами, еозинофілами і іншими лейкоцитами, а також дегрануляция лаброцитов. Який механізм гіперчутливості лежить в основі описаних змін?

Запальний

Імунокомплексний

Іммуннообусловленний клітинний цитоліз

Реагінового реакція

Аутоімунний

145 / 6854
У жінки напередодні пологів ШОЕ 40 мм / год. Така величина ШОЕ обумовлена тим, що в крові підвищений вміст:

Фібриногену

Еритроцитів

альбумін

ліпопротеїнів

Білків

146 / 6854
У хворого 42 років при дослідженні периферичної крові виявлено: гемоглобін 80 г / л, еритроцитів 3,2 * 1012 / л, лейкоцитів 25 * 109 / л ; лейкоцитарна формула: базофілів - 5%, еозинофілів - 9%, мієлобластів - 3%, промиелоцитов - 8%; нейтрофілів: мієлоцитів -11%, метамиелоцитов - 22%, паличкоядерних - 17%, сегментоядерних - 19%, лімфоцитів - 3%, моноцитів - 3%. Яка патологія крові найбільш імовірна у хворого?

Еритромієлоз

промієлоцитарному лейкоз

Хронічний мієлолейкоз

панміелофтіз

Мієлобластний лейкоз

147 / 6854
У хворого 17 років інтраопераційно на нижній поверхні печінки виявлена пухлина з розмірами 4,5x5,0x3,5 см з субсерозной локалізацією, темно - червоного кольору, на розрізі представлена поло ня зі значним вмістом крові. Який попередній діагноз?

Гемангіоендотеліома

Капілярна гемангіома

Лимфангиома

Кавернозная гемангіома

Гемангіоперицитома

148 / 6854
Після травми передньої поверхні верхньої третини передпліччя у хворого спостерігається утруднення пронации, ослаблення ладонного згинання кисті і порушення чутливості шкіри 1-3 пальців на долоні. Який нерв пошкоджений?

П. medianus

п. cutaneus antebrachii medialis

п. ulnaris

п. musculocutaneus

п. radialis

149 / 6854
У жінки 63 років є ознаки ревматоїдного артриту. Підвищення рівня якого з нижчеперелічених показників крові буде найбільш значущим для підтвердження діагнозу?

ліпопротеїди

Кислого фосфатази

Сумарних гликозаминогликанов

R-глікозідази

Спільного холестерину

150 / 6854
На аутопсії померлої, яка тривалий час хворіла на цистит і дискінезією сечоводів, виявлені морфологічні ознаки уремії. Нирка була нерівномірно рубцево - зморщена. В просвіті мисок - дрібні уратні камені і пісок. Гістологічно виявлено ' щитовидну нирку ', осередки інтерстіціонного запалення. Який з нижчеперелічених діагнозів є найбільш імовірним?

Хронічний пієлонефрит

атеросклеротичні зморщена нирка

амілоїдні зморщена нирка

Первинно зморщена нирка

Гострий пієлонефрит

151 / 6854
У хворого на глаукому спостерігається підвищення внутрішньоочного тиску при нормальній секреції водянистої вологи циліарним тілом. З пошкодженням яких структур стінки очного яблука пов'язане порушення відтоку рідини з передньої камери?

Заднього епітелію рогівки

ресничном м'язи

цилиарном тіла

Венозного синуса

Судинної оболонки

152 / 6854
Зменшення кровопостачання органів обумовлює розвиток гіпоксії, яка активізує функцію фібробластів. Обсяг яких елементів збільшується в цій ситуації?

Нервових елементів

Судин мікроциркуляторного русла

Міжклітинних речовини

Паренхіматозних елементів органу

Лімфатичних судин

153 / 6854
У хворого на цукровий діабет змінилося значення рН крові і стало рівним 7,3. Визначення компонентів який буферної системи використовується для діагностики розладів кислотно - лужного рівноваги?

фосфатні

гемоглобінового

Білкової

бікарбонатному

Оксигемоглобіновой

154 / 6854
Згідно моделі подвійної спіралі ДНК, запропонованої Уотсоном і Криком, було встановлено, що одна з ланцюгів зберігається при реплікації, а друга синтезується комплементарно першої. Як називається цей спосіб реплікації?

Ідентичний

Дисперсний

Аналогічний

Консервативний

напівконсервативним

155 / 6854
Підліток 15 років скаржиться на брак повітря, загальну слабкість, серцебиття. ЧСС -130 уд./ хв., АТ -100/60 мм рт. ст, на ЕКГ комплекс QRS нормальної форми і тривалості. Число зубців Р і шлуночкових комплексів однакове, зубець Т слі т з зубцем Р. Яка аритмія серця спостерігається у підлітка?

Мерехтіння передсердь

Передсердна пароксизмальна тахікардія

Тремтіння передсердь

Синусова екстрасистолія

Синусова тахікардія

156 / 6854
При мікроскопічному дослідженні пухлини верхньої губи виявлено, що вона побудована з численних щілиноподібних порожнин. Стінки порожнин встелені плоским ендотелієм. Порожнини заповнені рідкою кров'ю і згустками. Поставте діагноз.

гломуса - ангіома

Гемангіоперицитома

Кавернозная гемангіома

Капілярна гемангіома

Венозна гемангіома

157 / 6854
Пацієнт скаржиться на запаморочення, відчуття спраги, ускладнене ковтання, погане бачення близьких предметів. Об'єктивно: часте дихання, розширені зіниці, загальне збудження, балакучість, але мова малозрозумілий. АТ - 110 /70 мм рт. ст., пульс 110 хв. Про передозуванні якого препарату можуть свідчити зазначені симптоми?

Аміназину

Атропіну

Кофеїн

Морфін

Ефедрину

158 / 6854
У дитини, який одужує після кору, розвинулася пневмонія, викликана умовно - патогенними бактеріями. Яка найбільш ймовірна форма цієї інфекції?

Госпітальна інфекція

Персистирующая інфекція

Вторинна інфекція '

Реінфекція

Суперінфекція

159 / 6854
У чоловіка, при житті страждав бронхоектатичної хворобою, пневмосклерозом з вираженими явищами кахексії, на розтині серце зменшене в розмірах, стінки стоншені, в'ялої консистенції, на розрізі тканина бурого кольору. Відкладення якого пігменту спостерігається в міокарді?

Ліпофусцин

Ліпохроми

Гемосидерин

Гематоїдин

Цероїд

160 / 6854
В гастроентерологічне відділення потрапив хворий 57 років з підозрою на синдром Золлінгера - Еллісона, про що свідчило різке збільшення рівня гастрину в сироватці крові. Яке порушення секреторної функції шлунка найбільш імовірно?

Гиперсекреция гіперацидних

Гіпосекреція гіпоацидного

Гиперсекреция гіпоацидного

Гіпосекреція гіперацидних

Ахілія

161 / 6854
В інфекційне відділення лікарні госпіталізовано хворого з діагнозом бактеріальної дизентерії. Лабораторними дослідженнями встановлено, що збудник чутливий до багатьох протимікробних засобів, але у хворого виявлені явища анемії. Який препарат протипоказаний хворому?

Фталазол

Ентеросептол

Ампіцилін

Фуразолідон

Левоміцетин

162 / 6854
Мати виявила у 5 річної дочки на періанальних складках білих ' черв'ячків ',' які викликали у дівчинки свербіж і занепокоєння, і доставила їх в лабораторію. Виявлено білі гельмінти 0,5-1 см завдовжки, ниткоподібної форми з загостреними кінцями, у деяких вони закручені. Який діагноз можна поставити?

Аскаридоз

Дифиллоботриоз

Опісторхоз

Теніоз

Ентеробіоз

163 / 6854
Який механізм тепловіддачі найбільш ефективно спрацьовує при перебуванні людини в умовах 80% вологості повітря і температурі навколишнього середовища +35°С?

Теплопровідність

Радіація

Конвекція

Потовиділення

164 / 6854
У жінки 68 років після інсульту відсутні рухи в верхній і нижній правій кінцівках. Тонус м'язів цих кінцівок і рефлексів у них збільшені. Є патологічні рефлекси. Яка це форма паралічу?

Дисоціація

Тетраплегія

моноплегии

параплена Егія

Геміплегія

165 / 6854
Для гістологічного дослідження доставлена віддалена на операції матка. Під слизової оболонкою визначені численні округлої форми вузли, чітко відокремлені від навколишнього тканини. Мікроскопічно пухлина побудована з пучків гладкої мускулатури, з явищами тканинного атипизма. Який найбільш ймовірний діагноз?

Хоріонепітеліома

Лейоміосаркома

Лейоміома

Рак матки

Фіброміоми

166 / 6854
В інфекційну лікарню поступив хворий з періодично повторюється гарячкою. У препараті крові (товста крапля), пофарбованому по методу Романовського - Гімза, виявлені спіралеподібні мікроорганізми з гострими кінцями синьо - фіолетового кольору. Який збудник виявлено?

Черевного тифу

Поворотного тифу

Малярії

лептоспіроз

висипного тифу

167 / 6854
В травмпункт звернувся чоловік 45 років після побутової травми плеча. Об'єктивно: відсутні функції розгинання, привиди і пронації плеча. Пошкодження який м'язи викликало таке стан?

Надосной м'язи

підлопаткова м'язи

Великий круглої м'язи

підоснов м'язи

Малої круглої м'язи

168 / 6854
Людина отримав електротравму. При цьому струм пройшов через серцевий м'яз. Які небезпечні порушення в роботі серця, що вимагають невідкладного втручання, можуть виникнути в цій ситуації?

Екстрасистолія

Тахікардія

Фібриляція передсердь

Фибрилляция шлуночків

Брадикардія

169 / 6854
В сім'ї росте донька 14 років, у якої спостерігаються деякі відхилення від норми: зріст її нижче, ніж у одноліток, відсутні ознаки статевого дозрівання, шия дуже коротка, плечі широкі, інтелект в нормі. Яке захворювання можна припустити?

Синдром Едвардса

Синдром Клайнфельтера

Синдром Шерешевського - Тернера

Синдром Дауна

Синдром Патау

170 / 6854
В приймальне відділення доставлений чоловік у непритомному зі стоянні. Об'єктивно: на зовнішні подразники хворий НЕ реагує, дихання періодичне по типу Чейн - Стокса, зіниці звужені, зіничний рефлекс відсутній. Було встановлено, що дані симптоми обумовлені використанням морфіну. Призначити антидотную терапію.

Кальцію хлорид

Налоксон

Протаміну сульфат

Унітіол

апоморфін гідрохлорид

171 / 6854
У хворої 43- х років на тлі септичного шоку відзначається тромбоцитопенія, зменшення фібриногену, поява в крові продуктів деградації фібрину, петехіальні крововиливи. Яка найбільш ймовірна причина даних змін?

Екзогенна інтоксикація

Геморагічний діатез

Порушення вироблення тромбоцитів

ДВС - синдром

Аутоіммунна тромбоцитопенія

172 / 6854
У хворого похилого віку спостерігали збільшення і потовщені ення пальців, кистей, стоп, носа і нижньої щелепи. З збільшенням виділення якого гормону пов'язані зазначені порушення?

Інсуліну

Тиреотропіну

Соматотропіну

Паратгормону

адренокортикотропіну

173 / 6854
При огляді 3 річного дитини виявлено порушення звапніння кісток, деформацію хребта і грудної клітини, викривлення нижніх кінцівок. Який препарат є найбільш ефективним в даній ситуації?

Адіурекрин

Тіаміну бромід

Токоферол

Ергокальциферол

Глюкоза

174 / 6854
В гістологічному препараті стінки серця між ендокардит і міокардом визначаються великі клітини зі світлою цитоплазмою і ексцентрично розміщеним ядром. Які це клітини?

Ендокринні клітини

Скоротливі кардіоміоцити

Клітини Пуркіньє

липоцитов

Пейсмейкерови клітини

175 / 6854
Хворий на ішемічну хворобу серця НЕ повідомив лікаря, що у нього бувають напади бронхоспазмов. Лікар призначив препарат, після прийому якого напади стенокардії стали рідше, але почастішали напади бронхоспазму. Вкажіть, який препарат був призначений.

Анаприлин

Нитросорбид

Атенолол

Дилтиазем

Верапаміл

176 / 6854
Хворому проведена субтотальна субфасціальна резекція щитовидної залози. В післяопераційному періоді тривалий час зберігається охриплість голосу. Який нерв пошкоджений в ході операції?

Нижньощелепний нерв

Під'язиковий нерв

Поворотний гортанний нерв

Верхній гортанний нерв

Язичний нерв

177 / 6854
Хворий 45 років госпіталізований в хірургічне відділення з скаргами на раптову гостру біль в надчеревній області. Після обстеження поставлений діагноз: перфоративная (проривна) виразка задньої стінки шлунка. Куди вилилося вміст шлунка в момент перфорації?

У правий брижових синус

У передшлункову сумку

В сальниковую сумку

В лівий брижових синус

В печінкову сумку

178 / 6854
У чоловіка 60 років після крововиливу в головний мозок настав тривалий сон. Пошкодження яких структур найбільш ймовірно призвело до цього стану?

гіпокамп

Ретикулярної формації

Кори великих півкуль

ядер черепних нервів

Чорної субстанції

179 / 6854
В культурі тканин ядерним випромінюванням пошкоджені ядерця ядер. Оновлення яких органел в цитоплазмі клітин стає проблематичним?

Лізосом

Рибосом

Комплексу Гольджі

Мікротрубочок

ендоплазматичний сітки

180 / 6854
При відтворенні артеріальної гіпертензії у собаки через 1 місяць товщина стінки лівого шлуночка збільшилася в 1,7 рази, а обсяг циркулюючої крові НЕ змінився порівняно з вихідними даними. Яка стадія гіпертрофії міокарда спостерігається у тварини?

Початкова

Декомпенсації

Аварійна

Закінченої гіпертензія рофіі

Прогресуючого кардіосклерозу

181 / 6854
В біоптаті бронха хворого, зловживає курінням, в потовщеною слизової оболонки виявлено хронічне запалення і трансформацію одношарового війчастого епітелію в багатошаровий плоский епітелій. Який з процесів найбільш вірогідний?

Гіпертрофія епітелію

Гіперплазія епітелію

Плоскоклітинний рак

Лейкоплакия

Метаплазія

182 / 6854
В препаратах представлені зрізи органів кровотворення і імуногенезу людини, для яких характерне наявність лімфоїдної тканини, яка формує різні структури (лімфатичні вузлики, часточки, тяжі). Визначте, в якому з органів відбувається антігеннезавісімая проліферація і диференціювання лімфоцитів.

Селезінка

Лімфатичні вузли

Тимус

Мигдалик

гемолімфатичних вузли

183 / 6854
При огляді зіву у хворого на ангіну визначається гіперемія слизової оболонки неба, мигдалини збільшені в розмірах, червоні, на їх поверхні помітні дрібні біло - жовті осередки. Який клініко - морфологічний варіант ангіни найбільш імовірний в даному випадку?

Гнійна

Фібринозна

Лакунарна

Катаральна

Некротична

184 / 6854
Хлопчик 13 років скаржиться на загальну слабкість, запаморочення, стомлюваність. Спостерігається відставання в розумовому розвитку. У крові і сечі висока концентрація валина, ізолейцину, лейцину. Сеча специфічного запаху. Який найбільш ймовірний діагноз?

Хвороба Аддісона

Гістидинемія

Тирозиноз

Хвороба кленового сиропу

Базедова хвороба

185 / 6854
В лікарню потрапив хворий з Східної Сибіру з жал Обами на біль в печінці. У фекаліях знайдені яйця до 30 мкм, що нагадують по формі насіння огірків. Який попередній діагноз можна поставити хворому?

Описторхоз

Парагонімоз

Дікроцеліоз

Тениаринхоз

Гіменолепідоз

186 / 6854
Після введення жабі стрихніну вона на мінімальне роздратування відповідає генералізованими судомами. Блокада який структури ЦНС є причиною цього?

Клітин Реншоу

холінорецептори

збуджує синапсів

адренорецептори

Гальмівних синапсів

187 / 6854
піддослідних собаці через зонд в порожнину шлунка ввели 150 мл м'ясного бульйону. Зміст якого з наведених речовин швидко збільшиться в крові тваринного?

Нейротензин

вазоінтестінальний поліпептид

Гастрін

Соматостатін

Інсулін

188 / 6854
Хворий 65 років, який страждав на атеросклероз, госпо алізірован в хірургічне відділення з приводу розлитого гнійного перитоніту. Під час операції діагностовано тромбоз брижових артерій. Яка найбільш ймовірна причина перитоніту?

Ішемія компресійна

Геморагічний інфаркт

Ішемічний інфаркт

Ішемія ангіоспастична

Стазі

189 / 6854
З випорожнень хворого дитини 6 місячного віку, який знаходився на штучному вигодовуванні виділена культура кишкової палички з антигенною структурою 0-111. Який діагноз можна поставити?

холероподібний захворювання

коліентеріт

Харчове отруєння

Гастроентерит

дізентеріеподобние захворювання

190 / 6854
У хворого діагностовано тиреотоксикоз. У крові знайдено антитиреоїдні тіла. Який тип алергічної реакції по Кумбсу і Джелу спостерігається п ри розвитку цього захворювання?

Анафілактичний

Стимулюючий

Цитотоксичний

Гіперчутливість сповільненого типу

Імунокомплексний

191 / 6854
У чоловіка 52 років через 3 роки після операції видалення шлунка зміст еритроцитів у крові становить 2,0 × 10 12 / л, Hb - 85 г / л, до. п. - 1,27. Порушення засвоєння якого вітаміну викликало такі зміни?

Р

В6

С

A

В12

192 / 6854
При пункційної біопсії в трансплантованою нирці виявлена дифузна інфільтрація строми лімфоцитами, плазмоцитами, лімфобластами, плазмобласти, а також некротичний артеріїт. Який патологічний процес розвинувся в трансплантаті?

Імунне відторгнення

Тубулонекроз

Пієлонефрит

Гломерулонефрит

Ішемічне пошкодження нирки

193 / 6854
При визначенні групи крові по системі АВ 0 аггл ютінацію еритроцитів досліджуваної крові викликали стандартні сироватки I і II груп та НЕ викликала III групи. Які агглютіногени містяться в цих еритроцитах?

D і C

А і В

А

B

С

194 / 6854
У людини 70 років швидкість поширення пульсової хвилі виявилася суттєво вище, ніж у 25- річного. Причиною цього є зниження:

Артеріального тиску

Серцевого викиду

Частоти серцевих скорочень

Еластичності судинної стінки

Швидкості кровотоку

195 / 6854
В групі дітей, які їли солодкий соковитий кавун, у двох з'явилися ознаки отруєння: різка слабкість, запаморочення, головний біль, блювота, задишка, тахікардія, синюшність губ, вух, кінчиків пальців. Лабораторний аналіз кавуна показав високу зміст нітратів. Який провідний механізм в патоген Езе отруєння тільки у двох дітей?

Недостатність глутатіон - піроксідази

Недостатність супероксиддисмутази

Блокада цитохромоксидази

Недостатність каталази

Недостатність мет -Hb- редуктази

196 / 6854
У хворого через 9 діб після введення лікувальної сироватки з'явилася кропив'янка, свербіж шкіри, набряк її та слизових оболонок, припухання лімфатичних вузлів. Яке захворювання розвинулося?

Набряк Квінке

Поліноз

Феномен Швартцмана

Феномен Овері

Сироваткова хвороба

197 / 6854
Хворий з інфекційним мононуклеозом на протязі 2 х тижнів приймав глюкокортикостероїдні препарати. Настала ремісія, але у нього виникло загострення хронічного тонзиліту. Результатом якого дії глюкокортикостероїдів є дане ускладнення?

імунодепресивними

антиалергічні

Протизапальної

антитоксически

протишокова

198 / 6854
При травмі у пацієнта 44 років виник розрив сухожиль м'язів лівої долоні, поверхневих кровоносних судин. Після оперативного втручання і видалення більшої частини некротичних зміненої м'язової тканини кровотік був відновлений. За рахунок яких судин?

Arcus palmaris superficialis

Arcus palmaris profundus

Аа. digitales palmares communes

Аа. perforantes

Aa. metacarpeae palmares

199 / 6854
При дослідженні каріотипу 5 річної дівчинки виявлено 46 хромосом. Одна з хромосом 15 й пари довше звичайної, т. до. до неї приєдналася хромосома з 21- ї пари. Який вид мутації має місце у цій дівчинки?

Інверсія

Брак

Транслокация

Дуплікація

Делеция

200 / 6854
У хворого з значними периферійними набряками почергове застосування діхлотіазіда, етакриновою кислоти і фуросеміду НЕ сприяло значному діуретичній ефекту. У крові значне підвищення кількості альдостерону. Вкажіть препарат вибору.

Вабить

Клопамід

Спіронолактон

Сечовина

Амілорид

201 / 6854
Через деякий час після інтенсивної фізичної тренування у спортсмена активується глюконеогенез, основним субстратом якого є:

Глутаминовая кислота

альфа - кетоглутарат

Аспарагінова кислота

Серин

Лактат

202 / 6854
На місці автокатастрофи пораненому водію наклали кровоспинний джгут на верхню третину стегна. У хірургічне відділення хворий був доставлений в задовільному стані тільки через 3 години. Після зняття джгута з'явився різкий набряк тканини стегна, частий пульс, холодний піт, різка гіпотонія . Яке патологічне явище розвинулося у потерпілого?

Токсеміческій шок

Анафілактичний шок

Геморагічний шок

Кардіогенний шок

Колапс

203 / 6854
Хворому з значними опіками зробили пересадку донорської шкіри, але на 8-му добу трансплантат набряк, змінився його колір і на 11-ту добу він почав відторгатися. Які клітини беруть в цьому участь?

Т - лімфоцити

Еозинофіли

Еритроцити

В - лімфоцити

Базофіли

204 / 6854
Хворому для купірування кишкової коліки призначений атропіну сульфат. Яке з названих захворювань може служити протипоказанням в цьому випадку?

Запаморочення

Синусова брадикардія

Глаукома

Гіпотонія

Бронхіальна астма

205 / 6854
Біля інфікованої рани збільшилися регіонарні лімфовузли. При гістологічному дослідженні в них виявлено збільшення кількості макрофагів, лімфоцитів і лімфатичних фолікулів в кірковому шарі, а також велику кількість плазматичних клітин. Який процес в лімфатичних вузлах відображають виявлені гістологічні зміни?

Уроджену недостатність лімфоїдної тканини

Придбану недостатність лімфоїдної тканини

Реакцію гіперчутливості

Пухлинну трансформацію

Антигенну стимуляцію

206 / 6854
У дитини підвищилася температура тіла, з'явилися болі при ковтанні. При огляді: піднебінні мигдалини збільшені, темно - червоні, покриті сіро - жовтими плівками, щільно спаяні з поверхнею мигдаликів. Яке запалення розвинулося в мигдалинах?

Дифтеритичне

Катаральне

Гнійне

Крупозне

Геморагічне

207 / 6854
У пацієнта, які тривалий час хворіє на туберкульоз, виявлено внутрішньоклітинне розташування мікобактерій. Який з вказаних препаратів обов'язково повинен бути включений в комплексну терапію туберкульозу?

Рифампіцин

Ізоніазид

Натрію парааміносаліцілат

Етіонамід

Етамбутол

208 / 6854
Після прийому жирної їжі у хворого з'являються нудота і печія, має місце стеаторея. Причиною такого стану може з'явитися:

Брак амілази

Порушення синтезу фосфоліпази

Брак жовчних кислот

Підвищений виділення ліпази

Порушення синтезу трипсину

209 / 6854
Під час здачі іспиту у студентів ' пересихає в роті '. Посилена реалізація яких рефлексів є механізмом, що обумовлює розвиток цього стану?

Незаперечних симпатичних

Умовних симпатичних

Незаперечних периферичних

умовних парасимпатических

Незаперечних парасимпатических

210 / 6854
Хворому з цирозом печінки разом з лікарськими препаратами внутрішньовенно ввели 500,0 мл 5% розчину глюкози. Яке порушення водного балансу найбільш ймовірно може виникнути у хворого?

Гіпоосмолярна гипергидратация

Ізоосмолярна гипергидратация

Порушень водного балансу НЕ відбудеться

Гиперосмолярная гипергидратация

Гіпоосмолярна дегідратація

211 / 6854
У гістологічному препараті визначається ться рецепторная зона сенсоепітеліального органу почуттів. Клітини даної зони лежать на базальної мембрані і включають такі види: зовнішні і внутрішні сенсорні, зовнішні і внутрішні фалангові, стовпові, зовнішні прикордонні і зовнішні підтримують. Якому органу почуттів належить дана рецепторная зона?

Органу рівноваги

Органу зору

Органу слуху

Органу нюху

Органу смаку

212 / 6854
У чоловіка при ураженні одного з відділів ЦНС спостерігається астенія, м'язова дистонія, порушення рівноваги. Який з відділів ЦНС вражений?

Чорна субстанція

Мозочок

Червоні ядра

Вестибулярні ядра

Ретикулярная формація

213 / 6854
Після введення пирога у хворого підвищилася температура, шкірні покриви стали блідими, холодними на дотик, з'явився озноб, зрілось споживання кисню. Як змінюються процеси терморегуляції в описаному періоді лихоманки?

Тепловіддача дорівнює теплопродукції

Знижується теплопродукція

Збільшується тепловіддача

Збільшується теплопродукція

Знижується тепловіддача

214 / 6854
У травмпункт звернувся чоловік 45 років після побутової травми плеча. Об'єктивно: відсутні функції розгинання, приведення і пронації плеча. Пошкодження який м'язи викликало таке стан?

надостьовий м'яз

великий круглий м'яз

малий круглий м'яз

підлопатковий м'яз

підостьовий м'яз

215 / 6854
У препараті червоного кісткового мозку людини визначаються скупчення гігантських клітин, що знаходяться в тісному контакті з синусоїдними капілярами. Які формені елементи крові утворюються з цих клітин?

Лейкоцити

Моноцити

Лімфоцити

Кров'яні пластинки

Еритроцити

216 / 6854
При гістологічному дослідженні стулки мітрального клапана серця виявлено: мукоїдному набряк, пошкодження ендотелію, утворення по замикаючому краю фібринових тромбів. Яка форма ревматичного ендокардиту спостерігається?

Дифузний ендокардит

Зворотно - бородавчастий ендокардит

Гострий бородавчастий ендокардит

Фібропластичний ендокардит

Поліпозно - виразковий ендокардит

217 / 6854
У дівчинки 5 років спостерігається висока температура і біль в горлі. Об'єктивно: набряк м'якого піднебіння, на мигдалинах сірі плівки, які відділяються з працею, залишаючи глибокі кровоточать, дефекти тканини. Яке з нижчеперелічених захворювань найбільш ймовірно?

Дифтерія зіва

Лакунарна ангіна

Інфекційний мононуклеоз

Некротична ангіна

Ангіна Симон івського - Венсана

218 / 6854
У хворого 70 років діагностовано крововилив у стовбур мозку. Обстеження виявило підвищення тонусу м'язів - згиначів на тлі зниження тонусу мишцразгібателей. Роздратуванням яких структур мозку можна пояснити зміни в тонусі м'язів?

Червоних ядер

ядер Голя

ядер Бурдаха

Чорної субстанції

ядер Дейтерса

219 / 6854
У хворого спостерігається атонія м'язів. Назвіть фермент м'язової тканини, активність якого може бути знижена при такому стані.

Каталаза

7- глутамінтрансферази

транскетолази

Амілаза

Креатинфосфокиназа

220 / 6854
У хворого виявлено різке зниження активності сурфактанту легень. Які зміни слід очікувати у цього хворого?

Розростання сполучної тканини легенів

Зміна еластичних властивостей легенів

Зменшення трахеобронхіального секрету

Порушення кровообігу в легенях

Схильність альвеол до спадання і неможливість їх швидкого розправлення

221 / 6854
52- річну пацієнтку на протязі останніх декількох днів турбують напади болю в правому підребер'ї після прийому жирної їжі. Візуально визначається пожовтіння склер і шкіри, ахолічний кал, сеча ' кольору пива? Присутність якого речовини в сечі пацієнтки зумовило темний колір сечі при обтураційній жовтяниці?

уробилин

Глюкози

білірубінглюкуроніди

стеркобіліна

Кетонових тел

222 / 6854
На обмеженому ділянці епідермісу внаслідок травми відсутні шари аж до росткового. Які клітини послужать джерелом його регенерації?

Шари базальних клітин

Шари крилатих і зернистих клітин незруйнованим ділянки

Шар базальних клітин незруйнованим ділянки

Шари крилатих і базальних клітин

Шари крилатих клітин

223 / 6854
У хворого спостерігається сльозотеча, підвищене слиновиділення. Це стан, спільно з іншими симптомами, расц енівается як роздратування волокон одного з черепних нервів. Якого саме нерва і яких його волокон? - Соматичні рухові волокна лицьового нерва

Соматичні рухові волокна окорухового нерва

Парасимпатические волокна окорухового нерва

Парасимпатические волокна блукаючого нерва

Парасимпатические волокна лицьового нерва

224 / 6854
У хворого з Тімом (пухлина вилочкової залози) спостерігається ціаноз, розширення підшкірної венозної сітки і набряк м'яких тканин обличчя, шиї, верхньої половини тулуба і верхніх кінцівок. Який венозний стовбур перетиснено пухлиною?

Внутрішня яремна вена

Верхня порожниста вена

Передня яремна вена

Підключичної вена

Зовнішня яремна вена

225 / 6854
Під час розтину в верхній частці правого легкого про назовні великий клиновидний вогнище темно - червоною щільною тканини. Під час гістологічного дослідження в ній виявлений некроз стінок альвеол ; просвіт альвеол щільно заповнений еритроцитами. Який процес розвинувся в легенях?

Геморагічний інфаркт легень

Гангрена легенів

карніфікація легких

Ателектаз легенів

Крововилив в легені

226 / 6854
При яких групах крові батьків по системі резус можлива резус - конфліктна ситуація під час вагітності?

Дружина Rh +, чоловік Rh + (гетерозигота)

Дружина Rh + (гетерозигота), чоловік Rh + (гомозигота)

Дружина Rh +, чоловік Rh + (гомозигота)

Дружина Rh-, чоловік Rh-

Дружина Rh-, чоловік Rh + (гомозигота)

227 / 6854
Мукополісахарідоз відноситься до хвороб накопичення. З - за відсутності ферментів порушується розщеплення полісахаридів. У хворих спостерігається вище ня виділення їх з сечею і накопичення в одному з органоїдів клітин. В яких органелах накопичуються мукополісахариди?

У комплексі Гольджі

У мітохондріях

В ендоплазматичному ретикулуме

В лізосомах

У клітинному центрі

228 / 6854
З метою аналгезії можуть бути використані речовини, що імітують ефекти морфіну, але виробляються в ЦНС. Вкажіть таке речовина.

бетаендорфін

Окситоцин

Соматоліберин

Кальцитонин

Вазопрессин

229 / 6854
У хворого, які тривалий час кращого тютюн, розвинувся рак легені. Які з перерахованих канцерогенних речовин містяться в тютюновому димі і відносяться до ПАУ (поліненасичених ароматичним вуглеводів)?

бета - нафтиламин

диметиламіноазобензолу

Бензпирен

Діетілнітрозамін

Ортоаміноазотолуол

230 / 6854
У померлого 58 років на розтині мітральний клапан деформований, потовщений, змикається НЕ до кінця. Мікроскопічно: вогнища колагенових волоконец еозинофільно, дають плюсову реакцію на фібрин. Найбільш ймовірно це:

Амилоидоз

Гіаліноз

Фібринозне запалений ие

Мукоїдне набухання

Фібриноїдне набухання

231 / 6854
Хворий 45 років госпіталізований в хірургічне відділення з скаргами на раптову гостру біль в надчеревній області. Після обстеження поставлений діагноз: перфоративная (проривна) виразка задньої стінки шлунка. Куди вилилося вміст шлунка в момент перфорації?

В сальниковую сумку

В лівий брижових синус

У передшлункову сумку

У правий брижових синус

В печінкову сумку

232 / 6854
При попаданні чужорідного тіла в дихальні шляхи на який бронх має бути направлено увагу лікаря в першу чергу для пошуку стороннього тіла і його видалення?

Праві часткові бронхи

Правий головний бронх

Ліві сегментарні бронхи

Ліві часткові бронхи

Лівий головний бронх

233 / 6854
Смерть хворого 16 років настала від розлитого (поширеного) фібрінозногнойного перитоніту. На розтині в нижньому ділянці тонкої кишки виявлена виразка, що повторює форму пеєрових бляшки, з перфорацією стінки кишки. Мікроскопічна дослідження виявило невизначеність малюнка лімфоїдної тканини, вите снення її проліферуючими моноцитами, які формують гранульоми. Ускладнення якого захворювання стало причиною смерті?

бруцельоз

черевного тифу

дизентерія

неспецифічні виразкового коліту

холери

234 / 6854
Після перенесеного запального захворювання у хворого виникло неповне відведення очного яблука в латеральну сторону. Який нерв пошкоджений у хворого?

Зоровий

Відвідний

Окоруховий

Особовий

Бічний

235 / 6854
У чоловіка 52 років діагностовано системний амебіаз з ураженням кишечника, печінки, легенів. Який препарат слід призначити?

Ентеросептол

Метронідазол

Хиниофон

Тетрациклін

Хінгамін

236 / 6854
У крові хворого виявлено низький рівень альбумінів і фібриногену. Зниження активності яких органел гепатоцитів печінки наиб олее ймовірно обумовлює це явище?

Комплексу Гольджі

Лізосом

Мітохондрій

Гранулярної ендоплазматичної сітки

Агранулярної ендоплазматичної сітки

237 / 6854
У хворого виявлено множинні синці на тілі, тривалість кровотечі по Дуке 25 хвилин, число тромбоцитів крові 25 * 109 / л. Для якого захворювання характерні такі ознаки?

Авітаміноз С

Спадковий дефект освіти тромбоцитів

Хвороба Віллебранда

Гемофілія В

Гемофілія А

238 / 6854
У хворого жовтяницею отримані такі дані лабораторного обстеження: HbsAg-, HBeAs-, анти HBsG +, анти HBsM-, HCAg +. Який діагноз підтверджується на основі цих даних?

Рецидив гепатиту В

Гепатит С

П овторное зараження гепатитом В

Гепатит С, в анамнезі - гепатит

Хронічний гепатит В з низькою реплікативної активністю

239 / 6854
Хворому з бактеріальною інфекцією лікар призначив сульфадимезин в таблетках, порадивши запивати його 1,5-2 л лужної мінеральної води щодоби. Чим обумовлена необхідність даної рекомендації?

Для нейтралізації кислоти шлункового соку

Для зсуву рН крові в лужну сторону

Для профілактики кристалізації ацетильних похідних препарату в ниркових канальцях

Для зниження дратівної впливу на шлунок

Для пролонгування дії

240 / 6854
У хворого 30 років з дизентерією, підтвердженої бактеріологічно, з'явилися ознаки парапроктиту. Про який стадії місцевих змін найбільш ймовірно йде мова у даного хворого?

Фіб рінозний коліт

Стадія загоєння виразок

Фолікулярний коліт

Катаральний коліт

Стадія освіти виразок

241 / 6854
У клінічно здорових людей в умовах високогір'я виявляються ознаки анемії. При аналізі крові виявляють серповидні клітини. Визначте генотип цих людей:

ХсХс

ХСХс

Aa

АА

аа

242 / 6854
При визначенні групи крові по системі АВ 0 агглютинацию еритроцитів досліджуваної крові викликали стандартні сироватки Г і II груп і НЕ викликала сироватка III групи. Які агглютіногени містяться в цих еритроцитах?

В

А і В

D і С

А

С

243 / 6854
З випорожнень хворого дитини 6 місячного віку, який знаходився на штучному вигодовуванні, виділена культура кишкової палички з антигенною структурою 0-111. Який діагноз можна поставити?

Харчове отруєння

холероподібний захворювання

дізентеріеподобние захворювання

Коли - ентерит

Гастроентерит

244 / 6854
Малюк попросив Вас надути повітряний кульку як можна сильніше за один видих. Яким обсягом повітря Ви скористаєтеся?

Життєвої ємністю легких

Ємністю вдиху

Резервним об'ємом вдиху

Функціональної залишкової ємністю

Загальною ємністю легких

245 / 6854
Під час хірургічної операції пацієнту проведено переливання крові. На антигени якого збудника необхідно перевірити цю кров?

Вірусу гепатиту В

Вірусу гепатиту А

аденовіруси

ентеровірус

Вірусу гепатиту Е

246 / 6854
У 6 місячного дитини спостерігалися часті і сильні підшкірні кровотечі. Призначення синтетичного аналога вітаміну К (вікасолу) дало позитивний ефект. У гамма - карбоксилювання глутамінової кислоти якого білка системи згортання крові приймає участь цей вітамін?

Фактора Розенталя

Фактора Хагемана

Фібриногену

антигемофільний глобуліну А

протромбіну

247 / 6854
Хворому з гіперсекрецією шлункового соку лікар рекомендував виключити з дієти насичені бульйони і овочеві відвари, тому що вони стимулюють шлункову секрецію. Який переважний механізм стимуляції шлункової секреції в даному випадку?

Роздратування механорецепторів порожнини рота

Стимуляція вироблення секретину в 12-палої кишці

Роздратування механорецепторів шлунка

Стимуляція вироблення гастрит ина G-клітинами

Роздратування смакових рецепторів

248 / 6854
Яка частина товстої кишки найбільш ймовірно буде пошкоджена при пораненні в праву половину живота?

Низхідна ободова кишка

Поперечна ободова кишка

Висхідна ободова кишка

Пряма кишка

Сигмовидная ободова кишка

249 / 6854
Дитина 7 років НЕ може відвести плече і підняти його до горизонтального рівня, до особі руку приводить лише тильною стороною при деякій абдукції плеча (за рахунок надостной м'язи) - рука ' сурмача '. Активна функція якої м'язи відсутній?

дельтоподібними

подостной

Великий круглої

Малої круглої

Великий грудної

250 / 6854
З нітратів, нітритів і нітрозамінів в організмі утворюється азотистая кислота, яка обумовлює окисне дезамінування азотистих про снованіі нуклеотидів. Це може привести до точкової мутації - заміни цитозину на:

Урацил

Тимин

гуанін

Инозин

Аденін

251 / 6854
Мати виявила у 5 річної дочки на періанальних складках білих ' черв'ячків ',' які викликали у неї свербіння і занепокоєння, і доставила їх в лабораторію. При огляді лікар побачив білих гельмінтів 0,5-1 см завдовжки, ниткоподібної форми з загостреними кінцями, у деяких вони закручені. Який діагноз можна поставити?

Ентеробіоз

Дифиллоботриоз

Описторхоз

Аскаридоз

Теніоз

252 / 6854
У людини діагностовано галактоземія - хвороба накопичення. Внаслідок порушення якої клітинної структури виникла ця хвороба?

Центросоми

Клітинного центру

Мітохондрій

Комплексу Гольджі

Лізосом

253 / 6854
При дисбактеріозах, що супроводжуються процесом гниття (протей, псевдомонади) і підвищенням рН фекалій, необхідно призначати біологічні препарати, Подкисляющие середу і проявляють антагоністичну дію. Які мікроорганізми для цього підходять?

серрацій

Клебсієли

ентеробактерій

Біфідумбактеріі

азотобактер

254 / 6854
Після введення жабі стрихніну вона на мінімальне роздратування відповідає генералізованими судомами. Блокада який структури ЦНС є причиною цього?

адренорецептори

Клітин Реншоу

Гальмівних синапсів

збуджує синапсів

холінорецептори

255 / 6854
Хворому з підозрою на черевний тиф лікар - інфекціоніст призначив бактеріологічне дослідження крові. Доцільність цього призначення пояснюється тим, що в першу тиждень захворювання тифо - паратифами спостерігається:

Септицемія

Септикопіємія

Бактериемия

токсінеміі

Вірусемія

256 / 6854
У жінки 32 років з безсимптомним перебігом хвороби у другій раз народився мертвий дитина з вираженою мікроцефалією. Про якому захворюванні в першу чергу слід подумати лікаря?

Токсоплазмоз

Лістеріоз

Бруцельоз

Гістоплазмоз

Сифіліс

257 / 6854
Для попередження нападу бронхіальної астми лікар призначив хворому кромолин натрію. Який з наведених механізмів характерний для цього кошти?

Інактивація гістаміну

Зниження концентрації імуноглобулінів

Блокада гістамінових рецепторів

Стабілізація мембран огрядних кліток

Зв'язування вільного гістаміну

258 / 6854
Дівчинка 10 років часто хворіє на гострі респіраторні інфекції, після яких спостерігаються множинні точкові крововиливи я в місцях тертя одягу. Гіповітаміноз якого вітаміну має місце у дівчинки?

B1

B2

В 6

C

A

259 / 6854
Людина отримав електротравму. При цьому струм пройшов через серцевий м'яз. Які небезпечні порушення в роботі серця, що вимагають невідкладного втручання, можуть виникнути в цій ситуації?

Екстрасистолія

Фибрилляция шлуночків

Брадикардія

Тахікардія

Фібриляція передсердь

260 / 6854
З допомогою шпателя зроблений зішкріб слизової рота людини. У незруйнованих епітеліальних клітинах пофарбованого мазка добре видно овальні ядра, неоднакові по розмірах. Яким шляхом відбувалося поділ цих клітин?

Амитоз

Шизогонія

Бінарне розподіл

Мейоз

Митоз

261 / 6854
Під час ЕКГ - дослідження хворий виявлено періодичне поява шлуночкової екстрасистоли. При цьому встановлено, що перед екстрасистолою відсутня зубець Е. Яка найбільш ймовірна причина його зникнення? - Виникнення рефрактерного періоду в шлуночках

Виникнення рефрактерного періоду в передсерді

Блокада імпульсу в синусовому у злі

Неможливість ретроградного проведення через AV-вузол

Блокада проведення імпульсу по передсердях

262 / 6854
54- річний чоловік скаржиться на відсутність чутливості шкіри нижнього століття, зовнішньої латеральної поверхні носа, верхньої губи. Лікар при огляді встановлює запалення другої гілки трійчастого нерва. Через якийсь отвір виходить з черепа ця гілка?

Верхня глазничная щілину

Кругле отвір

Овальний отвір

Остистий отвір

рвані отвір

263 / 6854
Хворий з діагнозом вогнищевого туберкульозу верхньої частки правого легкого отримує ізоніазид в складі комбінованої терапії. Через деякий час пацієнт почав скаржитися на м'язову слабкість, зниження чутливості шкіри, порушення зору, координації рухів. Який вітамінний пре парат доцільно використовувати для усунення цих явищ?

Вітамін В6

Вітамін В12

Вітамін D

Вітамін А

Вітамін С

264 / 6854
Хворий скаржиться на диспептичні розлади, мелену, гемороїдальні кровотечі. При дослідженні виявлено розширення сітки венозних судин на передній стінці живота в комплексі з збільшенням його розмірів. Яка патологія шлунково-кишкового тракту може проявитися такими симптомами?

Ентерит

Портальная гіпертензія

Виразкова хвороба

Кишкова аутоинтоксикация

Коліт

265 / 6854
У хворого з гіпертонічною хворобою при систематичному лікуванні антигіпертензивним засобом з'явився кашель. Яке з названих засобів може бути причиною даного побічного ефекту?

Діхлотіазід

Еналаприл

Верапаміл

Празозин

Клофелін

266 / 6854
У експерименті досліджували поріг сили подразнення клітин різних тканин. В яких з наступних клітин він виявиться найнижчими?

У міоцитах гладкких м'язів

У залізистих клітинах

В кардіоміоцитах

В мотонейронах спинного мозку

У міоцитах скелетних м'язів

267 / 6854
У хворого, які тривалий час брав глюкокортикоїди, в результаті відміни препарату виникло загострення наявного захворювання, зниження артеріального тиску, слабкість. Чим можна пояснити ці явища?

кумуляції

звикання до препарату

Виникненням недостатності надниркових залоз

сенсибілізація

гіперпродукція АКТГ

268 / 6854
У психіатричну клініку доставлений хворий 40 років в стані збудження, агресії, марення. Який препарат слід ввести хворому?

Седуксен (діазепам)

Настоянку валеріани

Натрію бромід

Резерпін

Аміназин

269 / 6854
Хворий госпіталізований з скаргами на блювоту, запаморочення, двоїння в очах, утруднене ковтання. Лікар запідозрив ботулізм. Які методи діагностики доцільно використовувати для підтвердження діагнозу?

Біологічну пробу, бактеріологічний

Бактеріологічний, мікологічний

Протозоологіческій, мікроскопічний

Алергійну пробу, серологічний

270 / 6854
У хворого нефритом виявлена глюкозурія і аминоацидурия. Порушення якого механізму реабсорбції глюкози і амінокислот є причиною цього?

піноцитоз

фагоцитозу

Простий дифузії

Вторинного Na+ - залежного транспорту

Первинного активного транспорту

271 / 6854
У хворого 27 років виявлені патологічні зміни печінки і головного мозку. У плазмі крові виявлено різке зниження, а в сечі збільшення вмісту міді. Поставлено діагноз - хвороба Вільсона. Активність якого ферменту в сироватці крові необхідно дослідити для підтвердження діагнозу?

Карбоангідрази

Церулоплазміну

лейцінамінопептідази

ксантиноксидазу

Алкогольдегідрогенази

272 / 6854
У юнака під час фізичної навантаження хвилинне споживання кисню і хвилинне виділення вуглекислого газу дорівнюють 1000 мл. Які субстрати окислюються в клітинах його організму?

Вуглеводи і жири

Жири

Вуглеводи

Вуглеводи і білки

Білки

273 / 6854
У гастроентерологічне відділення потрапив хворий 57 років з підозрою на синдром Золінгера - Еллісона, про що свідчило різке збільшення рівня гастрину в сироватці крові. Яке порушення секреторної функції шлунка найбільш імовірно?

Гиперсекреция гіпоацидного

Гіпосекреція гіпоацидного

Гиперсекреция гіперацидних

Ахілія

Гіпосекреція гіперацидних

274 / 6854
Дитина 5 років поступив в ЛОР - відділення з діагнозом 'гнійне запалення середнього вуха.' Захворювання почалося з запалення носоглотки. Через якийсь канал скроневої кістки інфекція потрапила в барабанну порожнину?

канадець барабанної струни

Сонний канал

Барабанний каналець

Мишечнотрубний канал

Сонно - барабанні канальці

275 / 6854
При видаленні зуба для знеболювання використовують розчин новокаїну. З якої метою його вводять НЕ в ясна біля зуба, а в область проходження чутливого нерва?

Порушується формування потенціалу дії больових рецепторів

Блокується проведення больових імпульсів

Змінюється рН тканин в області анестезії

Пригноблюється аксональний транспорт

Підвищується збудливість больових рецепторів

276 / 6854
У хворого сухим плевритом вислуховується шум тертя плеври. При ураженні якого виду епітелію відзначається цей симптом?

Перехідний епітелій

Одношаровий плоский епітелій

Одношаровий кубічний епітелій

Одношаровий призматичний епітелій

багатошарове ий епітелій

277 / 6854
метильних групи (-СН3) використовуються в організмі для синтезу таких важливих сполук як креатин, холін, адреналін і ін. Яка з незамінних амінокислот, є джерелом цих груп є одна?

Метіонін

Ізолейцин

Валін

Лейцин

Триптофан

278 / 6854
При розтині померлого дитини, який страждав діареєю, виявлено ексікоз і розлитої фібринозний коліт. В мазку - відбитку слизової оболонки виявлені грамнегативні палички. Який найбільш ймовірний діагноз?

Холера

Сальмонельоз

Дизентерія

Черевний тиф

Стафилококковая кишкова інфекція

279 / 6854
Поступив з їжею глікоген Гідролізований в шлунково - кишковому тракті. Який кінцевий продукт утворився в результаті цього процесу?

Лактат

Глюкоза

Галактоза

Лактоза

Фруктоза

280 / 6854
При обстеженні хворого виявлено новоутворення в білому речовині півкуль великого мозку з локалізацією в коліні і передньому відділі задньої ніжки внутрішньої капсули. Волокна якого провідного шляху мозку будуть зруйновані?

Tr. parietooccipitopontinus

Tr. thalamocorticalis

Tr. pyramidalis

Tr. frontothalamicus

Tr. frontopontinus

281 / 6854
У людини внаслідок довільної затримки дихання на 40 с зросли частота серцевих скорочень і системне артеріальний тиск. Реалізація будь механізмів регуляції зумовила зміна показників?

Безумовні симпатичні рефлекси

Умовні парасимпатичні рефлекси -

Умовні симпатичні рефлекси

Безумовні парасимпатичні рефлекси

282 / 6854
На протязі двох тижнів хвора приймала мікстуру, прописану невропатологом по приводу неврастенії. Самопочуття хворої декілька покращилось, однак незабаром з'явилися скарги на нежить, кон'юнктивіт, шкірну висип, млявість і ослаблення пам'яті. Був встановлений діагноз ' бромізм '. Що доцільно призначити для послаблення симптомів?

Натрію хлорид

Полиглюкин

Аспаркам

Розчин глюкози 5%

283 / 6854
Хворий НЕ може підняти опущену вниз нижню щелепу. Які м'язи НЕ можуть виконати свої функції?

Мімічні м'язи

Жувальні м'язи

Кругова м'яз рота

М'яз, що піднімає кут рота

надчерепной м'яз

284 / 6854
До лікаря звернувся хворий з пошкодженої стопою, забинтованою брудною марлевою пов'язкою, просоченої гнійними виділеннями. Спроба зняти пов'язку для огляду і обробки рани викликає гостру біль, так як вона прилипла до рани поверхні. Який антисептик полегшить видалення пов'язки і очистить рану від бруду і гною?

Етоній

Етакрідін

Перекис водню

Калію перманганат

Фурацилин

285 / 6854
Хворому з непереносимістю антибіотиків для лікування пневмонії призначений сульфален. Через кілька днів у хворого розвинувся гемоліз еритроцитів. Недостатність якого ферменту в організмі хворого сприяла розвитку цього побічного ефекту?

Урідіндіфосфатглюкуроновой трансферази

холінестерази

ацетальдегіддегідрогенази

Глюкозо-6-фосфатдегідрогенази

N-ацетилтрансферази

286 / 6854
До лікаря звернувся хворий з скаргами на постійну спрагу. Виявлено гіперглікемія, поліурія та підвищений змісту 17- кетостероїдів в сечі. Яке захворювання найбільш ймовірно?

Аддисонова хвороба

Інсулінозалежний діабет

Мікседема

Стероїдний діабет

Глікогеноз I типу

287 / 6854
При рентгенологічному дослідженні кісток основи черепа виявлено збільшення порожнини турецького сідла, витончення передніх похилих відростків, руйнування різних ділянок турецького сідла. Пухлина який ендокринної залози може викликати таке руйнування кісток?

вилочкова залози

Гіпофіза

Наднирників

Епіфіза

щитовидної залози

288 / 6854
Людина стоїть в кімнаті в легкій одязі, температура повітря +14°С. Вікна та двері закриті. Яким шляхом він віддає більше всього тепла?

перспірація

Теплорадіація

Конвекція

Теплопроведение

Випаровування

289 / 6854
Клінічні дослідження крові рекомендується проводити натще і вранці. Які зміни компонентів крові можливі, якщо провести забір крові після прийому їжі?

Збільшення числа еритроцитів

Зниження числа еритроцитів

Зниження числа тромбоцитів

Збільшення білків плазми

Збільшення числа лейкоцитів

290 / 6854
Після фізичної навантаження підвищився артеріальний тиск крові. Чому?

Збільшилася кількість функціонуючих капілярів

Збільшилася зміст води в плазмі крові

Збільшилася зміст іонів калію в плазмі крові

Збільшилася кількість гемоглобіну

Збільшився хвилин ний обсяг кровообігу

291 / 6854
У хворого з запаленням легенів непереносимість антибіотиків. Який з комбінованих сульфаніламідних препаратів слід призначити хворому?

Етазол

Сульфацил натрію

Сульфадиметоксин

Бісептол

Стрептоцид

292 / 6854
Тривале перебування в умовах спеки викликало у людини спрагу. Сигналізація від яких рецепторів, в першу чергу, зумовила її розвиток?

осморецептори печінки

барорецепторами дуги аорти

Осморецептори гіпоталамуса

натрієві рецепторів гіпоталамуса

глюкорецептори гіпоталамуса

293 / 6854
У юнака 20 років травмовано праве яєчко. Яку небезпеку це становить для лівого (здорового) яєчка?

Демаскування антигену і виникнення пошкодження антитілами

Розвиток гіпертрофії

Розвиток атрофії

Розвиток инфекц іонного процесу

Чи не загрожує нічим

294 / 6854
При обстеженні хворого визначається наявність гіперглікемії, кетонурія, поліурії, гиперстенурия і глюкозурії. Яка форма порушення кислотно - лужного балансу має місце в даній ситуації?

Метаболічний алкалоз

Метаболічний ацидоз

Негазовий алкалоз

Газовий алкалоз

Газовий ацидоз

295 / 6854
У хворого 45 років на правій нозі спостерігається блідість шкіри гомілки і стопи, визначається відсутність пульсації тильної артерії стопи та задньої великогомілкової артерії. Пульсація стегнової артерії збережена. Поранення який артерії сталося?

Низхідній колінної - Малогомілкової

Зовнішньої клубової

Глибокої артерії стегна

Підколінної

296 / 6854
Чоловікові 70 років, яка страждає хронічним бронхітом, призначений противокашлевой препарат - кодеїн. Який механізм забезпечує противокашлевой ефект?

Периферичний дію

Конкурентний

Місцеве дію

Рефлекторний

Центральний

297 / 6854
Людина сидів у кріслі з закритими очима, коли задзвонив телефон. Як зміниться у нього ЕЕГ?

Збільшиться тета - ритм

Бета - ритм зміниться на тета - ритм

Альфа - ритм зміниться на бета - ритм

Тета - ритм зміниться на альфа - ритм

Збільшиться альфа - ритм

298 / 6854
У приймальне відділення доставлений хворий у непритомному стані. Шкіра холодна, зіниці звужені, дихання з утрудненням, відзначається періодичність по типу Чейна - Стокса, артеріальний тиск знижений, сечовий міхур переповнений. Отруєння яким речовиною най більш ймовірно?

Наркотичними анальгетиками

ненаркотичних анальгетиками

транквілізатори

М-холиноблокаторами

299 / 6854
У хворого гострим мієлобластний лейкоз виявлено: збільшення печінки, селезінки, анемія, мієлобласти в периферичної крові. Який принциповий ознака дозволяє відрізнити гострий мієлобластний лейкоз від хронічного?

Владні клітини в периферичної крові

Анемія

Тромбоцитопенія

Лейкемічний провал

Панцитопенія

300 / 6854
При дослідженні крові хворого виявлено значне збільшення активності МВ - форм КФК (креатинфосфокінази) та ЛДГ-1. Яка найбільш ймовірна патологія?

Панкреатит

Ревматизм

Гепатит

Холецистит

Інфаркт міокарда

301 / 6854
Сульфаніламідні препарати нагадують по структурі парааминобензойную кислоту. В ніж з остоит молекулярна основа їх фармакологічного ефекту?

В порушення синтезу вітаміну

В активації ліполізу

В руйнуванні клітинної мембрани

У ингибировании гліколізу

У зв'язуванні з ДНК

302 / 6854
На прийом до лікаря прийшов пацієнт дуже високого зростання, з довгими товстими пальцями рук, великою нижньою щелепою і відвислою нижньою губою. Підвищену секрецію якого гормону який залози можна підозрювати?

гонадотропні гормону передньої частки гіпофіза

Соматотропного гормону передньої частки гіпофіза

Гормонів надниркової залози з групи глюкокортикоїдів

антидіуретичного гормону задньої частки гіпофіза

Гормонів щитовидної залози

303 / 6854
Хворий 37 років, що страждає на облітеруючий ендартеріїтом судин нижніх кінцівок, отримує фенилин в добовій дозі 60 мг / кг. У зв'язку з проявами судомного синдрому (в анамнезі ЧМТ) призначений фенобарбітал, після відміни якого у хворого виникло носове кровотеча. Дане ускладнення пов'язане з:

Гальмуванням фенобарбіталом мікросомального окислення в печінці

окислювальний дезаминированием фенилина

кон'югації фенилина з глюкуроновою кислотою

Індукцією феноба рбіталом ферментів мікросомального окислення в печінці

аліфатичні гідроксилюванням фенобарбіталу

304 / 6854
У хворого 30 років діагностовано ішемічна хвороба серця. Напади стенокардії турбують вже 3 роки. Виявлено спадковий характер захворювання. Який вид гиперлипопротеидемии найбільш ймовірно буде виявлено у хворого?

V (гіперпребеталіпопротеідемія і гіперхіломікронемія).

III (дісбеталіпопротеідемія).

I (гіперхіломікронемія).

IV (гіперпребеталіпопротеідемія).

II (гіпербеталіпопротеідемія).

305 / 6854
Жінка 53 років, зріст 163 см, вага 92 кг, рівномірне відкладення жиру, особа одутле, малорухлива, апатична. При натисканні шкіри ноги залишається ямка. Порушенням функції якої залози обумовлено стан хворої?

Паращитовидних

Щитоподібної

Гіпофіза

надниркові

Полових

306 / 6854
В приймальне відділення доставлений дитина 1,5 років з ознаками отруєння нітратами: стійкий ціаноз, задишка, судоми. Який пат огенетіческій механізм лежить в основі цих симптомів?

Освіта скороченої гемоглобіну

Освіта оксигемоглобина

Освіта карбогемоглобін

Освіта карбоксигемоглобина

Освіта метгемоглобіну

307 / 6854
Під час обіду дитина поперхнувся і аспірованої їжу. Почався сильний кашель, шкіра і слизові ціанотичні, пульс прискорений, дихання рідкісне, видих подовжений. Яке порушення зовнішнього дихання розвинулося у дитини?

Стенотичне дихання

Дихання Біота

Стадія инспираторной задишки при асфіксії

альтернирующая дихання

Стадія експіраторной задишки при асфіксії

308 / 6854
У померлого від малярії виражена жовтушність шкіри, склер та слизових оболонок. При розтині: селезінка збільшена в розмірі, аспидно - сірого кольору. Аспидно - сіра забарвлення селезінки обуслов лена наявністю:

ліпофусцин

Гемопорфіріна

гемомеланін

Меланіну

гемосидерину

309 / 6854
Хворому з маніакально - депресивним синдромом в стадії депресії, скаржилися на відчуття тривоги, страху, був призначений антидепресант з супутнім псіхоседатівним ефектом. Який це був препарат?

інказан

Имизин (іміпрамін)

Сіднофен

Ніаламід

Амитриптилин

310 / 6854
Який препарат слід призначити хворому, у якого з - за висипу алергічного характеру з почервонінням, набряком, сильним свербінням з'явилася безсоння?

Фенобарбітал

Нітразепам

Димедрол

Хлоралгідрат

Натрію оксибутират

311 / 6854
З метою перевірки крові донорів на наявність антигенів гепатиту B необхідно застосовувати високочутливі методи. Яку з названих реакцій слід застосує ь з вказаною метою?

Твердофазний імуноферментний аналіз

Реакцію непрямої гемаглютинації

Реакцію непрямої імунофлуоресценції

Іммуноелектрофорез

Реакцію зв'язування комплементу

312 / 6854
У пацієнта після травми виникли паралічі, розлади больової чутливості справа ; зліва паралічі відсутні, але порушена больова і температурна чутливість. Яка причина такого явища?

Одностороннє ураження спинного мозку з правої сторони

Пошкодження рухової зони кори головного мозку

Пошкодження стовбура мозку

Пошкодження мозочка

Пошкодження середнього мозку

313 / 6854
При обстеженні хворого встановлено діагноз - кліщовий поворотний тиф. Яким шляхом міг заразитися хворий?

Через укус собачого кліща

Через укус селищного кліща

Через укус тайгового кліща

Через укус москіта

Через укус малярійного комара

314 / 6854
В шкірі виявлена щільна, рухома, чітко відмежована від оточуючих тканин пухлина. У розрізі вона білого кольору, представлена волокнистою тканиною. Мікроскопічно - хаотично переплетені колагенові волокна, клітин мало. Яка це пухлина?

гістіоцитома

Тверда фіброма

Дерматофіброма

Десмоїд

М'яка фіброма

315 / 6854
В печінці хворого порушена детоксикація природних метаболітів і ксенобіотиків. Активність якого цитохрому може бути знижена?

цитохроми З-1

цитохроми

Гемоглобіну

цитохромоксидази

Цитохроми Р-450

316 / 6854
У хворого тромбоз коронарних судин. Необхідно введення речовини, що відновлює їх прохідність. Яке з пропонованих речовин про ладает необхідними властивостями?

Стрептокиназа

Гепарин

Ацетилсаліцилова кислота

Пентоксифілін

Фенилин

317 / 6854
При обстеженні хворого з гемофілією виявлено зміна деяких показників крові. Який з перерахованих ознак відповідає цьому захворюванню?

Час згортання крові загальмовано

Тромбоцитопенія

Еритроцитоз

Еозинофілія

афібриногенемією

318 / 6854
У хворого діагностовано гострий інфаркт міокарда, що супроводжується стійкими болями за грудиною. Неефективність раніше введених препаратів дала підставу лікарю провести нейролептанальгезию. Який нейролептик треба використовувати?

Аміназин

Дроперидол

Галоперидол

Резерпін

Метаперазін

319 / 6854
Відомо, що вірус імунодефіциту людини належить до сімейства ретровірусів. Вкажіть осно вной ознака, що характеризує дане сімейство.

Відсутність інтеграції нуклеїнової кислоти в геном господаря

Виявлення антигенів реакцією імуноферментного аналізу

Наявність ферменту зворотної транскриптази

Наявність мінус - РНК

320 / 6854
При мікроскопічному дослідженні нефробіоптата виявлено наявність півмісяців більш ніж в 50% клубочків, капілярні петлі некротизованих, в просвіті їх виявлено фібринові тромби, виражений тубуло - інтерстиціальний компонент. Про якому захворюванні нирок слід думати?

Ліпоїдний нефроз

Швидкопрогресуючий гломерулонефрит

Некротический нефроз

Хронічний гломерулонефрит

Амилоидоз

321 / 6854
Морфологічні дослідження селезінки виявили активізацію імунних реакцій в організмі. В яких структурах даного органу починається антигензависимая проліферація Т - лімфоцитів?

Періартеріального зона білої пульпи

Червона пульпа

Центральна зона білої пульпи

Мантійна зона білої пульпи

Маргінальна зона білої пульпи

322 / 6854
При обстеженні хворого з травматичним пошкодженням головного мозку виявлено, що він перестав розрізняти переміщення предмета по шкірі. Який відділ кори мозку пошкоджений?

Потилична частка кори

Лобова частка кори

Передня центральна звивина

Тім'яна частка кори

Задня центральна звивина

323 / 6854
Після внебольничного аборту у жінки прогресував гнійний ендоміометрит зі смертельним результатом. При розтині померлої виявлені численні абсцеси легких, субкапсулярні гнійники в нирках, гіперплазія селезінки. Яка форма сепсису виникла у хворої?

уросепсисі

хроніосепсису

Легеневий сепсис

З ептікопіемія

Септицемія

324 / 6854
Хвора 45 років скаржиться на задишку при невеликій фізичної навантаженні, набряки на балках, в анамнезі часті ангіни, хворіє на протязі двох років. Діагностовано міокардит, комбінований мітральний порок серця, недостатність кровообігу. Який гемодинамічний механізм декомпенсації серця у хворої?

Тахікардія

Підвищення артеріального тиску

Зменшення обсягу циркулюючої крові

Зменшення хвилинного обсягу серця

Зниження венозного тиску

325 / 6854
У дитини 7 років на шкірі розгинальних поверхонь ліктьових і колінних суглобів з'явилися щільні, безболісні вузлики розміром 1-2 мм. У біоптаті вузликів - великий осередок фибриноидного некрозу сполучної тканини з лімфоцитами і макрофагами по периферії. При якому захворюванні спостерігаються такі вузлики?

Ревматизм

Системна червона вовчак

Вузликовий періартеріїт

Склеродермия

Ревматоїдний артрит

326 / 6854
Для лікування урогенітальних інфекцій використовують хінолони - інгібітори ферменту ДНК - гірази. Який процес порушується під впливом хінолонів в першу чергу?

Репарація

Зворотній транскрипція

Ампліфікація генів

Рекомбінація генів

Реплікація

327 / 6854
При мікроскопічному дослідженні тканини печінки було виявлено, що деякі клітини розпалися на невеликі фрагменти з окремими органелами і залишками ядра, оточені мембраною. Запальна реакція відсутня. Для якого патологічного процесу характерні ці зміни?

Апоптоз

Некроз

Пла зморексіс

Плазмоліз

каріорексисом

328 / 6854
У хворого має місце позаклітинний набряк тканин (збільшені розміри м'яких тканин кінцівок, печінки і т. д.). Зменшення якого параметра гомеостазу є найбільш вірогідною причиною розвитку набряку?

в'язкості

Онкотичного тиску плазми крові

гематокриту

Осмотичного тиску плазми крові

329 / 6854
При обстеженні юнака з розумовою відсталістю виявлено евнухоідний будова тіла, недорозвиненість статевих органів. У клітинах порожнини рота - статевий хроматин. Який метод генетичного дослідження слід використовувати для уточнення діагнозу?

Популяційно-статистичний

дерматогліфіки

Цитологічний

Клініко-генеалогічний

Біохімічний

330 / 6854
У людини зареєстрована електрокардіограма зі зниженою амплітудою зубця R. Що означає цей зубець на ЕКГ?

Електричну діастолу серця

Поширення збудження від передсердь до шлуночків

Електричну систолу серця

Поширення збудження по передсердям

Поширення збудження по шлуночках

331 / 6854
У жінки 68 років після інсульту відсутні рухи в верхній і нижній правих кінцівках. Тонус м'язів цих кінцівок і рефлекси в них підвищені. Є патологічні рефлекси. Яка це форма паралічу?

Тетраплегія

Геміплегія

Дисоціація

моноплегии

Параплегія

332 / 6854
У хворого 17 років інтраопераційно на нижній поверхні печінки виявлена пухлина розмірами 4,5x5,0x3,5 см з субсерозной локалізацією, темно - червоного кольору, на розрізі представлена порожнинами зі значним вмістом крові. До акой найбільш ймовірний попередній діагноз?

Гемангіоендотеліома

Капілярна гемангіома

Кавернозная гемангіома

Гемангіоперицитома

Лимфангиома

333 / 6854
У хворого при гастроскопії виявлено недостатнє кількість слизу, що покриває слизову оболонку. З порушенням функції яких клітин стінки шлунка це пов'язано?

B- клітин призматичного залозистого епітелію

шеечную клітин

ендокріноціти

Парієтальних клітин залоз шлунка

Головних екзокриноцитів

334 / 6854
Особам, які бажають схуднути, рекомендують включати в харчовий раціон побільше нежирної яловичини. Якими властивостями білків це пояснюється?

Поганим всмоктуванням

Тривалою затримкою в шлунку

Низькою калорійністю

Наявністю найбільшого специфічно-динамічного дії

Швидким насиченням

335 / 6854
Чоловік чину 65 років, що страждає на подагру, скаржиться на болі в області нирок. При ультразвуковому дослідженні встановлено наявність ниркових каменів. В результаті якого процесу утворюються ниркові камені?

Відновлення цистеїну

Розпаду пуринових нуклеотидів

катаболізм білків

орнітінового циклу

Розпаду гема

336 / 6854
У хворого з діагнозом ' злоякісний карціноід ' різко підвищено вміст серотоніну в крові. З якої амінокислоти може утворитися даний біогенний амін?

Триптофану

Метіоніну

аланін

треоніну

лейцин

337 / 6854
При електронномікроскопіческом дослідженні гіалінового хряща виявляються клітини з добре розвиненою гранулярной ендоплазматичної мережею, комплексом Гольджі. Яку функцію виконують ці клітини?

Освіта міжклітинної речовини

Депонування жиру

Депонування глікогену

Руйнування межкл еточного речовини хряща

Трофіка хрящової тканини

338 / 6854
У хворого на шкірі обличчя поступово розвинулася бляшка з некрозом і виразкою в центрі. При патогистологическом дослідженні біоптату виявлено розростання атипових епітеліальних клітин з великою кількістю патологічних мітозів. Який найбільш ймовірний діагноз?

Рак шкіри

Саркома

Трофическая виразка

Фиброма

Папілома

339 / 6854
У хворого міастенією після призначення прозерину з'явилася нудота, діарея, посмикування м'язів мови і скелетних м'язів. Чим можна купірувати інтоксикацію?

мезатон

ізадрину

фізостигмін

Пиридостигмина бромидом

метацин

340 / 6854
У хворого внутрішньосуглобове перелом шийки стегнової кістки. Спостерігається асиметрія головки. До акая структура буде найбільш ймовірно пошкоджена?

Зв'язка головки стегнової кістки

Стегновий нерв

запирательной артерія

Круговий пояс

Запірательний нерв

341 / 6854
У хворого інфаркт міокарда в області передньої стінки лівого шлуночка. У басейні якої артерії виникло порушення кровообігу?

Передніх шлуночкових гілок правої вінцевої артерії

Лівою крайової гілки лівої вінцевої артерії

Огинає гілки лівої вінцевої артерії

Предсердно - шлуночкових гілок лівої вінцевої артерії

Передньої міжшлуночкової гілки лівої вінцевої артерії

342 / 6854
Хворий, що страждав хронічним гнійним остеомієлітом, помер від хронічної ниркової недостатності. При розтині виявлено великі щільні нирки беложелтого кольору з сальним блиском на зрізі. Який найбільш веро ятний діагноз?

Підгострий гломерулонефрит

Септичний нефрит

Гострий некротичний нефроз

Амилоидоз нирок

Хронічний гломерулонефрит

343 / 6854
До лікаря звернулися пацієнти з подібними скаргами: слабість, болі в кишечнику, розлад шлунково-кишкового тракту. Після дослідження фекалій з'ясувалося, що термінової госпіталізації підлягає один з пацієнтів, у якого виявлено цисти з чотирма ядрами. Для якого найпростішого характерні такі цисти?

Кишкова амеба

Тріхомонада

Лямблії

балантидії

Дизентерийная амеба

344 / 6854
У хворого після важкої травми розвинувся шок і з'явилися ознаки гострої ниркової недостатності (ОПН). Що є провідним механізмом розвитку гострої ниркової недостатності в даному випадку?

Підвищення тиску в капсулі клубочка

Падіння артеріального тиску

Зниження онкотичного тиску крові

Порушення відтоку сечі

Підвищення тиску в ниркових артеріях

345 / 6854
У немовляти відзначається блювота і пронос, загальна дистрофія, гепато- і спленомегалія. При припинення вигодовування молоком симптоми зменшуються. Який основний наследст венний дефект буде відзначатися в патогенезі?

Недостатність глюкозо-6-фосфатдегідрогенази

Порушення обміну галактози

Порушення обміну тирозину

Гиперсекреция залоз зовнішньої секреції

Порушення обміну фенілаланіну

346 / 6854
З хімічного виробництва в токсикологічне відділення доставлений хворий з отруєнням ртуттю. Який препарат слід використати в даній ситуації?

Налоксон

Ентеросорбент СКН

ізонітрозину

Унітіол

Активоване вугілля

347 / 6854
У хворого в 2 рази збільшена щитовидна залоза. При пальпації заліза щільна, поверхня нерівномірна, горбиста. При гістологічному дослідженні - дифузна інфільтрація тканин залози лімфоцитами, плазматичними клітинами з утворенням фолікулів і посилене розростання сполучної тканини. Який най більш ймовірний діагноз?

Зоб Хасімото

Зоб Ріделя

Спорадичний зоб

Ендемічний зоб

Дифузний токсичний зоб

348 / 6854
Дитина 10 років переніс кілька атак ревматизму. При його клінічному обстеженні було встановлено, що мали місце запальні явища в суглобах, і виявилися ознаки недостатності мітрального клапана. Яке з патологічних явищ у даного хворого можна віднести до поняття 'хвороба'?

Запалення суглобів

Ревматизм

Недостатність мітрального клапана

Артрит

Порок мітрального клапана

349 / 6854
У хворого діагностовано мегалобластная анемія. Недостатнє кількість якого речовини може призводити до розвитку цієї хвороби?

Гліцину

Магнію

Меди

ціанокобаламін

холекальциферол

350 / 6854
При дослідженні крові у хворого в иявлена виражена гіпоглюкоземія натщесерце. При дослідженні біоптату печінки виявилося, що в клітинах печінки НЕ відбувається синтез глікогену. Недостатність якого ферменту є причиною захворювання?

фосфорилазу

Фруктозодіфосфатази

глікогенсинтетазу

піруваткарбоксілази

альдолаза

351 / 6854
При розтині жінки 40 років, яка страждала ревматоїдним артритом, виявлено збільшену щільну селезінку. На розрізі її тканина коричнево - червоного кольору з збільшеними фолікулами, які мають вигляд напівпрозорих сірувато - білуватих зерен. Який патологічний процес найбільш вірогідний?

Сальна селезінка

Гіаліноз селезінки

Сагова селезінка

порфірний селезінка

глазурного селезінка

352 / 6854
В лабораторії досліджують мокроту хворого на туберкульоз. Який метод пофарбовані ания слід використовувати для виявлення збудників туберкульозу?

Нейссера

Грама - Синьова

Гімза - Романовського

Бурри - Гінса

Циля - Нільсена

353 / 6854
У новонародженого діагностовано порушення розвитку міокарда шлуночка. З порушенням розвитку якого ембріонального джерела пов'язана ця патологія?

парієтальної спланхноплеври

Вісцеральної спланхноплеври

мезенхіми

ектодерма

ентодерми

354 / 6854
У хворого запальний гнійний процес шкіри першого межпальцевого проміжку ноги. Які лімфатичні вузли є регіональними для зазначеного ділянки і відреагують болем і припухлістю?

Поверхневі пахові

Передні великогомілкової

Задні великогомілкової і підколінні

Зовнішні клубові

Поверхневі і глибокі пахові

355 / 6854
Аспірин надає проти вовоспалітельное дію, так як пригнічує активність циклооксигенази. Рівень будь біологічно активних речовин буде при цьому знижуватися?

Катехоламінів

лейкотрієнів

простагландинів

біогенні амінів

йодтироніни

356 / 6854
З гнійної рани хворого виділений патогенний стафілокок і визначена чутливість його до антибіотиків: пеніцилін - зона затримки росту 8 мм ; оксацилін - 9 мм ; ампіцилін - 10 мм ; гентаміцин - 22 мм ; линкомицин - 11 мм. Який антибіотик необхідно вибрати для лікування хворого?

Пеніцилін

Ампіцилін

Линкомицин

Гентамицин

Оксациллин

357 / 6854
В медико - генетичну консультацію звернулася жінка з приводу ризику захворювання на гемофілію у свого сина. Її чоловік страждає цим захворюванням з народження. Жінка і її батьки здорові в відношенні гемофілії. Яка ймовірність появи хвороби у хлопчика в даній родині?

Все хлопчики будуть хворими

Усі хлопчики будуть здоровими

50% хлопчиків будуть хворими

25% хлопчиків будуть хворими

75% хлопчиків будуть хворими

358 / 6854
Хворий 55 років спостерігається у ендокринолога з приводу порушення ендокринної функції підшлункової залози, що проявляється зменшенням кількості гормону глюкагону в крові. Функція яких клітин цієї залози порушена в цьому випадку?

S-клітини острівців Лангерганса

А-клітини острівців Лангерганса

бета-клітини острівців Лангерганса

B2-клітини острівців Лангерганса

РР-клітини острівців Лангерганса

359 / 6854
У чоловіка 42 років, який страждає на подагру, в крові підвищена концентрація сечової кислоти. Для зниження рівня сечової кислот и йому призначений аллопуринол. Конкурентною інгібітором якого ферменту є алопуринол?

аденозіндезамінази

Аденінфосфорібозілтрансферази

Гуаніндезамінази

ксантиноксидазу

Гіпоксантінфосфорібозілтрансферази

360 / 6854
У молодого людини в м'яких тканинах лівого стегна з'явилось безболісне новоутворення без чітких меж. У біоптаті тканини новоутворення нагадують риб'яче м'ясо, складаються з незрілих фібробластоподібних клітин з численними митозами, проростають в м'язи. Який найбільш ймовірний діагноз?

міосаркома

Міома

Фиброма

Рак

Фібросаркома

361 / 6854
У хворого 24 років через півтори тижні після важкої стрептококової ангіни проявилася набряклість обличчя, підвищився артеріальний тиск. Гематурія і протеїнурія 1,2 г / л. У крові виявлено антистрептококових анти тіла і зниження компонентів комплементу. У микрососудах яких структур найбільш імовірна локалізація скупчень імунних комплексів, що зумовили розвиток нефропатії?

Проксимальний відділ канальців

Петля Генле

Спадний відділ канальців

Клубочки

Піраміди

362 / 6854
У чоловіка 36 років з черепно - мозкової травмою дихання слабке, пульс ниткоподібний, рефлекси відсутні. Який шлях введення пірацетаму найбільш доцільний в даному випадку?

Внутрішньовенний

Пероральний

Підшкірний

Ректальний

Інгаляційний

363 / 6854
При мікроскопічному дослідженні біоптату шийки матки у хворої на хронічний ендоцервікоз виявлена клітинна і ядерна атипія багатошарового плоского епітелію, патологічні мітози, а також рогові перлини в глибині епітеліальних шарів. Який наиболе е ймовірний діагноз?

Залозистий рак

Перехідно рак

Анапластичний рак

Плоскоклітинний рак з зроговінням

Плоскоклітинний рак без зроговіння

364 / 6854
У хворого виник спазм гладенької мускулатури бронхів. Фізіологічно обгрунтованим буде dикористання для зняття нападу активаторів:

α- і β-адренорецепторів

Н-холінорецепторів

М-холінорецепторів

Β-адренорецепторів

α-адренорецепторів

365 / 6854
При дослідженні каріотипу у пацієнта були виявлені два типи клітин в однаковому кількості з хромосомними наборами 46XY і 47XXY. Який найбільш ймовірний діагноз?

Синдром Дауна

Синдром Патау

Нормальний каріотип

Моносомія - Х

Синдром Клайнфельтера

366 / 6854
При всіх формах розмноження (статеве і нестатеве розмноження) елементарними дискретними єдиний іцамі спадковості є:

Один ген

Одна пара нуклеотидів

Дві ланцюжка молекули ДНК

Одна ланцюжок молекули ДНК

Один нуклеотид

367 / 6854
В хірургічний кабінет звернувся чоловік, якого покусала невідома собака. Широкі рвані рани локалізовані на обличчі. Яку лікувально - профілактичну допомогу потрібно надати для профілактики сказу?

Почати імунізацію антирабічною вакциною

Госпіталізувати хворого і містити під наглядом лікаря

Призначити комбіновану антибіотикотерапію

Терміново ввести вакцину АКДС

Терміново ввести нормальний гамма - глобулін

368 / 6854
При аналізі крові у хворого залишковий азот склав 48 ммоль / л, сечовина -15,3 ммоль / л. Про захворюванні якого органу свідчать результати цього аналізу?

Кишечника

Шлунка

Печінки

Нирок

селезінки

369 / 6854
У жінки 62 років розвинулася катаракта (помутніння кришталика) на тлі цукрового діабету. Посилення якого процесу при діабеті є причиною помутніння кришталика?

протеолізу білків

глюконеогенез

Глікозіт ілірованія білків

кетогенез

ліполіз

370 / 6854
Терапія анаприліном позитивно вплинула на динаміку хвороби у жінки 44 років, яка страждає стенокардією. Який головний механізм дії цього препарату?

Зменшення окислювального обміну в міокарді внаслідок блокади ферментів циклу Кребса

Блокада β - адреноблокатори і зниження потреби міокарда в кисні

Зниження потреби і збільшення надходження кисню в міокард

Зменшення енерговитрат міокарда внаслідок зниження навантаження

Збільшення надходження кисню в міокард

371 / 6854
У людини внаслідок тривалого голодування швидкість клубочкової фільтрації зросла на 20%. Що є найбільш вірогідною причиною зміни фільтрації в зазначених умовах?

Зниження онкотичного тиску плазми крові

Вище ня проникності ниркового фільтру

Підвищення коефіцієнта фільтрації

Підвищення системного артеріального тиску

Підвищення ниркового плазмотока

372 / 6854
В яких клітинах на протязі життя НЕ спостерігається мітоз, і кількісне зміст ДНК залишається постійним?

В кровотворних

У епідермісі

В нейронах

В м'язових (поперечно - смугастих)

В м'язових (гладких)

373 / 6854
У хворої 55 років виявлено збільшення розмірів гіпофіза, гіперплазія кори надниркових залоз. АТ - 190/90 мм рт. ст.; зміст глюкози в крові - 20 ммоль / л, має місце глюкозурія, ожиріння, гірсутизм. Для якої патології характерні виявлені зміни?

Синдром Іценко - Кушинга

Хвороба Іценко - Кушинга

Хвороба Барракера - Симмондса

Адіпозогенітальная дистрофія

Хвороба Аддісона

374 / 6854
При аутопсії померлого на шкірі дистальних відділів нижніх кінцівок виявлено багряно - червоні плями, бляшки і вузли (саркома Капоши). Виявлено також гостра пневмонія, викликана пневмоцистами. Для якого захворювання характерні дані зміни?

Сибірська виразка

Дифтерія

Кір

Грип

СНІД

375 / 6854
У жінки 45 років в період цвітіння трав з'явилося гостре запальне захворювання верхніх дихальних шляхів і очей: гіперемія, набряк, слизові виділення. Який вид лейкоцитозу буде найбільш характерним при цьому?

Еозинофілія

Базофілія

Нейтрофілія

Моноцитоз

Лимфоцитоз

376 / 6854
У жінки виявлено пухлину яєчника. Показана операція. Яку зв'язку повинен перерізати хірург, щоб відділити яєчник від матки?

Круглу зв'язку матки

Бічну пупкову зв'язку

Широку зв'язку матки

Зв'язку, підвішуючу яєчник

Власну зв'язку яєчника

377 / 6854
У хворого в анамнезі відзначений тривалий зростання кінцівок, подовжені ' павукові ' пальці, дефекти кришталика ока, аномалії серцево - судинної системи. Інтелект в нормі. Які ф енотіпіческіе ознаки ще можуть бути у цього хворого:

Плоске обличчя і широке перенісся

Порушення розвитку сполучної тканини

Недорозвинення нижньої щелепи

Недорозвинення гонад

Розщеплення м'якого та твердого піднебіння

378 / 6854
У хворого з головним болем, високою температурою, ознобом, кашлем з мокротиння виділили палички овоидной форми з біполярної забарвленням, грамнегативні. В мазку з бульонной культури розташовуються ланцюжками, на агарі утворюють колонії R- форми. Для якого захворювання це характерно?

Туберкульоз

Дифтерія

Стрептококова ангіна

Чума

Менінгококовий назофарингіт

379 / 6854
При гістологічному дослідженні стінки аорти виявлено велику кількість ксантомних клітин, розташованих переважно в інтимі. При якому захворюванні можлива така картина?

нодозной периартериит

Атеросклероз

Сифілітичний мезаортит

Гіпертонічна хвороба

Неспецифічний аортоартеріїт

380 / 6854
В приймально - діагностичне відділення доставлена жінка 38 років з маточним кровотечею. Які найбільш ймовірні змінений ія з боку крові відбудуться?

Лейкоцитоз

Зменшення гематокритного числа

Лейкопенія

Збільшення гематокритного числа

Еритроцитоз

381 / 6854
У хворого камінь загального жовчного протока припинив надходження жовчі в кишечник. Порушення якого процесу травлення при цьому спостерігається?

Переваривание білків

Переваривание жирів

Всмоктування білків

Переваривание вуглеводів

Всмоктування вуглеводів

382 / 6854
В лікарню доставили хворого на цукровий діабет в несвідомому стані. Дихання типу Кусмауля, артерії ально тиск 80/50 мм рт. ст., запах ацетону з рота. Накопиченням в організмі яких речовин можна пояснити виникнення даних розладів?

Модифікованих ліпопротеїдів

Кетонових тел

Складних вуглеводів

Молочної кислоти

Вугільної кислоти

383 / 6854
В відділення травматології доставлений хворий з розчавлених м'язової тканини. Який біохімічний показник сечі при цьому буде збільшений?

Глюкоза

Креатинін

Загальні ліпіди

Сечова кислота

Мінеральні солі

384 / 6854
У хворого на туберкульоз після тривалого лікування з'явилися шум і дзвін у вухах, зниження слуху, висип на шкірі, набряк слизових оболонок і порушилася координація рухів. Після скасування препарату стан хворого значно покращився. Який препарат приймав хворий?

Стрептомицина сульфат

Ізоніазид

Етамбутол

Рифампіцин

Бепаск

385 / 6854
В лікарню швидкої допомоги доставили дитину 7 років в стані алергічного шоку, що розвинувся після укусу оси. У крові підвищена концентрація гістаміну. В результаті якої реакції утворюється цей амін?

дегидрированием

Дезамінування

Відновлення

декарбоксилюванню

Гідроокісленія

386 / 6854
У дитини, хворої на дифтерію, через 10 днів після введення антитоксичної протидифтерійної сироватки з'явилися висипання на шкірі, які супроводжувалися сильним свербінням, підвищилася температура тіла до 38 ° С, з'явилися болі в суглобах. Яка причина цих явищ?

Атопія

Анафілактична реакція

Гіперчутливість сповільненого типу

Контактна алергія

Сироваткова хвороба

387 / 6854
У пацієнта, який півтора місяці тому назад переніс інфаркт міокарда, діагностований синдром Дресслера з характерною тріадою: перикардит, плеврит, пневмонія. Причиною його розвитку вважається:

Інтоксикація організму продуктами некрозу

Зниження резистентності до інфекційних агентів

Викид в кров міокардіальних ферментів

Сенсибілізація організму антигенами міокарда

Активація сапрофітної мікрофлори

388 / 6854
Під час емоційного збудження частота серцевих скорочень у людини 30 років досягла 112 в хв. Який відділ проводить системи серця відповідає за це зміна?

Атріовентрикулярний вузол

Ніжки пучка Гіса

Пучок Гіса

Волокна Пуркіньє

Синоатріальна вузол

389 / 6854
Хворий госпіталізований з гострою серцево - судинної недостатністю. Який препарат необхідно використовувати в даному випадку?

Дигоксин в таблетках

Адреналін

Строфантин

Аміодарон

Дигитоксин

390 / 6854
Гризуни є резервуаром збудників багатьох хвороб. З чим це пов'язано в першу чергу?

Приналежність гризунів до найбільш численному ряду класу ссавців

Властивість гризунів швидко розмножуватися

Проживання в умовах, де ектопаразити використовують гризунів як джерело живлення

Біологічні особливості гризунів, що сприяють обміну паразитами і збудниками з людиною

Приналежність гризунів до важливим компонентам наземних біоцен озов

391 / 6854
У 4 місячного дитини яскраво виражені прояви рахіту. Розладів травлення НЕ відзначається. Дитина багато знаходиться на сонці. В протягом 2 х місяців дитина отримував вітамін D 3, однак прояви рахіту НЕ зменшилися. Порушенням синтезу якої речовини можна пояснити розвиток рахіту у цього дитини?

Інсуліну

Паратгормону

Тироксину

кальцитоніну

кальцитріолом

392 / 6854
Тварині, сенсибілізованій туберкуліном, внутрибрюшинно введений туберкулін. Через 24 години при лапаратомии виявлено венозну гіперемію і набряк очеревини. В мазках - відбитках з очеревини велика кількість лімфоцитів і моноцитів. Який патологічний процес у тварини?

Серозное запалення

Гнійне запалення

Асептичне запалення

Фібринозне запалення

Алергічне запалення

393 / 6854
Альбіноси погано переносять сонячне загар, у них з'являються опіки. Порушення метаболізму який кислоти лежить в основі цього явища?

Гістидину

Метіоніну

фенілаланін

глутамінова кислоти

Триптофану

394 / 6854
При повному (з водою) аліментарному голодуванні розвинулись генералізовані набряки. Який з патогенетичних факторів у цьому випадку є провідним?

Підвищення осмотичного тиску міжклітинної рідини

Підвищення онкотичного тиску тканинної рідини

Зниження гідростатичного тиску міжклітинної рідини

Зниження осмотичного тиску плазми крові

Зниження онкотичного тиску плазми крові

395 / 6854
При штовханні штанги спортсмен закидає голову назад для максимального підвищення тонус а м'язів - розгиначів верхніх кінцівок. Де розташовані центри рефлексів, що виникають при цьому?

У спинному мозку

В червоних ядрах

В ядрах Дейтерса

У рухової корі

В базальних гангліях

396 / 6854
Хворому проводять правобічну пульмонектомію по приводу раку легкого. Який порядок розміщення анатомічних утворень кореня правого легкого (в напрямку зверху вниз)?

Відня, артерія, бронх

Артерія, вени, бронх

Артерія, бронх, вени

Бронх, артерія, вени

397 / 6854
При реєстрації ЕКГ хворого з гіперфункцією щитовидної залози зареєстровано збільшення частоти серцевих скорочень. Скорочення якого елемента ЕКГ про це свідчить?

Інтервалу Р-Т

Сегмента PQ

Комплексу QRS

Інтервалу RR

Інтервалу PQ

398 / 6854
При розтині труп а хворого, який помер від легеневої недостатності, виявлено збільшене легке з вогнищами темно - червоного, білого, рожево - жовтого кольору, некротичний трахеобронхіт. Яким захворюванням найбільш ймовірно страждав хворий?

Фиброзирующий альвеоліт

Корова пневмонія

Токсична форма грипу

Грипозна пневмонія

Крупозная пневмонія

399 / 6854
При обстеженні у хворого виявлено підвищений вміст ліпопротеїнів низької щільності в сироватці крові. Яке захворювання можна очікувати у цього хворого?

Атеросклероз

Запалення легенів

Гастрит

Пошкодження нирок

Гострий панкреатит

400 / 6854
Внаслідок блокади іонних каналів мембрани клітини її потенціал спокою зменшився з -90 мВ до -70 мВ. Які канали заблоковані?

Хлорні

Магнієві

Натрієві

Калієві

Кальцієві

401 / 6854
Підвищена ламкість судин, руйнування емалі і дентину у хворих на цингу здебільшого обумовлені порушенням дозрівання колагену. Який етап модифікації проколагену порушений при цьому авітамінозі?

Утворення поліпептидних ланцюгів

Видалення з проколагену С-кінцевого пептиду

Відщеплення N-кінцевого пептиду

Гідроксилювання проліну

Глікозилювання гидроксилізинових залишків

402 / 6854
Дитина 10 років скаржиться на слабкість, нудоту, дратівливість. На білизні знайдені гельмінти білого кольору завдовжки 5-10 мм. Під час мікроскопії зіскрібка з періанальних складок виявлені безбарвні яйця несиметричної форми. Укажіть, який гельмінт паразитує у дитини?

Аскарида людська

Волосоголовець

Трихінела

Гострик

Кривоголовка

403 / 6854
Під час розтину мертвонародженої дитини виявлено аномалію розвитку серця: шлуночки не розмежовані, з правої частини виходить суцільний артеріальний стовбур. Для якого класу хребетних тварин характерна подібна будова серця?

Амфібії

Риби

Птахи

Ссавці

Рептилії

404 / 6854
Чоловік 37 років надійшов до хірургічного відділення з явищами гострого панкреатиту: блювання, пронос, брадикардія, гіпотензія, слабість, явища зневоднювання організму. Який препарат найбільш доцільно використати в першу чергу?

Контрикал

Но-шпу

Платифілін

Ефедрин

Етаперазин

405 / 6854
У 12-річного хлопця часто виникають вірусні та бактеріальні інфекції, екзематозні ураження шкіри. Під час обстеження виявлено зменшення Тлімфоцитів та IgМ при нормальному вмісті IgA і IgG. Якій вид патології імунної системи спостерігається у хворого?

Гіпогаммаглобулінемія Брутона

Синдром Шерешевського-Тернера

Спадковий дефіцит системи комплементу

Гіпоплазія тимуса

Комбінований імунодефіцит

406 / 6854
Під час обстеження людини встановлено, що хвилинний об'єм серця дорівнює 3500 мл, систолічний об'єм – 50 мл. Якою є частота серцевих скорочень у цієї людини?

70 скорочень за хвилину

50 скорочень за хвилину

90 скорочень за хвилину

80 скорочень за хвилину

60 скорочень за хвилину

407 / 6854
У жінки виявлено пухлину яєчника. Показана операція. Яку зв`язку повинен перерізати хірург, щоб відділити яєчник від матки?

Бічну пупкову зв’язку

Зв’язку, що підвішує яєчник

Широку зв’язку матки

Власну зв’язку яєчника

Круглу зв’язку матки

408 / 6854
На рентгенограмі таза у новонародженого визначаються три самостійні кістки, які з’єднані хрящем у ділянці кульшової западини. Які це кістки?

Сіднична, стегнова, крижова

Клубова, лобкова, сіднична

Крижова, лобкова, куприкова

Клубова, крижова, куприкова

Лобкова, сіднична, стегнова

409 / 6854
У крові чоловіка 26 років виявлено 18% еритроцитів сферичної, сплощеної, кулеподібної та остистої форми. Інші еритроцити були у формі двоввігнутих дисків. Як називається таке явище?

Фізіологічний пойкілоцитоз

Фізіологічний анізоцитоз

Еритроцитоз

Патологічний анізоцитоз

Патологічний пойкілоцитоз

410 / 6854
Під час розтину тіла чоловіка, який тривалий час зловживав алкоголем, виявлено: печінка малих розмірів, щільна, дрібногорбиста. Мікроскопічно: псевдочасточки дрібні, розподілені вузькими прошарками сполучної тканини з лімфомакрофагальними інфільтратами; гепатоцити у стані великокрапельної жирової дистрофії. Який найбільш імовірний діагноз?

Жировий гепатоз

Алкогольний цироз

Хронічний активний алкогольний гепатит

Токсична дистрофія печінки

Хронічний персистуючий алкогольний гепатит

411 / 6854
У крові пацієнта вміст глюкози натщесерце був 5,65 ммоль/л, через 1 годину після цукрового навантаження становив 8,55 ммоль/л, а через 2 години – 4,95 ммоль/л. Такі показники характерні для:

Здорової людини

Хворого з тиреотоксикозом

Хворого з прихованим цукровим діабетом

Хворого з інсуліннезалежним цукровим діабетом

Хворого з інсулінозалежним цукровим діабетом

412 / 6854
У хворого, що тривалий час приймав глюкокортикоїди, в результаті відміни препарату виникло загострення наявного захворювання, зниження артеріального тиску, слабкість. Чим можна пояснити ці явища?

Виникненням недостатності наднирників

Кумуляцією

Звиканням до препарату

Сенсибілізацією

Гіперпродукцією АКТГ

413 / 6854
Хворий похилого віку скаржиться на головний біль, запаморочення, швидку втомлюваність, погіршення пам’яті. В анамнезі черепно-мозкова травма. Препарат якої групи необхідно призначити хворому?

Снодійні

Нейролептики

Аналгетики

Ноотропні

Транквілізатори

414 / 6854
З хімічного виробництва в токсикологічне відділення доставлено хворого з отруєнням ртуттю. Який препарат слід використати в даній ситуації?

Налоксон

Активоване вугілля

Унітіол

Ентеросорбент СКН

Ізонітрозин

415 / 6854
При обстеженні хворого виявлене новоутворення в білій речовині півкуль великого мозку з локалізацією у коліні та передньому відділі задньої ніжки внутрішньої капсули. Волокна якого провідного шляху мозку будуть зруйновані?

Tr. frontothalamicus

Tr. parietooccipitopontinus

Тr. thalamocorticalis

Tr. frontopontinus

Tr. pyramidalis

416 / 6854
При бактеріологічному дослідженні випорожнень чотиримісячної дитини з симптомами гострої кишкової інфекції на середовищі Ендо виросли у великій кількості червоні колонії. Які це можуть бути мікроорганізми?

Шигели

Стрептококи

Сальмонели

Ешерихії

Стафілококи

417 / 6854
Хворий протягом двох тижнів отримував медикаментозну терапію з приводу психозу. Стан хворого поліпшився, однак невдовзі з’явились ригідність, тремор, гіпокінезія. Який з наведених препаратів викликає такі ускладення?

Дифенін

Хлордіазепоксид

Аміназин

Сиднокарб

Імізин

418 / 6854
На розтині тіла померлого виявлено: м’яка мозкова оболонка верхніх відділів півкуль головного мозку різко повнокровна, жовто-зеленого кольору, просочена гнійним та фібринозним ексудатом, що нагадує чіпець. Для якого захворювання характерна така картина?

Менінгіт при сибірці

Грипозний менінгіт

Менінгококовий менінгіт

Менінгіт при висипному тифі

Туберкульозний менінгіт

419 / 6854
У молодих здорових батьків народилася дівчинка, білява, з блакитними очима. У перші ж місяці життя у дитини розвинулися дратівливість, неспокій , порушення сну і харчування, а обстеження невропатолога показало відставання у розвитку дитини. Який метод генетичного обстеження дитини слід застосувати для точного встановлення діагнозу?

Біохімічний

Цитологічний

Генеалогічний

Популяційно-статистичний

Близнюковий

420 / 6854
Дистрофічні зміни серцевого м’яза супроводжуються розширенням порожнин серця, зниженням сили серцевих скорочень, збільшенням об’єму крові , що залишається під час систоли в порожнині серця, переповненням вен. Для якого стану серця це характерно?

Тоногенна дилатація

Міогенна дилатація

Тампонада серця

Кардіосклероз

Аварійна стадія гіперфункції та гіпертрофії

421 / 6854
При визначенні групи крові за системою АВ0 аглютинацію еритроцитів досліджуваної крові викликали стандартні сироватки I та II груп і не викликала III групи. Які аглютиногени містяться в цих еритроцитах?

С

В

А та В

D та C

А

422 / 6854
У жінки з первинним гіперпаратиреоїдизмом періодично повторюються напади ниркової коліки. Ультразвукове обстеження показало наявність дрібних каменів у нирках. Яка найбільш імовірна причина утворення цих каменів?

Гіперурикемія

Гіперкаліємія

Гіперфосфатемія

Гіперхолестеринемія

Гіперкальціємія

423 / 6854
У хворого на глаукому спостерігається підвищення внутрішньоочного тиску при нормальній секреції водянистої вологи циліарним тілом. З ушкодженням яких структур стінки очного яблука пов’язане порушення відтоку рідини з передньої камери?

Війкового м’яза

Циліарного тіла

Венозного синуса

Судинної оболонки

Заднього епітелію рогівки

424 / 6854
Який механізм тепловіддачі найбільш ефективно спрацьовує при перебуванні людини в умовах 80% вологості повітря та температурі навколишнього середовища +35°С?

--

Конвекція

Радіація

Теплопроведення

Випаровування

425 / 6854
Провідником наукової експедиції по Індії був місцевий житель, який ніколи не розлучався зі своїм улюбленим собакою. Якими інвазійними захворюваннями можуть бути заражені члени експедиції при контакті з цим собакою, якщо він є джерелом інвазії?

Теніозом

Парагонімозом

Ехінококозом

Фасціольозом

Дикроцеліозом

426 / 6854
Хворій 34 років 3 роки тому було встановлено діагноз гломерулонефриту. За останні 6 місяців з'явилися набряки. Що лежить в основі їх розвитку?

Протеїнурія

Гіперосмолярність плазми

Порушення білковоутворюючої функції печінки

Гіперпродукція вазопресину

Гіперальдостеронізм

427 / 6854
При жировій інфільтрації печінки порушується синтез фосфоліпідів. Вкажіть, яка з наведених речовин може посилювати процеси метилювання в процесі синтезу фосфоліпідів?

Глюкоза

Метіонін

Гліцерин

Аскорбінова кислота

Цитрат

428 / 6854
Хворому 50 років з метою лікування черевного тифу призначений левоміцетин, але на наступний день стан хворого погіршився, температура підвищилася до 39,6°С. Чим пояснити погіршення стану хворого?

Алергічною реакцією

Реінфекцією

Дією ендотоксинів збудника

Приєднанням вторинної інфекції

Нечутливістю збудника до левоміцетину

429 / 6854
У хворого 30 років з дизентерією, підтвердженою бактеріологічно, з'явились ознаки парапроктиту. Про яку стадію місцевих змін найбільш імовірно йде мова у даного хворого?

Фібринозний коліт

Стадія утворення виразок

Катаральний коліт

Фолікулярний коліт

Стадія загоєння виразок

430 / 6854
Тривале перебування в умовах спеки викликало у людини спрагу. Сигналізація від яких рецепторів, перш за все, зумовила її розвиток?

Глюкорецепторів гіпоталамусу

Осморецепторів печінки

Барорецепторів дуги аорти

Осморецепторів гіпоталамусу

Натрієвих рецепторів гіпоталамусу

431 / 6854
Протягом двох тижнів хвора приймала мікстуру, призначену невропатологом з приводу неврастенії. Самопочуття хворої дещо поліпшилося, однак незабаром з’явились скарги на нежить, кон’юнктивіт, шкірні висипи, млявість та послаблення пам’яті. Був встановлений діагноз “бромізм”. Що доцільно призначити для послаблення симптомів?

Поліглюкін

Аспаркам

Розчин глюкози 5%

Натрію хлорид

432 / 6854
Хворого з явищами енцефалопатії госпіталізували до неврологічного стаціонару і виявили кореляцію між наростанням енцефалопатії і речовинами, що надходять із кишечнику до загального кровотоку. Які з'єднання, що утворюються в кишечнику, можуть викликати ендотоксемію?

Біотин

Ацетоацетат

Бутират

Індол

Орнітин

433 / 6854
В експерименті у кролика було видалено верхній шийний вузол симпатичного стовбура. На боці видалення спостерігаються почервоніння і підвищення температури шкіри голови. Яка форма порушень периферичного кровообігу розвинулася у

Венозна гіперемія

Нейротонічна артеріальна гіперемія

Нейропаралітична артеріальна гіперемія

Стаз

Метаболічна артеріальна гіперемія

434 / 6854
До травматологічного відділення доставлено чоловіка з закритою травмою живота праворуч та підозрою на розрив печінки. В якому з приведених утворень очеревини слід чекати накопичення крові?

Excavatio rectovesicalis

Recessus duodenalis inferior

Recessus intersigmoideus

Fossa ischio-analis

Bursa omentalis

435 / 6854
У пацієнта, який чітко виконував рекомендації по дотриманню певної дієти протягом 10 днів, було проведене дослідження величини дихального коефіцієнту. Встановлено, що він дорівнює 1. Якої дієти дотримувався пацієнт?

З переважним вмістом вуглеводів

З переважним вмістом жирів і вуглеводів

Змішаної

З переважним вмістом білків і жирів

З переважним вмістом білків і вуглеводів

436 / 6854
У препаратах подані зрізи органів кровотворення та імуногенезу людини, для яких характерна наявність лімфоїдної тканини, що формує різноманітні структури (лімфатичні вузлики, дольки, тяжі). Визначте, в якому з органів відбувається антигеннезалежна проліферація та диференціювання лімфоцитів?

Лімфатичні вузли

Гемолімфатичні вузли

Тимус

Селезінка

Мигдалик

437 / 6854
За даними бактеріоскопії мазку з уретри у хворого виявлено гонорею. Враховуючи, що препаратами вибору для лікування гонореї є фторхінолони, хворому необхідно призначити:

Цефазолін

Фуразолідон

Уросульфан

Фторурацил

Ципрофлоксацин

438 / 6854
Хворому з метою відновлення дихання при отруєнні чадним газом було введено аналептичний засіб рефлекторного типу дії з групи Н-холіноміметиків. Який засіб було призначено хворому?

Пентамін

Атропіну сульфат

Мезатон

Лобеліну гідрохлорид

Адреналіну гідрохлорид

439 / 6854
Молода людина під час активного підтягування на перекладині відчула різкий біль у спині. Об'єктивно: біль при спробах руху верхньою кінцівкою, обмеження функцій приведення та пронації. Розтягнення якого м’яза, найбільш імовірно, відбулося?

М. romboideus major

М. subscapularis

М. trapezius

М. latissimus dorsi

М. levator scapulae

440 / 6854
З метою серологічної діагностики коклюшу поставлена розгорнута реакція з коклюшним та паракоклюшним діагностикумами. На дні пробірок, до яких було внесено діагностикум з Bordetella parapertussis, утворився зернистий осад. Які антитіла виявила ця реакція?

Опсоніни

Преципітини

Антитоксини

Бактеріолізини

Аглютиніни

441 / 6854
При тиреотоксикозі підвищується продукція тиреоїдних гормонів Т3 та Т4, розвивається схуднення, тахікардія, психічне збудження та інше. Як саме впливають тиреоїдні гормони на енергетичний обмін в мітохондріях клітин?

Блокують дихальний ланцюг

Блокують субстратне фосфорилювання

Активують субстратне фосфорилювання

Активують окисне фосфорилювання.

Роз`єднують окислення та окисне фосфорилювання

442 / 6854
У комплексному лікуванні гіпертонічної хвороби хворому був призначений сечогінний препарат. Через кілька днів АТ знизився, але виникли ознаки гіпокаліємії. Який препарат міг викликати таке ускладнення?

Фуросемід

Еналаприл

Клофелін

Тріамтерен

Спіронолактон

443 / 6854
Хворий 45 років госпіталізований до хірургічного відділення зі скаргами на раптовий гострий біль в надчеревній ділянці. Після обстеження встановлено діагноз: перфоративна (проривна) виразка задньої стінки шлунка. Куди вилився вміст шлунка в момент перфорації?

В правий брижовий синус

В сальникову сумку

В лівий брижовий синус

В печінкову сумку

В передшлункову сумку

444 / 6854
В медико-генетичну консультацію звернулась жінка з приводу ризику захворювання на гемофілію у свого сина. Її чоловік страждає на дане захворювання з народження. Жінка та її батьки здорові стосовно гемофілії. Яка вірогідність появи хвороби у хлопчика в даній сім'ї?

25% хлопчиків будуть хворими

Всі хлопчики будуть хворі

50% хлопчиків будуть хворими

Усі хлопчики будуть здорові

75% хлопчиків будуть хворими

445 / 6854
При розтині тіла жінки 40 років, яка страждала ревматоїдним артритом, знайдено збільшену щільну селезінку. На розрізі її тканина коричневочервоного кольору зі збільшеними фолікулами, які мають вигляд напівпрозорих сірувато-білуватих зерен. Який патологічний процес найбільш вірогідний?

Глазурна селезінка

Сагова селезінка

Гіаліноз селезінки

Масна селезінка

Порфірна селезінка

446 / 6854
У хворого відзначається підвищена чутливість шкіри до сонячного світла. Його сеча при тривалому стоянні набуває темно-червоного кольору. Яка найбільш імовірна причина такого стану?

Альбінізм

Алкаптонурія

Порфірія

Пелагра

Гемолітична жовтяниця

447 / 6854
Терапія анаприліном позитивно вплинула на динаміку хвороби у жінки 44 років, яка страждає стенокардією. Який головний механізм дії цього препарату?

Збільшення надходження кисню в міокард

Блокада β-адренорецепторів і зниження потреби міокарда в кисні

Зниження потреби і збільшення надходження кисню в міокард

Зменшення енергозатрат міокарда внаслідок зниження навантаження

Зменшення окислювального обміну в міокарді внаслідок блокади ферментів циклу Кребса

448 / 6854
В хірургічний кабінет звернулась людина, яку покусав невідомий собака. Широкі рвані рани локалізовані на обличчі. Яку лікувально-профілактичну допомогу потрібно надати для профілактики сказу?

Призначити комбіновану антибіотикотерапію

Розпочати імунізацію антирабічною вакциною

Госпіталізувати хворого і утримувати під наглядом лікаря

Терміново ввести вакцину АКДП

Терміново ввести нормальний гама-глобулін

449 / 6854
До приймального відділення доставлений хворий зі скаргами на сухість у роті, світлобоязнь та порушення зору. Шкіра гіперемійована, суха, зіниці розширені, тахікардія. При подальшому обстеженні був встановлений діагноз: отруєння алкалоїдами красавки. Який із лікарських засобів доцільно застосувати?

Дипіроксим

Армін

Прозерин

Діазепам

Пілокарпін

450 / 6854
При моделюванні запалення нижньої кінцівки у тварини підвищилася температура тіла, збільшився вміст антитіл та лейкоцитів у крові. Які речовини зумовили розвиток цих загальних реакцій організму при запаленні?

Мінералокортикоїди

Лейкотриєни

Глюкокортикоїди

Інтерлейкіни

Соматомедини

451 / 6854
В крові хворого виявлено низький рівень альбумінів і фібриногену. Зниження активності яких органел гепатоцитів печінки найбільш вірогідно обумовлює це явище?

Комплексу Гольджі

Гранулярної ендоплазматичної сітки

Агранулярної ендоплазматичної сітки

Лізосом

Мітохондрій

452 / 6854
Під час патронажу лікар виявив у дитини симетричну шорсткість щік, діарею, порушення нервової діяльності. Нестача яких харчових факторів є причиною такого стану?

Метіонін, ліпоєва кислота

Лізин, аскорбінова кислота

Фенілаланін, пангамова кислота

Нікотинова кислота, триптофан

Треонін, пантотенова кислота

453 / 6854
Дівчинка 10 років часто хворіє на гострі респіраторні інфекції, після яких спостерігаються множинні точкові крововиливи в місцях тертя одягу. Гіповітаміноз якого вітаміну має місце в дівчинки?

В6

В1

С

В2

А

454 / 6854
У хворого з гіпохромною анемією січеться і випадає волосся, відзначається підвищена ламкість нігтів, порушений смак. Який механізм розвитку зазначених симптомів?

Дефіцит вітаміну А

Дефіцит вітаміну В12

Зниження продукції паратирину

Зниження продукції тиреоїдних гормонів

Дефіцит залізовмісних ферментів

455 / 6854
Аспірин має протизапальну дію, оскільки пригнічує активність циклооксигенази. Рівень яких біологічно активних речовин буде при цьому знижуватися?

Йодтиронінів

Біогенних амінів

Простагландинів

Лейкотриєнів

Катехоламінів

456 / 6854
У чоловіка 60 років після крововиливу в головний мозок настав тривалий сон. Пошкодження яких структур найімовірніше призвело до цього стану?

Чорної субстанції

Ретикулярної формації

Кори великих півкуль

Ядер черепних нервів

Гіпокампа

457 / 6854
Клітину лабораторної тварини піддали надмірному рентгенівському опроміненню. У результаті утворились білкові фрагменти в цитоплазмі. Який органоїд клітини візьме участь у їх утилізації?

Лізосоми

Рибосоми

Комплекс Гольджі

Клітинний центр

Ендоплазматичний ретикулум

458 / 6854
У чоловіка 52 років діагностовано системний амебіаз з ураженням кишечнику, печінки, легень. Який препарат слід призначити?

Метронідазол

Хініофон

Ентеросептол

Тетрациклін

Хінгамін

459 / 6854
До лікаря звернувся хворий зі скаргами на постійну спрагу. Виявлена гіперглікемія, поліурія та підвищений вміст 17-кетостероїдів у сечі. Яке захворювання найбільш імовірне?

Мікседема

Інсулінозалежний діабет

Стероїдний діабет

Глікогеноз I типу

Аддісонова хвороба

460 / 6854
У біоптаті бронха хворого, який зловживає палінням, у потовщеній слизовій оболонці виявлено хронічне запалення і трансформацію одношарового війчастого епітелію в багатошаровий плоский епітелій. Який із процесів найбільш імовірний?

Лейкоплакія

Метаплазія

Гіперплазія епітелію

Плоскоклітинний рак

Гіпертрофія епітелію

461 / 6854
Пацієнту з гострим інфарктом міокарда внутрішньовенно крапельно введено 1500 мл різних розчинів протягом 8 годин, кисень інтраназально. Смерть настала від набряку легень. Що спричинило набряк легень?

Алергічна реакція

Зменшення онкотичного тиску за рахунок гемодилюції

Перевантаження лівого шлуночка об'ємом

Нейрогенна реакція

Інгаляція кисню

462 / 6854
У хворої 43 років на фоні септичного шоку визначається тромбоцитопенія, зменшення фібриногену, поява в крові продуктів деградації фібрину, петехіальні крововиливи. Яка найбільш вірогідна причина цих змін?

ДВС-синдром

Геморагічний діатез

Екзогенна інтоксикація

Аутоімунна тромбоцитопенія

Порушення виробляння тромбоцитів

463 / 6854
До лікаря звернулися пацієнти з подібними скаргами: слабкість, болі в кишечнику, розлад ШКТ. Після дослідження фекалій з'ясувалось, що терміновій госпіталізації підлягає один з пацієнтів, у якого були виявлені цисти з чотирма ядрами. Для якого найпростішого характерні такі цисти?

Кишкова амеба

Лямблія

Трихомонада

Балантидій

Дизентерійна амеба

464 / 6854
До клініки доставили пацієнта 32 років з масивною крововтратою внаслідок автодорожної травми. Пульс 110 уд/хв., частота дихання - 22 за хв., АТ - 100/60 мм рт.ст. Яка зміна крові із перелічених буде найбільш характерною через 1 годину після крововтрати?

Еритропенія

Лейкопенія

Гіпопротеїнемія

Гіповолемія

Гіпохромія еритроцитів

465 / 6854
При диспансерному обстеженні хлопчику 7 років встановлено діагноз - дальтонізм. Батьки здорові, кольоровий зір нормальний. Але у дідуся по материнській лінії така ж аномалія. Який тип успадкування цієї аномалії?

Рецесивний, зчеплений зі статтю

Домінантний, зчеплений зі статтю

Неповне домінування

Аутосомно-домінантний

Аутосомно-рецесивний

466 / 6854
Під час мікроскопічного дослідження збільшеного шийного лімфатичного вузла дівчинки 14 років було знайдено, що тканинна будова вузла порушена, лімфоїдні фолікули відсутні, є ділянки склерозу та вогнища некрозу, клітинний склад вузла поліморфний, присутні лімфоцити, еозинофіли, атипові клітини великих розмірів з багаточасточковими ядрами (клітини БерезовськогоШтернберга) та одноядерні клітини також великих розмірів. Який найбільш імовірний діагноз?

Лімфома Беркітта

Гострий лімфолейкоз

Грибоподібний мікоз

Хронічний лімфолейкоз

Лімфогранулематоз

467 / 6854
Хворий на цукровий діабет після ін'єкції інсуліну знепритомнів, почалися судоми. Який результат може дати біохімічний аналіз крові на вміст цукру?

3,3 ммоль/л

5,5 ммоль/л

8,0 ммоль/л

10,0 ммоль/л

1,5 ммоль/л

468 / 6854
Жінка 45 років страждає на сезонний алергічний риніт, пов’язаний з цвітінням амброзії. Який лікарський засіб з групи стабілізаторів тучних клітин можна застосувати для профілактики даного захворювання?

Діазолін

Фенкарол

Димедрол

Тавегіл

Кетотифен

469 / 6854
Тварині в експерименті перерізали передні корінці п’яти сегментів спинного мозку. Які зміни відбудуться в зоні

Втрата температурної чутливості

Гіперчутливість

Втрата дотикової чутливості

Втрата пропріоцептивної чутливості

Втрата рухів

470 / 6854
У хворого із значними периферійними набряками почергове застосування дихлотіазиду, етакринової кислоти і фуросеміду не сприяло значному діуретичному ефекту. У крові - значне підвищення кількості альдостерону. Вкажіть препарат вибору.

Спіронолактон

Амілорид

Сечовина

Клопамід

Маніт

471 / 6854
У досліді на тварини зруйновано середню частину завитка внутрішнього вуха. Це призведе до порушення сприйняття звуків такої частоти:

Висока

Висока та низька

Низька

Середня

Порушень не буде

472 / 6854
Після накладання джгута у досліджуваного виявили точкові крововиливи. З порушенням функції яких клітин крові це пов’язано?

Еозинофіли

Лімфоцити

Нейтрофіли

Тромбоцити

Моноцити

473 / 6854
Чоловік 65 років, який страждає на подагру, скаржиться на болі в області нирок. При ультразвуковому обстеженні встановлена наявність ниркових каменів. У результаті якого процесу утворюються ниркові камені?

Розпаду пуринових нуклеотидів

Орнітинового циклу

Розпаду гему

Відновлення цистеїну

Катаболізму білків

474 / 6854
Артеріальна гіпертензія у хворого обумовлена стенозом ниркових артерій. Активація якої системи є головною ланкою в патогенезі цієї форми гіпертензії?

Ренін-ангіотензинова

Симпатоадреналова

Парасимпатична

Гіпоталамо-гіпофізарна

Калікреїн-кінінова

475 / 6854
З носоглотки дитини 5 років виділено мікроорганізм, який за морфологічними та біохімічними ознаками ідентичний Corynebacterium diphtheriae, але не утворює екзотоксин. У результаті якого процесу цей мікроорганізм може стати токсигенним?

Пасаж через організм чутливих тварин

Хромосомна мутація

Фагова конверсія

Вирощування у присутності антитоксичної сироватки

Культивування на телуритовому середовищі

476 / 6854
В експерименті ізольований м'яз жаби ритмічно подразнюють електричними імпульсами. Кожний наступний імпульс припадає на період розслаблення попереднього скорочення. Яке скорочення м'язу виникає?

Суцільний тетанус

Одиночне

Зубчастий тетанус

Асинхронне

Тонічне

477 / 6854
Для підвищення результатів спортсмену рекомендовано застосовувати препарат, що містить карнітин. Який процес у найбільшому ступені активується карнітином?

Синтез стероїдних гормонів

Синтез кетонових тіл

Транспорт жирних кислот у мітохондрії

Синтез ліпідів

Тканинне дихання

478 / 6854
Хлопчик 13 років скаржиться на загальну слабість, запаморочення, втомлюваність. Спостерігається відставання у розумовому розвитку. У крові та сечі висока концентрація валіну, ізолейцину, лейцину. Сеча специфічного запаху. Який найбільш імовірний діагноз?

Тирозиноз

Хвороба Аддісона

Гістидинемія

Хвороба кленового сиропу

Базедова хвороба

479 / 6854
У собаки в досліді подразнювали на шиї периферичний відрізок блукаючого нерва, при цьому спостерігали такі зміни серцевої діяльності:

Збільшення атріовентрикулярного проведення

Збільшення збудливості міокарда

Зменшення частоти скорочень

Збільшення частоти та сили скорочень

Збільшення сили скорочень

480 / 6854
Після позалікарняного аборту у жінки прогресував гнійний ендоміометрит зі смертельним наслідком. При розтині померлої виявлені чисельні абсцеси легень, субкапсулярні гнійнички в нирках, гіперплазія селезінки. Яка форма сепсису виникла у хворої?

Легеневий сепсис

Уросепсис

Хроніосепсис

Септицемія

Септикопіємія

481 / 6854
До лікаря-інфекціоніста з хворою дитиною звернулися батьки, які тривалий час працювали в одній азіатській країні. У дитини шкіра землистого кольору, втрата апетиту, в’ялість, збільшені печінка, селезінка, периферичні лімфатичні вузли. Яке протозойне захворювання можна припустити у дитини?

Вісцеральний лейшманіоз

Токсоплазмоз

Балантидіаз

Амебіаз

Лямбліоз

482 / 6854
До травматологічного пункту доставлено хворого з пошкодженням м'язів нижніх кінцівок. За рахунок яких клітин можлива репаративна регенерація м'язових волокон і відновлення функції м'язів?

Міофібробластів

Клітин-сателітів

Фібробластів

Міоепітеліальних клітин

Міобластів

483 / 6854
Хворого госпіталізовано зі скаргами на блювання, запаморочення, двоїння в очах, утруднене ковтання. Лікар запідозрив ботулізм. Які методи діагностики доцільно використати для підтвердження діагнозу?

Протозоологічний, мікроскопічний

Біологічну пробу, бактеріологічний

Алергічну пробу, серологічний

Бактеріологічний, мікологічний

484 / 6854
У хворого на ревматоїдний артрит, який лікувався індометацином, виникли ознаки гастропатії. З якою дією препарату можна пов’язати виникнення цього ускладнення?

Місцевоподразнююча

Антикінінова

Антисеротонінова

Антициклооксигеназна

Антигістамінна

485 / 6854
На розтині виявлено: на зовнішній поверхні аортального клапана великі (1-2 см) буровато-червоні, крихкі нашарування, які прикривають виразкові дефекти. Який найбільш імовірний діагноз?

Зворотній бородавчастий ендокардит

Гострий бородавчастий ендокардит

Фібропластичний ендокардит

Поліпозно-виразковий ендокардит

Дифузний ендокардит

486 / 6854
Під час інтраопераційної біопсії молочної залози виявлено ознаки тканинного атипізму, що виражається у порушенні співвідношення паренхіми і строми з переважанням останньої, різних розмірів і форми залозисті структури, вистелені одношаровим проліферуючим епітелієм. Який найбільш імовірний діагноз?

Мастит

Фіброаденома

Інфільтруючий рак

Папілома

Неінфільтруючий рак

487 / 6854
Хворий скаржиться на слабкість, задишку, набряки нижніх кінцівок. Діагноз – хронічна серцева недостатність. Який засіб необхідно призначити хворому в першу чергу?

Анаприлін

Кофеїн

Дигітоксин

Папаверин

Раунатин

488 / 6854
При нанесенні стоматологом пероксиду водню на слизову оболонку порожнини рота з'явилася інтенсивна піна. Який фермент спричиняє такий ефект?

Метгемоглобінредуктаза

Холінестераза

Ацетилтрансфераза

Каталаза

Глюкозо-6-фосфатдегідрогеназа

489 / 6854
Через декілька годин після опіку в ділянці гіперемії та набряку шкіри у хворого з’явилось вогнище некрозу. Який головний механізм забезпечує посилення руйнівних явищ в осередку запалення?

Еміграція лімфоцитів

Проліферація фібробластів

Вторинна альтерація

Діапедез еритроцитів

Первинна альтерація

490 / 6854
У потерпілого травма ліктьового суглоба з відриванням медіального надвиростка плечової кістки. Який нерв може бути пошкоджено при цій травмі?

Серединний

Променевий

М'язово-шкірний

Медіальний шкірний нерв передпліччя

Ліктьовий

491 / 6854
У хворого на цукровий діабет змінилось значення рH та стало дорівнювати 7,3. Визначення компонентів якої буферної системи використовується для діагностики розладів кислотно-лужної рівноваги?

Оксигемоглобінової

Білкової

Фосфатної

Бікарбонатної

Гемоглобінової

492 / 6854
У хворого М., 45 років, при аналізі ЕКГ встановлено: ритм синусовий, число передсердних комплексів більше числа шлуночкових комплексів; прогресуюче подовження інтервалу P-Q від комплексу до комплексу; випадання окремих шлуночкових комплексів; зубці Р та комплекси QRST без змін. Назвіть тип порушення серцевого ритму.

Атріовентрикулярна блокада II ступеня

Атріовентрикулярна блокада I ступеня

Повна атріовентрикулярна блокада

Синоаурикулярна блокада

Внутрішньопередсердна блокада

493 / 6854
Хвора 27 років закрапала в очі краплі, до складу яких входить пеніцилін. Через декілька хвилин з’явилися свербіння та печіння шкіри, набряк губ та повік, кашель зі свистом, став знижуватися артеріальний тиск. Які імуноглобуліни беруть участь у розвитку цієї алергічної реакції?

IgG і IgD

IgE і IgG

IgM і IgG

IgM і IgD

IgA і IgM

494 / 6854
У клітині, яка мітотично ділиться, спостерігається розходження дочірніх хроматид до полюсів клітини. На якій стадії мітотичного циклу знаходиться клітина?

Метафаза

Інтерфаза

Телофаза

Профаза

Анафаза

495 / 6854
В приймальне відділення доставлено хворого у непритомному стані. Шкіра холодна, зіниці звужені, дихання з утрудненням, відзначається періодичність по типу Чейна-Стокса, артеріальний тиск знижений, сечовий міхур переповнений. Отруєння якою речовиною найбільш вірогідне?

Ненаркотичними аналгетиками

Наркотичними аналгетиками

М-холіноблокаторами

Транквілізаторами

496 / 6854
При дослідженні гнійних виділень з шийки матки бактеріоскопічно виявлено присутність грамнегативних бобоподібних диплококів, які знаходилися як всередині, так і поза лейкоцитами. Назвіть чинника гнійного запалення шийки матки.

Haemophilus vaginalis

Trichomonas vaginalis

Chlamidia trachomatis

Neisseria gonorroeae

Calymmatobacterium granulomatis

497 / 6854
У хворого 60 років унаслідок злоякісної пухлини великого сосочка дванадцятипалої кишки виникла обтураційна жовтяниця. Просвіт якої анатомічної структури стискується пухлиною?

Міхурова протока

Ліва печінкова протока

Загальна печінкова протока

Права печінкова протока

Печінково-підшлункова ампула

498 / 6854
Хворий 60 років госпіталізований до хірургічного відділення в зв’язку з інфекцією, викликаною синьогнійною паличкою, чутливою до антибіотика пеніцилінового ряду. Вкажіть, який з наведених пеніцилінів має виражену активність по відношенню до Pseudomonas aeruginosa?

Карбеніцилін

Бензилпеніцилін

Феноксиметилпеніцилін

Метицилін

Оксацилін

499 / 6854
На електронній мікрофотографії біопсійного матеріалу подано легеню недоношеної дитини. Виявлено злипання стінки альвеол через відсутність сурфактанту. Порушення функції яких клітин стінки альвеоли зумовлює дану картину?

Альвеолоцитів II типу

Альвеолярних макрофагів

Альвеолоцитів I типу

Секреторних клітин

Фібробластів

500 / 6854
В приймально-діагностичне відділення доставлено жінку 38 років з матковою кровотечею. Які найбільш вірогідні зміни зі сторони крові відбудуться?

Зменшення гематокритного числа

Збільшення гематокритного числа

Лейкоцитоз

Еритроцитоз

Лейкопенія

501 / 6854
На базарі громадянин А. продавав ковбасу під назвою “свиняча домашня”. У держсанінспекції виникла підозра фальсифікації ковбаси. За допомогою якої серологічної реакції імунітету можна ідентифікувати харчовий продукт?

Преципітації

РЗК

Імунофлуоресценції

РНГА

Аглютинації

502 / 6854
У хворого виник спазм гладенької мускулатури бронхів. Використання активаторів яких мембранних циторецепторів фізіологічно обгрунтовано для зняття нападу?

β-адренорецепторів

α- та β-адренорецепторів

М-холінорецепторів

Н-холінорецепторів

α-адренорецепторів

503 / 6854
У хворої з клінічними ознаками імунодефіциту кількість та функціональна активність Т- і В-лімфоцитів не змінені. Під час обстеження на молекулярному рівні виявлено дефект, при якому порушена функція антигенпрезентації імунокомпетентним клітинам. Дефект яких клітин є найбільш імовірним?

NK-клітини

Т-лімфоцити, В-лімфоцити

Фібробласти, Т-лімфоцити, В-лімфоцити

Макрофаги, моноцити

0-лімфоцити

504 / 6854
Хвора 25 років звернулася зі скаргами на погіршення зору. При огляді виявлено порушення акомодації, зіниця розширена, не реагує на світло. Функція яких м'язів порушена?

М'яз, що розширює зіницю, війковий

Верхній навскісний, війковий

М'яз, що звужує і м'яз, що розширює зіницю

М'яз, що звужує зіницю, війковий

Латеральний прямий, м'яз, що звужує зіницю

505 / 6854
Внаслідок дії електричного струму на збудливу клітину виникла деполяризація її мембрани. Рух яких іонів через мембрану є причиною деполяризації?

Cl–

К+

НСО3–

Са2+

Na+

506 / 6854
У міокарді шлуночків досліджуваної людини порушені процеси реполяризації. Це призведе до порушення амплітуди, конфігурації, тривалості зубця:

S

P

Q

Т

R

507 / 6854
Хворий 50 років скаржиться на спрагу, п'є багато води, виражена поліурія. Глюкоза крові 4,8 ммоль/л, в сечі глюкози і ацетону немає, сеча безбарвна, питома вага 1,002-1,004. Яка причина поліурії?

Нестача вазопресину

Інсулінова недостатність

Гіпотиреоз

Альдостеронізм

Тиреотоксикоз

508 / 6854
Хвора на бронхіальну астму приймала таблетки, які викликали безсоння, головний біль і підвищення артеріального тиску. Який препарат міг стати причиною таких ускладнень?

Еуфілін

Адреналін

Ефедрин

Ізадрин

Кромолін натрію

509 / 6854
До гастроентерологічного відділення потрапив хворий 57 років з підозрою на синдром Золінгера-Еллісона, про що свідчило різке збільшення рівня гастрину у сироватці крові. Яке порушення секреторної функції шлунка найбільш імовірне?

Ахілія

Гіпосекреція гіпоацидна

Гіпосекреція гіперацидна

Гіперсекреція гіпоацидна

Гіперсекреція гіперацидна

510 / 6854
У хворого 17 років інтраопераційно на нижній поверхні печінки виявлена пухлина розмірами 4,5х5,0х3,5 см із субсерозною локалізацією, темночервоного кольору, на розрізі представлена порожнинами зі значним вмістом крові. Поставте попередній діагноз.

Капілярна гемангіома

Гемангіоендотеліома

Гемангіоперицитома

Кавернозна гемангіома

Лімфангіома

511 / 6854
У дитини виявлено галактоземію. Концентрація глюкози в крові суттєво не змінена. Дефіцитом якого ферменту зумовлене це захворювання?

Галактокіназа

Галактозо-1-фосфат-уридилтрансфераза

Аміло-1,6-глюкозидаза

Гексокіназа

Фосфоглюкомутаза

512 / 6854
Під час гістологічного дослідження шлунка виявлено, що у залозах міститься дуже мало парієтальних клітин або вони повністю відсутні. Слизову оболонку якої ділянки шлунка вивчали?

Тіло шлунка

Кардіальний відділ

Пілоричний відділ

Дно шлунка

513 / 6854
У результаті черепно-мозкової травми у хворого були виявлені такі симптоми: інтенційний тремор, дисметрія, адіадохокінез, дизартрія. Яка структура головного мозку ушкоджена?

Мозочок

Стріатум

Блідий шар

Рухова кора

Чорна речовина

514 / 6854
До травмпункту звернувся чоловік 45 років після побутової травми плеча. Об'єктивно: відсутні функції розгинання, приведення та пронації плеча. Пошкодження якого м’яза викликало такий стан?

Підлопатковий м’яз

Малий круглий м’яз

Надосний м’яз

Великий круглий м’яз

Підосний м’яз

515 / 6854
У крові хворого збільшена концентрація пірувату, значна кількість його екскретується з сечею. Який авітаміноз спостерігається у хворого?

Авітаміноз В6

Авітаміноз Е

Авітаміноз В1

Авітаміноз В2

Авітаміноз В3

516 / 6854
Внаслідок впливу гама-випромінювання ділянка ланцюга ДНК повернулась на 180 градусів. Який з наведених видів мутацій відбувся в ланцюгу ДНК?

Реплікація

Транслокація

Делеція

Інверсія

Дуплікація

517 / 6854
Куди треба провести катетер для забору лімфи з грудної лімфатичної протоки?

У правий венозний кут

У нижню порожнисту вену

У ліву пахову вену

У верхню порожнисту вену

У лівий венозний кут

518 / 6854
У хворого із запаленням легень непереносимість антибіотиків. Який з комбінованих сульфаніламідних препаратів слід призначити хворому?

Стрептоцид

Сульфацил натрію

Бісептол

Сульфадиметоксин

Етазол

519 / 6854
У хворого видалено дванадцятипалу кишку. Це призвело до зменшення секреції, перш за все, таких гастроінтестинальних гормонів:

Гістамін

Холецистокінін та секретин

Гастрин

Гастрин та гістамін

Нейротензин

520 / 6854
У померлого 58 років на розтині мітральний клапан деформований, потовщений, змикається не до кінця. Мікроскопічно: вогнища колагенових волокнинок еозинофільні, дають плюсову реакцію на фібрин. Найвірогідніше це:

Амілоїдоз

Гіаліноз

Фібриноїдне набухання

Фібринозне запалення

Мукоїдне набухання

521 / 6854
В організм людини введено живу вакцину. На підвищення активності яких клітин сполучної тканини можна очікувати?

Адипоцити і адвентиційні клітини

Фібробласти і лаброцити

Плазмоцити і лімфоцити

Макрофаги і фібробласти

Пігментоцити і перицити

522 / 6854
Тварині, сенсибілізованій туберкуліном, внутрішньоочеревинно введений туберкулін. Через 24 години при лапаротомії виявлено венозну гіперемію та набряк очеревини. У мазках-відбитках з очеревини велика кількість лімфоцитів та моноцитів. Який патологічний процес у тварини?

Алергійне запалення

Серозне запалення

Асептичне запалення

Фібринозне запалення

Гнійне запалення

523 / 6854
У чоловіка 42 років, який страждає на подагру, в крові підвищена концентрація сечової кислоти. Для зниження рівня сечової кислоти йому призначено алопуринол. Конкурентним інгібітором якого ферменту є алопуринол?

Аденозиндезамінази

Гіпоксантинфосфорибозилтрансферази

Аденінфосфорибозилтрансферази

Ксантиноксидази

Гуаніндезамінази

524 / 6854
Під час мікроскопії волосини хворого, взятої з уражених ділянок, виявлені обривки міцелію гриба, спори, пухирці повітря і крапельки жиру. Для якого грибкового захворювання характерна така мікроскопічна картина волосини?

Споротрихоз

Епідермофітія

Мікроспорія

Трихофітія

Фавус

525 / 6854
У людини зменшений діурез, гіпернатріємія, гіпокаліємія. Гіперсекреція якого гормону може бути причиною таких змін?

Адреналін

Паратгормон

Альдостерон

Вазопресин

Передсердний натрійуретичний фактор

526 / 6854
Послаблення кровопостачання органа зумовлює розвиток гіпоксії, яка активізує функцію фібробластів. Об’єм яких елементів нарощується в цій ситуації?

Судин мікроциркуляторного русла

Лімфатичних судин

Міжклітинної речовини

Паренхіматозних елементів органа

Нервових елементів

527 / 6854
Хворий скаржиться на часте та утруднене сечовиділення. Порушення структури якого з наведених утворень є причиною цього?

Бульбоуретральні залози

Яєчка

Сім’яні міхурці

Простата

Придатки яєчка

528 / 6854
У результаті виснажливої м'язової роботи у робіткика значно зменшилась буферна ємність крові. Надходженням якої кислої речовини до крові можна пояснити це явище?

Лактату

α-кетоглутарату

Пірувату

1,3-бісфосфогліцерату

3-фосфогліцерату

529 / 6854
У групі дітей, які їли солодкий соковитий кавун, у двох з'явились ознаки отруєння: різка слабість, запаморочення, головний біль, блювання, задишка, тахікардія, синюшність губ, вух, кінчиків пальців. Лабораторний аналіз кавуна показав високий вміст нітратів. Який провідний механізм у патогенезі отруєння тільки у двох дітей?

Недостатність глутатіон-пероксидази

Недостатність супероксиддисмутази

Блокада цитохромоксидази

Недостатність каталази

Недостатність мет-Hb-редуктази

530 / 6854
У спортсмена на старті перед змаганнями відзначається підвищення артеріального тиску та частоти серцевих скорочень. Впливом яких відділів ЦНС можна пояснити вказані зміни?

Кори великих півкуль

Проміжного мозку

Середнього мозку

Довгастого мозку

Гіпоталамуса

531 / 6854
Під час запалення відзначається підвищення проникливості судин мікроциркуляторного русла, збільшення в них гідродинамічного тиску крові. У міжклітинній рідині має місце підвищення осмотичної концентрації і дисперсності білкових структур. Який вид набряку буде спостерігатися в цьому

Лімфогенний

Змішаний

Мембраногенний

Гідродинамічний

Колоїдно-осмотичний

532 / 6854
На розтині виявлено, що нирки збільшені в розмірах, поверхня крупногорбиста за рахунок наявності численних порожнин з гладенькою стінкою, заповнених прозорою рідиною. Яке захворювання нирок мало місце у хворого?

Полікістоз

Пієлонефрит

Інфаркт

Некротичний нефроз

Гломерулонефрит

533 / 6854
Для лікування злоякісних пухлин призначають метотрексат - структурний аналог фолієвої кислоти, який є конкурентним інгібітором дигідрофолатредуктази. На якому рівні метотрексат пригнічує синтез нуклеїнових кислот?

Реплікація

Репарація

Процесинг

Транскрипція

Синтез мононуклеотидів

534 / 6854
У результаті пошкодження одного з реакторів АЕС відбулося витікання радіоактивних продуктів. Люди, які знаходилися в зоні підвищеної радіації, орієнтовно отримали по 250-300 Р. Їх негайно госпіталізовано. Які зміни складу крові будуть характерними для потерпілих?

Тромбоцитопенія

Нейтропенія

Лімфопенія

Анемія

Лейкопенія

535 / 6854
Тестостерон і його аналоги збільшують масу скелетний м'язів, що дозволяє використовувати їх для лікування дистрофій. Взаємодією гормону з яким клітинним субстратом зумовлена ця дія?

Хроматин

Рибосоми

Білки-активатори транскрипції

Ядерні рецептори

Мембранні рецептори

536 / 6854
До інфекційної лікарні надійшов пацієнт з діареєю. Під час бактеріоскопічного дослідження фекальних мас виявили грамнегативні зігнуті палички. Яке захворювання можна припустити у хворого?

Черевний тиф

Кишкова форма чуми

Холера

Сальмонельозний гастроентерит

Дифтерія

537 / 6854
При дослідженні каріотипу 5-річної дівчинки виявлено 46 хромосом. Одна з хромосом 15-ї пари довша від звичайної, тому що до неї приєдналась хромосома з 21-ї пари. Який вид мутації має місце у цієї дівчинки?

Делеція

Інверсія

Дуплікація

Транслокація

Нестача

538 / 6854
Дитина 7 років не може відвести плече і підняти його до горизонтального рівня, до обличчя руку приводить лише тильною стороною при деякій абдукції плеча (за рахунок надостного м'яза) - рука “сурмача”. Активна функція якого м'яза

Підостного

Дельтоподібного

Малого круглого

Великого грудного

Великого круглого

539 / 6854
Трансмуральний інфаркт міокарда у хворого ускладнився розвитком гострої лівошлуночкової недостатності. Що є найбільш типовим для цього стану?

Ціаноз

Набряк кінцівок

Набряк легенів

Артеріальна гіпертензія

540 / 6854
У крові хворих на цукровий діабет спостерігається підвищення вмісту вільних жирних кислот. Причиною цього може бути:

Активація утилізації кетонових тіл

Накопичення в цитозолі пальмітоїл-КоА

Зниження активності фосфатидилхолін-холестерин-ацилтрансферази плазми

Підвищення активності тригліцеридліпази адипоцитів

Активація синтезу аполіпопротеїнів А-1, А-2, А-4

541 / 6854
Для постановки туберкулінової проби дитині внутрішньошкірно введено туберкулін. Через 24 години в місці введення відзначена виражена гіперемія, ущільнення тканин. Який механізм лежить в основі розвитку даних змін?

Клітинна цитотоксичність

Імунокомплексна цитотоксичність

Утворення гранульом

Цитотоксичність реагінового типу

Антитільна цитотоксичність

542 / 6854
У жінки 63 років є ознаки ревматоїдного артриту. Підвищення рівня якого з перерахованих нижче показників крові буде найбільш значущим для підтвердження діагнозу?

Ліпопротеїдів

Загального холестерину

Сумарних глікозаміногліканів

Кислої фосфатази

R-глікозидази

543 / 6854
Чоловік 59 років має ознаки паренхіматозної жовтяниці та портальної гіпертензії. Під час гістологічного дослідження пункційного біоптату печінки знайдено: балково-часточкова будова порушена, частина гепатоцитів має ознаки жирової дистрофії, утворюються порто-портальні сполучнотканинні септи з формуванням псевдочасточок, з наявністю перипортальних лімфомакрофагальних інфільтратів. Який найбільш імовірний діагноз?

Хронічний гепатоз

Токсична дистрофія

Цироз печінки

Алкогольний гепатит

Вірусний гепатит

544 / 6854
Жінка 49 років тривалий час хворіла на хронічний гломерулонефрит, який призвів до смерті. На розтині встановлено, що нирки мають розміри 7х3х2.5 см, масу 65,0 г, щільні, дрібнозернисті. Мікроскопічно: фібринозне запалення серозних і слизових оболонок, дистрофічні зміни паренхіматозних органів, набряк головного мозку. Яке ускладнення призвело до вказаних змін серозних оболонок і

ДВЗ-снндром

Уремія

Сепсис

Тромбоцитопенія

Анемія

545 / 6854
У внутрішньоутробному періоді розвитку в судинній системі плода функціонує крупна артеріальна (боталова) протока, яка після народження перетворюється в lig. arteriosum. Які анатомічні утворення з’єднує між собою ця протока?

Легеневий стовбур та аорта

Аорта та верхня порожниста вена

Аорта та нижня порожниста вена

Легеневий стовбур та верхня порожниста вена

Праве та ліве передсердя

546 / 6854
Внаслідок активації іонних каналів зовнішньої мембрани збудливої клітини значно збільшився її потенціал спокою. Які канали були активовані?

Повільні кальцієві

Натрієві та кальцієві

Калієві

Швидкі кальцієві

Натрієві

547 / 6854
Біля інфікованої рани збільшилися регіонарні лімфовузли. При гістологічному дослідженні в них виявлено збільшення кількості макрофагів, лімфоцитів і лімфатичних фолікулів в кірковому шарі, а також велику кількість плазматичних клітин. Який процес в лімфатичних вузлах відображають виявлені гістологічні зміни?

Набуту недостатність лімфоїдної тканини

Антигенну стимуляцію

Реакцію гіперчутливості

Пухлинну трансформацію

Природжену недостатність лімфоїдної тканини

548 / 6854
Хворому 56 років із скаргами на спрагу та часте сечовиділення ендокринологом було встановлено діагноз цукрового діабету та призначено бутамід. Яким є механізм дії цього препарату?

Пригнічує всмоктування глюкози в кишечнику

Полегшує транспорт глюкози через клітинні мембрани

Пригнічує альфа-клітини острівців Лангерганса

Стимулює бета-клітини острівців Лангерганса

Сприяє засвоєнню глюкози клітинами тканин організму

549 / 6854
Хвора звернулася до травмпункту з приводу нагноєння різаної рани. Лікар для очищення рани від гнійних виділень промив її 3% розчином перекису водню. При цьому піна не утворилася. З чим пов'язана відсутність дії препарату?

Наявність у рані гнійного вмісту

Неглибока рана

Низька концентрація H2O2

Спадкова недостатність фосфатдегідрогенази еритроцитів

Спадкова недостатність каталази

550 / 6854
Хворий 62 років надійшов до неврологічного відділення з приводу мозкового крововиливу. Стан тяжкий. Спостерігається наростання глибини і частоти дихання, а потім його зменшення до апное, після чого цикл дихальних рухів відновлюється. Який тип дихання виник у хворого?

Чейна–Стокса

Біота

Апнеїстичне

Кусмауля

Гаспінг–дихання

551 / 6854
Хворому з гіперсекрецією шлункового соку лікар рекомендував виключити з дієти насичені бульйони і овочеві відвари, тому що вони стимулюють шлункову секрецію. Який переважний механізм стимуляції шлункової секреції у цьому випадку?

Стимуляція вироблення секретину в 12-палій кишці

Подразнення механорецепторів ротової порожнини

Стимуляція вироблення гастрину G-клітинами

Подразнення смакових рецепторів

Подразнення механорецепторів шлунка

552 / 6854
При дослідженні крові хворого виявлено значне збільшення активності МВ-форм КФК (креатинфосфокінази) та ЛДГ-1. Яка найбільш імовірна патологія?

Панкреатит

Ревматизм

Інфаркт міокарда

Гепатит

Холецистит

553 / 6854
До інфекційного відділення лікарні госпіталізовано хворого з діагнозом бактеріальної дизентерії. Лабораторними дослідженнями встановлено, що збудник чутливий до багатьох протимікробних засобів, однак у хворого виявлені явища анемії. Який препарат протипоказаний хворому?

Ентеросептол

Фуразолідон

Фталазол

Ампіцилін

Левоміцетин

554 / 6854
На мікропрепараті червоного кісткового мозку виявляються численні капіляри, через стінку яких у кровоносне русло виходять зрілі формені елементи крові. До якого типу належать ці капіляри ?

Соматичні

Лімфатичні

Синусоїдні

Вісцеральні

Фенестровані

555 / 6854
В експерименті на тварині видалення ділянки кори півкуль мозку усунуло раніше вироблені умовні рефлекси на світлове подразнення. Яку ділянку кори було видалено?

Скронева доля

Потилична кора

Постцентральна звивина

Лімбічна кора

Прецентральна звивина

556 / 6854
В експерименті на спинному мозку при збудженні альфа-мотонейронів м'язів-згиначів відзначено гальмування альфа-мотонейронів м'язів-розгиначів. Який вид гальмування лежить в основі цього явища?

Деполяризаційне

Пресинаптичне

Зворотне

Латеральне

Реципрокне

557 / 6854
Під час емоційного збудження частота серцевих скорочень у людини 30 років досягла 112 на хв. Який відділ провідної системи серця є відповідальним за цю зміну?

Пучок Гіса

Волокна Пуркіньє

Атріовентрикулярний вузол

Синоатріальний вузол

Ніжки пучка Гіса

558 / 6854
При розтині померлого від поширеного перитоніту в дистальних відділах тонкої кишки виявлено численні виразки овальної форми, які розташовані вздовж кишки. Дно виразок чисте, гладеньке, утворене м'язовою або серозною оболонкою, краї виразок рівні, закруглені. У двох виразках є перфоративні отвори діаметром до 0,5 см. Яке захворювання треба

Черевний тиф

Туберкульоз

Дизентерія

Холера

Висипний тиф

559 / 6854
Малюк попросив Вас надути повітряну кульку якомога дужче за один видих. Яким об'ємом повітря Ви скористаєтесь?

Резервним об'ємом вдиху

Ємністю вдиху

Загальною ємністю легень

Функціональною залишковою ємністю

Життєвою ємністю легень

560 / 6854
Під час вивчення родоводу сім'ї, в якій спостерігається гіпертрихоз (надмірне оволосіння вушних раковин), виявлено, що ця ознака зустрічається в усіх поколіннях тільки у чоловіків і успадковується від батька до сина. Визначте тип успадкування гіпертрихозу:

Аутосомно-рецесивний

Зчеплений з Х-хромосомою домінантний

Зчеплений з Y-хромосомою

Аутосомно-домінантний

Зчеплений з Х-хромосомою рецесивний

561 / 6854
Приймання оральних контрацептивів, які містять статеві гормони, пригнічує секрецію гормонів гіпофіза. Секреція якого з наведених гормонів пригнічується при прийманні оральних контрацептивів, які містять статеві гормони?

Фолікулостимулюючий

Окситоцин

Тиреотропний

Соматотропний

Вазопресин

562 / 6854
Внаслідок росту пухлини у порожнину III шлуночка головного мозку у пацієнта розвиваються вегетативні розлади у вигляді порушення сну, терморегуляції, усіх видів обміну, нецукрового діабету. Подразнення ядер якої ділянки головного мозку викликає ці симптоми?

Гіпоталамус

Міст

Довгастий мозок

Покришка середнього мозку

Ніжки мозку

563 / 6854
У хворого виявлена болючість по ходу великих нервових стволів та підвищений вміст пірувату в крові. Нестача якого вітаміну може викликати такі зміни?

В1

В2

РР

Біотин

Пантотенова кислота

564 / 6854
Територію старого худобомогильника, який не використовувався більше 50 років, планується відвести під житлове будівництво. Однак дослідження грунту виявило наявність життєздатних спор збудника особливо небезпечного захворювання. Який із вказаних мікроорганізмів найбільш імовірно міг зберігатися у грунті протягом такого тривалого

Brucella abortus

Yersinia pestis

Mycobacterium bovis

Francisella tularensis

Bacillus anthracis

565 / 6854
Дитина 5 років надійшла до ЛОР-відділення з діагнозом гнійне запалення середнього вуха. Захворювання розпочалося із запалення носоглотки. Через який канал скроневої кістки інфекція потрапила в барабанну порожнину?

Сонно-барабанні канальці

М’язовотрубний канал

Сонний канал

Каналець барабанної струни

Барабанний каналець

566 / 6854
Хворий на цукровий діабет вчасно не отримав ін’єкцію інсуліну, що призвело до розвитку гіперглікемічної коми (вміст глюкози в крові – 50 ммоль/л). Який механізм є головним у розвитку цієї коми?

Гіпокаліємія

Гіпонатріємія

Ацидоз

Гіпоксія

Гіперосмія

567 / 6854
У тяжко травмованої людини поступово настала біологічна смерть. Свідченням цього є:

Невпорядкованість хімічних процесів

Непритомність

Відсутність серцебиття і дихання

Відсутність рухливості

У клітинах відбувається автоліз і розкладання

568 / 6854
У жінки 68 років після інсульту відсутні рухи в верхній та нижній правій кінцівках. Тонус м’язів цих кінцівок і рефлекси в них підвищені. Є патологічні рефлекси. Яка це форма паралічу?

Параплегія

Геміплегія

Тетраплегія

Дисоціація

Моноплегія

569 / 6854
У хворого з верхнім типом ожиріння клінічно тривало відзначалися артеріальна гіпертонія, гіперглікемія, глюкозурія. Смерть настала від крововиливу в головний мозок. Під час патоморфологічного дослідження виявлені базофільна аденома гіпофіза, гіперплазія кори наднирників. Який найбільш імовірний діагноз?

Цукровий діабет

Гіпофізарний нанізм

Адипозогенітальна дистрофія

Акромегалія

Хвороба Іценко-Кушінга

570 / 6854
У хворого після важкої травми розвинувся шок та з’явилися ознаки гострої ниркової недостатності [ГНН]. Що є провідним механізмом розвитку ГНН в даному випадку?

Порушення відтоку сечі

Підвищення тиску в ниркових артеріях

Підвищення тиску в капсулі нефрону

Зниження онкотичного тиску крові

Падіння артеріального тиску

571 / 6854
У людини внаслідок тривалого голодування швидкість клубочкової фільтрації зросла на 20%. Найбільш імовірною причиною змін фільтрації в зазначених умовах є:

Збільшення системного артеріального тиску

Збільшення коефіцієнту фільтрації

Збільшення проникності ниркового фільтру

Зменшення онкотичного тиску плазми крові

Збільшення ниркового плазмотоку

572 / 6854
При розтині померлої в комі молодої людини виявлено поширений тромбоемболічний інфаркт лівої півкулі мозку, велика септична селезінка, імунокомплексний гломерулонефрит, виразки в стулках аортального клапана, прикриті поліпоподібними тромбами з колоніями стафілококів. Яке захворювання викликало церебральну тромбоемболію?

Ревматичний тромбоендокардит

Септичний бактеріальний ендокардит

Гострий ревматичний вальвуліт

Септицемія

Септикопіємія

573 / 6854
Жінці з дисфункціональною матковою кровотечею зробили діагностичне вишкрябання. Гістологічно у зіскрібку виявлено велику кількість звивистих залоз, просвіти окремих залоз кістозно розширені. Назвіть різновид загальнопатологічного процесу в ендометрії:

Атрофія

Дисплазія

Метаплазія

Гіпертрофічні розростання

Залозисто-кістозна гіперплазія

574 / 6854
У людини частота серцевих скорочень утримується на рівні, що не перевищує 40 разів за хвилину. Що є водієм ритму серця у цієї людини?

Волокна Пуркіньє

Атріовентрикулярний вузол

Пучок Гіса

Ніжки пучка Гіса

Синоатріальний вузол

575 / 6854
У хворого виявлено порушення прохідності дихальних шляхів на рівні дрібних і середніх бронхів. Які порушення кислотно-лужної рівноваги можна виявити у крові в даному

Респіраторний ацидоз

Метаболічний алкалоз

Метаболічний ацидоз

Респіраторний алкалоз

576 / 6854
У хворих з непрохідністю жовчовивідних шляхів пригнічується зсідання крові, виникають кровотечі, що є наслідком недостатнього засвоєння вітаміну:

К

Каротину

Е

D

А

577 / 6854
На розтині тіла померлого виявлено, що вся права легеня збільшена у розмірі, щільна, на плеврі нашарування фібрину, на розрізі тканина легені сірого кольору, з неї стікає каламутна рідина. Для якого захворювання легень характерна така картина?

Фіброзуючий альвеоліт

Крупозна пневмонія

Інтерстиціальна пневмонія

Вогнищева пневмонія

Гангрена легені

578 / 6854
У людини вимірюють внутрішньоплевральний тиск. У якій фазі людина затримала дихання, якщо величина тиску дорівнює – 25 см вод.ст.?

Форсований видих

Спокійний видих

Спокійний вдих

Форсований вдих

579 / 6854
У хворого 27 років виявлено патологічні зміни печінки і головного мозку. У плазмі крові виявлено різке зниження, а в сечі підвищення вмісту міді. Поставлено діагноз - хвороба Вільсона. Активність якого ферменту в сироватці крові необхідно дослідити для підтвердження діагнозу?

Карбоангідрази

Ксантиноксидази

Алкогольдегідрогенази

Церулоплазміну

Лейцинамінопептидази

580 / 6854
До лікаря звернувся хворий зі скаргами на постійну спрагу. Виявлена гіперглікемія, поліурія та підвищений вміст 17-кетостероїдів у сечі. Яке захворювання найбільш імовірне?

Глікогеноз I типу

Стероїдний діабет

Аддісонова хвороба

Мікседема

Інсулінозалежний діабет

581 / 6854
Під час обіду дитина похлинулася і аспірувала їжу. Почався сильний кашель, шкіра і слизові ціанотичні, пульс прискорений, дихання рідке, видих подовжений. Яке порушення зовнішнього дихання розвинулось у дитини?

Стадія інспіраторної задишки при асфіксії

Стенотичне дихання

Дихання Біота

Альтернуюче дихання

Стадія експіраторної задишки при асфіксії

582 / 6854
У лабораторії при мікроскопії харкотиння хворого на пневмонію випадково виявлені личинки. При аналізі крові виявлена еозинофілія. Який гельмінтоз можна передбачити?

Трихоцефальоз

Опісторхоз

Аскаридоз

Парагонімоз

Ентеробіоз

583 / 6854
У хворого з тимомою (пухлиною вилочкової залози) спостерігається ціаноз, розширення підшкірної венозної сітки і набряк м’яких тканин обличчя, шиї, верхньої половини тулуба і верхніх кінцівок. Який венозний стовбур перетиснено

Підключична вена

Передня яремна вена

Внутрішня яремна вена

Верхня порожниста вена

Зовнішня яремна вена

584 / 6854
Під час дослідження первинної структури молекули глобіну виявлено заміну глутамінової кислоти на валін. Для якої спадкової патології це характерно?

Гемоглобіноз

Хвороба Мінковського-Шоффара

Серпоподібноклітинна анемія

Фавізм

Таласемія

585 / 6854
При травмі у пацієнта 44 років виник розрив сухожилків м’язів лівої долоні, поверхневих кровоносних судин. Після оперативного втручання і видалення більшої частини некротично зміненої м’язової тканини кровотік був відновлений. За рахунок яких судин?

A. perforantes

Arcus palmaris superficialis

Arcus palmaris profundus

A. digitales palmares communes

A. metacarpeae palmares

586 / 6854
У хворої, яка страждає на тромбофлебіт глибоких вен гомілки, раптово настала смерть. На розтині трупа у загальному стовбурі і біфуркації легеневої артерії знайдено червоні пухкі маси з тьмяною гофрованою поверхнею, що лежать вільно. Який патологічний процес виявив патологоанатом?

Тканинна емболія

Тромбоемболія

Жирова емболія

Тромбоз

Емболія стороннім тілом

587 / 6854
При оформленні дитини до школи для вирішення питання про необхідність ревакцинації поставлено пробу Манту, яка виявилася негативною. Про що свідчить такий результат

Про наявність антитіл до туберкульозних бактерій

Про відсутність клітинного імунітету до туберкульозу

Про наявність клітинного імунітету до туберкульозу

Про відсутність антитоксичного імунітету до туберкульозу

Про відсутність антитіл до туберкульозних бактерій

588 / 6854
Під час бактеріологічного дослідження промивних вод хворого на харчове отруєння висіяли чисту культуру бактерій з такими властивостями: грамнегативна рухлива паличка, на середовищі Ендо росте у вигляді безбарвних колоній. Представником якого роду було зумовлене захворювання?

Shigella

Escherichia

Citrobacter

Salmonella

Yersinia

589 / 6854
До клініки потрапив чоловік 54 років зі скаргами на болі в правій підреберній ділянці, блювоту з кров'ю. Об'єктивно: збільшення розмірів печінки, варикозне розширення вен стравоходу і шлунка, кровотеча з них. Порушення функції якої судини, ймовірніше за все, мало місце?

Aorta abdominalis

Vena porta

Vena cava inferior

Vena hepatica

Vena cava superior

590 / 6854
Під час обстеження донора, який тривалий час не здавав кров, за допомогою методу ІФА виявлені анти-НBs антитіла. Про що свідчить у цьому випадку позитивний результат ІФА?

Про гострий гепатит С

Про хронічний гепатит С

Про перенесений гепатит В

Про хронічний гепатит В

Про гострий гепатит В

591 / 6854
Дитина вдихнула ґудзик. Куди найбільш імовірно він потрапить?

У правий головний бронх

У гортань

У лівий головний бронх

У стравохід

У трахею

592 / 6854
У молодої людини в м'яких тканинах лівого стегна з'явилось безболісне новоутворення без чітких меж. В біоптаті тканини новоутворення нагадують риб'яче м'ясо, складаються з незрілих фібробластоподібних клітин з численними мітозами, які проростають в м'язи. Який найбільш вірогідний діагноз?

Міосаркома

Фіброма

Рак

Фібросаркома

Міома

593 / 6854
У дитини 2 років виникли судоми внаслідок зниження концентрації іонів кальцію в плазмі крові. Це зумовлено зниженням функції:

Кори наднирників

Прищитоподібних залоз

Тимуса

Шишкоподібної залози

Гіпофіза

594 / 6854
У процесі метаболізму в організмі людини утворюються активні форми кисню, у тому числі супероксидний аніон-радикал. За допомогою якого ферменту інактивується цей аніон?

Супероксиддисмутаза

Пероксидаза

Каталаза

Глутатіонпероксидаза

Глутатіонредуктаза

595 / 6854
У загальному вигляді генетичний апарат еукаріот є таким: екзон – інтрон – екзон. Така структурно-функціональна організація гена зумовлює особливості транскрипції. Якою буде про-іРНК відповідно до згаданої схеми?

Екзон-екзон

Екзон-екзон-інтрон

Екзон-інтрон-екзон

Інтрон-екзон

Екзон-інтрон

596 / 6854
При отруєнні аманітином (отрутою блідої поганки) блокується РНКполімераза В(ІІ). При цьому припиняється:

Зворотна транскрипція

Синтез мРНК

Синтез тРНК

Дозрівання мРНК

Синтез праймерів

597 / 6854
Чоловіку 70 років, який страждає на хронічний бронхіт, призначений протикашльовий препарат - кодеїн. Який механізм забезпечує протикашльовий ефект?

Місцева дія

Рефлекторний

Центральний

Конкурентний

Периферична дія

598 / 6854
Хворий 46 років скаржиться на утруднення носового дихання. У біоптаті потовщеної слизової носа знайдені клітини Мікуліча, скупчення епітеліоїдних клітин, плазмоцити, лімфоцити, гіалінові кулі. Який найбільш імовірний діагноз?

Аденовірусний риніт

Менінгококовий назофарингіт

Риновірусна інфекція

Склерома

Алергічний риніт

599 / 6854
З випорожнень хворої дитини 6-місячного віку, яка знаходилась на штучному вигодовуванні, виділена культура кишкової палички з антигенною структурою 0-111. Який діагноз можна апідозрити?

Холероподібне захворювання

Колі-ентерит

Гастроентерит

Дизентерієподібне захворювання

Харчове отруєння

600 / 6854
На препараті яєчника, забарвленому гематоксиліном-еозіном, визначається фолікул, в якому клітини фолікулярного епітелію розміщені в 1-2 шари і мають кубічну форму, навколо овоцита видно оболонку яскраво-червоного кольору. Назвіть цей фолікул:

Зрілий

Атретичний

Первинний

Вторинний

Примордіальний

601 / 6854
Пацієнт під час роботи швидко стомлюється. У положенні стоячи з заплющеними очима він похитується, втрачає рівновагу. Тонус скелетних м'язів знижений. Яка з наведених структур мозку уражена у цієї людини?

Таламус

Базальні ганглії

Прецентральна звивина кори великих півкуль

Мозочок

Гіпоталамус

602 / 6854
На розтині тіла чоловіка 35 років у другому сегменті правої легені виявлено вогнище ущільнення діаметром 5 см, оточене тонкою капсулою. Вогнище представлене сухою крихкою тканиною з тьмяною поверхнею. Для якого захворювання характерні такі морфологічні зміни?

Хондрома

Післязапальний пневмосклероз

Рак легені

Туберкульома

Туморозна форма силікозу

603 / 6854
Під час розтину тіла померлого 58 років знайдено, що мітральний клапан деформований, потовщений, змикається не до кінця. Мікроскопічно: вогнища колагенових волокнинок еозинофільні, дають позитивну реакцію на фібрин. Найвірогідніше це:

Фібриноїдне набухання

Амілоїдоз

Мукоїдне набухання

Фібринозне запалення

Гіаліноз

604 / 6854
У чоловіка 42 років, який страждає на подагру, в крові підвищена концентрація сечової кислоти. Для зниження рівня сечової кислоти йому призначено алопуринол. Конкурентним інгібітором якого ферменту є алопуринол?

Гіпоксантинфосфорибозилтрансфераза

Гуаніндезаміназа

Аденозиндезаміназа

Ксантиноксидаза

Аденінфосфорибозилтрансфераза

605 / 6854
У юнака енерговитрати збільшились з 500 до 2000 кДж за годину. Що з наведеного може бути причиною цього?

Фізичне навантаження

Прийом їжі

Розумова праця

Перехід від сну до бадьорості

Підвищення зовнішньої температури

606 / 6854
Під час розтину тіла померлої в комі молодої людини виявлено поширений тромбоемболічний інфаркт лівої півкулі мозку, велика септична селезінка, імунокомплексний гломерулонефрит, виразки в стулках аортального клапана, прикриті поліпоподібними тромбами з колоніями стафілококів. Яке захворювання викликало церебральну тромбоемболію?

Ревматичний тромбоендокардит

Гострий ревматичний вальвуліт

Септицемія

Септикопіємія

Септичний бактеріальний ендокардит

607 / 6854
У чоловіка 60 років після крововиливу в головний мозок настав тривалий сон. Пошкодження якої структури найімовірніше призвело до цього стану?

Чорна субстанція

Гіпокамп

Ретикулярна формація

Ядра черепних нервів

Кора великих півкуль

608 / 6854
Тестостерон і його аналоги збільшують масу скелетних м'язів, що дозволяє використовувати їх для лікування дистрофій. Взаємодією з яким клітинним субстратом зумовлена ця дія?

Рибосоми

Ядерні рецептори

Білки-активатори транскрипції

Мембранні рецептори

Хроматин

609 / 6854
При жировій інфільтрації печінки порушується синтез фосфоліпідів. Яка з перелічених речовин може посилювати процеси метилювання в синтезі фосфоліпідів?

Метіонін

Цитрат

Глюкоза

Аскорбінова кислота

Гліцерин

610 / 6854
Під час обстеження людини встановлено, що хвилинний об'єм крові дорівнює 3500 мл, систолічний об'єм - 50 мл. Якою є частота серцевих скорочень у цієї людини?

50 скорочень за хвилину

60 скорочень за хвилину

70 скорочень за хвилину

90 скорочень за хвилину

80 скорочень за хвилину

611 / 6854
Чоловік 40 років пробіг 10 км за 60 хвилин. Які зміни енергетичного обміну відбудуться у його м'язах?

Посилиться протеоліз

Посилиться глікогеноліз

Збільшиться швидкість окислення жирних кислот

Посилиться гліколіз

Посилиться глюконеогенез

612 / 6854
У результаті виснажливої м'язової роботи у робітника значно зменшилась буферна ємність крові. Надходженням якої кислої речовини до крові можна пояснити це явище?

Піруват

α-кетоглутарат

1,3-бісфосфогліцерат

3-фосфогліцерат

Лактат

613 / 6854
До фібрилярних елементів сполучної тканини належать колаген, еластин та ретикулін. Вкажіть амінокислоту, яка входить тільки до складу колагену, і визначення якої в біологічних рідинах використовується для діагностики захворювань сполучної тканини?

Фенілаланін

Гідроксипролін

Гліцин

Лізин

Пролін

614 / 6854
У тяжко травмованої людини поступово настала біологічна смерть. Свідченням цього є:

Непритомність

У клітинах відбувається автоліз і розкладання

Відсутність рухливості

Невпорядкованість хімічних процесів

Відсутність серцебиття і дихання

615 / 6854
Хворому на тромбофлебіт призначено комплексну терапію, яка діє на різні етапи тромбоутворення. Яке з перелічених засобів сприяє відновленню прохідності судини?

Неодикумарин

Гепарин

Дипіридамол

Ацетилсаліцилова кислота

Фібринолізин

616 / 6854
Жінка 42 років із невралгією трійчастого нерва скаржиться на періодичне почервоніння правої половини обличчя та шиї, відчуття припливу тепла та підвищення шкірної чутливості. Якою у цьому випадку є артеріальна гіперемія за патофізіологічним механізмом?

Метаболічна

Нейропаралітична

Нейротонічна

Робоча

Реактивна

617 / 6854
У мікропрепараті, виготовленому з пунктату регіонарного лімфовузла хворого та зафарбованому за Романовським-Гімзою, лікар виявив тонкі мікроорганізми з 12-14 рівномірними завитками, з гострими кінцями, довжиною 10-13 мкм блідо-рожевого кольору. Про збудника якої інфекційної хвороби може йти мова у даному випад- ку?

Лептоспіроз

Поворотний тиф

Лейшманіоз

Сифіліс

Трипаносомоз

618 / 6854
Група чоловіків звернулася до лікаря зі скаргами на підвищення температури, головні болі, набряки повік та обличчя, болі в м'язах. З анамнезу стало відомо, що всі вони мисливці і часто вживають в їжу м'ясо диких тварин. Який найбільш імовірний діагноз?

Трихінельоз

Теніаринхоз

Теніоз

Цистицеркоз

Філяріатоз

619 / 6854
У загальному вигляді генетичний апарат еукаріот є таким: екзон-інтрон-екзон. Така структурно-функціональна організація гена зумовлює особливості транскрипції. Якою буде про-іРНК відповідно до згаданої схеми?

Екзон-екзон-інтрон

Інтрон-екзон

Екзон-інтрон

Екзон-екзон

Екзон-інтрон-екзон

620 / 6854
Хворий 43 років чотири місяця тому переніс травматичну ампутацію лівої нижньої кінцівки. На момент огляду скаржиться на відчуття наявності ампутованої кінцівки і постійний сильний, іноді нестерпний біль у ній. Який вид болю у хворого?

Таламічний

Невралгія

Рефлекторний

Каузалгія

Фантомний

621 / 6854
Під час футбольного матчу гравець отримав травму колінного суглоба. На рентгенівському знімку виявлено перелом кістки, що лежить у товщі сухожилка чотириголового м'яза стегна. До якої з перелічених груп кісток належить ця кістка?

Повітроносні

Сесамоподібні

Трубчасті

Змішані

Плоскі

622 / 6854
У пацієнта після травми виникли паралічі, розлади больової чутливості справа; зліва - паралічі відсутні, але порушена больова і температурна чутливість. Яка причина такого явища?

Пошкодження рухової зони кори головного мозку

Пошкодження середнього мозку

Пошкодження мозочка

Однобічне пошкодження спинного мозку з правого боку

Пошкодження стовбура мозку

623 / 6854
Під час дослідження сироватки крові у пацієнта з ознаками імунодефіциту виявлено антитіла до білків gP120 і gP41. Наявність якої інфекції у хворого підтверджує цей результат?

HBV-інфекція

ЕСНО-інфекція

TORCH-інфекція

ВІЛ-інфекція

HLTV-1-інфекція

624 / 6854
У дитини було діагностовано перелом плечової кістки. Зламана рука почала відставати в рості. Яка частина кістки постраждала?

Кістковомозковий канал

Метафіз

Діафіз

Епіфіз

Апофіз

625 / 6854
Через деякий час після інтенсивного фізичного тренування у спортсмена активується глюконеогенез. Що є його основним субстратом?

α-кетоглутарат

Лактат

Аспарагінова кислота

Серин

Глутамінова кислота

626 / 6854
До інфекційного відділення лікарні госпіталізовано хворого з діагнозом бактеріальної дизентерії. Лабораторними дослідженнями встановлено, що збудник чутливий до багатьох протимікробних засобів, однак у хворого виявлені явища анемії. Який препарат протипоказаний хворому?

Фталазол

Ентеросептол

Фуразолідон

Левоміцетин

Ампіцилін

627 / 6854
У людини частота серцевих скорочень утримується на рівні, що не перевищує 40 разів за хвилину. Що є водієм ритму серця у цієї людини?

Ніжки пучка Гіса

Пучок Гіса

Синоатріальний вузол

Атріовентрикулярний вузол

Волокна Пуркіньє

628 / 6854
У пацієнта під час відвідування стоматолога виникла різка гіпотензія. Який із пре- паратів, що стимулює адренергічні структури, слід використати для нормалізації артеріального тиску?

Ерготамін

Мезатон

Доксазозин

Ксилометазолін

Санорин

629 / 6854
Прозерин при систематичному введенні щуру підвищує тонус скелетних м'язів. Фторотан викликає релаксацію скелетних м'язів і послаблює ефекти прозерину. Визначте характер взаємодії прозерину і фторотану.

Конкурентний антагонізм

Прямий функціональний антагонізм

Непрямий функціональний антагонізм

Незалежний антагонізм

Неконкурентний антагонізм

630 / 6854
У зв'язку з підозрою на внутрішньолікарняну інфекцію у відділенні новонароджених пологового будинку проведено обстеження. У кількох дітей, а також на деяких предметах догляду виявлено золотистий стафілокок. Які властивості виділених культур дають можливість встановити їхнє походження з одного джерела?

Антигенна структура

Фаготип

Антибіотикограма

Біохімічна активність

Пігментоутворення

631 / 6854
На базарі громадянин продавав ковбасу під назвою 'свиняча домашня'. У держсанінспекції виникла підозра фальсифікації ковбаси . За допомогою якої серологічної реакції імунітету можна ідентифікувати харчовий продукт?

Аглютинації

PЗK Е. РНГА

Імунофлуоресценції

Преципітації

632 / 6854
У хворого після важкої травми розвинувся шок та з'явилися ознаки гострої ниркової недостатності [ГНН]. Що є провідним механізмом розвитку ГНН у даному випадку?

Порушення відтоку сечі

Зниження онкотичного тиску крові

Підвищення тиску в капсулі нефрону

Підвищення тиску в ниркових артеріях

Падіння артеріального тиску

633 / 6854
Студент старанно конспектує лекцію. Якість конспектування значно погіршилася, коли сусіди стали розмовляти. Який вид гальмування в корі головного мозку є причиною цього?

Позамежне

Зовнішнє

Запізніле

Диференціювальне

Згасаюче

634 / 6854
У хворого 39 років після променевої терапії з приводу пухлини печінки утворилася виразка тонкої кишки внаслідок пригнічення мітотичної активності клітин, за рахунок яких відбувається поновлення покривного епітелію тонкої кишки. Мітотичну активність яких клітин пригнічено у даного хворого?

Келихоподібних екзокриноцитів

Екзокриноцитів з ацидофільною зернистістю

Стовпчастих епітеліоцитів

Ендокринних клітин

Стовпчастих клітин крипт без облямівки

635 / 6854
Хворій 34 років 3 роки тому було встановлено діагноз гломерулонефриту. За останні 6 місяців з'явилися набряки. Що лежить в основі їх розвитку?

Порушення білковоутворюючої функції печінки

Гіперальдостеронізм

Гіперпродукція вазопресину

Протеїнурія

Гіперосмолярність плазми

636 / 6854
Під час бактеріологічного дослідження гнійних виділень з уретри було встановлено наявність бактерій, які за Грамом фарбувалися негативно, нагадували кавові зернини, розщеплювали глюкозу і мальтозу до кислоти, знаходилися всередині лейкоцитів. Збудниками якої хвороби є ці мікроорганізми?

Сифіліс

М'який шанкр

Венеричний лімфогранулематоз

Гонорея

Меліоїдоз

637 / 6854
Проводиться каріотипування клітин здорової людини. У каріотипі знайдено дрібну акроцентричну непарну хромосому. Якою хромосомою вона може бути?

Хромосома групи А

Хромосома групи В

Х-хромосома

Хромосома групи С

Y-хромосома

638 / 6854
При травмі у пацієнта 44 років виник розрив сухожилків м'язів лівої долоні, поверхневих кровоносних судин. Після оперативного втручання і видалення більшої частини некротично зміненої м'язової тканини кровотік був відновлений. За рахунок яких судин?

Аа. metacarpeae palmares

Аа. perforantes

Arcus palmaris profundus

Arcus palmaris superficialis

Аа. digitales palmares communes

639 / 6854
У хворого 30 років з різаною раною передпліччя виникло порушення розгинання пальців кисті. Про пошкодження якого нерва це свідчить?

Медіальний шкірний нерв передпліччя

Ліктьовий

М'язовошкірний

Серединний

Променевий

640 / 6854
До приймального відділення доставлено хворого у непритомному стані. Шкіра холодна, зіниці звужені, дихання з утрудненням, відзначається періодичність по типу Чейна-Стокса, артеріальний тиск знижений, сечовий міхур переповнений. Отруєння якою речовиною найбільш вірогідне?

Ненаркотичними анальгетиками

М-холіноблокаторами

Транквілізаторами

Наркотичними анальгетиками

641 / 6854
Малюк попросив Вас надути повітряну кульку якомога дужче за один видих. Яким об'ємом повітря Ви скористаєтесь?

Загальна ємність легень

Резервний об'єм вдиху

Функціональна залишкова ємність

Ємність вдиху

Життєва ємність легень

642 / 6854
Жінка 33 років, яка тривалий час лікується з приводу хронічного поліартриту, скаржиться на підвищення артеріального тиску, зміни розподілу жирової тканини, порушення менструального циклу. З прийманням якого препарату пов'язані ці скарги?

Синафлан

Бутадіон

Індометацин

Преднізолон

Беклометазон

643 / 6854
Хворому на сечокам'яну хворобу після обстеження призначили алопуринол - конкурентний інгібітор ксантиноксидази. Підставою для цього був хімічний аналіз ниркових каменів, які складалися переважно з:

Сульфату кальцію

Урату натрію

Фосфату кальцію

Дигідрату оксалату кальцію

Моногідрату оксалату кальцію

644 / 6854
У людини хірургічно видалили ушкоджену патологічним процесом дистальну чверть тонкої кишки. Як це позначиться на всмоктуванні поживних речовин при звичайному харчовому раціоні?

Зменшиться всмоктування жирів

Зменшиться всмоктування вуглеводів

Всмоктування суттєво не зміниться

Зменшиться всмоктування води

Зменшиться всмоктування білків

645 / 6854
Під час дослідження біоптату шкіри хворого на алергічний васкуліт знайдено: стінка судин потовщена, гомогенна, пікрофуксином забарвлюється у жовтий колір, ШІК-позитивна. Який патологічний процес розвинувся в стінках судин?

Фібриноїдне набухання

Амілоїдоз

Мукоїдне набухання

Ліпідоз

Гіаліноз

646 / 6854
У людини внаслідок патологічного процесу збільшена товщина альвеолокапілярної мембрани. Безпосереднім наслідком цього буде зменшення:

Кисневої ємкості крові

Резервного об'єму видиху

Альвеолярної вентиляції легень

Хвилинного об'єму дихання

Дифузійної здатності легень

647 / 6854
З метою серологічної діагностики коклюшу поставлена розгорнута реакція з коклюшним та паракоклюшним діагностикумами. На дні пробірок, до яких було внесено діагностикум з Bordetella parapertussis, утворився зернистий осад. Які антитіла виявила ця реакція?

Бактеріолізини

Преципітини

Аглютиніни

Опсоніни

Антитоксини

648 / 6854
У хворих на цукровий діабет і під час голодування в крові збільшується вміст ацетонових тіл, що використовуються в якості енергетичного матеріалу. З якої речовини вони синтезуються?

Цитрат

Ацетил-КоА

Сукциніл-КоА

Кетоглутарат

Малат

649 / 6854
На мікропрепараті червоного кісткового мозку виявляються численні капіляри, через стінку яких у кровоносне русло виходять зрілі формені елементи крові. До якого типу належать ці капіляри?

Вісцеральні

Синусоїдні

Фенестровані

Соматичні

Лімфатичні

650 / 6854
Хворий помер при явищах уремії. На розтині нирки збільшені, в'ялої консистенції, кірковий шар широкий, набряклий, з червоним крапом; мозкова речовина темночервона. Мікроскопічно у порожнині капсули клубочків виявлені 'півмісяці', які стискають капіляри, дистрофія нефроцитів, набряк та інфільтрація строми. Яке захворювання було причиною смерті цього чоловіка?

Нефротичний синдром

Гломерулонефрит

Пієлонефрит

Нефролітіаз

Амілоїдоз нирок

651 / 6854
Під час обстеження хворого виявлене новоутворення у білій речовині півкуль великого мозку з локалізацією у коліні та передньому відділі задньої ніжки внутрішньої капсули. Волокна якого провідного шляху будуть зруйновані?

Tr. frontothalamicus

Tr. frontopontinus

Tr. thalamocorticalis

Tr. parietooccipitopontinus

Tr. pyramidalis

652 / 6854
Під час обстеження юнака з розумовою відсталістю виявлено євнухоїдну будову тіла, недорозвиненість статевих органів. У клітинах порожнини рота - статевий хроматин. Який метод генетичного дослідження слід застосувати для уточнення діагнозу?

Клініко-генеалогічний

Цитологічний

Популяційно-статистичний

Дерматогліфіку

Біохімічний

653 / 6854
У хворого лікар діагностував гостру гонорею. З анамнезу стало відомо, що раніше він переніс гонорею і вилікування було повним. До якої категорії інфекцій можна віднести це нове захворювання?

Автоінфекція

Рецидив

Реінфекція

Вторинна інфекція

Суперінфекція

654 / 6854
Недбалий студент раптово зустрівся з деканом. Концентрація якого гормону найшвидше збільшиться в крові студента?

Кортикотропін

Адреналін

Соматотропін

Тироліберин

Кортизол

655 / 6854
Хворий 50 років скаржиться на спрагу, п'є багато води, виражена поліурія. Глюкоза крові 4,8 ммоль/л, в сечі глюкози і ацетону немає, сеча безбарвна, питома вага 1,002-1,004. Яка причина поліурії?

Альдостеронізм

Нестача вазопресину

Тиреотоксикоз

Гіпотиреоз

Інсулінова недостатність

656 / 6854
Після аналізу родоводу, лікар – генетик встановив: ознака проявляється у кожному поколінні, жінки та чоловіки успадковують ознаку з однаковою частотою, батьки в однаковій мірі передають ознаку своїм дітям. Який тип успадкування має досліджувана ознака?

Аутосомно- домінантний

Y-зчеплене успадкування

Аутосомно- рецесивний

Х-зчеплене домінантне успадкування

Х-зчеплене рецесивне успадкування

657 / 6854
Чоловік 59 років має ознаки паренхіматозної жовтяниці та портальної гіпертензії. Під час гістологічного дослідження пункційного біоптату печінки знайдено: балково-часточкова будова порушена, частина гепатоцитів має ознаки жирової дистрофії, утворюються порто-портальні сполучнотканинні септи з формуванням псевдочасточок, з наявністю перипортальних лімфо-макрофагальних інфільтратів. Який найбільш імовірний діагноз?

Хронічний гепатоз

Токсична дистрофія

Вірусний гепатит

Цироз печінки

Алкогольний гепатит

658 / 6854
В експерименті у кроля було видалено верхній шийний вузол симпатичного стовбура. На боці видалення спостерігається почервоніння і підвищення температури шкіри голови. Яка форма порушень периферичного кровообігу розвинулася в цьому випадку?

Нейротонічна артеріальна гіперемія

Метаболічна артеріальна гіперемія

Нейропаралітична артеріальна гіперемія

Стаз

Венозна гіперемія

659 / 6854
Забір крові для загального аналізу рекомендують проводити натщесерце і зранку. Які зміни складу крові можливі, якщо провести забір крові після прийому їжі?

Збільшення білків плазми

Збільшення кількості лейкоцитів

Зниження кількості тромбоцитів

Зниження кількості еритроцитів

Збільшення кількості еритроцитів

660 / 6854
У хворого із значними периферійними набряками почергове застосування дихлотіазиду, етакринової кислоти і фуросеміду не сприяло значному діуретичному ефекту. У крові - значне підвищення кількості альдостерону. Вкажіть препарат вибору.

Клопамід

Маніт

Спіронолактон

Сечовина

Амілорид

661 / 6854
Під час мікроскопії біоптату печінки виявлені гранульоми, які складаються з плазматичних, лімфоїдних, гігантських багатоядерних клітин типу Пирогова- Ланхганса, дрібних судин з ознаками ендо- і периваскуліту, зустрічаються осередки казеозного некрозу. Для якого захворювання характерно утворення таких гранульом?

Лепра

Сифіліс

Риносклерома

Туберкульоз

Сап

662 / 6854
Хвора 25 років звернулася зі скаргами на погіршення зору. При огляді виявлено порушення акомодації, зіниця розширена, не реагує на світло. Функція яких м'язів порушена?

М'яз, що звужує і м'яз, що розширює зіницю

Латеральний прямий, м'яз, що звужує зіницю

М'яз, що звужує зіницю, війковий

Верхній косий, війковий

М'яз, що розширює зіницю, війковий

663 / 6854
У 6-місячної дитини спостерігалися часті та інтенсивні підшкірні крововиливи. Призначення синтетичного аналога вітаміну К (вікасолу) дало позитивний ефект. В гама-карбоксилюванні глутамінової кислоти якого білку згортаючої системи крові бере участь цей вітамін?

Фактор Розенталя

Фактор Хагемана

Фібриноген

Антигемофільний глобулін А

Протромбін

664 / 6854
У результаті землетрусу чоловік 50 років два дні перебував під завалом. Після звільнення з-під завалу у нього було встановлено діагноз синдрому тривалого розчавлення. Виникнення якого ускладнення в подальшому найбільш імовірне у цьому випадку?

Гостра ниркова недостатність

Гостра дихальна недостатність

Гостра печінкова недостатність

Гостра судинна недостатність

Гостра серцева недостатність

665 / 6854
У хворого 27 років діагностовано гнійне запалення жовчного міхура. До якого відділу очеревинної порожнини потрапить гній під час розриву жовчного міхура при його типовому положенні.

До чепцевої сумки

До печінкової сумки

До верхнього дванадцятипалого закутка

До передшлункової сумки

До лівого бічного каналу

666 / 6854
До хірурга звернулася хвора з приводу наявності ущільнення у молочній залозі. У якому напрямку має здійснити розріз хірург під час операції для зменшення травмування часточки?

Дугоподібно

Поперечно

Радіально

Вертикально

667 / 6854
Хворий помер на 3-й день після операції з приводу перфорації стінки товстої кишки з явищами розлитого гнійного перитоніту. На розтині: слизова оболонка стінки товстої кишки потовщена, вкрита фібриновою плівкою , поодинокі виразки проникають на різну глибину. Гістологічно: некроз слизової, інфільтрація лейкоцитами з фокусами геморагій. Ускладнення якого захворювання стало причиною смерті хворого?

Амебіаз

Хвороба Крона

Дизентерія

Неспецифічний виразковий коліт

Черевний тиф

668 / 6854
Під час обертання на каруселі у жінки 25 років з'явилися нудота, блювання, посилення потовиділення. Активація яких рецепторів зумовила рефлекторний розвиток цих симптомів?

Отолітових вестибулярних рецепторів

Зорових рецепторів

Вестибулярних рецепторів півколових каналів

Пропріорецепторів скелетних м'язів

Рецепторів кортієвого органу

669 / 6854
До хірургічного відділення надійшов чоловік 35 років з гнійною раною шиї попереду трахеї (в ділянці передвісцерального простору). Куди може поширитися інфекція, якщо терміново не буде проведене оперативне втручання?

У переднє середостіння

У надгруднинний міжапоневротичний простір

У ретровісцеральний простір

У заднє середостіння

У середнє середостіння

670 / 6854
Через два тижні після переливання крові у реципієнта виникла пропасниця. Про яке протозойне захворювання можна думати?

Лейшманіоз

Токсоплазмоз

Трипаносомоз

Амебіаз

Малярію

671 / 6854
До лікаря звернулися пацієнти з подібними скаргами: слабкість, болі у животі, рідкі випорожнення. Після дослідження фекалій з'ясувалося, що терміновій госпіталізації підлягає один з пацієнтів, у якого були виявлені цисти з чотирма ядрами. Для якого найпростішого характерні такі цисти?

Трихомонада

Балантидій

Дизентерійна амеба

Кишкова амеба

Лямблія

672 / 6854
У людини внаслідок втрати 1,5 л крові різко зменшився діурез. Посилена секреція якого гормону спричинила зміни діурезу?

Кортизол

Вазопресин

Паратгормон

Кортикотропіну

Натрійуретичний

673 / 6854
Хворому 56 років із скаргами на спрагу та часте сечовиділення було встановлено діагноз цукрового діабету та призначено бутамід. Яким є механізм дії цього препарату?

Стимулює бета-клітини острівців Лангерганса

Полегшує транспорт глюкози через клітинні мембрани

Пригнічує альфа-клітини острівців Лан герганса

Пригнічує всмоктування глюкози в кишечнику

Сприяє засвоєнню глюкози клітинами тканин організму

674 / 6854
Для знеболювання маніпуляції, пов'язаної із обробкою опеченої поверхні, ввели внутрішньовенно препарат для короткочасного наркозу. Наркоз настав через хвилину, під час нього спостерігалися підвищення AT, тахікардія, підвищення тонусу скелетних м'язів; рефлекси збережені. Після виходу із наркозу у хворого відзначалися дезорієнтація, зорові галюцинації. Який препарат ввели хворому?

Закис азоту

Тіопентал натрію

Сомбревін

Кетамін

Ефір для наркозу

675 / 6854
Під час бактеріологічного дослідження випорожнень чотиримісячної дитини з симптомами гострої кишкової інфекції на середовищі Ендо виросли у великій кількості червоні колонії. Які це можуть бути мікроорганізми?

Стрептококи

Ешерихії

Сальмонели

Стафілококи

Шигели

676 / 6854
У здорової дорослої людини швидкість проведення збудження через атріовентрикулярний вузол дорівнює 0,02-0,05 м/с. Що забезпечує атріовентрикулярна затримка?

Послідовність скорочення передсердів та шлуночків

Одночасність скорочення обох перед- сердів

Достатню силу скорочення шлуночків

Одночасність скорочення обох шлуночків

Достатню силу скорочення передсердь

677 / 6854
У дитини, яка страждає на пілоростеноз, що супроводжується частими блюваннями, розвинулися ознаки зневоднення організму. Яка форма порушення кислотнолужного стану може розвинутися у даному випадку?

Газовий ацидоз

Негазовий ацидоз

Негазовий алкалоз

Метаболічний ацидоз

Газовий алкалоз

678 / 6854
При ентеробіозі призначають акрихін - структурний аналог вітаміну В2. Порушення синтезу яких ферментів у мікроорганізмів викликає цей препарат?

Пептидази

Цитохромоксидази

ФАД-залежні дегідрогенази

Амінотрансферази

НАД-залежні дегідрогенази

679 / 6854
Хворому на бронхіальну астму призначено інгаляції 0,5% розчину ізадрину. Бронхоспазм був усунутий, але пацієнт почав скаржитися на болі в серці та серцебиття. Це пов'язано зі стимуляцією:

Синтезу ацетилхоліну

α1-адренорецепторів

β2-адренорецепторів

β1-адренорецепторів

М-холінорецепторів

680 / 6854
Хвора 13 років знаходиться на стаціонарному лікуванні в гематологічному відділенні обласної дитячої лікарні з діагнозом залізодефіцитна анемія. Який тип гіпоксії має місце у цієї хворої?

Змішана

Тканинна

Циркуляторна

Дихальна

Гемічна

681 / 6854
Для запобігання віддалених результатів чотириденної малярії пацієнту 42 років призначили примахін. Вже на 3 добу від початку лікування терапевтичними дозами препарату у пацієнта з'явилися болі в животі та в ділянці серця, диспептичні розлади, загальний ціаноз, гемоглобінурія. Що є причиною розвитку побічної дії препарату?

Генетична недостатність глюкозо-6-фосфат-дегідрогенази

Кумуляція лікарського засобу

Зниження активності мікросомальних ферментів печінки

Потенціювання дії іншими препаратами

Сповільнення екскреції препарату з сечею

682 / 6854
У хворого, який проходить курс лікувального голодування, нормальний рівень глюкози у крові підтримується головним чином за рахунок глюконеогенезу. З якої амінокислоти у печінці людини найбільш активно синтезується глюкоза?

Аланін

Валін

Лізин

Лейцин

Глутамінова кислота

683 / 6854
Хворому з метою відновлення дихання при отруєнні чадним газом було введено аналептичний засіб рефлекторного типу дії з групи Н-холіноміметиків. Який засіб було призначено хворому?

Пентамін

Мезатон

Лобеліну гідрохлорид

Адреналіну гідрохлорид

Атропіну сульфат

684 / 6854
У людини внаслідок довільної затримки дихання на 40 с зросли системний артеріальний тиск та частота серцевих скорочень. Реалізація яких механізмів регуляції зумовлює зміни показників?

Умовні симпатичні рефлекси

Безумовні симпатичні рефлекси

Безумовні парасимпатичні рефлекси

Умовні парасимпатичні рефлекси

685 / 6854
У хлопчика 2 років спостерігається збільшення у розмірах печінки та селезінки, катаракта. У крові підвищена концентрація цукру, однак тест толерантності до глюкози в нормі. Спадкове порушення обміну якої речовини є причиною цього стану?

Галактоза

Мальтоза

Глюкоза

Сахароза

Фруктоза

686 / 6854
Протягом двох тижнів хвора приймала мікстуру, призначену невропатологом з приводу неврастенії. Самопочуття хворої дещо покращилося, однак незабаром з'явилися скарги на нежить, кон'юнктивіт, шкірні висипи, млявість та послаблення пам'яті. Був встановлений діагноз 'бромізм'. Що доцільно призначити для послаблення симптомів?

Аспаркам

Розчин глюкози 5%

Поліглюкін

Натрію хлорид

687 / 6854
Під час вивчення фаз мітотичного циклу корінця цибулі знайдено клітину, в якій хромосоми лежать в екваторіальній площині, створюючи зірку. На якій стадії мітозу перебуває клітина?

Метафаза

Анафаза

Інтерфаза

Профаза

Телофаза

688 / 6854
У хворих на колагеноз має місце процес деструкції сполучної тканин. Це підтверджується збільшенням у крові:

Активності трансаміназ

Вмісту оксипроліну та оксилізину

Активності ізоферментів ЛДГ

Вмісту уратів

Вмісту креатину та креатиніну

689 / 6854
У хворого з опіковою хворобою наступила смерть внаслідок прогресування сепсису. Під час розтину тіла в ділянці передньої черевної стінки за допомогою мікроскопічного дослідження виявлено дифузну інфільтрацію міжм'язевих проміжків сегментоядерними лейкоцитами, набряк тканин, лізис м'язових волокон. Визначте характер патологічного процесу.

Флегмона

Катаральне запалення

Некроз

Дифтеритичне запалення

Абсцес

690 / 6854
У хворого з періодичними нападами ядухи, які виникають при вдиханні різних ароматичних речовин, діагностована атопічна бронхіальна астма. Визначено збільшення Ig Е. Для якого типу реакцій це характерно?

Імунокомплексні

Анафілактичні

Гіперчутливості сповільненого типу

Цитотоксичні

Автоімунні

691 / 6854
В експерименті збільшили проникність мембрани збудливої клітини для іонів калію. Які зміни електричного стану мембрани виникатимуть при цьому?

Деполяризація

Потенціал дії

Гіперполяризація

Локальна відповідь

Змін не буде

692 / 6854
Жінка 26 років скаржиться на сухість у роті, спрагу. Під час обстеження: глюкоза крові - 6,5 ммоль/л, глюкозурія. Для якого з зазначених нижче станів найбільш характерні ці симптоми?

Нирковий діабет

Цукровий діабет

Стероїдний діабет

Нецукровий діабет

Аліментарна глюкозурія

693 / 6854
У хворого 30 років з дизентерією, підтвердженою бактеріологічно, з'явились ознаки парапроктиту. Про яку стадію місцевих змін найбільш імовірно йде мова у даного хворого?

Катаральний коліт

Фібринозний коліт

Утворення виразок

Фолікулярний коліт

Загоєння виразок

694 / 6854
На прийом до лікаря звернувся хворий з симетричним дерматитом відкритих ділянок шкіри. З бесіди з пацієнтом встановлено, що він харчується здебільшого крупами і їсть мало м'яса, молока і яєць. Дефіцит якого з перерахованих вітамінів домінує у цього пацієнта?

Кальциферол

Токоферол

Фолієва кислотна

Нікотинамід

Біотин

695 / 6854
Хворий надійшов до клініки з пораненням у ділянці шиї. Під час обстеження виявлено пошкоджений нерв, розташований попереду переднього драбинчастого м'яза. Який нерв ушкоджено?

Під'язиковий

Язикоглотковий

Шийний відділ симпатичного стовбуру

Блукаючий

Діафрагмальний

696 / 6854
У харкотинні хворого з підозрою на пневмонію виявлено грампозитивні диплококи, подовжені, з дещо загостреними протилежними кінцями. Які мікроорганізми виявлені у харкотинні?

Staphylococcus aureus

Streptococcus pneumoniae

Neisseria meningitidis

Neisseria gonorrhoeae

Klebsiella pneumoniae

697 / 6854
Під час інтраопераційної біопсії щитоподібної залози гістологічно серед фолікулів, заповнених колоїдом, виявлені лімфоїдні структури із центрами росту. Яке захворювання у хворого?

Базедів зоб

Спорадичний зоб

Ендемічний зоб

Зоб Хашимото

Зоб Ріделя

698 / 6854
Скарги та об'єктивні дані дозволяють припустити наявність у хворого запального процесу в жовчному міхурі, порушення колоїдних властивостей жовчі, імовірність утворення жовчних каменів. Що головним чином може спричинити утворення каменів?

Хлориди

Холестерин

Урати

Оксалати

Фосфати

699 / 6854
Людина після травми головного мозку втратила зір. Пошкодження якої зони кори головного мозку може бути причиною цього?

Потилична

Лобна

Скронева

Тім'яна

Скронева та тім'яна

700 / 6854
У хворого спостерігаються часті кровотечі з внутрішніх органів, слизових оболонок. Аналіз виявив недостатність гідроксипроліну та гідроксилізину у складі колагенових волокон. Через нестачу якого вітаміну порушено в організмі пацієнта процеси гідроксилювання названих амінокислот?

Вітамін Н

Вітамін РР

Вітамін С

Вітамін К

Вітамін А

701 / 6854
У хворого на сухий плеврит вислуховується шум тертя плеври. При ураженні якого виду епітелію відмічається цей симптом?

Одношаровий плоский епітелій

Одношаровий кубічний епітелій

Багатошаровий епітелій

Перехідний епітелій

Одношаровий призматичний епітелій

702 / 6854
До лікаря звернулася хвора зі скаргами на розлади травлення, розлитий біль у животі. Під час обстеження лікар виявив різко виражене зниження гемоглобіну в крові. З опитування виявилося, що, проживаючи на Далекому Сході, хвора часто вживала у їжу малосольну рибну ікру. Аналогічний стан відзначений у деяких родичів, що проживають з нею. Яке захворювання діагностував лікар у цієї хворої?

Дифілоботріоз

Трихінельоз

Ехінококоз

Аскаридоз

Теніоз

703 / 6854
Хворому на миготливу аритмію, у якого в анамнезі бронхіальна астма, треба призначити протиаритмічний засіб. Який препарат з цієї групи протипоказаний хворому?

Новокаїнамід

Верапаміл

Ніфедипін

Анаприлін

Аймалін

704 / 6854
У чоловіка 52 років діагностовано системний амебіаз з ураженням кишечнику, печінки, легень. Який препарат слід призначити?

Хінгамін

Тетрациклін

Ентеросептол

Метронідазол

Хініофон

705 / 6854
У хворого з розладом мозкового кровотоку порушено акт ковтання, він може поперхнутися під час прийому рідкої їжі. Який відділ мозку уражено?

Проміжний мозок

Шийний відділ спинного мозку

Мозочок

Середній мозок

Довгастий мозок

706 / 6854
Під час постсинтетичного періоду мітотичного циклу було порушено синтез білків - тубулінів, які беруть участь у побудові веретена поділу Це може призвести до порушення:

Цитокінезу

Розходження хромосом

Деспіралізації хромосом

Тривалості мітозу

Спіралізації хромосом

707 / 6854
У 2-річної дитини кишковий дисбактеріоз, на фоні якого виник геморагічний синдром. Що є найбільш імовірною причиною геморагій у цієї дитини?

Нестача вітаміну К

Дефіцит фібриногену

Активація тромбопластину тканин

Гіповітаміноз РР

Гіпокальціємія

708 / 6854
До лікаря-генетика звернувся юнак 18 років астенічної будови тіла: вузькі плечі, широкий таз, високий зріст; незначне оволосіння обличчя. Виражена розумова відсталість. Було встановлено попередній діагноз: синдром Клайнфельтера. Який метод медичної генетики дозволяє підтвердити цей діагноз?

Дерматогліфіка

Популяційно-статистичний

Генеалогічний

Близнюковий

Цитогенетичний

709 / 6854
У хворого на глаукому спостерігається підвищення внутрішньоочного тиску при нормальній секреції водянистої вологи циліарним тілом. З ушкодженням яких структур стінки очного яблука пов'язане порушення відтоку рідини з передньої камери?

Задній епітелій рогівки

Війковий м'яз

Циліарне тіло

Венозний синус

Судинна оболонка

710 / 6854
Під час вивчення родоводу сім'ї, в якій спостерігається гіпертрихоз (надмірне оволосіння вушних раковин) встановлено, що ця ознака трапляється в усіх поколіннях тільки у чоловіків і успадковується від батька до сина. Який тип успадкування гіпертрихозу?

Зчеплений з Y-хромосомою

Зчеплений з Х-хромосомою рецесивний

Зчеплений з Х-хромосомою домінантний

Аутосомно-домінантний

Аутосомно-рецесивний

711 / 6854
У положенні на спині дитина 6 місяців задихається. Пальпаторно на передній стінці трахеї до яремної вирізки визначається пухлиноподібне утворення, яке про- довжується в переднє середостіння. Що з наведеного може стискати трахею?

Щитоподібна залоза

Навколощитоподібні лімфатичні вузли

Загруднинна залоза

Навколотрахеальні лімфатичні вузли

Паращитоподібні залози

712 / 6854
При визначенні групи крові за системою АВ0 аглютинацію еритроцитів досліджуваної крові викликали стандартні сироватки І та II груп і не викликала III групи. Які аглютиногени містяться в цих еритроцитах?

D та C

В

А та В

А

C

713 / 6854
У померлого від ядухи чоловіка, який багато років страждав на бронхіальну астму, при гістологічному дослідженні легень виявлено: у просвіті бронхіол та дрібних бронхів багато слизу з вмістом еозинофілів, склероз міжальвеолярних перетинок, розширення просвіту альвеол. Який з механізмів розвитку реакції гіперчутливості має місце?

Реагіновий

Гранулематоз

Цитотоксичний

Імунокомплексний

Цитоліз, обумовлений лімфоцитами

714 / 6854
У хворого спостерігаються напади артеріальної гіпертензії, які супроводжуються тахікардією, різким болем у надчеревній ділянці. Для якого з перелічених захворювань найбільш характерні ці симптоми?

Феохромоцитома

Аденома щитовидної залози

Базофільна аденома гіпофіза

Пухлина яєчників

Аденома клубочкової зони наднирників

715 / 6854
У жінки 35 років із хронічним захворюванням нирок розвинувся остеопороз. Дефіцит якого з нижчеперерахованих речовин є основною причиною цього ускладнення?

Холестерин

D2

1.25(ОH)2D3

25OHD3

D3

716 / 6854
У хворого спостерігається ішемія тканин нижче колінного суглоба, що супроводжується «переміжною кульгавістю». Про оклюзію якої артерії слід думати?

Малогомілкова

Задня великогомілкова

Підколінна

Передня великогомілкова

Проксимальна частина стегнової

717 / 6854
У хворого з захворюванням печінки виявлено зниження вмісту протромбіну в крові. Це призведе, перш за все, до порушення:

Фібринолізу

Антикоагулянтних властивостей крові

Першої фази коагуляційного гемостазу

Другої фази коагуляційного гемостазу

Судинно-тромбоцитарного гемостазу

718 / 6854
Під час фарбування мазка з харкотиння хворого з підозрою на крупозну пневмонію були використані наступні барвники і реактиви: розчин генціанвіолету, розчин Люголя, 96° спирт, водний фуксин. Який спосіб фарбування застосований у даному випадку?

За Нейсером

За Романовським

За Грамом

За Лефлером

За Цилем-Нільсеном

719 / 6854
У чоловіка 50 років раптово виникли сильне серцебиття, біль у серці, різка слабкість, підвищення AT; пульс аритмічний, з дефіцитом. На ЕКГ виявлено відсутність зубців Р і різна тривалість інтервалів R-R. Яке порушення серцевого ритму у хворого?

Синусова екстрасистолія

Дихальна аритмія

Поперечна блокада серця

Пароксизмальна тахікардія

Миготлива аритмія

720 / 6854
У збудливій клітині заблокували іонні канали. Це суттєво не змінило рівень потенціалу спокою, але клітина втратила здатність до генерації ПД. Які канали заблоковано?

Кальцієві

Натрієві та калієві

Натрієві

Калієві

Хлорні

721 / 6854
У печінці хворого порушена детоксикація природних метаболітів та ксенобіотиків. Знижена активність якого хромопротеїду може бути причиною цього?

Гемоглобін

Цитохром В

Цитохром С-1

Цитохромоксидаза

Цитохром Р-450

722 / 6854
У хворого із запаленням легень непе- реносимість антибіотиків. Який з комбінованих сульфаніламідних препаратів слід призначити хворому?

Стрептоцид

Бісептол

Сульфадиметоксин

Етазол

Сульфацил натрію

723 / 6854
Хворому 65 років з ознаками загального ожиріння, небезпекою жирової дистрофії печінки рекомендована дієта, збагачена ліпотропними речовинами. Вміст якої речовини у раціоні має найбільше значення у даному випадку?

Метіонін

Гліцин

Холестерин

Вітамін С

Глюкоза

724 / 6854
У хворого на гострий мієлобластний лейкоз виявлено: збільшення печінки, селезінки, анемія, мієлобласти в периферичній крові. Яка принципова ознака дозволяє відрізнити гострий мієлобластний лейкоз від хронічного?

Бластні клітини в периферичній крові

Панцитопенія

Анемія

Лейкемічний провал

Тромбоцитопенія

725 / 6854
У молодої людини в м'яких тканинах лівого стегна з'явилося безболісне новоутворення без чітких меж. У біоптаті тканини новоутворення нагадує риб'яче м'ясо, складаються з незрілих фібробластоподібних клітин з численними мітозами, які проростають в м'язи. Який найбільш імовірний діагноз?

Міосаркома

Фіброма

Міома

Фібросаркома

Рaк

726 / 6854
Літня людина перенесла інфаркт правої півкулі головного мозку. Через рік, враховуючи відсутність рухів лівих кінцівок, проведено комп'ютерну томографію мозку, під час якої в правій півкулі знайдено порожнину з гладенькими стінками, заповнену ліквором. Який патологічний процес виявлено у головному мозку?

Гематома

Інфаркт мозку

Гідроцефалія

Постінфарктна кіста

Сіре розм'якшення мозку

727 / 6854
У хворого, який страждає на вроджену еритропоетичну порфірію, відзначена світлочутливість шкіри. Накопиченням якої сполуки в клітинах шкіри це обумовлено?

Копропорфіриноген III

Гем

Уропорфіриноген II

Уропорфіриноген І

Протопорфірин

728 / 6854
На 8 день після введення протиправцевої сироватки з приводу брудної рани стопи у пацієнта температура підвищилася до 38°C, з'явилися болі в суглобах, висипка, свербіж. З боку крові спостерігається лейкопенія і тромбоцитопенія. Алергічна реакція якого типу розвинулася в цьому випадку?

Стимулюючого

Гіперчутливості уповільненого типу

Анафілактичного

Імунокомплексного

Цитотоксичного

729 / 6854
Хвора 44 років надійшла до терапевтичного відділення з діагнозом правобічний плеврит. Обстеження підтвердило наявність рідини в плевральній порожнині. У якому закутку плеври буде найбільше накопичення серозної рідини?

Правому реберно-середостінному

Правому середостінно-діафрагмальному

Правому реберно-діафрагмальному

Лівому реберно-середостінному

Лівому середостінно-діафрагмальному

730 / 6854
В експерименті ізольований м'яз жаби ритмічно подразнюють електричними імпульсами. Кожний наступний імпульс припадає на період розслаблення попереднього скорочення. Яке скорочення м'язу виникає?

Асинхронне

Зубчастий тетанус

Тонічне

Одиночне

Суцільний тетанус

731 / 6854
Після радіаційного опромінення у хворого зруйновано стовбурові клітини. Відновлення яких клітин пухкої волокнистої сполучної тканини буде порушено?

Перицити

Адипоцити

Пігментні клітини

Макрофаги

Фібробласти

732 / 6854
У малярійного плазмодія – збудника триденної малярії - розрізняють два штами: південний та північний. Вони відрізняються тривалістю інкубаційного періоду: у південного він короткий, а у північного - довгий. Дія якого відбору проявляється у даному випадку?

Штучний

Розсікаючий

Статевий

Стабілізуючий

Руховий

733 / 6854
У хворого в крові підвищений вміст сечової кислоти, що клінічно проявляється больовим синдромом внаслідок відкладення уратів у суглобах. У результаті якого процесу утворюється ця кислота?

Реутилізація пуринових основ

Катаболізм гему

Розпад пуринових нуклеотидів

Розщеплення білків

Розпад піримідинових нуклеотидів

734 / 6854
У хворого після видалення зуба з'явився стійкий біль за грудиною. Після вживання під язик антиангінального засобу біль за грудиною зник, але хворий поскаржився на головний біль і запаморочення. Який препарат використав хворий?

Метопролол

Верапаміл

Валідол

Нітрогліцерин

Анаприлін

735 / 6854
До терапевтичного відділення надійшов хворий з тривалою бронхопневмонією. Антибіотикотерапія не дала належного ефекту. Який препарат для підвищення імунного статусу слід додати до комплексної терапії цього хворого?

Сульфокамфокаїн

Димедрол

Аналгін

Парацетамол

Тималін

736 / 6854
Дистрофічні зміни серцевого м'яза супроводжуються розширенням порожнин серця, зниженням сили серцевих скорочень, збільшенням об'єму крові, що залишається під час систоли в порожнині серця, переповненням вен. Для якого стану серця це характерно?

Тоногенна дилатація

Аварійна стадія гіперфункції та гіпертрофії

Кардіосклероз

Міогенна дилатація

Тампонада серця

737 / 6854
У гістопрепараті представлено паренхіматозний орган, що має кіркову і мозкову речовину. Кіркова речовина утворена тяжами епітеліоцитів, між якими проходять кровоносні капіляри; тяжі формують три зони. Мозкова речовина складається з хромафіноцитів і венозних синусоїдів. Який орган має дані морфологічні ознаки?

Щитоподібна залоза

Тимус

Наднирник

Нирка

Лімфатичний вузол

738 / 6854
Під час огляду хворого з ранами, що кровоточать, лікар виявив пошкодження тканин личинками, а також локальні місця нагноєння, та встановив діагноз: облігатний міаз. Личинки яких комах є збудниками цього захворювання?

Тріатомовий клоп

Жигалка осіння

Муха вольфартова

Муха хатня

Муха цеце

739 / 6854
У мазку крові, забарвленому за Романовським-Гімзою, виявляється 20% великих (діаметром 20 мкм), округлих клітин з блідобазофільною цитоплазмою і бобоподібним ядром. Як клінічно характеризується це явище?

Нейтрофільоз

Лімфоцитоз

Ретикулоцитоз

Моноцитоз

Лейкопенія

740 / 6854
Під час тяжких пологів відбувся розрив лобкового симфізу. Який орган найбільш імовірно буде травмований?

Маткові труби

Сечовий міхур

Пряма кишка

Яєчники

Матка

741 / 6854
Під час гістологічного дослідження серця померлого від гострої серцевої недостатності у міокарді лівого шлуночка виявлена ділянка омертвіння, відмежована від неушкодженої тканини зоною повнокровних судин, дрібних крововиливів та лейкоцитарною інфільтрацією. Який діагноз найбільш імовірний?

Осередковий ексудативний міокардит

Дифузний ексудативний міокардит

Інфаркт міокарда

Продуктивний міокардит

Ішемічна дистрофія міокарда

742 / 6854
У міокарді шлуночків досліджуваної людини порушені процеси реполяризації. Це призведе до порушення амплітуди, конфігурації та тривалості зубця:

R

S

Q

P

Т

743 / 6854
У чоловіка 46 років на шкірі голо- ви визначалася пігментна пляма темного кольору, яка дещо вибухала та не спричиняла турбот. Останнім часом пляма почала збільшуватися, з'явився біль, колір став чорно-коричневим, у її основі пальпується вузлик . Під час гістологічного дослідження видаленої тканини визначалися веретеноподібні і поліморфні клітини, цитоплазма яких вміщувала пігмент бурого кольору. Про яку пухлину іде мова?

Гемангіома

Карциноїд

Гематома

Базаліома

Меланома

744 / 6854
У процесі метаболізму в організмі людини утворюються активні форми кисню, у тому числі супероксидний аніон-радикал. За допомогою якого ферменту інактивується цей аніон?

Пероксидаза

Супероксиддисмутаза

Каталаза

Глутатіонпероксидаза

Глутатіонредуктаза

745 / 6854
До медико-генетичної консультації звернулася жінка з приводу ризику захворювання на гемофілію у свого сина. Її чоловік страждає на дане захворювання з народження. Жінка та її батьки здорові стосовно гемофілії. Яка імовірність народження хворого хлопчика в цій сім'ї?

25 % хлопчиків будуть хворими

75 % хлопчиків будуть хворими

Усі хлопчики будуть хворі

Всі хлопчики будуть здорові

50% хлопчиків будуть хворими

746 / 6854
Під час огляду хворого, який знаходився на гірських пасовиськах і надійшов у тяжкому стані з високою температурою, лікар відзначив, що пахвинні лімфатичні вузли збільшені до 8 см, зпаяні з оточуючими тканинами, нерухливі, шкіра над ними червона, різко болюча. Під час мікроскопічного дослідження в лімфатичному вузлі найгостріше серозно-геморагічне запалення. Для якого захворювання це характерно?

Туляремія

Сифіліс

Чума

Бруцельоз

Сибірка

747 / 6854
У хворого на цукровий діабет з'явився різкий біль у правій стопі. Під час огляду великий палець стопи чорного кольору, тканини стопи набряклі, осередки відшарування епідермісу, виділення з неприємним запахом. Яка клініко-морфологічна форма некрозу розвинулася у хворого?

Гангрена волога

Пролежень

Інфаркт

Секвестр

Гангрена суха

748 / 6854
У хворої 38 років після прийому аспірину і сульфаніламідів спостерігається посилений гемоліз еритроцитів, викликаний недостатністю глюкозо-6фосфатдегідрогенази. Порушенням утворення якого коферменту зумовлена ця патологія?

ФАДН2

НАДФН

ФМНН2

Убіхінон

Піридоксальфосфат

749 / 6854
Під час ультразвукового обстеження серця лікар спостерігає за стулками мітрального клапана. Що відбувається з ними під час систоли передсердь?

Притискаються до стінок передсердя

Вивертаються в порожнину передсердя

Притискаються до стінки судини

Стуляються, закриваючи просвіт отвору

Вивертаються в порожнину шлуночка

750 / 6854
У мікропрепараті спинного мозку необхідно проаналізувати стан ядра, нейрони якого утворюють моторні закінчення у скелетній мускулатурі. Про яке ядро спинного мозку йдеться?

Проміжне латеральне ядро

Власне ядро сірої речовини

Власне ядро заднього рога

Грудне ядро

Власне ядро переднього рога

751 / 6854
Пацієнтка 58 років скаржиться на підвищену втомлюваність, зниження працездатності, сонливість, задишку під час швидкої ходи. У крові: еритроцити - 4,6 * 1012 /л, гемоглобін - 92 г/л, кольоровий показник - 0,6. У мазку крові - велика кількість анулоцитів і мікроцитів. Для якої анемії це характерно?

Серпоподібноклітинна

Перніціозна

Постгеморагічна

Залізодефіцитна

Гемолітична

752 / 6854
Під час дослідження первинної структури молекули глобіну виявлено заміну глутамінової кислоти на валін. Для якої спадкової патології це характерно?

Хвороба Мінковського-Шоффара

Фавізм

Таласемія

Серпоподібноклітинна анемія

Гемоглобіноз

753 / 6854
У хворої на дифтерію дитини через 10 днів після введення антитоксичної протидифтерійної сироватки з'явилася висипка на шкірі, яка супроводжувалася сильним свербінням, підвищилася температура тіла до 38°С, з'явилися болі в суглобах. Яка причина цих явищ?

Сироваткова хвороба

Анафілактична реакція

Контактна алергія

Гіперчутливість сповільненого типу

Атопія

754 / 6854
У хворого внаслідок хронічного захворювання органів дихання на фоні задишки, тахікардії і ціанозу під час дослідження газового складу крові виявлено розвиток гіпоксемії і гіперкапнії. Яке порушення зовнішнього дихання спостерігається у хворого?

Гіпервентиляція

Гіпоперфузія

Гіпердифузія

Гіповентиляція

Гіперперфузія

755 / 6854
У людини в стані спокою збільшена робота м'язів, що забезпечують вдих. Що з наведеного може бути причиною цього?

Рідке дихання

Зменшення хвилинного об'єму дихання

Негативний внутрішньоплевральний тиск

Поверхневе дихання

Звуження дихальних шляхів

756 / 6854
Біля інфікованої рани збільшилися регіонарні лімфовузли. Під час гістологічного дослідження в них виявлено збільшення кількості макрофагів, лімфоцитів і лімфатичних фолікулів у кірковому шарі, а також велику кількість плазматичних клітин. Який процес у лімфатичних вузлах відображують виявлені гістологічні зміни?

Антигенну стимуляцію

Пухлинну трансформацію

Реакцію гіперчутливості

Природжену недостатність лімфоїдної тканини

Набуту недостатність лімфоїдної тканини

757 / 6854
Послаблення кровопостачання органа зумовлює розвиток гіпоксії, яка активізує функцію фібробластів. Об'єм яких елементів збільшується в цій ситуації?

Лімфатичні судини

Судини мікроциркуляторного русла

Нервові елементи

Паренхіматозні елементи органа

Міжклітинна речовина

758 / 6854
Хворому, який звернувся до лікаря з приводу безпліддя, було призначено обстеження на токсоплазмоз і хронічну гонорею. Яку реакцію слід поставити для виявлення прихованого токсоплазмозу і хронічної гонореї у цього хворого?

Імуноблотингу

РЗНГА

РІФ

РЗПГА

РЗК

759 / 6854
Хвора на бронхіальну астму приймала протягом 2-х місяців преднізолон у таблетках. Внаслідок значного покращання стану раптово припинила його прийом. Розвиток якого ускладнення високоімовірний у цьому випадку?

Шлункова кровотеча

Ожиріння верхньої половини тулуба

Синдром Іценко-Кушінга

Синдром відміни

Гіпотонія

760 / 6854
До ендокринолога звернувся хворий зі скаргами на схуднення на 10 кг за 2 місяці, серцебиття, витрішкуватість. Для гіперфункції якої ендокринної залози (яких залоз) ці скарги найбільш характерні?

Підшлункова

Надниркові залози

Щитоподібна

Паращитоподібні залози

Яєчники

761 / 6854
Чоловік 63 років страждає на рак стравоходу, є метастази в лімфатичні вузли середостіння, ракова кахексія. Яка патогенетична стадія пухлинного процесу у цьому випадку?

Прогресії

Ініціації

Промоції

Трансформації

762 / 6854
У хворого через наявність каменя у загальній жовчній протоці припинилося надходження жовчі у кишечник. Порушення якого з процесів спостерігається при цьому?

Перетравлення вуглеводів

Перетравлення жирів

Всмоктування вуглеводів

Всмоктування білків

Перетравлення білків

763 / 6854
У пацієнта 48 років після сильного пси- хоемоційного навантаження раптово з'явився гострий біль у ділянці серця з іррадіацією у ліву руку. Нітрогліцерин зняв напад болю через 10 хвилин. Який патогенетичний механізм є провідним у розвитку болю у даному випадку?

Підвищення потреби міокарда у кисні

Стиснення коронарних судин

Спазм коронарних судин

Закупорка коронарних судин

Розширення периферичних судин

764 / 6854
Під час розтину тіла померлого виявлено, що печінка збільшена у розмірах, щільна, краї заокруглені, на розрізі тканина жовтувато-коричневого кольору з темно-червоними крапками та смужками, що нагадує малюнок мускатного горіху. Який патологічний процес лежить в основі таких змін печінки?

Артеріальне повнокров'я

Гостре венозне повнокров'я

Хронічна кровотеча

Артеріальне недокрів'я

Хронічне венозне повнокров'я

765 / 6854
У здорової людини визначили величину енерговитрат. У якому стані знаходилася людина, якщо її енерговитрати виявилися меншими за основний обмін?

Спокій

Легка робота

Сон

Відпочинок

766 / 6854
У чоловіка 58 років клінічна картина гострого панкреатиту. Підвищення в сечі якої з нижчеперерахованих речовин буде підтвердженням діагнозу?

Альбумін

Залишковий азот

Амілаза

Сечова кислота

Сечовина

767 / 6854
Під час інтраопераційної біопсії молочної залози виявлено ознаки тканинного атипізму, що виражається у порушенні співвідношення паренхіми і строми з переважанням останньої, різних розмірів і форми залозисті структури, вистелені одношаровим проліферуючим епітелієм. Який найбільш імовірний діагноз?

Неінфільтруючий рак

Фіброаденома

Папілома

Інфільтруючий рак

Мастит

768 / 6854
До інфекційної лікарні надійшов пацієнт з клінічними ознаками енцефаліту. В анамнезі - укус кліща. Під час постановки реакції затримки гемаглютинації вияв- лено антитіла проти збудника кліщового енцефаліту в розведенні 1:20, що не є діагностичним. Які наступні дії лікаря після одержання вказаного результату?

Дослідити цю ж сироватку повторно

Повторити дослідження із сироваткою, взятою через 10 днів

Повторити дослідження з іншим діагностикумом

Відхилити діагноз кліщового енцефаліту

Використати чутливішу реакцію

769 / 6854
У 40-річної вагітної проведено амніоцентез. Під час дослідження каріотипу плоду одержано результат: 47XY+21. Яку патологію плода виявлено?

Синдром Дауна

Хвороба Патау

Фенілкетонурія

Хвороба Шерешевського-Тернера

Синдром Клайнфельтера

770 / 6854
До травмпункту звернувся чоловік 45 років після побутової травми плеча. Об'єктивно: відсутні функції розгинання, приведення та пронації плеча. Пошкодження якого м'яза викликало такий стан?

Надостного

Великого круглого

Підосного

Малого круглого

Підлопаткового

771 / 6854
У хворого 43 років спостерігається хронічний атрофічний гастрит, мегалобластна гіперхромна анемія. Підвищено виділення метилмалонової кислоти з сечею. Недостатністю якого вітаміну обумовлене виникнення зазначеного симптомокомплексу?

Вітамін В2

Вітамін В5

Вітамін В12

Вітамін В1

Вітамін В3

772 / 6854
Під час обстеження у іноземного громадянина виявили кишковий шистосомоз. Яким шляхом він міг заразитися?

При вживання в їжу риби

При вживанні в їжу м'яса

Через брудні руки

Через укуси комарів

Під час купання в річці

773 / 6854
Після вживання ацетилсаліцилової кислоти у пацієнта з'явився біль у ділянці шлунка як наслідок загострення виразкової хвороби. Що лежить в основі ульцерогенності вказаного препарату?

Імунодепресивний ефект

Спазм судин

Антипростагландинова дія

Стимуляція виділення пепсину

Жовчогінна дія

774 / 6854
До лікаря звернулася жінка зі скаргами на те, що у весняний період у неї з'являється нежить, сиплість голосу, почервоніння повік зі сльозотечею. Який тип алергічної реакції за Джелом і Кумбсом розвивається у цьому випадку?

Імунокомплексний

Стимулюючий

Цитотоксичний

Анафілактичний

Гіперчутливості сповільненого типу

775 / 6854
У чоловіка 39 років спостерігається підвищений ризик розвитку інфекційних процесів, гіперкератоз, порушення присмеркового зору. Який вітамінний препарат необхідно призначити?

Ретинолу ацетат

Токоферолу ацетат

Піридоксину гідрохлорид

Рибофлавін

Ергокальциферол

776 / 6854
У жінки, яка тривалий час знаходилася на дієті з використанням очищеного рису, виявлено поліневрит (хвороба бері-бері). Відсутність якого вітаміну в раціоні призводить до розвитку цього захворювання?

Тіамін

Аскорбінова кислота

Піридоксаль

Рибофлавін

Фолієва кислота

777 / 6854
При повному (з водою) аліментарному голодуванні розвинулись генералізовані набряки. Який з патогенетичних факторів у цьому випадку є провідним?

Підвищення осмотичного тиску міжклітинної рідини

Зниження онкотичного тиску плазми крові

Зниження осмотичного тиску плазми крові

Підвищення онкотичного тиску міжклітинної рідини

Зниження гідростатичного тиску міжклітинної рідини

778 / 6854
Куди треба провести катетер для забору лімфи з грудної лімфатичної протоки?

У ліву пахову вену

У верхню порожнисту вену

У лівий венозний кут

У правий венозний кут

У нижню порожнисту вену

779 / 6854
До лікаря звернувся хворий із скаргами на постійну спрагу. Виявлена гіперглікемія, поліурія та підвищений вміст 17кетостероїдів у сечі. Яке захворювання найбільш імовірне?

Аддісонова хвороба

Інсулінозалежний діабет

Мікседема

Стероїдний діабет

Глікогеноз І типу

780 / 6854
Смерть хворого 16 років настала від розлитого (поширеного) фібринозногнійного перитоніту. Під час розтину в нижній ділянці тонкої кишки виявлена виразка, яка повторювала форму пейєрової бляшки, з перфорацією стінки кишки. Мікроскопічне дослідження виявило невизначеність малюнку лімфоїдної тканини, витіснення її проліферуючими моноцитами, які формують гранульоми. Ускладнення якого захворювання є причиною смерті?

Холера

Неспецифічний виразковий коліт

Черевний тиф

Бруцельоз

Дизентерія

781 / 6854
Чоловік 42 років надійшов до кардіологічного відділення з діагнозом стенокардія. До комплексу препаратів, призначених хворому, включений інгібітор ферменту фосфодіестерази. Концентрація якої речовини в серцевому м'язі буде збільшуватись?

ГМФ

Цикло-АМФ

АМФ

АТФ

АДФ

782 / 6854
У хворого спостерігається гемералопія (куряча сліпота). Яка з перелічених речовин матиме лікувальну дію?

Кератин

Карнітин

Карнозин

Каротин

Креатин

783 / 6854
Хворому на аскаридоз призначили лікарський засіб, який має також вплив на імунну систему і використовується як імуномодулятор. Який це препарат?

Нафтамон

Левамізол

Пірантел В. Фенасал

Піперазину адипінат

784 / 6854
У хворого інфаркт міокарда в ділянці передньої стінки лівого шлуночка. В басейні якої артерії виникло порушення кровообігу?

Ліва крайова гілки лівої вінцевої артерії

Передня міжшлуночкова гілка лівої вінцевої артерії

Огинаюча гілка лівої вінцевої артерії

Передні шлуночкові гілки правої вінце- вої артерії

Передсердно-шлуночкова гілка лівої вінцевої артерії

785 / 6854
У хворого напади бронхіальної астми виникають звичайно вночі, супроводжуються брадикардією, спастичними болями у кишечнику, проносом. Препарати якої групи можуть усунути ці симптоми?

β-адреноблокатори

Салуретики

М-холіноблокатори

Н-холіноблокатори

Симпатолітики

786 / 6854
Під час аналізу електрокардіограми встановлено, що тривалість серцевого циклу у людини дорівнює 1 сек. Якою є частота серцевих скорочень за хвилину?

50

70

100

80

60

787 / 6854
У хворого на хронічну серцеву недостатність у процесі лікування препаратами наперстянки виникли симптоми, що свідчать про початок токсичної дії серцевих глікозидів. Який препарат необхідно призначити для зменшення негативної дії серцевих глікозидів?

Етимізол

Дипіроксим

Атропіну сульфат

Кофеїн-натрія бензоат

Калію хлорид

788 / 6854
Хворого госпіталізовано зі скаргами на блювання, запаморочення, двоїння в очах, утруднене ковтання. Лікар запідозрив ботулізм. Які методи діагностики доцільно використати для підтвердження діагнозу?

Бактеріологічний, мікологічний

Біологічну пробу, бактеріологічний

Алергічну пробу, серологічний

Протозоологічний, мікроскопічний

789 / 6854
Під час розтину тіла чоловіка, який багато років працював на шахті і помер від хронічної легенево-серцевої недостатності, виявлено, що легені малоповітряні, значно ущільнені, склерозовані, верхівки емфізематозно змінені, поверхня сірочорного кольору, на розрізі тканина легенів аспідно-чорного кольору. Яке захворювання стало причиною смерті цього чоловіка?

Силікоз

Талькоз

Алюміноз

Антракоз

Азбестоз

790 / 6854
У чоловіка після гіпертонічного кризу відзначається відсутність довільних рухів у правих руці і нозі, тонус м'язів у цих кінцівках підвищений. Який вид порушення функції нервової системи спостерігається у даному випадку?

Центральний параліч

Рефлекторний парез

Периферичний парез

Центральний парез

Периферичний параліч

791 / 6854
Під час патронажу лікар виявив у дитини симетричну шорсткість щік, діарею, порушення нервової діяльності. Нестача яких харчових факторів є причиною такого стану?

Лізин, аскорбінова кислота

Фенілаланін, пангамова кислота

Метіонін, ліпоєва кислота

Треонін, пантотенова кислота

Нікотинова кислота, триптофан

792 / 6854
Під час мікроскопічного дослідження змиву з рани хворого з гострим рановим процесом гомілки виявлено велику кількість клітин неправильної витягнутої форми, з щільним ядром, у базофільній цитоплазмі яких міститься багато лізосом, фагосом, піноцитозних пухирців. Які клітини виявлено в рані?

Тканинні базофіли

Фіброцити

Плазмоциди

Макрофаги сполучної тканини

Фібробласти

793 / 6854
Чоловік 40 років госпіталізований до хірургічного відділення з діагнозом розрив селезінки. У якому анатомічному утворі буде накопичуватися кров?

У чепцевій сумці

У печінковій сумці

У прямокишково-міхуровій заглибині

У правому бічному каналі

У передшлунковій сумці

794 / 6854
На електронній мікрофотографії фрагменту нирки представлена приносна артеріола, у якій під ендотелієм видно великі клітини, які містять секреторні гранули. Назвіть цей вид клітин:

Гладеньком'язові

Юкставаскулярні

Мезангіальні

Юкстагломерулярні

Інтерстиціальні

795 / 6854
Хворий взимку впав в ополонку, замерз на вітрі, захворів. Температура тіла підвищилася до 39,7°С і коливалася від 39,0°С до 39,8°С. Назвіть тип температурної кривої у хворого.

Febris recurrens

Febris hectica

Febris intermittens

Febris remittens

Febris continua

796 / 6854
У жінки, що хворіє на міастенію, виникли розлади дихання, що вимагало застосування штучної вентиляції легень. Який вид дихальної недостатності розвинувся у даної хворої?

Рестриктивний

Торакодіафрагмальний

Центрогенний

Нервово-м'язовий

Обструктивний

797 / 6854
Піддослідному собаці через зонд у порожнину шлунка ввели 150 мл м'ясного бульйону. Вміст якої з приведених речовин швидко збільшиться в крові тварини?

Інсулін

Нейротензин

Вазоінтестінальний поліпептид

Соматостатин

Гастрин

798 / 6854
У хворого, що тривалий час палить тютюн, розвинувся рак легені. Які з перерахованих канцерогенних речовин містяться в тютюновому диму і відносяться до ПАВ (поліненасичених ароматичних вуглеводів)?

Діетилнітрозамін

β-нафтиламін

Диметиламіноазобензол

Бензпірен

Ортоаміноазотолуол

799 / 6854
У хворого 56 років з серцевою недостатністю спостерігається набряк стоп та гомілок, шкіра в місці набряку бліда і холодна. Яка провідна ланка патогенезу набряку у цього хворого?

Зменшення онкотичного тиску в капілярах

Порушення лімфовідтоку

Позитивний водний баланс

Підвищення проникливості капілярів

Підвищення гідростатичного тиску в венулах

800 / 6854
При спалаху гострої респіраторної інфекції з метою встановлення діагнозу грип проводиться експрес-діагностика, яка грунтується на виявленні специфічного вірусного антигену в досліджуваному матеріалі (змив носоглотки). Яку серологічну реакцію використовують для цього?

Реакція преципітації

Реакція опсонізації

Реакція аглютинації

Реакція імунофлюоресценції

Реакція зв’язування комплементу

801 / 6854
До хірургічного відділення надійшов чоловік 35 років з гнійною раною шиї попереду трахеї (у ділянці передвісцерального простору). Куди може поширитися інфекція, якщо терміново не буде проведене оперативне втручання?

У заднє середостіння

У середнє середостіння

У ретровісцеральний простір

У переднє середостіння

У надгруднинний міжапоневротичний простір

802 / 6854
У хворого з гіпохромною анемією січеться та випадає волосся, відзначається підвищена ламкість нігтів, порушений смак. Якою є причина розвитку зазначених симптомів?

Дефіцит вітаміну В12

Дефіцит залізовмісних ферментів

Зниження продукції тиреоїдних гормонів

Зниження продукції паратирину

Дефіцит вітаміну А

803 / 6854
До медико-генетичної консультації звернулася жінка з приводу ризику захворювання на гемофілію у свого сина. Її чоловік страждає на це захворювання від народження. Жінка та її родичі не страждали на це захворювання. Визначте вірогідність народження хлопчика з гемофілією у цій сім’ї:

25% хлопчиків будуть хворими

75% хлопчиків будуть хворими

50% хлопчиків будуть хворими

100%

0%

804 / 6854
Послаблення кровопостачання органа зумовлює розвиток гіпоксії, яка активізує функцію фібробластів. Об’єм яких елементів збільшується в цій ситуації?

Судини мікроциркуляторного русла

Міжклітинна речовина

Паренхіматозні елементи органа

Лімфатичні судини

Нервові елементи

805 / 6854
При мікроскопії зіскобу з періанальних складок виявлені безбарвні яйця, що мають форму несиметричних овалів, розміром 50x23 мкм. Про який вид гельмінту йде мова?

Аскарида

Карликовий ціп’як

Волосоголовець

Гострик

Кривоголовка

806 / 6854
У здорового обстежуваного в стані спокою кількість еритроцитів становить 5,65*1012/л. Причиною цього може бути те, що обстежуваний:

Студент

Вагітна жінка

Відповідальний працівник міністерства

Шахтар

Мешканець високогір’я

807 / 6854
До лікаря звернулася мати, син якої за літо виріс на 18 см. При обстеженні хлопця 12 років: зріст - 180 см, вага 68 кг. З гіперфункцією якої ендокринної залози це може бути пов’язано?

Щитоподібна

Гіпофіз

Епіфіз

Наднирник

Підшлункова

808 / 6854
Хворий не розуміє змісту слів, а також не розуміє власної мови (словесна глухота). Яка із структур кори великих півкуль головного мозку уражена?

Задня центральна звивина

Верхня скронева звивина

Верхня тім’яна часточка

Нижня лобова звивина

Нижня тім’яна часточка

809 / 6854
Територію старого худобомогильника, який не використовувався більше 50 років, планується відвести під житлове будівництво. Однак дослідження грунту виявило наявність життєздатних спор збудника особливо небезпечного захворювання. Який із вказаних мікроорганізмів найбільш вірогідно міг зберігатися у грунті протягом такого тривалого часу?

Yerarna pestis

Frantisella tularensis

Brucella abortus

Mycobacterium bovis

Batillus anthratis

810 / 6854
Молода людина під час активного підтягування на перекладині відчула різкий біль у спині. Об’єктивно: біль при спробах руху верхньою кінцівкою, обмеження функцій приведення та пронації. Розтягнення якого м’яза, найбільш вірогідно, відбулося?

М.latissimus dorsi

М.levator scapulae

М.subscapularis

М.romboideus major

M.trapezrns

811 / 6854
Людина хворіє на цукровий дiабет, що супроводжується гіперглікемією натще понад 7,2 ммоль/л. Рівень якого білка плазми крові дозволяє ретроспективно (за попередні 4-8 тижні до обстеження) оцінити рівень глікемії?

Церулоплазмін

Фібриноген

Альбумін

С-реактивний білок

Ілікозильований гемоглобін

812 / 6854
У хворого із значними периферічними набряками почергове застосування дихлотіазиду, етакринової кислоти та фуросеміду не викликало значного діуретичного ефекту. У крові значне підвищення кількості альдостерону. Вкажіть препарат вибору:

Клопамід

Амілорид

Маніт

Спіронолактон

Сечовина

813 / 6854
Під час лапаротомії хірург виявив гангренозне ураження низхідної ободової кишки. Тромбоз якої артерії зумовив цей стан?

Ліва ободова

Права ободова

Клубово-ободова

Серединна ободова

Верхня брижова

814 / 6854
Під час розтину тіла померлої в комі молодої людини, виявлено: поширений тромбоемболічний інфаркт лівої півкулі мозку, велика септична селезінка, імунокомплексний гломерулонефрит, виразки на стулках аортального клапану, що прикриті поліпоподібними тромбами з колоніями стафілококів. Яке захворювання викликало церебральну тромбоемболію?

Гострий ревматичний вальвуліт

Септицемія

Ревматичний тромбендокардит

Септичний бактеріальний ендокардит

Септикопіємія

815 / 6854
Внаслідок дії електричного струму на збудливу клітину виникла деполяризація її мембрани. Рух яких іонів через мембрану є причиною деполяризації?

K+

НСО

Na+

Cl-

Са2+

816 / 6854
Постраждалому з колотою раною передньої стінки шлунка надається хірургічна допомога. До якого утворення порожнини очеревини потрапив вміст шлунка?

Правий мезентеріальний синус

Передшлункова сумка

Печінкова сумка

Сальникова сумка

Лівий мезентеріальний синус

817 / 6854
Вагітній жінці, що мала в анамнезі декілька викиднів, призначена терапія, яка містить вітамінні препарати. Укажіть вітамін, який сприяє виношуванню вагітності:

Рутін

Цианкобаламін

Піридоксальфосфат

Фолієва кислота

α-токоферол

818 / 6854
У хворого з аневризмою правої підключичної артерії спостерігається осиплість голосу. З подразненням якого нерву це може бути пов’язане?

N.laringeus superior smster

N.laringeus superior dexter

N.laringeus reccurens dexter

N.laringeus reccurens srnlster

N.laringeus inferior smster

819 / 6854
До приймального відділення доставлений хворий зі скаргами на сухість у роті, світлобоязнь та порушення зору. Шкіра гіперемована, суха, зіниці розширені, тахікардія. При подальшому обстеженні був встановлений діагноз: отруєння алкалоїдами красавки. Який із лікарських засобів доцільно застосувати?

Прозерин

Армін

Дипіроксим

Діазепам

Пілокарпін

820 / 6854
У пацієнта після травми виникли паралічі, розлади дотикової чутливості з правого боку; зліва паралічі відсутні, але порушена больова та температурна чутливість. Яка причина цього явища?

Пошкодження середнього мозку

Однобічне пошкодження спинного мозку з правого боку

Пошкодження мозочка

Пошкодження стовбура мозку

Пошкодження рухової зони кори головного мозку

821 / 6854
Оглядаючи черевну порожнину потерпілого з проникним пораненням передньої черевної стінки, хірург помітив пошкодження малого сальника, з якого витікала яскраво-червона (артеріальна) кров. Яка артерія виявилася пошкодженою?

A.gastroepiploica dextra

A.lienalis

A.hepatica propria

A.gastroepiploica sinistra

A.hepatica communis

822 / 6854
У пологовому будинку народилася дитина з численними порушеннями як зовнішніх так і внутрішніх органів - серця, нирок, травної системи. Був встановлений попередній діагноз - синдром Дауна. За допомогою якого методу можна підтвердити цей діагноз?

Цитогенетичний

Близнюковий

Популяційно-статистичний

Генеалогічний

Біохімічний

823 / 6854
У процесі метаболізму в організмі людини утворюються активні форми кисню, у тому числі супероксидний аніонрадикал. За допомогою якого ферменту інактивується цей аніон?

Супероксиддисмутаза

Каталаза

Глутатіонпероксидаза

Глутатіонредуктаза

Пероксидаза

824 / 6854
Експериментатору необхідно якнайшвидше виробити умовний рефлекс у собаки. На базі якого безумовного рефлексу доцільно виробляти умовний рефлекс у цьому випадку?

Статевий

Орієнтувальний

Захисний

Травний

Міостатичний

825 / 6854
Студент старанно конспектує лекцію. Якість конспектування значно погіршилася, коли сусіди стали розмовляти. Який вид гальмування в корі головного мозку є причиною цього?

Згасаюче

Диференціювальне

Позамежове

Зовнішнє

Запізніле

826 / 6854
Під час патронажу лікар виявив у дитини симетричну шорсткість щік, діарею, порушення нервової діяльності. Нестача яких харчових факторів є причиною такого стану?

Лізин, аскорбінова кислота

Треонін, пантотенова кислота

Нікотинова кислота, триптофан

Метіонін, ліпоєва кислота

Фенілаланін, пангамова кислота

827 / 6854
Для вивчення локалізації біосинтезу білка в клітинах, миші ввели мічені амінокислоти аланін та триптофан. Біля яких органел буде спостерігатися накопичення мічених амінокислот?

Лізосоми

Апарат Гольджі

Клітинний центр

Гладенька ЕПС

Рибосоми

828 / 6854
У хворих на колагеноз має місце процес деструкції сполучної тканини. Це підтверджується збільшенням у крові:

Активності трансаміназ

Вмісту уратів

Вмісту креатину та креатиніну

Вмісту оксипроліну та оксилізину

Активності ізоферментів ЛДГ

829 / 6854
У 6-місячної дитини спостерігалися часті та інтенсивні підшкірні крововиливи. Призначення синтетичного аналога вітаміну K (вікасолу) дало позитивний ефект. У укарбоксилюванні глутамінової кислоти якого білка зсідальної системи крові бере участь цей вітамін?

Фібриноген

Протромбін

Антигемофільний глобулін A

Фактор Хагемана

Фактор Розенталя

830 / 6854
У хворого 60 років виявлено розширення вен стравоходу, прямої кишки та підшкірних вен передньої черевної стінки. Система якої вени ушкоджена?

Непарна

Ворітна

Нижня порожниста

Верхня брижова

Верхня порожниста

831 / 6854
У хворого лікар діагностував гостру гонорею. З анамнезу відомо, що раніше він переніс гонорею та вилікування було повним. До якої категорії інфекцій можна віднести це нове захворювання?

Вторинна інфекція

Реінфекція

Рецидив

Суперінфекція

Аутоінфекція

832 / 6854
У людини зменшений діурез, гіпернатріємія, гіпокаліємія. Гіперсекреція якого гормону може бути причиною таких змін?

Передсердний натрійуретичний фактор

Альдостерон

Вазопресин

Паратгормон

Адреналін

833 / 6854
Під час вивчення фаз мітотичного циклу корінця цибулі знайдено клітину, в якій хромосоми лежать в екваторіальній площині, утворюючи зірку. На якій стадії мітозу перебуває клітина?

Інтерфаза

Профаза

Анафаза

Телофаза

Метафаза

834 / 6854
У хворого напад тахікардії. Які мембранні циторецептори кардіоміоцитів доцільно заблокувати, щоб припинити напад?

М- та Н-холінорецептори

в-адренорецептори

Н-холінорецептори

М-холінорецептори

а-адренорецептори

835 / 6854
У крові хворого концентрація альбумінів складає 2,8 г/л, підвищена концентрація лактатдегідрогенази 5 (ЛДГ5). Про захворювання якого органа це свідчить?

Печінка

Серце

Селезінка

Легеня

Нирка

836 / 6854
Під час розтину тіла мертвонародженої дитини виявлено аномалію розвитку серця: шлуночки не розмежовані, з правої частини виходить суцільний артеріальний стовбур. Для якого класу хребетних тварин характерна подібна будова серця?

Рептилії

Амфібії

Птахи

Риби

Ссавці

837 / 6854
У чоловіка 59 років спострерігаються ознаки паренхіматозної жовтяниці та портальної гіпертензії. Під час гістологічного дослідження пункційного біоптату печінки знайдено: балково-часточкова будова порушена, частина гепатоцитів має ознаки жирової дистрофії, утворюються порто-портальні сполучнотканинні септи з формуванням псевдочасточок, з наявністю пері-портальних лімфомакрофагальних інфільтратів. Який найбільш вірогідний діагноз?

Алкогольний гепатит

Вірусний гепатит

Цироз печінки

Хронічний гепатоз

Токсична дистрофія

838 / 6854
До гастроентерологічного відділення надійшов хворий із запаленням жовчних шляхів. У порціях жовчі виявлено рухомі найпростіші грушоподібної форми, двоядерні, з опорним стрижнем - аксостилем. Яке протозойне захворювання діагностується у хворого?

Балантидіаз кишковий

Трихомоноз

Лямбліоз

Амебіаз кишковий

Амебна дизєнтєрія

839 / 6854
У хворого 30 років з рiзаною раною передпліччя виникло порушення розгинання пальців кисті. Про пошкодження якого нерва це свідчить?

Серединний

М’язовошкірний

Променевий

Ліктьовий

Присередній шкірний нерв передпліччя

840 / 6854
Під час обробки атипових кардіо-міоцитів синоатріального вузла біологічно активною речовиною, зареєстровано збільшення їх мембранного потенціалу через збільшену проникність для іонів калію. Яка біологічно активна речовина впливала на кардіоміоцити?

Норадреналін

Адреналін

Ацетилхолін

Тироксин

Атріопептид

841 / 6854
Під час експерименту подразнюють скелетний м’яз серією електричних імпульсів. Який вид м’язового скорочення буде виникати, якщо кожний наступний імпульс надходить у періоді вкорочення попереднього поодинокого м’язового скорочення?

Контрактура м’яза

Асинхронний тетанус

Суцільний тетанус

Серія поодиноких скорочень

Зубчастий тетанус

842 / 6854
У хворого через декілька годин після опіку в ділянці гіперемії та набряку шкіри з’явилося вогнище некрозу. Який головний механізм забезпечує посилення руйнівних явищ в осередку запалення?

Проліферація фібробластів

Вторинна альтерація

Еміграція лімфоцитів

Діапедез еритроцитів

Первинна альтерація

843 / 6854
У чоловіка 42 років, який страждає на подагру, в крові підвищена концентрація сечової кислоти. Для зниження рівня сечової кислоти йому призначено алопуринол. Конкурентним інгібітором якого ферменту є алопуринол?

Гуаніндезаміназа

Аденінфосфорибозилтрансфераза

Ксантиноксидаза

Аденозиндезаміназа

Гіпоксантинфосфорибозилтрансфераза

844 / 6854
Хлопчик 13 років скаржиться на загальну слабкість, запаморочення, втомлюваність. Спостерігається відставання у розумовому розвитку. При обстеженні виявлено високу концентрацію валіну, ізолейцину, лейцину в крові та сечі. Сеча специфічного запаху. Який найбільш вірогідний діагноз?

Базедова хвороба

Гістидинемія

Тирозиноз

Хвороба 'кленового сиропу'

Хвороба Аддісона

845 / 6854
Хворий звернувся із скаргами на загальну слабкість, головний біль, нудоту, блювання, рідкі випорожнення з домішками слизу та крові. При мікроскопії дуоденального вмісту та при дослідженні свіжих фекалій виявлено рухомі личинки. Який найбільш вірогідний діагноз?

Дракункульоз

Стронгілоїдоз

Трихоцефальоз

Ентеробіоз

Анкілостомідоз

846 / 6854
Біля інфікованої рани збільшилися регіонарні лімфовузли. Під час гістологічного дослідження в них виявлено збільшення кількості макрофагів, лімфоцитів та лімфатичних фолікулів у кірковому шарі, а також велику кількість плазматичних клітин. Який процес відображують виявлені у лімфатичних вузлах гістологічні зміни?

Набута недостатність лімфоїдної тканини

Природжена недостатність лімфоїдної тканини

Антигенна стимуляція

Реакція гіперчутливості

Пухлинна трансформація

847 / 6854
Жінка 62 років скаржиться на частий біль у ділянці грудної клітки та хребта, переломи ребер. Лікар припустив мієломну хворобу (плазмоцитому). Який з перерахованих нижче лабораторних показників буде мати найбільше діагностичне значення?

Гiперальбумiнемiя

Гіпопротеїнемія

Протєїнурія

Парапротеїнемiя

Гіпогло6улінємія

848 / 6854
У гістопрепараті представлено паренхіматозний орган, що має кіркову та мозкову речовину. Кіркова речовина утворена тяжами епітеліоцитів, між якими проходять кровоносні капіляри; тяжі формують три зони. Мозкова речовина складається з хромафіноцитів та венозних синусоїдів. Який орган має зазначені морфологічні ознаки?

Тимус

Нирка

Щитовидна залоза

Лімфатичний вузол

Наднирник

849 / 6854
Хворий надійшов зі скаргами на диспептичні розлади, мелену, гемороїдальні кровотечі. Під час дослідження виявлено розширення сітки венозних судин на передній стінці живота, збільшення його розмірів. Яка патологія може мати такі симптоми?

Виразкова хвороба

Ентерит

Коліт

Кишкова аутоінтоксикація

Портальна гіпертензія

850 / 6854
Хворому, що страждає на склеродермію (колагенове захворювання), було призначено імунодепресант, який відноситься до групи протималярійних засобів, похідних хіноліну. Хворий приймав препарат тривало. Внаслідок цього у нього знизилася вага, посивіло волосся, зменшилася гострота зору, з’явилося миготіння перед очима. Було призначено обстеження у окуліста, який відмінив препарат. Визначте, яким препаратом лікували хворого:

Кризанол

Метотрексат

Хінгамін

Хіноцид

Циклофосфан

851 / 6854
У загальному вигляді генетичний апарат еукаріот є таким: ек-зон-інтрон-екзон. Така структурно-функціональна організація гена зумовлює особливості транскрипції. Якою буде про-і-РНК відповідно до згаданої схеми?

Інтрон-екзон

Екзон-інтрон

Екзон-екзон-інтрон

Екзон-інтрон-екзон

Екзон-екзон

852 / 6854
хворого виявлено підвищення активності ЛДГ1 ЛДГ2 52. , АсАТ, креатинкінази. В якому органі хворого найбільш вірогідний розвиток патологічного процесу?

Серце

Скелетні м’язи

Нирки

Печінка

Підшлункова залоза

853 / 6854
Споживання пацієнтом протягом тривалого часу забруднених овочів та фруктів призвело до отруєння нітратами. Яке похідне гемоглобіну утворилося у крові даного хворого?

Hb O2

Hb CN

Hb СО

Hb NHCOOH

Hb-OH

854 / 6854
3 роки тому хворій 34 років було встановлено діагноз хронічного гломерулонефриту. За останні 6 місяців з’явилися набряки. Що лежить в основі їх розвитку?

Гіперосмолярність плазми

Протеїнурія

Порушення білковоутворюючої функції печінки

Гіперальдостеронізм

Гіперпродукція вазопресину

855 / 6854
Під час роботи щодо ліквідації наслідків аварії на АЕС, робітник одержав дозу опромінення 500 рентген. Скаржиться на головний біль, нудоту, запаморочення. Які зміни кількості лейкоцитів можна очікувати у хворого через 10 годин після опромінення?

Лейкопенія

Агранулоцитоз

Лімфоцитоз

Лейкемія

Нейтрофільний лейкоцитоз

856 / 6854
З хімічного виробництва до токсикологічного відділення доставлений хворий з отруєнням парами ртуті. Який препарат слід використати в даній ситуації?

Ентеросорбент СКН

Активоване вугілля

Унітіол

Налоксон

Ізонітрозин

857 / 6854
У раціоні людини велика кількість вуглеводів. Кількість яких структур збільшиться у цитоплазмі гепатоцитів?

Вільні рибосоми

Включення ліпофусцину

Гранули глікогену

Краплини жиру

Лізосоми

858 / 6854
Жінка 63 років має ознаки ревматоїдного артриту. Підвищення рівня якого з перерахованих нижче показників крові буде найбільш значущим для підтвердження діагнозу?

R-глікозидаза

Кисла фосфатаза

Ліпопротеїди

Сумарні глікозаміноглікани

Загальний холестерин

859 / 6854
Під час розтину на зовнішній поверхні аортального клапана, виявлені великі (1-2 см) буровато-червоні, крихкі нашарування, які прикривають виразкові дефекти. Який найбільш вірогідний діагноз?

Дифузний ендокардит

Поліпозно-виразковий ендокардит

Поворотній бородавчастий ендокардит

Гострий бородавчастий ендокардит

Фібропластичний ендокардит

860 / 6854
Після крововиливу в мозок у хворого стали неможливими активні рухи лівих руки та ноги. Тонус м’язів цих кінцівок підвищено, їх спінальні рефлекси різко підсилені, розширено зони рефлексів. Рефлекс Бабінського зліва. Який вид розладу рухів має місце у хворого?

Спінальний шок

Периферичний параліч

В’ялий параліч

Рефлекторний параліч

Центральний параліч

861 / 6854
Під час рентгенологічного дослідження кісток основи черепа виявлено збільшення турецького сідла та деструкція окремих його ділянок. Пухлинне розростання якого анатомічного утворення може спричинити таке руйнування кістки?

Епіфіз

Зоровий горб

Гіпофіз

Чотиригорбове тіло

Колінчасті тіла

862 / 6854
Хворий відзначає часті проноси, особливо після вживанняжирної їжі, схуднення. Лабораторні дослідження показали наявність стеатореї; кал гіпохолічний. Що може бути причиною такого стану?

Обтурація жовчних шляхів

Незбалансована дієта

Запалення слизової оболонки тонкої кишки

Недостатність панкреатичної ліпази

Недостатність панкреатичної фосфо-ліпази

863 / 6854
Під час розтину тіла чоловіка 56 років, який страждав на фіброзно-кавернозний туберкульоз легень, знайдено збільшену у розмірах щільну селезінку. На розрізі тканина її коричнево-рожевого кольору, гладенька, з віскоподібною поверхнею. Який з перелічених патологічних процесів у селезінці найбільш вірогідний?

Цианотична індурація

Глазурна селезінка

Порфірна селезінка

Сальна селезінка

Сагова селезінка

864 / 6854
У хворого з гнійничковими ураженнями шкіри виділений збудник, який на кров’яному агарі утворює жовті колонії округлої форми, середніх розмірів, оточені зоною гемолізу. У мазках з колоній - коки, розташовані скупченнями неправильної форми, грампозитивні. Виділена культура оксидазо- і каталазопозитивна, ферментує маніт, синтезує плазмокоагулазу. Який вид збудника виділений?

Streptococcus agalactiae

Staphylococcus saprophyticus

Staphylococcus epidermidis

Staphylococcus aureus

Streptococcus pyogenes

865 / 6854
До стоматолога звернулася мати зі скаргами на руйнування зубів у дитини 2-х років. Під час огляду молочні зуби деформовані, уражені карієсом, у шийки коричнева облямівка. З анамнезу встановлено, що мати під час вагітності приймала антибіотики без контролю лікаря. Вкажіть, антибютики якої групи, що мають найбільш виражену тератогенну дію, могла приймати мати?

Макроліди

Тетрацикліни

Пєніциліни

Цефалоспорини

Аміноглікозиди

866 / 6854
У хворого на хронічну серцеву недостатність у процесі лікування препаратами наперстянки виникли симптоми, що свідчать про початок токсичної дії серцевих глікозидів. Який препарат необхідно призначити для зменшення негативної дії серцевих глікозидів?

Етімізол

Атропіну сульфат

Кофеїн-бензоат натрію

Діпіроксим

Калію хлорид

867 / 6854
До лікаря звернувся хворий зі скаргами на непереносимість сонячної радіації. Мають місце опіки шкіри та порушення зору. Попередній діагноз: альбінізм. Порушення обміну якої амінокислоти відзначається у цього пацієнта?

Лізин

Аланін

Тирозин

Пролін

Триптофан

868 / 6854
До лікарні швидкої допомоги доставили дитину 7 років у стані алергічного шоку, який розвинувся після того, як її вжалила оса. У крові підвищена концентрація гістаміну. В результаті якої реакції утворився цей амін?

Відновлення

Дегідрування

Декарбоксилювання

Гідрооксилювання

Дезамінування

869 / 6854
Під час огляду хворого лікар запідозрив синдром Іценка-Кушинга. Визначення якої речовини в крові хворого підтвердить припущення лікаря?

Токоферол

Ретинол

Кортизол

Адреналін

Холестерин

870 / 6854
У дитини 6 місяців спостерігається різке відставання в психомоторному розвитку, напади судом, бліда шкіра з екзематозними змінами, біляве волосся, блакитні очі. У цієї дитини найбільш вірогідно дозволить встановити діагноз визначення концентрації у крові та сечі:

Гістидину

Триптофану

Лейцину

Фенілпірувату

Валінду

871 / 6854
Тромбоз коронарної артерії спричинив розвиток інфаркту міокарда. Які механізми ушкодження кардіоміоцитів є домінуючими при цьому захворюванні?

Кальцієві

Протеїнові

Ацидотичні

Ліпідні

Електролітно-осмотичні

872 / 6854
У крові хворого виявлено низький рівень альбумінів та фібриногену. Зниження активності яких органел гепатоцитів найбільш вірогідно обумовлює це явище?

Комплекс Гольджі

Гранулярна ендоплазматична сітка

Агранулярна ендоплазматична сітка

Лізосоми

Мітохондрії

873 / 6854
Під час обстеження хворого була виявлена недостатня кількість імуноглобулінів. Які з клітин імунної системи їх продукують?

Т-супресори

Т-кілери

Т-хелпери

Плазматичні

Плазмобласти

874 / 6854
Жінка 33 років страждає на гепатоцеребральну дистрофію (хвороба Вільсона). У крові - знижений вміст церулоплазміну. У сечі - різко підвищений вміст амінокислот. Посиленням якого процесу зумовлені ці зміни?

Комплексоутворення амінокислот з міддю

Розпад тканинних 6ілків

Переамінування амінокислот

Ілюконеогенез

Синтез сечовини

875 / 6854
В одному з гірських селищ спостерігалася масова загибель гризунів, що супроводжувалася захворюванням мешканців селища. Для цієї хвороби було притаманне швидке підвищення t0 до 400 С, виражена інтоксикація, збільшення пахвинних лімфовузлів. У препаратах-мазках з трупного матеріалу виявлені грамнегативні палички овоїдної форми з біполярним забарвленням. Які мікроорганізми є збудниками цього інфекційного захворювання?

Збудник сибірки

Клостридії

Збудник туляремії

Стафілокок

Збудник чуми

876 / 6854
До терапевтичного відділення надійшов хворий з тривалою бронхопневмонією. Антибіотикотерапія не дала належного ефекту. Який препарат для підвищення імунного статусу доцільно додати до комплексної терапії цього хворого?

Тималін

Парацетамол

Аналгін

Димедрол

Сульфокамфокаїн

877 / 6854
У хворого на хронічний цистіт у біоптаті слизової оболонки сечового міхура разом з перехідним епітелієм виявлені вогнища багатошарового плоского незроговілого епітелію. Який процес лежить в основі зазначених змін в епітелії?

Дистрофія

Метаплазія

Гіперкератоз

Дисплазія

Гіперплазія

878 / 6854
Який механізм тепловіддачі найбільш ефективно реалізується під час перебування людини при температурі навколишнього середовища +350 С та 80% вологості повітря?

Теплопроведення

Радіація

Конвекція

Випаровування

879 / 6854
У хворого спостерігається підвищення опору вигнанню крові з лівого шлуночка. При якому із перерахованих патологічних процесів може виникнути така ситуація?

Недостатність аортального клапана

Аортальний стеноз

Емболія легеневої артерії

Артеріальна гіпотензія

Мітральний стеноз

880 / 6854
У представників однієї з людських популяцій тіло подовжене, широка варіабельність зросту, знижений об’єм м’язової маси, подовжені кінцівки, зменшена у розмірах і об’ємі грудна клітка, підвищене потовиділення, знижені показники основного обміну та синтезу жирів. До якого адаптивного типу людей відноситься дана популяція?

Тип зони помірного клімату

Арктичний

Гірський

Проміжний

Тропічний

881 / 6854
Під час гістологічного дослідження біопсійного матеріалу шлунка виявили малу кількість чи повну відсутність парієтальних клітин у залозах. Слизову оболонку якої ділянки шлунку вивчали?

Кардіальний відділ

Пілоричний відділ

Дно

Тіло

882 / 6854
До травматологічного пункту доставлено хворого з ушкодженням нижньої кінцівки внаслідок прямого удару по внутрішній поверхні середньої третини гомілки. Перелом якого анатомічного утворення найбільш вірогідний?

Проксимальний епіфіз малогомілкової кістки

Проксимальний епіфіз великогомілкової кістки

Дистальний епіфіз малогомілкової кістки

Дистальний епіфіз великогомілкової кістки

Діафіз великогомілкової кістки

883 / 6854
Тестостерон та його аналоги збільшують масу скелетних м’язів, що дозволяє використовувати їх для лікування дистрофій. Взаємодією з яким клітинним субстратом зумовлена ця дія?

Хроматин

Рибосоми

Білки-активатори транскрипції

Ядерні рецептори

Мембранні рецептори

884 / 6854
У вогнищі запалення підвищується проникність судин мікроциркуляторного русла, у них збільшується гідродинамічний опір. У міжклітинній рідині підвищується осмотична концентрація та дисперсність білкових структур. Який вид набряку буде спостерігатися у даному випадку?

Змішаний

Лімфогенний

Колоїдно-осмотичний

Мембраногенний

Гідродинамічний

885 / 6854
Під час дослідження коронарних артерій виявлені атеросклеротичні бляшки з кальцинозом, що закривають просвіт судин на 1/3. У м’язі дрібні множинні білуваті прошарки сполучної тканини. Як називається процес, виявлений у міокарді?

Міокардит

Інфаркт міокарда

Післяінфарктний кардіосклероз

Дифузний кардіосклероз

Тигрове серце

886 / 6854
У процесі онтогенезу у людини на організменному рівні проявилися наступні зміни: зменшилася життєва ємність легень, збільшився артеріальний тиск, розвинувся атеросклероз. Який період онтогенезу найбільш вірогідний у цьому випадку?

Похилий вік

Підлітковий

Молодий вік

Юнацький

Початок зрілого віку

887 / 6854
Хворий 50 років з метою лікування черевного тифу почав приймати левоміцетин, але на наступний день його стан погіршився, температура підвищилася до 39,60 С. Чим пояснити погіршення стану хворого?

Алергічною реакцією

Дією ендотоксинів збудника

Реінфекцією

Нечутливістю збудника до левоміцетину

Приєднанням вторинної інфекції

888 / 6854
Як зміниться фармакологічна активність препарату з високою спорідненістю до білків плазми крові при виникненні гіпоальбумінемії?

Дещо зменшиться

Суттєво зменшиться

Підвищиться

Не зміниться

Зникне

889 / 6854
У людини в стані спокою значно збільшена робота м’язів видиху. Що з наведеного може бути причиною цього?

Негативний внутрішньоплевральний тиск

Зменшення хвилинного об’єму дихання

Звуження дихальних шляхів

Поверхневе дихання

Рідке дихання

890 / 6854
У місті епідемія грипу. Який препарат доцільно використати для неспецифічної профілактики захворювання?

Протигрипозний імуноглобулін

Пеніцилін

Протигрипозна вакцина

Лейкоцитарний інтерферон

Протигрипозна сироватка

891 / 6854
Під час розтину трупа чоловіка 56 років у термінальному відділі тонкої кишки виявлено декілька виразок діаметром 4-5 см. Краї виразок піднімаються над поверхнею слизової оболонки; стінки вкриті сірувато-жовтуватими масами, які кришаться. Реакція Відаля позитивна. Який найбільш вірогідний діагноз?

Поворотний тиф

Дизентерія

Хвороба Крона

Черевний тиф

Паратиф

892 / 6854
Під час огляду дитини, яка перехворіла на кір, у м’яких тканинах щік та промежини виявлено нечітко відмежовані, набряклі, червоно-чорного кольору ділянки, у яких виявляється помірна флюктуація. Яке ускладнення розвинулося у дитини?

Пролежень

Суха гангрена

Волога гангрена (нома)

Трофічна виразка

Газова гангрена

893 / 6854
У людини вимірють внутрішньо-плевральний тиск. У якій фазі людина затримала дихання, якщо величина тиску дорівнює - 7,5 см вод.ст.?

Спокійний видих

Форсований вдих

Форсований видих

Спокійний вдих

894 / 6854
В умовах експерименту у кролика перев’язали ниркову артерію, що через 2 тижні призвело до суттєвого збільшення артеріального тиску. У результаті збільшення секреції якої біологічно активної речовини це відбулося?

Норадреналін

Ренін

Натрійуретичний гормон

Вазопресин

Адреналін

895 / 6854
Куди треба провести катетер для забору лімфи з грудної лімфатичної протоки?

У нижню порожнисту вену

У правий венозний кут

У верхню порожнисту вену

У лівий венозний кут

У ліву пахвинну вену

896 / 6854
У дитячому садку планується проведення вакцинації проти коклюшу. Який препарат необхідно використати з цією метою?

Типоспецифічна сироватка

Вакцина АКДП

АДП анатоксин

Нормальний у-глобулін

Вакцина БЦЖ

897 / 6854
У студента перед іспитом виникла тахікардія. Які зміни на ЕКГ свідчитимуть про її наявність?

Вкорочення інтервалу P-Q

Подовження сегменту Q-T

Вкорочення інтервалу R-R

Подовження комплексу QRS

Подовження інтервалу R-R

898 / 6854
Перед відрядженням за кордон лікарю, з метою профілактики малярії, призначено засіб з гістомизонтоцидною дією. Який препарат він одержував?

Хінін

Примахін

Доксициклін

Бісептол

Хлоридин

899 / 6854
У хлопчика 5 років, якого непокоїть анальний свербіж, виявлені черв’яки класу нематод (гострики). Оберіть лікарський засіб для дегельмінтації дитини:

Аміноакрихін

Празіквантель

Сім’я гарбуза

Фенасал

Мебендазол

900 / 6854
Після ремонту автомобілю в гаражному приміщенні водій потрапив до лікарні з симптомами отруєння вихлопними газами. Вміст якої речовини у крові буде підвищено?

Карбгемоглобін

Карбоксигемоглобін

Глікозильований гемоглобін

Метгемоглобін

Оксигемоглобін

901 / 6854
У жінки через 6 місяців після пологів розвинулася маткова кровотеча. Під час гінекологічного обстеження у порожнині матки виявлена тканина темно-червоного кольору з множинними порожнинами, що нагадує 'губку'. Під час мікроскопічного дослідження пухлини у лакунах крові виявлені атипові світлі епітеліальні клітини Лангханса та гігантські клітини синцитіотрофобласта. Яка це пухлина?

Аденокарцинома

Міхуровий занос

Плоскоклітинний незроговілий рак

Хоріонепітеліома

Фіброміома

902 / 6854
Хвора 13 років знаходиться на стаціонарному лікуванні в гематологічному відділенні обласної дитячої лікарні з діагнозом залізодефіцитна анемія. Який тип гіпоксії має місце у цієї хворої?

Змішана

Гемічна

Дихальна

Тканинна

Циркуляторна

903 / 6854
Хворому, що страждає на тромбоемболічну хворобу, призначений штучний антикоагулянт пелентан. Антагоністом якого вітаміну є цей препарат?

Вітамін D

Вітамін С

Вітамін А

Вітамін Е

Вітамін К

904 / 6854
У хворої гінекологічного відділення виникли симптоми внутрішньої кровотечі. Який засіб слід призначити з метою пригнічення фібринолізу та зупинки кровотечі?

Контрикал

Дицинон

Хлористий кальцій

Фібриноген

Вікасол

905 / 6854
У хворого з клінічними ознаками первинного імунодефіциту виявлено порушення функції антигенпрезентації імунокомпетентним клітинам. Порушення функціонування яких клітін може бути причиною цього?

Фібробласти

0-лімфоцити

В-лімфоцити

Макрофаги, моноцити

Т-лімфоцити

906 / 6854
У хворого 37 років на фоні тривалого застосування антибіотиків спостерігається підвищена кровоточивість при невеликих пошкодженнях. У крові - зниження активності II, VII, X факторів згортання крові; подовження часу згортання крові. Нестачею якого вітаміну обумовлені ці зміни?

Вітамін D

Вітамін А

Вітамін С

Вітамін Е

Вітамін К

907 / 6854
У хворого на хронічний гломерулонефрит швидкість клубочкової фільтрації (ШКФ) знижена до 20% від нормальної. Що є основною причиною зниження ШКФ у даному випадку?

Ішемія нирок

Тромбоз ниркових артерій

Обтурація сечовивідних шляхів

Зменшення кількості функціонуючих нефронів

Тубулопатія

908 / 6854
5 років температура підвищилася до 40о 108. С, з’явився різкий головний біль, блювання, неспокій, озноб. Через 4 дні з’явилася геморагічна висипка на шкірі, олігоурія та надниркова недостаність, що і стало причиною смерті. При бактеріологічному дослідженні мазків з глотки виявлено менінгокок. Яка форма хвороби виявлена?

Менінгококовий менінгіт

Менінгоенцефаліт

Менінгококовий назофарингіт

Гідроцефалія

Менінгококцемія

909 / 6854
У жінки 37 років під час оперативного втручання на органах малого тазу виникла необхідність перев’язати маткову артерію. Яке з утворень може бути випадково перев’язаним разом з нею?

Внутрішня клубова вена

Маткова труба

Сечовід

Сечівник

Кругла зв’язка матки

910 / 6854
Під час гістологічного дослідження тимуса чоловіка 40 років, визначено зменшення частки паренхіматозних елементів залози, збільшення частки жирової та пухкої сполучної тканини, збагачення її тимусними тільцями при незмінній загальній масі органу. Як зветься таке явище?

Акцідентальна інволюція

Вікова інволюція

Атрофія

Гіпотрофія

Дистрофія

911 / 6854
Підліток 12 років протягом 3 місяців втратив 7 кг маси тіла. Вміст глюкози у крові становить 20 ммоль/л. Несподівано розвинулася кома. Який вид цукрового діабету найбільш вірогідний у хлопчика?

Гіпофізарний

Гіпертіреоїдний

Стероїдний

Інсулінозалежний (I тип)

Інсулінонезалежний (II тип)

912 / 6854
Внаслідок тривалого голодування в організмі людини швидко зникають резерви вуглеводів. Який з процесів метаболізму підтримує при цьому вміст глюкози в крові?

Глюконеогенез

Пентозофосфатний цикл

Глікогеноліз

Анаеробний гліколіз

Аеробний гліколіз

913 / 6854
У чоловіка 55 років, який протягом багатьох років страждає на недостатність мітрального клапана, виникла гостра серцева недостатність. Який патофізіологічний варіант недостатності серця спостерігається у цьому випадку?

Гіпоксичне ушкодження серця

Перевантаження серця об’ємом

Нейрогенне ушкодження серця

Коронарогенне ушкодження серця

Гостра тампонада серця

914 / 6854
У бактеріологічній лабораторії проводиться дослідження м‘ясних консервів на вміст ботулінічного токсину. Для цього дослідній групі мишей ввели екстракт із досліджуваного матеріалу та антитоксичну протиботулінічну сироватку типів А, В, Е; контрольній групі мишей ввели екстракт без протиботулінічної сироватки. Яку серологічну реакцію було використано?

Опсоно-фагоцитарна

Зв‘язування комплементу

Нейтралізації

Подвійної імунної дифузії

Преципітації

915 / 6854
Хворому, що страждає на хронічну серцеву недостатність, лікар порадив провести профілактичний курс лікування кардіотонічним препаратом з групи серцевих глікозидів, який приймають ентерально. Який препарат було рекомендовано хворому?

Корглікон

Дигоксин

Кордарон

Кордіамін

Строфантин

916 / 6854
Офтальмолог з діагностичною метою (розширення зіниць для огляду очного дна) використав 1 % розчин мезатону. Чим обумовлений мідріаз, викликаний препаратом?

Активація α1 адренорецепторів

Блокада α1 адренорецепторів

Активація α2 адренорецепторів

Активація М-холінорецепторів

Активація β1 адренорецепторів

917 / 6854
Чоловіка 45 років протягом останніх 3 років непокоїв сухий кашель; наростала задишка, легенева недостатність, швидко втрачалася вага. На розтині: легеневе серце; у легенях різко виражений фіброз з наявністю порожнин, що створюють картину 'медових сот'. Гістологічно: інтерстиційний фіброз з вираженою інфільтрацією строми лімфогістіоцитами з домішками нейтрофілів. Який найбільш вірогідний діагноз?

Пиловий пневмосклероз

Бронхоектатична хвороба

Післязапальний пневмосклероз

Хронічна бульозна емфізема

Фіброзуючий альвеоліт

918 / 6854
Під час огляду дитини педіатр відзначив відставання у фізичному та розумовому розвитку. У сечі різко підвищений вміст кетокислоти, що дає якісну кольорову реакцію з хлорним залізом. Яке порушення обміну речовин було виявлено?

Фенілкетонурія

Альбінізм

Тирозинемія

Алкаптонурія

Цистинурія

919 / 6854
Хвора 40 років надійшла до інфекційного відділення лікарні з високою температурою тіла. Об’єктивно: виражені менінгеальні симптоми. Проведено спиномозкову пункцію. Яке анатомічне утворення було пропунктовано?

Spatium subdurale

Spatium epidurale

Cavum trigeminale

Spatium subarachnoideum

Cisterna cerebellomedullaris posterior

920 / 6854
Тварині внутрішньовенно ввели концентрований розчин хлориду натрію, що зумовило зниження реабсорбції іонів натрію у канальцях нирок. Внаслідок яких змін секреції гормонів це відбувається?

Зменшення натрійуретичного фактора

Збільшення вазопресину

Зменшення вазопресину

Зменшення альдостерону

Збільшення альдостерону

921 / 6854
У холодну погоду з вітром люди замерзають швидше, ніж за відсутності вітру. Причиною цього є те, що вітер, вперш за все, збільшує віддачу тепла шляхом:

Випаровування

Конвекції

Радіації

Теплопроведення

922 / 6854
Під час бігу на довгі дистанції скелетна мускулатура тренованої людини використовує глюкозу з метою отримання енергії АТФ для м’язового скорочення. Вкажіть основний процес утилізації глюкози в цих умовах:

Ілюконеогенез

Аеробний гліколіз

Анаеробний гліколіз

Ілікогеноліз

Ілікогенез

923 / 6854
Дитина 10 років під час гри порізала ногу відламком скла та була направлена до поліклініки для введення протиправцевої сироватки. З метою попередження розвитку анафілактичного шоку лікувальну сироватку вводили за Безредкою. Який механізм лежить в основі подібного способу гіпосенсибілізації організму?

Зв’язування фіксованих на тучних клітинах IgE

Стимуляція імунологічної толерантності до антигену

Блокування синтезу медіаторів тучних клітин

Зв’язування рецепторів до IgE на тучних клітинах

Стимуляція синтезу антиген-специфічних IgG2

924 / 6854
У хворого через добу після апендектомії у крові визначається нейтрофільний лейкоцитоз із регенеративним зсувом. Який найбільш вірогідний механізм розвитку лейкоцитозу у даному випадку?

Уповільнення руйнування лейкоцитів

Посилення лейкопоезу

Посилення лейкопоезу та уповільнення еміграції лейкоцитів у тканини

Уповільнення еміграції лейкоцитів у тканини

Перерозподіл лейкоцитів у організмі

925 / 6854
У хворого з широким інфарктом міокарда розвинулася серцева недостатність. Який патогенетичний механізм її розвитку?

Реперфузійне ураження міокарда

Перевантаження серця тиском

Перевантаження серця об’ємом

Гостра тампонада серця

Зменшення маси функціонуючих кардіоміоцитів

926 / 6854
Під час визначення групової належності крові за системою АВ0, аглютинацію еритроцитів досліджуваної крові викликали стандартні сироватки I та II груп, та не викликала сироватка III групи. Кров якої групи досліджується?

АВ(IV)

В(III)α

Неможливо визначити

O(I)α та β

А(II)β

927 / 6854
Хворий 50 років скаржиться на спрагу, п’є багато води; виражена поліурія. Глюкоза крові - 4,8 ммоль/л. У сечі глюкози та ацетонових тіл немає, сеча безбарвна, питома вага- 1,002 - 1,004. Яка причина поліурії?

Інсулінова недостатність

Гіпотиреоз

Тиреотоксикоз

Альдостеронізм

Нестача вазопресину

928 / 6854
До ендокринолога звернулася хвора 45 років із скаргами на підвищенний апетит, сухість слизових оболонок ротової порожнини, зростання діурезу. При обстеженні вперше виявлено інсуліннезалежний діабет. Який з названих препаратів доцільно призначити хворій?

Глібенкламід

Вазопресин

Адіурекрин

Окситоцин

Інсулін

929 / 6854
До реанімаційного відділення надійшов хворий із симптомами гострого отруєння морфіном - непритомність, гіпотермія, дихання Чейн-Стокса, гіпотензія, брадикардія, міоз. Який з перерахованих препаратів буде найефективнішим у даній ситуації?

Кофеїн

Налоксон

Етимізол

Камфора

Кордіамін

930 / 6854
До лікаря звернувся хворий із скаргами на біль у лівій половині шиї, який посилюється при рухах голови. Положення при якому біль не турбує - це нахил голови ліворуч з поверненим обличчям праворуч. Ураження якого м’яза спричиняє біль у даному випадку?

M.longus со11і

M.stemocleidomastoideus dexter

M.platisma sister

М.stemocleidomastoideu sinister

M.platisma dexter

931 / 6854
Хвора у віці 69 років страждає хронічними запорами, в основі яких лежить гіпотонія товстої кишки. Який з перерахованих препаратів доцільно призначити?

Натрію сульфат

Прозерин

Магнію сульфат

Бісакоділ

Касторова олія

932 / 6854
Хворий 45 років госпіталізований до лікарні зі скаргами на високу температуру, біль під час дихання, задишку та кашель. Після обстеження, лабораторної та рентгенодіагностики було встановлено діагноз - плеврит. Для евакуації ексудату була призначена плевральна пункція. В якому місці плевральної порожнини буде знаходитися найбільша кількість ексудату?

Діафрагмально-медіастинальний синус

Реберно-медіастинальний синус

Під коренем легенів

Під куполом плеври

Реберно-діафрагмальний синус

933 / 6854
Під час розтину трупа жінки 69 років підвищеного живлення, яка померла від гострого інфаркту міокарда, в інтимі коронарних артерій знайдені численні білуваті, щільні, вибухаючі формування, що різко звужують просвіт судин. Для якої стадії атеросклерозу властиві такі зміни?

Атероматоз

Ліпоїдоз

Атерокальциноз

Стадія утворення атероматозної виразки

Ліпосклероз

934 / 6854
Під час обстеження чоловіка 45 років, який перебуває довгий час на вегетеріанській рослинній дієті, виявлено негативний азотистий баланс. Яка особливість раціону стала причиною цього?

Недостатня кількість жирів

Надлишкова кількість води

Надлишкова кількість вуглеводів

Недостатня кількість білків

Недостатня кількість вітамінів

935 / 6854
Хворій на ревматоїдний поліартрит призначили нестероїдний протизапальний засіб - диклофенак натрію. Через деякий час у хворої виникло загострення супутнього захворювання, що змусило відмінити препарат. Яке супутнє захворювання могло призвести до відміни препарату?

Ішемічна хвороба серця

Цукровий діабет

Гіпертонічна хвороба

Бронхіальна астма

Виразкова хвороба шлунка

936 / 6854
Хворий, що лікувався з приводу неврозу сибазоном, скаржиться на зубний біль. Лікар призначив йому знеболювальний засіб у дозі, яка меньша за середню терапевтичну. Яке явище взяв до уваги лікар, зменшуючи дозу препарату?

Сумація

Потенціювання

Кумуляція

Лікарська залежiсть

Толерантність

937 / 6854
Під час дослідження сироватки крові пацієнта з ознаками імунодефіциту виявлено антитіла до білків gP120 та gP41. Наявність якої інфекції у хворого підтверджує цей результат?

ВІЛ-інфекція

HLTV-1-інфекція

ЕСНО-інфекція

TORCH-інфекція

HBV-інфекція

938 / 6854
На розтині тіла чоловіка 46 років на слизовій оболонці прямої та сигмовидної кишок виявилено множинні коричнево-зелені нашарування, крововиливи; у просвіті кишки слиз, невелика кількість крові; гістологічно - фібринозний коліт. При бактеріологічному дослідженні вмісту кишки висіяно S.Sonne. Який найбільш вірогідний діагноз?

Холера

Ієрсініоз

Хвороба Крона

Дизентерія

Сальмонельоз

939 / 6854
Під час розтину тіла чоловіка, який служив на підводному атомному човні, виявили наступне: спустошення в кістковому мозку (панмієлофтиз), анемія, лейкопенія, тромбоцитопенія, розпад лімфоцитів у лімфатичних вузлах, селезінці, лімфатичному апараті шлунково-кишкового тракту, крововиливи в на-днирники. Який найбільш вірогідний діагноз?

Гостра анемія

Гостра променева хвороба

Вібраційна хвороба

Гострий лейкоз

Кесонна хвороба

940 / 6854
Пацієнтка 58 років скаржиться на підвищену втомлюваність, зниження працездатності, сонливість, задишку під час швидкої ходи. У крові: ер.-4,61012/л, Hb- 92 г/л, кол.показн.- 0,6. У мазку

крові велика кількість анулоцитів та мікроцитів. Для якої анемії це притаманно?

Гемолітична

Залізодефіцитна

Серповидноклітинна

Постгеморагічна

Перніціозна

941 / 6854
Хворий багато років страждав на бронхіальну астму та помер від нападу ядухи. Під час гістологічного дослідження легень виявлено: в просвіті бронхіол та дрібних бронхів багато слизу з домішкою еозинофілів, склероз міжальвеолярних перетинок, розширення просвіту альвеол. Який механізм розвитку реакції гіперчутливості мав місце?

Реагінова реакція

Цитотоксична реакція

Імунокомплексна реакція

Гранулематоз

Цитоліз, обумовлений лімфоцитами

942 / 6854
У хворого 53 років після поранення в ділянку промежини відмічається довільне сечовиділення. Який з м’язів найбільш вірогідно ушкоджений?

M.transversus реrineі superficialis

M.transversus реrіnеі profundus

M.sphyncter uretrae

M.ischiocavemosus

M.bulbospongiosus

943 / 6854
При обстеженні на бактеріоносійство черевного тифу у сироватці крові кухарки шкільної їдальні виявлені Vі-антитіла. Яка з перелічених реакцій була використана у данному випадку?

Реакція Відаля

РЗК

ІФА

РНГА

РІФ

944 / 6854
Хвора надійшла до клініки зі скаргами на загальну слабкість, запаморочення, задишку. Незадовго до звернення у клініку вона приймала левоміцетин для профілактики кишкових інфекцій. У крові:

ер.- 1,91012/л, Hb- 58 г/л, колірний показник- 0,9; лейк.- 2,2109/л. Про яку анемію це свідчить?

Гіпопластична

Залізодефіцитна

Апластична

Метапластична

Гемолітична

945 / 6854
До клініки потрапила дитина 1 року з ознаками ураження м’язів. Після обстеження виявлений дефіцит карнитину у м’язах. Порушення якого процесу є біохімічною основою цієї патології?

Синтез актину та міозину

Утилізація молочної кислоти

Субстратне фосфорилювання

Транспорт жирних кислот у мітохондрії Регуляція рівня Ca2+ B. в мітохондріях

946 / 6854
Жінка 38 років скаржиться на підвищену пітливість, серцебиття, підвищення температури тіла у вечірні години. Основний обмін збільшений на 60%. Лікар встановив діагноз тиреотоксикоз. Які властивості тироксину приводять до підсилення теплопродукції?

Розщеплює окисне фосфорилування

Підвищує спряження окиснення та фосфорилування

Сприяє накопиченню ацетил-КоА

Зменшує в-окиснення жирних кислот

Зменшує дезамінування амінокислот

947 / 6854
У людини збережена смакова, але втрачена загальна чутливість структур ротової порожнини. Про ураження якого нерва це свідчить?

N.glossopharyngeus та n.vagus

N.vagus

N.trigemmus

N.hypoglossus

N.glossopharyngeus

948 / 6854
До приймального відділення лікарні надійшов непритомний юнак з ознаками отруєння морфіном. Відзначається поверхневе та рідке дихання, яке обумовлене пригніченням дихального центру. Який тип недостатності дихання виник при цьому?

Перфузійна

Вентиляційна обструктивна

Вентиляційна дисрегуляторна

Дифузійна

Вентиляційна рестриктивна

949 / 6854
У хворого з серцевою недостатністю виникла аритмія, під час якої частота скорочень передсердь була 70, а шлуночків - 35/хв. Порушення якої функції провідної системи серця спостерігалося у хворого?

Збудливість

Провідність

Скоротливість

Автоматизм

Збудливість та провідність

950 / 6854
До лікаря звернувся пацієнт з приводу пожовтіння склер та шкіри. Під час обстеження не виявлено ознак енцефалопатії, холемічного та ахолічного синдромів. Яка жовтяниця розвинулася у цього пацієнта?

Механічна

Гемолітична

Паренхіматозна

Ядерна

Ензимопатична

951 / 6854
Жінка 63 років приймала пірацетам для відновлення функції ЦНС після ішемічного інсульту. Стан хворої значно покращився. Який механізм дії даного препарату?

Блокада дофамін-гідроксилази

Покращення метаболізму в ЦНС

Блокада моноамінооксидази

Блокада холінацетази

Блокада катехол-о-метилтрансферази

952 / 6854
У хворого на рентгенограмі легень виявлено затемнення. Під час діагностичної експресбіопсії лімфатичного вузла бронха виявлено: сирний некроз, навколо якого розташовані епітеліоїдні та лімфоїдні пласти з домішками багатоядерних гігантських клітин. Вкажіть причину лімфаденіту:

Туберкульоз

Пневмонія

Метастази раку

Сифіліс

Аденовірусна інфекція

953 / 6854
Хворий помер з явищами уремії. На розтині нирки збільшені, в’ялої консистенції, кіркова речовина широка, набрякла, з червоним крапом; мозкова речовина темно-червона. Мікроскопічно у порожнині капсули ниркового тільця виявлені “півмісяці' які стискають капіляри, дистрофія нефроцитів, набряк та інфільтрація строми. Яке захворювання стало причиною смерті цього чоловіка?

Амілоїдоз нирок

Нефротичний синдром

Гломерулонефрит

Нефролітіаз

Пієлонефрит

954 / 6854
У хворого струс головного мозку, що супроводжується повторним блюванням та задишкою. В артерiальнiй крові: рН- 7,62; PC02 - 40 мм рт.ст. Яке порушення кислотноосновного стану у хворого?

Негазовий ацидоз

Негазовий алкалоз

Газовий алкалоз

Газовий ацидоз

955 / 6854
У хворого на жовчокам’яну хворобу розвинулася механічна жовтяниця. Під час обстеження встановлено, що камінь знаходиться у загальній жовчній протоці. Які жовчовивідні протоки утворюють обтуровану протоку?

Ductus hepaticus dexter et ductus cysticus

Ductus hepaticus smster et ductus cysticus

Ductus hepaticus dexter et smster

Ductus hepaticus commurns et ductus choledochus

Ductus hepaticus commurns et ductus cysticus

956 / 6854
У дитини першого року життя під час профілактичного огляду виявлено порушення мінералізації кісток. Нестача якого вітаміну може бути причиною цього?

Кобаламін

Токоферол

Кальциферол

Рибофлавін

Фолієва кислота

957 / 6854
У чоловіка 30 років методом непрямої калориметрії встановлено, що його основний обмін на 30% менше від належного. Знижена секреція гормонів якої залози (яких залоз) є причиною цього?

Підшлункова

Наднирники

Щитовидна

Прищитоподібні

Епіфіз

958 / 6854
Новонароджена дитина має недорозвинений тимус. Який вид гемопоезу буде порушений?

Гранулоцитопоез

Мегакаріоцитопоез

Лімфопоез

Еритропоез

Моноцитопоез

959 / 6854
На мікропрепараті серця розрізняються клітини прямокутної форми, розмірами від 50 до 120 мкм, з центрально розташованим ядром, розвиненими міофібрилами, зв’язані між собою вставними дисками. Яку функцію виконують ці клітини?

Регенераторна

Проведення імпульсів

Ендокринна

Скорочення серця

Захисна

960 / 6854
У збудливій клітині заблокували іонні канали. Це суттєво не змінило рівень потенціалу спокою, але клітина втратила здатність до генерації ПД. Які канали заблоковано?

Кальцієві

Хлорні

Натрієві

Калієві

Натрієві та калієві

961 / 6854
У людини крововилив у задню центральну звивину призвів до порушення чутливості з протилежного боку. Який вид чутливості порушений?

Зорова

Шкірна та пропріоцептивна

Нюхова та смакова

Слухова

Слухова та зорова

962 / 6854
У пацієнта розвинулися порушення рухової активності: тремор, атаксія та асинергія рухів, дизартрія. Яка структура найбільш вірогідно уражена?

Лімбічна система

Базальні ганглії

Довгастий мозок

Мозочок

Стовбур мозку

963 / 6854
Хворому на миготливу аритмію, в анамнезі якого бронхіальна астма, треба призначити протиаритмічний засіб. Який препарат з цієї групи ПРОТИПОКАЗАНИЙ хворому?

Анаприлін

Новокаїнамід

Верапаміл

Ніфедипін

Аймалін

964 / 6854
На електронній мікрофотографії представлена клітина нейрального походження, що знаходиться у складі епітелію слизової оболонки. Дистальна частина периферійного відростку клітини має булавоподібне потовщення, від якого відходять 10-12 війок. Що це за клітина?

Колбочкова зорова клітина

Сенсорна клітина смакової цибулини

Палочкова зорова клітина

Біполярний нейрон спинномозкового вузла

Нюхова

965 / 6854
Ізольована клітина серця людини автоматично генерує імпульси збудження з частотою 60 разів за хвилину. З якої структури серця отримано цю клітину?

Синоатриальний вузол

Пучок Гіса

Шлуночок

Передсердя

Атріовентрикулярний вузол

966 / 6854
У людини через 10 хвилин після початку інтенсивної фізичної роботи кількість еритроцитів у крові збільшилася з 4,01012/л до 4,51012/л. Що є основною причиною цього?

Активація еритропоезу

Вихід еритроцитів з депо

Збільшення хвилинного об’єму крові

Втрата води організмом

Пригнічення руйнування еритроцитів

967 / 6854
У людини в артеріальній крові напруга кисню збільшена до 104 мм рт.ст., а вуглекислого газу зменшена до 36 мм рт.ст. Що може бути причиною цього?

Перебування у горах

Затримка дихання

Довільна гіпервентиляція

Помірне фізичне навантаження

Інтенсивне фізичне навантаження

968 / 6854
У чоловіка 33 років діагностовано прободіння шлунка та запалення очеревини, що призвело до напруження м’язів передньої черевної стінки (“доскоподібний живіт”). Який рефлекс забезпечує цей симптом?

Сомато-вісцеральний

Вісцеро-вісцеральний

Вісцеро-соматичний

Кутанно-вісцеральний

Вісцеро-кутанний

969 / 6854
У хворого глибока рвана рана із нерівними краями, вкрита гноєм. У крайових відділах сочна грануляційна тканина, яка не здіймається над рівнем рани. Назвіть вид загоєння рани:

Безпосереднє закриття дефекту епітеліальної тканини

Вторинним натягом

Під струпом

Первинним натягом

970 / 6854
Під час гістологічного дослідження лімфовузла хворого 18 років з ділянки заднього трикутника шиї морфолог виявив скопичення клітин, серед яких поодинокі багатоядерні клітини Березовського-Штернберга, великі клітини Ходжкіна, малі клітини Ходжкіна та багато лімфоцитів, поодинокі плазматичні клітини, еозинофіли. Яке захворювання у хворого?

Лімфоцитарна лімфома

Хронічний лімфоїдний лейкоз

Пухлина Беркіта

Лімфогранулематоз

Нодулярна лімфома

971 / 6854
Дівчинка 6 років захворіла на дифтерію та померла від асфіксії на третю добу. На аутопсії слизова оболонка трахеї та бронхів стовщена, набрякла, тьмяна, вкрита сіруватими плівками, які легко відокремлюються. Про який вид запалення свідчать морфологічні зміни?

Крупозне

Катаральне

Дифтеритичне

Серозне

Геморагічне

972 / 6854
Хворому з травмою передпліччя під час репозиції кісток для міорелаксації введено дитилін. Повне відновлення тонусу та функції м’язів відбулося більш, ніж через годину. Чим можна пояснити значне подовження курареподібної дії препарату?

Утворенням активного метаболіту

Генетичним дефіцитом гідроксилази

Пригнічєнням мікросомного окиснення

Генетичним дефіцитом бутирилхолінестерази

Генетичним дєфіцитом моноамінооксидази

973 / 6854
У людини, що виконувала важку фізичну роботу в умовах підвищеної температури навколишнього середовища, змінилася кількість білків плазми крові. Що саме має місце у даному випадку?

Парапротеїнемія

Абсолютна гіпепротеїнемія

Відносна гіперпротеїнемія

Абсолютна гіпопротеїнемія

Диспротеїнемія

974 / 6854
Хворому, що страждає на стенокардію та приймає ізосорбіда мононітрат, було додатково призначено лікарський засіб з дезагрегантним ефектом. Визначте цей препарат:

Анаприлін

Нітрогліцерин

Ацетилсаліцилова кислота

Ніфедипін

Валідол

975 / 6854
Хворому в післяопераційному періоді для стимуляції перистальтики кишечника та тонусу сечового міхура було призначено препарат з групи антихолінестеразних засобів. Визначте його серед нижченаведених препаратів:

Дихлотіазид

Резерпін

Маніт

Анаприлін

Прозерин

976 / 6854
У хворого діагностовано ураження стулок правого передсердно-шлуночкового клапана. Внаслідок запального процесу якої анатомічної структури серця відбулися зміни у стулках?

Епікард

Ендокард

Міокард

Серозний перикард

Фіброзний перикард

977 / 6854
У хворого на обличчі вугрі. Під час мікроскопії зскрібків із уражених ділянок виявлені живі членистоногі розміром 0,2-0,5 мм., які мають витягнуту червоподібну форму, чотири пари коротких кінцівок, що розташовані у середній частині тіла. Який лабораторний діагноз?

Фтиріоз

Міаз

Педикульоз

Демодекоз

Короста

978 / 6854
У хворого, який тривалий час страждає на хронічний ентероколіт, після вживання молока виникли метеоризм, діарея, коліки. З нестачею якого ферменту в кишечнику це пов’язано?

Сахараза

Лактаза

Амілаза

Глікогенситнетаза

Мальтаза

979 / 6854
У новонародженної дитини виявлено гнійні виділення з кон’юктиви ока. Під час мікроскопії мазка з кон’юктиви знайдено велику кількість лейкоцитів та грамнегативні бобоподібні диплококи, що знаходяться всередині лейкоцитів. Який збудник є причиною цього захворювання?

Staphylococcus aureus

Streptococcus pyogenes

Neisseria gonorrhoeae

Neisseria catarrhalis

Staphylococcus epidermidis

980 / 6854
Жінці 30 років на 32 тижні вагітності лікар, в складі комплексної терапії виразкової хвороби шлунка, призначив антибіотик. Вкажіть, який препарат можна застосувати у даному випадку?

Тетрациклін

Азитроміцин

Левоміцетин

Гентаміцин

Бензилпеніцилін

981 / 6854
Під час ультразвукового обстеження вагітної жінки було діагностовано багатоводдя. З порушенням діяльності яких позазародкових органів можна пов’язати даний патологічний стан?

Жовтковий мішок

Алантоїс

Хоріон

Плацента

Амніотична оболонка

982 / 6854
У хворого з гострим циститом під час дослідження сечі виявили лейкоцити та багато грамнегативних паличок. При посіві виросли колонії слизового характеру, які утворювали зелений розчинний пігмент. Який мікроорганізм, найбільш вірогідно, є причиною захворювання?

Salmonella enteritidis

Pseudomonas aeruginosa

Escherihia coli

Proteus mirabilis

Klebsiella pneumoniae

983 / 6854
Чоловіка 49 років доставлено з місця автомобільної аварії до лікарні у непритомному стані. Шкірні покриви бліді, пульс частий та поверхневий. Переломів кісток та пошкодження головного мозку не виявлено. Під час пункції черевної порожнини отримано значну кількість крові. Що є первинною причиною важкого стану потерпілого?

Еритропенія

Гіпопротеїнемія

Гіпонатріємія

Гіповолемія

Гіпоінсулінемія

984 / 6854
У жінки 45 років народився хлопчик з розщепленням верхньої щелепи (“заяча губа” та “вовча паща”). Під час додаткового обстеження виявлені значні порушення з боку нервової, серцево-судинної систем та зору. При дослідженні каріотипу діагностована трісомія по 13 хромосомі. Який синдром має місце у хлопчика?

Едвардса

Дауна

Клайнфельтера

Шерешевського-Тернера

Патау

985 / 6854
Під час медико-генетичного консультування родини зі спадковою патологією виявлено, що аномалія проявляється через покоління у чоловіків. Який тип успадковування притаманний для цієї спадкової аномалії?

Х-зчеплене домінантне

Х-зчеплене рецесивне

Аутосомне рецесивне

Y-зчеплене

Аутосомне домінантне

986 / 6854
Людина 28 років споживає надмірну кількість вуглеводів (600 г на добу), що перевищує її енергетичні потреби. Який процес буде активуватися у даному випадку?

Окислення жирних кислот

Ліпогенез

Гліколіз

Глюконеогенез

Ліполіз

987 / 6854
У дитини 5 років діагностовано хворобу Брутона, яка проявляється у важкому перебігу бактеріальних інфекцій, відсутності В-лімфоцитів та плазматичних клітин. Які зміни вмісту імуноглобулінів будуть спостерігатися у сироватці крові цієї дитини?

Зменшення IgD,IgE

Збільшення IgD,IgE

Зменшення IgA, IgM

Збільшення IgA, IgM

Змін не буде

988 / 6854
У жінки 30 років виявлено недостатність зовнішньосекреторної функції підшлункової залози. Гідроліз яких поживних речовин буде порушений?

Білки, жири

Білки

Білки, жири, вуглеводи

Білки, вуглеводи

Жири, вуглеводи

989 / 6854
На практичному занятті з мікробіології студентам запропоновано пофарбувати готові зафіксовані мазки із мокротиння хворого на туберкульоз. Який метод фарбування треба використати у данному випадку?

Буррі

Гінса

Циля-Нільсена

Грама

Романовського-Гімза

990 / 6854
Під час обстеження на бактеріоносійство працівників дитячих закладів у виховательки виділена С.diphtheriae. Було проведено дослідження на токсигенність збудника, яке показало, що цей штам С.diphtheriae не продукує екзотоксин. Яку реакцію провели при дослідженні на токсигенність дифтерійних бактерій?

Реакція кільцепреципітації

Реакція зв’язування комплементу

Реакція аглютинації

Реакція преципітації в агаровому гелі

Реакція імунофлуоресценсії

991 / 6854
У хворої параліч мімічної мускулатури, розлад сприйняття смаку в передніх 2/3 язика, зменшення слиновиділення. Який з черепних нервів уражений?

N.facialis

N.hypoglossus

N.glossphapyngeus

N.vagus

N.trigeminus

992 / 6854
Лікар-дослідник у складі альпіністської експедиції піднявся у базовий табір, розташований на висоті 5000 м. На 3-й день перебування у нього з’явилися ознаки гірської хвороби: задишка, головний біль, втрата апетиту, загальна слабкість, ціаноз. Який тип гіпоксії має місце в цьому випадку?

Змішана

Циркуляторна

Гіпоксична

Тканинна

Гемічна

993 / 6854
У хворого на колагеноз після тривалого прийому преднізолону з’явився спастичний біль скелетних м’язів внаслідок розвитку гипокаліємії. Який препарат треба використати для корекції обміну калію?

Тирокальцитонін

Діазепам

Панангін

Дитилін

Но-шпа

994 / 6854
У робітника 37 років, який працював у кесоні, після підйому на поверхню раптово з’явилися ознаки гострого порушення мозкового кровообігу, непритомність. Через декілька днів він помер. На розтині в лівій півкулі головного мозку виявлено вогнище сірого кольору м’якої консистенції, неправильної форми, розмірами 5х6х3,5 см. Який процес мав місце у головному мозку?

Пухлина

Абсцес

Кіста

Ішемічний інсульт

Геморагічний інсульт

995 / 6854
Зі слизових оболонок та з харкотиння хворого, який тривалий час приймав імунодепресанти, були виділені грампозитивні великі овальні клітини з брунькуванням, що розташовані хаотично, та подовжені клітини у вигляді ланцюжків. Який збудник виділений?

Ієрсінії

Актиноміцети

Стрептококи

Кандиди

Стрептобактерії

996 / 6854
У здорової дорослої людини проводять зондування порожнин серця. Зонд знаходится у лівому шлуночку. Під час якої фази (періоду) серцевого циклу буде зареєстровано збільшення тиску від 8 до 70 мм рт.ст.?

Фаза повільного вигнання

Період вигнання

Фаза швидкого вигнання

Фаза асинхронного скорочення

Фаза ізометричного скорочення

997 / 6854
Людина за призначенням лікаря тривалий час приймала препарат з групи глюкокортикоїдних гормонів. Секреція якого (яких) з наведених гормонів буде пригнічена внаслідок цього?

Соматотропний

Тиротропний

Мінералокортикоїди

Кортикотропний

Статеві

998 / 6854
У студента через 2 години після іспиту в аналізі крові виявлено лейкоцитоз без істотних змін у лейкоцитарній формулі. Який найбільш вірогідний механізм розвитку лейкоцитозу?

Уповільнення міграції лейкоцитів у тканини

Перерозподіл лейкоцитів в організмі

Посилення лейкопоезу та зменшення руйнування лейкоцитів

Посилення лейкопоезу

Уповільнення руйнування лейкоцитів

999 / 6854
У молодої жінки видалили пухлину дистального кінця стегнової кістки, яка швидко зростала. Макроскопічно: пухлина строкатого вигляду - від біло-сірого до коричневочервоного кольору, пухкої консистенції. Мікроскопічно: основний тканинний компонент пухлини представлений кістковими та остеоїдними структурами, вистеленими атиповими остеобластами з патологічними мітозами; безліч тонкостінних судин. Який найбільш вірогідний діагноз?

Остеома

Хондрома

Остеосаркома

Ангіосаркома

Саркома Юїнга

1000 / 6854
У хворого підвищений вміст глюкози в крові; є цукор в сeчі. Під час пункційної біопсії нирки виявлено: розширення мезангія з осередковим накопиченням мембраноподібної речовини з перигломерулярним склерозом деяких клубочків, гіаліноз та плазматичне просякання артеріол, лімфогістіоцитарна інфільтрація строми з наявністю поліморфноядерних лейкоцитів; глікогенна інфільтрація нефроцитів вузького сегмента. Який найбільш вірогідний діагноз?

Підгострий гломерулонефрит

Пієлонефрит

Діабетичний гломерулосклероз

Гострий гломерулонефрит

Хронічний гломерулонефрит

1001 / 6854
До хірургічного відділення надійшов чоловік 35-ти років з гнійною раною шиї попереду трахеї (в ділянці передвісцерального простору). Куди може поширитися інфекція, якщо терміново не буде проведене оперативне втручання?

У надгруднинний міжапоневротичний простір

У заднє середостіння

У переднє середостіння

У ретровісцеральний простір

У середнє середостіння

1002 / 6854
Тривале перебування в умовах спеки викликало в людини спрагу. Сигналізація від яких рецепторів, перш за все, зумовила її розвиток?

Осморецептори гіпоталамуса

Осморецептори печінки

Глюкорецептори гіпоталамуса

Барорецептори дуги аорти

Натрієві рецептори гіпоталамуса

1003 / 6854
Людина, яка проживала в ендемічному вогнищі, перехворіла триденною малярією. Через півтора року після переїзду в іншу місцевість захворіла малярією знову. Яка найбільш вірогідна форма цього захворювання?

Суперінфекція

Реінфекція

Вторинна інфекція

Персистуюча інфекція

Рецидив

1004 / 6854
До приймального відділення в важкому стані надійшов чоловік 38-ми років, який отруївся сулемою. Який антидот треба негайно ввести хворому?

Ізонітрозин

Атропін

Налорфін

Унітіол

Дипіроксим

1005 / 6854
Людина хворіє на цукровий діабет, що супроводжується гіперглікемією натще понад 7,2 ммоль/л. Рівень якого білка плазми крові дозволяє ретроспективно (за попередні 4-8 тижні до обстеження) оцінити рівень глікемії?

Глікозильований гемоглобін

Церулоплазмін

Фібриноген

Альбумін

С-реактивний білок

1006 / 6854
У дітей часто можна спостерігати утруднене носове дихання, яке пов’язане з надмірним розвитком лімфоїдної тканини слизової оболонки глотки. Розростання яких мигдаликів може спричинити це явище?

Tonsilla palatina

Tonsilla tubaria

Tonsilla pharyngea

Tonsilla lingualis

Усі названі мигдалики

1007 / 6854
У чоловіка 50-ти років раптово виникли сильне серцебиття, біль у серці, різка слабкість, підвищення артеріального тиску; пульс аритмічний, з дефіцитом. На ЕКГ виявлено відсутність зубців Р та різна тривалість інтервалів R-R. Яке порушення серцевого ритму в хворого?

Синусова екстрасистолія

Поперечна блокада серця

Миготлива аритмія

Пароксизмальна тахікардія

Дихальна аритмія

1008 / 6854
У пацієнта 60-ти років виявлено погіршення сприйняття звуків високої частоти. Порушення стану яких структур слухового аналізатора зумовило ці зміни?

М’язи середнього вуха

Барабанна перетинка

Євстахієва труба

Основна мембрана завитки біля овального віконця

Основна мембрана завитки біля гелікотреми

1009 / 6854
У хворого спостерігається ішемія тканин нижче колінного суглоба, що супроводжується 'переміжною кульгавістю'. Про оклюзію якої артерії слід думати?

Проксимальна частина стегнової

Передня великогомілкова

Задня великогомілкова

Підколінна

Малогомілкова

1010 / 6854
В експерименті було показано, що опромінені ультрафіолетом клітини шкіри хворих на пігментну ксеродерму, через дефект ферменту репарації, повільніше відновлюють нативну структуру ДНК, ніж клітини здорових людей. За допомогою якого ферменту відбувається цей процес?

Ендонуклеаза

Праймаза

ДНК-полімераза III

ДНК-гіраза

РнК-лігаза

1011 / 6854
Ветеринарний фельдшер, що працював на тваринницькій ферму звернувся до лікаря зі скаргами на 6іль у суглобах, лихоманку, нездужання, пітливість по ночам. Хворіє близько місяця. Враховуючи скарги та професійний анамнез, лікар запідозрив у нього бруцельоз. Який матеріал, взятий у цього хворого, підлягає дослідженню в звичайній мікробіологічній лабораторії?

Випорожнення

Блювотні маси

Сеча

Спинномозкова рідина

Сироватка крові

1012 / 6854
У хворого виявлено зниження вмісту іонів магнію, які потрібні для прикріплення рибосом до гранулярної ендоплазматичної сітки. Відомо, що це призводить до порушення біосинтезу білка. Який саме етап біосинтезу білка буде порушено?

Реплікація

Трансляція

Активація амінокислот

Транскрипція

Термінація

1013 / 6854
У пацієнта після травми виникли паралічі, розлади больової чутливості праворуч; з лівого боку - паралічі відсутні, але порушена больова та температурна чутливість. Яка причина такого явища?

Пошкодження стовбура мозку

Пошкодження мозочка

Пошкодження рухової зони кори головного мозку

Пошкодження середнього мозку

Однобічне пошкодження спинного мозку з правого боку

1014 / 6854
Студент старанно конспектує лекцію. Якість конспектування значно погіршилася, коли сусіди стали розмовляти. Який вид гальмування в корі головного мозку є причиною цього?

Позамежове

Запізніле

Згасаюче

Диференціювальне

Зовнішнє

1015 / 6854
При рентгенологічному дослідженні кісток основи черепа виявлено збільшення порожнини турецького сідла, витончення передніх нахилених відростків, руйнування різних ділянок турецького сідла. Пухлина якої ендокринної залози може спричинити таке руйнування кісток?

Наднирники

Гіпофіз

Епіфіз

Щитоподібна залоза

Вилочкова залоза

1016 / 6854
До лікаря-невропатолога звернулася хвора 52-х років із скаргами на втрату чутливості шкіри правої половини обличчя в ділянці нижньої повіки, спинки носа та верхньої губи. Вкажіть, яка гілка якого нерва при цьому ушкоджена:

Великий кам’янистий нерв лицевого нерва

Нижньощелепна гілка трійчастого нерва

Барабанна струна лицевого нерва

Очна гілка трійчастого нерва

Верхньощелепна гілка трійчастого нерва

1017 / 6854
Хворий з нейродермітом протягом тривалого часу вживав преднізолон. При обстеженні в нього виявили підвищення рівня цукру в крові. Вплив препарату на яку ланку вуглеводного обміну призводить до виникнення цього ускладнення?

Активація глюконеогенезу

Посилення всмоктування глюкози в кишечнику

Пригнічення синтезу глікогену

Активація розщеплення інсуліну

Активація глікогеногенезу

1018 / 6854
Надмірне оволосіння вушних раковин (гіпертрихоз) визначається геном, локалізованим у Y-хромосомі. Цю ознаку має батько. Яка вірогідність народження хлопчика з такою аномалією?

35%

0%

75%

100%

25%

1019 / 6854
У дівчини виявлена диспропорція тіла, крилоподібні складки на шиї. При цитогенетичному дослідженні в ядрах лейкоцитів не виявлені 'барабанні палички', а в ядрах букального епітелію відсутні тільця Барра. Який найбільш вірогідний діагноз?

Синдром Едвардса

Синдром Патау

Синдром Шерешевського-Тернера

Синдром Клайнфельтера

Синдром Дауна

1020 / 6854
У хворих на колагеноз має місце процес деструкції сполучної тканини. Це підтверджується збільшенням у крові:

Активності трансаміназ

Вмісту уратів

Активності ізоферментів ЛДГ

Вмісту креатину та креатиніну

Вмісту оксипроліну та оксилізину

1021 / 6854
Хворому з частими нападами стенокардії був призначений сустак-форте по 1 табл. 2 рази на день. Спочатку відмічався позитивний ефект, однак, на другу добу, напади стенокардії відновилися. Чим можна пояснити неефективність призначеного препарату?

Тахіфілаксія

Сенсибілізація

Ідіосинкразія

Кумуляція

Залежність

1022 / 6854
На розтині тіла чоловіка 35-ти років у другому сегменті правої легені виявлено вогнище ущільнення діаметром 5 см, оточене тонкою капсулою. Вогнище представлене сухою крихкою тканиною з тьмяною поверхнею. Для якого захворювання характерні такі морфологічні зміни?

Хондрома

Післязапальний пневмосклероз

Рак легені

Туморозна форма силікозу

Туберкулома

1023 / 6854
У хворого лікар діагностував гостру гонорею. З анамнезу стало відомо, що раніше він переніс гонорею і вилікування було повним. До якої категорії інфекцій можна віднести це нове захворювання?

Вторинна інфекція

Суперінфекція

Автоінфекція

Рецидив

Реінфекція

1024 / 6854
Хворій проводиться операція на щитоподібній залозі. Гілки яких артерій має перев’язати хірург під час операції?

Верхня та нижня гортанні

Висхідна та глибока артерії шиї

Верхня щитоподібна та щитошийний стовбур

Верхня та нижня щитоподібні

Верхня щитоподібна та висхідна артерія шиї

1025 / 6854
Під час вивчення фаз мітотичного циклу корінця цибулі знайдено клітину, в якій хромосоми лежать в екваторіальній площині, утворюючи зірку. На якій стадії мітозу перебуває клітина?

Профаза

Анафаза

Телофаза

Інтерфаза

Метафаза

1026 / 6854
Запалення барабанної порожнини (гнійний середній отит) у хворого ускладнилося запаленням комірок соскоподібного відростка. Через яку стінку барабанної порожнини проникнув гній до комірок?

Верхня

Задня

Передня

Латеральна

Медіальна

1027 / 6854
На спеціальному живильному середовищі після посіву виділення гною з уретри, виросли ніжні голубуваті колонії. При мікроскопії препаратів з них виявлені грамнегативні бобовинні диплококи. Збудником якої хвороби вони є?

Меліоїдоз

Хламідіоз

Сифіліс

Гонорея

Туляремія

1028 / 6854
В бактеріологічній лабораторії досліджували в’ялену рибу домашнього виготовлення, яка стала причиною важкого харчового отруєння. При мікроскопії виділеної на середовищі Кіта-Тароцці культури виявлені мікроорганізми, схожі на тенісну ракетку. Який найбільш вірогідний діагноз?

Сальмонельоз

Дизентерія

Ботулізм

Черевний тиф

Холера

1029 / 6854
До лікаря звернулася хвора зі скаргами на розлади травлення, розлитий біль у животі. При обстеженні лікар виявив різко виражене зниження гемоглобіну в крові. З опитування виявилося, що, під час проживання на Далекому Сході, хвора часто вживала в їжу малосольну рибну ікру. Аналогічний стан відзначений у деяких родичів, що проживають з нею. Яке захворювання діагностував лікар у цієї хворої?

Ехінококоз

Теніоз

Трихінельоз

Аскаридоз

Дифілоботріоз

1030 / 6854
У клітині повністю заблокований синтез АТФ. Як зміниться величина мембранного потенціалу спокою цієї клітини?

Незначно збільшиться

Спочатку зменшиться, потім збільшиться

Істотно збільшиться

Спочатку збільшиться, потім зменшиться

Зникне

1031 / 6854
Хворій, що перенесла мастектомію в зв’язку з раком молочної залози, був призначений курс променевої терапії. Який з перерахованих вітамінних препаратів має виражену радіопротекторну дію, зумовлену антиоксидантною активністю?

Фолієва кислота

Ергокальциферол

Тіаміну хлорид

Токоферолу ацетат

Рибофлавін

1032 / 6854
У померлого від хронічної серцево-судинної недостатності на розтині виявлене 'тигрове серце'. З боку ендокарду помітна жовтувато-біла посмугованість; міокард тьмяний, глинисто-жовтий. Який процес зумовив дану патологію?

Вуглеводна дистрофія

Амілоїдоз

Гіаліново-краплинна дистрофія

Жирова судинно-стромальна дистрофія

Жирова паренхіматозна дистрофія

1033 / 6854
У хворої дитини періодично з’являються рідкі випорожнення, іноді біль у ділянці живота, нудота, блювання. Зі слів матері, одного разу в дитини з блювотними масами виділився гельмінт веретеноподібної форми, розміром 20 см. Причиною такого стану може бути:

Анкілостомоз

Дракункульоз

Трихоцефальоз

Аскаридоз

Трихінельоз

1034 / 6854
Дистрофічні зміни серцевого м’яза супроводжуються розширенням порожнин серця, зниженням сили серцевих скорочень, збільшенням об’єму крові, що залишається під час систоли в порожнині серця, переповненням вен. Для якого стану серця це характерно?

Тампонада серця

Тоногенна дилатація

Аварійна стадія гіперфункції та гіпертрофії

Кардіосклероз

Міогенна дилатація

1035 / 6854
У хворого 27-ми років діагностовано гнійне запалення жовчного міхура. До якого відділу очеревинної порожнини потрапить гній під час розриву жовчного міхура при його типовому положенні?

До верхнього дванадцятипалого закутка

До чепцевої сумки

До печінкової сумки

До передшлункової сумки

До лівого бічного каналу

1036 / 6854
У хворого, який тривалий час страждає на переміжну кульгавість, тканини пальців стопи сухі, чорного кольору, нагадують мумію. На невеликій відстані від почорнілої ділянки розташована двоколірна лінія (червона лінія прилягає до практично незмінених тканин, а біло-жовта - до змінених тканин). Який вид некрозу в даного хворого?

Мацерація

Інфаркт

Секвестр

Гангрена

Пролежень

1037 / 6854
Збудник гепатиту D (дельта-агент) є дефектним вірусом, який може реплікуватись лише в клітинах, що вже інфіковані одним з вірусів. Який саме вірус необхідний для розмноження дельта-вірусу?

Вірус Епстайна-Барр

Вірус гепатиту Е

Вірус гепатиту В

Вірус імунодефіциту людини

Вірус гепатиту А

1038 / 6854
Хворому призначено дигоксин. Через декілька днів виявлено ознаки передозування препарату, його вміст у крові значно перевищує верхню межу терапевтичної концентрації. Як називається такий варіант дії лікарських речовин?

Антагонізм

Звикання

Тахіфілаксія

Потенціювання

Кумуляція

1039 / 6854
Хворий чоловік госпіталізований на 5-й день хвороби з проявами жовтяниці, болем у м’язах, ознобом, носовими кровотечами. Під час проведення лабораторної діагностики бактеріолог виконав темнопольну мікроскопію краплини крові хворого. Назвіть збудника хвороби:

Leptospira interrogans

Calymmatobacterium granulomatis

Rickettsia mooseri

Bartonella bacilloformis

Borrelia dutlonii

1040 / 6854
При переведенні погляду на близько розташований об’єкт, заломна сила оптичних середовищ ока збільшилася на 10 діоптрій. Це є результатом зміни стану такої структури ока:

Кришталик

Рогівка

Скловидне тіло

М’яз, що розширює зіницю

Волога передньої камери ока

1041 / 6854
Хворому з метою відновлення дихання, при отруєнні чадним газом, було введено аналептичний засіб рефлекторного типу дії з групи Н-холіноміметиків. Який засіб було призначено хворому?

Атропіну сульфат

Лобеліну гідрохлорид

Мезатон

Адреналіну гідрохлорид

Пентамін

1042 / 6854
У хворого, прооперованого з приводу ускладненого апендициту, в крові відзначаються наступні зміни: ер.-4,0•1012/л, Нb-120 г/л, КП- 0,9, лейк.-18•109 /л, б - 0, е - 0, мієлоц.- 0, ю - 0, п - 20, с - 53, л - 21, м - 5. Як називається такий ядерний зсув лейкоцитарної формули?

Гіперрегенеративний

Дегенеративний зсув вліво

Регенеративний зсув вліво

Регенеративно-дегенеративний

Зсув вправо

1043 / 6854
Забір крові для загального аналізу рекомендують проводити натщесерце та зранку. Які зміни складу крові можливі, якщо провести забір крові після приймання їжі?

Збільшення кількості еритроцитів

Збільшення білків плазми

Зниження кількості тромбоцитів

Зниження кількості еритроцитів

Збільшення кількості лейкоцитів

1044 / 6854
Для серологічної діагностики сифілісу в реакції Вассермана лікар-лаборант підготував такі реактиви: кардіоліпіновий антиген, спиртовий екстракт ліпоїдів із серцевого м’яза бика з холестерином, антиген з трепонем, зруйнованих ультразвуком, гемолітична система, фізіологічний розчин, досліджувані сироватки. Який ще компонент необхідний для постановки реакції?

Живі трепонеми

Комплемент

Еритроцити барана

Антиглобулінова сироватка

Діагностична преципітуюча сироватка

1045 / 6854
До відділення реанімації надійшов чоловік 47-ми років з діагнозом інфаркт міокарду. Яка з фракцій лактатдегідрогенази (ЛДГ) буде переважати в сироватці крові протягом перших двох діб?

ЛДГ3

ЛДГ1

ЛДГ4

ЛДГ2

ЛДГ5

1046 / 6854
Під час операції на головному мозку відмічено, що подразнення певних зон кори великих півкуль викликало в хворого i тактильні і температурні відчуття. На яку зону кори діяли подразники?

Поясна звивина

Верхня латеральна звивина

Парагіпокампова звивина

Прецентральна звивина

Постцентральна звивина

1047 / 6854
До відділення травматології надійшов хворий із значним пошкодженням м’язової тканини. Який біохімічний показник сечі буде збільшений при цьому?

Глюкоза

Загальні ліпіди

Сечова кислота

Креатинін

Мінеральні солі

1048 / 6854
У хворої 43-х років на фоні септичного шоку відзначається тромбоцитопенія, зниження фібриногену, поява в крові продуктів дегенерації фібрину, поява петехіальних крововиливів. Вкажіть причину виникнення даних змін:

Автоімунна тромбоцитопенія

Порушення утворення тромбоцитів

Екзогенна інтоксикація

ДВЗ-синдром

Геморагічний діатез

1049 / 6854
Хворому з великими опіками зробили пересадку донорської шкіри. На 8-му добу трансплантат набряк, змінився його колір; на 11 добу почав відторгатися. Які клітини беруть у цьому участь?

Базофіли

Еозинофіли

В-лімфоцити

Т-лімфоцити

Еритроцити

1050 / 6854
Хворий з інфекційним мононуклеозом протягом двох тижнів приймав глюкокортикостероїдні препарати. Наступила ремісія, проте в нього виникло загострення хронічного тонзиліту. Результатом якої дії глюкокортикостероїдів є дане ускладнення?

Антиалергічна

Протизапальна

Протишокова

Імунодепресивна

Антитоксична

1051 / 6854
Під час розтину трупа дівчинки 12-ти років виявлено: множинні крововиливи у шкірі (переважно сідниць, нижніх кінцівок), серозних та слизових оболонок, у головному мозку. У наднирниках вогнищевий некроз та масивні крововиливи, у нирках - некротичний нефроз, гнійний артрит, іридоцикліт, васкуліт. Який найбільш вірогідний діагноз?

Вузликовий періартеріїт

Менінгококцемія

Системний червоний вівчак

Висипний тиф

Променева хвороба

1052 / 6854
Більша частина учасників експедиції Магелана до Америки загинула від авітамінозу. Це захворювання проявлялося загальною слабкістю, підшкірними крововиливами, випадінням зубів, кровотечею з ясен. Як називається цей авітаміноз?

Поліневрит (бері-бері)

Пелагра

Рахіт

Анемія Бірмера

Скорбут (цинга)

1053 / 6854
При розтині тіла жінки 40-а років, яка страждала на ревматоїдний артрит, знайдена збільшена щільна селезінка. На розрізі її тканина коричнево-червоного кольору зі збільшеними фолікулами, які мають вигляд напівпрозорих сірувато-білуватих зерен. Вкажіть, який з перелічених патологічних процесів найбільш вірогідний?

Гіаліноз селезінки

Сагова селезінка

Сальна селезінка

Глазурна селезінка

Порфірна селезінка

1054 / 6854
Чоловік 28-ми років надійшов зі скаргами на нудоту, блювання, біль у правому підребер’ї. Об’єктивно: жовтяничність шкіри, склер; температура тіла підвищена, печінка збільшена, сеча темна, кал гіпохолічний. Гіпербілірубінемія (білірубін прямий та непрямий), білірубінурія, уробілінурія, гіпопротеїнемія, зниження зсідання крові. Для якого з перелічених нижче станів найбільш характерні ці зміни?

Клітинно-паренхіматозна жовтяниця

Надпечінкова гемолітична жовтяниця

Гострий холецистит

Підпечінкова жовтяниця

Гострий панкреатит

1055 / 6854
Хворому на сечокам’яну хворобу після обстеження призначили алопуринол - конкурентний інгібітор ксантиноксидази. Підставою для цього був хімічний аналіз ниркових каменів, які складалися переважно з:

Сульфату кальцію

Урату натрію

Моногідрата оксалату кальцію

Дигідрата оксалату кальцію

Фосфату кальцію

1056 / 6854
Під час обстеження підлітка, що страждає на ксантоматоз, виявлена сімейна гіперхолестеринемія. Концентрація яких ліпопротеїнів значно підвищена в крові при цій патології?

НЕЖК

ЛПДНГ

Хіломікрони

ЛПНГ

ЛПВГ

1057 / 6854
У жінки, що хворіє на міастенію, виникли розлади дихання, що вимагало застосування штучної вентиляції легень. Який вид дихальної недостатності розвинувся в даної хворої?

Центрогенний

Нервово-м’язовий

Торакодіафрагмальний

Рестриктивний

Обструктивний

1058 / 6854
У вагітної жінки розвинувся токсикоз з важкими повторними блюваннями протягом доби. До кінця доби почали з’являтися тетанічні судоми та зневоднення організму. Який зсув кислотно-лужного стану викликав вказані зміни?

Видільний ацидоз

Метаболічний ацидоз

Видільний алкалоз

Газовий алкалоз

Газовий ацидоз

1059 / 6854
У хворої 18-ти років пахові лімфатичні вузли збільшені в розмірах, не болючі, ущільнені при пальпації. У ділянці слизової оболонки геніталій невеликих розмірів виразка з ущільненими краями та 'лакованим'дном сіруватого кольору. Який найбільш вірогідний діагноз?

Сифіліс

Туберкульоз

Гонорея

Трофічна виразка

Лепра

1060 / 6854
У хворого з підозрою на дифтерію під час бактеріоскопічного дослідження мазка з зіву виявлені паличкоподібні бактерії з зернами волютину. Який етіотропний препарат є препаратом вибору в даному випадку?

Дифтерійний анатоксин

Інтерферон

Протидифтерійна антитоксична сироватка

Еубіотик

Бактеріофаг

1061 / 6854
Чоловік 26-ти років скаржиться на безплідність. Об’єктивно: зріст 186 см, довгі кінцівки, гінекомастія, гіпоплазія яєчок, у зіскрібку слизової оболонки щоки знайдені тільця Барра. Діагностований синдром Клайнфельтера. Який механізм хромосомної аномалії має місце при даному захворюванні?

Інверсія хромосоми

Нерозходження гетосом у мейозі

Делеція хромосоми

Транслокація

Нерозходження хроматид у мітозі

1062 / 6854
Чоловік 32-х років чотири роки страждає на хронічний гломерулонефрит з нефротичним синдромом. Відзначаються явні набряки на обличчі, в останній час з’явилися набряки на ногах та тулубі. Який із механізмів розвитку набряків найбільш вірогідний у цього хворого?

Зниження онкотичного тиску крові

Підвищення гідростатичного тиску крові в капілярах

Підвищення проникливості капілярів

Утруднення лімфовідтоку

Підвищення онкотичного тиску тканинної рідини

1063 / 6854
У жінки, яка тривалий час знаходилася на дієті з використанням очищеного рису, виявлено поліневрит (хвороба Бері-Бері). Відсутність якого вітаміну в раціоні призводить до розвитку цього захворювання?

Піридоксаль

Рибофлавін

Тіамін

Аскорбінова кислота

Фолiєва кислота

1064 / 6854
У хворого після видалення жовчного мiхура утруднені процеси всмоктування Ca через стінку кишечнику. Призначення якого вітаміну буде стимулювати цей процес?

В12

К

РР

D3

С

1065 / 6854
У хворого з гіпертонічною хворобою з’явилися головний біль, шум у вухах, блювання. Артеріальний тиск підвищився до 220/160 мм рт.ст. Під час обстеження виявлена асиметрія обличчя з правого боку, відсутність довільних рухів, підвищення сухожилкових рефлексів та тонусу м’язів правих руки і ноги. Яка форма розладів рухової функції має місце в цьому випадку?

Параплегія

Геміплегія

Гіперкінез

Тетраплегія

Моноплегія

1066 / 6854
У жінки, яка хворіє на гіпертонічну хворобу, розвинувся напад бронхіальної астми. Який засіб слід призначити для зняття нападу?

Сальбутамол

Адреналін

Еуфілін

Ізадрин

Ефедрин

1067 / 6854
Фенілкетонурія успадковується як аутосомно-рецесивна ознака. У здорових батьків народилася дитина, хвора на фенілкетонурію. Які генотипи батьків?

аа х аа

Аа х Аа

Аа х аа

АА х АА

АА х Аа

1068 / 6854
Хворий відзначає часті проноси, особливо після вживання жирної їжі, схуднення. Лабораторні дослідження показали наявність стеатореї; кал гіпохолічний. Що може бути причиною такого стану?

Незбалансована дієта

Запалення слизової оболонки тонкої кишки

Недостатність панкреатичної ліпази

Обтурація жовчних шляхів

Недостатність панкреатичної фосфоліпази

1069 / 6854
У хворої жінки після парентерального введення гормону відбулося підвищення артеріального тиску і також підвищилися рівні глюкози та ліпідів у крові. Який гормон було введено?

Фолікулін

Прогестерон

Інсулін

Адреналін

Глюкагон

1070 / 6854
У дівчинки 7-ми років ознаки анемії. Лабораторно встановлений дефіцит піруваткінази в еритроцитах. Порушення якого процесу грає головну роль в розвитку анемії у дівчинки?

Дезамінування амінокислот

Окислювальне фосфорилювання

Анаеробний гліколіз

Розклад пероксидів

Тканинне дихання

1071 / 6854
У дитини 5-ти років, що часто хворіє на респіраторні захворювання, відмічаються екзематозні явища після вживання деяких харчових продуктів, схильність до затяжного перебігу запальних процесів. Який вид діатезу можна припустити в даному випадку?

Геморагічний

Ексудативно-катаральний

Лімфатико-гіпопластичний

Астенічний

Нервово-артритичний

1072 / 6854
Хворий сонливий, свідомість потьмарена, реакції на сильні подразники загальмовані. Шкіра бліда, суха, виражені набряки. М’язові фібрилярні посмикування. Мідріаз. Дихання типу Чейна-Стокса з аміачним запахом. При аускультації серця - шум тертя перикарда. Який вид коми розвинувся в хворого?

Печінкова.

Ниркова

Кетоацидотична

Апоплексична

Гіперосмолярна

1073 / 6854
Дитина 10-ти місячного віку, батьки якої брюнети, має світлє волосся, дуже світлу шкіру та блакитні очі. Зовнішньо при народженні виглядала нормально, але протягом останніх 3 місяців спостерігалися порушення мозкового кровообігу, відставання в розумовому розвитку. Причиною такого стану може бути:

Гістидинемія

Гостра порфірія

Галактоземія

Глікогеноз

Фенілкетонурія

1074 / 6854
У грудної дитини спостерігається забарвлення склер, слизових оболонок. Виділяється сеча, яка темніє на повітрі. В крові та сечі виявлено гомогентизинову кислоту. Що може бути причиною даного стану?

Альбінізм

Цистинурія

Алкаптонурія

Галактоземія

Гістидинемія

1075 / 6854
У мазку дуоденального вмісту хворого з розладом травлення виявлено найпростіших розміром 10-18 мкм. Тіло грушоподібної форми, 4 пари джгутиків, у розширеній передній частині тіла два ядра, які розміщені симетрично. Який вид найпростіших найбільш вірогідний?

Балантидій

Кишкова амеба

Дизентерійна амеба

Трихомонада

Лямблія

1076 / 6854
У потерпілого травма м’яких тканин голови та тім’яних кісток в ділянці їх з’єднання між собою, яка супроводжується сильною кровотечею. Яке судинне утворення пошкоджене?

Sinus petrosus superior

Sinus sagittalis superior

Sinus transversus

Sinus sagittalis inferior

Sinus rectus

1077 / 6854
Жінка 44-х років скаржиться на загальну слабкість, біль у ділянці серця, значне збільшення маси тіла. Об’єктивно: обличчя місяцеподібне, гірсутизм, АТ- 165/100 мм рт.ст., зріст -164 см, вага - 103 кг; переважно накопичення жиру на шиї, верхньому плечовому поясі, животі. Що є основним патогенетичним механізмом ожиріння у жінки?

Підвищення продукції інсуліну

Підвищення продукції мінералокортикоїдів

Зниження продукції глюкагону

Підвищення продукції глюкокортикоїдів

Зниження продукції тиреоїдних гормонів

1078 / 6854
У дитини 3-х років після перенесеної важкої вірусної інфекції відзначаються повторне блювання, непритомність, судоми. При дослідженні виявлена гіперамоніємія. З чим може бути пов’язана зміна біохімічних показників крові у цієї дитини?

Активація процесів декарбоксилювання амінокислот

Пригнічення активності ферментів трансамінування

Порушення знешкодження біогенних амінів

Порушення знешкодження аміаку в орнітиновому циклі

Посилення гниття білків у кишечникУ

1079 / 6854
На гістологічному препараті в сполучній тканині знайдено великі клітини, заповнені базофільною метахроматичною зернистістю; гістохімічно встановлено, що гранули містять гепарин та гістамін. Які клітини найбільш вірогідно знайдено в препараті?

Фібробласти

Тучні клітини

Макрофаги

Плазмоцити

Адипоцити

1080 / 6854
Альбіноси погано переносять вплив сонця - засмага не розвивається, а з’являються опіки. Порушення метаболізму якої амінокислоти лежить в основі цього явища?

Метіонін

Триптофан

Фенілаланін

Гістидин

Глутамінова

1081 / 6854
У хворого 25-ти років з численних шкірних пустул висівається золотистий стафілокок в асоцiацiї з епідермальним стафілококом. В аналiзi харкотиння виявлена пневмоциста карінії, у випорожненнях - криптоспоридії, вульгарний протей та гриби роду кандіда. При якому захворюванні зустрічається таке множинне інфікування умовно-патогенними мікроорганізмами?

Дисбактерiоз

Медикаментозний агранулоцитоз

Сепсис

Цукровий діабет

СНІД

1082 / 6854
До терапевтичного відділєння надійшов хворий з тривалою бронхопневмонією. Антибіотикотерапія не дала належного ефекту. Який препарат для підвищення імунного статусу слід додати до комплексної терапії цього хворого?

Димедрол

Тималін

Парацетамол

Анальгін

Сульфокамфокаїн

1083 / 6854
У пацієнта цироз печінки. Дослідження якої з перелічених речовин, що екскретуються з сечею, може характеризувати стан антитоксичної функції печінки?

Гіпурова кислота

Сечова кислота

Креатинін

Амінокислоти

Амонійні солі

1084 / 6854
Під час розтину трупа чоловіка 60-ти років у міокарді передньої стінки лівого шлуночка виявлене сіре неправильної форми щільне вогнище 5х4 см з чіткими межами, волокнистої структури. Який діагноз найбільш вірогідний?

Кардіоміопатія

Інфаркт

Післяінфарктний міокардіосклероз

Міокардит

Дифузний дрібновогнищевий міокардіосклероз

1085 / 6854
При аналізі спірограми в обстежуваного встановлено зменшення частоти та глибини дихання. До зменшення якого показника це призведе?

Резервний об’єм видиху

Хвилинний об’єм дихання

Життєва ємність легень

Залишковий об’єм

Резервний об’єм вдиху

1086 / 6854
У внутрішньоутробному періоді розвитку в судинній системі плода функціонує крупна артеріальна (боталова) протока, яка після народження перетворюється в lig. arteriosum. Що з’єднує ця протока?

Праве та ліве передсердя

Легеневий стовбур та верхню порожнисту вену

Аорту та нижню порожнисту вену

Аорту та верхню порожнисту вену

Легеневий стовбур та аорту

1087 / 6854
Хворий під час курсу лікування метронідазолом вжив невелику кількість алкоголю, внаслідок чого розвинулось тяжке отруєння. Поясніть причину цього:

Невралгічні розлади

Алергічна реакція

Порушення функції нирок

Накопичення ацетальдегіду

Серцево-судинна недостатність

1088 / 6854
До пологового відділення надійшла вагітна жінка зі слабкою пологовою діяльністю. Призначте засіб гормональної природи для посилення пологової діяльності:

Окситоцин

Гідрокортизон

Прогестерон

Метандростенолон

АКТГ

1089 / 6854
Оперуючи молочну залозу, хірурги надають перевагу радіальним розрізам. З особливостями будови яких анатомічних структур пов’язана така техніка операції?

Часточки залози розташовані поперечно

Часточки залози розташовані вертикально

Верхівки часточок сходяться до соска

Основи часточок звернені до соска

1090 / 6854
У пацієнта, який суворо виконував рекомендації щодо дотримання певної дієти протягом 10 днів, було проведено дослідження величини дихального коефіцієнта. Результат: ДК=1,0. Якої дієти дотримувався пацієнт?

Змішана

З переважним вмістом вуглєводів

З переважним вмістом жирів та вуглеводів

З переважним вмістом білків та жирів

З переважним вмістом білків та вуглеводів

1091 / 6854
У мікропрепараті спинного мозку необхідно проаналізувати стан ядра, нейрони якого утворюють моторні закінчення в скелетній мускулатурі. Про яке ядро спинного мозку йде мова?

Власне ядро переднього рога

Проміжне латеральне ядро

Власне ядро сірої речовини

Власне ядро заднього рога

Грудне ядро

1092 / 6854
У процесі розвитку дитини хребет поступово набув два лордози та два кіфози. Це пояснюється розвитком здатності до:

Прямоходіння

Сидіння

Лежання

Повзання

Плавання

1093 / 6854
При лабораторному дослідженні сечі в нормі в ній не виявляються формені елементи крові. Яка структура нефрону найбільше перешкоджає їх надходженню до первинної сечі?

Юкставаскулярні клітини

Базальна мембрана капілярів клубочка

Епітелій петлі Іенле

Епітелій зовнішнього листка капсули клубочка

Мезангіальні клітини

1094 / 6854
На розтині тіла жінки 27-ми років, що померла від хронічної ниркової недостатності, виявлені множинні рубці та інфаркти в селезінці, нирках. Під час мікроскопічного дослідження виявлені зміни в артеріях середнього та дрібного калібру, які характеризуються склерозом стінок, помірною проліферацією ендотелію і вираженою лімфогістіоцитарною інфільтрацією навколо. На яке захворювання страждала померла?

Атеросклероз

Гіпертонічна хвороба

Вісцеральний сифіліс

Хвороба Марфана

Вузликовий періартеріїт

1095 / 6854
Хворому з облітеруючим ендартеріїтом проведена гангліонарна симпатектомія. Позитивний лікувальний ефект цієї операції пов’язаний з розвитком артеріальної гіперемії нижніх кінцівок. Як визначається ця гіперемія за механізмом розвитку?

Метаболічна

Нейропаралітична

Реактивна

Нейротонічна

Робоча

1096 / 6854
Під час аналізу електрокардіограми встановлено, що тривалість серцевого циклу в людини дорівнює 1 секунді. Яка частота серцевих скорочень за хвилину?

100

80

50

60

70

1097 / 6854
Нервово-м’язовий препарат жаби обробили отрутою. Після цього зберігається здатність м’яза до скорочення у відповідь на пряму стимуляцію, але втрачається у відповідь на стимуляцію нерва. Що блокує ця отрута?

Натрієві канали

Нервово-м’язовий синапс

Процеси енергоутворення

Спряження збудження та скорочення в м’язі

Калієві канали

1098 / 6854
У місті епідемія грипу. Який препарат доцільно використати для неспецифічної профілактики захворювання?

Протигрипозна сироватка

Лейкоцитарний інтерферон

Протигрипозна вакцина

Протигрипозний імуноглобулін

Пеніцилін

1099 / 6854
Швидкою допомогою до приймального відділення доставлений хворий з кривавим блюванням. В анамнезі - цироз печінки. Пошкодження яких вен найбільш вірогідне у даному випадку?

Верхня брижова

Селезінкова

Стравохідні

Нижня брижова

Печінкові

1100 / 6854
Хворий скаржиться на біль при рухах у лівій половині шиї. Найкраще положення, при якому хворий не відчуває болю - нахил шиї вліво з одночасним підняттям голови і поверненням обличчя в протилежний бік. Функція якого м’яза порушена?

M.trapezius зліва

M.sternocleidomastoideus з правого боку

M.sternocleidomastoideus зліва

M.trapezius з правого боку

M.sternohyoideus

1101 / 6854
У людини вимірюють внутрішньо-плевральний тиск. У якій фазі людина затримала дихання, якщо величина тиску дорівнює - 7,5 см вод.ст.?

Форсований вдих

Спокійний вдих

Форсований видих

Спокійний видих

1102 / 6854
Жінка 49 років звернулася до лікаря зі скаргами на підвищену втомлюваність та появу задишки під час фізичного навантаження. На ЕКГ: ЧСС-50/хв.; PQ- подовжений, QRS- не змінений, кількість зубців Р перевищує кількість комплексів QRS. Який вид аритмії у пацієнтки?

Екстрасистолія

Миготлива аритмія

Синусова брадикардія

Синоатріальна блокада

Атріовентрикулярна блокада

1103 / 6854
На препараті одного з відділів дихальної системи виявлений трубчастий орган, в якому визначається невисокий епітелій, добре розвинена м’язова оболонка, відсутні залози та хрящ. Назвіть цей орган:

Великі бронхи

Середні бронхи

Малі бронхи

Гортань

Трахея

1104 / 6854
На препараті яєчника, забарвленому гематоксиліном-еозіном, визначається фолікул, в якому клітини фолікулярного епітелію розміщені в 1-2 шари та мають кубічну форму, навколо овоциту видно оболонку яскраво-червоного кольору. Назвіть цей фолікул:

Первинний

Вторинний

Примордіальний

Зрілий

Атретичний

1105 / 6854
Для розвитку гарячкових станів характерним є зростання рівня білків 'гострої фази' церулоплазміну, фібриногену, С-реактивного протеїну. Вкажіть можливий механізм цього явища:

Проліферативна дія ІЛ-2 на Т-лімфоцити

Руйнівна дія підвищеної температури на клітини організму

Дегрануляція тканинних базофілів

Стимулюючий вплив ІЛ-1 на гепатоцити

1106 / 6854
Хворий протягом двох тижнів отримував медикаментозну терапію з приводу психозу. Стан хворого поліпшився, однак невдовзі з’явилися ригідність, тремор, гіпокінезія. Який з перелічених препаратів викликає вказані ускладнення?

Імізін

Сиднокарб

Аміназин

Дифенін

Хлордіазепоксид

1107 / 6854
При огляді лікарем у хворого виявлена стегнова кила. Через яке анатомічне утворення вона виходить під шкіру?

Canalis adductorius

Hiatus saphenus

Anulus femoralis

Anulus inguinalis superficialis

Lacuna musculorum

1108 / 6854
Внаслідок впливу у-випромінювання ділянка ланцюга ДНК повернулася на 180 градусів. Яка з перелічених видів мутацій відбулася в ланцюзі ДНК?

Реплікація

Транслокація

Інверсія

Дуплікація

Делеція

1109 / 6854
В експерименті збільшили проникність мембрани збудливої клітини для іонів калію. До яких змін мембранного потенціалу це призведе?

Потенціал дії

Змін не буде

Локальна відповідь

Деполяризація

Гіперполяризація

1110 / 6854
При розтині тіла чоловіка, який тривалий час зловживав алкоголем, печінка малих розмірів, щільна, дрібно-бугриста. Мікроскопічно: псевдочасточки дрібні, розподілені вузькими прошарками сполучної тканини з лімфомакрофагальними інфільтратами; гепатоцити в стані великокрапельної жирової дистрофії. Який з перелічених діагнозів найбільш вірогідний?

Хронічний персистуючий алкогольний гепатит

Жировий гепатоз

Токсична дистрофія печінки

Хронічний активний алкогольний гепатит

Алкогольний цироз

1111 / 6854
У чоловіка виявлено гонорею за даними бактеріоскопії мазка з уретри. Враховуючи, що препаратами вибору для лікування гонореї є фторхінолони, хворому необхідно призначити:

Фторурацил

Цефазолін

Ципрофлоксацин

Фуразолідон

Уросульфан

1112 / 6854
Хворий з ревматоїдним артритом приймає диклофенак-натрій. Після чергового аналізу крові лікар відмінив цей препарат. Яке ускладнення з боку крові викликав диклофенакнатрій?

Зниження гемокоагуляції

Лейкопенія

Лейкоцитоз

Еозинофілія

Посилення гемокоагуляції

1113 / 6854
У хворого, який тривалий час приймав тетрациклін, виник кандидоз слизових оболонок. Який лікарський препарат слід призначити для його лікування?

Гризеофульвін

Фурадонін

Ітраконазол

Нітрофунгін

Амфотеріцин

1114 / 6854
Хворий 40-а років висуває скарги на сильне серцебиття, пітливість, нудоту, порушення зору, тремор рук, підвищення артеріального тиску. З анамнезу: 2 роки тому було встановлено діагноз феохромоцитома. Гіперпродукція яких гормонів зумовлює цю патологію?

Альдостерон

Глюкокортикоїди

Катехоламіни

Тиреоїдні гормони

АКТГ

1115 / 6854
Хвора 56-ти років тривалий час хворіє на тиреотоксикоз. Який тип гіпоксії може розвинутися у цієї хворої?

Гемічна

Дихальна

Циркуляторна

Тканинна

Змішана

1116 / 6854
У хворої симптоми запального процесу сечостатевих шляхів. У мазку із слизової оболонки піхви виявлено великі одноклітинні організми грушоподібної форми з загостреним шипом на задньому кінці тіла, великим ядром та ундулюючою мембраною. Які найпростіші знайдені в мазку?

Trichomonas vaginalis

Trichomonas hominis

Trichomonas buccalis

Lamblia intestinalis

Trypanosoma gambiense

1117 / 6854
Ціаніди є надзвичайно потужними клітинними отрутами, які при надходженні до організму людини можуть спричинити смерть. Блокування якого ферменту тканинного дихання лежить в основі такої їх дії?

Цитохромоксидаза

Ферохелатаза

Гемоглобінредуктаза

Каталаза

Глюкозо-6-фосфатдегідрогеназа

1118 / 6854
Під час гістологічного дослідження тимуса чоловіка 40-а років, визначено зменшення частки паренхіматозних елементів залози, збільшення частки жирової та пухкої сполучної тканини, збагачення її тимусними тільцями при незмінній загальній масі органа. Як зветься таке явище?

Атрофія

Акцідентальна інволюція

Гіпотрофія

Вікова інволюція

Дистрофія

1119 / 6854
У хворого спостерігається збільшений тонус артеріол за нормальних показників насосної функції серця. Як це вплине на величину артеріального тиску?

Зменшиться переважно систолічний

Зменшиться переважно діастолічний

Тиск не зміниться

Зросте переважно систолічний

Зросте переважно діастолічний

1120 / 6854
Хворий 50-ти років страждає на гіпертонічну хворобу. Під час фізичного навантаження у нього з’явилися відчуття м’язової слабкості, нестачі повітря, синюшність губ, шкіри, обличчя; дихання супроводжувалося відчутними на відстані вологими хрипами. Який механізм лежить в основі виникнення такого синдрому?

Хронічна лівошлуночкова недостатність

Гостра лівошлуночкова недостатність

Тампонада серця

Хронічна правошлуночкова недостатність

Колапс

1121 / 6854
У хворого 58-ми років з гострою серцевою недостатністю спостерігається зменшення добової кількості сечі - олігурія. Який механізм цього явища?

Зниження проникності ниркового фільтру

Зниження онкотичного тиску крові

Підвищення гідростатичного тиску крові в капілярах

Зниження клубочкової фільтрації

Зниження кількості функціонуючих клубочків

1122 / 6854
Хворий 32-х років надійшов до стаціонару зі скаргами на загальне нездужання, блювання, біль праворуч внизу живота. Після огляду лікарем був поставлений діагноз - гострий апендицит. Який вид болю в хворого?

Фантомний

Соматичний поверхневий ранній

Вісцеральний

Соматичний поверхневий пізній

Соматичний глибокий

1123 / 6854
У новонародженого фізіологічна жовтяниця. Рівень вільного білірубіну в крові значно перевищує норму. Нестачею якого ферменту це обумовлено?

Аденозиндезаміназа

УДФ-глюкуронілтрансфераза

Гем-оксигеназа

Ксантиноксидаза

Трансаміназа

1124 / 6854
У бактеріологічній лабораторії проводиться дослідження м’ясних консервів на вміст ботулінічного токсину. Для цього дослідній групі мишей ввели екстракт із досліджуваного матеріалу та антитоксичну протиботулінічну сироватку типів А, В, Е; контрольній групі мишей ввели екстракт без протиботулінічної сироватки. Яку серологічну реакцію було використано?

Нейтралізації

Зв’язування комплементу

Преципітації

Подвійної імунної дифузії

Опсонофагоцитарна

1125 / 6854
У хворого на цукровий діабет після введення інсуліну настала непритомність, спостерігаються судоми. Який результат біохімічного аналізу крові на вміст цукру?

5,5 ммоль/л

1,5 ммоль/л

3,3 ммоль/л

8 ммоль/л

10 ммоль/л

1126 / 6854
При зовнішньому огляді новонародженого виявлені виражені зміни шкірних покривів усього тіла. Шкіра суха, тьмяна, з нерівною поверхнею та з наявністю сірих пластин, які відшаровуються. З яким видом дистрофії пов’язана ця патологія?

Фібриноїдне набухання

Рогова

Гідропічна

Мукоїдне набухання

Гіаліново-крапельна

1127 / 6854
Лікування туберкульозу здійснюється за допомогою комбінованої хіміотерапії, що включає речовини різного механізму дії. Яке з протитуберкульозних засобів пригнічує в мiкобактерiях транскрипцію РНК на ДНК?

Стрептоміцин

Ізоніазид

Рiфампiцин

Етіонамід

ПАСК

1128 / 6854
Чоловіка 45-ти років протягом останніх 3-х років непокоїв сухий кашель, наростала задишка, легенева недостатність, швидко втрачалася вага. На розтині: легеневе серце; у легенях різко виражений фіброз з наявністю порожнин, що створюють картину 'медових сот'. Гістологічно: інтерстиційний фіброз з вираженою інфільтрацією строми лімфогістіоцитами з домішками нейтрофілів. Який найбільш вірогідний діагноз?

Післязапальний пневмосклероз

Фіброзуючий альвеоліт

Пиловий пневмосклероз

Хронічна бульозна емфізема

Бронхоектатична хвороба

1129 / 6854
У хворого вдень піднялася температура тіла до 39oС і через 6 годин повернулася до норми. На другу добу напад повторився: в період пароксизму температура досягла 41oС, період апірексії настав через 8 годин. Як називається такий тип температурної кривої?

Зворотній

Постійний

Септичний

Гектичний

Інтермітуючий

1130 / 6854
Травма мозку викликала підвищене утворення аміаку. Яка амінокислота бере участь у видаленні аміаку з мозкової тканини?

Валін

Глутамінова

Тирозин

Лізин

Триптофан

1131 / 6854
Хвора 40-а років надійшла до інфекційного відділення лікарні з високою температурою тіла. Об’єктивно: виражені менінгеальні симптоми. Проведено спинномозкову пункцію. Яке анатомічне утворення було пропунктовано?

Spatium epidurale

Spatium subarachnoideum

Cavum trigeminale

Cisterna cerebellomedullaris posterior

Spatium subdurale

1132 / 6854
У вагітної жінки 29-ти років лікар визначив розміри таза. За допомогою циркуля була виміряна відстань між двома верхніми передніми клубовими остями. Який розмір великого таза був визначений?

Conjugata vera

Distantia cristarum

Distantia spinarum

Distantia trochanterica

Conjugata anatomica

1133 / 6854
Після проведення туберкулінової проби (проба Манту) у дитини через 48 годин на місці ведення туберкуліну утворилася папула до 10 мм у діаметрі. Який механізм гіперчутливості лежить в основі розвитку вказаних змін?

Антитілозалежна цитотоксичність

Анафілаксія

Клітинна цитотоксичність

Імунокомплексна цитотоксичність

Гранулематоз

1134 / 6854
У людини хірургічно видалили ушкоджену патологічним процесом дистальну чверть тонкої кишки. Як це позначиться на всмоктуванні поживних речовин при звичайному харчовому раціоні?

Зменшиться всмоктування води

Зменшиться всмоктування вуглеводів

Всмоктування не зміниться

Зменшиться всмоктування білків

Зменшиться всмоктування жирів

1135 / 6854
У новонародженого хлопчика під час огляду зовнішніх статевих органів виявлена розщілина сечівника, яка відкривається на нижній поверхні статевого члена. Про яку аномалію йдеться?

Епіспадія

Монорхізм

Крипторхізм

Гіпоспадія

Гермафродитизм

1136 / 6854
У ході катаболізму гістидину утворюється біогенний амін, що має потужну судинорозширюючу дію. Назвіть його:

Норадреналін

Серотонін

Дофамін

Гістамін

ДОФА

1137 / 6854
У чоловіка швидкість клубочкової фільтрації 80 мл/хв (норма - 125 ± 25 мл/хв). Збільшення якого показника може бути причиною цього?

Нирковий кровотік

Онкотичний тиск плазми крові

Гідростатичний тиск крові в капілярах клубочків

Проникність ниркового фільтру

Ефективний фільтраційний тиск

1138 / 6854
Хворому внутрішньовенно ввели гіпертонічний розчин глюкози. Це підсилить рух води:

З міжклітинної рідини до капілярів

З клітин до міжклітинної рідини

З капілярів до міжклітинної рідини

З міжклітинної рідини до клітин

Змін руху води не буде

1139 / 6854
Хвора 45-ти років звернулася до лікаря з скаргами на порушення сну, що проявлялося погіршенням засинання та пробудженнями серед ночі. Лікар призначив хворій снодійний засіб. Який вид фармакотерапії використав лікар?

Патогенетична

Рефлекторна

Вибіркова

Етіотропна

Симптоматична

1140 / 6854
Для запобігання віддалених результатів чотиридобової малярії пацієнту 42-х років призначили примахін. Вже на 3 добу від початку лікування терапевтичними дозами препарату в пацієнта з’явилися біль у животі та в ділянці серця, диспепсичні розлади, загальний ціаноз, гемоглобінурія. Що стало причиною розвитку побічної дії препарату?

Кумуляція лікарського засобу

Сповільнення екскреції препарату з сечею

Потенціювання дії іншими препаратами

Зниження активності мікросомаль-них ферментів печінки

Генетична недостатність глюкозо-6-фосфат-дегідрогенази

1141 / 6854
До ендокринолога звернулася хво- ра 45-ти років із скаргами на підвищений апетит, сухість слизових оболонок ротової порожнини, зростання діурезу. При обстеженні вперше виявлено інсулінонезалежний діабет. Який з названих препаратів доцільно призначити хворій?

Адіурекрин

Вазопресин

Глібенкламід

Інсулін

Окситоцин

1142 / 6854
У хворого спостерігається зниження тактильної та смакової чутливості слизової оболонки задньої третини спинки язика. З патологією якого нерва це пов’язано?

III гілка трійчастого нерва

Під’язиковий

Язикоглотковий

Лицевий

II гілка трійчастого нерва

1143 / 6854
На 8-й день після введення протиправцевої сироватки з приводу брудної рани стопи у пацієнта підвищилася температура тіла до 380 С, з’явилися біль у суглобах, висипка, свербіж. У крові - лейкопенія і тромбоцитопенія. Який тип алергічної реакції розвинувся?

Імунокомплексна

Цитотоксична

Стимулююча

Гіперчутливість уповільненого типу

Анафілактична

1144 / 6854
Юнак 17-ти років захворів гостро, температура тіла підвищилася до 38,5o С, з’явилися кашель, нежить, сльозотеча, виділення з носу. Яке запалення розвинулося у юнака?

Катаральне

Фібринозне

Гнійне

Серозне

Геморагічне

1145 / 6854
У добовій сечі хворого знайдені вилужені еритроцити. Для якої патології нирок найбільш притаманний виявлений симптом?

Нирковокам’яна хвороба

Пієлонефрит

Дифузний гломерулонефрит

Гостра ниркова недостатність

Нефротичний синдром

1146 / 6854
При аналізі родоводу лікар-генетик встановив, що хвороба зустрічається в осі6 чоловічої та жіночої статей, не в усіх поколіннях, і що у здорових батьків можуть народжуватися хворі діти. Який тип успадкування хвороби?

Х-зчеплений домінантний

Аутосомно-домінантний

Х -зчеплений рецесивний

Y-зчеплений

Аутосомно-рецесивний

1147 / 6854
Хворий на бронхіальну астму не повідомив лікаря, що в нього бувають напади стенокардії. Лікар призначив препарат, після прийому якого напади бронхіальної астми стали рідшими, однак, почастішали напади стенокардії. Який препарат був призначений?

Ізадрин

Сальбутамол

Еуфілін

Фенотерол

Кромолін-натрій

1148 / 6854
Хворий, що лікувався з приводу неврозу сибазоном, скаржиться на зубний біль. Лікар призначив йому знеболювальний засіб у дозі, яка менша за середню терапевтичну. Яке явище взяв до уваги лікар, зменшуючи дозу препарату?

Потенціювання

Лікарська залежність

Сумація

Кумуляція

Толерантність

1149 / 6854
Запальний процес видозміненої підсерозної основи навколо шийки матки спричинив інтенсивний больовий симптом у пацієнтки. Патологічний процес якої ділянки статевих органів встановив лікар?

Perimetrium

Endometrium

Mesometrium

Myometrium

Parametrium

1150 / 6854
При дослідженні амніотичної рідини, одержаної під час амніоцентезу (прокол амніотичної оболонки), виявлені клітини, ядра яких містять статевий хроматин (тільце Барра). Про що це може свідчити?

Розвиток плода жіночої статі

Трисомія

Генетичні порушення розвитку плода

Поліплоїдія

Розвиток плода чоловічої статі

1151 / 6854
На гістологічному препараті, забарвленому орсеїном, у середній оболонці судини виявлено від 40 до 60 вікончастих еластичних мембран. Назвіть цю судину:

Артерія м’язового типу

Вена безм’язового типу

Артерія змішаного типу

Артерія еластичного типу

Вена м’язового типу

1152 / 6854
У людей після тривалого фізичного навантаження виникає інтенсивний біль у м’язах. Які зміни в м’язах є найбільш вірогідною причиною цього?

Підвищена збудливість

Посилений розпад білків

Накопичення молочної кислоти

Накопичення креатиніну

Підвищення вмісту АДФ

1153 / 6854
Під час операції у пацієнта було видалено частину легені, яка вентилюється бронхом третього порядку, що супроводжується гілками легеневої артерії та інших судин. Яка частина легені була видалена?

Сегмент легені

Середня частка

Верхня частка

Нижня частка

Легенева часточка

1154 / 6854
При спадковій оротацидурії виділення оротової кислоти в багато разів перевищує норму. Синтез яких речовин буде порушений при цій патології?

Сечова кислота

Сечовина

Піримідинові нуклеотиди

Пуринові нуклеотиди

Біогенні аміни

1155 / 6854
На розтині тіла чоловіка 46-ти років на слизовій оболонці прямої та сигмоподібної кишок виявлено множинні коричнево-зелені нашарування, крововиливи; у просвіті кишки слиз, невелика кількість крові; гістологічно - фібринозний коліт. При бактеріологічному дослідженні вмісту кишки висіяна S.Sonne. Який найбільш вірогідний діагноз?

Хвороба Крона

Дизентерія

Холера

Сальмонельоз

Ієрсиніоз

1156 / 6854
Чоловік 40-а років протягом 10 років після перелому великогомілкової кістки страждав на хронічний остеомієліт. Три роки тому з’явився нефротичний синдром. Смерть настала від уремії. На секції: нирки щільні, білі, з рубцями в кірковому шарі на розрізі з сальним блиском. Яка патологія нирок розвинулася?

Первинний амілоїдоз

Хронічний гломерулонефрит

Ідіопатичний амілоїдоз

Хронічний пієлонефрит

Вторинний амілоїдоз

1157 / 6854
Пацієнтка 58-ми років скаржиться на підвищену втомлюваність, зниження працездатності, сонливість, задишку під час швидкої ходи. У крові: ер.-4,6•1012/л, Hb- 92 г/л, КП- 0,6. У мазку крові - велика кількість анулоцитів та мікроцитів. Для якої анемії це характерно?

Перніціозна

Гемолітична

Постгеморагічна

Залізодефіцитна

Серповидноклітинна

1158 / 6854
У жінки 30-ти років виникли набряки обличчя. При обстеженні виявлені протеїнурія (5,87 г/л), гіпопротеїнемія, диспротеїнемія, гіперліпідемія. Для якого стану характерно таке поєднання симптомів?

Хронічний пієлонефрит

Нефритичний синдром

Хронічна ниркова недостатність

Гостра ниркова недостатність

Нефротичний синдром

1159 / 6854
Хворий багато років страждав на бронхіальну астму та помер від нападу ядухи. Під час гістологічного дослідження легень виявлено: в просвіті бронхіол та дрібних бронхів багато слизу з домішкою еозинофілів, склероз міжальвеолярних перетинок, розширення просвіту альвеол. Який механізм розвитку реакції гіперчутливості мав місце?

Імунокомплексна реакція

Цитотоксична реакція

Гранулематоз

Реагінова реакція

Цитоліз, обумовлений лімфоцитами

1160 / 6854
У хворого знижений синтез вазопресину, що призводить до поліурії і, як наслідок, до вираженої дегідратації організму. У чому полягає механізм розвитку поліурії?

Зниження реабсорбції глюкози

Зниження канальцевої реабсорбції білку

Збільшення швидкості клубочкової фільтрації

Зниження канальцевої реабсорбції води

Зниження канальцевої реабсорбції іонів Na

1161 / 6854
У медичній практиці для профілактики алкоголізму широко використовують тетурам, який є інгібітором альдегіддегідрогенази. Підвищення в крові якого метаболіту викликає відразу до алкоголю?

Метанол

Малоновий альдегід

Ацетальдегід

Пропіоновий альдегід

Етанол

1162 / 6854
До клініки надійшла дитина 1 року з ознаками ураження м’язів. Після обстеження виявлений дефіцит карнітину в м’язах. Порушення якого процесу є біохімічною основою цієї патології?

Утилізація молочної кислоти

Субстратне фосфорилювання

Синтез актину та міозину

Транспорт жирних кислот у мітохондрії

Регуляція рівня Ca2+ в мітохондріях

1163 / 6854
В експерименті електричними імпульсами подразнюють нерв, що призводить до виділення привушною залозою великої кількості рідкої слини. Який нерв стимулюють?

N.trigeminus

N. facialis

N.vagus

N.glossopharyngeus

N.sympathicus

1164 / 6854
До щелепно-лицьового відділення надійшов хворий з переломом нижньої щелепи. Було вирішено з’єднання кісток провести хірургічним методом під наркозом. Після внутрішньовенного введення міорелаксанту спостерігались короткочасні фібрилярні скорочення м’язів обличчя хворого. Який міорелаксант було застосовано?

Дитилін

Меліктин

Діазепам

Тубокурарина хлорид

Пипекуроній бромід

1165 / 6854
У хворого відзначається атрофія альвеолярних відростків щелепи після видалення зубів. Це є прикладом:

патологічної реакції

патологічного процесу

патологічного стану

хвороби

структурного сліду адаптації

1166 / 6854
Під час гістологічного дослідження слизової оболонки матки знайдено звивисті залози, пилко- та штопороподібні, подовжені, розростання строми з гіперплазією її клітин. Який найбільш вірогідний діагноз?

Лейоміома

Плацентарний поліп

Пухирний занос

Гострий ендометрит

Залозиста гіперплазія ендометрія

1167 / 6854
При лабораторному дослідженні крові пацієнта 44-х років виявлено, що вміст білків в плазмі становить 40 г/л. Як це впливає на транскапілярний обмін води?

Зменшуються фільтрація та реабсорбція

Збільшується фільтрація, зменшується реабсорбція

Обмін не змінюється

Зменшується фільтрація, збільшується реабсорбція

Збільшуються фільтрація та реабсорбція

1168 / 6854
В гістологічному препараті органу ротової порожнини видно, що передня поверхня вистелена багатошаровим плоским незроговілим епітелієм, а задня поверхня - багаторядним війчастим епітелієм. Що це за орган?

Ясна

Щока

Тверде піднебіння

М’яке піднебіння

Губа

1169 / 6854
Хворому на гостру серцеву недостатність було введено серцевий глікозид швидкої дії. Який з перелічених засобів було введено?

Целанід

Мілринон

Дигітоксин

Адонізид

Корглікон

1170 / 6854
У нирках досліджуваного збільшена реабсорбція іонів кальцію і зменшена - фосфатних іонів. Впливом якого гормону це зумовлено?

Тирокальцитонін

Альдостерон

Вазопресин

Паратгормон

Гормональна форма вітаміну D3

1171 / 6854
У людини вміст глюкози в крові 15 ммоль/л (поріг реабсорбції - 10 ммоль/л). Наслідком цього буде:

Зменшення секреції вазопресину

Зменшення діурезу

Зменшення реабсорбції глюкози

Глюкозурія

Зменшення секреції альдостерону

1172 / 6854
Людина після травми головного мозку втратила зір. Пошкодження яких зон кори головного мозку може бути причиною цього?

Тім’яна

Потилична

Скронева

Скронева та тім’яна

Лобна

1173 / 6854
При аналізі ЕКГ людини з’ясовано, що у другому стандартному відведенні від кінцівок зубці Т позитивні, їх амплітуда та тривалість відповідає нормі. Вірним є висновок про те, що в шлуночках серця нормально відбувається процес:

Скорочення

Збудження

Розслаблення

Реполяризація

Деполяризація

1174 / 6854
Ізольована клітина серця людини автоматично генерує імпульси збудження з частотою 60 разiв за хвилину. З якої структури серця отримано цю клітину?

Передсердя

Шлуночок

Атріовентрикулярний вузол

Пучок Пса

Синоатрiальний вузол

1175 / 6854
Під час підготовки пацієнта до операції проведено вимірювання тиску в камерах серця. В одній з них тиск протягом серцевого циклу змінювався від 0 до 120 мм рт.ст. Назвіть цю камеру серця:

Правий шлуночок

Праве передсердя

Лівий шлуночок

Ліве передсердя

1176 / 6854
Під час помірного фізичного навантаження хвилинний об’єм крові в досліджуваного становить 10 л/хв. Який об’єм крові проходить у нього за хвилину через судини легень?

4 л/хв

6 л/хв

7 л/хв

10 л/хв

5 л/хв

1177 / 6854
У немовляти, народженої передчасно, частина альвеол не розправилася за рахунок підвищеної еластичної тяги легень. Яким чином можна зменшити цю силу?

Введення глюкози

Відсмоктування рідини з дихальних шляхів

Вдихання чистого кисню

Введення сурфактантів

Штучна вентиляція легень

1178 / 6854
Під час розтину тіла померлого в прямій та сигмоподібній кишках видно дефекти слизової оболонки неправильної форми з нерівними контурами, вони зливаються між собою, залишаючи невеликі островки збереженої слизової оболонки. Про який різновид коліту йдеться?

Гнійний

Катаральний

Виразковий

Фібринозний

Фолікулярний

1179 / 6854
У дитини з гострою кишковою інфекцією швидко розвинулись ознаки зневоднення, з’явилася кров у випорожненнях. Педіатром було запідозрено коліентерит. Яким методом необхідно скористатись для діагностики ентерального ешерихіозу?

Алергічний

Біологічний

Бактеріологічний

Мікроскопічний

Серологічний

1180 / 6854
У жінки 22-х років, при гістологічному дослідженні збільшених шийних лімфатичних вузлів, виявлені вузлики, що складаються переважно з плоских, дещо витягнутих, з блідо забарвленим ядром, гігантських клітин округло-овальної форми з блідо-рожевою цитоплазмою та з ядрами, розташованими на периферії (у вигляді частоколу), а також дрібних округлої форми клітин з вузьким обідком цитоплазми; у центрі деяких вузликів безструктурні маси, забарвлені в блідо-рожевий колір. Для якого захворювання характерні такі зміни?

Саркоїдоз

Лімфогранулематоз

Бруцельоз

Туберкульоз

Неспецифічний гіперпластичний лімфаденіт

1181 / 6854
У хворого під час об’єктивного огляду виявлено: тонка фігура, великий череп, сильно розвинена лобна частина обличчя, короткі кінцівки. Для якого конституціонального типу це характерно?

Дигестивний

Мускульний

Респіраторний

Змішаний

Церебральний

1182 / 6854
Хвора 38-ми років померла під час нападу бронхіальної астми, що не вдалося купірувати. Під час гістологічного дослідження в просвіті бронхів виявлені скупчення слизу, в стінці бронхів багато тучних клітин (лаброцитів), багато з них у стані дегрануляції, а також багато еозинофілів. Який патогенез цих змін у бронхах?

Цитотоксична, цитолітична дія антитіл

Атопія

Гранулематоз

Клітинно обумовлений цитоліз

Імунокомплексний механізм

1183 / 6854
У хворого на тромбофлебіт нижніх кінцівок з’явився біль у грудній клітці, кровохаркання, наростаюча дихальна недостатність, при явищах якої він помер. На розтині діагностовано множинні інфаркти легень. Яка найбільш вірогідна причина їх розвитку в цьому випадку?

Тромбоз гілок легеневої артерії

Тромбоз бронхіальних артерій

Тромбоз легеневих вен

Тромбоемболія гілок легеневої артерії

Тромбоемболія бронхіальних артерій

1184 / 6854
У клітинах мозку лисиці, яка була спіймана в межі міста, виявлені включення у вигляді тілець Бабеша-Негрі. Джерелом якого захворювання є ця тварина?

Кліщовий енцефаліт

Сказ

Інфекційний мононуклеоз

Грип

Вітряна віспа

1185 / 6854
Хворому, що страждає на стенокардію та приймає ізосорбіда мононітрат, було додатково призначено лікарський засіб з дезагрегантним ефектом. Визначте цей препарат:

Ацетилсаліцилова кислота

Анаприлін

Нітрогліцерин

Валідол

Ніфедипін

1186 / 6854
У хворого на обличчі вугрі. Під час мікроскопії зіскрібків із уражених ділянок виявлені живі членистоногі розміром 0,2-0,5 мм, які мають витягнуту червоподібну форму, чотири пари коротких кінцівок, що розташовані в середній частині тіла. Який лабораторний діагноз?

Фтиріоз

Демодекоз

Короста

Педикульоз

Міаз

1187 / 6854
У хворого гостра серцево-легенева недостатність, що супроводжується набряком легень. Який препарат з групи діуретиків необхідно призначити?

Спіронолактон

Діакарб

Фуросемід

Дихлотіазид

Тріамтерен

1188 / 6854
До лікаря звернулася хвора зі скаргами на нежить, який посилюється навесні в період цвітіння рослин. Було встановлено діагноз алергійного риніту. Які зміни лейкоцитарної формули можна очікувати в аналізі крові цієї хворої?

Зсув формули вліво

Еозинопенія

Еозинофілія

Лімфопенія

Лімфоцитоз

1189 / 6854
В крові пацієнта вміст глюкози натщесерце 5,6 ммоль/л, через 1 годину після цукрового навантаження - 13,8 ммоль/л, а через 3 години - 9,2 ммоль/л. Для якої патології характерні такі показники?

Тиреотоксикоз

Здорова людина

Акромегалія

Прихована форма цукрового діабету

Хвороба Іценко-Кушінга

1190 / 6854
До терапевтичного відділення надійшов хворий на виразкову хворобу шлунка з гіперацидним синдромом. Який препарат з перелічених груп лікарських засобів необхідно використати у комплексній терапії пацієнта?

Стероїдні протизапальні засоби

Блокатори Н1-гістамінових рецепторів

Блокатори Н2-гістамінових рецепторів

Нестероїдні протизапальні засоби

Блокатори кальцієвих каналів

1191 / 6854
Чоловік 45-ти років звернувся до лікаря з приводу бляшкоподібного утвору на шиї. В біоптаті шкіри гістологічно виявлено пухлинні клітини, розташовані гніздами, які мають круглу та овальну форму з вузьким ободком базофільної цитоплазми та схожі на клітини базального шару епідермісу. Як називається пухлина в пацієнта?

Сирінгоаденома

Епідермальний рак

Трихоепітеліома

Гідраденома

Базаліома

1192 / 6854
У хворого з діагнозом хвороба Іценка-Кушинга (гіперпродукція гормонів корою наднирників) в крові визначено підвищену концентрацію глюкози, кетонових тіл, натрію. Який біохімічний механізм є провідним у виникненні гіперглікемії?

Глікогеноліз

Гліколіз

Аеробний гліколіз

Глюконеогенез

Глікогенез

1193 / 6854
При цукровому діабеті внаслідок активації процесів окиснення жирних кислот виникає кетоз. До яких порушень кислотно-лужної рівноваги може призвести надмірне накопичення кетонових тіл у крові?

Дихальний ацидоз

Змін не відбуватиметься

Метаболічний ацидоз

Дихальний алкалоз

Метаболічний алкалоз

1194 / 6854
У жінки, що має 0 (I) групу крові, народилася дитина з групою крові AB. Чоловік цієї жінки має групу крові A. Які з наведених нижче видів взаємодії генів пояснюють це явище?

Полімерія

Комплементарність

Епістаз рецесивний

Неповне домінування

Кодомінування

1195 / 6854
У дитини 5-ти років діагностовано хворобу Брутона, яка проявляється у важкому перебігу бактеріальних інфекцій, відсутності В-лімфоцитів та плазматичних клітин. Які зміни вмісту імуноглобулінів будуть спостерігатися в сироватці крові цієї дитини?

Зменшення IgA, IgM

Збільшення IgA, IgM

Зменшення IgD, IgE

Змін не буде

Збільшення IgD, IgE

1196 / 6854
У жінки 30-ти років виявлено недостатність зовнішньосекреторної функції підшлункової залози. Гідроліз яких поживних речовин буде порушений?

Білки

Білки, жири, вуглеводи

Білки, жири

Жири, вуглеводи

Білки, вуглеводи

1197 / 6854
При розслідуванні спалаху внутрішньолікарняної інфекції виділені чисті культури золотистого стафілококу від хворих, медперсоналу та деяких об’єктів навколишнього середовища. Яке дослідження необхідно провести, щоб визначити ідентичність виділених стафілококів і встановити джерело госпітальної інфекції?

Визначення патогенності

Фаготипування

Зараження тварин

Визначення біовару

Серотипування

1198 / 6854
У студента через 2 години після іспиту в аналізі крові виявлено лейкоцитоз без істотних змін у лейкоцитарній формулі. Який найбільш вірогідний механізм розвитку лейкоцитозу?

Посилення лейкопоезу та зменшення руйнування лейкоцитів

Уповільнення міграції лейкоцитів у тканини

Посилення лейкопоезу

Уповільнення руйнування лейкоцитів

Перерозподіл лейкоцитів в організмі

1199 / 6854
У молодої жінки видалили пухлину дистального кінця стегнової кістки, яка швидко зростала. Макроскопічно: пухлина строкатого вигляду - від біло-сірого до коричневочервоного кольору, пухкої консистенції. Мікроскопічно: основний тканинний компонент пухлини представлений кістковими та остеоїдними структурами, вистеленими атиповими остеобластами з патологічними мітозами; безліч тонкостінних судин. Який найбільш вірогідний діагноз?

Остеосаркома

Саркома Юїнга

Остеома

Хондрома

Ангіосаркома

1200 / 6854
У жінки 39-ти років відмічається мокнуття в ділянці соска молочної залози, наявність неглибокої виразки з запальною гіперемією та набряком шкіри. При гістологічному дослідженні біоптату цієї ділянки в ростковому шарі потовщеного епідермісу виявлені атипові великі клітини зі світлою та оптично пустою цитоплазмою, з відсутністю міжклітинних містків. Такі клітини знайдені і в усті великих протоків залози. Який найбільш вірогідний діагноз?

Хвороба Педжета

Базально-клітинний рак

Меланома

Внутрішньопротоковий рак

Плоскоклітинний рак

1201 / 6854
Хворому перед операцією був ведений дитилін (лістенон) і проведена інтубація. Після закінчення операції та припинення наркозу самостійне дихання не відновилося. Дефіцит якого ферменту в організмі хворого подовжує дію м’язового релаксанту?

Карбангідраза

Псевдохолінестераза

Сукцинатдегідрогеназа

N-ацетилтрансфераза

K — Na-АТФ-аза

1202 / 6854
Людина хворіє на цукровий діабет, що супроводжується гіперглікемією натще понад 7,2 ммоль/л. Рівень якого білка плазми крові дозволяє ретроспективно (за попередні 4-8 тижні до обстеження) оцінити рівень глікемії?

Фібриноген

Альбумін

С-реактивний білок

Глікозильований гемоглобін

Церулоплазмін

1203 / 6854
У пацієнта 60-ти років виявлено погіршення сприйняття звуків високої частоти. Порушення стану яких структур слухового аналізатора зумовило ці зміни?

Основна мембрана завитки біля гелікотреми

Євстахієва труба

М’язи середнього вуха

Основна мембрана завитки біля овального віконця

Барабанна перетинка

1204 / 6854
У хворого на цукровий діабет з’явився різкий біль у правій стопі. Об’єктивно: великий палець стопи чорного кольору, тканини стопи набряклі, осередки відшарування епідермісу, виділення з неприємним запахом. Яка клініко-морфологічна форма некрозу розвинулася в хворого?

Секвестр

Інфаркт

Іангрена суха

Пролежень

Гангрена волога

1205 / 6854
Постраждалому з колотою раною передньої стінки шлунка наддається хірургічна допомога. До якого утворення порожнини очеревини потрапив вміст шлунка?

Печінкова сумка

Сальникова сумка

Лівий мезентеріальний синус

Правий мезентеріальний синус

Передшлункова сумка

1206 / 6854
До клініки надійшла дитина 4-х років з ознаками тривалого білкового голодування: затримка росту, анемія, набряки, розумова відсталість. Причиною розвитку набряків у цієї дитини є зниження синтезу:

Глобулінів

Ліпопротеїнів

Альбумінів

Ілікопротеїнів

Гемоглобіну

1207 / 6854
Під час розтину трупа чоловіка, що страждав на крупозну пневмонію і помер від пневмококового сепсису, у правій плевральній порожнині містилося 900 мл каламутної зеленувато-жовтого кольору рідини. Листки плеври тьмяні, повнокровні. Назвіть клінікоморфологічну форму запалення в плевральній порожнині:

Флегмона

Емпієма

Гострий абсцес

Хронічний абсцес

Фібринозне запалення

1208 / 6854
У хворого виявлено зниження вмісту іонів магнію, які потрібні для прикріплення рибосом до гранулярної ендоплазматичної сітки. Відомо, що це призводить до порушення біосинтезу білка. Який саме етап біосинтезу білка буде порушено?

Термінація

Транскрипція

Трансляція

Реплікація

Активація амінокислот

1209 / 6854
При дослідженні сироватки крові хворого виявлене підвищення рівня аланінамінотрансферази (АЛТ) та аспартатамінотрансферази (АСТ). Які зміни на клітинному рівні можуть призвести до подібної ситуації?

Порушення функції енергозабезпечення клітин

Порушення ферментних систем клітин

Порушення міжклітинних взаємовідношень

Руйнування генетичного апарату клітин

Руйнування клітин

1210 / 6854
У юнака енерговитрати збільшились з 500 до 2000 кДж за годину. Що з наведеного може бути причиною цього?

Перехід від сну до бадьорості

Підвищення зовнішньої температури

Розумова праця

Прийом їжі

Фізичне навантаження

1211 / 6854
У хворого виявлено підвищення активності ЛДГ1 ,2 , АсАТ, креатинфосфокінази. В якому органі (органах) найбільш вірогідний розвиток патологічного процесу?

Печінка та нирки

Серцевий м’яз

Сполучна тканина

Скелетні м’язи

Нирки та надниркові залози

1212 / 6854
За даними ВООЗ на малярію щорічно на Землі хворіють приблизно 250 млн. чоловік. Ця хвороба зустрічається переважно у тропічних і субтропічних областях. Межі її розповсюдження співпадають з ареалами комарів роду:

Кулізета

Анофелес

Мансоніа

Аедес

Кулекс

1213 / 6854
У померлого 58-ми років на розтині: мітральний клапан деформований, потовщений, змикається не до кінця. Мікроскопічно: вогнища колагенових волоконець еозінофільні, дають позитивну реакцію на фібрин. Найвірогідніше це:

Амілоїдоз

Гіаліноз

Фібриноїдне набухання

Мукоїдне набухання

Фібринозне запалення

1214 / 6854
Для вивчення локалізації біосинтезу білка в клітинах, миші ввели мічені амінокислоти аланін та триптофан. Біля яких органел буде спостерігатися накопичення мічених амінокислот?

Клітинний центр

Рибосоми

Апарат Гольджі

Гладенька ЕПС

Лізосоми

1215 / 6854
У районах Південної Африки у людей розповсюджена серпоподібно-клітинна анемія, при якій еритроцити мають форму серпа внаслідок заміни в молекулі гемоглобіну амінокислоти глутаміну на валін. Чим викликана ця хвороба?

Трансдукція

Генна мутація

Геномні мутації

Кросинговер

Порушення механізмів реалізації генетичної інформації

1216 / 6854
Хворий помер при явищах серцево-судинної недостатності. Результати розтину: післяінфарктний кардіосклероз, гіпертрофія міокарда і дилятація його порожнин, особливо правого шлуночка. Печінка збільшена, з гладенькою поверхнею, на розрізі повнокровна, з темно-червоними крапками на буруватому фоні тканини. Гістологічно: повнокров’я центральних відділів часточок; у периферичних відділах навколо портальних трактів - гепатоцити у стані жирової дистрофії. Як називаються описані вище зміни печінки?

Несправжньомускатна печінка

Мускатна печінка

Цироз печінки

Стеатоз печінки

Амілоїдоз

1217 / 6854
У померлого від хронічної серцево-судинної недостатності на розтині виявлене 'тигрове серце'. З боку ендокарду помітна жовтувато-біла посмугованість; міокард тьмяний, глинисто-жовтий. Який процес зумовив дану патологію?

Жирова судинно-стромальна дистрофія

Гіаліново-краплинна дистрофія

Вуглеводна дистрофія

Жирова паренхіматозна дистрофія

Амілоїдоз

1218 / 6854
Під час експерименту подразнюють скелетний м’яз серією електричних імпульсів. Який вид м’язового скорочення буде виникати, якщо кожний наступний імпульс надходить у періоді вкорочення попереднього поодинокого м’язового скорочення?

Зубчастий тетанус

Суцільний тетанус

Серія поодиноких скорочень

Контрактура м’яза

Асинхронний тетанус

1219 / 6854
Амоніак є дуже отруйною речовиною, особливо для нервової системи. Яка речовина бере особливо активну участь у знешкодженні амоніаку в тканинах мозку?

Глутамінова кислота

Лізин

Аланін

Гістидин

Пролін

1220 / 6854
Хворий чоловік госпіталізований на 5-й день хвороби з проявами жовтяниці, болем у м’язах, ознобом, носовими кровотечами. Під час проведення лабораторної діагностики бактеріолог виконав темнопольну мікроскопію краплини крові хворого. Назвіть збудника хвороби:

Bartonella bacilloformis

Leptospira interrogans

Borrelia dutlonii

Rickettsia mooseri

Calymmatobacterium granulomatis

1221 / 6854
У хворого поперечний розрив спинного мозку нижче VI грудного сегменту. Як внаслідок цього зміниться дихання?

Зупиниться

Стане більш глибоким

Стане більш рідким

Суттєво не зміниться

Стане більш частим

1222 / 6854
У хворого через 12 годин після гострого нападу загруднинного болю знайдено різке підвищення активності АсАТ у сироватці крові. Вкажіть патологію, для якої характерне це зміщення:

Колагеноз

Інфаркт міокарда

Нецукровий діабет

Вірусний гепатит

Цукровий діабет

1223 / 6854
Людина стоїть у кімнаті в легкому одязі, температура повітря +140 C, вікна і двері зачинені. Яким шляхом вона віддає найбільше тепла?

Випаровування

Перспірація

Теплопроведення

Теплорадіація

Конвекція

1224 / 6854
Жінка 30-ти років хворіє близько року, коли вперше з’явився біль у ділянці суглобів, їх припухлість, почервоніння шкіри над ними. Попередній діагноз - ревматоїдний артрит. Зміна якого компоненту в структурі білка сполучної тканини є однією з причин цього захворювання?

Міозин

Овоальбумін

Колаген

Тропонін

Муцин

1225 / 6854
Хворий з інфекційним мононуклеозом протягом двох тижнів приймав глюкокортикостероїдні препарати. Наступила ремісія, проте в нього виникло загострення хронічного тонзиліту. Результатом якої дії глюкокортикостероїдів є дане ускладнення?

Антитоксична

Імунодепресивна

Антиалергічна

Протишокова

Протизапальна

1226 / 6854
У собаки в досліді подразнювали на шиї периферичний відрізок блукаючого нерва. При цьому спостерігали такі зміни серцевої діяльності:

Збільшення частоти та сили скорочень

Зменшення частоти скорочень

Збільшення швидкості атріовентрикулярного проведення

Збільшення збудливості міокарда

Збільшення сили скорочень

1227 / 6854
У результаті виснажуючої м’язової праці у робочого значно зменшилася буферна ємність крові. Надходження якої речовини у кров може бути причиною цього явища?

1,3-бісфосфогліцерат

Піруват

3-фосфогліцерат

Лактат

1228 / 6854
Хвора доставлена бригадою швидкої допомоги. Об’єктивно: стан важкий, свідомість відсутня, адинамія. Шкірні покриви сухі, запалі очі, ціаноз обличчя, тахікардія, запах ацетону з рота. Результати аналізів: глюкоза крові -20,1 ммоль/л (у нормі - 3,3-5,5 ммоль/л), у сечі - 3,5% (у нормі - 0). Який найбільш вірогідний діагноз?

Анафілактичний шок

Гіперглікемічна кома

Гіпоглікемічна кома

Гостре алкогольне отруєння

Гостра серцева недостатність

1229 / 6854
Жінка 62-х років скаржиться на частий біль у ділянці грудної клітки та хребта, переломи ребер. Лікар припустив мієломну хворобу (плазмоцитому). Який з перерахованих нижче лабораторних показників буде мати найбільше діагностичне значення?

Гіпопротеїнемія

Гіпоглобулінемія

Гіперальбумінемія

Парапротеїнемія

Протеїнурія

1230 / 6854
У новонародженої дитини на пелюшках виявлені темні плями, що свідчать про утворення гомогентизинової кислоти. З порушенням обміну якої речовини це пов’язане?

Галактоза

Холестерин

Метіонін

Тирозин

Триптофан

1231 / 6854
Під час роботи щодо ліквідації наслідків аварії на АЕС, робітник одержав дозу опромінення 500 рентген. Скаржиться на головний біль, нудоту, запаморочення. Які зміни кількості лейкоцитів можна очікувати в хворого через 10 годин після опромінення?

Лейкопенія

Нейтрофільний лейкоцитоз

Лейкемія

Агранулоцитоз

Лімфоцитоз

1232 / 6854
У підлітка 12-ти років, який хворіє на бронхіальну астму, виник тяжкий напад астми: виражена експіраторна задишка, блідість шкірних покривів. Який вид порушення альвеолярної вентиляції має місце?

Нервово-м’язовий

Рестриктивний

Торако-діафрагмальний

Обструктивний

Центральний

1233 / 6854
У жінки, що тривалий час дотримувалася дієти з використанням очищеного рису, виявлений поліневрит (хвороба Бері-Бері). Відсутність якого вітаміну в їжі призводить до розвитку цього захворювання?

Тіамін

Рибофлавін

Фолієва кислота

Піридоксин

Аскорбінова кислота

1234 / 6854
У хворого після видалення жовчного міхура утруднені процеси всмоктування Ca через стінку кишечнику. Призначення якого вітаміну буде стимулювати цей процес?

PP

C

В12

D3

K

1235 / 6854
У населеному пункті зареєстрований спалах гепатиту, який зв’язують з водним фактором. Який вірус гепатиту міг викликати спалах захворювань у цьому населеному пункті?

E

B

D

G

C

1236 / 6854
У хворого 43-х років у шлунку погано перетравлюються білки. Аналіз шлункового соку виявив низьку кислотність. Функція яких клітин шлунка порушена в даному випадку?

Слизові клітини (мукоцити)

Шиєчні мукоцити

Парієтальні екзокриноцити

Ендокринні клітини

Головні екзокриноцити

1237 / 6854
До навчального закладу вступив юнак 16-ти років з сільської місцевості. При плановому проведенні реакції Манту виявилося, що у цього юнака вона негативна. Яка найбільш раціональна тактика лікаря?

Терміново ізолювати юнака з навчального колективу

Провести прискорену дiагностику туберкульозу методом Прайса

Повторити реакцію через 1 місяць

Провести серодiагностику туберкульозу

Зробити щеплення БЦЖ

1238 / 6854
Хворий відзначає частi проноси, особливо після вживання жирної їжі, схуднення. Лабораторні дослідження показали наявність стеатореї; кал гіпохолічний. Що може бути причиною такого стану?

Недостатність панкреатичної фосфо-ліпази

Обтурація жовчних шляхів

Запалення слизової оболонки тонкої кишки

Незбалансована дієта

Недостатність панкреатичної ліпази

1239 / 6854
Хворий звернувся до лікаря зі скаргами на дисфункцію кишечнику. Лікар констатував симптоми дуоденіту і ентериту. Під час лабораторного дослідження встановлено діагноз: лямбліоз. Застосування якого препарату показане?

Мономіцин

Тетрациклін

Еритроміцин

Хінгамін

Метронідазол

1240 / 6854
У групи альпіністів на висоті 3000 метрів було зроблено аналіз крові. Виявлене зниження HCO3 до 15 ммоль/л (норма 22-26 ммоль/л). Який механізм зниження HCO3 крові?

Зниження реабсорбції бікарбонатів у нирках

Посилення ацидогенезу

Гіпервентиляція

Зниження амоніогенезу

Гіповентиляція

1241 / 6854
Після введення лікарської речовини у піддослідної тварини зменшилося виділення слини, розширилися зіниці, а при наступному введенні у вену ацетилхоліну частота скорочень серця істотно не змінилася. Вкажіть назву цієї речовини:

Атропін

Анаприлін

Сальбутамол

Прозерин

Адреналін

1242 / 6854
При тривалому використанні препарату в хворого можуть мати місце остеопороз, ерозії слизової шлунка, гіпокаліємія, затримка натрію і води, зменшення вмісту кортикотропіну в крові. Укажіть цей препарат:

Індометацин

Дигоксин

Преднізолон

Резерпін

Гіпотіазид

1243 / 6854
Дитина квола, апатична. Печінка збільшена, при її біопсії виявлено значний надлишок глікогену. Концентрація глюкози в крові нижче норми. У чому причина зниженої концентрації глюкози у крові цієї хворої?

Підвищена активність глікогенсинтетази у печінці

Понижена (відсутня) активність глюкозо-6-фосфатази у печінці

Дефіцит гену, який відповідає за синтез глюкозо-1-фосфатуридинтрансферази

Понижена (відсутня) активність гексокінази у печінці

Понижена (відсутня) активність глікоген-фосфорилази у печінці

1244 / 6854
До лікарні швидкої допомоги госпіталізований чоловік 63-х років з явищами колапсу. Для боротьби з гіпотензією лікар вибрав норадреналін. Який механізм дії цього препарату?

Активація серотонінових рецепторів

Блокада M-холінорецепторів

Активація β-адренорецепторів

Активація дофамінових рецепторів

Активація α-адренорецепторів

1245 / 6854
У дитини з підозрою на дифтерію з зіву виділена чиста культура мікроорганізмів та вивчені їх морфологічні, тинкторіальні, культуральні та біохімічні властивості, які виявилися типовими для збудників дифтерії. Яке дослідження необхідно ще провести для видачі висновку про те, що виділена патогенна дифтерійна паличка?

Визначення уреазної активності

Визначення токсигенних властивостей

Визначення властивосгі розщеплювати крохмаль

Визначення протеолітичних властивостей

Визначення цистиназної активносгі

1246 / 6854
У дитини на слизовій оболонці щік та на язиці виявлені білуваті плями, які нагадують молоко, що скипілося. У виготовлених препаратах-мазках знайдені грампозитивні овальнi дріжджоподі-6ні клітини. Які це збудники?

Стафілококи

Дифтерійна паличка

Гриби роду Кандіда

Актиноміцети

Фузобактерії

1247 / 6854
У хворого 35-ти років, який часто вживає алкоголь, на фоні лікування сечогінними засобами, виникли сильна м’язова і серцева слабкість, блювання, діарея, АТ- 100/60 мм рт.ст., депресія. Причиною такого стану є посилене виділення з сечею:

Хлору

Фосфатів

Калію

Натрію

Кальцію

1248 / 6854
Хворий після вживання жирної їжі відчуває нудоту, млявість; з часом з’явилися ознаки стеатореї. У крові холестерин - 9,2 ммоль/л. Причиною такого стану є нестача у кишечнику:

Тригліцеридів

Фосфоліпідів

Жирних кислот

Хіломікронів

Жовчних кислот

1249 / 6854
У чоловіка, який тривалий час не вживав з їжею жирів, але отримував достатню кількість вуглеводів і білків, виявлено дерматит, погане загоювання ран, погіршення зору. Дефіцит яких компонентів є причиною порушення обміну речовин?

Мінеральні солі

Пальмітинова кислота

Олеїнова кислота

Лінолева кислота, вітаміни A, D, E, K

Вітаміни PP, H

1250 / 6854
Експериментальній тварині давали надлишкову кількість глюкози, міченої за вуглецем, протягом тижня. У якій сполуці можна виявити мітку?

Вітамін A

Метіонін

Арахідонова кислота

Пальмітинова кислота

Холін

1251 / 6854
РНК, що містить вірус імунодефіциту людини, проникла всередину лейкоцита і за допомогою ферменту ревертази змусила клітину синтезувати вірусну ДНК. В основі цього явища лежить:

Дерепресія оперона

Зворотня транскрипція

Конваріантна реплікація

Репресія оперона

Зворотня трансляція

1252 / 6854
У хворого з частими кровотечами з внутрішніх органів і слизових оболонок виявлені пролін і лізин у складі колагенових волокон. Через відсутність якого вітаміну порушено їх гідроксилювання?

Тіамін

Вітамін A

Вітамін E

Вітамін K

Вітамін C

1253 / 6854
У хворого із запаленням легень спостерігається підвищення температури тіла. Яка біологічно активна речовина відіграє провідну роль у виникненні цього прояву?

Інтерлейкін-I

Брадикінін

Гістамін

Серотонін

Лейкотрієни

1254 / 6854
Хвора 27-ми років закрапала в очі краплі, до складу яких входить пеніцилін. Через декілька хвилин з’явився свербіж та печіння тіла, набряк губ та повік, свистячий кашель; став падати артеріальний тиск. Які імуноглобуліни беруть участь в розвитку даної алергічної реакції?

IgM та IgG

IgM та IgD

IgE та IgG

IgG та IgD

IgA та IgM

1255 / 6854
У пацієнта цироз печінки. Дослідження якої з пєрєлічєних речовин, що екскретуються з сечею, може характеризувати стан антитоксичної функції печінки?

Сечова кислота

Креатинін

Гіпурова кислота

Амонійні солі

Амінокислоти

1256 / 6854
У дитини 2-х років виникли судоми внаслідок зниження концентрації іонів кальцію в плазмі крові. Функція якого ендокринного органу знижена?

Гіпофіз

Шишкоподібна залоза

Тимус

Прищитоподібні залози

Кора наднирників

1257 / 6854
При запаленні ока у хворого відмічалося накопичення мутної рідини з високим вмістом білку на дні передньої камери, яке отримало назву - гіпопіон. Який процес лежить в основі зазначених змін?

Порушення мікроциркуляції

Первинна альтерація

Вторинна альтерація

Проліферація

1258 / 6854
У людини частота серцевих скорочень постійно утримується на рівні 40 разів за хвилину. Що є водієм ритму серця у неї?

Атріовентрикулярний вузол

Пучок Гіса

Ніжки пучка Гіса

Волокна Пуркін’є

Синоатріальний вузол

1259 / 6854
Хвора 48-ми років надійшла до клініки із скаргами на слабкість, дратівливість, порушення сну. Об’єктивно: шкіра та склери жовтого кольору. У крові: підвищення рівня загального білірубіну з переважанням прямого. Кал - ахолічний. Сеча - темного кольору (жовчні пігменти). Яка жовтяниця має місце в хворої?

Синдром Кріглера-Найяра

Механічна

Синдром Жільбера

Паренхіматозна

Гемолітична

1260 / 6854
При бактеріологічному дослідженні промивних вод хворого на харчове отруєння висіяли чисту культуру бактерій з такими властивостями: грамнегативна рухлива паличка, на середовищі Ендо росте у вигляді безбарвних колоній. Представником якого роду було зумовлене захворювання?

Shigella

Salmonella

Esherichia

Iersinia

Citrobacter

1261 / 6854
Хворому встановлено діагноз - активний вогнищевий туберкульоз легень. Вкажіть, який із препаратів найбільш доцільно призначити в першу чергу?

Етіонамід

Ізоніазид

Циклосерін

Сульфален

Етоксид

1262 / 6854
При обстеженні молодого чоловіка у центрі по боротьбі зі СНІДом отримано позитивний результат ІФА з антигенами ВІЛ. Скарги на стан здоров’я відсутні. Про що може свідчити результат ІФА?

Про перенесене захворювання на СНІД

Про персистенцію ВГВ

Про захворювання на СНІД

Про інфікування ВІЛ

Про інфікування ВГВ

1263 / 6854
До лікаря звернулися батьки хлопчика 10-ти років, у якого відзначалося збільшення волосяного покриву на тілі, ріст бороди і вус, низький голос. Збільшення секреції якого гормону можна припустити?

Кортизол

Естроген

Соматотропін

Тестостерон

Прогестерон

1264 / 6854
У хворого, що страждає на важку форму порушення водно-сольового обміну, настала зупинка серця в діастолі. Який найбільш вірогідний механізм зупинки серця в діастолі?

Гіпернатріємія

Дегідратація організму

Гіпокаліємія

Гіперкаліємія

Гіпонатріємія

1265 / 6854
Під час дослідження коронарних артерій виявлені атеросклеротичні бляшки з кальцинозом, що закривають просвіт судин на 1/3. У м’язі дрібні множинні білуваті прошарки сполучної тканини. Як називається процес, виявлений у міокарді?

Інфаркт міокарда

Дифузний кардіосклероз

Тигрове серце

Післяінфарктний кардіосклероз

Міокардит

1266 / 6854
У реакції пасивної гемаглютинації, поставленої з еритроцитарним черевнотифозним Viдіагностикумом, виявлені антитіла у розведенні сироватки обстежуваного до 1:80, що вище діагностичного титру. Такий результат свідчить про наступне:

Інкубаційний період черевного тифу

Рецидив черевного тифу

Гостре захворювання на черевний тиф

Реконвалесценції хворого на черевний тиф

Можливе носійство паличок черевного тифу

1267 / 6854
У підлітка внаслідок радіоактивного опромінення значно постраждала лімфоїдна система, відбувся розпад великої кількості лімфоцитів. Відновлення нормальної формули крові можливо завдяки діяльності залози:

Тимус

Наднирники

Щитоподібна

Печінка

Підшлункова

1268 / 6854
До приймального відділення доставлено чоловіка з різаною раною підошви правої стопи. У постраждалого обмежене підняття латерального краю стопи. Під час обробки виявлено пошкодження сухожилка м’яза. Який м’яз ушкоджений?

Довгий розгинач пальців

Триголовий м’яз гомілки

Передній великогомілковий

Короткий малогомілковий

Довгий малогомілковий

1269 / 6854
У хворого, що страждає на серцеву недостатність, спостерігаються збільшення печінки, набряки нижніх кінцівок, асцит. Який механізм є провідним в утворенні даного набряку?

Лімфогенний

Мембраногенний

Гідродинамічний

Колоїдно-осмотичний

1270 / 6854
У мужчини 32-х років високий зріст, гінекомастія, жіночий тип оволосіння, високий голос, розумова відсталість, безпліддя. Попередній діагноз - синдром Клайнфельтера. Що необхідно дослідити для його уточнення?

Сперматогенез

Каріотип

Родовід

Група крові

Лейкоцитарна формула

1271 / 6854
У шахтаря виявлено фіброз легень, що супроводжувався порушенням альвеолярної вентиляції. Який механізм виникнення цього порушення є провідним?

Звуження верхніх дихальних шляхів

Обмеження дихальної поверхні легень

Порушення нервової регуляції дихання

Спазм бронхів

Обмеження рухомості грудної клітки

1272 / 6854
Людина зробила спокійних видих. Як називається об’єм повітря, який міститься у неї в легенях при цьому?

Життєва ємність легень

Функціональна залишкова ємність легень

Залишковий об’єм

Резервний об’єм видиху

Дихальний об’єм

1273 / 6854
При дослідженні ізольованого кардіоміоциту встановлено, що він не генерує імпульси збудження автоматично. З якої структури серця отримано кардіоміоцит?

Атріовентрикулярний вузол

Пучок Пса

Волокна Пуркін’є

Сино-атріальний вузол

Шлуночок

1274 / 6854
При обстеженні людини встановлено, що хвилинний об’єм серця дорівнює 3500 мл, систолічний об’єм - 50 мл. Якою є частота серцевих скорочень за хвилину у людини?

50

90

60

80

70

1275 / 6854
У людини, яка обертається на каруселі, збільшилися частота серцевих скорочень, потовиділення, з’явилася нудота. З подразненням яких рецепторів, перш за все, це пов’язано?

Вестибулярні ампулярні

Зорові

Слухові

Пропріоцептори

Вестибулярні отолітові

1276 / 6854
З метою встановлення токсигенно-ті виділених від пацієнтів збудників дифтерії, культури висіяли на чашку Петрі з поживним агаром по обидва боки від розташованої в центрі смужки фільтрувального паперу, змоченого протидифтерійною антитоксичною сироваткою. Після інкубації посівів в агарі між окремими культурами і смужкою фільтрувального паперу виявлено смужкоподібні ділянки помутніння середовища. Яку імунологічну реакцію було виконано?

Реакція аглютинації

Реакція Кумбса

Реакція опсонізації

Реакція кільцепреципітації

Реакція преципітації в гелі

1277 / 6854
У людини вимірюють внутрішньо-плевральний тиск. У якій фазі людина затримала дихання, якщо величина тиску дорівнює - 7,5 см вод.ст?

Спокійний вдих

Спокійний видих

Форсований вдих

Форсований видих

1278 / 6854
Жінка 49-ти років звернулася до лікаря зі скаргами на підвищену втомлюваність та появу задишки під час фізичного навантаження. На ЕКГ: ЧСС-50/хв, PQ- подовжений, QRS- не змінений, кількість зубців P перевищує кількість комплексів QRS. Який вид аритмії у пацієнтки?

Екстрасистолія

Атріовентрикулярна блокада

Синусова брадикардія

Миготлива аритмія

Синоатріальна блокада

1279 / 6854
У хворого з невритом стегнового нерва порушено згинання стегна та розгинання гомілки у колінному суглобі. Функція якого м’яза при цьому порушена?

Триголовий м’яз стегна

Двоголовий м’яз стегна

Напівперетинчастий м’яз

Чотирьохголовий м’яз стегна

Півсухожилковий м’яз

1280 / 6854
При розтині трупа новонародженого хлопчика виявлені полідактилія, мікроцефалія, незрощення верхньої губи та твердого піднебіння, а також гіпертрофія паренхіматозних органів. Вказані вади відповідають синдрому Патау. Яка найбільш вірогідна причина даної патології?

Часткова моносомія

Нерозходження статевих хромосом

Трисомія 18-ої хромосоми

Трисомія 21-ої хромосоми

Трисомія 13-ої хромосоми

1281 / 6854
На препараті яєчника, забарвленому гематоксиліном-еозіном, визначається фолікул, в якому клітини фолікулярного епітелію розміщені в 1-2 шари та мають кубічну форму, навколо овоциту видно оболонку яскраво-червоного кольору. Назвіть цей фолікул:

Первинний

Атретичний

Примордіальний

Зрілий

Вторинний

1282 / 6854
Після тижневого застосування нового косметичного засобу у жінки розвинулося запалення повік з гіперемією, інфільтрацією та болючістю. Алергічна реакція якого типу розвинулася у пацієнтки?

IV

I

V

III

II

1283 / 6854
Під час огляду хворого відзначаються різке звуження зіниць, сонливість, рідке дихання за типом Чейна-Стокса, затримка сечі, сповільнення серцевого ритму, підвищення спинномозкових рефлексів. Яка речовина викликала отруєння?

Барбітал

Кофеїн

Атропін

Фосфакол

Морфін

1284 / 6854
Хворий знаходиться на обліку в ендокринологічному диспансері з приводу гіпертиреозу. До схуднення, тахікардії, тремтіння пальців рук, приєдналися симптоми гіпоксії - головний біль, втомлюваність, мерехтіння 'мушок' перед очима. Який механізм дії тиреоїдних гормонів лежить в основі розвитку гіпоксії?

Конкурентне гальмування дихальних ферментів

Специфічне зв’язування активних центрів дихальних ферментів

Роз’єднання окиснення та фосфорилювання

Посилення синтезу дихальних ферментів

Гальмування синтезу дихальних ферментів

1285 / 6854
При алергічному дерматиті лікар призначив хворому в складі комплексної терапії Яігістаміноблокатор. Визначте цей препарат:

Адреналін

Гідрокортизон

Преднізолон

Лоратадин

Кромолін-натрій

1286 / 6854
У пораненого кровотеча із гілок сонної артерії. Для тимчасового припинення кровотечі сонну артерію треба притиснути до горбика поперечного відростка шийного хребця. До якого саме хребця притискається артерія у таких випадках?

IV

VI

II

V

III

1287 / 6854
Внаслідок аварії у постраждалого виникли сильний біль та набряк передньої поверхні гомілки; тильне згинання ступні утруднене. Функція якого з названих м’язів гомілки постраждала?

M.tibialis anterior

M.flexor digitorum longus

M.peroneus brevis

M.flexor hallucis longus

M.peroneus longus

1288 / 6854
В першому класі було проведене медичне обстеження учнів з метою відбору дітей для ревакцинації проти туберкульозу. Яку з наведених нижче проб при цьому використали?

Проба Шика

Проба Бюрне

Нашкірна проба з тулярином

Проба з антраксином

Проба Манту

1289 / 6854
В експерименті збільшили проникність мембрани збудливої клітини для іонів калію. До яких змін мембранного потенціалу це призведе?

Локальна відповідь

Змін не буде

Гіперполяризація

Деполяризація

Потенціал дії

1290 / 6854
Хворий на гіпертонічну хворобу II стадії з лікувальною метою приймав один з гіпотензивних препаратів. Через деякий час артеріальний тиск знизився, але хворий став скаржитися на в’ялість, сонливість, байдужість. Пізніше з’явився біль у шлунку. Було діагностовано виразкову хворобу. Який гіпотензивний препарат приймав хворий?

Дибазол

Фуросемід

Резерпін

Каптоприл

Верапаміл

1291 / 6854
У жінки через 6 місяців після пологів розвинулася маткова кровотеча. Під час гінекологічного обстеження у порожнині матки виявлена тканина темно-червоного кольору з множинними порожнинами, що нагадує 'губку'. Під час мікроскопічного дослідження пухлини, у лакунах крові виявлені атипові світлі епітеліальні клітини Лангханса та гігантські клітини синцитіотрофобласта. Яка це пухлина?

Аденокарцинома

Плоскоклітинний незроговілий рак

Фіброміома

Хоріонепітеліома

Міхуровий занос

1292 / 6854
Хворий 42-х років висуває скарги на сильне серцебиття, пітливість, нудоту, порушення зору, тремор рук, підвищення артеріального тиску. З анамнезу: 2 роки тому було встановлено діагноз феохромоцитома. Гіперпродукція яких гормонів зумовлює цю патологію?

Катехоламіни

Тиреоїдні гормони

Глюкокортикоїди

Альдостерон

АКТГ

1293 / 6854
Хвора 56-ти років тривалий час хворіє на тиреотоксикоз. Який тип гіпоксії може розвинутися у цієї хворої?

Дихальна

Циркуляторна

Змішана

Тканинна

Гемічна

1294 / 6854
Для підвищення спортивних результатів чоловіку рекомендували застосовувати препарат, що містить карнітин. Який процес у найбільшому ступені активізується карнітином?

Тканинне дихання

Синтез кетонових тіл

Синтез стероїдних гормонів

Синтез ліпідів

Транспорт жирних кислот до міто-хондрій

1295 / 6854
У хворої симптоми запального процесу сечостатевих шляхів. У мазку із слизової оболонки піхви виявлено великі одноклітинні організми грушоподібної форми з загостреним шипом на задньому кінці тіла, великим ядром та ундулюючою мембраною. Які найпростіші знайдені в мазку?

Trichomonas vaginalis

Trichomonas hominis

Lamblia intestinalis

Trichomonas buccalis

Trypanosoma gambiense

1296 / 6854
У хворого виявлено екстрасистолію. На ЕКГ при екстрасистолічному скороченні відсутній зубець P, комплекс QRS деформований, є повна компенсаторна пауза. Які це екстрасистоли?

Передсердно-шлуночкові

Синусні

Передсердні

Шлуночкові

1297 / 6854
У хворого на рак спинки язика виникла сильна кровотеча внаслідок ураження пухлиною дорзальної артерії язика. Яку судину повинен перев’язати лікар для зупинки кровотечі?

Лицева артерія

Язикова артерія

Висхідна артерія глотки

Дорзальна артерія язика

Глибока артерія язика

1298 / 6854
В експерименті певним чином зруйнована значна кількість стовбурових клітин червоного кісткового мозку. Оновлення яких популяцій клітин у складі пухкої сполучної тканини буде загальмовано?

Перицити

Фібробласти

Ліпоцити

Макрофаги

Пігментні клітини

1299 / 6854
Під час гістологічного дослідження тимуса чоловіка 40-ка років, визначено зменшення частки паренхіматозних елементів залози, збільшення частки жирової та пухкої сполучної тканини, збагачення її тимусними тільцями при незмінній загальній масі органу. Як зветься таке явище?

Акцідентальна інволюція

Гіпотрофія

Дистрофія

Вікова інволюція

Атрофія

1300 / 6854
На слизовій оболонці правого піднебінного мигдалика спостерігається безболісна виразка з гладеньким лакованим дном та рівними хрящоподібної консистенції краями. Мікроскопічно: запальний інфільтрат, що складається з лімфоцитів, плазмоцитів, невеликої кількості нейтрофілів та епітеліоїдних клітин, та наявність ендо- та периваскуліту. Про яке захворювання йдеться?

Сифіліс

Туберкульоз

Виразково-некротична ангіна Венсана

Дифтерія зіву

Актиномікоз

1301 / 6854
У бактеріологічній лабораторії проводиться дослідження м’ясних консервів на вміст ботулінічного токсину. Для цього дослідній групі мишей ввели екстракт із досліджуваного матеріалу та антитоксичну протиботулінічну сироватку типів А, В, Е; контрольній групі мишей ввели екстракт без протиботулінічної сироватки. Яку серологічну реакцію було використано?

Зв’язування комплементу

Опсонофагоцитарна

Подвійної імунної дифузії

Преципітації

Нейтралізації

1302 / 6854
Для запобігання післяопераційної кровотечі 6-ти річній дитині рекомендовано приймати вікасол, який є синтетичним аналогом вітаміну K. Вкажіть, які посттрансляційні зміни факторів згортання крові активуються під впливом вікасолу?

Карбоксилювання глутамінової кислоти

Полімеризація

Частковий протеоліз

Фосфорилювання радикалів серину

Глікозилювання

1303 / 6854
У чоловіка 33-х років внаслідок спинномозкової травми порушена больова та температурна чутливість, що обумовлено пошкодженням таких висхідних шляхів:

Медіальний спинокортикальний

Латеральний спинокортикальний

Передній спиномозочковий

Спиноталамічні

Задній спиномозочковий

1304 / 6854
У хворого для обробки опікової поверхні шкіри було використано препарат, антисептичні властивості якого забезпечуються вільним киснем, що відщеплюється у присутності органічних речовин. Який лікарський засіб був використаний?

Кислота борна

Фурацилін

Натрію гідрокарбонат

Хлоргексидин

Калію перманганат

1305 / 6854
У дитини 2-х років після грипу з’явилися скарги на біль у вусі. Лікар виявив зниження слуху та запалення середнього вуха. Яким шляхом інфекція потрапила до середнього вуха?

Через foramen jugularis

Через canalis caroticus

Через canalis nasolacrimalis

Через atrium mastoideum

Через слухову трубу

1306 / 6854
Хворому 50-ти років з хронічною серцевою недостатністю і тахіаритмією призначили кардіотонічний препарат. Який з препаратів призначили хворому?

Аміодарон

Дигоксин

Дофамін

Добутамін

Мілдронат

1307 / 6854
При мікроскопічному дослідженні оперативно видаленого апендикса відзначався набряк, дифузна нейтрофільна інфільтрація стінки з некрозом та наявністю дефекту слизової оболонки з ураженням її м’язової пластинки. Яка форма апендициту розвинулася в хворого?

Поверхнева

Флегмонозно-виразкова

Гангренозна

Флегмонозна

Апостематозна

1308 / 6854
У жінки 39-ти років під час операції були видалені збільшена у розмірах маткова труба та частина яєчника з великою кістою. При гістологічному дослідженні стінки труби виявлені децидуальні клітини, ворсини хоріону. Який найбільш вірогідний діагноз було встановлено при дослідженні маточної труби?

'Паперовий' плід

Трубна вагітність

Плацентарний поліп

Хоріонкарцинома

Літопедіон

1309 / 6854
У хлопчика 4-х років після перенесеного важкого вірусного гепатиту мають місце блювання, втрата свідомості, судоми. У крові - гіперамоніємія. Порушення якого біохімічного процесу викликало патологічний стан хворого?

Порушення знешкодження аміаку в печінці

Посилення гниття білків у кишечнику

Порушення знешкодження біогенних амінів

Активація декарбоксилування амінокислот

Пригнічення ферментів трансаміну-вання

1310 / 6854
У хворого виявлено гіперкаліємію та гіпонатріємію. Знижена секреція якого гормону може спричинити такі зміни?

Альдостерон

Паратгормон

Натрійуретичний

Кортизол

Вазопресин

1311 / 6854
При непрямому гістогенезі кісткової тканини трубчастих кісток між епіфізарним та діафізарним центрами окостеніння утворюється пластинка, що в подальшому забезпечує ріст кісток у довжину. Як називається ця структура?

Метафізарна пластинка

Остеон

Кісткова манжетка

Кісткова пластинка

Шар внутрішніх генеральних пластинок

1312 / 6854
Хвора 40-ка років надійшла до інфекційного відділення лікарні з високою температурою тіла. Об’єктивно: виражені менінгеальні симптоми. Проведено спинномозкову пункцію. Яке анатомічне утворення було пропунктовано?

Spatium subarachnoideum

Cisterna cerebellomedullaris posterior

Cavum trigeminale

Spatium subdurale

Spatium epidurale

1313 / 6854
Після резекції середньої третини облітерованої тромбом стегнової артерії нижня кінцівка кровопостачається за рахунок обхідних анастомозів. Назвіть артерію, яка має основне значення у відновленні кровотоку:

Поверхнева огинальна артерія клубової кістки

Глибока зовнішня соромітня артерія

Поверхнева надчеревна артерія

Глибока стегнова артерія

Низхідна колінна артерія

1314 / 6854
У хворого не розгинається колінний суглоб, відсутній колінний рефлекс, порушена чутливість шкіри на передній поверхні стегна. Який нерв уражений?

Стегновий

Нижній сідничний

Затульний

Великий малогомілковий

Верхній сідничний

1315 / 6854
На електронній мікрофотографії фрагменту нирки представлена приносна артеріола, у якій під ендотелієм видно великі клітини, що містять секреторні гранули. Назвіть даний вид клітин:

Гладеньком’язові

Інтерстиційні

Мезангіальні

Юкставаскулярні

Юкстагломерулярні

1316 / 6854
Для вирішення питання ретроспективної діагностики перенесеної бактеріальної дизентерії було призначено серологічне дослідження сироватки крові з метою встановлення титру антитіл до шигел. Яку з перелічених реакцій доцільно використати для цього?

Бактеріоліз

Преципітація

Гемоліз

Зв’язування комплементу

Пасивна гемаглютинація

1317 / 6854
У чоловіка 43-х років з видаленою ниркою були виявлені симптоми анемії. Що зумовило появу цих симптомів?

Зниження синтезу еритропоетинів

Нестача заліза

Нестача вітаміну B12

Підвищене руйнування еритроцитів

Нестача фолієвої кислоти

1318 / 6854
Хворий 50-ти років потрапив до лікарні зі скаргами на біль за грудниною, ядуху при фізичних навантаженнях. Після ангіографії виявлені патологічні зміни в задній міжшлуночковій гілці правої вінцевої артерії. Які ділянки серця уражені?

Передня стінка правого і лівого шлуночків

Правий передсердно-шлуночковий клапан

Ліве передсердя

Праве передсердя

Задня стінка правого і лівого шлуночків

1319 / 6854
Після проведення туберкулінової проби (проба Манту) у дитини через 48 годин на місці ведення туберкуліну утворилася папула до 10 мм у діаметрі. Який механізм гіперчутливості лежить в основі розвитку вказаних змін?

Клітинна цитотоксичність

Гранулематоз

Антитілозалежна цитотоксичність

Імунокомплексна цитотоксичність

Анафілаксія

1320 / 6854
У новонародженого хлопчика під час огляду зовнішніх статевих органів виявлена розщілина сечівника, яка відкривається на нижній поверхні статевого члена. Про яку аномалію йдеться?

Епіспадія

Гермафродитизм

Монорхізм

Крипторхізм

Гіпоспадія

1321 / 6854
У людини порушено всмоктування продуктів гідролізу жирів. Причиною цього може бути дефіцит у порожнині тонкої кишки наступних компонентів:

Жовчні кислоти

Іони натрію

Жовчні пігменти

Ліполітичні ферменти

Жиророзчинні вітаміни

1322 / 6854
У мешканців територій з холодним кліматом в крові збільшений вміст гормону, що має пристосувальне терморегуляторне значення. Про який гормон йдеться?

Інсулін

Соматотропін

Кортизол

Тироксин

Глюкагон

1323 / 6854
У результаті порушення техніки безпеки відбулося отруєння сулемою (хлористою ртуттю). Через 2 дні добовий діурез склав 620 мл. У хворого з’явилися головний біль, блювання, судоми, задишка, у легенях - вологі хрипи. Яка патологія має місце?

Гломерулонефрит

Гостра ниркова недостатність

Пієлонефрит

Уремічна кома

Хронічна ниркова недостатність

1324 / 6854
У новонародженої дитини з пілоростенозом часте блювання, що супроводжується апатією, слабкістю, підвищенням тонусу м’язів, інколи судомами. Яка форма порушення кислотно-основного стану розвинулася в хворого?

Негазовий алкалоз

Газовий ацидоз

Газовий алкалоз

Метаболічний ацидоз

Видільний ацидоз

1325 / 6854
При декарбоксилуванні глутамату в ЦНС утворюється медіатор гальмування. Назвіть його:

Аспарагін

Серотонін

ГАМК

Гістамін

Глутатіон

1326 / 6854
Під час катаболізму гістидину утворюється біогенний амін, що має потужну судинорозширюючу дію. Назвіть його:

Дофамін

ДОФА

Серотонін

Гістамін

Норадреналін

1327 / 6854
При утилізації арахідонової кислоти за циклооксигеназним шляхом утворюються біологічно активні речовини. Вкажіть їх:

Тироксин

Інсуліноподібні фактори росту

Соматомедини

Простагландини

Біогенні аміни

1328 / 6854
Внаслідок руйнування певних структур стовбуру мозку тварина втратила орієнтувальні рефлекси. Які структури було зруйновано?

Чорна речовина

Червоні ядра

Вестибулярні ядра

Чотиригорбкова структура

Медіальні ядра ретикулярної формації

1329 / 6854
У людини осмотичний тиск плазми крові 350 мосмоль/л (норма - 300 мосмоль/л). Це спричинить, перш за все, посилену секрецію такого гормону:

Адренокортикотропін

Альдостерон

Кортизол

Натрійуретичний

Вазопресин

1330 / 6854
Молодий чоловік звернувся до лікарні зі скаргами на порушення сечовипускання. Під час обстеження зовнішніх статевих органів виявлено, що сечівник розщеплений зверху і сеча витікає через цей отвір. Який вид аномалії розвитку зовнішніх статевих органів спостерігається у цьому випадку?

Фімоз

Парафімоз

Епіспадія

Гермафродитизм

Гіпоспадія

1331 / 6854
Хворий скаржиться на біль у ділянці печінки. При дослідженні жовчі, отриманої під час дуоденального зондування, виявлені жовтуваті яйця овальної форми, звужені до полюсів, на кінці одного полюсу знаходиться кришечка. Розміри цих яєць найменші серед яєць усіх гельмінтів. Який найбільш вірогідний діагноз?

Дифілоботріоз

Ехінококоз

Опісторхоз

Теніарінхоз

Теніоз

1332 / 6854
У хворого з дизентерією при колоноскопії виявлено, що слизова оболонка товстої кишки гіперемована, набрякла, її поверхня вкрита сіро-зеленими плівками. Назвіть морфологічну форму дизентерійного коліту:

Фібринозний

Гнійний

Некротичний

Катаральний

Виразковий

1333 / 6854
У хворого через добу після апендектомії у крові визначається нейтрофільний лейкоцитоз із регенеративним зсувом. Який найбільш вірогідний механізм розвитку лейкоцитозу в даному випадку?

Уповільнення руйнування лейкоцитів

Посилення лейкопоезу

Перерозподіл лейкоцитів у організмі

Уповільнення міграції лейкоцитів у тканини

Посилення лейкопоезу та уповільнення міграції лейкоцитів у тканини

1334 / 6854
У хірурга після проведення тривалої операції підвищився артеріальний тиск до 140/110 мм рт.ст. Які зміни гуморальної регуляції можуть бути причиною підвищення артеріального тиску в даному випадку?

Активація калікреїн-кінінової системи

Активація ренін-ангіотензинової системи

Активація симпатоадреналової системи

Активація утворення і виділення альдостерону

Гальмування симпатоадреналової системи

1335 / 6854
Хворому внутрішньовенно ввели гіпертонічний розчин глюкози. Це підсилить рух води:

З міжклітинної рідини до клітин

З капілярів до міжклітинної рідини

З міжклітинної рідини до капілярів

З клітин до міжклітинної рідини

Змін руху води не буде

1336 / 6854
У чоловіка 36-ти років черепно-мозкова травма. Об’єктивно: дихання слабке, пульс ниткоподібний, рефлекси відсутні. Який шлях введення пірацетама найбільш доцільний у даному випадку?

Внутрішньовенний

Підшкірний

Пероральний

Ректальний

Інгаляційний

1337 / 6854
Дитина скаржиться на загальну слабкість, відсутність апетиту, неспокійний сон, свербіж у періанальній ділянці. Встановлено діагноз: ентеробіоз. Для уточнення діагнозу слід провести:

Рентгеноскопічне дослідження

Біопсія м’язової тканини

Аналіз дуоденального вмісту

Імунодіагностика

Зіскоб з періанальних складок

1338 / 6854
У хворого опікова хвороба ускладнилася ДВЗ-синдромом. Яку стадію ДВЗ-синдрому можна запідозрити, якщо відомо, що кров хворого згортається менше ніж за 3 хвилини?

Гіпокоагуляції

Фібриноліз

Перехідна

Гіперкоагуляції

Термінальна

1339 / 6854
На практичному занятті студенти вивчали забарвлений мазок крові миші з бактеріями, фагоцитованими лейкоцитами. Яка органела клітини завершує перетравлення цих бактерій?

Апарат Гольджі

Рибосоми

Гранулярна ендоплазматична сітка

Мітохондрії

Лізосоми

1340 / 6854
Жінка 55-ти років звернулася зі скаргами на тривалі циклічні маткові кровотечі протягом року, слабкість, запаморочення. Об’єктивно: блідість шкіри. У крові: Hb- 70 г/л, ер.- 3,2•1012/л, КП- 0,6, лейк.- 6,0•109 /л, ретикулоцити -1%; гіпохромія еритроцитів. Яка анемія у хворої?

В12-фолієводефіцитна

Хронічна постгеморагічна

Гемолітична

Апластична

Залізодефіцитна

1341 / 6854
У пацієнта за даними аудіометрії виявлено порушення сприйняття звуків середньої частоти. Причиною цього може бути пошкодження:

Чотиригорбикової структури

Латеральних колінчастих тіл

Спірального ганглія

Кохлеарних ядер

Середньої частини завитки

1342 / 6854
Госпіталізовано хворого з діагнозом карциноїд кишечнику. Аналіз виявив підвищену продукцію серотоніну. Відомо, що ця речовина утворюється з амінокислоти триптофану. Який біохімічний механізм лежить в основі даного процесу?

Декарбоксилювання

Дезамінування

Утворення парних сполук

Трансамінування

Мікросомальне окиснення

1343 / 6854
-х років скаржиться на слабкість, підвищення температури до 38 — 400 143. C. Об’єктивно: печінка і селезінка збільшені. У крові: Hb- 100 г/л, ер.- 2,9•1012/л, лейк.- 4,4•109 /л, тромб.-48 • 109 /л, нейтрофіли сегментоядерні -17%, лімфоцити - 15%, бластні клітини - 68%. Всі цитохімічні реакції негативні. Дайте гематологічний висновок:

Гострий еритромієлоз

Хронічний мієлолейкоз

Гострий лімфобластний лейкоз

Гострий мієлобластний лейкоз

Недиференційований лейкоз

1344 / 6854
Хворий на бронхіальну астму не повідомив лікаря, що в нього бувають напади стенокардії. Лікар призначив препарат, після прийому якого напади бронхіальної астми стали рідшими, однак почастішали напади стенокардії. Який препарат був призначений?

Сальбутамол

Еуфілін

Кромолін-натрій

Ізадрин

Фенотерол

1345 / 6854
Хворий на хронічну серцеву недостатність протягом декількох місяців приймав в амбулаторних умовах дигоксин. На певному етапі лікування в нього виникли симптоми передозування препарату. Яке явище лежить в основі розвитку цього ускладнення?

Матеріальна кумуляція

Тахіфілаксія

Функціональна кумуляція

Сенсибілізація

Звикання

1346 / 6854
Хворий, що лікувався з приводу неврозу сибазоном, скаржиться на зубний біль. Лікар призначив йому знеболювальний засіб у дозі, яка менша за середню терапевтичну. Яке явище взяв до уваги лікар, зменшуючи дозу препарату?

Потенціювання

Толерантність

Сумація

Лікарська залежність

Кумуляція

1347 / 6854
У крові дитини виявлено високий вміст галактози, концентрація глюкози понижена. Спостерігаються катаракта, розумова відсталість, розвивається жирове переродження печінки. Яке захворювання має місце?

Лактоземія

Цукровий діабет

Фруктоземія

Галактоземія

Стероїдний діабет

1348 / 6854
З урахуванням клінічної картини хворому призначено піридоксальфосфат. Для корекції яких процесів рекомендований цей препарат?

Окисне декарбоксилювання кетокислот

Трансамінування і декарбоксилювання амінокислот

Дезамінування пуринових нуклеотидів

Синтез пуринових та піримідинових основ

Синтез білку

1349 / 6854
Під час операції у пацієнта було видалено частину легені, яка вентилюється бронхом третього порядку, що супроводжується гілками легеневої артерії та інших судин. Яка частина легені була видалена?

Верхня частка

Нижня частка

Легенева часточка

Сегмент легені

Середня частка

1350 / 6854
У хворого з клінічними ознаками імунодефіциту проведено імунологічні дослідження. Виявлено значне зниження кількості клітин, що утворюють розетки з еритроцитами барана. Який висновок слід зробити на основі даних аналізу?

Зниження рівня Б-лімфоцитів

Зниження рівня натуральних кілерів (УХ-клітин)

Зниження рівня системи комплементу

Зниження рівня T-лімфоцитів

Недостатність клітин-ефекторів гуморального імунітету

1351 / 6854
У жінки з III (Б), Rh- групою крові народилась дитина з II (A) групою крові. У дитини діагностовано гемолітичну хворобу новонародженого внаслідок резус-конфлікту. Яка група крові за системою АБо та резус-належність можливі у батька?

II (A), Rh

II (A), Rh+

III (Б), Rh+

I (O), Rh

I (O), Rh+

1352 / 6854
У жінки 45-ти років хвороба Іценко-Кушінга - стероїдний діабет. При біохімічному обстеженні: гіперглікемія, гіпохлоремія. Який з перерахованих нижче процесів активується у жінки в першу чергу?

Транспорт глюкози в клітину

Гліколіз

Реабсорбція глюкози

Глюконеогенез

Глікогеноліз

1353 / 6854
На гістологічному препараті видно судину, стінка якої складається з ендотелію, базальної мембрани та пухкої сполучної тканини. Назвіть тип судини:

Лімфокапіляр

Вена м’язового типу

Вена безм’язового типу

Гемокапіляр

Артерія

1354 / 6854
При розтині тіла померлого чоловіка 48-ми років, у ділянці 1-го сегменту правої легені виявлено округлий утвір діаметром 5 см з чіткими контурами, оточений тонким прошарком сполучної тканини, виповнений білими крихкими масами. Діагностуйте форму вторинного туберкульозу:

Гострий вогнищевий туберкульоз

Гострий кавернозний туберкульоз

Туберкулома

Фіброзно-кавернозний туберкульоз

Казеозна пневмонія

1355 / 6854
У хворого на гострий міокардит з’явилися клінічні ознаки кардіогенного шоку. Який із вказаних нижче патогенетичних механізмів є провідним в розвитку шоку?

Збільшення периферичного опору судин

Зниження насосної функції серця

Зниження судинного тонусу

Зниження діастолічного притоку до серця

Депонування крові в органах

1356 / 6854
У хворого з гострою нирковою недостатністю на 6-й день проведення терапевтичних заходів виникла поліурія. Чим зумовлене зростання діурезу на початку поліуричної стадії гострої ниркової недостатності?

Зменшенням вазопресину в плазмі

Зменшенням альдостерону в плазмі

Відновлення фільтрації в нефронах

Збільшенням натрійуретичного фактора

Збільшенням об’єму циркулюючої крові

1357 / 6854
При тривалому лікуванні голодуванням у пацієнта зменшилося співвідношення альбумінів і глобулінів у плазмі крові. Що з наведеного буде наслідком цих змін?

Зниження гематокритного показника

Гіперкоагуляція

Збільшення ШЗЕ

Збільшення гематокритного показника

Зниження ШЗЕ

1358 / 6854
Експериментальній тварині, після попередньої сенсибілізації, підшкірно введено дозу антигену. У місці ін’єкції розвинулось фібринозне запалення з альтерацією стінок судин, основної речовини та волокнистих структур сполучної тканини у вигляді мукоїдного та фібриноїдного набухання і некрозу. Яка імунологічна реакція має місце?

Гіперчутливість негайного типу

Нормергічна реакція

Гранульоматоз

Гіперчутливість сповільненого типу

Реакція трансплантаційного імунітету

1359 / 6854
У хворого спостерігається порушення зору - гемералопія ('куряча сліпота'). Який вітамінний препарат треба вживати хворому, щоб відновити зір?

Вікасол

Тіаміну хлорид

Токоферолу ацетат

Ретинолу ацетат

Піридоксин

1360 / 6854
До клініки госпіталізована дитина 1-го року з ознаками ураження м’язів. Після обстеження виявлений дефіцит карнітину в м’язах. Біохімічною основою цієї патології є порушення процесу:

Регуляції рівня Ca2+ в мітохондріях

Синтезу актину та міозину

Транспорту жирних кислот до міто-хондрій

Субстратного фосфорилювання

Утилізації молочної кислоти

1361 / 6854
У хворого важка післяопераційна псевдомонадна інфекція. Який з перелічених антибіотиків показаний хворому?

Амікацину сульфат

Цефазолін

Еритроміцин

Бензилпеніцилін

Доксициклін

1362 / 6854
У дитини 9-ти місяців спостерігається запізніле прорізування зубів, порушення порядку прорізування. Конфігурація верхньої щелепи - у горизонтальному напрямку ('високе піднебіння'); мікроскопічно - у зубах нерівномірна мінералізація емалі, зморщені емалеві призми, деякі з них вакуолізовані. Розширення зони предентину; зустрічаються поодинокі дентиклі. Яке захворювання у дитини?

Гіпервітаміноз D

Остеомаляція

Пізній рахіт

Подагра

Ранній рахіт

1363 / 6854
Під час обстеження у хворого виявлено абсцес крило-піднебінної ямки. Куди може розповсюдитись інфекція при несвоєчасному наданні медичної допомоги?

Орбіта

Міжкрилоподібний простір

Підапоневротичний скроневий простір

Барабанна порожнина

Лобова пазуха

1364 / 6854
При мікроскопічному дослідженні нирок померлої від ниркової недостатності жінки 36-ти років, в клубочках виявлено проліферацію нефротелію капсули, подоцитів та макрофагів з утворенням 'півмісяців', некроз капілярних петель, фібринові тромби в їх просвітах, а також склероз та гіаліноз клубочків, атрофію канальців та фіброз строми нирок. Який з перелічених діагнозів найбільш вірогідний?

Хронічний гломерулонефрит

Мембранозна нефропатія

Фокальний сегментарний склероз

Гострий гломерулонефрит

Підгострий гломерулонефрит

1365 / 6854
На судово-медичній експертизі знаходиться тіло чоловіка 58-ми років, який тривалий час зловживав алкоголем. Помер вдома. Макроскопічно: права легеня щільна і збільшена в розмірах, тканина на розрізі сіруватого кольору, однорідна, плевра вкрита сіруватими плівчастими нашаруваннями. Мікроскопічно - порожнини альвеол містять нитки фібрину, гемолізовані еритроцити. Діагностуйте захворювання:

Вогнищева пневмонія

Первинний туберкульоз легень

Крупозна пневмонія

Інтерстиційна пневмонія

Казеозна пневмонія

1366 / 6854
Через кілька днів після споживання копченої свинини у хворого з’явилися набряки обличчя та повік, шлунково-кишкові розлади, різке підвищення температури, м’язовий біль. В аналізі крові різко виражена еозинофілія. Яким гельмінтом могла заразитися людина через свинину?

Волосоголовець

Анкілостома

Аскарида

Трихінела

Гострик

1367 / 6854
У хворого після видалення зуба з’явився стійкий біль за грудниною. Після сублінгвального вживання антиангінального засобу біль за грудниною зник, але хворий поскаржився на головний біль та запаморочення. Для якого препарату притаманні ці властивості?

Анаприлін

Валідол

Верапаміл

Метопролол

Нітрогліцерин

1368 / 6854
До щелепно-лицьового відділення надійшов хворий з переломом нижньої щелепи. Було вирішено з’єднання кісток провести хірургічним методом під наркозом. Після внутрішньовенного введення міорелаксанту спостерігались короткочасні фібрилярні скорочення м’язів обличчя хворого. Який міорелаксант було застосовано?

Пипекуроній бромід

Діазепам

Меліктин

Дитилін

Тубокурарина хлорид

1369 / 6854
При розтині тіла померлого 56-ти років у правій скроневій частці головного мозку знайдено великий осередок розм’якшеної сірої речовини кашицеподібної консистенції, блідо-сірого кольору. В артеріях основи мозку чисельні білувато-жовті потовщення інтими, які різко звужують просвіт. Який найбільш вірогідний діагноз?

Крововилив

Абсцес мозку

Набряк мозку

Ішемічний інсульт

Геморагічний інсульт

1370 / 6854
При нестачі вітаміну A у людини відбувається порушення сутінкового зору. Вкажіть клітини, яким належить означена фоторецепторна функція:

Колбочкові нейросенсорні клітини

Горизонтальні нейроцити

Гангліонарні нервові клітини

Біполярні нейрони

Паличкові нейросенсорні клітини

1371 / 6854
У хворого 65-ти років під час неврологічного обстеження виявлено крововилив у межах верхньої скроневої звивини. У зоні кровопостачання якої артерії воно знаходиться?

Задня мозкова артерія

Передня сполучна артерія

Передня мозкова артерія

Основна артерія

Середня мозкова артерія

1372 / 6854
Чоловік 70-ти років під час гоління зрізав гнійник в ділянці соскоподібного відростка. Через 2 доби він був доставлений до лікарні з дiагнозом запалення оболонок головного мозку. Яким шляхом інфекція потрапила в порожнину черепа?

V.v.labyrinthi

V.v.auriculares

V.facialis

V.v.tympanicae

V.emissariae mastoideae

1373 / 6854
У жінки 22-х років виявлєні збіль-шєні лімфатичні вузли. Гістологічно: у лімфатичному вузлі наявні лімфоцити, гістіоцити, ретикулярні клітини, малі та великі клітини Ходжкіна, багатоядерні клітини Березовського-Штернберга, поодинокі осередки казе-озного некрозу. Для якого захворювання характерні такі зміни?

Лімфогранулематоз

Лімфосаркома

Хронічний лейкоз

Гострий лейкоз

Метастаз рака легень

1374 / 6854
У хворого з підозрою на одне з протозойних захворювань досліджено пунктат лімфатичного вузла. В препараті, забарвленому за Романовським-Гімзою, виявлені тільця півмісяцевої форми із загостреним кінцем, блакитною цитоплазмою, ядром червоного кольору. Яких найпростіших виявлено в мазках?

Дерматотропні лейшманії

Трипаносоми

Вісцеротропні лейшманії

Малярійні плазмодії

Токсоплазми

1375 / 6854
Під час статевого дозрівання клітини чоловічих статевих залоз починають продукувати чоловічий статевий гормон тестостерон, який обумовлює появу вторинних статевих ознак. Які клітини чоловічих статевих залоз продукують цей гормон?

Клітини Сертолі

Підтримуючі клітини

Сустентоцити

Клітини Лейдіга

Сперматозоїди

1376 / 6854
Ьольована клітина серця людини автоматично генерує імпульси збудження з частотою 60 разів за хвилину. З якої структури серця отримано цю клітину?

Пучок Гіса

Шлуночок

Передсердя

Синоатріальний вузол

Атріовентрикулярний вузол

1377 / 6854
При обстежені пацієнта встановили сильний, врівноважений, інертний тип вищої нервової діяльності за Павловим. Якому темпераменту за Гіппократом відповідає пацієнт?

Холерик

Флегматик

Сангвінік

Меланхолік

1378 / 6854
У хворого крововилив у задню центральну звивину. До порушення якого виду чутливості з протилежного боку це призведе?

Слухова

Шкірна та пропріоцептивна

Слухова і зорова

Нюхова та смакова

Зорова

1379 / 6854
У пункційному біоптаті печінки виявлена дистрофія гепатоцитів з некрозами, а також склероз з порушеннями балкової та часточкової будови, з утворенням несправжніх часточок і регенераторних вузлів. Оберіть найбільш вірогідний діагноз:

Прогресуючий масивний некроз печінки

Цироз печінки

Хронічний гепатит

Гострий гепатит

Хронічний гепатоз

1380 / 6854
У 60-ти річного пацієнта була виявлена гіперглікемія та глюкозурія. Для лікування цього хворого лікар призначив препарат для приймання всередину. Який це препарат?

Фуросемід

Корглікон

Глібенкламід

Панкреатин

Окситоцин

1381 / 6854
У людини, що виконувала важку фізичну роботу в умовах підвищеної температури навколишнього середовища, змінилася кількість білків плазми крові. Що саме має місце у даному випадку?

Відносна гіперпротеїнемія

Парапротеїнемія

Абсолютна гіпепротеїнемія

Абсолютна гіпопротеїнемія

Диспротеїнемія

1382 / 6854
У піддослідного щура з паралічем кінцівки спостерігається зникнення сухожилкових і шкірних рефлексів, зниження м’язового тонусу, при цьому зберігається здатність м’язів ураженої кінцівки відповідати збудженням на пряму дію постійного струму. Який тип паралічу відзначається у тварини?

В’ялий центральний

Спастичний центральний

Спастичний периферичний

Екстрапірамідний

В’ялий периферичний

1383 / 6854
Ліквідатора аварії на АЕС, який отримав опромінення, стало турбувати блювання, яке виникає несподівано. Який препарат слід призначити хворому?

Резерпін

Атропін

Де-нол

Метоклопрамід

Аерон

1384 / 6854
У хворого з гострим циститом під час дослідження сечі виявили лейкоцити та багато грамнегативних паличок. При посіві виросли колонії слизового характеру, які утворювали зелений розчинний пігмент. Який мікроорганізм, найбільш вірогідно, є причиною захворювання?

Escherihia coli

Proteus mirabilis

Salmonella enteritidis

Klebsiella pneumoniae

Pseudomonas aeruginosa

1385 / 6854
Чоловік 45-ти років звернувся до лікаря з приводу бляшкоподібного утвору на шиї. В біоптаті шкіри гістологічно виявлено пухлинні клітини, розташовані гніздами, які мають круглу та овальну форму з вузьким ободком базофільної цитоплазми та схожі на клітини базального шару епідермісу. Як називається пухлина в пацієнта?

Сирінгоаденома

Гідраденома

Епідермальний рак

Трихоепітеліома

Базаліома

1386 / 6854
У чоловіка з’явилися біль, набряк та почервоніння шкіри у передньо-верхній частині стегна та великого пальця стопи. Які лімфатичні вузли нижньої кінцівки відреагували на запальний процес?

Загальні повздовжні

Глибокі пахвинні

Внутрішні повздовжні

Поверхневі повздовжні

Поверхневі пахвинні

1387 / 6854
До лабораторії надійшов матеріал із рани хворого. Попередній діагноз - газова гангрена. Яким мікробіологічним методом можна встановити видову приналежність збудника?

Серологічний

Бактеріологічний

РІА

Бактеріоскопічний

Алергічний

1388 / 6854
У культурі клітин, отриманих від хворого з лізосомною патологією, виявлено накопичення значної кількості ліпідів у лізосомах. При якому з перелічених захворювань має місце це порушення?

Хвороба Тея-Сакса

Фенілкетонурія

Подагра

Хвороба Вільсона-Коновалова

Галактоземія

1389 / 6854
У чоловіка після черепно-мозкової травми виникла правобічна косоокість, що сходиться. Пошкодження якого черепно-мозкового нерва призвело до таких наслідків?

n.facialis

n.trochlearis

n.trigeminus

n.oculomotorius

n.abducens

1390 / 6854
При проникаючому пораненні передньої черевної стінки раневий канал пройшов над малою кривиною шлунка. Яке утворення очеревини найвірогідніше пошкоджене?

Ligamentum hepatogastricum

Ligamentum triangulare sinistrum

Ligamentum hepatorenale

Ligamentum hepatoduoduodenale

Ligamentum gastrocolicum

1391 / 6854
У кроля перерізали нерв, що іннервує праве вухо, і видалили правий верхній шийний симпатичний вузол. Одразу після операції провели вимірювання температури шкіри вух. Виявилося, що температура шкіри вуха кролика на боці денервації на 1,50 C вища, ніж на протилежному інтактному боці. Що з наведеного є найбільш вірогідною причиною вказаних явищ?

Артеріальна гіперемія нейропаралітичного типу

Фізіологічна артеріальна гіперемія

Артеріальна гіперемія нейротонічного типу

Артеріальна гіперемія, обумовлена метаболічними факторами

Реактивна артеріальна гіперемія

1392 / 6854
У чоловіка 63-х років захворювання почалось гостро з явищ гострого трахеїту і бронхіту, до яких приєдналася бронхопневмонія. На 10-ту добу хворий помер від легенево-серцевої недостатності. На розтині виявлений фібринозно-геморагічний ларинготра-хеобронхіт; легені збільшені в об’ємі, на розрізі мають 'пістрявий'вигляд за рахунок чергування ділянок бронхопневмонії, крововиливів у легеневу паренхіму, гострих абсцесів і ателектазів. У внутрішніх органах - дисциркуляторні та дистрофічні зміни. Який діагноз найбільш вірогідний?

Респіраторно-синцитіальна інфекція

Грип середньої важкості

Аденовірусна інфекція

Грип, важка форма

Парагрип

1393 / 6854
На аутопсії померлого від грипу чоловіка відзначено, що серце дещо збільшене у розмірах, пастозно, на розрізі міокард тьмяний, з крапом. Мікроскопічно: у міокарді на всьому протязі ознаки паренхіматозної жирової і гідропічної дистрофії, строма набрякла, з незначною макрофагально-лімфоцитарною інфільтрацією, судини повнокровні; периваскулярно - петехіальні крововиливи. Який вид міокардиту розвинувся в даному випадку?

Серозний вогнищевий

Серозний дифузний

Проміжний проліферативний

Гранулематозний

Гнійний

1394 / 6854
У хворого на хронічну серцеву недостатність, незважаючи на терапію кардіотонічними засобами і тіазидовим діуретиком, зберігаються набряки і виникла загроза асциту. Який препарат слід призначити для підсилення діуретичного ефекту застосованих ліків?

Фуросемід

Клопамід

Спіронолактон

Манітол

Амілорид

1395 / 6854
У хворого на колагеноз після тривалого прийому преднізолону з’явився спастичний біль скелетних м’язів внаслідок розвитку гипокаліємії. Який препарат треба використати для корекції обміну калію?

Тирокальцитонін

Діазепам

Дитилін

Но-шпа

Панангін

1396 / 6854
Хлопчик 7-ми років. Об’єктивно: на гіперемованому фоні шкіри дрібно-крапчастий яскраво-рожевий висип на лобі, шиї, внизу живота, підколінних ямках; носогубний трикутник блідий. В ротоглотці - відмежована яскраво-червона гіперемія; мигдалики набряклі, пухкі, в лакунах є гній, язик малиновий. Шийні лімфовузли збільшені, щільні, болючі. Який найбільш вірогідний діагноз?

Скарлатина

Інфекційний мононуклеоз

Дифтерія

Краснуха

Кашлюк

1397 / 6854
Пацієнт скаржиться на сухість шкіри голови, свербіж, ламкість і випадіння волосся. При обстеженні встановлений діагноз: себорея. З порушенням діяльності яких клітин це пов’язано?

Епітеліоцити

Меланоцити

Клітини потових залоз

Адипоцити

Клітини сальних залоз

1398 / 6854
В хірургічному відділенні лікарні виник спалах госпітальної інфекції, що проявлялася в частому нагноєнні післяопераційних ран. При бактеріологічному дослідженні гною був виділений золотистий стафілокок. Яке дослідження треба використати для виявлення джерела цього збудника серед персоналу відділення?

Визначення чутливості до антибіотиків

Серологічна ідентифікація

Фаготипування

Біохімічна ідентифікація

Мікроскопія

1399 / 6854
Внаслідок короткочасного фізичного навантаження у людини рефлекторно зросли частота серцевих скорочень та системний артеріальний тиск. Активація яких рецепторів найбільшою мірою зумовила реалізацію пресорного рефлексу в цій ситуації?

Терморецептори гіпоталамуса

Барорецептори судин

Пропріорецептори працюючих м’язів

Волюморецептори судин

Хеморецептори судин

1400 / 6854
В експерименті подразнюють скелетний м’яз серією електричних імпульсів. Який вид м’язового скорочення буде виникати, якщо кожний наступний імпульс припадає на період розслаблення поодинокого м’язового скорочення?

Суцільний тетанус

Серія поодиноких скорочень

Контрактура м’яза

Зубчастий тетанус

Асинхронний тетанус

1401 / 6854
Після вживання жирної їжі у хворого з’являються нудота та печія, має місце стеаторея. Причиною такого стану може бути:

Нестача амілази

Порушення синтезу фосфоліпази

Підвищене виділення ліпази

Порушення синтезу трипсину

Нестача жовчних кислот

1402 / 6854
У немовляти внаслідок неправильного годування виникла виражена діарея. Одним з основних наслiдкiв діареї є екскреція великої кількості бікарбонату натрію. Яка форма порушення кислотно-лужного балансу має місце в цьому випадку?

Респіраторний алкалоз

Метаболічний ацидоз

Не буде порушень кислотно-лужного балансу

Метаболічний алкалоз

Респіраторний ацидоз

1403 / 6854
У трирічної дитини з підвищеною температурою тіла після прийому аспірину спостерігається посилений гемоліз еритроцитів. Вроджена недостатність якого ферменту могла викликати у дитини гемолітичну анемію?

Глюкозо-6-фосфатдегідрогеназа

Глюкозо-6-фосфатаза

Гліцеролфосфатдегідрогеназа

Глікогенфосфорилаза

γ-глутамілтрансфераза

1404 / 6854
Людина хворіє на цукровий діабет, що супроводжується гіперглікемією натще понад 7,2 ммоль/л. Рівень якого білку плазми крові дозволяє ретроспективно (за попередні 4-8 тижні до обстеження) оцінити рівень глікемії?

С-реактивний білок

Альбумін

Фібриноген

Глікозильований гемоглобін

Церулоплазмін

1405 / 6854
Під час лапаротомії хірург виявив гангренозне ураження низхідної ободової кишки. Тромбоз якої артерії зумовив цей стан?

Права ободова

Клубово-ободова

Серединна ободова

Ліва ободова

Верхня брижова

1406 / 6854
У пацієнта 60-ти років виявлено погіршення сприйняття звуків високої частоти. Порушення стану яких структур слухового аналізатора зумовило ці зміни?

Основна мембрана завитки біля гелікотреми

Барабанна перетинка

Євстахієва труба

М’язи середнього вуха

Основна мембрана завитки біля овального віконця

1407 / 6854
З сироватки крові людини виділили п’ять ізоферментних форм лактатдегідрогенази і вивчили їх властивості. Яка властивість доводить, що виділені ізоферментні форми одного і того ж ферменту?

Каталізують одну і ту ж реакцію

Тканинна локалізація

Однакова молекулярна маса

Однакова електрофоретична рухливість

Однакові фізико-хімічні властивості

1408 / 6854
Було доведено, що молекула незрілої і-РНК (про-і-РНК) містить більше триплетів, чим знайдено амінокислот у синтезованому білку. Це пояснюється тим, що трансляції у нормі передує:

Ініціація

Реплікація

Мутація

Процесінг

Репарація

1409 / 6854
У хворого виявлено зниження вмісту іонів магнію, які потрібні для прикріплення рибосом до гранулярної ендоплазматичної сітки. Відомо, що це призводить до порушення біосинтезу білка. Який саме етап біосинтезу білка буде порушено?

Реплікація

Транскрипція

Трансляція

Термінація

Активація амінокислот

1410 / 6854
Студент старанно конспектує лекцію. Якість конспектування значно погіршилась, коли сусіди стали розмовляти. Який вид гальмування умовних рефлексів є причиною цього?

Запізніле

Позамежове

Диференційоване

Згасаюче

Зовнішнє

1411 / 6854
Хворий з нейродермітом протягом тривалого часу вживав преднізолон. При обстеженні в нього виявили підвищення рівня цукру в крові. Вплив препарату на яку ланку вуглеводного обміну призводить до виникнення цього ускладнення?

Посилення всмоктування глюкози в кишечнику

Активація розщеплення інсуліну

Пригнічення синтезу глікогену

Активація глікогеногенезу

Активація глюконеогенезу

1412 / 6854
Для вивчення локалiзацiї біосинтезу бiлка в клітинах, миші ввели мічені амінокислоти аланін та триптофан. Біля яких органел буде спостерігатися накопичення мічених амінокислот?

Клітинний центр

Гладенька ЕПС

Рибосоми

Апарат Гольджі

Лізосоми

1413 / 6854
У хворого виявили туберкульоз легень. Який антибіотик слід йому призначити поряд з іншими протитуберкульозними засобами?

Азітроміцин

Кефзол

Левоміцетин

Ріфампіцин

Тетрациклін

1414 / 6854
Запалення барабанної порожнини (гнійний середній отит) у хворого ускладнилося запаленням комірок соскоподібного відростка. Через яку стінку барабанної порожнини потрапив гній до комірок?

Передня

Латеральна

Задня

Верхня

Медіальна

1415 / 6854
Для прискорення загоєння рани слизової оболонки в ротовій порожнині хворому призначено препарат, який являє собою термостабільний білок, що міститься у людини в сльозах, слині, грудному молоці матері, а також його можна виявити в свіжознесеному курячому яйці. Відомо, що він являє собою фактор природної резистентності організму і має назву:

Інтерферон

Лізоцим

Інтерлейкін

Комплемент

Іманін

1416 / 6854
Внаслідок стресу у похилої людини підвищився артеріальний тиск. Причиною цього є активація:

Функції щитоподібної залози

Функції кори наднирників

Парасимпатичного ядра блукаючого нерва

Функції гіпофізу

Симпато-адреналової системи

1417 / 6854
Хворий 20-ти років скаржиться на загальну слабкість, запаморочення, швидку втомлюваність. У крові: Hb- 80 г/л. Мікроскопічно: еритроцити зміненої форми. Причиною цього стану може бути:

Гостра переміжна порфірія

Серпоподібноклітинна анемія

Хвороба Аддісона

Обтураційна жовтяниця

Паренхіматозна жовтяниця

1418 / 6854
У жінки 30-ти років хвилинний об’єм крові у стані спокою становить 5 л/хв. Який об’єм крові проходить у неї через судини легень за 1 хвилину?

2,5 л

3,75 л

5 л

2,0 л

1,5 л

1419 / 6854
У людини внаслідок тривалого голодування швидкість клубочкової фільтрації зросла на 20%. Найбільш вірогідною причиною змін фільтрації в зазначених умовах є:

Зменшення онкотичного тиску плазми крові

Збільшення проникності ниркового фільтру

Збільшення коефіцієнта фільтрації

Збільшення системного артеріального тиску

Збільшення ниркового плазмотоку

1420 / 6854
Під час експерименту подразнюють скелетний м’яз серією електричних імпульсів. Який вид м’язового скорочення буде виникати, якщо кожний наступний імпульс надходить у пєріоді вкорочення попереднього поодинокого м’язового скорочення?

Суцільний тетанус

Контрактура м’яза

Асинхронний тетанус

Серія поодиноких скорочень

Зубчастий тетанус

1421 / 6854
У пацієнта, що звернувся до лікаря, спостерігається жовте забарвлення шкіри, сеча темна, кал темно-жовтого кольору. Підвищення концентрації якої речовини буде спостерігатися в сироватці крові?

Білівердин

Вердоглобін

Кон’ югований білірубін

Вільний білірубін

Мезобілірубін

1422 / 6854
На прийом до лікаря звернувся хворий з симетричним дерматитом відкритих ділянок шкіри. З бесіди з пацієнтом встановлено, що він харчується, в основному, крупами і вживає мало м’яса, молока та яєць. Дефіцит якого вітаміну є провідним у цього пацієнта?

Біотин

Токоферол

Кальциферол

Фолієва кислота

Нікотинамід

1423 / 6854
Хворий 46-ти років звернувся до лікаря зі скаргою на біль в суглобах, який посилюється напередодні зміни погоди. У крові виявлено підвищення концентрації сечової кислоти. Посилений розпад якої речовини є найвірогіднішою причиною захворювання?

УМФ

ТМФ

ЦМФ

АМФ

УТФ

1424 / 6854
У хворої 38-ми років ревматизм в активній фазі. Визначення якого лабораторного показника сироватки крові має діагностичне значення при даній патології?

С-реактивний білок

Сечова кислота

Сечовина

Креатинін

Трансферин

1425 / 6854
Хворий з інфекційним мононуклеозом протягом двох тижнів приймав глюкокортикостероїдні препарати. Настала ремісія, проте в нього виникло загострення хронічного тонзиліту. Результатом якої дії глюкокортикостероїдів є дане ускладнення?

Антитоксична

Антиалергічна

Протизапальна

Імунодепресивна

Протишокова

1426 / 6854
Під час реєстрації ЕКГ хворого з гіперфункцією щитоподібної залози зареєстровано збільшення частоти серцевих скорочень. Вкорочення якого елементу ЕКГ про це свідчить?

Інтервал P — T

Комплекс QRS

Інтервал P — Q

Інтервал R — R

Сегмент P — Q

1427 / 6854
Чоловіку 70-ти років, що хворіє на хворобу Паркінсона, був призначений препарат леводопа. Через тиждень стан хворого значно покращився. Який механізм дії цього лікарського засобу?

Активація дофамінової системи

Гальмування холінергічної системи

Гальмування серотонінергічної системи

Активація енкефалінової системи

Гальмування гістамінергічної системи

1428 / 6854
Жінка 62-х років скаржиться на частий біль у ділянці грудної клітки та хребта, переломи ребер. Лікар припустив мієломну хворобу (плазмоцитому). Який з перерахованих нижче лабораторних показників буде мати найбільше діагностичне значення?

Парапротеїнемія

Протеїнурія

Гіперальбумінемія

Гіпоглобулінемія

Гіпопротеїнемія

1429 / 6854
Під час гри у волейбол спортсмен після стрибка приземлився на зовнішній край стопи. Виник гострий біль в гомілковостопному суглобі, активні рухи в ньому обмежені, пасивні - в повному обсязі, але болісні. Потім розвинулася припухлість у ділянці зовнішньої кісточки, шкіра почервоніла, стала теплішою на дотик. Який вид розладу периферичного кровообігу розвинувся в даному випадку?

Тромбоз

Артеріальна гіперемія

Стаз

Емболія

Венозна гіперемія

1430 / 6854
Хворий надійшов до стаціонару з гострою серцево-судинною недостатністю. Який препарат необхідно призначити хворому в даному випадку?

Дигітоксин

Адреналіну гідрохлорид

Дигоксин в таблетках

Корглікон

Аміодарон

1431 / 6854
У хворого з підозрою на дифтерію під час бактеріоскопічного дослідження мазка з зіву виявлені паличкоподібні бактерії з зернами волютину. Який етіотропний препарат є препаратом вибору в даному випадку?

Еубіотик

Протидифтерійна антитоксична сироватка

Інтерферон

Дифтерійний анатоксин

Бактеріофаг

1432 / 6854
У хворого після видалення жовчного міхура утруднені процеси всмоктування Ca через стінку кишечнику. Призначення якого вітаміну буде стимулювати цей процес?

C

D3

В12

PP

K

1433 / 6854
Жінці виконана операція з приводу позаматкової (трубної) вагітності. Гілки яких артерій має перев’язати хірург під час операції?

Маткова та нижня міхурна

Маткова та яєчникова

Маткова та верхня міхурна

Верхня міхурна та яєчникова

Нижня міхурна та яєчникова

1434 / 6854
Дитині 6-ти місяців з бронхітом зроблена рентгенограма грудної клітки. Крім змін пов’язаних з бронхами, на R-грамі визначається тінь вилочкової залози (тимуса). З чим можуть бути пов’язані ці зміни?

Є наслідком неправильного анатомічного розташування

Є наслідком запалення вилочкової залози

Є наслідком бронхіту

Є наслідком пухлинного процесу

Для цього віку вказаний стан є варіантом норми

1435 / 6854
Хворий відзначає часті проноси, особливо після вживання жирної їжі, схуднення. Лабораторні дослідження показали наявність стеатореї; кал гіпохолічний. Що може бути причиною такого стану?

Недостатність панкреатичної фосфоліпази

Незбалансована дієта

Обтурація жовчних шляхів

Недостатність панкреатичної ліпази

Запалення слизової оболонки тонкої кишки

1436 / 6854
У хворого з гнійничковими ураженнями шкіри виділений збудник, який на кров’яному агарі утворює жовті колонії округлої форми, середніх розмірів, оточені зоною гемолізу. У мазках з колоній - коки, розташовані скупченнями неправильної форми, грампозитивні. Виділена культура оксидазо- і каталазопозитивна, ферментує маніт, синтезує плазмокоагулазу. Який вид збудника виділений?

Streptococcus pyogenes

Staphylococcus saprophyticus

Staphylococcus aureus

Staphylococcus epidermidis

Streptococcus agalactiae

1437 / 6854
Чоловік 65-ти років, який страждає на подагру, скаржиться на біль в ділянці нирок. При ультразвуковому обстеженні встановлена наявність ниркових каменів. Підвищення концентрації якої речовини є найбільш вірогідною причиною утворення каменів у даному випадку?

Сечовина

Білірубін

Сечова кислота

Цистин

Холестерин

1438 / 6854
При мікроскопії зіскрібку з язика, забарвленого за Грамом, знайдені овальні, округлі, темно-фіолетового кольору, видовжені ланцюжки клітин, що брунькуються. Про збудника якого захворювання може йти мова?

Актиномікоз

Кандидоз

Стрептококова інфекція

Дифтерія

Стафілококова інфекція

1439 / 6854
Жінка 44-х років скаржиться на загальну слабкість, біль у ділянці серця, значне збільшення маси тіла. Об’єктивно: обличчя місяцеподібне, гірсутизм, АТ- 165/100 мм рт.ст., зріст -164 см, вага - 103 кг; переважно накопичення жиру на шиї, верхньому плечовому поясі, животі. Що є основним патогенетичним механізмом ожиріння у жінки?

Зниження продукції глюкагону

Підвищення продукції інсуліну

Підвищення продукції глюкокорти-коїдів

Підвищення продукції мінералокор-тикоїдів

Зниження продукції тиреоїдних гормонів

1440 / 6854
Кров, яку взяли у хворого з підозрою на сепсис, посіяли на цукровий бульйон. У цукровому бульйоні утворився придонний осад. При пересіві на кров’яний агар виросли дрібні, прозорі, кулясті колонії, оточені зоною гемолізу. У мазку, приготованому з осаду, визначалися грампозитивні коки, що розташовувалися у вигляді довгих ланцюжків. Які мікроорганізми присутні у крові цього хворого?

Стафілококи

Сарцини

Стрептококи

Тетракоки

Мікрококи

1441 / 6854
Хворий 2 роки тому переніс операцію резекції пілоричного відділу шлунка. Спостерігається слабкість, періодична поява темних кіл під очима, задишка. У крові: Hb70 г/л, ер.- 3,0 • 1012/л, КП- 0,7 Які зміни еритроцитів у мазках крові найбільш характерні для даного стану?

Мегалоцити

Мікроцити

Макроцити

Овалоцити

Шизоцити

1442 / 6854
У пацієнта цироз печінки. Дослідження якої з перелічених речовин, що екскретуються з сечею, може характеризувати стан антитоксичної функції печінки?

Амінокислоти

Гіпурова кислота

Сечова кислота

Креатинін

Амонійні солі

1443 / 6854
Територію старого худобомогильника, який не використовувався більше 50 років, планується відвести під житлове будівництво. Однак дослідження ґрунту показало наявність життєздатних спор збудника особливо небезпечного захворювання. Який із вказаних мікроорганізмів найбільш вірогідно міг зберегтися у ґрунті протягом такого тривалого часу?

Brucella abortus

Francisella tularensis

Mycobacterium bovis

Bacillus anthracis

Yersinia pestis

1444 / 6854
На розтині хворого, який багато років працював на шахті і помер від хронічної легеневосерцевої недостатності, виявлено, що легені малоповітряні, значно ущільнені, склерозовані, верхівки емфізематозно змінені, поверхня сіро-чорного кольору, на розрізі тканина легенів аспідно-чорного кольору. Від якої хвороби настала смерть?

Алюміноз

Талькоз

Азбестоз

Антракоз

Силікоз

1445 / 6854
У баклабораторії під час мікроскопії мазків з харкотиння хворого на хронічне легеневе захворювання, забарвлених за Цілем-Нільсеном, виявлені червоні палички. Яка властивість туберкульозної палички виявлена при цьому?

Лугостійкість

Капсулоутворення

Кислотостійкість

Спороутворення

Спиртостійкість

1446 / 6854
Під час дослідження коронарних артерій виявлені атеросклеротичні бляшки з кальцинозом, що закривають просвіт судин на 1/3. У м’язі дрібні множинні білуваті прошарки сполучної тканини. Як називається процес, виявлений у міокарді?

Тигрове серце

Післяінфарктний кардіосклероз

Інфаркт міокарда

Міокардит

Дифузний кардіосклероз

1447 / 6854
Встановлено ураження вірусом ВІЛ Т-лімфоцитів. При цьому фермент вірусу зворотня траскриптаза (РНК-залежна ДНК-полімераза) каталізує синтез:

і-РНК на матриці вірусного білка

ДНК на матриці вірусної і-РНК

Вірусна і-РНК на матриці ДНК

Вірусна ДНК на матриці ДНК

ДНК на вірусній р-РНК

1448 / 6854
До травматологічного відділення госпіталізовано чоловіка 35-ти років з травмою лівої кисті. Об’єктивно: різана рана долонної поверхні лівої кисті; середні фаланги II-V пальців не згинаються. Які м’язи пошкоджені?

Глибокий м’яз-згинач пальців

Поверхневий м’яз-згинач пальців

Червоподібні м’язи

Долонні міжкісткові м’язи

Тильні міжкісткові м’язи

1449 / 6854
При гістологічному дослідженні біоптату шкіри виявлені гранульоми, які складаються з макрофагальних вузликів з наявністю лімфоцитів та плазматичних клітин. Крім того, зустрічаються великі макрофаги з жировими вакуолями, які містять запакованих у вигляді куль збудників захворювання (клітини Вірхова). Грануляційна тканина добре васкуляризована. Для якого захворювання характерна описана гранульома?

Риносклерома

Сап

Сифіліс

Туберкульоз

Лепра

1450 / 6854
На препараті яєчника, забарвленому гематоксиліном-еозином, визначається фолікул, в якому клітини фолікулярного епітелію розміщені в 1-2 шари та мають кубічну форму, навколо овоциту наявна оболонка яскраво-червоного кольору. Назвіть цей фолікул:

Вторинний

Атретичний

Примордіальний

Зрілий

Первинний

1451 / 6854
До інфекційної лікарні надійшов пацієнт з клінічними ознаками енцефаліту. В анамнезі - укус кліща. В реакції гальмування гемаглютинації виявлено антитіла проти збудника кліщового енцефаліту в розведенні 1:20, що не є діагностичним. Вкажіть наступні дії лікаря після одержання зазначеного результату:

Повторити дослідження із сироваткою, взятою через 10 днів

Повторити дослідження з іншим діагностикумом

Дослідити цю ж сироватку повторно

Відхилити діагноз кліщового енцефаліту

Використати чутливішу реакцію

1452 / 6854
У чоловіка 40-ка років у ділянці шиї виникло почервоніння та набряк шкіри і з часом розвинувся невеликий гнійник. На розрізі осередок щільний, жовто-зеленого забарвлення. В гнійних масах видно білі крупинки. Гістологічно виявлено друзи грибка, плазматичні та ксантомні клітини, макрофаги. Який найбільш вірогідний вид мікозу?

Кокцидіомікоз

Споротрихоз

Аспергільоз

Актиномікоз

Кандидоз

1453 / 6854
Тривале вживання деяких лікарських засобів, що передують вагітності, збільшують ризик народження дитини з генетичними вадами. Як називається ця дія?

Фетотоксичний ефект

Мутагенний ефект

Ембріотоксичний ефект

Бластомогенний ефект

Тератогенний ефект

1454 / 6854
Під час розтину чоловіка, у якого після поранення кінцівки виникло тривале нагноєння рани, і який помер при явищах інтоксикації, знайдено загальне виснаження, зневоднення, бура атрофія печінки, міокарду, селезінки, поперечносмугастої мускулатури та амілоїдоз нирок. Який з перелічених діагнозів найбільш вірогідний?

Хвороба Чорногубова

Хроніосепсис

Септицемія

Бруцельоз

Септикопіємія

1455 / 6854
Жінка 19-ти років, хвора на первинний сифіліс, отримує комплексну терапію, до складу якої входить натрієва сіль бензилпеніциліну. Вкажіть механізм дії цього препарату:

Блокада синтезу петдидогліканів мікробної оболонки

Блокада синтезу РНК

Блокада синтезу білків цитоплазми

Блокада синтезу ДНК

Блокада тіолових груп ензимів

1456 / 6854
У жінки через 6 місяців після пологів розвинулася маткова кровотеча. Під час гінекологічного обстеження у порожнині матки виявлена тканина темно-червоного кольору з множинними порожнинами, що нагадує 'губку'. Під час мікроскопічного дослідження пухлини у лакунах крові виявлені атипові світлі епітеліальні клітини Лангханса та гігантські клітини синцитіотрофобласта. Яка це пухлина?

Фіброміома

Хоріонепітеліома

Аденокарцинома

Міхуровий занос

Плоскоклітинний незроговілий рак

1457 / 6854
У тварини з недостатністю аортальних клапанів розвинулась гіпертрофія лівого шлуночка серця. В окремих його ділянках визначаються локальні контрактури. Накопичення якої речовини в міокардіоцитах обумовило контрактури?

Калій

Вуглекислий газ

Молочна кислота

Кальцій

Натрій

1458 / 6854
У дівчинки діагностований адреногенітальний синдром (псевдогермафродитизм). Надмірна секреція якого гормону наднирників обумовила дану патологію?

Естроген

Адреналін

Кортизол

Андроген

Альдостерон

1459 / 6854
Під час гістологічного дослідження тимуса чоловіка 40-ка років, визначено зменшення частки паренхіматозних елементів залози, збільшення частки жирової та пухкої сполучної тканини, збагачення її тимусними тільцями при незмінній загальній масі органу. Як зветься таке явище?

Гіпотрофія

Вікова інволюція

Дистрофія

Акцидентальна інволюція

Атрофія

1460 / 6854
При обстеженні хворого 70-ти років виявлено інсулінонезалежний цукровий діабет. Який препарат доцільно призначити хворому?

Мерказоліл

Кортизон

Глібенкламід

Паратіреоїдин

Інсулін

1461 / 6854
Охолодження тіла людини у воді виникає значно швидше, ніж на повітрі. Який шлях тепловіддачі у воді значно ефективніший?

Конвекція

Випаровування поту

Теплопроведення

Тепловипромінювання

1462 / 6854
У бактеріологічній лабораторії проводиться дослідження м’ясних консервів на вміст ботулінічного токсину. Для цього дослідній групі мишей ввели екстракт із досліджуваного матеріалу та антитоксичну протиботулінічну сироватку типів А, В, Е; контрольній групі мишей ввели екстракт без протиботулінічної сироватки. Яку серологічну реакцію було використано?

Нейтралізації

Подвійної імунної дифузії

Преципітації

Зв’язування комплементу

Опсонофагоцитарна

1463 / 6854
Хворому, що страждає на хронічну серцеву недостатність, лікар порадив провести профілактичний курс лікування кардіотонічним препаратом з групи серцевих глікозидів, який приймають ентерально. Який препарат було рекомендовано хворому?

Строфантин

Корглікон

Кордарон

Дигоксин

Кордіамін

1464 / 6854
Під час диспансерного огляду у чоловіка 36-ти років, водія за професією, артеріальний тиск склав 150/90 мм рт.ст. Скарги на шум у вухах наприкінці робочого дня та загальне нездужання, які зникають після відпочинку. Діагностована гіпертонічна хвороба. Який провідний патогенетичний механізм у цьому випадку?

Нирковий

Ендокринний

Нейрогенний

Рефлексогенний

Гуморальний

1465 / 6854
Травма мозку викликала підвищене утворення аміаку. Яка амінокислота бере участь у видаленні аміаку з мозкової тканини?

Триптофан

Лізин

Валін

Тирозин

Глутамінова

1466 / 6854
Проводиться вивчення максимально спіралізованих хромосом каріотипу людини. При цьому процес поділу клітини припинили на стадії:

Інтерфаза

Телофаза

Анафаза

Метафаза

Профаза

1467 / 6854
Після проведення туберкулінової проби (проба Манту) у дитини через 48 годин на місці введення туберкуліну утворилася папула до 10 мм у діаметрі. Який механізм гіперчутливості лежить в основі розвитку вказаних змін?

Імунокомплексна цитотоксичність

Антитілозалежна цитотоксичність

Гранулематоз

Клітинна цитотоксичність

Анафілаксія

1468 / 6854
Хворий госпіталізований до хірургічного відділення з підозрою на запалення дивертикулу Меккеля. Обстеження якого відділу кишечнику необхідно провести для виявлення дивертикулу під час операції?

Duodenum

Jejunum

Ileum

Colon ascendens

Caecum

1469 / 6854
У людини визначили величину енерговитрат. У якому стані знаходилась людина, якщо її енерговитрати виявилися меншими за основний обмін?

Нервове напруження

Відпочинок

Легка робота

Сон

Спокій

1470 / 6854
У новонародженого хлопчика під час огляду зовнішніх статевих органів виявлена розщілина сечівника, яка відкривається на нижній поверхні статевого члена. Про яку аномалію йдеться?

Монорхізм

Гіпоспадія

Крипторхізм

Епіспадія

Гермафродитизм

1471 / 6854
Тварині внутрішньовенно ввели концентрований розчин хлориду натрію, що зумовило зниження реабсорбції іонів натрію у канальцях нирок. Внаслідок яких змін секреції гормонів це відбувається?

Зменшення вазопресину

Зменшення натрійуретичного фактора

Збільшення альдостерону

Зменшення альдостерону

Збільшення вазопресину

1472 / 6854
Внаслідок руйнування певних структур стовбуру мозку тварина втратила орієнтувальні рефлекси у відповідь на сильні світлові подразники. Які структури було зруйновано?

Передні горбки чотиригорбкового тіла

Вестибулярні ядра

Чорна речовина

Задні горбки чотиригорбкового тіла

Червоні ядра

1473 / 6854
У цитоплазмі міоцитів розчинена велика кількість метаболітів окиснення глюкози. Назвіть один з них, який безпосередньо перетворюється на лактат:

Оксалоацетат

Фруктозо-6-фосфат

Глюкозо-6-фосфат

Гліцерофосфат

Піруват

1474 / 6854
До стоматолога звернулася мати дитини 2-х років, яка під час вагітності, у зв’язку з інфекційним захворюванням, безсистемно приймала антибіотики. Під час огляду у дитини відзначається руйнування різців, емаль зубів жовтого кольору, у шийки зубів коричнева облямівка. Який з препаратів має виражену тератогенну дію?

Фуросемід

Доксацикліну гідрохлорид

Ампіокс

Ксантинола нікотинат

Октадин

1475 / 6854
У хворого 59-ти років, директора підприємства, після перевірки податкової інспекції з’явився інтенсивний пекучий біль, локалізований за грудниною, який іррадіює в ліву руку. Через 15 хвилин стан хворого нормалізувався. Який можливий механізм стенокардії є провідним у цього хворого?

Атеросклероз коронарних судин

Тромбоз коронарних судин

Внутрішньосудинна агрегація формених елементів

Підвищення у крові рівня катехола-мінів

Функціональне перевантаження серця

1476 / 6854
Пацієнт одужав після перенесеної дизентерії Зонне і повторно заразився цим же збудником. Як називається така форма інфекції?

Персистуюча інфекція

Хронічна інфекція

Рецидив

Реінфекція

Суперінфекція

1477 / 6854
Чоловік 56-ти років потрапив до клініки зі скаргами на загальну слабкість, біль і печіння в язиці, відчуття оніміння в кінцівках. У минулому переніс резекцію кардіального відділу шлунка. У крові: Hb- 80 г/л; ер.- 2,0 • 1012/л; КП-1,2, лейк.- 3,5 • 109 /л. Який вид анемії у цього хворого?

Залізодефіцитна

Апластична

Постгеморагічна

В12 -фолієводефіцитна

Гемолітична

1478 / 6854
Хворий 35-ти років звернувся до лікаря із скаргами на сильну нежить та втрату відчуття запахів протягом тижня. Об’єктивно: в носовій порожнині велика кількість слизу, що вкриває слизову оболонку та блокує рецептори нюху. Де в носовій порожнині розташовані ці рецептори?

Середня носова раковина

Присінок носа

Загальний носовий хід

Верхня носова раковина

Нижня носова раковина

1479 / 6854
Юнак 17-ти років захворів гостро, температура тіла підвищилася до 38,5o C, з’явилися кашель, нежить, сльозотеча, виділення з носу. Яке запалення розвинулося у юнака?

Геморагічне

Серозне

Фібринозне

Гнійне

Катаральне

1480 / 6854
У хворого з порушенням мозкового кровотоку порушений акт ковтання. Який відділ мозку постраждав?

Шийний відділ спинного мозку

Передній мозок

Проміжний мозок

Середній мозок

Стовбур мозку

1481 / 6854
Внаслідок дефіциту вітаміну B1 порушується окисне декарбоксилювання α-кетоглутарової кислоти. Синтез якого з наведених коферментів порушується при цьому?

Нікотинамідаденіндинуклеотид

Коензим А

Флавінаденіндинуклеотид

Тіамінпірофосфат

Ліпоєва кислота

1482 / 6854
Після хімічного опіку в хворого розвинувся стеноз стравоходу. Виникло різке схуднення від затрудненого прийому їжі. У крові: ер.- 3,0 • 1012/л, Hb-106 г/л, загальний білок - 57 г/л. Який вид голодування в хворого?

Повне

Абсолютне

Водне

Білкове

Неповне

1483 / 6854
У жінки 30-ти років виникли набряки обличчя. При обстеженні виявлені протеїнурія (5,87 г/л), гіпопротеїнемія, диспротеїнемія, гіперліпідемія. Для якого стану характерно таке поєднання симптомів?

Нефротичний синдром

Хронічна ниркова недостатність

Нефритичний синдром

Хронічний пієлонефрит

Гостра ниркова недостатність

1484 / 6854
У померлого внаслідок серцевої недостатності на шкірі відзначаються сліди висипу у вигляді плям і крапок. У ділянці крижі, остистих відростків хребців - пролежні. При мікроскопічному дослідженні ЦНС, шкіри, надниркових залоз, в судинах мікроциркуляторного русла і дрібних артеріях - деструктивно-проліферативний ендотромбоваскуліт з наявністю гранульом Попова; в серці - інтерстиційний міокардит. Який з перелічених діагнозів найбільш вірогідний?

Ку -гарячка

Вузликовий періартеріїт

ВІЛ-інфекція

Черевний тиф

Висипний тиф

1485 / 6854
На розтині померлого від сепсису, в стегновій кістці нижньої кінцівки виявлено флегмонозне запалення, що охоплює кістковий мозок, гаверсові канали та періост. Під періостом - множинні абсцеси, в навколишніх м’яких тканинах стегна - також флегмонозне запалення. Який патологічний процес має місце?

Остеопетроз

Гострий гематогенний остеомієліт

Остеопороз

Хронічний гематогенний остеомієліт

1486 / 6854
У хворого знижений синтез вазопресину, що призводить до поліурії і, як наслідок, до вираженої дегідратації організму. У чому полягає механізм розвитку поліурії?

Зниження канальцевої реабсорбції білку

Зниження реабсорбції глюкози

Зниження канальцевої реабсорбції іонів Na

Зниження канальцевої реабсорбції води

Збільшення швидкості клубочкової фільтрації

1487 / 6854
У хворого з нефротичним синдромом спостерігаються масивні набряки обличчя та кінцівок. Який патогенетичний механізм є провідним в розвитку набряків?

Підвищення гідродинамічного тиску крові

Підвищення лімфовідтоку

Лімфостаз

Підвищення судинної проникності

Зниження онкотичного тиску крові

1488 / 6854
Під шкірою піднижньощелепної ділянки у жінки 55-ти років виявлено рухоме утворення 1,0x0,7 см, з чіткими межами, тістуватої консистенції, з повільним ростом. При гістологічному дослідженні утворення виявлені ліпоцити, які формують часточки, різні за формою та розміром, розмежовані тонкими прошарками сполучної тканини із судинами. Який найбільш вірогідний діагноз?

Фібросаркома

Ангіома

Ліпосаркома

Фіброма

Ліпома

1489 / 6854
При щелепно-лицевій операції в стаціонарі, лікар-стоматолог для премедикації, з метою зменшення почуття страху та для потенціювання дії анестетиків, призначив хворому транквілізатор - похідне бензодіазепіну. Який з перерахованих засобів призначив лікар?

Дроперідол

Атропіну сульфат

Діазепам

Сульпірид

Аміназин

1490 / 6854
До приймального відділення лікарні надійшов непритомний юнак з ознаками отруєння морфіном. Відзначається поверхневе та рідке дихання, яке обумовлене пригніченням дихального центру. Який тип недостатності дихання виник при цьому?

Дифузійна

Вентиляційна рестриктивна

Вентиляційна обструктивна

Перфузійна

Вентиляційна дисрегуляторна

1491 / 6854
У хворого на жовчнокам’яну хворобу розвинулася механічна жовтяниця. Під час обстеження встановлено, що камінь знаходиться у загальній жовчній протоці. Які жовчовивідні протоки утворюють обтуровану протоку?

Ductus hepaticus dexter et ductus cysticus

Ductus hepaticus communis et ductus choledochus

Ductus hepaticus communis et ductus cysticus

Ductus hepaticus dexter et sinister

Ductus hepaticus sinister et ductus cysticus

1492 / 6854
При лабораторному дослідженні крові пацієнта 44-х років виявлено, що вміст білків у плазмі становить 40 г/л. Як це впливає на транскапілярний обмін води?

Зменшується фільтрація, збільшується реабсорбція

Обмін не змінюється

Збільшується фільтрація, зменшується реабсорбція

Збільшуються фільтрація та реабсорбція

Зменшуються фільтрація та реабсорбція

1493 / 6854
З метою диференційної діагностики менінгітів проводять дослідження спинномозкової рідини. В якому місці люмбальна пункція є безпечною?

LII -LIII

LI -LII

LV -SI

ThXII -LI

LIII -LIV

1494 / 6854
Ізольована клітина серця людини автоматично генерує імпульси збудження з частотою 60 разів за хвилину. З якої структури серця отримано цю клітину?

Шлуночок

Передсердя

Пучок Гіса

Синоатріальний вузол

Атріовентрикулярний вузол

1495 / 6854
Внаслідок посттрансляційних змін деяких білків, що приймають участь в зсіданні крові, зокрема протромбіну, вони набувають здатності зв’язувати кальцій. В цьому процесі бере участь вітамін:

C

В1

B2

K

А

1496 / 6854
У людини, що виконувала важку фізичну роботу в умовах підвищеної температури навколишнього середовища, змінилася кількість білків плазми крові. Що саме має місце у даному випадку?

Парапротеїнемія

Диспротеїнемія

Абсолютна гіперпротеїнемія

Абсолютна гіпопротеїнемія

Відносна гіперпротеїнемія

1497 / 6854
У хворого на тромбофлебіт нижніх кінцівок з’явився біль у грудній клітці, кровохаркання, наростаюча дихальна недостатність, при явищах якої він помер. На розтині діагностовано множинні інфаркти легень. Яка найбільш вірогідна причина їх розвитку в цьому випадку?

Тромбоемболія гілок легеневої артерії

Тромбоз легеневих вен

Тромбоемболія бронхіальних артерій

Тромбоз бронхіальних артерій

Тромбоз гілок легеневої артерії

1498 / 6854
У жінки, що хворіє на остеохондроз, з’явився різкий біль у плечовому суглобі, який посилювався при відведенні плеча. Ураження якого нерва може бути причиною цих симптомів?

Підключичний нерв

Пахвовий нерв

Підлопатковий нерв

Дорсальний нерв лопатки

Грудо-спинний нерв

1499 / 6854
У хворого глибока різана рана на задній поверхи плеча в середній його третині. Про ушкодження якого м’яза можна думати?

Ліктьовий м’яз

Плечовий м’яз

Двоголовий м’яз плеча

Дзьобоподібно-плечовий м’яз

Трьохголовий м’яз плеча

1500 / 6854
Хворому, що страждає на стенокардію та приймає ізосорбіду мононітрат, було додатково призначено лікарський засіб з дезагрегантним ефектом. Визначте цей препарат:

Ацетилсаліцилова кислота

Нітрогліцерин

Анаприлін

Валідол

Ніфедипін

1501 / 6854
Хворому в післяопераційному періоді для стимуляції перистальтики кишечника та тонусу сечового міхура було призначено препарат з групи антихолінестеразних засобів. Визначте його серед нижче наведених препаратів:

Маніт

Резерпін

Анаприлін

Прозерин

Дихлотіазид

1502 / 6854
У хворого на обличчі вугри. Під час мікроскопії зіскрібків із уражених ділянок виявлені живі членистоногі розміром 0,2-0,5 мм, які мають витягнуту червоподібну форму, чотири пари коротких кінцівок, що розташовані у середній частині тіла. Який лабораторний діагноз?

Демодекоз

Фтиріоз

Педикульоз

Міаз

Короста

1503 / 6854
У чоловіка 28-ми років після вогнепального поранення гомілки розвинулася виразка на боці пошкодження. Що є основним у патогенезі нейродистрофії в даному випадку?

Порушення мікроциркуляції

Інфекція

Психічний стрес

Пошкодження тканини

Травматизація периферичного нерва

1504 / 6854
Чоловік 45-ти років звернувся до лікаря з приводу бляшкоподібного утворення на шиї. В біоптаті шкіри гістологічно виявлено пухлинні клітини, розташовані гніздами, які мають круглу та овальну форму з вузьким ободком базофільної цитоплазми та схожі на клітини базального шару епідермісу. Як називається пухлина в пацієнта?

Сирінгоаденома

Базаліома

Гідраденома

Епідермальний рак

Трихоепітеліома

1505 / 6854
Хворому на гіпертонічну хворобу лікар запропонував засіб, який припиняє тромбоутворення, вводиться парентерально. Оберіть цей засіб:

Неодикумарин

Гепарин

Протаміну сульфат

Амбен

Синкумар

1506 / 6854
У людини до травми гематокритний показник 40%. Яким він буде через добу після втрати 750 мл крові?

45%

40%

55%

50%

30%

1507 / 6854
У студента через 2 години після іспиту в аналізі крові виявлено лейкоцитоз без істотних змін у лейкоцитарній формулі. Який найбільш вірогідний механізм розвитку лейкоцитозу?

Посилення лейкопоезу

Уповільнення руйнування лейкоцитів

Перерозподіл лейкоцитів в організмі

Уповільнення міграції лейкоцитів у тканини

Посилення лейкопоезу та зменшення руйнування лейкоцитів

1508 / 6854
Комплекс Гольджі виводить речовини із клітини завдяки злиттю мембранного мішечка з мембраною клітини. При цьому вміст мішечка виливається назовні. Який процес тут проявляється?

Ендоцитоз

Екзоцитоз

Активний транспорт

Полегшена дифузія

Жодна відповідь невірна

1509 / 6854
Відпочиваючи на дачі, хлопчик знайшов павука з наступними морфологічними особливостями: довжина - 2 см, кулясте черевце чорного кольору, на спинному боці якого видно червоні плямочки у два ряди, чотири пари членистих кінцівок вкриті дрібними чорними волосками. Визначте дане членистоноге:

Кліщ

Скорпіон

Фаланги

Тарантул

Каракурт

1510 / 6854
Дитина 3-х років із симптомами стоматиту, гінгівіту, дерматиту відкритих ділянок шкіри була госпіталізована. При обстеженні встановлено спадкове порушення транспорту нейтральних амінокислот у кишечнику. Нестача якого вітаміну зумовила дані симптоми?

Пантотенова кислота

Біотин

Ніацин

Кобаламін

Вітамін A

1511 / 6854
Хворому з підозрою на діагноз 'прогресуюча м’язова дистрофія' був зроблений аналіз сечі. Наявність якої сполуки в сечі підтверджує діагноз?

Креатин

Колаген

Кальмодулін

Міоглобін

Порфірин

1512 / 6854
У хворого, який знаходиться на лікуванні з приводу вірусного гепатиту В, з’явилися ознаки печінкової недостатності. Які зміни крові, що свідчать про порушення білкового обміну, найбільш вірогідно спостерігатимуться у цьому випадку?

Абсолютна гіперфібриногенемія

Білковий склад крові не змінений

Абсолютна гіпоальбумінемія

Абсолютна гіперальбумінемія

Абсолютна гіперглобулінемія

1513 / 6854
У вагітної жінки визначили групу крові. Реакція аглютинації еритроцитів відбулася зі стандартними сироватками груп 0ав (I), Bα (III) і не відбулася зі стандартною сироваткою групи Aβ (II). Досліджувана кров належить до групи:

Aβ (II)

Bα (III)

0αβ (I)

AB (IV)

1514 / 6854
Чутливий нервовий ганглій складається з нейроцитів кулястої форми з одним відростком, який на певній відстані від перикаріону поділяється на аксон і дендрит. Як називаються такі клітини?

уніполярні

біполярні

псевдоуніполярні

аполярні

мультиполярні

1515 / 6854
При обстеженні дитини лікар виявив ознаки рахіту. Нестача якої сполуки в організмі дитини сприяє розвитку цього захворювання?

1,25 [ОН]- дигідроксіхолекальциферол

Токоферол

Ретинол

Біотин

Нафтохінон

1516 / 6854
У зародка порушено процес сегментації дорзальної мезодерми та утворення сомітів. В якій частині шкіри можливі порушення розвитку?

Сальні залози

Волосся

Епідерміс

Потові залози

Дерма

1517 / 6854
Встановлено, що деякі сполуки, наприклад, токсини грибів та деякі антибіотики, можуть пригнічувати активність РНК-полімерази. Порушення якого процесу відбувається в клітині у випадку пригнічування даного ферменту?

Трансляція

Реплікація

Транскрипція

Репарація

Процесінг

1518 / 6854
Матеріалом від дитини з попереднім діагнозом 'ентеровірусна інфекція'заразили культуру клітин мавпи (Vero) і мишат-сисунків, в результаті не виявлено цитопатичного ефекту на культурі клітин, але зареєстрована загибель мишат-сисунків. Які ентеровіруси могли викликати захворювання у цієї дитини?

Коксакі А

Некласифіковані ентеровіруси 68-71

Коксакі В

Поліовіруси

ECHO

1519 / 6854
Хворий помер від наростаючої легенево-серцевої недостатності. При гістологічному дослідженні виявлено: дифузне ураження легенів з інтерстиціальним набряком, інфільтрацією інтерстиціальної тканини лімфоцитами, макрофагами, плазмоцидами; пневмофіброз, панацинарна емфізема. Який найбільш вірогідний діагноз?

Хронічний бронхіт

Бронхопневмонія

Фіброзуючий альвеоліт

Бронхіальна астма

Ателектаз легенів

1520 / 6854
В експерименті подразнюють гілочки блукаючого нерва, які іннервують серце. Це призвело до того, що припинилося проведення збудження від передсердь до шлуночків. Електрофізіологічні зміни в яких структурах серця є причиною цього?

Передсердя

Синоатріальний вузол

Шлуночки

Пучок Гіса

Атріовентрикулярний вузол

1521 / 6854
У людини з нападом бронхоспазму необхідно зменшити вплив блукаючого нерва на гладеньку мускулатуру бронхів. Які мембранні циторецептори доцільно заблокувати для цього?

α- та β-адренорецептори

β-адренорецептори

H-холінорецептори

M-холінорецептори

α-адренорецептори

1522 / 6854
У пацієнтки хірургічного відділення з’явилися скарги на біль у попереку та в низу живота, болісне і часте сечовипускання. Після бактеріологічного дослідження сечі виявлені грамнегативні оксидазапозитивні паличкоподібні бактерії, що утворюють мукоїдні колонії зеленуватого кольору зі специфічним запахом. Про який збудник можна думати?

Mycoplasma pneumonie

Str.pyogenes

Pseudomonas aeruginosa

Proteus mirabilis

E.coli

1523 / 6854
Виділяють декілька груп молекулярних механізмів, які мають важливе значення в патогенезі ушкодження клітин, що сприяє розвитку патології. Які процеси забезпечують протеїнові механізми ушкодження?

Активація фосфоліпаз

Ацидоз

Перекисне окиснення ліпідів

Пригнічення ферментів

Осмотичне розтягнення мембран

1524 / 6854
Дитина народилася з вовчою пащею. При обстеженні виявлено вади аорти, в крові - зменшення Т-лімфоцитів. Який імунодефіцитний синдром у новонародженого?

Ді Джорджі

Луї-Бар

Чедіака-Хігасі

Віскотта-Олдріча

Швейцарський тип

1525 / 6854
У юнака 16-ти років після перенесеного захворювання знижена функція синтезу білків у печінці внаслідок нестачі вітаміну K. Це може призвести до порушення:

Утворення антикоагулянтів

Утворення еритропоетинів

Осмотичного тиску крові

Зсідання крові

Швидкості зсідання еритроцитів

1526 / 6854
У життєвому циклі клітини відбувається процес самоподвоєння ДНК. В результаті цього однохроматидні хромосоми стають двохроматидними. У який період клітинного циклу спостерігається це явище?

G1

G0

G2

M

S

1527 / 6854
У водія, який потрапив у ДТП, отримав травму та знаходиться у стані шоку, спостерiгається зменшення добової кількості сєчі до 300 мл. Який основний патогенетичний фактор цієї зміни діурезу?

Вторинний гіперальдостеронізм

Зниження онкотичного тиску крові

Падіння артеріального тиску

Зменшення кількості функціонуючих клубочків

Підвищення проникності судин

1528 / 6854
У дорослої людини системний артеріальний тиск знизився з 120/70 до 90/50 мм рт.ст., що викликало рефлекторне звуження судин. У якому з зазначених органів звуження судин буде найбільшим?

Головний мозок

Наднирники

Кишечник

Нирки

Серце

1529 / 6854
Під час обстеження 12-ти річного хлопчика, який відстає у рості, виявлена ахондроплазія: непропорційна тілобудова з помітним вкороченням рук і ніг, внаслідок порушення росту епіфізарних хрящів довгих трубчастих кісток. Дане захворювання є:

Вродженим

Спадковим за рецесивним типом

Набутим

Спадковим, зчепленим зі статтю

Спадковим за домінантним типом

1530 / 6854
У хворого виявлена аутоімунна гемолітична анемія, що розвивається за цитотоксичним типом. Які речовини є антигенами при алергічних реакціях II типу?

Модифіковані рецептори клітинних мембран

Сироваткові білки

Модулятори запалення

Антибіотики

Гормони

1531 / 6854
При виконанні оперативного втручання в ділянці пахвової порожнини, хірургу необхідно виділити артеріальну судину, що оточена пучками плечового сплетіння. Яка це артерія?

A.profunda brachii

A.vertebralis

A.subscapularis

A.axillaris

A.transversa colli

1532 / 6854
Для лікування хронічної серцевої недостатності хворий приймає дигоксин. Який діуретичний засіб може збільшити токсичність дигоксину за рахунок посиленого виведення з організму іонів K+?

Лізиноприл

Силібор

Гідрохлортіазид

Спіронолактон

Панангін

1533 / 6854
Під час голодування м’язові білки розпадаються до вільних амінокислот. В який процес найімовірніше будуть втягуватися ці сполуки за таких умов?

Глікогеноліз

Глюконеогенез у м’язах

Глюконеогенез у печінці

Декарбоксилування

Синтез вищих жирних кислот

1534 / 6854
Хворій на виразкову хворобу шлунка в якості етіотропної терапії призначили препарат, що пригнічує ріст і розмноження H.pylori. Назвіть цей препарат:

Метронідазол

Празозин

Корглікон

Glauvent

Фуросемід

1535 / 6854
Після оперативного видалення частини шлунка у хворого порушилося всмоктування вітаміну B12, він виводиться з калом. Розвинулася анемія. Який фактор необхідний для всмоктування цього вітаміну?

Гастрин

Соляна кислота

Гастромукопротеїн

Пепсин

Фолієва кислота

1536 / 6854
Хворому на хронічну серцеву недостатність призначили дигоксин у середньотерапевтичній дозі. Через 2 тижні після початку прийому препарату у нього з’явились ознаки інтоксикації препаратом (брадикардія, екстрасистолія, нудота). Як називається явище, яке призвело до накопичення в організмі препарату?

Кумуляція

Толерантність

Тахіфілаксія

Сенсибілізація

Ідіосинкразія

1537 / 6854
При проходженні профілактичного огляду у шахтаря лікар встановив зміни функціонального стану серця, що свідчать про серцеву недостатність в стадії компенсації. Що з нижче перерахованого є головним підтвердженням компенсації діяльності серця?

Гіпертрофія міокарда

Ціаноз

Збільшення артеріального тиску

Тахікардія

Задишка

1538 / 6854
Потенціал спокою клітини дорівнює -80 мВ. Під час якої фази ПД величина мембранного потенціалу склала +30 мВ?

Слідової гіперполяризації

Деполяризації

Слідової деполяризації

Реверсполяризації

1539 / 6854
У результаті поранення у чоловіка 35-ти років настав повний розрив спинного мозку на рівні першого шийного сегменту. Як зміниться характер дихання?

Не зміниться

Збережеться діафрагмальне, щезне грудне

Збережеться грудне, щезне діафрагмальне

Стане рідким та глибоким

Зупиниться

1540 / 6854
У піддослідної тварини під час експерименту подразнюють периферичний відрізок блукаючого нерва. Які з наведених змін будуть спостерігатися при цьому?

Збільшення частоти серцевих скорочень

Розширення зіниць

Розширення бронхів

Збільшення частоти дихання

Зменшення частоти серцевих скорочень

1541 / 6854
У хворого 27-ми років з політравмою (закрита травма грудної клітини, закритий перелом правого стегна) через дві години після проведення скелетного витягнення різко погіршився стан і, на фоні гострої легенево-серцевої недостатності, настала смерть. При гістологічному дослідженні кровоносних судин легень та головного мозку померлого, при забарвленні суданом ІІІ, виявлені краплі помаранчевого кольору, які закупорюють просвіт судин. Яке ускладнення політравми розвинулося у хворого?

Тромбоемболія

Повітряна емболія

Газова емболія

Мікробна емболія

Жирова емболія

1542 / 6854
У хворого з важким перебігом респіраторної вірусної інфекції з’явилися клінічні ознаки прогресуючої серцевої недостатності, яка призвела до смерті хворого на 2-му тижні захворювання. На розтині серце зі значним розширенням порожнин, в’яле. Гістологічно в міокарді виявляється повнокров’я мікросудин і дифузна інфільтрація строми лімфоцитами та гістіоцитами. Вкажіть найбільш вірогідний діагноз:

Міокардит

Кардіоміопатія

Стенокардія

Гостра коронарна недостатність

Інфаркт міокарда

1543 / 6854
В препараті, зафарбованому за методом Ожешко, видно паличкоподібні мікроорганізми, зафарбовані в синій колір, в яких термінально розміщені компоненти округлої форми, зафарбовані в червоний колір. Як називаються ці компоненти?

Спори

Війки

Джгутики

Капсули

Мезосоми

1544 / 6854
Проведена пункційна біопсія печінки хворому 38-ми років з виразною жовтяницею, дрібними крововиливами у шкірі, загальною слабкістю, втратою апетиту. Патологічне дослідження виявило поширену дистрофію, некроз гепатоцитів, наявність тілець Каунсілмена. По периферії часточок значна інфільтрація лімфоцитами, зустрічаються окремі багатоядерні гепатоцити. Який найбільш вірогідний діагноз?

Вірусний гепатит гострий

Алкогольний гепатит гострий

Хронічний гепатит

Міліарний цироз печінки

Токсична дистрофія печінки

1545 / 6854
До реанімаційного відділення надійшов чоловік з пораненням задньої ділянки шиї (regio nuchae). Який з м’язів тіла займає цю ділянку?

M.latissimus dorsi

M.scalenus anterior

M.sternocleidomastoideus

M.trapezius

M.rhomboideus minor

1546 / 6854
В експерименті на жабі зруйнували лабіринт з правого боку. До зниження тонусу яких м’язів це призведе?

Екстензори праворуч та ліворуч

Екстензори ліворуч

Екстензори праворуч

Флексори ліворуч

Флексори праворуч

1547 / 6854
Хвора 20-ти років померла від інтоксикації через 8 днів після штучного позалікарняного аборту при терміні вагітності 14-15 тижнів. На розтині тіла померлої: жовтувате забарвлення склери очей, шкіри, гнійно-некротичний ендометрит, багаточисельні гнійники в легенях, гіперплазія селезінки з великою кількістю нейтрофілів в її синусах. Яке ускладнення після аборту розвинулося у хворої?

Хроніосепсис

Геморагічний шок

Вірусний гепатит А

Септицемія

Септикопіємія

1548 / 6854
Хворий скаржиться на запаморочення і втрату слуху. Який нерв ушкоджений?

Блукаючий

Трійчастий

Під’язиковий

Блоковий

Присінково-завитковий

1549 / 6854
До лікарні надійшла дитина 6-ти років. Під час обстеження було виявлено, що дитина не може фіксувати погляд, не слідкує за іграшками, на очному дні відзначається симптом 'вишневої кістки'. Лабораторні аналізи показали, що у мозку, печінці та селезінці - підвищений рівень гангліозиду глікометиду. Яке спадкове захворювання у дитини?

Хвороба Тея-Сакса

Хвороба Німана-Піка

Синдром Шерешевського-Тернера

Хвороба Вільсона-Коновалова

Хвороба Мак-Аргдля

1550 / 6854
У хворого на шиї виявлено пакет спаяних між собою лімфовузлів щільної консистенції. При гістологічному обстеженні видаленого лімфовузла відмічається проліферація ретикулярних клітин, наявність клітин Березовського-Штернберга. Про яке захворювання йдеться?

Мієлоцитарний лейкоз

Лімфогранулематоз

Лімфоцитарний лейкоз

Лімфобластний лейкоз

Мієлобластний лейкоз

1551 / 6854
Під час пальпації молочної залози у хворої виявлено ущільнення у вигляді вузла у нижньому медіальному квадранті. У які лімфатичні вузли можуть поширитися метастази при цьому?

Задні середостінні

Верхні діафрагмальні

Бронхолегеневі

Глибокі латеральні шийні

Пригруднинні

1552 / 6854
Вагітній жінці, під час встановлення на облік у жіночу консультацію, було проведено клінічне обстеження на ряд інфекцій. У сироватці крові були виявлені lgM до вірусу краснухи. Про що свідчить такий результат?

Хронічний процес

Жінка здорова

Загострення хронічного процесу

Повторне інфікування вірусом краснухи

Первинне зараження жінки

1553 / 6854
Батьки для профілактики кишкових інфекцій у дитини 3-х років тривало застосовували антибіотики. Через місяць стан дитини погіршився. У крові - виражена лейкопенія і гранулоцитопенія. Який найбільш вірогідний механізм виявлених змін у крові?

Віковий

Гемолітичний

Перерозподільний

Мієлотоксичний

Аутоімунний

1554 / 6854
У результаті патологічного процесу в бронхах відбувається десквамація епітелію. За рахунок яких клітин відбуватиметься регенерація бронхіального епітелію?

Ендокринні

Келихоподібні

Базальні

Вставочні

Війчасті

1555 / 6854
Хворий, що страждав на туберкульоз, помер від прогресуючої легенево-серцевої недостатності. На розтині в ділянці верхівки правої легені визначається порожнина діаметром 5 см, яка сполучається з просвітом сегментарного бронха. Стінки порожнини з середини покриті сирнистими масами, під якими знаходяться епітеліоїдні клітини і клітини Пирогова-Лангханса. Вкажіть морфологічну форму туберкульозу:

Інфільтративний туберкульоз

Казеозна пневмонія

Гострий осередковий туберкульоз

Туберкульома

Гострий кавернозний туберкульоз

1556 / 6854
Робітники парникового господарства працюють в умовах несприятливого мікроклімату: температура повітря +37oC, відносна вологість 90%, швидкість руху повітря 0,2 м/с. Яким шляхом здійснюється тепловіддача за цих умов?

Усі зазначені шляхи

Радіація

Конвекція

Теплопроведення

Випаровування

1557 / 6854
При огляді хворої лікар-гінеколог відмітив симптоми запалення статевих шляхів. У мазку, взятому із піхви виявлено овально-грушоподібні найпростіші з шипом, з передньої частини яких відходять джгутики; наявна ундулююча мембрана. Яке захворювання підозрює лікар у хворої?

Кишковий трихомоноз

Урогенітальний трихомоноз

Балантидіоз

Токсоплазмоз

Лямбліоз

1558 / 6854
У хлопчика I (IoIo) група крові, а в його сестри IV(IAIB). Які групи крові у батьків цих дітей?

III (IBIo) та IV (IАIB)

II (ІАІА) та III (IBIo)

I (IoIo) та IV (IAIB)

I (IoIo) та III (IBIo)

II (IAIо) та III (IBIo)

1559 / 6854
У хворого на ентерит, що супроводжувався значною діареєю, спостерігається зменшення кількості води в позаклітинному просторі, збільшення її всередині клітин та зниження осмолярності крові. Як називають таке порушення водно-електролітного обміну?

Осмолярна гіпогідратація

Гіпоосмолярна гіпергідратація

Гіперосмолярна гіпогідратація

Гіпоосмолярна гіпогідратація

Гіперосмолярна гіпергідратація

1560 / 6854
У клініці для лікування інфаркту міокарда пацієнту введено ембріональні стовбурові клітини, що одержано шляхом терапевтичного клонування в цього ж пацієнта. Як називається цей вид трансплантації?

Аутотрансплантація

Гетеротрансплантація

Ізотрансплантація

Алотрансплантація

Ксенотрансплантація

1561 / 6854
На гістологічному препараті селезінки виявлена судина, стінка якої складається з ендотелію та субендотеліального шару, середня оболонка відсутня, зовнішня оболонка зрощена зі сполучнотканинними прошарками селезінки. Що це за судина?

Артерія м’язового типу

Капіляр

Вена м’язового типу

Вена безм’язового типу

Артеріола

1562 / 6854
В умовному експерименті повністю інгібовано розвиток клітин мезенхіми. Порушення розвитку якої м’язової тканини при цьому буде спостерігатися?

Гладенька м’язова тканина

М ’язова тканина нейрального походження

Серцева м’язова тканина

М’язова тканина епідермального походження

Скелетна м’язова тканина

1563 / 6854
Хворому на хронічний гастрит зроблена внутрішньошлункова рН-метрія, за допомогою якої встановлено зниження кислотності шлункового соку. Функція яких клітин знижена?

Додаткові клітини

Парієтальні екзокриноцити

Ендокриноцити

Шийкові клітини

Головні екзокриноцити

1564 / 6854
В експерименті на тварині здійснили перерізку блукаючих нервів з обох боків. Як при цьому зміниться характер дихання?

Стане глибоким і рідким

Стане поверхневим та рідким

Дихання не зміниться

Стане поверхневим та частим

Стане глибоким і частим

1565 / 6854
Чоловік 70-ти років хворіє на атеросклероз судин нижніх кінцівок та ішемічну хворобу серця. Під час обстеження виявлено порушення ліпідного складу крові. Надлишок яких ліпопротеїнів є головною ланкою в патогенезі атеросклерозу?

Хіломікрони

Низької щільності

Високої щільності

Проміжної щільності

Холестерин

1566 / 6854
У хворого після операції резекції шлунка на 2-3-й день не відновилася перистальтика кишок. Що потрібно призначити хворому для стимуляції функції шлунково-кишкового тракту?

Норадреналіну гідротартат

Празозин

Циклодол

Атропіну сульфат

Прозерин

1567 / 6854
Хворому чоловіку 75-ти років, у якого частота серцевих скорочень була 40/хвилину, імплантували серцевий електростимулятор. Після цього частота серцевих скорочень зросла до 70/хв. Функцію якого відділу серця взяв на себе електростимулятор?

Волокна Пуркін’є

Ніжки Гіса

Атріовентрикулярний вузол

Синоатріальний вузол

Волокна пучка Гіса

1568 / 6854
Стоматолог з метою зменшення салівації під час пломбування зуба хворому призначив препарат. Що це за лікарський засіб?

Мезатон

Прозерин

Пілокарпіну гідрохлорид

Адреналіну гідрохлорид

Атропіну сульфат

1569 / 6854
У жінки 30-ти років після тривалого використання губної помади з флюоресцуючою речовиною на облямівці губ розвинулася обмежена еритема, незначне лущення, пізніше поперечні дрібні борозни та тріщини. Після спеціальних методик при мікроскопічному дослідженні цієї зони ураження: в сполучній тканині наявність сенсибілізованих лімфоцитів і макрофагів, явища цитолізу. Який тип імунологічної гіперчутливості розвинувся на губі?

III тип (імунокомплексна цитотокси-чність)

I тип (реагінового типу)

IV тип (клітинна цитотоксичність)

Гранулематоз

II тип (антитільна цитотоксичність)

1570 / 6854
У хворого на ессенціальну артеріальну гіпертензію розвинувся гіпертонічний криз, що призвело до нападу серцевої астми. Який механізм серцевої недостатності є провідним в даному випадку?

Перевантаження серця збільшеним об’ємом крові

Порушення надходження крові до серця

Перевантаження серця підвищеним опором

Пошкодження міокарда

Абсолютна коронарна недостатність

1571 / 6854
Людина, що тривалий час приймає ліки, не може різко припинити їх вживання, оскільки при цьому виникають порушення психічних та соматичних функцій. Як називається синдром різних порушень при відмові від прийому речовини?

Сенсибілізація

Тахіфілаксія

Абстиненція

ідіосинкразія

Кумуляція

1572 / 6854
Хворий звернувся до стоматолога зі скаргами на гнійне запалення ясен. Який препарат буде найбільш ефективним, якщо припускається анаеробна природа збудника?

Нітроксолін

Ко-тримоксазол

Гентаміцин

Метронідазол

Оксацилін-натрій

1573 / 6854
У щурів, що знаходяться у стані стресу, підвищені м’язовий тонус та артеріальний тиск, збільшений вміст глюкози у крові, посилена секреція кортикотропіну і кортикостероїдів. У якій фазі стресу знаходяться ці тварини?

Еректильна

Виснаження

Фаза шоку

Термінальна

Фаза протишоку

1574 / 6854
При ультрамікроскопічному дослідженні популяції 'темних' гепатоцитів у цитоплазмі клітин визначено розвинену гранулярну ендоплазматичну сітку. Яку функцію в даних клітинах виконує ця органела?

Депонування іонів кальцію

Продукція жовчі

Синтез білків плазми крові

Синтез вуглеводнів

Дезінтоксикаційна

1575 / 6854
В лікарню звернувся хворий зі скаргами на швидку стомлюваність і виражену м’язову слабкість. При обстеженні виявлено аутоімунне захворювання, внаслідок якого порушується функціональний стан рецепторів у нервово-м’язових синапсах. Дія якого медіатора буде заблокована?

Ацетилхолін

Серотонін

Гліцин

Норадреналін

Дофамін

1576 / 6854
У пацієнта 35-ти років виявили підвищену кислотність шлункового соку. Блокада яких рецепторів може спричинити її зниження?

α1 -адренорецептори

Гістамінові

β1 -адренорецептори

β2 -адренорецептори

α2 -адренорецептори

1577 / 6854
По ходу слухового нерва у молодої жінки виявлена пухлина у вигляді вузла до 3 см в діаметрі, м’яко-еластичної консистенції,

рожево-білого кольору, однорідна. Мікроскопічно пухлина містить пучки клітин з овальними ядрами. Клітинно-волокнисті пучки формують ритмічні структури, створені паралельними рядами, правильно орієнтованими клітинами, розташованими у вигляді частоколу, поміж яких знаходиться безклітинна гомогенна зона (тільця Верокаї). Що це за пухлина?

Невринома

Нейробластома

Гангліоневрома

Гангліонейробластома

Злоякісна невринома

1578 / 6854
Жінка 45-ти років хворіє на рак лівої молочної залози. На лівій руці є ознаки недостатності лімфатичної системи - набряк кінцівки, збільшення лімфовузлів. Яка форма недостатності лімфообміну спостерігається у хворої?

Резорбційна недостатність

Механічна недостатність

Динамічна недостатність

Змішана недостатність

1579 / 6854
Для морфологічного дослідження представлена ендокринна залоза, паренхіма якої складається з епітелію та нервової тканини. В епітеліальних трабекулах виявляється два типи клітин: хромофільні та хромофобні. Визначте даний орган:

Щитоподібна залоза

Гіпоталамус

Надниркова залоза

Прищитоподiбна залоза

Гіпофіз

1580 / 6854
Під час операції на пахвинному каналі з приводу грижі, хірург пошкодив його вміст. Що саме пошкодив хірург?

Urarchus

Lig. inguinale

Lig. teres uteri

Funiculus spermaticus

1581 / 6854
У дитини 12-ти років непереносимість ряду харчових продуктів. Їх вживання викликає алергічну реакцію у вигляді висипань на шкірі, що сверблять. Який протигістамінний засіб слід призначити, щоб не заважати шкільним заняттям дитини (не викликати сонливість)?

Еуфілін

Диклофенак-натрію

Мезатон

Димедрол

Лоратадин

1582 / 6854
Чоловік 26-ти років перебуває в торпідній стадії шоку внаслідок автомобільної аварії. В крові: лейк.- 3, 2 • 109 /л. Який головний механізм в розвитку лейкопенії?

Пригнічення лейкопоезу

Руйнування лейкоцитів у кровотворних органах

Підвищення виділення лейкоцитів з організму

Перерозподіл лейкоцитів у судинному руслі

Порушення виходу зрілих лейкоцитів з кісткового мозку в кров

1583 / 6854
У хворого геморагічний інсульт. Виявлено в крові підвищену концентрацію кінінів. Лікар призначив хворому контрікал. Для гальмування якої протеїнази було зроблено це призначення?

Трипсин

Колагеназа

Калікреїн

Пепсин

Хімотрипсин

1584 / 6854
Для лікування бактеріальної пневмонії було призначено бензилпеніциліну натрієву сіль. Який механізм антимікробної дії препарату?

Антагонізм з параамінобензойною кислотою

Пригнічення SH-груп ферментів мікроорганізмів

Пригнічення активності холінестерази

Пригнічення синтезу клітинної стінки мікроорганізмів

Пригнічення внутрішньоклітинного синтезу білка

1585 / 6854
Хвороба Хартнепа зумовлена точковою мутацією лише одного гена, наслідком чого є порушення всмоктування амінокислоти триптофану в кишечнику та реабсорбції її в ниркових канальцях. Це призводить до одночасних розладів у травній і сечовидільній системах. Яке генетичне явище спостерігається в цьому випадку?

Плейотропія

Кодомінування

Неповне домінування

Полімерія

Комплементарна взаємодія

1586 / 6854
З метою прискорення загоєння променевої виразки був призначений вітамінний препарат. Вкажіть його:

Ретаболіл

Ретинолу ацетат

Преднізолон

Метилурацил

Левамізол

1587 / 6854
При посіві матеріалу із зіву від хворого ангіною на кров’яно-телуритовий агар виросли колонії діаметром 4-5 мм, сірого кольору, радіально покреслені (у вигляді розеток). Під мікроскопом - грампозитивні палички із булавоподібними потовщеннями на кінцях, що розміщені у вигляді розтопирених пальців. Які це мікроорганізми?

Стрептококи

Клостридії ботулізму

Стрептобацили

Дифтероїди

Коринебактерії дифтерії

1588 / 6854
Плазмові фактори згортання крові зазнають посттрансляційної модифікації за участю вітаміну K. Як кофактор, він потрібен у ферментній системі у-карбоксилювання білкових факторів коагуляції крові, завдяки збільшенню спорідненості їх молекул з іонами кальцію. Яка амінокислота карбоксилюється в цих білках?

Фенілаланін

Аргінін

Валін

Глутамінова

Серин

1589 / 6854
Фармакологічні ефекти антидепресантів пов’язані з блокуванням (інгібуванням) ними ферменту, який каталізує розпад таких біогенних амінів, як норадреналін і серотонін в мітохондріях нейронів головного мозку. Який фермент бере участь у цьому процесі?

Пептидаза

Трансаміназа

Ліаза

Декарбоксилаза

Моноамінооксидаза

1590 / 6854
Онкологічному хворому призначили препарат метотрексат, до якого з часом клітинимішені пухлини втратили чутливість. Експресія гену якого ферменту при цьому змінюється?

Фолатоксидаза

Тиміназа

Дегідрофолатредуктаза

Фолатдекарбоксилаза

Дезаміназа

1591 / 6854
При диспансерному обстеженні хлопчику 7-ми років встановлено діагноз - синдром ЛешаНайхана (хворіють тільки хлопчики). Батьки здорові, але у дідуся за материнською лінією таке саме захворювання. Який тип успадкування захворювання?

Домінантний, зчеплений із статтю

Рецесивний, зчеплений із статтю

Автосомно-домінантний

Неповне домінування

Автосомно-рецесивний

1592 / 6854
Значну роль у профілактиці туберкульозу відіграє планова масова вакцинація проти туберкульозу новонароджених дітей віком 5-7 днів життя. При цьому застосовують вакцину:

АКП

АД

БЦЖ

АКДП

АДП

1593 / 6854
До кардіологічного відділення надійшов хворий з гіпертонічним кризом, йому внутрішньовенно ввели антигіпертензивний засіб - сіль лужноземельного металу. Який препарат ввели хворому?

Кальцію лактат

Бензогексоній

Натрію гідрокарбонат

Калію хлорид

Магнію сульфат

1594 / 6854
Під час оперативного втручання на малому тазі жінки виникла необхідність перев’язати яєчникову артерію. Яке з утворень може бути випадково перев’язаним разом з нею?

Кругла зв’язка матки

Внутрішня клубова вена

Сечівник

Маткова труба

Сечовід

1595 / 6854
До приймального відділення лікарні доставлено людину з гострим отруєнням морфіном. Який специфічний антагоніст наркотичних анальгетиків необхідно застосувати в цьому випадку?

Налоксон

Парацетамол

Метацин

Унітіол

Дигоксин

1596 / 6854
Верхня частка правої легені збільшена, сірого кольору, безповітряна, з поверхні розрізу стікає каламутна рідина, на плеврі багато фібринозних плівок; мікроскопічно в альвеолах виявляється ексудат з присутністю нейтрофілів, десквамованих альвеолоцитів і ниток фібрину. Стінка бронха інтактна. Який найбільш вірогідний діагноз?

Крупозна пневмонія

Грипозна пневмонія

Інтерстиціальна пневмонія

Вогнищева пневмонія

Абсцес легень

1597 / 6854
У хворого 28-ми років відзначалися підвищення артеріального тиску, гематурія та набряки на обличчі. Незважаючи на лікування, наростали явища ниркової недостатності. Через 6 місяців хворий помер від уремії. Мікроскопічно при дослідженні нирок у клубочках виявлена проліферація нефротелію капсули, подоцитів з утворенням 'півмісяців', склероз та гіаліноз клубочків. Який найбільш вірогідний діагноз?

Нефротичний синдром

Гострий пієлонефрит

Хронічний гломерулонефрит

Підгострий гломерулонефрит

Гострий гломерулонефрит

1598 / 6854
До клініки швидкої допомоги надійшов хворий з ножовим пораненням лівої поперекової ділянки. У ході операції хірург встановив, що внутрішні органи не пошкоджені, але лезо ножа пошкодило один із м’язів ниркового ложа. Назвіть цей м’яз:

Великий поперековий м’яз

Зовнішній косий м’яз живота

Клубовий м’яз

Випрямляч хребта

Внутрішній косий м’яз живота

1599 / 6854
Чоловік з колотою раною в ділянці чотирьохстороннього отвору звернувся до лікаря. При обстеженні виявлено, що потерпілий не може відвести руку від тулуба. Який нерв вірогідно ушкоджений?

N.medianus

N.radialis

N.ulnaris

N.subclavius

N.axillaris

1600 / 6854
Під час розтину тіла чоловіка, що за життя страждав на важку форму гіпотиреозу, виявлено: сполучна тканина, строма органів, жирова і хрящова тканини набряклі, напівпрозорі, слизоподібні. Мікроскопічно у тканинах виявляються зірчасті клітини з відростками, між якими знаходиться слиз. Назвіть вид дистрофії:

Паренхіматозна жирова

Стромально-судинна білкова

Паренхіматозна білкова

Стромально-судинна вуглеводна

Стромально-судинна жирова

1601 / 6854
Хворий 30-ти років звернувся до лікаря зі скаргами на пронос і біль у животі протягом 5-ти днів, підвищення температури тіла до 37,5oC з ознобами. Напередодні хворий був у лісі, де випив води з відкритого водоймища. Встановлено лабораторно підтверджений діагноз: амебна дизентерія. Вкажіть препарат вибору для лікування цього захворювання:

Еметина гідрохлорид

Фуразолідон

Левоміцетин

Метронідазол

Фталазол

1602 / 6854
Чоловік, що страждає на спадкову хворобу, одружився із здоровою жінкою. У них було 5 дітей, три дівчинки і два хлопчика. Усі дівчатка успадкували хворобу батька. Який тип спадкування цього захворювання?

Зчеплений з Y-хромосомою

Рецесивний, зчеплений з X -хромосомою

Домінантний, зчеплений з X -хромосомою

Аутосомно-домінантний

Аутосомно-рецесивний

1603 / 6854
На електронній мікрофотографії представлена клітина макрофагічної природи, вздовж відростків якої розташовуються еритроцити на різних стадіях диференціювання. Клітина якого органу представлена?

Лімфатичний вузол

Селезінка

Тимус

Червоний кістковий мозок

Мигдалик

1604 / 6854
Досить часто причиною набутих імунодефіцитів є інфекційне ураження організму, при якому збудники безпосередньо розмножуються в клітинах імунної системи і руйнують їх. Виберіть серед перерахованих ті захворювання, при яких має місце вищезгадане:

Туберкульоз, мікобактеріоз

Інфекційний мононуклеоз, СНІД

Дизентерія, холера

Ку-гарячка, висипний тиф

Поліомієліт, гепатит А

1605 / 6854
В шкірі виявлена щільна, рухома, чітко відмежована від оточуючих тканин пухлина. На розрізі вона білого кольору, представлена волокнистою тканиною. Мікроскопічно: хаотично переплетені колагенові волокна, клітин мало. Що це за пухлина?

Фіброма

Гістіоцитома

Дерматофіброма

Міома

Десмоїд

1606 / 6854
У хворого після оперативного видалення кісти підшлункової залози виник геморагічний синдром з вираженим порушенням зсідання крові. Розвиток цього ускладнення пояснюється:

Активацією фібринолітичної системи

Активацією фактору Крисмаса

Активацією протизгортальної системи

Недостатнім утворенням фібрину

Зменшенням кількості тромбоцитів

1607 / 6854
Мати звернулася до лікаря з приводу того, що у дитини 5-ти років під дією сонячних променів на шкірі з’являються еритеми, везикулярний висип, свербіж шкіри. Лабораторні дослідження виявили зменшення вмісту заліза у сироватці крові, збільшення виділення з сечею уропорфіриногену І. Найбільш вірогідною спадковою патологією у дитини є:

Еритропоетична порфірія

Метгемоглобінемія

Копропорфірія

Печінкова порфірія

Інтермітуюча порфірія

1608 / 6854
Немовля відмовляється від годування груддю, збудливе, дихання неритмічне, сеча має специфічний запах 'пивної закваски' або 'кленового сиропу'. Вроджений дефект якого ферменту викликав дану патологію?

Аспартатамінотрансфераза

Глюкозо-6-фосфатдегідрогеназа

УДФ-глюкуронілтрансфераза

Гліцеролкіназа

Дегідрогеназа розгалужених альфа-кетокислот

1609 / 6854
У мікропрепараті, виготовленому з пунктату регіонарного лімфовузла хворого, зафарбованому за Романовським-Гімзою, лікар виявив тонкі мікроорганізми з 12-14 рівномірними завитками з гострими кінцями, довжиною 10-13 мкм блідо-рожевого кольору. Про збудника якої інфекційної хвороби може йти мова в даному випадку?

Лептоспіроз

Поворотний тиф

Лейшманіоз

Трипаносомоз

Сифіліс

1610 / 6854
При санітарно-бактеріологічному дослідженні води методом мембранних фільтрів виявлено дві червоні колонії на мембранному фільтрі (середовище Ендо), через який пропустили 500 мл досліджуваної води. Розрахуйте колі-індекс та колі-титр досліджуваної води:

2 та 500

250 та 2

4 та 250

250 та 4

500 та 2

1611 / 6854
Лікар-отоларинголог при огляді хворого відмітив гіперемію, значний набряк мигдаликів з сірим нальотом на них. При мікроскопії нальоту було виявлено грам-позитивні палички, розташовані під кутом одна до одної. Про яке захворювання слід думати?

Менінгоназофарингіт

Ангіна

Скарлатина

Епідемічний паротит

Дифтерія

1612 / 6854
До приймального відділення доставлено жінку 38-ми років з матковою кровотечею, що триває другу добу. Що з наведеного буде виявлено при аналізі крові хворої?

Лейкоцитоз

Збільшення колірного показника

Уповільнення ШОЕ

Зменшення гематокритного показника

Еозинофілія

1613 / 6854
У чоловіка після гіпертонічної кризи відзначається відсутність довільних рухів в правих руці та нозі, тонус м’язів у цих кінцівках підвищений. Який вид розладу рухової функції спостерігається у даному випадку?

Центральний параліч

Периферичний парез

Рефлекторний парез

Центральний парез

Периферичний параліч

1614 / 6854
У хворого в організмі знижений вміст іонів магнію, які потрібні для прикріплення рибосом до гранулярної ендоплазматичної сітки. Відомо, що це призводить до порушення біосинтезу білка. Який саме етап біосинтезу білка буде порушено?

Термінація

Реплікація

Активація амінокислот

Транскрипція

Трансляція

1615 / 6854
Хворий з виразковою хворобою шлунка приймав антацидний препарат альмагель. Для лікування гострого бронхіту йому призначили антибіотик метициклін. Проте протягом 5- ти днів температура не знизилася, кашель і характер харкотиння не змінились. Лікар прийшов до висновку про несумісність ліків при їх взаємодії. Про який саме вид несумісності ліків йдеться?

Прямий антагонізм

Фармакокінетична на етапі біотрансформації

Фармакодинамічна

Фармакокінетична на етапі всмоктування

Фармацевтична

1616 / 6854
У хворого 70-ти років атеросклероз ускладнився тромбозом судин нижніх кінцівок, виникла гангрена пальців лівої стопи. Початок тромбоутворення, найбільш вірогідно, пов’язаний з:

Адгезією тромбоцитів

Зниженням синтезу гепарину

Активацією протромбінази

Перетворенням фібриногену в фібрин

Перетворенням протромбіну в тромбін

1617 / 6854
У хворого 44-х років на ЕКГ виявлені ознаки гіпертрофії обох шлуночків та правого передсердя. Діагностовано недостатність тристулкового клапана. Який патогенетичний варіант порушення функції серця має місце при цій недостатності?

Коронарна недостатність

Перевантаження серця об’ємом

Тампонада серця

Перевантаження серця опором

Первинна міокардіальна недостатність

1618 / 6854
Бактеріологічне дослідження гнійних виділень з уретри виявило наявність бактерій, які за Грамом фарбувалися негативно, нагадували кавові зернини, розкладали глюкозу і мальтозу до кислоти, розташовувалися в лейкоцитах. Збудники якої хвороби виявлені?

Меліоїдоз

Сифілісу

М ’який шанкр

Венеричний лімфогранулематоз

Гонорея

1619 / 6854
Прозерин при системному введенні підвищує тонус скелетних м’язів. Фторотан викликає релаксацію м’язів і послаблює ефекти прозерину. Який характер взаємодії прозерину та фторотану?

Конкурентний антагонізм

Прямий функціональний антагонізм

Незалежний антагонізм

Непрямий функціональний антагонізм

Неконкурентний антагонізм

1620 / 6854
У жінки 62-х років розвинулася катаракта (помутніння кришталика) на фоні цукрового діабету. Який тип модифікації білків має місце при діабетичній катаракті?

Фосфорилювання

АДФ-рибозилювання

Глікозилювання

Обмежений протеоліз

Метилювання

1621 / 6854
У дитини 2-х років виявлено відставання у фізичному розвитку, часті пневмонії. Встановлено діагноз - незарощення артеріальної протоки. Сполучення яких кровоносних судин викликало порушення гемодинаміки?

Аорта і легеневі вени

Аорта і легеневий стовбур

Легеневий стовбур і легеневі вени

Верхня порожниста вена і аорта

Верхня порожниста вена і легеневий стовбур

1622 / 6854
в середовищі з температурою 38o 22. C та відносною вологістю повітря 50%. Які шляхи тепловіддачі зумовлюють підтримку постійної температури ядра тіла за цих умов?

Випаровування

Конвекція і теплопроведення

Теплопроведення

Конвекція

Радіація

1623 / 6854
До медико-генетичної консультації звернулося подружжя з питанням про вірогідність народження у них дітей, хворих на гемофілію. Подружжя здорове, але батько дружини хворий на гемофілію. В цій родині на гемофілію можуть захворіти:

Сини та дочки

Тільки дочки

Половина синів

Всі діти

Половина дочок

1624 / 6854
У хворого з пересадженим серцем при фізичному навантаженні збільшився хвилинний об’єм крові. Який механізм регуляції забезпечує ці зміни?

Симпатичні умовні рефлекси

Катехоламіни

Парасимпатичні безумовні рефлекси

Парасимпатичні умовні рефлекси

Симпатичні безумовні рефлекси

1625 / 6854
На прийом до лікаря прийшла хвора зі скаргами на розлад травлення, розлитий біль у животі. При обстеженні виявлено різке зниження вмісту гемоглобіну в крові. З анамнезу відомо, що, під час перебування на Далекому Сході, вона часто вживала в їжу малосольну риб’ячу ікру. Аналогічний стан відзначений у деяких родичів, що проживають з нею. Яке захворювання найбільш вірогідне?

Теніоз

Ехінококоз

Трихінельоз

Аскаридоз

Дифілоботріоз

1626 / 6854
У людини нормальна чутливість шкіри пальця, але він не відчуває наявності на ньому обручки. Який процес, спричинений впливом обручки, є причиною цього?

Адаптація рецепторів

Порушення структури рецепторів

Порушення структури епідермісу

Порушення кровообігу

Розвиток фіброзної тканини

1627 / 6854
У хворого 45-ти років з підозрою на запалення оболонок мозку потрібно було отримати спинномозкову рідину Зроблено діагностичну пункцію між дугами поперекових хребців (L3 — S4).Через яку зв’язку пройшла голка під час пункції?

Міжпоперечна

Клубово-поперекова

Передня поздовжня

Жовта

Задня поздовжня

1628 / 6854
При аналізі ЕКГ людини з’ясовано, що у стандартних відведеннях від кінцівок зубці T позитивні, їх амплітуда та тривалість нормальні. Вірним є висновок, що у шлуночках серця нормально відбувається такий процес:

Реполяризація

Деполяризація

Збудження

Скорочення

Розслаблення

1629 / 6854
У хворого в крові збільшена концентрація пірувату. Значна його кількість екскретується з сечею. Дефіцит якого вітаміну має місце у хворого?

В2

В3

B1

E

B6

1630 / 6854
Людина захворіла на пелагру. При опитуванні стало відомо, що впродовж тривалого часу вона харчувалася переважно кукурудзою, мало вживала м’яса. Дефіцит якої речовини у кукурудзі спричинив розвиток хвороби?

Аланін

Тирозин

Гістидин

Триптофан

Пролін

1631 / 6854
При забарвленні бакпрепаратів, виготовлених з харкотиння, методом Ціля-Нільсена виявлено наявність яскраво-червоних паличок, які розміщувалися поодиноко або групами, не чутливі до дії кислот. На живильних середовищах перші ознаки росту з’являються на 10-і5-ту добу. До якої родини відносяться виявлені бактерії?

Micobacterium tuberculosis

Coxiella burnettii

Yersinia pseudotuberculosis

Klebsiella rhinoscleromatis

Histoplasma dubrosii

1632 / 6854
Пацієнтці з високим ступенем ожиріння у якості харчової добавки було рекомендовано карнітин для поліпшення 'спалювання'жиру. Яку безпосередню участь бере карнітин у процесі окиснення жирів?

Активація внутрішньоклітинного ліполізу

Транспорт ВЖК з жирових депо до тканин

Бере участь в одній з реакцій бета-окиснення ВЖК

Транспорт ВЖК з цитозоля до мітохондрій

Активація ВЖК

1633 / 6854
У дитини 2-х років з катаральними явищами та висипом на шкірі лікар запідозрив скарлатину. Внутрішньошкірно дитині ввели невелику кількість сироватки до еритрогенного токсину стрептокока, на місці ін’єкції висип зник. Що означають результати реакції?

У дитини підвищена чутливість до еритрогенного токсину

Захворювання викликав не гемолітичний стрептокок

Клінічний діагноз підтвердився

Імунна система дитини дуже ослаблена

Всю дозу сироватки можна вводити внутрішньовенно

1634 / 6854
У хворого поперечний розрив спинного мозку нижче VI грудного сегменту. Як внаслідок цього зміниться дихання?

Суттєво не зміниться

Стане більш частим

Стане більш глибоким

Стане більш рідким

Зупиниться

1635 / 6854
У хворої на бронхіальну астму вірусне інфікування спровокувало астматичний статус зі смертельним наслідком. При гістологічному дослідженні легень виявлено: спазм і набрякання бронхіол, в їх стінках виражена інфільтрація лімфоцитами, еозинофілами та іншими лейкоцитами, а також дегрануляція лаброцитів. Який механізм гіперчутливості лежить в основі зазначених змін?

Запальний

Аутоімунний

Імунокомплексний

Реагінова реакція

Імунозумовлений клітинний цитоліз

1636 / 6854
Мати виявила у 5-річної доньки на періанальних складках білих 'черв’ячків ', які викликали у дитини свербіж і неспокій, і доставила їх до лабораторії. Під час огляду лікар побачив білих гельмінтів 0,5-1 см довжиною, ниткоподібної форми з загостреними кінцями, у деяких кінці були закручені. Який найбільш вірогідний діагноз?

Дифілоботріоз

Теніоз

Аскаридоз

Опісторхоз

Ентеробіоз

1637 / 6854
У хворого 27-ми років виявлено патологічні зміни печінки і головного мозку. У плазмі крові виявлено різке зниження, а в сечі - підвищення вмісту міді. Встановлено діагноз - хвороба Вільсона. Активність якого ферменту в сироватці крові необхідно дослідити для підтвердження діагнозу?

Церулоплазмін

Карбоангідраза

Ксантиноксидаза

Лейцинамінопептидаза

Алкогольдегідрогеназа

1638 / 6854
Пацієнт звернувся до лікаря зі скаргами на задишку, що виникала після фізичного навантаження. Клінічне обстеження виявило анемію та наявність парапротеїну в зоні гамма-глобулінів. Який показник у сечі необхідно визначити для підтвердження діагнозу мієломи?

Антитрипсин

Церулоплазмін

Білок Бенс-Джонса

Гемоглобін

Білірубін

1639 / 6854
У чоловіка 45-ти років після зна- чного психоемоційного навантаження раптово з’явився стискаючий біль в ділянці серця з іррадіацією в ліву руку, шию, під ліву лопатку. Обличчя стало блідим, вкрилося холодним потом. Нітрогліцерин усунув напад болю. Який процес розвинувся у хворого?

Психогенний шок

Перфорація виразки шлунка

Інфаркт міокарда

Інсульт

Стенокардія

1640 / 6854
Жінка 62-х років скаржиться на частий біль у ділянці грудної клітки та хребта, переломи ребер. Лікар припустив мієломну хворобу (плазмоцитому). Який з перерахованих нижче лабораторних показників буде мати найбільше діагностичне значення?

Гіперальбумінемія

Парапротеїнемія

Гіпопротеїнемія

Гіпоглобулінемія

Протеїнурія

1641 / 6854
Дитині 8-ми років, що надійшла до інфекційного відділення з високою температурою (до 38oC), дрібноточковим яскраво-червоним висипом, було встановлено діагноз скарлатини. Об’єктивно: слизова оболонка зіву яскраво гіперемована, набрякла, мигдалики різко збільшені, з тьмяними вогнищами жовтувато-сірого кольору і ділянками чорного кольору. Яке запалення лежить в основі змін у зіві?

Катаральне

Серозне

Фібринозне

Геморагічне

Гнійно-некротичне

1642 / 6854
У новонародженої дитини на пелюшках виявлені темні плями, що свідчать про утворення гомогентизинової кислоти. З порушенням обміну якої речовини це пов’язане?

Тирозин

Триптофан

Холестерин

Метіонін

Галактоза

1643 / 6854
У гістологічному препараті визначаються рецепторна зона сенсоепітеліального органа чуттів. Клітини даної зони лежать на базальній мембрані і включають наступні види: зовнішні та внутрішні сенсорні, зовнішні та внутрішні фалангові, стовбурові, зовнішні межові і зовнішні підтримуючі. Вкажіть, якому органу чуттів належить зазначена рецепторна зона:

Слуху

Нюху

Рівноваги

Зору

Смаку

1644 / 6854
Хворий надійшов до відділення з нападом задухи, зумовленої спазмом гладенької мускулатури дихальних шляхів. Назвіть відділи повітроносних шляхів, зміна стану яких, в основному, спричинила даний напад:

Респіраторний відділ

Бронхи великого калібру

Кінцеві бронхіоли

Бронхи малого калібру

Бронхи середнього калібру

1645 / 6854
Під час обіду дитина поперхнулася і аспірувала їжу. Почався сильний кашель, шкіра і слизові ціанотичні, пульс частий, дихання рідке, видих подовжений. Яке порушення зовнішнього дихання розвинулося у дитини?

Інспіраторна задишка при асфіксії

Дихання Біота

Альтернуюче дихання

Експіраторна задишка при асфіксії

Стенотичне дихання

1646 / 6854
Хворий 50-ти років звернувся до клініки зі скаргами на загальну слабкість, втрату апетиту, аритмію серця. Спостерігається гіпотонія м’язів, мляві паралічі, послаблення перистальтики кишечнику. Причиною такого стану може бути:

Гіперкаліємія

Гіпонатріємія

Гіпокаліємія

Гіпопротеїнемія

Гіпофосфатемія

1647 / 6854
Під час гри у волейбол спортсмен після стрибка приземлився на зовнішній край стопи. Виник гострий біль у гомілковостопному суглобі, активні рухи в ньому обмежені, пасивні - в повному обсязі, але болісні. Потім розвинулася припухлість у ділянці зовнішньої щиколотки, шкіра почервоніла, стала теплішою на дотик. Який вид розладу периферичного кровообігу розвинувся в даному випадку?

Венозна гіперемія

Артеріальна гіперемія

Емболія

Стаз

Тромбоз

1648 / 6854
У хворого на міастенію після призначення прозерину з’явилися нудота, діарея, посмикування м’язів язика і скелетних м’язів. Чим можна усунути інтоксикацію?

Ізадрин

Мезатон

Фізостигмін

Атропіну сульфат

Пірідостигміну бромід

1649 / 6854
Чоловік помер від гострого інфекційного захворювання, яке супроводжувалось гарячкою, жовтяницею, геморагічною висипкою на шкірі та слизових оболонках, а також гострою нирковою недостатністю. При гістологічному дослідженні тканини нирки (забарвлення за Романовським-Гімзою) виявлені звивисті бактерії, які мають вигляд букв С та S. Які бактерії були виявлені?

Лептоспіри

Боррелії

Кампілобактерії

Трепонеми

Спіролли

1650 / 6854
У підлітка 12-ти років, який хворіє на бронхіальну астму, виник тяжкий напад астми: виражена експіраторна задишка, блідість шкірних покривів. Який вид порушення альвеолярної вентиляції має місце?

Торако-діафрагмальний

Нервово-м’язовий

Центральний

Обструктивний

Рестриктивний

1651 / 6854
Чоловіку 46-ти років, що хворіє на дифузний токсичний зоб, була проведена операція резекції щитоподібної залози. Після операції відмічаються відсутність апетиту, диспепсія, підвищена нервово-м’язова збудливість. Маса тіла не збільшилася. Температура тіла у нормі. Чим, із нижче переліченого, обумовлений стан хворого?

Зниженням продукції тироксину

Підвищенням продукції тиреоліберину

Зниженням продукції паратгормону

Підвищенням продукції кальцитоніну

Підвищенням продукції тироксину

1652 / 6854
Хворому, який переніс інфаркт міокарда, призначена ацетилсаліцилова кислота по 75 мг щоденно. З якою метою призначено препарат?

Розширення коронарних судин

Зменшення болю

Зменшення запалення

Зниження температури

Зменшення агрегації тромбоцитів

1653 / 6854
При медичному огляді у військкоматі був виявлений хлопчик 15-ти років, високого зросту, з євнухоїдними пропорціями тіла, гінекомастією, з ростом волосся на лобку за жіночим типом. Відмічається відкладання жиру на стегнах, відсутність росту волосся на обличчі, високий голос; коефіцієнт інтелекту знижений. Виберіть каріотип, що відповідає даному захворюванню:

45, XO

46, XY

47, XXY

47, XXX

46, XX

1654 / 6854
З метою попередження відторгнення трансплантата після пересадки органів обов’язковим є проведення курсу гормонотерапії з метою імуносупресії. Які гормони застосовують з цією метою?

Статеві гормони

Глюкокортикоїди

Тиреоїдні

Катехоламіни

Мінералокортикоїди

1655 / 6854
У хворої 53-х років внаслідок тупої травми живота діагностований розрив печінки. У якому анатомічному утворенні збереться кров, що вилилася?

Міхурово-маткове заглиблення

Правий брижовий синус

Лівий брижовий синус

Сальникова сумка

Прямокишково-маткове заглиблення

1656 / 6854
Хвора скаржиться на набряк ніг, посиніння шкіри, невеличкі виразки збоку латерального виростку. При обстеженні відмічено: припухлість, збільшення розмірів вен, утворення вузлів. З боку якої вени відмічається патологія?

V. saphena parva

V iliaca externa

V saphena magna

V profunda femoris

V femoralis

1657 / 6854
У дитини 1,5 років спостерігається відставання в розумовому і фізичному розвитку, посвітління шкіри і волосся, зниження вмісту в крові катехоламінів. При додаванні до свіжої сечі декількох крапель 5 % розчину трихлороцтового заліза з’являється оливковозелене забарвлення. Для якої патології обміну амінокислот характерні дані зміни?

Алкаптонурія

Фенілкетонурія

Ксантонурія

Тирозиноз

Альбінізм

1658 / 6854
При розтині тіла померлого чоловіка 73-х років, який довго хворів на ішемічну хворобу серця з серцевою недостатністю, знайдено: 'мускатна'печінка, бура індурація легень, ціанотична індурація нирок та селезінки. Вкажіть, який з видів порушення кровообігу в хворого призвів до таких наслідків?

Гостре загальне венозне повнокров’я

Артеріальна гіперемія

Хронічне загальне венозне повнокров’я

Гостре недокрів’я

Хронічне недокрів’я

1659 / 6854
Хворий з хронічним гіперацидним гастритом для усунення печії застосовує антацидний засіб, після прийому якого відзначає поліпшення, однак разом з тим виникає відчуття розпирання у шлунку. Який з наведених препаратів міг викликати вказаний побічний ефект?

Пепсин

Магнію трисилікат

Натрію гідрокарбонат

Алюмінію гідроокис

Магнію окис

1660 / 6854
При тривалому використанні препарату в хворого можуть мати місце остеопороз, ерозії слизової шлунка, гіпокаліємія, затримка натрію i води в організмі, зменшення вмісту кортикотропіну в крові. Укажіть цей препарат:

Резерпін

Індометацин

Преднізолон

Гіпотіазид

Дигоксин

1661 / 6854
Хворому тривалий час вводили високі дози гідрокортизону, внаслідок чого настала атрофія однієї з зон кори наднирників. Яка це зона?

Сітчаста

Клубочкова і сітчаста

Клубочкова

Пучкова

1662 / 6854
До лікарні швидкої допомоги госпіталізований чоловік 63-х років з явищами колапсу. Для боротьби з гіпотензією лікар вибрав норадреналін. Який механізм дії цього препарату?

Активація серотонінових рецепторів

Блокада M-холінорецепторів

Активація α1 -адренорецепторів

Активація дофамінових рецепторів

Активація β-адренорецепторів

1663 / 6854
Основна маса азоту з організму виводиться у вигляді сечовини. Зниження активності якого ферменту в печінці призводить до гальмування синтезу сечовини і нагромадження амоніаку в крові і тканинах?

Аспартатамінотрансфераза

Уреаза

Пепсин

Амілаза

Карбамоїлфосфатсинтаза

1664 / 6854
У хворого на сифіліс при лікуванні препаратами вісмуту з’явилися сірі плями на слизовій оболонці ротової порожнини та симптоми нефропатії. Який засіб доцільно використати у хворого для лікування отруєння препаратами вісмуту?

Бемегрид

Налорфін

Унітіол

Налоксон

Метиленовий синій

1665 / 6854
На ЕКГ пацієнта мають місце такі зміни: зубець P - нормальний, інтервал P — Q - вкорочений, шлуночковий комплекс QRST - розширений, зубець R -двогорбий або двофазний. Яка із форм аритмії має місце у даного пацієнта?

Синдром Фредеріка (тріпотіння передсердь)

Миготіння шлуночків

Миготлива аритмія

Синдром WPW (Вольфа-Паркінсона-Уайта)

Атріовентрикулярна блокада

1666 / 6854
У гістологічному препараті паренхіма органа представлена лімфоїдною тканиною, яка утворює лімфатичні вузлики; останні розташовуються дифузно і містять центральну артерію. Яке анатомічне утворення має таку морфологічну будову?

Мигдалик

Селезінка

Лімфатичний вузол

Тимус

Червоний кістковий мозок

1667 / 6854
У хворого з частими кровотечами з внутрішніх органів і слизових оболонок виявлені пролін і лізин у складі колагенових волокон. Через відсутність якого вітаміну порушено їх гідрокси-лювання?

Вітамін A

Вітамін C

Вітамін K

Тіамін

Вітамін E

1668 / 6854
При диспансерному обстеженні хлопчику 7-ми років встановлено діагноз - дальтонізм. Батьки здорові, кольоровий зір у них у нормі, але у дідуся по материнській лінії така ж аномалія. Який тип успадкування цієї аномалії?

Рецесивний, зчеплений зі статтю

Неповне домінування

Аутосомно-домінантний

Домінантний, зчеплений зі статтю

Аутосомно-рецесивний

1669 / 6854
У здорової дорослої людини швидкість проведення збудження через атріовентрикулярний вузол дорівнює 0,02-0,05 м/с. Атріовентрикулярна затримка забезпечує:

Одночасність скорочення обох шлуночків

Достатню силу скорочення передсердь

Одночасність скорочення обох передсердь

Послідовність скорочення передсердь та шлуночків

Достатню силу скорочення шлуночків

1670 / 6854
На гістологічному препарат нирки представлена ділянка дистального канальцю нефрону, що проходить між приносною та виносною артеріолами. В клітинах, що складають стінку ка-нальцю, наявні ущільнені ядра, відсутня базальна мембрана. Як зветься це структурне утворення?

Щільна пляма

Юкстагломерулярні клітини

Юкставаскулярні клітини

Клітини Гурмагтига

Мезангіальні клітини

1671 / 6854
Внаслідок дії електричного струму на збудливу клітину виникла деполяризація її мембрани. Вхід яких іонів через мембрану до клітини відіграв основну роль в розвитку деполяризації?

Cl

HCO3-

K+

Na+

Ca2+

1672 / 6854
Продуктами гідролізу та модифікації деяких білків є біологічно активні речовини - гормони. З якого із наведених білків у гіпофізі утворюються ліпотропін, кортикотропін, меланотропін та ендорфіни?

Нейроглобулін

Нейроальбумін

Нейростромін

Тиреоглобулін

Проопіомеланокортин (ПОМК)

1673 / 6854
У хворих з непрохідністю жовчовивідних шляхів пригнічується зсідання крові, виникають кровотечі, що є наслідком недостатнього засвоєння такого вітаміну:

D

A

K

E

C

1674 / 6854
Хвора 48-ми років надійшла до клініки із скаргами на слабкість, дратівливість, порушення сну. Об’єктивно: шкіра та склери жовтого кольору. У крові: підвищення рівня загального білірубіну з переважанням прямого. Кал - ахолічний. Сеча - темного кольору (жовчні пігменти). Яка жовтяниця має місце в хворої?

Синдром Жільбера

Гемолітична

Паренхіматозна

Синдром Кріглера-Найяра

Механічна

1675 / 6854
У баклабораторії під час мікроскопії мазків з харкотиння хворого на хронічне легеневе захворювання, забарвлених за Цілем-Нільсеном, виявлені червоні палички. Яка властивість туберкульозної палички виявлена при цьому?

Кислотостійкість

Лугостійкість

Спиртостійкість

Капсулоутворення

Спороутворення

1676 / 6854
Під час мікроскопічного дослідження збільшеного шийного лімфатичного вузла визначається стертя його структури, лімфоїдні фолікули відсутні, усі поля зору представлені клітинами з округлими ядрами і вузьким обідком базофільної цитоплазми. З клінічних даних відомо, що збільшені і інші групи лімфовузлів, а також селезінка та печінка. Про яке захворювання слід думати?

Лімфогранулематоз

Лімфоїдний лейкоз

Мієлоїдний лейкоз

Мієломна хвороба

Лімфосаркома

1677 / 6854
У юнака 20-ти років, через 2 тижні після перенесеної лакунарної ангіни, з’явилися скарги на загальну слабкість, набряки під очима. Після обстеження хворому встановлено діагноз: гострий гломерулонефрит. Які патологічні зміни у складі сечі найбільш вірогідні?

Натрійурія

Наявність свіжих еритроцитів

Піурія

Протеїнурія

Циліндрурія

1678 / 6854
До травматологічного пункту звернувся чоловік 38-ми років з травмою правої кисті. При огляді встановлено: різана рана в ділянці підвищення великого пальця правої кисті; дистальна фаланга I пальця не згинається. Який м’яз пошкоджено?

Короткий м’яз-згинач великого пальця

Короткий відвідний м’яз великого пальця

Довгий м’яз-згинач великого пальця

Протиставний м’яз великого пальця

Привідний м’яз великого пальця

1679 / 6854
У людини збільшена вентиляція легень внаслідок фізичного навантаження. Який з наведених показників зовнішнього дихання у неї значно більший, ніж у стані спокою?

Загальна ємність легень

Резервний об’єм вдиху

Дихальний об’єм

Життєва ємність легень

Резервний об’єм видиху

1680 / 6854
У людини внаслідок тривалого голодування швидкість клубочкової фільтрації зросла на 20%. Найбільш вірогідною причиною змін фільтрації в зазначених умовах є:

Зменшення онкотичного тиску плазми крові

Збільшення ниркового кровотоку

Збільшення проникності ниркового фільтру

Збільшення системного артеріального тиску

Збільшення коефіцієнта фільтрації

1681 / 6854
У пацієнта після переливання 200 мл крові підвищилася температура тіла до 37,9o C. Яка з наведених речовин найбільш вірогідно призвела до підвищення температури?

Фактор некрозу пухлин

Інтерлейкін-2

Інтерлейкін-4

Інтерлейкін-3

Інтерлейкін-1

1682 / 6854
У людини, яка обертається на каруселі, збільшилися частота серцевих скорочень, потовиділення, з’явилася нудота. З подразненням яких рецепторів, перш за все, це пов’язано?

Зорові

Вестибулярні ампулярні

Вестибулярні отолітові

Пропріоцептори

Слухові

1683 / 6854
Робітник тваринницької ферми гостро захворів і при наростаючих явищах інтоксикації помер. При розтині встановлено: селезінка збільшена, в’яла, на розрізі темно-вишневого кольору, зішкріб пульпи рясний. М’які мозкові оболонки на склепінні та основі мозку набряклі, просякнуті кров’ю, мають темно-червоний колір ('шапочка кардинала'). Мікроскопічно: серозно-геморагічне запалення оболонок і тканин головного мозку з руйнуванням стінок дрібних судин. Який найбільш вірогідний діагноз?

Туляремія

Чума

Холера

Сибірка

Бруцельоз

1684 / 6854
Жінка 49-ти років звернулася до лікаря зі скаргами на підвищену втомлюваність та появу задишки під час фізичного навантаження. На ЕКГ: ЧСС-50/хв, інтервал PQ- подовжений, комплекс QRS - не змінений, кількість зубців P перевищує кількість комплексів QRS. Який вид аритмії у пацієнтки?

Миготлива аритмія

Синоатріальна блокада

Атріовентрикулярна блокада

Синусова брадикардія

Екстрасистолія

1685 / 6854
Хворий на ішемічну хворобу серця, з метою усунення нападів стенокардії, впродовж дня багаторазово приймав препарат, який в подальшому з причини передозування привів до отруєння. Об’єктивно: ціаноз шкіри та слизових оболонок, різке зниження артеріального тиску, тахікардія, пригнічення дихання. У крові підвищений вміст метгемоглобіну. Препарат якої групи приймав хворий?

Блокатори кальцієвих каналів

Міотропні спазмолітики

Органічні нітрати

Препарати аденозинового ряду

α-адреноблокатори

1686 / 6854
У хворого з верхнім типом ожиріння тривало відзначалися артеріальна гіпертонія, гіперглікемія, глюкозурія. Смерть настала від крововиливу у головний мозок. Під час патоморфологічного дослідження виявлені базофільна аденома гіпофізу, гіперплазія кори наднирників. Який найбільш вірогідний діагноз?

Акромегалія

Адипозогенітальна дистрофія

Гіпофізарний нанізм

Цукровий діабет

Хвороба Іценка-Кушінга

1687 / 6854
Чоловік середнього віку виїхав до іншої країни на обіцяну йому роботу, але працевлаштуватися тривалий час йому не вдавалося. Які з ендокринних залоз були виснажені у цієї людини найбільше?

Підгрудинна

Щитоподібна

Наднирники

Прищитоподібні

Сім’яники

1688 / 6854
У дівчинки діагностований адреногенітальний синдром (псевдогермафродитизм). Надмірна секреція яких гормонів наднирників обумовила дану патологію?

Глюкокортикоїди

Мінералокортикоїди

Катехоламіни

Андрогени

Естрогени

1689 / 6854
У жінки народилась мертва дитина з багатьма вадами розвитку. Яке протозойне захворювання могло спричинити внутрішньоутробну загибель?

Амебіаз

Лямбліоз

Малярія

Лейшманіоз

Токсоплазмоз

1690 / 6854
У хворого на рак спинки язика виникла сильна кровотеча внаслідок ураження пухлиною дорзальної артерії язика. Яку судину повинен перев’язати лікар для зупинки кровотечі?

Язикова артерія

Глибока артерія язика

Лицева артерія

Висхідна артерія глотки

Дорзальна артерія язика

1691 / 6854
У підлітка 12-ти років, який впродовж 3-х місяців різко схуд, вміст глюкози у крові 50 ммоль/л. У нього розвинулася кома. Який головний механізм її розвитку?

Гіпоксичний

Кетонемічний

Гіпоглікемічний

Лактацидемічний

Гіперосмолярний

1692 / 6854
У хворого на мікросфероцитарну гемолітичну анемію (хворобу Мінковського-Шоффара), внаслідок підвищення проникливості мембрани еритроцитів, у клітину надходять іони натрію та вода. Еритроцити набувають форму сфероцитів і легко руйнуються. Який провідний механізм пошкодження еритроцитів має місце в даному випадку?

Кальцієвий

Нуклеїновий

Ацидотичний

Протеїновий

Електролітно-осмотичний

1693 / 6854
На гістологічному препараті представлено кровоносну судину. Внутрішня оболонка складається з ендотелію, підендотелію та внутрішньої еластичної мембрани. Середня оболонка збагачена гладенькими міоцитами. Вкажіть, для якої судини характерні дані морфологічні ознаки:

Капіляр

Артерія еластичного типу

Вена м’язового типу

Артерія м’язового типу

Вена безм’язового типу

1694 / 6854
Охолодження тіла людини у воді виникає значно швидше, ніж на повітрі. Який шлях тепловіддачі у воді значно ефективніший?

Конвекція

Тепловипромінювання

Випаровування поту

Теплопроведення

1695 / 6854
До лікарні після автокатастрофи надійшов юнак 18-ти років. У травматологічному відділенні виявлені численні травми м’яких тканин обличчя в ділянці медіального кута ока, які призвели до масивної кровотечі. Який артеріальний анастомоз міг бути пошкоджений у цьому регіоні?

a. carotis externa et a. subclavia

a. subclavia et a. ophthalmica

a. carotis interna et a. ophthalmica

a. carotis interna et a. subclavia

a. carotis externa et a. carotis interna

1696 / 6854
Жінці 36-ти років після хірургічного втручання внутрішньовенно ввели концентрований розчин альбуміну. Це спричинило посилений рух води у такому напрямку:

З міжклітинної рідини до клітин

З міжклітинної рідини до капілярів

Із клітин до міжклітинної рідини

Із капілярів до міжклітинної рідини

Змін руху води не відбуватиметься

1697 / 6854
Під час розтину трупа чоловіка зі злоякісною пухлиною шлунка, що помер від ракової інтоксикації, в задньонижніх відділах легень виявлені щільні сіро-червоного кольору неправильної форми осередки, які виступають над поверхнею розрізу. Мікроскопічно: у просвіті, стінках дрібних бронхів та альвеолах виявляється ексудат, в якому багато нейтрофілів. Про яке захворювання свідчать зміни у легенях померлого?

Крупозна пневмонія

Гострий бронхіт

Проміжна пневмонія

Гостра гнійна бронхопневмонія

Гостра серозна бронхопневмонія

1698 / 6854
Під час розтину тіла померлої дитини 1,5 років виявлені: геморагічний висип на шкірі, помірна гіперемія та набряк слизової оболонки носоглотки, дрібні крововиливи у слизових оболонках і внутрішніх органах, різкі дистрофічні зміни у печінці, міокарді, гострий некротичний нефроз, масивні крововиливи у наднирниках. Для якого захворювання найбільш характерні виявлені зміни?

Дифтерія

Висипний тиф

Скарлатина

Менінгококова інфекція

Кір

1699 / 6854
Під час огляду дитини 11-ти місяців педіатр виявив викривлення кісток нижніх кінцівок і затримку мінералізації кісток черепа. Нестача якого вітаміну призводить до даної патології?

Тіамін

Рибофлавін

Пантотенова кислота

Біофлавоноїди

Холекальциферол

1700 / 6854
У хворого на хронічний гепатит виявлено значне зниження синтезу і секреції жовчних кислот. Який процес у найбільшій мірі буде порушений у кишечнику цього хворого?

Всмоктування гліцерину

Емульгування жирів

Травлення білків

Травлення вуглеводів

Всмоктування амінокислот

1701 / 6854
Для вирішення питання ретроспективної діагностики перенесеної бактеріальної дизентерії було призначено серологічне дослідження сироватки крові з метою встановлення титру антитіл до шигел. Яку з перелічених реакцій доцільно використати для цього?

Гемоліз

Бактеріоліз

Зв’язування комплементу

Преципітація

Пасивна гемаглютинація

1702 / 6854
Вивчається робота оперону бактерії. Відбулося звільнення гена-оператора від білка репресора. Безпосередньо після цього в клітині почнеться:

Процесінг

Трансляція

Реплікація

Транскрипція

Репресія

1703 / 6854
При визначенні енерговитрат організму людини встановлено, що дихальний коефіцієнт дорівнює 1,0. Це означає, що у клітинах досліджуваного переважно окислюються:

Білки

Вуглеводи та жири

Жири

Білки і вуглеводи

Вуглеводи

1704 / 6854
У тварини в експерименті перерізали задні корінці спинного мозку. Які зміні відбуватимуться в зоні іннервації?

Втрата чутливості

Втрата рухових функцій

Втрата чутливості і рухових функцій

Підвищення тонусу м’язів

Зниження тонусу м’язів

1705 / 6854
Внаслідок руйнування певних структур стовбуру мозку тварина втратила орієнтувальні рефлекси. Які структури було зруйновано?

Вестибулярні ядра

Медіальні ядра ретикулярної формації

Чорна речовина

Червоні ядра

Чотиригорбкова структура

1706 / 6854
У людини осмотичний тиск плазми крові 350 мосмоль/л (норма - 300 мосмоль/л). Це спричинить, перш за все, посилену секрецію такого гормону:

Альдостерон

Адренокортикотропін

Вазопресин

Кортизол

Натрійуретичний

1707 / 6854
Під час бігу на короткі дистанції у нетренованої людини виникає м’язова гіпоксія. До накопичення якого метаболіту в м’язах це призводить?

Ацетил-КоА

Лактат

Кетонові тіла

Глюкозо-6-фосфат

Оксалоацетат

1708 / 6854
У цитоплазмі міоцитів розчинена велика кількість метаболітів окиснення глюкози. Назвіть один з них, який безпосередньо перетворюється на лактат:

Фруктозо-6-фосфат

Піруват

Глюкозо-6-фосфат

Гліцерофосфат

Оксалоацетат

1709 / 6854
Молодий чоловік звернувся до лікарні зі скаргами на порушення сечовипускання. Під час обстеження зовнішніх статевих органів виявлено, що сечівник розщеплений зверху і сеча витікає через цей отвір. Який вид аномалії розвитку зовнішніх статевих органів спостерігається у цьому випадку?

Парафімоз

Гіпоспадія

Гермафродитизм

Епіспадія

Фімоз

1710 / 6854
Хворий звернувся до лікаря-уролога зі скаргами на біль під час сечовипускання. У сечі, що отримана на аналіз у денний час, були виявлені яйця з характерним шипом. З анамнезу відомо, що хворий недавно повернувся з Австралії. Який найбільш вірогідний діагноз?

Шистосомоз японський

Опісторхоз

Дикроцеліоз

Шистосомоз кишковий

Шистосомоз урогенітальний

1711 / 6854
У хворого на дизентерію при колоноскопії виявлено, що слизова оболонка товстої кишки гіперемована, набрякла, її поверхня вкрита сіро-зеленими плівками. Назвіть морфологічну форму дизентерійного коліту:

Катаральний

Виразковий

Гнійний

Некротичний

Фібринозний

1712 / 6854
У хворого через добу після апендектомії у крові визначається нейтрофільний лейкоцитоз із регенеративним зсувом. Який найбільш вірогідний механізм розвитку лейкоцитозу в даному випадку?

Уповільнення міграції лейкоцитів у тканини

Уповільнення руйнування лейкоцитів

Посилення лейкопоезу

Посилення лейкопоезу та уповільнення міграції лейкоцитів у тканини

Перерозподіл лейкоцитів у організмі

1713 / 6854
Хворий 35-ти років звернувся до лікаря із скаргами на сильний нежить та втрату відчуття запахів протягом тижня. Об’єктивно: в носовій порожнині велика кількість слизу, що вкриває слизову оболонку та блокує рецептори нюху. Де в носовій порожнині розташовані ці рецептори?

Нижня носова раковина

Середня носова раковина

Присінок носа

Верхня носова раковина

Загальний носовий хід

1714 / 6854
У дитини 10-ти років поставлено пробу Манту (з туберкуліном). Через 48 годин на місці введення туберкуліну з’явилася папула розміром до 8 мм у діаметрі. Який тип реакції гіперчутливості розвинувся після введення туберкуліну?

Реакція гіперчутливості IV типу

Реакція типу сироваткової хвороби

Реакція гіперчутливості II типу

Атопічна реакція

Реакція типу феномен Артюса

1715 / 6854
вітаміну В1 115. порушується окисне декарбоксилювання α-кетоглутарової кислоти. Синтез якого з наведених коферментів порушується при цьому?

Ліпоєва кислота

Флавінаденіндинуклеотид

Нікотинамідаденіндинуклеотид

Тіамінпірофосфат

Коензим А

1716 / 6854
З урахуванням клінічної картини хворому призначено піридоксальфосфат. Для корекції яких процесів рекомендований цей препарат?

Окисне декарбоксилювання кетокислот

Синтез білку

Дезамінування пуринових нуклеотидів

Синтез пуринових та піримідинових основ

Трансамінування і декарбоксилювання амінокислот

1717 / 6854
У хворого з клінічними ознаками імунодефіциту проведено імунологічні дослідження. Виявлено значне зниження кількості клітин, що утворюють розетки з еритроцитами барана. Який висновок слід зробити аналізуючи дані дослідження?

Недостатність клітин-ефекторів гуморального імунітету

Зниження рівня В-лімфоцитів

Зниження рівня натуральних кілерів (NK-клітин)

Зниження рівня T-лімфоцитів

Зниження рівня системи комплементу

1718 / 6854
У померлого, що понад 20-ти років працював на вугільній шахті, при розтині тіла знайдені ущільнені легені сіро-чорного кольору зі значними ділянками новоутвореної сполучної тканини та наявністю великої кількості макрофагів з пігментом чорного кольору у цитоплазмі. Який з перелічених діагнозів найбільш вірогідний?

Антракосилікоз

Антракоз

Талькоз

Силікоантракоз

Сидероз

1719 / 6854
На розтині тіла померлого від сепсису, в стегновій кістці нижньої кінцівки виявлено флегмонозне запалення, що охоплює кістковий мозок, гаверсові канали та періост. Під періостом - множинні абсцеси, в навколишніх м’яких тканинах стегна - також флегмонозне запалення. Який патологічний процес має місце?

Хронічний гематогенний остеомієліт

Гострий гематогенний остеомієліт

Остеопетроз

Остеопороз

1720 / 6854
Після попередньої сенсибілізації експериментальній тварині підшкірно ввели дозу антигену. У місці ін’єкції розвинулось фібринозне запалення з альтерацією стінок судин, основної речовини та волокнистих структур сполучної тканини у вигляді мукоїдного та фібриноїдного набухання і некрозу. Яка імунологічна реакція має місцє?

Гіперчутливість негайного типу

Гіперчутливість сповільненого типу

Реакція трансплантаційного імунітету

Гранулематоз

Нормергічна реакція

1721 / 6854
Стоматолог під час огляду порожнини рота на межі середньої і задньої третини спинки язика виявив запалені сосочки. Які сосочки язика запалені?

Papillae fungiformes

Papillae conicae

Papillae filiformes

Papillae foliatae

Papillae vallatae

1722 / 6854
При розтині трупа чоловіка 50-ти років виявлено наступні зміни: права легеня у всіх відділах помірно щільна, на розрізі тканина безповітряна, дрібнозерниста, сухувата. Вісцеральна плевра з нашаруванням фібрину сіро-коричневого кольору. Який найбільш вірогідний діагноз?

Інтерстиціальна пневмонія

Крупозна пневмонія

Туберкульоз

Пневмофіброз

Бронхопневмонія

1723 / 6854
Під час статевого дозрівання клітини чоловічих статевих залоз починають продукувати чоловічий статевий гормон тестостерон, який обумовлює появу вторинних статевих ознак. Які клітини чоловічих статевих залоз продукують цей гормон?

Сустентоцити

Сперматозоїди

Клітини Лейдіга

Клітини Сертолі

Підтримуючі клітини

1724 / 6854
При огляді пацієнта виявлене надмірне розростання кісток і м’яких тканин обличчя, збільшені розміри язика, розширені міжзубні проміжки в збільшеній зубній дузі. Які зміни секреції гормонів у нього найбільш вірогідні?

Зменшена секреція інсуліну

Збільшена секреція соматотропного гормону

Зменшена секреція соматотропного гормону

Збільшена секреція інсуліну

Зменшена секреція тироксину

1725 / 6854
На розтині тіла померлого від ниркової недостатності, який протягом останніх 5-ти років хворів на бронхоектатичну хворобу, виявлені збільшені в розмірах нирки щільної консистенції з потовщеним кірковим шаром білого кольору та сальним блиском. Про яке захворювання нирок можна думати?

Некротичний нефроз

Вторинний амілоїдоз

Гломерулонефрит

Хронічний пієлонефрит

1726 / 6854
На розтині тіла жінки 49-ти років, що померла від хронічної ниркової недостатності, виявлено: нирки ущільнені, зменшені, строкаті, з ділянками крововиливів. Мікроскопічно: у ядрах епітелію канальців гематоксилінові тільця, потовщення базальних мембран капілярів клубочків, які мають вигляд дротяних петель, подекуди в капілярах - гіалінові тромби та вогнища фібриноїдного некрозу. Який найбільш вірогідний діагноз?

Ревматизм

Системний червоний вовчак

Амілоїдоз

Атеросклеротичний нефросклероз

Артеріосклеротичний нефросклероз

1727 / 6854
У хворого 37-ми років внаслідок тривалої антибіотикотерапії розвинувся дисбактеріоз кишечнику. Який вид препаратів необхідно використати для нормалізації кишкової мікрофлори?

Бактеріофаги

Аутовакцини

Сульфаніламіди

Еубіотики

Вітаміни

1728 / 6854
До генетичної консультації звернулася сімейна пара, в якій чоловік хворіє на інсулінозалежний цукровий діабет, а жінка здорова. Яка вірогідність появи інсулінозалежного діабету у дитини цього подружжя?

Більше, ніж в популяції

100%

Така сама, як в популяції

50%

Нижче, ніж в популяції

1729 / 6854
Хвора похилого віку хворіє на цукровий діабет 2-го типу, який супроводжується ожирінням, атеросклерозом, ішемічною хворобою серця. При цьому виявлена базальна гіперінсулінемія. Запропонуйте хворій адекватне лікування:

Глібенкламід

Ретаболіл

Інсулін

Ловастатин

Амлодипін

1730 / 6854
До лікаря звернулася жінка 32-х років зі скаргами на відсутність лактації після народження дитини. Дефіцитом якого гормону можна пояснити дане порушення?

Соматотропін

Вазопресин

Пролактин

Тиреокальцитонін

Глюкагон

1731 / 6854
У хворої під час профілактичного обстеження на медіальній стінці лівої пахвової западини виявлений збільшений лімфовузол метастатичного походження. Вкажіть найбільш вірогідну локалізацію первинної пухлини:

Молочна залоза

Легеня

Піднижньощелепна слинна залоза

Шлунок

Щитоподібна залоза

1732 / 6854
У чоловіка, що хворіє на остеохондроз, з’явився різкий біль у м’язах живота (бічних та передніх). При об’єктивному обстеженні лікар констатував підвищену больову чутливість шкіри підчеревної ділянки. Ураження якого нерва могло спричинити цій біль?

Сідничний

Статево-стегновий

Затульний

Стегновий

Клубово-підчеревний

1733 / 6854
На рентгенограмі нирок при пієлографії лікар виявив ниркову миску, в яку безпосередньо впадали малі чашечки (великі були відсутні). Яку форму сечовивідних шляхів нирки виявив лікар?

Фетальна

Ампулярна

Деревоподібна

Зріла

Ембріональна

1734 / 6854
На аутопсії померлого від грипу чоловіка відзначено, що серце дещо збільшене у розмірах, пастозне, на розрізі міокард тьмяний, з крапом. Мікроскопічно: у міокарді на всьому протязі ознаки паренхіматозної жирової і гідропічної дистрофії, строма набрякла, з незначною макрофагально-лімфоцитарною інфільтрацією, судини повнокровні; периваскулярно - петехіальні крововиливи. Який вид міокардиту розвинувся в даному випадку?

Гранулематозний

Проміжний проліферативний

Гнійний

Серозний вогнищевий

Серозний дифузний

1735 / 6854
У хворого на хронічну серцеву недостатність, незважаючи на терапію кардіотонічними засобами і тіазидовим діуретиком, зберігаються набряки і виникла загроза асциту. Який препарат слід призначити для підсилення діуретичного ефекту застосованих ліків?

Амілорид

Фуросемід

Клопамід

Спіронолактон

Манітол

1736 / 6854
При гінекологічному огляді жінки 30-ти років на шийці матки виявлені яскраво-червоні блискучі плями, які при дотику легко кровоточать. На біопсії: шматочок шийки матки вкритий циліндричним епітелієм із сосочковими виростами, в товщині тканини розростання залоз. Яка патологія шийки матки виявлена?

Залозиста гіперплазія

Псевдоерозія

Лейкоплакія

Ендоцервіцит

Справжня ерозія

1737 / 6854
У мертвонародженої дитини шкіра потовщена, нагадує панцир черепахи, вушні раковини недорозвинені. Гістологічно в шкірі: надмірне зроговіння, атрофiя зернистого шару епідермісу, відсутні запальні зміни. Яке захворювання найбільш вірогідно?

Ксеродермія

Еритроплакія

Іхтіоз

Дерматоміозит

Лейкоплакія

1738 / 6854
У хворого спостерігається погіршення сутінкового зору. Який з вітамінних препаратів слід призначити пацієнту?

Ретинолу ацетат

Кислота нікотинова

Піридоксину гідрохлорид

Кислота аскорбінова

Ціанокобаламін

1739 / 6854
Хвора звернулась до лікаря із скаргами на біль та обмеження рухів у колінних суглобах. Який з нестероїдних протизапальних засобів краще призначити, враховуючи наявність в анамнезі хронічного гастродуоденіту?

Целекоксиб

Кислота ацетилсаліцилова

Промедол

Диклофенак-натрій

Бутадіон

1740 / 6854
Депресії та емоційні розлади є наслідком нестачі у головному мозку норадреналіну, серотоніну та інших біогенних амінів. Збільшення їх вмісту у синапсах можна досягти за рахунок антидепресантів, які гальмують такий фермент:

Моноамінооксидаза

Фенілаланін-4-монооксигеназа

Оксидаза L-амінокислот

Диамінооксидаза

Оксидаза D-амінокислот

1741 / 6854
У хворого з’явилися жовтушність шкіри, склер та слизових оболонок. У плазмі крові підвищений рівень загального білірубіну, в калі - рівень стеркобіліну, в сечі - уробіліну. Який вид жовтяниці у хворого?

Хвороба Жільбера

Холестатична

Паренхіматозна

Обтураційна

Гемолітична

1742 / 6854
До патогістологічої лабораторії доставлено червоподібний відросток товщиною до 2,0 см. Серозна оболонка його тьмяна, потовщена, вкрита жовто-зеленими плівковими нашаруваннями. Стінка в’яла, сіро-червона. Просвіт відростка розширено, заповнено жовто-зеленими масами. При гістологічному дослідженні виявлено, що стінка інфільтрована нейтрофілами. Визначте захворювання апендикса:

Хронічний апендицит

Гострий простий апендицит

Гострий флегмонозний апендицит

Гострий поверхневий апендицит

Гострий гангренозний апендицит

1743 / 6854
Відпочиваючи на дачі, хлопчик знайшов павука з наступними морфологічними особливостями: довжина - 2 см, кулясте черевце чорного кольору, на спинному боці якого видно червоні плямочки у два ряди, чотири пари членистих кінцівок вкриті дрібними чорними волосками. Визначте дане членистоноге:

Каракурт

Тарантул

Скорпіон

Кліщ

Фаланги

1744 / 6854
У хворого внаслідок травми розвинувся травматичний шок, у перебігу якого мали місце наступні порушення: АТ- 140/90 мм рт.ст., Ps- 120/хв. Хворий метушливий, багатослівний, блідий. Якій стадії шоку відповідає цей стан?

Кінцева

Латентний період

Торпідна

Термінальна

Еректильна

1745 / 6854
У хворого, що надійшов до хірургічного відділення з ознаками гострого апендициту, виявлені наступні зміни білої крові: загальна кількість лейкоцитів - 16 • 109 /л. Лейкоцитарна формула: б.- 0, е.- 2%, ю.- 2%, п.- 8%, с.- 59%, л.- 25%, м.- 4%. Як класифікуються зазначені зміни?

Нейтрофілія з зсувом вправо

Лейкемоїдна реакція за нейтрофільним типом

Нейтрофілія з гіперрегенеративним зсувом вліво

Нейтрофілія з дегенеративним зсувом вліво

Нейтрофілія з регенеративним зсувом вліво

1746 / 6854
Після травми хворий не може розігнути руку в ліктьовому суглобі. Порушення функції якого з основних м’язів може це спричинити?

m. subscapularis

m. teres major

m. infraspinatus

m. triceps brachii

m. levator scapulae

1747 / 6854
До лікаря звернувся студент з проханням призначити препарат для лікування алергічного риніту, який виник у нього під час цвітіння липи. Який засіб можна застосувати?

Анаприлін

Лоратадин

Лозартан

Норадреналіну гідротартрат

Амброксол

1748 / 6854
У зародка порушено процес сегментації дорзальної мезодерми та утворення сомітів. В якій частині шкіри можливі порушення розвитку?

Сальні залози

Дерма

Потові залози

Волосся

Епідерміс

1749 / 6854
Дитина 9-ми місяців харчується штучними сумішами, які не збалансовані за вмістом вітаміну В6 . У дитини спостерігається пелагроподібний дерматит, судоми, анемія. Розвиток судом може бути пов’язаний з порушенням утворення:

Гістаміну

Серотоніну

ДОФА

ГАМК

Дофаміну

1750 / 6854
В експерименті на жабі вивчали міотатичний рефлекс. Однак при розтяганні скелетного м’яза він рефлекторно не скоротився. Порушення функції яких рецепторів може бути причиною цього?

Суглобові

Больові

Дотикові

М’язові веретена

Сухожильні рецептори Гольджі

1751 / 6854
В експерименті подразнюють гілочки блукаючого нерва, які іннервують серце. Це призвело до того, що припинилося проведення збудження від передсердь до шлуночків. Електрофізіологічні зміни в яких структурах серця є причиною цього?

Пучок Гіса

Шлуночки

Передсердя

Синоатріальний вузол

Атріовентрикулярний вузол

1752 / 6854
Чоловік протягом 3-х років працював в одній із африканських країн. Через місяць після переїзду до України звернувся до офтальмолога зі скаргами на біль в очах, набряки повік, сльозоточивість і тимчасове послаблення зору. Під кон’юнктивою ока були виявлені гельмінти розмірами 30-50 мм, які мали видовжене ниткоподібне тіло. Який найбільш вірогідний діагноз?

Філяріоз

Аскаридоз

Ентеробіоз

Трихоцефальоз

Дифілоботріоз

1753 / 6854
У юнака 16-ти років після перенесеного захворювання знижена функція синтезу білків у печінці внаслідок нестачі вітаміну K. Це може призвести до порушення:

Зсідання крові

Утворення еритропоетинів

Швидкості осідання еритроцитів

Осмотичного тиску крові

Утворення антикоагулянтів

1754 / 6854
На перехід із горизонтального положення у вертикальне система кровообігу відповідає розвитком рефлекторної пресорної реакції. Що з наведеного є її обов’язковим компонентом?

Системне розширення артеріальних судин опору

Зменшення насосної функції серця

Зменшення об’єму циркулюючої крові

Системне звуження венозних судин ємності

Зменшення частоти серцевих скорочень

1755 / 6854
У життєвому циклі клітини відбувається процес самоподвоєння ДНК. В результаті цього однохроматидні хромосоми стають двохроматидними. У який період клітинного циклу спостерігається це явище?

S

G1

G0

G2

M

1756 / 6854
У водія, який потрапив у ДТП, отримав травму та знаходиться у стані шоку, спостерігається зменшення добової кількості сечі до 300 мл. Який основний патогенетичний фактор цієї зміни діурезу?

Зменшення кількості функціонуючих клубочків

Вторинний гіперальдостеронізм

Зниження онкотичного тиску крові

Підвищення проникності судин

Падіння артеріального тиску

1757 / 6854
У хворого виявлена аутоімунна гемолітична анемія, що розвивається за цитотоксичним типом. Які речовини є антигенами при алергічних реакціях II типу?

Модифіковані рецептори клітинних мембран

Сироваткові білки

Модулятори запалення

Гормони

Антибіотики

1758 / 6854
Пацієнт страждає на геморагічний синдром, що проявляється частими носовими кровотечами, посттравматичними та спонтанними внутрішньо-шкірними та внутрішньосуглобовими крововиливами. Після лабораторного обстеження було діагностовано гемофілію В. Дефіцит якого фактора згортання крові обумовлює дане захворювання?

VIII

XI

V

VII

IX

1759 / 6854
Після черепно-мозкової травми у хворого спостерігається втрата можливості виконувати знайомі до травми складно-координовані рухи (апраксія). В якій ділянці кори великих півкуль найімовірніше локалізується ушкодження?

Gyrus paracentralis

Gyrus supramarginalis

Gyrus angularis

Gyrus lingualis

Gyrus parahippocampalis

1760 / 6854
Чоловік 58-ми років хворіє на атеросклероз судин головного мозку. При обстеженні виявлена гіперліпідемія. Вміст якого класу ліпопротеїдів у сироватці крові даного чоловіка найбільш вірогідно буде підвищений?

Ліпопротеїди високої щільності

Комплекси жирних кислот з альбумінами

Холестерин

Хіломікрони

Ліпопротеїди низької щільності

1761 / 6854
Хворий звернувся до лікаря зі скаргами на ригідність м’язів, скутість рухів, постійний тремор рук. Встановлено діагноз: хвороба Паркінсона. Який препарат найбільш раціонально призначити?

Етосуксимід

Дифенін

Сибазон

Леводопа

Фенобарбітал

1762 / 6854
До кардіологічного відділення надійшов хворий на ішемічну хворобу серця. Для профілактики нападів стенокардії призначено лікарський засіб з групи βадреноблокаторів. Назвіть цей препарат:

Окситоцин

Атропіну сульфат

Морфіну гідрохлорид

Фуросемід

Метопролол

1763 / 6854
До гінеколога звернулася жінка 28-ми років з приводу безпліддя. При обстеженні знайдено: недорозвинені яєчники та матка, нерегулярний менструальний цикл. При досліджєнні статевого хроматину в 6ільшості соматичних клітин виявлено 2 тільця Бара. Яка хромосомна хвороба найбільш вірогідна у жінки?

Синдром Едвардса

Синдром Патау

Синдром трипло-Х

Синдром Клайнфельтера

Синдром Шерешевського-Тернера

1764 / 6854
При отруєнні невідомим препаратом у пацієнта спостерігались сухість слизової оболонки рота та розширення зіниць. З яким впливом пов’язана дія цього препарату?

Стимуляція M-холінорецепторів

Блокада адренорецепторів

Стимуляція H-холінорецепторів

Блокада M-холінорецепторів

Стимуляція адренорецепторів

1765 / 6854
У новонародженого спостерігається диспепсія після годування молоком. При заміні молока розчином глюкози симптоми диспепсії зникають. Недостатня активність якого ферменту спостерігається у новонародженого?

Ізомальтаза

Лактаза

Сахараза

Амілаза

Мальтаза

1766 / 6854
У людини збільшена частота серцевих скорочень, розширені зіниці, сухість у роті. Наслідком активації в організмі якої системи регуляції функцій це викликано?

Ваго-інсулярна

Парасимпатична

Метасимпатична

Гипоталамо-гіпофізарно-наднирникова

Симпатична

1767 / 6854
Хворому з переломом кінцівки необхідно призначити препарат з групи міорелаксантів деполяризуючого типу дії для проведення нетривалого хірургічного втручання. Що це за засіб?

Цитітон

Пентамін

Атропіну сульфат

Тубокурарину хлорид

Дитилін

1768 / 6854
Пацієнт, що хворіє на хронічний бронхіт, приймає синтетичний муколітичний препарат, який сприяє розріджуванню харкотиння. Назвіть цей препарат:

Еналаприл

Діазепам

Ацетилцистеїн

Фуросемід

Гепарин

1769 / 6854
У хворого з масивними опіками розвинулась гостра недостатність нирок, що характеризується значним і швидким зменшенням швидкості клубочкової фільтрації. Який механізм її розвитку?

Зменшення ниркового кровотоку

Емболія ниркової артерії

Збільшення тиску канальцевої рідини

Зменшення кількості функціонуючих нефронів

Ушкодження клубочкового фільтра

1770 / 6854
У хворого з важким перебігом респіраторної вірусної інфекції з’явилися клінічні ознаки прогресуючої серцевої недостатності, яка призвела до смерті хворого на 2-му тижні захворювання. На аутопсії: серце зі значним розширенням порожнин, в’яле. Гістологічно в міокарді виявляється повнокров’я мікросудин і дифузна інфільтрація строми лімфоцитами та гістіоцитами. Який найбільш вірогідний діагноз?

Кардіоміопатія

Гостра коронарна недостатність

Інфаркт міокарда

Стенокардія

Міокардит

1771 / 6854
При поточному контролі санітарно-епідемічного стану аптеки проведено бактеріологічне дослідження повітря. Встановлено наявність у ньому бацил, дріжджеподібних грибів, гемолітичних стрептококів, мікрококів. Які з виявлених мікроорганізмів свідчать про пряму епідемічну небезпеку?

Гемолітичні стрептококи

Мікрококи

Дріжджеподібні гриби

Бацили

1772 / 6854
До реанімаційного відділення надійшов чоловік з пораненням задньої ділянки шиї (regio nuchae). Який з м’язів тіла займає цю ділянку?

m. sternocleidomastoideus

m. latissimus dorsi

m. rhomboideus minor

m. scalenus anterior

m. trapezius

1773 / 6854
На секції в лівій легені виявлено ділянку щільної тканини червоного кольору. Ділянка має форму конуса, чітко відмежована від здорової тканини, основою обернена до плеври. Тканина на розрізі зерниста, темно-червона. Який найбільш вірогідний діагноз?

Абсцес легені

Крупозна пневмонія

Геморагічний інфаркт

Первинний туберкульозний афект

Гангрена легені

1774 / 6854
У тварини збільшений тонус м’язів-розгиначів. Це є наслідком посиленої передачі інформації до мотонейронів спинного мозку такими низхідними шляхами:

Вестибулоспінальні

Латеральні кортикоспінальні

Медіальні кортикоспінальні

Ретикулоспінальні

Руброспінальні

1775 / 6854
Після опромінювання у людини з’явилася велика кількість мутантних клітин. Через деякий час більшість із них були розпізнані і знищені клітинами імунної системи, а саме:

T-лімфоцитами-супресорами

Стовбуровими клітинами

Плазмобластами

T-лімфоцитами-кілерами

В-лімфоцитами

1776 / 6854
У чоловіка 65-ти років розвинувся гнійний абсцес на шиї. Виділена культура грампозитивних коків, яка має плазмокоагулазну активність. Більш за все, це:

Staphylococcus saprophyticus

Streptococcus pyogenes

Staphylococcus epidermidis

Staphylococcus aureus

1777 / 6854
У клінічній практиці для лікування туберкульозу застосовують препарат ізоніазид - антивітамін, який здатний проникати у туберкульозну паличку. Туберкулостатичний ефект обумовлений порушенням процесів реплікації, окисно-відновних реакцій, завдяки утворенню несправжнього коферменту з:

НАД

ФМН

КоQ

ФАД

ТДФ

1778 / 6854
У новонародженої дитини спостерігаються зниження інтенсивності смоктання, часте блювання, гіпотонія. У сечі та крові значно підвищена концентрація цитруліну. Який метаболічний процес порушений?

Глюконеогенез

Гліколіз

Цикл Корі

ЦТК

Орнітиновий цикл

1779 / 6854
В експерименті на тварині здійснили перерізку блукаючих нервів з обох боків. Як при цьому зміниться характер дихання?

Дихання не зміниться

Стане поверхневим та рідким

Стане глибоким і частим

Стане поверхневим та частим

Стане глибоким і рідким

1780 / 6854
При підготовці до видалення зуба стоматолог порекомендував пацієнтові з метою профілактики кровотечі приймати препарат. Який засіб був рекомендований?

Вікасол

Магнію сульфат

Гепарин

Аспаркам

Димедрол

1781 / 6854
Людина, що тривалий час приймає ліки, не може різко припинити їх вживання, оскільки при цьому виникають порушення психічних та соматичних функцій. Як називається синдром різних порушень при відмові від прийому речовини?

Кумуляція

Сенсибілізація

Абстиненція

ідіосинкразія

Тахіфілаксія

1782 / 6854
У хворої 45-ти років невроз, що проявляється дратівливістю, безсонням, немотивованою тривогою. Який лікарський засіб усуне всі симптоми?

Леводопа

Діазепам

Екстракт валеріани

Кофеїн-бензоат натрію

Пірацетам

1783 / 6854
У хворого після вживання недоброякісної їжі розвинувся багаторазовий пронос. На наступний день у нього знизився артеріальний тиск, з’явились тахікардія, екстрасистолія. pH крові складає 7,18. Ці порушення є наслідком розвитку:

Метаболічного алкалозу

Негазового ацидозу

Негазового алкалозу

Газового алкалозу

Газового ацидозу

1784 / 6854
Яким буде скорочення м’язів верхньої кінцівки при утриманні (але не переміщенні) вантажу в певному положенні?

Ауксотонічним

Ексцентричним

Концентричним

Ізометричним

Ізотонічним

1785 / 6854
Хворий не відчуває дотику до шкіри в ділянці присередньої поверхні плеча. Порушення функції якого нерва спостерігається у хворого?

Шкірний присередній нерв плеча

Пахвовий нерв

Променевий нерв

Ліктьовий нерв

Шкірний присередній нерв передпліччя

1786 / 6854
По ходу слухового нерва у молодої жінки виявлена пухлина у вигляді вузла до 3 см в діаметрі, м’яко-еластичної консистенції, рожево-білого кольору, однорідна. Мікроскопічно пухлина містить пучки клітин з овальними ядрами. Клітинно-волокнисті пучки формують ритмічні структури, створені паралельними рядами, правильно орієнтованими клітинами, розташованими у вигляді частоколу, поміж яких знаходиться безклітинна гомогенна зона (тільця Верокаї). Що це за пухлина?

Гангліонейробластома

Нейробластома

Злоякісна невринома

Невринома

Гангліоневрома

1787 / 6854
До кардіологічного відділення надійшов хворий з інфарктом міокарда. Для усунення болю було вирішено потенціювати дію фентаніла нейролептиком. Який з перерахованих нейролептиків найбільш придатний для проведення нейролептаналгезії?

Аміназин

Галоперидол

Сульпірид

Трифтазін

Дроперидол

1788 / 6854
Для лікування бактеріальної пневмонії було призначено бензилпеніциліну натрієву сіль. Який механізм антимікробної дії препарату?

Пригнічення активності холінестерази

Пригнічення синтезу клітинної стінки мікроорганізмів

Пригнічення внутрішньоклітинного синтезу білка

Пригнічення SH-груп ферментів мікроорганізмів

Антагонізм з параамінобензойною кислотою

1789 / 6854
Хворий 49-ти років, водій за професією, скаржиться на нестерпний стискаючий біль за грудниною, що 'віддає' у ділянку шиї. Біль виник 2 години тому. Об’єктивно: стан важкий, блідість, тони серця послаблені. Лабораторне обстеження показало високу активність креатинкінази та ЛДГ1 . Для якого захворювання характерні такі симптоми?

Жовчнокам’яна хвороба

Цукровий діабет

Гострий інфаркт міокарда

Стенокардія

Гострий панкреатит

1790 / 6854
У чоловіка 60-ти років спостерігається послаблення перистальтики кишечнику. Який з наведених харчових продуктів буде стимулювати перистальтику найбільше?

Чорний хліб

Сало

Чай

Білий хліб

М’ясо

1791 / 6854
При посіві матеріалу із зіву від хворого ангіною на кров’яно-телуритовий агар виросли колонії діаметром 4-5 мм, сірого кольору, радіально посмуговані (у вигляді розеток). Під мікроскопом - грампозитивні палички із булавоподібними потовщеннями на кінцях, що розміщені у вигляді розчепірюваних пальців. Які це мікроорганізми?

Дифтероїди

Коринебактерії дифтерії

Клостридії ботулізму

Стрептококи

Стрептобацили

1792 / 6854
На мікропрепараті підщелепної слинної залози навколо кінцевих відділів і вивідних проток розрізняються кошикоподібні клітини, які охоплюють основи сероцитів і називаються міоепітеліоцити. До якої тканини належать ці клітини?

Нервова

Пухка волокниста сполучна

Сполучна зі спеціальними властивостями

Епітеліальна

М’язова

1793 / 6854
Встановлено, що аглютинація еритроцитів крові реципієнта викликали стандартні сироватки I та II груп і не викликали - сироватка III групи і антирезусна сироватка. Кров якої групи за системами AB0 і резус можна переливати реципієнту?

B,α (III)Rh-

AB(IV), Rh-

0,α,β, (I)Rh+

AB(IV), Rh+

1794 / 6854
Фармакологічні ефекти антидепресантів пов’язані з блокуванням (інгібуванням) ними ферменту, який каталізує розпад таких біогенних амінів, як норадреналін і серотонін в мітохондріях нейронів головного мозку. Який фермент бере участь у цьому процесі?

Декарбоксилаза

Моноамінооксидаза

Пептидаза

Ліаза

Трансаміназа

1795 / 6854
Онкологічному хворому призначили препарат метотрексат, до якого з часом клітинимішені пухлини втратили чутливість. Експресія гену якого ферменту при цьому змінюється?

Фолатдекарбоксилаза

Дегідрофолатредуктаза

Тиміназа

Фолатоксидаза

Дезаміназа

1796 / 6854
До кардіологічного відділення надійшов хворий з гіпертонічним кризом, йому внутрішньовенно ввели антигіпертензивний засіб - сіль лужноземельного металу. Який препарат ввели хворому?

Кальцію лактат

Магнію сульфат

Бензогексоній

Натрію гідрокарбонат

Калію хлорид

1797 / 6854
Постраждалий доставлений до хірургічного відділення із проникним пораненням у лівій бічній ділянці живота. Який відділ товстої кишки, найімовірніше, ушкоджений?

Colon descendens

Colon ascendens

Colon transverses

Rectum

Caecum

1798 / 6854
Постраждалого в аварії водія госпіталізовано до стаціонару з ушкодженням медіального надвиростка плечової кістки. Який нерв при цьому може бути ушкоджений?

n. ulnaris

n. muscolocutaneus

n. axillaris

n. radialis

n. medianus

1799 / 6854
Чоловік з колотою раною в ділянці чотирьохстороннього отвору звернувся до лікаря. При обстеженні виявлено, що потерпший не може відвести руку від тулуба. Який нерв вірогідно ушкоджений?

n. subclavius

n. ulnaris

n. radialis

n. axillaris

n. medianus

1800 / 6854
У вагітної жінки на передній черевній стінці виявлено пухлиноподібне утворення, яке виникло на місці видаленої два роки тому пухлини. Утворення має щільну консистенцію і розміри 2х1 см, з чіткими межами. При гістологічному дослідженні виявлено, що пухлина побудована з диференційованої сполучної тканини з переважанням колагенових волокон. Про яку пухлину слід думати?

Десмоїд

Ліпома

Фібросаркома

Лейоміома

Гібернома

1801 / 6854
Під час емоційного збудження частота серцевих скорочень (ЧСС) у людини 30-ти років досягла 112/хв. Зміна стану якої структури провідної системи серця є причиною збільшення ЧСС?

Ніжки пучка Пса

Пучок Гіса

Атріовентрикулярний вузол

Волокна Пуркін’є

Синоатрiальний вузол

1802 / 6854
При санітарно-бактеріологічному дослідженні води методом мембранних фільтрів виявлено дві червоні колонії на мембранному фільтрі (середовище Ендо), через який пропустили 500 мл досліджуваної води. Розрахуйте колі-індекс та колі-титр досліджуваної води:

250 та 2

500 та 2

2 та 500

250 та 4

4 та 250

1803 / 6854
Хвора 46-ти років довгий час страждає на прогресуючу м’язову дистрофію (Дюшена). Зміни рівня якого ферменту крові є діагностичним тестом в даному випадку?

Піруватдегідрогеназа

Креатинфосфокіназа

Глутаматдегідрогеназа

Аденілаткіназа

Лактатдегідрогеназа

1804 / 6854
Дівчинка 10-ти років часто хворіє на гострі респіраторні інфекції, після яких спостерігаються множинні точкові крововиливи в місцях тертя одягу. Який гіповітаміноз має місце в дівчинки:

C

A

В6

B2

B1

1805 / 6854
З сироватки крові людини виділили п’ять ізоферментних форм лактатдегідрогенази і вивчили їх властивості. Яка властивість доводить, що виділені ізоферментні форми одного і того ж ферменту?

Каталізують одну і ту ж реакцію

Тканинна локалізація

Однакові фізико-хімічні властивості

Однакова електрофоретична рухливість

Однакова молекулярна маса

1806 / 6854
Хворий з виразковою хворобою шлунка приймав антацидний препарат альмагель. Для лікування гострого бронхіту йому призначили антибіотик метициклін. Проте протягом 5- ти днів температура не знизилася, кашель і характер харкотиння не змінились. Лікар прийшов до висновку про несумісність ліків при їх взаємодії. Про який саме вид несумісності ліків йдеться?

Фармакодинамічна

Прямий антагонізм

Фармакокінетична на етапі всмоктування

Фармацевтична

Фармакокінетична на етапі біотрансформації

1807 / 6854
При гістологічному дослідженні біоптата перегородки носа хворого, який страждає на утруднене носове дихання, в слизовій оболонці знайдено гранулематозне запалення з наявністю в гранульомах клітин Микуліча і бактерій Волковича-Фріша. Який найбільш вірогідний діагноз?

Туберкульоз

Риносклерома

Сап

Лепра

Сифіліс

1808 / 6854
При аналізі ЕКГ необхідно визначити, що є водієм ритму серця. Зробити це можна на підставі вимірювання:

Амплітуди зубців

Тривалості комплексу QRST

Тривалості зубців

Тривалості інтервалу R — R

Напрямку зубців

1809 / 6854
У новонародженого хлопчика спостерігається деформація мозкового та лицьового черепа, мікрофтальмія, деформація вушної раковини, вовча паща, і т.ін. Каріотип дитини - 47, XY, 13+. Про яку хворобу йде мова?

Синдром Едвардса

Синдром Патау

Синдром Клайнфельтера

Синдром Дауна

Синдром Шерешевського-Тернера

1810 / 6854
Прозерин при системному введеннї підвищує тонус скелетних м’язів. Фторотан викликає релаксацію м’язів і послаблює ефекти прозерину. Який характер взаємодії прозерину та фторотану?

Неконкурентний антагонізм

Незалежний антагонізм

Непрямий функціональний антагонізм

Прямий функціональний антагонізм

Конкурентний антагонізм

1811 / 6854
Жінка в період вагітності тривалий час без контролю лікаря приймала антибіотик. Через деякий час у неї погіршився апетит, з’явились нудота, пронос, зміни слизових оболонок ротової порожнини та шлунково-кишкового тракту. З часом виникла жовтяниця. У новонародженого відмічено порушення росту кісток. Який препарат приймала жінка?

Доксациклін

Левоміцетин

Еритроміцин

Бісептол

Ампіцилін

1812 / 6854
До кардіологічного відділення доставлений хворий з діагнозом: гострий інфаркт міокарда. Для усунення больового синдрому (нейролептанальгезії) пацієнту необхідно ввести таку комбінацію лікарських препаратів:

Аміназин + діазепам

Анальгін + піпольфен

Морфін + атропін

Дроперідол + фентаніл

Но-шпа + димедрол

1813 / 6854
У хворого з пересадженим серцем при фізичному навантаженні збільшився хвилинний об’єм крові. Який механізм регуляції забезпечує ці зміни?

Парасимпатичні умовні рефлекси

Катехоламіни

Парасимпатичні безумовні рефлекси

Симпатичні умовні рефлекси

Симпатичні безумовні рефлекси

1814 / 6854
У хворого 53-х років, що тривало страждав на бронхоектатичну хворобу та кровохаркання, з’явилися набряки на обличчі і у ділянці попереку; у сечі - білок 33 мг/л. Смерть настала від легеневої кровотечі. Результати аутопсії: нирки збільшені в об’ємі, ущільнені, поверхня розрізу має сальний вигляд. Гістологічно відзначено відкладення у клубочках і по ходу канальців гомогенних еозинофільних мас, які вибірково забарвлюються конго-рот і дають метахромазію з метиловим фіолетовим. Який патологічний процес мав місце в нирках у даному випадку?

Гіаліноз

Фібриноїдне набухання

Жирова дистрофія

Амілоїдоз

Мукоїдне набухання

1815 / 6854
Людині внутрішньовенно ввели 0,5 л ізотонічного розчину лікарської речовини. Які з рецепторів насамперед прореагують на зміни водно-сольового балансу організму?

Натрієві рецептори гіпоталамуса

Волюморецептори порожнистих вен і передсердь

Осморецептори гіпоталамусу

Осморецептори печінки

Барорецептори дуги аорти

1816 / 6854
Отруєння ботулінічним токсином, який блокує вхід іонів кальцію до нервових закінчень аксонів мотонейронів, небезпечно для життя, бо загрожує:

Зупинкою дихання

Розвитком проносу

Розвитком блювання

Розладом тонусу судин

Зупинкою серця

1817 / 6854
Недбалий студент раптово зустрівся з деканом. Концентрація якого гормону найшвидше збільшиться в крові студента?

Кортизол

Тиреоліберин

Соматотропін

Кортикотропін

Адреналін

1818 / 6854
У студента після вживання м’яса, консервованого в домашніх умовах, з’явились диплопія, порушення мови та параліч дихання. Чим обумовлені такі симптоми ботулізму?

Секреція ентеротоксину

Дія нейротоксину

Ендотоксичний шок

Інвазія Cl. eotulinum в епітелій кишечнику

Активація аденілатциклази

1819 / 6854
У клінічно здорових батьків народилася дитина, хвора на фенілкетонурію (аутосомнорецесивне спадкове захворювання). Які генотипи батьків?

Аа х аа

Аа х Аа

АА х Аа

аа х аа

АА х АА

1820 / 6854
Хворого 55-ти років госпіталізовано до хірургічної клініки з підозрою на сепсис. Який матеріал для дослідження необхідно взяти від хворого і на яке середовище його слід засіяти?

Пунктат лімфовузла, цистеїновий агар

Сеча, м’ясо-пептонний бульйон

Ліквор, сироватковий агар

Гній, жовточно-сольовий агар

Кров, цукровий бульйон

1821 / 6854
У хворого діагностовано діабетичну кому. Концентрація цукру в крові становить 18,44 ммоль/л. Який з цукрознижуючих препаратів необхідно призначити даному хворому?

Інсулін короткої дії

Препарат із групи бігуанідів

Інсулін середньої тривалості дії

Препарат із групи похідних сульфо-нілсечовини

Інсулін тривалої дії

1822 / 6854
Людина захворіла на пелагру. При опитуванні стало відомо, що впродовж тривалого часу вона харчувалася переважно кукурудзою, мало вживала м’яса. Дефіцит якої речовини у кукурудзі спричинив розвиток хвороби?

Аланін

Пролін

Тирозин

Триптофан

Гістидин

1823 / 6854
До клініки надійшов хворий зі скаргами на біль у правій підреберній ділянці, блювання з кров’ю. При дослідженні було встановлено збільшення печінки, розширення підшкірних вен передньої стінки живота. В якій судині утруднений кровотік?

Верхня порожниста вена

Ворітна вена

Печінкові вени

Нижня порожниста вена

Черевна аорта

1824 / 6854
До лабораторії направлено матеріал білуватого нашарування із слизових оболонок ротової порожнини. Висів матеріалу зроблено на середовище Сабуро, відмічено ріст сметаноподібних колоній. Бактеріоскопія виявила короткі бруньковані нитки. До збудників якої інфекції відносять ізольовані мікроорганізми?

Хламідіоз

Мікоз

Мікоплазмоз

Рикетсіоз

Спірохетоз

1825 / 6854
У хворого, який проходить курс лікувального голодування, нормальний рівень глюкози у крові підтримується головним чином за рахунок глюконеогенезу. З якої амінокислоти при цьому у печінці людини найбільш активно синтезується глюкоза?

Лейцин

Глутамінова кислота

Аланін

Лізин

Валін

1826 / 6854
Оглядаючи дитину 6-ти років, лікар помітив на глоткових мигдаликах сірувату плівку, при спробі видалення якої виникла помірна кровотеча. Бактеріоскопія мазків з мигдаликів показала наявність грампозитивних бактерій булавоподібної форми. Які симптоми можуть виникнути у дитини у найближчі дні, якщо не буде проведене специфічне лікування?

Токсичні ураження серцевого м’яза, печінки, нирок

Хвильоподібна лихоманка

Папульозні висипи на шкірі

Набряк легенів

Дуже сильний нападоподібний кашель

1827 / 6854
Хворий 46-ти років звернувся до лікаря зі скаргою на біль в суглобах, який посилюється напередодні зміни погоди. У крові виявлено підвищення концентрації сечової кислоти. Посилений розпад якої речовини є найвірогіднішою причиною захворювання?

АМФ

ТМФ

УТФ

УМФ

ЦМФ

1828 / 6854
Чоловік 65-ти років надійшов до неврологічного відділєння з дiагнозом постінсультний синдром. Який препарат найбільш доцільно призначити хворому для прискорення одужання?

Ацеклідин

Ізонітрозин

Галантаміну гідрохлорид

Іпратропіум бромід

Дипіроксим

1829 / 6854
Через 2-3 години після парентерального введення препарату у пацієнта розвинувся коматозний стан, спостерігається дихання типу Чейн-Стокса, зіниці різко звужені, колінний рефлекс збережений. Який препарат міг спричинити отруєння?

Сибазон

Морфін

Аміназин

Спирт етиловий

Фенобарбітал

1830 / 6854
Дитина 3-х років надійшла до клініки з діагнозом отит. Є вірогідність розповсюдження гною із барабанної порожнини через задню стінку. Куди найвірогідніше може потрапити гній?

В зовнішній слуховий прохід

В задню черепну ямку

У внутрішнє вухо

В слухову трубу

В соскоподібну печеру

1831 / 6854
В медико-генетичній консультації при обстеженні хворого хлопчика в крові були виявлені нейтрофільні лейкоцити з однією 'барабанною паличкою'. Наявність якого синдрому можна запідозрити у хлопчика?

Синдром Шерешевського-Тернера

Синдром Клайнфельтера

Синдром трисомії-Х

Синдром Едвардса

Синдром Дауна

1832 / 6854
Після травми передньої поверхні верхньої третини передпліччя у хворого утруднення пронації, послаблення долонного згинання кисті та порушення чутливості шкіри 1-3 пальців на долоні. Який нерв ушкоджено?

n. ulnaris

n. musculocutaneus

n. radialis

n. medianus

n. cutaneus antebrachii medialis

1833 / 6854
До лікарні доставлено дитину 2-х років з уповільненим розумовим і фізичним розвитком, що страждає на часті блювання після прийому їжі. У сечі визначена фенілпіровиноградна кислота. Наслідком якого порушення є дана патологія?

Водно-сольовий обмін

Фосфорно-кальцієвий обмін

Ліпідний обмін

Обмін амінокислот

Вуглеводний обмін

1834 / 6854
Під час патронажу лікар виявив у дитини симетричну шорсткість щік, діарею, порушення нервової діяльності. Нестача яких харчових факторів є причиною такого стану?

Нікотинова кислота, триптофан

Треонін, пантотенова кислота

Метіонін, ліпоєва кислота

Лізин, аскорбінова кислота

Фенілаланін, пангамова кислота

1835 / 6854
Чоловік 38-ми років загинув при спробі підйому вантажу. Розвинувся колаптоїдний стан. На аутопсії виявлений розрив обширної аневризми грудного відділу аорти. За життя страждав на вісцеральний сифіліс. Який патологічний процес в даному випадку обумовив зменшення міцності стінки аорти, її розширення і розрив?

Зникнення колагенових волокон

Зникнення еластичних волокон

Атрофія м’язового шару

Зміни інтими за типом 'шагреневої шкіри'

Новоутворення судин

1836 / 6854
У хворого на міастенію після призначення прозерину з’явилися нудота, діарея, посмикування м’язів язика і скелетних м’язів. Чим можна усунути інтоксикацію?

Атропіну сульфат

Пірідостигміну бромід

Фізостигмін

Ізадрин

Мезатон

1837 / 6854
У хворого з типовою клінічною картиною дизентерії, внаслідок раннього застосування антибіотиків, під час бактеріологічного дослідження випорожнень шигели не виявлені. Титр антишигельозних антитіл в РПГА у парних сироватках у даного хворого підвищився в 4 рази. Про що це свідчить?

Виключає діагноз дизентерії

Неспецифічна реакція

Підтверджує діагноз дизентерії

Переніс дизентерію раніше

Вакцинальна реакція

1838 / 6854
Чоловік помер від гострого інфекційного захворювання, яке супроводжувалось гарячкою, жовтяницею, геморагічною висипкою на шкірі та слизових оболонках, а також гострою нирковою недостатністю. При гістологічному дослідженні тканини нирки (забарвлення за Романовським-Гімзою) виявлені звивисті бактерії, які мають вигляд букв С та S. Які бактерії були виявлені?

Спіроли

Лептоспіри

Трепонеми

Борелії

Кампілобактерії

1839 / 6854
Чоловіку 46-ти років, що хворіє на дифузний токсичний зоб, була проведена операція резекції щитоподібної залози. Після операції відмічаються відсутність апетиту, диспепсія, підвищена нервово-м’язова збудливість. Маса тіла не збільшилася. Температура тіла у нормі. Чим, із нижче переліченого, обумовлений стан хворого?

Зниженням продукції тироксину

Зниженням продукції паратгормону

Підвищенням продукції тироксину

Підвищенням продукції кальцитоніну

Підвищенням продукції тиреоліберину

1840 / 6854
з дихальною недостатністю рН крові 7,35. Визначення pC02 40. показало наявність гіперкапнії. При дослідженні рН сечі відзначається підвищення її кислотності. Яка форма порушення кислотно-основного стану в даному випадку?

Алкалоз газовий, декомпенсований

Ацидоз газовий, компенсований

Ацидоз метаболічний, компенсований

Ацидоз метаболічний, декомпенсований

Алкалоз газовий, компенсований

1841 / 6854
Чоловіку з виразковою хворобою дванадцятипалої кишки лікар після курсу терапії пропонує вживання соків із капусти та картоплі. Вміст яких речовин в цих овочах сприяє профілактиці та загоєнню виразок?

Вітамін Бі

Вітамін K

Пантотенова кислота

Вітамін U

Вітамін C

1842 / 6854
У 12-ти річного хлопчика в сечі виявлено високий вміст усіх амінокислот аліфатичного ряду. При цьому відмічена найбільш висока екскреція цистину та цистеїну. Крім того, УЗД нирок показало наявність каменів у них. Яка патологія найбільш вірогідна?

Алкаптонурія

Фенілкетонурія

Хвороба Хартнупа

Цистинурія

Цистит

1843 / 6854
У дитини 5-ти років гостра правобічна нижньодольова пневмонія. При посіві харкотиння виявлено, що збудник захворювання стійкий до пеніциліну, але чутливий до макролідів. Який препарат найбільш доцільно використати у даному випадку?

Тетрациклін

Стрептоміцин

Гентаміцин

Ампіцилін

Азитроміцин

1844 / 6854
У дитини 10-ти років хірург запідозрив запалення дивертикула Меккеля, що потребує оперативного втручання. Ревізія якої ділянки кишечника необхідна, щоб знайти дивертикул?

0,5 м порожньої кишки від зв’язки Трейца

Висхідна ободова кишка

1м клубової кишки від місця впадіння її в товсту кишку

Низхідна ободова кишка

20 см клубової кишки від клубово-сліпокишкового кута

1845 / 6854
Чоловік 44-х років з інфарктом міокарда, помер від лівошлуночкової недостатності. На аутопсії: набряк легень, дрібнокраплинні крововиливи у серозних та слизових оболонках. Мікроскопічно: дистрофічні та некробіотичні зміни епітелію проксимальних канальців нирок, у печінці - центролобулярні крововиливи та осередки некрозу. Який з видів порушення кровообігу найбільш вірогідний?

Хронічне недокрів’я

Гостре загальне венозне повнокров’я

Хронічне загальне венозне повнокров’я

Артеріальна гіперемія

Гостре недокрів’я

1846 / 6854
У хворого гнійне запалення клиноподібної пазухи. В яку частину носової порожнини витікає гній?

Meatus nasi inferior

Meatus nasi superior

Meatus nasi communis

Meatus nasi medius

1847 / 6854
До стаціонару надійшов хворий з діагнозом: виразкова хвороба 12-палої кишки у фазі загострення, виразка цибулини 12-палої кишки. Аналіз шлункового соку показав підвищення секреторної та кислотоутворюючої функцій шлунка. Оберіть препарат, що знижує секреторну функцію залоз шлунка за рахунок блокади Н2 -рецепторів:

Екстракт беладони сухий

Платифілін

Атропіну сульфат

Метацин

Фамотидин

1848 / 6854
У хворого 30-ти років із гострим запаленням підшлункової залози (панкреатитом) виявлено порушення порожнинного травлення білків. Це може бути пов’язано із недостатнім синтезом та виділенням залозою такого ферменту:

Амілаза

Трипсин

Ліпаза

Пепсин

Дипептидаза

1849 / 6854
Хворому тривалий час вводили високі дози гідрокортизону, внаслідок чого настала атрофія однієї з зон кори наднирників. Яка це зона?

Клубочкова

Пучкова

Клубочкова і сітчаста

Сітчаста

1850 / 6854
Жінка 25-ти років скаржиться на постійний біль у ділянці серця, задишку під час рухів, загальну слабкість. Об’єктивно: шкіра бліда та холодна, акроціаноз. Ps- 96/хв., АТ- 105/70 мм рт.ст. Межа серця зміщена на 2 см вліво. Перший тон над верхівкою серця послаблений, систолічний шум над верхівкою. Діагностовано недостатність мітрального клапана серця. Чим обумовлене порушення кровообігу?

Перевантаження міокарда підвищеним опором відтоку крові

Збільшення об’єму судинного русла

Перевантаження міокарда збільшеним об’ємом крові

Зниження об’єму циркулюючої крові

Пошкодження міокарда

1851 / 6854
У потерпілого в аварії спостерігається кровотеча з м’яких тканин попереду від кута нижньої щелепи. Яку судину необхідно перев’язати для зупинки кровотечі?

A.facialis

A.carotis interna

A.alveolaris inferior

A.lingvalis

A.temporalis superficialis

1852 / 6854
При мікроскопії зіскрібку з язика, забарвленого за Грамом, знайдені овальні, округлі, темно-фіолетового кольору, видовжені ланцюжки клітин, що брунькуються. Про збудника якого захворювання може йти мова?

Актиномікоз

Стафілококова інфекція

Кандидоз

Дифтерія

Стрептококова інфекція

1853 / 6854
В препараті представлений порожнистий орган. Слизова оболонка вкрита дворядним війковим епітелієм, що переходить в однорядний. М’язова пластинка слизової добре розвинена по відношенню до товщини всієї стінки. Хряща і залоз немає. Який орган представлений в препараті?

Сечовий міхур

Гортань

Трахея

Дрібний бронх

Середній бронх

1854 / 6854
Основна маса азоту з організму виводиться у вигляді сечовини. Зниження активності якого ферменту в печінці призводить до гальмування синтезу сечовини і нагромадження амоніаку в крові і тканинах?

Уреаза

Пепсин

Аспартатамінотрансфераза

Карбамоїлфосфатсинтаза

Амілаза

1855 / 6854
Хвора 36-ти років страждає на колагеноз. Збільшення вмісту якого метаболіту найбільш вірогідно буде встановлено у сечі?

Креатинін

Сечовина

Уробіліноген

Індикан

Оксипролін

1856 / 6854
При копрологічному дослідженні встановлено, що кал знебарвлений, у ньому знайдено краплі нейтрального жиру. Найбільш вірогідною причиною цього є порушення:

Процесів всмоктування в кишечнику

Секреції кишкового соку

Секреції підшлункового соку

Кислотності шлункового соку

Надходження жовчі до кишечнику

1857 / 6854
При обстеженні хворого виявлені наступні клінічні прояви: шкірні покриви рожеві, теплі на дотик, сухі, ЧСС-92/хв.,ЧДР- 22/хв., температура тіла -39,2oC. Яке співвідношення процесів утворення і віддачі тепла в описаному періоді лихоманки?

Зниження тепловіддачі на фоні незміненої теплопродукції

Теплопродукція нижче за тепловіддачу

Теплопродукція перевищує тепловіддачу

Посилення теплопродукції без зміни тепловіддачі

Теплопродукція дорівнює тепловіддачі

1858 / 6854
Хворий 21-го року надійшов до стаціонару з загостренням хронічного тонзиліту. Скаржиться на слабкість, задуху при помірному фізичному навантаженні. Температура 37,5oC. ЧСС- 110/хв. ЕКГ: ритм синусовий, інтервал PQ подовжений. Яка аритмія у хворого?

Передсердно-шлуночкова блокада І ст.

Передсердно-шлуночкова екстрасистолія

Внутрішньопередсердна блокада

Передсердно-шлуночкова блокада ІІ ст.

Порушення внутрішньошлуночкової провідності

1859 / 6854
Секреція яких гормонів гіпофізу гальмується після прийому оральних контрацептивів, які містять статеві гормони?

Соматотропний

Вазопресин

Гонадотропні

Тиреотропні

Окситоцин

1860 / 6854
У хворого на гіпертонічну хворобу виявлено високий рівень реніну в крові. Якому з гіпотензивних засобів слід віддати перевагу в цьому випадку?

Празозин

Дихлотіазид

Ніфедіпін

Лізиноприл

Анаприлін

1861 / 6854
При гістологічному дослідженні нирки у кірковій речовині визначається каналець, вистелений одношаровим кубічним облямівчастим епітелієм, цитоплазма якого забарвлена оксифільно. Який сегмент нефрону виявлений у препараті?

Збірна трубочка

Дистальний прямий каналець

Дистальний звивистий каналець

Петля Генле

Проксимальний звивистий каналець

1862 / 6854
Встановлено ураження вірусом ВШ Т-лімфоцитів. При цьому фермент вірусу зворотня траскриптаза (РНК-залежна ДНК-полімераза) каталізує синтез:

ДНК на матриці вірусної і-РНК

Вірусної і-РНК на матриці ДНК

ДНК на вірусній р-РНК

Вірусної ДНК на матриці ДНК

і-РНК на матриці вірусного білка

1863 / 6854
Людина зробила максимально глибокий видих. Як називається об’єм повітря, що знаходиться в її легенях після цього?

Функціональна залишкова ємність легень

Альвеолярний об’єм

Залишковий об’єм

Резервний об’єм видиху

Ємність вдиху

1864 / 6854
Внаслідок активації іонних каналів зовнішньої мембрани збудливої клітини значно збільшився її потенціал спокою. Які канали були активовані?

Повільні кальцієві

Калієві

Швидкі кальцієві

Натрієві

Натрієві та кальцієві

1865 / 6854
Хворому з кардіогенним шоком, гіпотензією, ядухою і набряками ввели неглікозидний кардіотонік. Який саме препарат був введений хворому?

Бемегрид

Кордіамін

Кофеїн-бензоат натрію

Етимізол

Добутамін

1866 / 6854
Після травми хворий не може розігнути руку в ліктьовому суглобі. Порушення функції якого м’яза може бути причиною цього?

Musculus biceps brachii

Musculus subscapularis

Musculus triceps brachii

Musculus coraco-brachialis

Musculus brachialis

1867 / 6854
Під час гістологічного дослідження стулок мітрального клапана серця жінки 30-ти років було встановлено, що ендотеліальні клітини вогнищево десквамовані, в цих ділянках на поверхні стулки розташовані дрібні тромботичні нашарування, сполучна тканина стулки з явищами мукоїдного набухання з ділянками склерозу та васкуляризації. Діагностуйте вид клапанного ендокардиту:

Поліпозно-виразковий

Фібропластичний

Дифузний

Гострий бородавчастий

Поворотньо-бородавчастий

1868 / 6854
У збільшеному шийному лімфатичному вузлі дівчинки 14-ти років мікроскопічно було встановлено, що тканинна будова вузла порушена, лімфоїдні фолікули відсутні, є ділянки склерозу та вогнища некрозу, клітинний склад вузла поліморфний, присутні лімфоцити, еозинофіли, атипові клітини великих розмірів з багаточасточковими ядрами (клітини Березовського-Штернберга) та також одноядерні клітини великих розмірів. Діагностуйте захворювання:

Лімфома Беркіта

Лімфогранулематоз

Хронічний лімфолейкоз

Гострий лімфолейкоз

Грибоподібний мікоз

1869 / 6854
Який з перерахованих сечогінних засобів НЕ БУДЕ проявляти діуретичного ефекту у пацієнта з хворобою Аддісона?

Тріамтерен

Етакринова кислота

Фуросемід

Спіронолактон

Гіпотіазид

1870 / 6854
Під час розтину чоловіка, у якого після поранення кінцівки виникло тривале нагноєння рани, і який помер при явищах інтоксикації, знайдено загальне виснаження, зневоднення, бура атрофія печінки, міокарду, селезінки, поперечносмугастої мускулатури та амілоїдоз нирок. Який з перелічених діагнозів найбільш вірогідний?

Септикопіємія

Хвороба Чорногубова

Бруцельоз

Септицемія

Хроніосепсис

1871 / 6854
Препарат чинить згубний вплив на еритроцитарні форми малярійних плазмодіїв, дизентерійну амебу. Застосовується для лікування та профілактики малярії, лікування амебіазу і колагенозів. Визначте цей препарат:

Хінін

Еметину гідрохлорид

Еритроміцин

Тетрациклін

Хінгамін

1872 / 6854
У хворого у крові збільшення концентрації пірувату, значна кількість його екскретується з сечею. Який авітаміноз спостерігається у хворого?

В2

E

B1

B3

В6

1873 / 6854
Дитина страждає на ідіосинкразію лікарської речовини. Чим зумовлена ця реакція?

Інгібування мікросомальних ферментів печінки

Виснаження субстрату, з яким взаємодіє лікарська речовина

Супутнє захворювання органу-мішені

Накопичення лікарської речовини в організмі

Спадкова ензимопатія

1874 / 6854
У одного з батьків запідозрили носійство рецесивного гена фенілкетонурії. Який ризик народження у цій сім’ї дитини, хворої на фенілкетонурію?

75%

25%

50%

100%

0%

1875 / 6854
Патологоанатом у біоптаті шкіри побачив гостре серозно-геморагічне запалення і ділянку некрозу. З анамнезу: захворювання почалося з появи невеликої червоної плями, в центрі якої утворився міхур із серозно-геморагічною рідиною. Згодом центральна частина стала чорною. Який діагноз найбільш вірогідний?

Алергічний дерматит

Актиномікоз шкіри

Хімічний дерматит

Сибірковий карбункул

Карбункул стрептококовий

1876 / 6854
В легенях пацієнта, який протягом 9-ти років працював шліфувальником каменю, виявлені дрібні округлої форми щільні вузлики, що складаються зі сполучної тканини. На периферії цих вузликів розташовані макрофаги. Проявом якого захворювання є зміни в легенях?

Хронічний бронхіт

Бронхоектатична хвороба

Гостра пневмонія

Силікоз

Бронхіальна астма

1877 / 6854
У пацієнта тривалість інтервалу P — Q на ЕКГ перевищує норму при нормальній тривалості зубця P. Причиною цього є зменшення швидкості проведення збудження:

Сино-атріальним вузлом

Пучком Гіса

Волокнами Пуркін’є

Атріо-вентрикулярним вузлом

Ніжками пучка Гіса

1878 / 6854
У підлітка 12-ти років, який впродовж 3-х місяців різко схуд, вміст глюкози у крові 50 ммоль/л. У нього розвинулася кома. Який головний механізм її розвитку?

Лактацидемічний

Гіпоксичний

Гіпоглікемічний

Гіперосмолярний

Кетонемічний

1879 / 6854
Який шлях віддачі тепла тілом робітників парникового господарства є найефективнішим при температурі повітря 36oC та відносній його вологості -70%?

Конвекція

Теплопроведення

Теплорадіація

Випаровування рідини

1880 / 6854
Внаслідок землетрусу чоловік 50-ти років дві доби перебував під завалом. Після звільнення з-під завалу рятівниками у нього був встановлений синдром тривалого розчавлення. Виникнення якого ускладнення в подальшому найбільш вірогідне?

Гостра печінкова недостатшсть

Гостра судинна недостатшсть

Гостра ниркова недостатність

Гостра дихальна недостатшсть

Гостра серцева недостатшсть

1881 / 6854
У ліквідатора наслідків аварії на АЕС під час перебігу гострої променевої хвороби виник геморагічний синдром. Що має найбільше значення в патогенезі цього синдрому?

Тромбоцитопенія

Зменшення активності факторів зсідання крові

Порушення структури стінки судин

Підвищення активності факторів фібринолізу

Підвищення активності факторів систем протизсідання крові

1882 / 6854
Для лікування жовтяниць показано призначення барбітуратів, які індукують синтез УДФглюкуронілтрансферази. Лікувальний ефект при цьому обумовлений утворенням:

Білівердину

Гему

Протопорфирину

Прямого (кон’югованого) білірубіну

Непрямого (некон’югованого) білірубіну

1883 / 6854
До лікарні потрапив чоловік 35-ти років, який втратив зір на одне око. З анамнезу відомо, що хворий часто вживав недостатньо просмажений шашлик. Після рентгенологічного обстеження та проведення імунологічних реакцій лікар поставив діагноз цистіцеркоз. Який гельмінт є збудником цього захворювання?

Diphyllobothrium latum

Taenia solium

Taeniarhynchus saginatus

Trichocephalus trichiurus

Trichinella spiralis

1884 / 6854
До лікарні після автокатастрофи надійшов юнак 18-ти років. У травматологічному відділенні виявлені численні травми м’яких тканин обличчя в ділянці медіального кута ока, які призвели до масивної кровотечі. Який артеріальний анастомоз міг бути пошкоджений у цьому регіоні?

a. carotis externa et a. subclavia

a. carotis externa et a. carotis interna

a. carotis interna et a. subclavia

a. subclavia et a. ophthalmica

a. carotis interna et a. ophthalmica

1885 / 6854
У чоловіка 75-ти років, який довгий час страждав на атеросклероз церебральних судин, на аутопсії у правій тім’яно-скроневої ділянці головного мозку виявлено вогнище неправильної форми, млявої консистенції, сірого кольору. Яка найбільш вірогідна причина розвитку цього процесу?

Тромбоз правої передньої мозкової артерії

Тромбоз правої задньої мозкової артерії

Тромбоз правої середньої мозкової артерії

Тромбоз судини м’якої мозкової оболонки

Тромбоз базилярної артерії

1886 / 6854
Під час розтину трупа чоловіка зі злоякісною пухлиною шлунка, що помер від ракової інтоксикації, в задньонижніх відділах легень виявлені щільні сіро-червоного кольору неправильної форми осередки, які виступають над поверхнею розрізу. Мікроскопічно: у просвіті, стінках дрібних бронхів та альвеолах виявляється ексудат, в якому багато нейтрофілів. Про яке захворювання свідчать зміни у легенях померлого?

Проміжна пневмонія

Гостра гнійна бронхопневмонія

Гостра серозна бронхопневмонія

Крупозна пневмонія

Гострий бронхіт

1887 / 6854
При оперативному втручанні з приводу каменів жовчних ходів хірург повинен знайти загальну печінкову протоку. Між листками якої зв’язки вона знаходиться?

Печінково-шлункова

Кругла зв’язка печінки

Печінково-ниркова

Венозна зв’язка

Печінково-дванадцятипала

1888 / 6854
Під час розтину тіла померлої дитини 1,5 років виявлені: геморагічний висип на шкірі, помірна гіперемія та набряк слизової оболонки носоглотки, дрібні крововиливи у слизових оболонках і внутрішніх органах, різкі дистрофічні зміни у печінці, міокарді, гострий некротичний нефроз, масивні крововиливи у наднирниках. Для якого захворювання найбільш характери виявлєні зміни?

Скарлатина

Висипний тиф

Дифтєрія

Менінгококова інфекція

Кір

1889 / 6854
Хвора 75-ти років доставлена до офтальмологічного відділення лікарні зі скаргами на погіршення зору. При об’єктивному дослідженні встановлена наявність пухлини мозку, розташованої в ділянці лівого зорового тракту. При цьому у хворої спостерігається випадіння поля зору в:

Правих і лівих половинах сітківки правого ока

Правих і лівих половинах сітківок обох очей

Правих і лівих половинах сітківки лівого ока

Лівих половинах сітківки обох очей

Правих половинах сітківки обох очей

1890 / 6854
У хворого на ішемічну хворобу серця відзначається гіпертрофія міокарда, тахікардія, зниження ХОК. Який з механізмів є провідним в ушкодженні кардіоміоцитів у даному випадку?

Втрата Mg2+ кардіоміоцитами

Пошкодження специфічних мембранних насосів

Збільшення числа а та β-адренорецепторів

Дегідратація кардіоміоцитів

Втрата Ca2+ кардіоміоцитами

1891 / 6854
У хворого на жовтяницю у крові підвищений вміст прямого білірубіну та жовчних кислот; у сечі відсутній стеркобіліноген. При якій жовтяниці можлива наявність цих ознак?

Печінкова

Надпечінкова

Механічна

Гемолітична

Паренхіматозна

1892 / 6854
Проводиться вивчення максимально спіралізованих хромосом каріотипу людини. При цьому процес поділу клітини припинили на стадії:

Профаза

Інтерфаза

Метафаза

Анафаза

Телофаза

1893 / 6854
У людини вимірюють енерговитрати натщесерце, лежачи, в умовах фізичного і психічного спокою, при температурі комфорту. В який час енерговитрати будуть найбільшими?

14-16 годин дня

3-4 години ранку

10-12 годин дня

7-8 годин ранку

17-18 годин вечора

1894 / 6854
При визначенні енерговитрат організму людини методом непрямої калориметрії встановлено, що за одну хвилину споживається 1000 мл кисню і виділяється 800 мл вуглекислого газу. Який дихальний коефіцієнт у досліджуваної людини?

0,84

1,25

0,8

1,0

0,9

1895 / 6854
Тварині внутрішньовенно ввели концентрований розчин хлориду натрію, що зумовило зниження реабсорбції іонів натрію у канальцях нирок. Внаслідок яких змін секреції гормонів це відбувається?

Зменшення вазопресину

Зменшення натрійуретичного фактора

Збільшення вазопресину

Зменшення альдостерону

Збільшення альдостерону

1896 / 6854
У хворого діагностовано алкаптонурію. Вкажіть фермент, дефект якого є причиною цієї патології:

Глутаматдегідрогеназа

Піруватдегідрогеназа

ДОФА-декарбоксилаза

Фенілаланінгідроксилаза

Оксидаза гомогентизинової кислоти

1897 / 6854
При хронічному передозуванні глюкокортикоїдів розвивається гіперглікемія. Назвіть процес вуглеводного обміну, за рахунок якого збільшується концентрація глюкози у плазмі крові:

Шкогенез

Глюконеогенез

Пентозофосфатний цикл

Глікогеноліз

Аеробний гліколіз

1898 / 6854
У хворої 49-ти років відзначається обмеження довільних рухів у лівих кінцівках. Тонус м’язів у лівих руці та нозі підвищений за спастичним типом, посилені місцеві сухожилкові рефлекси, виявляються патологічні рефлекси. Який найбільш вірогідний механізм призвів до розвитку м’язової гіпертонії та гіперрефлексії?

Активація синаптичної передачі імпульсів

Активація мотонейронів внаслідок інсульту

Зниження гальмівних низхідних впливів

Гальмування мотонейронів кори головного мозку

Активація збуджуючих впливів з вогнища інсульту

1899 / 6854
При виконуванні вправ на колоді гімнастка втратила рівновагу і впала. Із збудження, перш за все, яких рецепторів розпочнуться рефлекси, що забезпечать відновлення порушеної пози?

Ампулярні вестибулорецептори

Отолітові вестибулорецептори

Вестибулорецептори

Пропріорецептори

Рецептори завитки

1900 / 6854
Вірус імунодефіциту людини, маючи на своїй поверхні антигени gp — 41 та gp — 120, взаємодіє з клітинами-мішенями організму. Виберіть серед перерахованих антигени лімфоцитів людини, з якими комплементарно зв’язується gp — 120 вірусу:

CD 19

CD 8

CD 4

CD 28

CD 3

1901 / 6854
В анотації до препарату вказано, що він містить антигени збудника черевного тифу, адсорбовані на стабілізованих еритроцитах барана. З якою метою використовують цей препарат?

Для виявлення антитіл в реакції зв’язування комплементу

Для виявлення антитіл в реакції затримки гемаглютинації

Для виявлення антитіл в реакції непрямої гемаглютинації

Для виявлення антитіл в реакції Відаля

Для серологічної ідентифікації збудника черевного тифу

1902 / 6854
При обстеженні вагітної жінки з резус-негативною кров’ю знайдено високий рівень антиеритроцитарних антитіл, для зниження якого їй було підшито шкірний клапоть її резус-позитивного чоловіка. Через 2 тижні клапоть відторгнувся, мікроскопічно в ньому знайдено порушення кровообігу, набряк, клітинну інфільтрацію переважно лімфоцитами, нейтрофілами та макрофагами. Який з перелічених патологічних процесів найбільш вірогідний?

Інтерстиційне запалення

Реакція гіперчутливості сповільненого типу

Реакція гіперчутливості негайного типу

Гранулематозне запалення

Трансплантаційний імунітет

1903 / 6854
Дівчина 17-ти років з метою суїциду прийняла велику дозу фенобарбіталу. Після прибуття на місце події лікар швидкої допомоги швидко промив шлунок, ввів бемегрид і розчин натрію гідрокарбонату внутрішньовенно. З якою метою лікар ввів натрію гідрокарбонат?

Для підвищення ниркової екскреції фенобарбіталу

Для нормалізації артеріального тиску

Для інактивації фенобарбіталу

Для стимуляції дихання

Для пробуджуючого ефекту

1904 / 6854
Стоматолог під час огляду порожнини рота на межі середньої і задньої третини спинки язика виявив запалені сосочки. Які сосочки язика запалені?

Papillae conicae

Papillae fungiformes

Papillae vallatae

Papillae foliatae

Papillae filiformes

1905 / 6854
При лабораторному дослідженні крові пацієнта 44-х років виявлено, що вміст білків у плазмі становить 40 г/л. Як це впливає на транскапілярний обмін води?

Зменшується фшьтращя, збільшується реабсорбція

Зменшуються фшьтращя та реабсорбція

Обмін не змінюється

Збільшується фільтрація, зменшується реабсорбція

З6ільшуються фiльтрацiя та реабсорбція

1906 / 6854
У людини звужені зіниці. Чим це зумовлено?

Збільшення активності симпато-адреналової системи

Зростання тонусу парасимпатичних центрів

Дія адреналіну

Зростання тонусу симпатичних центрів

Дія норадреналіну

1907 / 6854
У чоловіка 46-ти років на шкірі визначалась темна пляма, яка вибухала та не спричиняла турбот. З часом пляма почала збільшуватись, з’явився біль, колір став чорно-коричневим і почав пальпуватися вузлик. При гістологічному дослідженні видаленої тканини визначалися веретеноподібні і поліморфні клітини з численними мітозами, цитоплазма яких вміщувала пігмент бурого кольору. Про яку пухлину йдеться?

Невус

Меланома

Гемангіома

Базаліома

1908 / 6854
У немовля с пілороспазмом внаслідок блювання, що часто повторювалося, з’явилися слабкість, гіподинамія, іноді судоми. Яка форма порушення кислотно-основного стану в нього спостерігається?

Видільний алкалоз

Газовий алкалоз

Метаболічний ацидоз

Екзогенний негазовий ацидоз

Видільний ацидоз

1909 / 6854
На електронній мікрофотографії клітини, яка має паличкоподібне ядро та веретеноподібну форму, у цитоплазмі спостерігається велика кількість проміжних мікрофіламентів, які містять десмін. З якої тканини зроблено зріз?

М’язова

Епітеліальна

Нервова

Сполучна

1910 / 6854
У важкоатлета при підйомі штанги відбувся розрив грудної лімфатичної протоки. Вкажіть найбільш вірогідне місце пошкодження:

У задньому середостінні

У ділянці шиї

У ділянці попереково-крижового зчленування

У ділянці аортального отвору діафрагми

У місці впадіння у венозний кут

1911 / 6854
У жінки, що хворіє на остеохондроз, з’явився різкий біль у плечовому суглобі, який посилювався при відведенні плеча. Ураження якого нерва може бути причиною цих симптомів?

Грудо-спинний нерв

Дорсальний нерв лопатки

Підлопатковий нерв

Пахвовий нерв

Підключичний нерв

1912 / 6854
На рентгенограмі нирок при пієлографії лікар виявив ниркову миску, в яку безпосередньо впадали малі чашечки (великі були відсутні). Яку форму сечовивідних шляхів нирки виявив лікар?

Ампулярна

Деревоподібна

Зріла

Фетальна

Ембріональна

1913 / 6854
У хворої внаслідок запалення порушена ендокринна функція фолікулярних клітин фолікулів яєчника. Синтез яких гормонів буде пригнічений?

Естрогени

Фолістатин

Лютропін

Фолікулостимулюючий гормон

Прогестерон

1914 / 6854
У хворої 19-ти років з дитинства відмічалося зниження гемоглобіну до 90-95 г/л. У крові під час госпіталізації: ер.- 3, 2 • 1012/л, Hb- 85 г/л, КП- 0,78; лейк.- 5, 6 • 109/л, тромб.- 210 • 109 /л. В мазку: анізоцитоз, пойкілоцитоз, мішенеподібні еритроцити. Ретикулоцити - 6%. Лікування препаратами заліза було неефективне. Яку патологію системи крові можна запідозрити в даному випадку?

Ферментопагія

Таласемія

Фавізм

Серпоподібноклітинна анемiя

Мембранопагія

1915 / 6854
Хворому на гострий інфаркт міокарда у комплексній терапії було призначено гепарин. Через деякий час після введення даного препарату з’явилась гематурія. Який антагоніст гепарину необхідно ввести хворому для усунення даного ускладнення?

Неодикумарин

Амінокапронова кислота

Протаміну сульфат

Фібриноген

Вікасол

1916 / 6854
Пацієнта турбують поліурія (7л на добу) і полідипсія. При обстеженні не виявлено ніяких розладів вуглеводного обміну. Дисфункція якої ендокринної залози може бути причиною даних порушень?

Острівці підшлункової залози

Кора наднирників

Нейрогіпофіз

Мозкова речовина наднирників

Аденогіпофіз

1917 / 6854
У чоловіка 30-ти років перед операцією визначили групову належність крові. Кров резуспозитивна. Реакцію аглютинації еритроцитів не викликали стандартні сироватки груп 0αβ (I), Аβ (II), Вα (III). Досліджувана кров належить до групи:

Вα (III)

АВ (IV)

Аβ (II)

0αβ (I)

1918 / 6854
У жінки 20-ти років під час медичного огляду при пальпації в молочній залозі виявлено щільний інкапсульований вузол діаметром 1,0 см. Результат післяопераційного біопсійного дослідження: розростання сполучної тканини навколо протоків молочної залози та залозисті елементи різного діаметру, що не утворюють часточок, без ознак клітинного атипізму. Яка відповідь патологоанатома?

Фіброаденома

Фіброзний рак

Аденома

Метастаз раку

Фіброма

1919 / 6854
У хворого 38-ми років на 3-му році захворювання на системний червоний вівчак виявлене дифузне ураження нирок, що супроводжується масивними набряками і вираженою протеїнурією. Що є найбільш вірогідною причиною розвитку протеїнурії у пацієнта?

Аутоімунне ушкодження нирок

Немічне ушкодження нирок

Запальне ураження сечовивідних шляхів

Запальне ураження сечового міхура

Асептичне ураження нирок

1920 / 6854
Студент на екзамені не зміг вірно відповісти на питання екзаменаційного білету, що супроводжувалося почервонінням шкіри обличчя, відчуттям жару і невпевненістю поведінки. Який вид артеріальної гіперемії розвинувся у даному випадку?

Постішемічна

Патологічна

Нейропаралітична

Нейротонічна

Метаболічна

1921 / 6854
У хворого з кровотечею розвинулась гостра ниркова недостатність, що спричинила смерть. На аутопсії макроскопічно: нирки збільшені з широким блідо-рожевим кірковим шаром, різко відмежованим від темно-червоних пірамід. Мікроскопічно: відсутність ядер епітелію звивистих канальців, тубулорексіс, венозний застій, ядра клітин судинних клубочків та прямих канальців збережені. Яка патологія нирок розвинулась у хворого?

Інфаркт

Некронефроз

Гломерулонефрит

Нефроз

Пієлонефрит

1922 / 6854
Хвора звернулась до лікаря із скаргами на біль та обмеження рухів у колінних суглобах. Який з нестероїдних протизапальних засобів краще призначити, враховуючи наявність в анамнезі хронічного гастродуоденіту?

Целекоксиб

Бутадіон

Диклофенак-натрій

Промедол

Кислота ацетилсаліцилова

1923 / 6854
У хворого з’явилися жовтушність шкіри, склер та слизових оболонок. У плазмі крові підвищений рівень загального білірубіну, в калі - рівень стеркобіліну, в сечі - уробіліну. Який вид жовтяниці у хворого?

Обтураційна

Хвороба Жільбера

Холестатична

Гемолітична

Паренхіматозна

1924 / 6854
У плазмі крові здорової людини знаходиться декілька десятків білків. При захворюванні організму з’являються нові білки, зокрема 'білок гострої фази'. Таким білком є:

Протромбін

Фібриноген

Імуноглобулін G

С-реактивний білок

Імуноглобулін А

1925 / 6854
Під час бійки у чоловіка виникла зупинка серця внаслідок сильного удару у верхню ділянку передньої черевної стінки. Який із зазначених механізмів спричинив зупинку серця?

Парасимпатичні умовні рефлекси

Парасимпатичні безумовні рефлекси

Симпатичні умовні рефлекси

Симпатичні безумовні рефлекси

Периферичні рефлекси

1926 / 6854
Для людини існує суворе обмеження в часі перебування на висоті понад 800 метрів над рівнем моря без кисневих балонів. Що є лімітуючим фактором для життя в даному випадку?

Парціальний тиск кисню в повітрі

Рівень вологості

Сила земного тяжіння

Рівень ультрафіолетового опромінення

Температура

1927 / 6854
У хворого із вираженим пневмосклерозом після перенесеного інфільтративного туберкульозу легень розвинулась дихальна недостатність. До якого патогенетичного типу вона відноситься?

Рефлекторний

Обструктивний

Апнеїстичний

Рестриктивний

Дисрегуляційний

1928 / 6854
Під час проведення хірургічних маніпуляцій було використано новокаїн з метою знеболення. Через 10 хвилин у хворого з’явилася блідість шкірних покривів, задишка, гіпотензія. Алергічну реакцію якого типу можна запідозрити?

Клітинно-опосередкована

Цитотоксична

Стимулююча

Імунокомплексна

Анафілактична

1929 / 6854
Чутливий нервовий ганглій складається з нейроцитів кулястої форми з одним відростком, який на певній відстані від перикаріону поділяється на аксон і дендрит. Як називаються такі клітини?

біполярні

псевдоуніполярні

аполярні

мультиполярні

уніполярні

1930 / 6854
У препараті трубчастого органу, зафарбованому орсеїном, виявлено близько 50 товстих мембран, які мають хвилясту форму і складають основу середньої оболонки органу. Який це орган?

Стравохід

Аорта

Стінка серця

Трахея

Артерія м’язового типу

1931 / 6854
Встановлено, що деякі сполуки, наприклад, токсини грибів та деякі антибіотики, можуть пригнічувати активність РНК-полімерази. Порушення якого процесу відбувається в клітині у випадку пригнічування даного ферменту?

Процесінг

Трансляція

Реплікація

Транскрипція

Репарація

1932 / 6854
У дорослої людини системний артеріальний тиск знизився з 120/70 до 90/50 мм рт.ст., що викликало рефлекторне звуження судин. У якому з зазначених органів звуження судин буде найменшим?

Скелетні м’язи

Кишечник

Серце

Печінка

Шкіра

1933 / 6854
На електронній мікрофотографії фрагменту власної залози шлунка представлена велика клітина неправильної кулястої форми, у цитоплазмі якої є велика кількість внутрішньоклітинних канальців та мітохондрій. Визначте дану клітину:

Парієтальна

Головна

Слизова

Ендокринна

Недиференційована

1934 / 6854
крові у спортсмена виявлено: ер.- 5,5 • 1012/л, Hb- 180 г/л, лейк.- 7-109 134. /л,н.- 64%, б.- 0,5%,е.- 0,5%, м.-8%,л.-27%. Такі показники свідчать про стимуляцію, перш за все:

Еритропоезу

Лімфопоезу

Імуногенезу

Лейкопоезу

Гранулоцитопоезу

1935 / 6854
Хворий помер від наростаючої легенево-серцевої недостатності. При гістологічному дослідженні виявлено: дифузне ураження легенів з інтерстиціальним набряком, інфільтрацією інтерстиціальної тканини лімфоцитами, макрофагами, плазмоцидами; пневмофіброз, панацинарна емфізема. Який найбільш вірогідний діагноз?

Бронхіальна астма

Фіброзуючий альвеоліт

Ателектаз легенів

Хронічний бронхіт

Бронхопневмонія

1936 / 6854
У людини з нападом бронхоспазму необхідно зменшити вплив блукаючого нерва на гладеньку мускулатуру бронхів. Які мембранні циторецептори доцільно заблокувати для цього?

M-холінорецептори

α- та β-адренорецептори

β-адренорецептори

α-адренорецептори

H-холінорецептори

1937 / 6854
У хворого виявлена виразка шлунка. При бактеріологічному дослідженні біоптату, отриманого із ушкодженої ділянки шлунка, на 5-й день на шоколадному агарі виросли дрібні колонії грамнегативних оксидоредуктазопозитивних звивистих бактерій. Який із перерахованих мікроорганізмів є найбільш вірогідним збудником?

Helicobacter pylori

Campilobacter jejuni

Campilobacter fetus

Chlamydia trachomatis

Mycoplasma hominis

1938 / 6854
Виділяють декілька груп молекулярних механізмів, які мають важливе значення в патогенезі ушкодження клітин, що сприяє розвитку патології. Які процеси забезпечують протеїнові механізми ушкодження?

Осмотичне розтягнення мембран

Пригнічення ферментів

Ацидоз

Активація фосфоліпаз

Перекисне окиснення ліпідів

1939 / 6854
У пацієнта діагностовано компресійний перелом поперекового хребця. При цьому різко збільшилась кривизна лордозу поперекового відділу хребта. Ушкодженням якої зв’язки може супроводжуватись така зміна кривизни хребта?

Міжостиста

Задня поздовжня

Передня поздовжня

Клубово-поперекова

Жовта

1940 / 6854
При розтині тіла померлого 49-ти років, який хворів на крупозну пневмонію і помер від пневмококового сепсису, в лівій плевральній порожнині містилось до 700 мл каламутної рідини зеленувато-жовтого кольору з неприємним запахом. Листки плеври тьмяні, повнокровні. Назвіть клініко-морфологічну форму запалення в плевральній порожнині:

Емпієма

Гострий абсцес

Хронічний абсцес

Фі6ринознє запалення

Флегмона

1941 / 6854
У чоловіка 35-ти років під час тривалого 6ігу виникла гостра серцева недостатність. Які зміни іонного складу спостерігаються у серцевому м’язі при цьому стані?

Зменшення в клітинах міокарда іонів Na+ і Ca2+

Накопичення в клітинах міокарда іонів K + і Mg2+

Накопичення в клітинах міокарда іонів Na+ і Ca2+

Зменшення в позаклітинному просторі іонів K + і Mg2+

Збільшення в позаклітинному просторі іонів Na+ і Ca2+

1942 / 6854
У дитини, що часто хворіє на ангіни і фарингіти, відзначається збільшення лімфовузлів та селезінки. Зовнішній вигляд характеризується пастозністю і блідістю, м’язова тканина розвинена слабко. У крові спостерігається лімфоцитоз. Як називається такий вид діатезу?

Ексудативно-катаральний

Астенічний

Нервово-артритичний

Геморагічний

Лімфатико-гіпопластичний

1943 / 6854
На перехід із горизонтального положення у вертикальне система кровообігу відповідає розвитком рефлекторної пресорної реакції. Що з наведеного є її обов’язковим компонентом?

Системне звуження венозних судин ємності

Системне розширення артеріальних судин опору

Зменшення частоти серцевих скорочень

Зменшення об’єму циркулюючої крові

Зменшення насосної функції серця

1944 / 6854
Дитині після аналізу імунограми встановили діагноз первинний імунодефіцит гуморальної ланки імунітету. Яка з причин може призвести до розвитку первинного імунодефіциту в організмі дитини?

Порушення в процесі ембріонального розвитку

Спадкові порушення в імунній системі

Токсичне пошкодження В-лімфоцитів

Порушення обміну речовин в організмі матері

Порушення реактивності та резистентності організму

1945 / 6854
У життєвому циклі клітини відбувається процес самоподвоєння ДНК. В результаті цього однохроматидні хромосоми стають двохроматидними. У який період клітинного циклу спостерігається це явище?

M

G2

S

G0

G1

1946 / 6854
Через дві доби після пологів у жінки розвинулася клініка шоку з ДВЗ-синдромом, внаслідок чого породілля померла. На аутопсії виявлений гнійний ендоміометрит, регіональний гнійний лімфангіт і лімфаденіт, гнійний тромбофлебіт. У паренхіматозних органах - дистрофічні зміни і проміжне запалення. Який діагноз найбільш вірогідний?

Міхуровий занесок

Сепсис

Туберкульоз статевих органів

Сифіліс

Деструюючий міхуровий занесок

1947 / 6854
Інозитолтрифосфати в тканинах організму утворюються в результаті гідролізу фосф атидилінозитолдифосфатів і відіграють роль вторинних посередників (месенджерів) в механізмі дії гормонів. Їхній вплив у клітині спрямований на:

Вивільнення іонів кальцію з клітинних депо

Активацію аденілатциклази

Активацію протеїнкінази А

Гальмування протеїнкінази С

Гальмування фосфодіестерази

1948 / 6854
Хворому на ішемічну хворобу серця з аритмією призначено препарат, який переважно блокує калієві канали, суттєво подовжує потенціал дії. Який препарат призначено?

Корглікон

Лізиноприл

Аміодарон

Добутамін

Нітрогліцерин

1949 / 6854
Для лікування хронічної серцевої недостатності хворий приймає дигоксин. Який діуретичний засіб може збільшити токсичність дигоксину за рахунок посиленого виведення з організму іонів K+?

Лізиноприл

Силібор

Панангін

Спіронолактон

Гідрохлортіазид

1950 / 6854
Під час голодування м’язові білки розпадаються до вільних амінокислот. В який процес найімовірніше будуть втягуватися ці сполуки за таких умов?

Глікогеноліз

Глюконеогенез у м’язах

Декарбоксилування

Синтез вищих жирних кислот

Глюконеогенез у печінці

1951 / 6854
Хвора звернулася до лікаря зі скаргами на неприємні відчуття в епігастрії, нудоту, зниження апетиту. При дослідженні дуоденального вмісту виявлено лямблії. Який препарат слід призначити?

Ізоніазид

Рифампіцин

Ацикловір

Метронідазол

Хінгамін

1952 / 6854
До гінеколога звернулася жінка 28-ми років з приводу безпліддя. При обстеженні знайдено: недорозвинені яєчники та матка, нерегулярний менструальний цикл. При дослідженні статевого хроматину в більшості соматичних клітин виявлено два тільця Бара. Яка хромосомна хвороба найбільш вірогідна у жінки?

Синдром трипло-Х

Синдром Шерешевського-Тернера

Синдром Едвардса

Синдром Патау

Синдром Клайнфельтера

1953 / 6854
До клініки потрапила дитина 1-го року з ознаками ураження м’язів кінцівок та тулуба. Після обстеження виявлений дефіцит карнітину в м’язах. Біохімічною основою цієї патології є порушення процесу:

Регуляції рівня Ca2+ в мітохондріях

Транспорту жирних кислот у мітохондрії

Окисного фосфорилювання

Субстратного фосфорилювання

Утилізації молочної кислоти

1954 / 6854
У хворого при проведенні наркозу виникли ознаки підвищення тонусу парасимпатичної нервової системи: гіперсалівація, ларингоспазм. Введенням якого препарату можна було попередити небажані ефекти?

Адреналіну гідрохлорид

Атропіну сульфат

Прозерин

Анальгін

Пірацетам

1955 / 6854
Після тривалого фізичного навантаження під час заняття з фізичної культури у студентів розвинулась м’язова крепатура. Причиною її виникнення стало накопичення у скелетних м’язах молочної кислоти. Вона утворилась після активації в організмі студентів:

Ліполізу

Гліколізу

Глюконеогенезу

Глікогенезу

Пентозофосфатного циклу

1956 / 6854
На прохання лікаря хворий після звичайного вдиху зробив максимально глибокий видих. Скорочення яких із наведених м’язів бере участь у такому видиху?

Діафрагма

Зовнішні міжреберні

Живота

Трапецієподібні

Грудні

1957 / 6854
На електронній мікрофотографії науковець виявив структуру, утворену вісьмома молекулами білків-гістонів і ділянкою молекули ДНК, що робить близько 1,75 обертів навколо них. Яку структуру виявив дослідник?

Нуклеосома

Елементарна фібрила

Хромосома

Напівхроматида

Хроматида

1958 / 6854
У хворого з масивними опіками розвинулась гостра недостатність нирок, що характеризується значним і швидким зменшенням швидкості клубочкової фільтрації. Який механізм її розвитку?

Емболія ниркової артерії

Ушкодження клубочкового фільтра

Збільшення тиску канальцевої рідини

Зменшення кількості функціонуючих нефронів

Зменшення ниркового кровотоку

1959 / 6854
У фекаліях трирічної дитини із сильновираженою діареєю, яка хворіє впродовж 3-х днів, за методом імунної електронної мікроскопії виявлено віруси із двошаровим капсидом і псевдоо-болонкою, які нагадують дрібні коліщатка із спицями. Які віруси найбільш вірогідно виявлені?

Ротавіруси

Реовіруси

Коронавіруси

Коксакі-віруси

ЕСНО-віруси

1960 / 6854
У хворого з лихоманкою та висипкою на шкірі після обстеження за допомогою серологічних реакцій поставлений діагноз фасціольоз. Було встановлено, що хворий заразився шляхом споживання сирої води з річки. Яка стадія життєвого циклу фасціоли інвазійна для людини?

Фіна

Метацеркарій

Яйце

Мірацидій

Адолескарій

1961 / 6854
У хворого з високим титром антинуклеарних антитіл смерть настала від наростаючої ниркової недостатності. При патологоанатомічному дослідженні виявлений мезангіопроліферативний гломерулонефрит, абактеріальний бородавчастий ендокардит; в селезінці - періартеріальний цибулинний склероз, в шкірі - продуктивнопроліферативний васкуліт. Який найбільш вірогідний діагноз?

Дерматоміозит

Нефротичний синдром

Системний червоний вівчак

Ревматизм

Вузликовий періартеріїт

1962 / 6854
При харчовому отруєнні виділена культура анаеробних грампозитивних спороутворюючих паличок. До якого виду, найбільш вірогідно, відноситься виділений збудник?

P.mirabilis

Esherichia coli

C. perfringens

Vibrio parahemolyticus

Proteus vulgaris

1963 / 6854
У тварини збільшений тонус м’язів-розгиначів. Це є наслідком посиленої передачі інформації до мотонейронів спинного мозку такими низхідними шляхами:

Вестибулоспінальні

Руброспінальні

Ретикулоспінальні

Латеральні кортикоспінальні

Медіальні кортикоспінальні

1964 / 6854
Хворий 70-ти років помер від гострої коронарної недостатності. За життя відзначалась припухлість, деформація і біль колінних суглобів. При патоморфологічному дослідженні деформованих суглобів і синовіальних оболонок виявлено: гіперемія оболонок з множинними периваскулярними запальними інфільтратами з лімфоцитів, плазмоцитів, макрофагів. Скупчення організованого фібрину, який вкриває ділянки синовіальної оболонки і визначається в суглобовій рідині у вигляді рисових зерен. Який найбільш вірогідний діагноз?

Ревматоїдний артрит

Вузликовий періартеріїт

Туберкульозний артрит

Деформуючий артроз

Анкілозуючий спондилоартрит

1965 / 6854
До відділення нейрохірургії було доставлено чоловіка з втратою слуху внаслідок травми голови. Порушення якої частки кори головного мозку може бути причиною цього?

постцентральна звивина

потилична

лобова

тім’яна

скронева

1966 / 6854
Вагітній жінці, під час встановлення на облік у жіночу консультацію, було проведено клінічне обстеження на ряд інфекцій. У сироватці крові були виявлені lgM до вірусу краснухи. Про що свідчить такий результат?

Загострення хронічного процесу

Жінка здорова

Хронічний процес

Повторне інфікування вірусом краснухи

Первинне зараження жінки

1967 / 6854
Хворий, що страждав на туберкульоз, помер від прогресуючої легенево-серцевої недостатності. На розтині в ділянці верхівки правої легені визначається порожнина діаметром 5 см, яка сполучається з просвітом сегментарного бронха. Стінки порожнини з середини покриті сирнистими масами, під якими знаходяться епітеліоїдні клітини і клітини Пирогова-Лангханса. Вкажіть морфологічну форму туберкульозу:

Гострий осередковий туберкульоз

Гострий кавернозний туберкульоз

Казеозна пневмонія

Туберкульома

Інфільтративний туберкульоз

1968 / 6854
До бактеріологічної лабораторії доставлені блювотні маси хворого з підозрою на холеру. З матеріалу приготований препарат 'висяча крапля'. Який метод мікроскопії буде використаний для виявлення збудника по його рухливості?

Імунна електронна

Шерстна

Люмінесцентна

Електронна

Фазово-контрастна

1969 / 6854
У хворого на ентерит, що супроводжувався значною діареєю, спостерігається зменшення кількості води в позаклітинному просторі, збільшення її всередині клітин та зниження осмолярності крові. Як називають таке порушення водно-електролітного обміну?

Гіпоосмолярна гіпергідратація

Гіпоосмолярна гіпогідратація

Гіперосмолярна гіпергідратація

Гіперосмолярна гіпогідратація

Осмолярна гіпогідратація

1970 / 6854
При огляді 6-місячної дитини лікар виявив незакрите заднє тім’ячко. В якому віці воно закривається при нормальному розвитку дитини?

До 6-ти місяців

До кінця другого року життя

До народження

До кінця першого року життя

До 3-х місяців

1971 / 6854
У новонародженої дитини спостерігаються зниження інтенсивності смоктання, часте блювання, гіпотонія. У сечі та крові значно підвищена концентрація цитруліну. Який метаболічний процес порушений?

Глюконеогенез

ЦТК

Гліколіз

Цикл Корі

Орнітиновий цикл

1972 / 6854
При обстеженні в клініці у чоловіка діагностували гостру променеву хворобу. Лабораторно встановлено різке зниження вмісту серотоніну в тромбоцитах. Порушення метаболізму якої речовини є можливою причиною зниження тромбоцитарного серотоніну?

Тирозин

Серин

Гістидин

Фенілаланін

5-окситріптофан

1973 / 6854
У хворої дитини виявлена затримка розумового розвитку, збільшення печінки, погіршення зору. Лікар пов’язує ці симптоми з дефіцитом в організмі галактозо-1- фосфатуридилтрансферази. Який патологічний процес має місце у дитини?

Гіпоглікемія

Гіперглікемія

Галактоземія

Фруктоземія

Гіперлактатацидемія

1974 / 6854
В умовному експерименті повністю інгібовано розвиток клітин мезенхіми. Порушення розвитку якої м’язової тканини при цьому буде спостерігатися?

Серцева м’язова тканина

М’язова тканина епідермального походження

Скелетна м’язова тканина

Гладенька м’язова тканина

М ’язова тканина нейрального походження

1975 / 6854
На гістологічному препараті паренхіма органу представлена лімфоїдною тканиною, яка утворює лімфатичні вузлики, останні розташовані дифузно і містять центральну артерію. Яке анатомічне утворення має дану морфологічну будову?

Тимус

Лімфатичний вузол

Селезінка

Червоний кістковий мозок

Мигдалик

1976 / 6854
У потерпілого з травмою голови у скроневій ділянці діагностовано епідуральну гематому. Яка з артерій найвірогідніше пошкоджена?

Передня оболонкова

Поверхнева скронева

Середня мозкова

Задня вушна

Середня оболонкова

1977 / 6854
У дитини спостерігається затримка фізичного та розумового розвитку, глибокі порушення з боку сполучної тканини внутрішніх органів, у сечі виявлено кератансульфати. Обмін яких речовин порушений?

Еластин

Глікозаміноглікани

Гіалуронова кислота

Колаген

Фібронектин

1978 / 6854
Видалення зуба у пацієнта з хронічним персистуючим гепатитом ускладнилось тривалою кровотечею. Яка причина геморагічного синдрому?

Збільшення утворення тромбопла-стину

Зменшення утворення фібрину

Збільшення синтезу фібриногену

Зменшення утворення тромбіну

Посилення фібринолізу

1979 / 6854
У жінки під час гаметогенезу (в мейозі) статеві хромосоми не розійшлися до протилежних полюсів клітини. Яйцеклітина була запліднена нормальним сперматозоїдом. Яке хромосомне захворювання може бути у дитини?

Синдром Дауна

Синдром Шерешевського-Тернера

Синдром Едвардса

Синдром котячого крику

Синдром Патау

1980 / 6854
Хворій для лікування алергічного хейліту призначили лоратадин. Який механізм дії даного лікарського засобу?

Підвищує активність моноамінокси-дази

Блокує адренорецептори

Пригнічує активність Na,K — АТФ — ази

Блокує Н1-гістамінові рецептори

Пригнічує активність холінестерази

1981 / 6854
Хворому після апендектомії з метою профілактики інфекції призначено антибіотик групи цефалоспоринів. Порушення якого процесу лежить в основі протимікробної активності антибіотиків цієї групи?

Блокада холінестерази

Синтез нуклеїнових кислот

Утворення мікробної стінки

Рибосомальний синтез білку

Енергетичний обмін

1982 / 6854
За медичним показанням пацієнту було проведено видалення частини однієї із структур ЦНС. В результаті видалення у пацієнта розвинулися атонія, астазія, інтенційний тремор, атаксія, адіадохокінез. Частина якої структури ЦНС була вилучена?

Мозочок

Базальні ганглії

Мигдалеподібний комплекс

Гіпокамп

Лімбічна система

1983 / 6854
У хворого діагностовано грип. Після прийому антипіретиків стан його різко погіршився: свідомість потьмарена, АТ- 80/50 мм рт.ст., Ps-140/хв., температура тіла різко знизилась до 35,8oC. Яке ускладнення виникло у даного хворого?

Колапс

Алкалоз

Гіпертермія

Гіповолемія

Ацидоз

1984 / 6854
Пацієнт звернувся до лікаря з приводу втрати здатності розрізняти смаки на корені язика. Лікар встановив, що це пов’язано з ураженням нерва. Якого?

Лицевий

Блукаючий

Язикоглотковий

Трійчастий

Верхньогортанний

1985 / 6854
Яким буде скорочення м’язів верхньої кінцівки при утриманні (але не переміщенні) вантажу в певному положенні?

Ауксотонічним

Ексцентричним

Ізометричним

Ізотонічним

Концентричним

1986 / 6854
Жінка. похилого віку госпіталізована зі скаргами на різкий 6іль, набряк в ділянці правого кульшового суглоба, що з’явилися після падіння. Об’єктивно: стегно приведене досередини, рухи в кульшовому суглобі порушені. Перелом якої кістки або її частини визначив лікар?

Сіднична кістка

Шийка стегнової кістки

Виростки стегнової кістки

Тіло стегнової кістки

Лобкова кістка

1987 / 6854
Жінка 45-ти років хворіє на рак лівої молочної залози. На лівій руці є ознаки недостатності лімфатичної системи - набряк кінцівки, збільшення лімфовузлів. Яка форма недостатності лімфообміну спостерігається у хворої?

Динамічна недостатність

Механічна недостатність

Змішана недостатність

Резорбційна недостатність

1988 / 6854
До лікарні доставлений хворий з отруєнням інсектицидом - ротеноном. Яка ділянка мітохондріального ланцюга переносу електронів блокується цією речовиною?

Сукцинат-коензим Q-редуктаза

АТФ -синтетаза

НАДН-коензим Q-редуктаза

Коензим Q-цитохром С-редуктаза

Цитохром С-оксидаза

1989 / 6854
Під час операції на пахвинному каналі з приводу грижі, хірург пошкодив його вміст. Що саме пошкодив хірург?

Lig. teres uteri

Funiculus spermaticus

Urarchus

Lig. inguinale

1990 / 6854
Чоловік 26-ти років перебуває в торпідній стадії шоку внаслідок автомобільної аварії. В крові: лейк.- 3, 2 • 109 /л. Який головний механізм в розвитку лейкопенії?

Підвищення виділення лейкоцитів з організму

Пригнічення лейкопоезу

Порушення виходу зрілих лейкоцитів з кісткового мозку в кров

Перерозподіл лейкоцитів у судинному руслі

Руйнування лейкоцитів у кровотворних органах

1991 / 6854
Хворий звернувся до лікаря зі скаргами на порушення відчуття рівноваги, що з’явилося після травми. Який нерв пошкоджено?

Трійчастий

Присінково-завитковий

Лицевий

Проміжний

Блукаючий

1992 / 6854
Хлопчик 5-ти місяців госпіталізований з приводу тонічних судом. Хворіє з народження. Об’єктивно: волосся жорстке, нігті витончені та ламкі, шкірні покриви бліді та сухі. В біохімічному аналізі крові: кальцій - 0,5 ммоль/л (норма - 0,75-2,5 ммоль/л), фосфор -1,9 ммоль/л (норма - 0,646-1,292 ммоль/л). З чим пов’язані ці зміни?

Гіперальдостеронізм

Гіперпаратиреоз

Гіпоальдостеронізм

Гіпопаратиреоз

Гіпотиреоз

1993 / 6854
У хворого на алкоголізм почався алкогольний психоз із вираженим психомоторним збудженням. Який препарат з групи нейролептиків слід призначити для швидкої допомоги?

Резерпін

Аміназин

Фторотан

Натрію бромід

Діазепам

1994 / 6854
При пункційній біопсії печінки хворого з клінікою печінково-клітинної недостатності виявлена вакуольна, балонна дистрофія гепатоцитів, некроз окремих клітин, тільця Каунсильмена, інфільтрація портальної та часточкової строми переважно лімфоцитами та макрофагами з незначною кількістю поліморфноядерних лейкоцитів. Який найбільш вірогідний діагноз?

Аутоімунний гепатит

Гострий вірусний гепатит

Хронічний персистуючий гепатит

Хронічний активний гепатит

Алкогольний гепатит

1995 / 6854
З метою прискорення загоєння променевої виразки був призначений вітамінний препарат. Вкажіть його:

Левамiзол

Преднізолон

Ретинолу ацетат

Метилурацил

Ретаболіл

1996 / 6854
Дівчина 15-ти років була доставлена до лікарні з запаленням червоподібного відростку. Аналіз крові показав ознаки анемії. У фекаліях було виявлено яйця гельмінта, які мають лимоноподібну форму (50х30 мкм), з 'пробочками'на полюсах. Який вид гельмінту паразитує у хворої?

Волосоголовець

Карликовий ціп’як

Ехінокок

Анкілостома

Гострик

1997 / 6854
У хворого, який страждає на вугри та запальні зміни шкіри обличчя, при мікроскопії матеріалу з осередків ураження виявлені живі членистоногі розміром 0,2-0,5 мм. Вони мають витягнуту червоподібну форму, чотири пари коротких кінцівок, розміщених у середній частині тіла. Виявлені членистоногі викликають:

Фтіріоз

Коросту

Педикульоз

Демодекоз

Шкірний міаз

1998 / 6854
При огляді дитини 4-х років зі скаргами на загальну слабкість, біль у горлі та утруднене ковтання лікар запідозрив дифтерію та направив матеріал до бактеріологічної лабораторії. На яке диференціально-діагностичне поживне середовище слід засіяти матеріал для виділення збудника дифтерії?

Середовище Сабуро

Кров’яно-телуритовий агар

Середовище Плоскірєва

Середовище Ендо

Середовище Левенштейна-Йєнсена

1999 / 6854
Серед антиатеросклеротичних препаратів, що застосовуються з метою профілактики та лікування атеросклерозу, є левостатин. Він діє шляхом:

Гальмування біосинтезу холестерину

Активації метаболізму холестерину

Пригнічення всмоктування холестерину в кишечнику

Усіма наведеними шляхами

Стимулювання екскреції холестерину з організму

2000 / 6854
У вагітної жінки на передній черевній стінці виявлено пухлиноподібне утворення, яке виникло на місці видаленої два роки тому пухлини. Утворення має щільну консистенцію і розміри 2х1 см, з чіткими межами. При гістологічному дослідженні виявлено, що пухлина побудована з диференційованої сполучної тканини з переважанням колагенових волокон. Про яку пухлину слід думати?

Лейоміома

Фібросаркома

Ліпома

Десмоїд

Гібернома

2001 / 6854
До хірургічного відділення доставлено чоловіка 35-ти років з гнійною раною на шиї попереду трахеї (в ділянці передвісцерального простору). Куди може розповсюдитись інфекція, якщо хворому терміново не зроблять операцію?

В грудну порожнину - заднє середостіння

В грудну порожнину - переднє середостіння

В грудну порожнину - середнє середостіння

В надгрудинний міжапоневротичний простір

В ретровісцеральний простір

2002 / 6854
Внаслідок інсульту (крововилив в головний мозок) у хворого відсутні вольові рухи м’язів голови і шиї. Обстеження головного мозку за допомогою ЯМР показало, що гематома знаходиться в коліні внутрішньої капсули. Який провідний шлях пошкоджено у хворого?

Tr.thalamo-corticalis

Tr.cortico-nuclearis

Tr.cortico-spinalis

Tr.cortico-fronto-pontinus

Tr.cortico-thalamicus

2003 / 6854
Хворому з гіперсекрецією шлункового соку лікар рекомендував виключити з дієти насичені бульйони і овочеві відвари, тому що вони стимулюють шлункову секрецію переважно через активацію:

Вироблення секретину

Механорецепторів ротової порожнини

Механорецепторів шлунка

Смакових рецепторів

Вироблення гастрину

2004 / 6854
У хворого після короткочасного оперативного втручання, проведеного із застосуванням дитиліну, понад 30 хвилин відмічалось пригнічення дихання, не відновився попередній тонус м’язів. Яку допомогу необхідно надати хворому?

Гемосорбція

Перитонеальний діаліз

Форсований діурез

Гемодіаліз

Переливання крові

2005 / 6854
Хворий з діагнозом 'вогнищевий туберкульоз верхньої долі правої легені' в складі комбінованої терапії одержує ізоніазид. Через деякий час пацієнт почав пред’являти скарги на м’язову слабкість, зниження шкірної чутливості, порушення зору, координації рухів. Який вітамінний препарат доцільно використати для усунення даних явищ?

Вітамін A

Вітамін В12

Вітамін C

Вітамін D

Вітамін В6

2006 / 6854
Хворому на гіпертонічну хворобу з супутнім обструктивним бронхітом призначили гіпотензивний засіб. Через деякий час у пацієнта почали з’являтися напади ядухи, розвинулась виражена брадикардія. На ЕКГ відмічались ознаки порушення атріовентрикулярного проведення. При призначенні якого препарату найбільш імовірний розвиток подібних симптомів?

Верапаміл

Резерпін

Клофелін

Корданум

Анаприлін

2007 / 6854
Хворий 60-ти років впродовж 9-ти років хворіє на цукровий діабет, отримує для корекції гіперглікемії інсулін-семіленте. 10 днів тому почав лікування гіпертонічної хвороби анаприліном. Через годину після прийому антигіпертензивного препарату розвинулась гіпоглікемічна кома. Який механізм виникнення гіпоглікемії за умови прийому анаприліну?

Зменшення всмоктування глюкози

Пригнічення глікогенолізу

Збільшення періоду напіввиведення інсуліну-семіленте

Збільшення біодоступності інсуліну-семіленте

Зменшення періоду напіввиведення глюкагону

2008 / 6854
Хворий надійшов до хірургічного відділення з діагнозом: гострий панкреатит. Розпочато консервативне лікування. Призначення якого препарату є патогенетично обґрунтованим?

Хімотрипсин

Фібринолізин

Трипсин

Контрикал

Панкреатин

2009 / 6854
До приймального відділення доставлений хворий зі скаргами на сухість в роті, світлобоязнь та порушення зору. Об’єктивно: шкіра гіперемована, суха, зіниці розширені, тахікардія. При подальшому обстеженні був встановлений діагноз: отруєння алкалоїдами беладонни. Який лікарський засіб доцільно застосувати?

Армін

Пілокарпін

Дипіроксим

Прозерин

Ацеклідин

2010 / 6854
При обстеженні 2-місячного хлопчика педіатр звернув увагу, що плач дитини схожий на нявкання кішки, відзначаються мікроцефалія і вада серця. За допомогою цитогенетичного методу був встановлений каріотип - 46 XY, 5р- . На якій стадії мітозу досліджували каріотип хворого?

Метафаза

Прометафаза

Анафаза

Телофаза

Профаза

2011 / 6854
Чоловік, що страждає на спадкову хворобу, одружився із здоровою жінкою. У них було 5 дітей, три дівчинки і два хлопчика. Усі дівчатка успадкували хворобу батька. Який тип спадкування цього захворювання?

Домінантний, зчеплений з X -хромосомою

Рецесивний, зчеплений з X -хромосомою

Аутосомно-рецесивний

Зчеплений з Y-хромосомою

Аутосомно-домінантний

2012 / 6854
Похідні птерину (аміноптерин і метотрексат) - є конкурентними інгібіторами дигідрофолатредуктази, внаслідок чого вони пригнічують регенерацію тетрагідрофолієвої кислоти з дигідрофолату. Ці лікарські засоби призводять до гальмування міжмолекулярного транспорту одновуглецевих груп. Біосинтез якого полімеру при цьому пригнічується?

ДНК

Глікозаміноглікани

Гомополісахариди

Білок

Гангліозиди

2013 / 6854
До лікарні надійшов хворий із скаргами на головний біль, біль у м’язах під час руху, слабкість, температуру, набряк повік і обличчя. Лікар пов’язує цей стан із вживанням свинини, купленої у приватних осіб. Який попередній діагноз може поставити лікар?

Трихінельоз

Опісторхоз

Теніарінхоз

Фасціольоз

Теніоз

2014 / 6854
В шкірі виявлена щільна, рухома, чітко відмежована від оточуючих тканин пухлина. На розрізі вона білого кольору, представлена волокнистою тканиною. Мікроскопічно: хаотично переплетені колагенові волокна, клітин мало. Що це за пухлина?

Дерматофіброма

Десмоїд

Міома

Фіброма

Гістіоцитома

2015 / 6854
На розтині тіла померлої виявлено морфологічні зміни: стеноз лівого атріовентрикулярного отвору, недостатність мітрального клапана. Гістологічно в міокарді - вогнищевий кардіосклероз, наявність квітучих гранульом Ашофф-Талалаєва. Який з перерахованих нижче діагнозів найбільш імовірний?

Дерматоміозит

Склеродермія

Вузликовий періартеріїт

Системний червоний вовчак

Ревматизм

2016 / 6854
У постраждалого виявлено рану верхньої частини передньої ділянки плеча. При обстеженні встановлена втрата активного згинання в ліктьовому суглобі і чутливості шкіри передньобічної поверхні передпліччя. Порушення функції якого нерва має місце?

Серединний

Променевий

Ліктьовий

М’язово-шкірний

Пахвовий

2017 / 6854
При мікроскопічному дослідженні шийного лімфатичного вузла виявлено скупчення епітеліоїдних клітин, лімфоцитів і гігантських клітин Пирогова-Лангханса. В центрі - казеозний некроз. Вкажіть найбільш імовірну патологію:

Риносклерома

Саркоїдоз

Сифіліс

Сап

Туберкульоз

2018 / 6854
При гістологічному дослідженні новоутворення шкіри виявлено: паренхіма сформована з покривного епітелію із збільшеним числом шарів. Строма разом з розростаннями епітелію формує сосочки. Вкажіть вид атипізму:

Функціональний

Клітинний

Метаболічний

Гістохімічний

Тканинний

2019 / 6854
Після перенесеного сепсису у хворої 27-ми років з’явився бронзовий колір шкіри, характерний для аддісонової хвороби. Механізм гіперпігментації полягає в підвищенні секреції такого гормону:

Соматотропний

Гонадотропний

B-ліпотропний

Меланоцитстимулюючий

Тиреотропний

2020 / 6854
У пацієнта, який півтора місяця тому переніс інфаркт міокарда, діагностовано синдром Дреслера з характерною тріадою: перикардит, плеврит, пневмонія. Який головний механізм цього ускладнення?

Сенсибілізація організму антигенами міокарда

Активація сапрофітної мікрофлори

Викидання у кров міокардіальних ферментів

Зниження резистентності до інфекційних агентів

Інтоксикація організму продуктами некрозу

2021 / 6854
Жінка після перенесеного нервового потрясіння погано спить. Якому снодійному засобу слід віддати перевагу для лікування безсоння?

Барбітал

Барбаміл

Фенобарбітал

Етамінал-натрій

Нітразепам

2022 / 6854
У хворої на дифтерію дитини через 10 днів після введення антитоксичної протидифтерійної сироватки з’явилася висипка на шкірі, яка супроводжувалася сильним свербежем, підвищенням температура тіла до 38oC, появою болю у суглобах. Яку причину цих явищ можна припустити?

Гіперчутливість уповільненого типу

Атопія

Контактна алергія

Анафілактична реакція

Сироваткова хвороба

2023 / 6854
У хворого з розладом мозкового кровообігу порушений акт ковтання, він може поперхнутися при прийомі рідкої їжі. Який відділ мозку уражений?

Середній мозок

Довгастий мозок

Шийний відділ спинного мозку

Мозочок

Проміжний мозок

2024 / 6854
Молодий чоловік 19-ти років страждав з раннього дитинства на бронхоектатичну хворобу, помер від ниркової недостатності. На розтині окрім множинних бронхоектатичних каверн, заповнених гнійним ексудатом, виявлено збільшені в розмірах нирки щільної консистенції, кірковий шар потовщений, білого кольору, щільний. Піраміди нирки анемічні, чіткі. Назвіть процес, який розвинувся в нирках:

Вторинний амілоїдоз

Хронічний пієлонефрит

Вроджений кістоз нирок

Вторинний нефросклероз

Гломерулонефрит

2025 / 6854
Дитина 3-х років з множинними порушеннями розвитку кісток лицевого відділу черепа померла. Причина смерті - сепсис, який розвинувся на фоні бронхопневмонії. В крові вміст імуноглобулінів в межах фізіологічної норми. На розтині встановлена відсутність тимусу. Назвіть головну причину хвороби дитини:

Синдром комбінованого імунодефіциту

Вторинний імунодефіцитний синдром

Гострий лімфолейкоз

Синдром недостатності клітинного імунітету

Синдром хронічної інтоксикації

2026 / 6854
До лікарні доставлено пораненого вогнепальною зброєю з сильною кровотечею. При огляді хірургом установлено, що кульовий канал пройшов через передню стінку живота, склепіння шлунка i вийшов на рівні ІХ ребра по лівій середній пахвовій лінії. Який орган постраждав разом з пораненням шлунка?

Підшлункова залоза

Селезінка

Поперечна ободова кишка

Ліва частка печінки

Ліва нирка

2027 / 6854
У хворого із захворюванням печінки виявлено зниження вмісту протромбіну в крові. Це призведе, перш за все, до порушення:

Першої фази коагуляційного гемостазу

Фібринолізу

Судинно-тромбоцитарного гемостазу

Антикоагулянтних властивостей крові

Другої фази коагуляційного гемостазу

2028 / 6854
в крові: ер.- 3,0 • 1012 28. /л; Hb- 90г/л; ретикул.- 0,5%. В мазку: пойкілоцити, гіпохромні еритроцити. Залізо сироватки крові - 80 мкмоль/л. Для якої патології це характерно?

Залізодефіцитна анемія

Серпоподібноклітинна анемія

Залізорефрактерна анемія

Б12-дефіцитна анемія

Хвороба Мінковського-Шоффара

2029 / 6854
У фізично здорових молодих курсантів після важкого фізичного навантаження при одноденному пішому переході на 50 км в сечі виявлено білок, рівень якого в середньому не перевищував 1 г/л. Який різновид протеїнурії мав місце?

Аліментарна

Маршова

Несправжня

Органічна

Дегідратаційна

2030 / 6854
Яким буде скорочення м’язів верхньої кінцівки при намаганні підняти непосильний вантаж?

Одиночне

Ізотонічне

Фазичне

Ізометричне

Ауксотонічне

2031 / 6854
У хворих на поворотний тиф виникає лихоманка, яка характеризується кількаденними періодами високої гарячки, що чергується з періодами нормальної температури. Така температурна крива називається:

Febris atypica

Febris continua

Febris recurrens

Febris hectica

Febris intermittens

2032 / 6854
У хворого з алкогольним цирозом печінки скарги на загальну слабкість, задишку. Встановлено зниження артеріального тиску, асцит, розширення поверхневих вен передньої стінки живота, спленомегалію. Яке порушення гемодинаміки спостерігається у хворого?

Недостатність лівого шлуночка серця

Синдром портальної гіпертензії

Недостатність правого шлуночка серця

Тотальна серцева недостатність

Колапс

2033 / 6854
Під час обертання на каруселі у жінки 25-ти років з’явилися нудота, блювання, посилення потовиділення. Активація яких рецепторів зумовила рефлекторний розвиток цих симптомів?

Пропріорецептори скелетних м’язів

Кортієвого органу

Вестибулярні півколові канали

Зорові

Отолітові вестибуляри

2034 / 6854
У студента 18-ти років під час фізичного навантаження реографічно зареєстровано перерозподіл кровотоку органів. У яких судинах кровотік підвищився найбільшою мірою?

Головний мозок

Скелетні м’язи

Шлунково-кишковий тракт

Нирки

Печінка

2035 / 6854
Під час ревізії порожнини очеревини з приводу перитоніту виявлений обмежений гнійник у кореня брижі сигмоподібної кишки. У якому утворенні очеревини знаходиться гнійник?

Правий бічний канал

Лівий бічний канал

Лівий брижовий синус

Міжсигмоподібна ямка

Правий брижовий синус

2036 / 6854
Під час розтину трупа шахтаря, що проробив у шахті більше 10-ти років, у легені виявлені тяжі білуватої волокнистої тканини і вузлики 0,2-0,3 см у діаметрі. При гістологічному дослідженні у вузликах невелика кількість коричневого пилу, концентричні розростання сполучної тканини, бідної на клітини, з вираженим гіалінозом. Про який пневмоконіоз слід думати у даному випадку?

Бериліоз

Сидероз

Талькоз

Силікоз

Асбестоз

2037 / 6854
При ендоскопічному дослідженні сечового міхура проведена біопсія пухлини, що складається з тонких, розгалужених сосочків, вкритих декількома рядами клітин перехідного епітелію. Назвіть пухлину:

Фіброаденома

Перехідноклітинна карцинома

Плоскоклітинна карцинома

Папілома

Базаліома

2038 / 6854
Хворий 23-х років надійшов до лікарні із черепно-мозковою травмою у важкому стані. Дихання характеризується судомним тривалим вдихом, який переривається коротким видихом. Для якого типу дихання це характерно?

Апнейстичне

Біота

Чейн-Стокса

Куссмауля

Гаспінг-дихання

2039 / 6854
Внаслідок дії на організм електричного струму міської електромережі впродовж 0,1 сек у напрямку 'права рука-голова'у постраждалого спостерігалась зупинка дихання. Вкажіть найбільш імовірний механізм цього ускладнення:

Тотальний параліч дихального центру

Параліч дихальних м’язів

Емоційний стрес

Рефлекторна зупинка дихання (больовий шок)

Параліч центрів вдиху

2040 / 6854
В експерименті досліджували поріг сили подразника клітин різних тканин. Де він виявився найменшим?

Типові кардіоміоцити

Мотонейрони спинного мозку

Залозисті клітини

Міоцити гладенького м’яза

Міоцити скелетного м’яза

2041 / 6854
На ізольованому серці вивчалась швидкість проведення збудження у різних його ділянках. Де була виявлена найменша швидкість?

Атріовентрикулярний вузол

Пучок Гіса

Міокард шлуночків

Міокард передсердь

Волокна Пуркін’є

2042 / 6854
Під час операції в печінці хворого виявлені дрібні міхурці малих розмірів з незначною кількістю рідини, які щільно прилягають один до одного. Який гельмінтоз виявився у хворого?

Опісторхоз

Дікроцеліоз

Альвеококоз

Клонорхоз

Фасціольоз

2043 / 6854
У хворого 70-ти років атеросклероз ускладнився тромбозом судин нижніх кінцівок, виникла гангрена пальців лівої стопи. Початок тромбоутворення, найбільш імовірно, пов’язаний з:

Перетворенням протромбіну в тром-бін

Адгезією тромбоцитів

Перетворенням фібриногену в фібрин

Активацією протромбінази

Зниженням синтезу гепарину

2044 / 6854
Жінку госпіталізовано в клініку з симптомами гострого живота. При обстеженні виникла підозра на позаматкову вагітність. Яке з анатомічних утворень таза необхідно пропунктувати для підтвердження діагнозу?

Excavatio rectouterina

Excavatio vesicouterina

Fossa ischiorectalis

Excavatio rectovesicalis

Processus vaginalis peritonei

2045 / 6854
Дитина 10-ти років страждає на стафілококовий дерматит. Лікування бензилпеніциліном не дало результатів. Призначення комбінованого препарату пеніциліну з клавулановою кислотою дало швидке одужання. Яка причина позитивної дії цього препарату?

Гальмування транспептидази

Інактивація бета-лактамази

Гальмування аденозиндезамінази

Активація фосфодіестерази

Блокада транслокази

2046 / 6854
У крові хворого виявлено підвищення активності ЛДГ 4,5, АлАТ, карбамоїлорнітинтрансферази. В якому органі можна передбачити розвиток патологічного процесу?

Сполучна тканина

Нирки

Скелетні м’язи

Серцевий м’яз (можливий інфаркт міокарда)

Печінка (можливий гепатит)

2047 / 6854
Апендикс, надісланий до патоморфологічного відділення після апендектомії, потовщений і збільшений у розмірах, серозна оболонка тьмяна, судини повнокровні, з просвіту відростка на розрізі виділяється рідина жовто-зеленого кольору. При якій формі апендициту виникають такі зміни?

Поверхневий катаральний

Апостематозний

Флегмонозний

Гангренозний

Простий катаральний

2048 / 6854
Внаслідок травми у чоловіка 40-ка років зруйновані задні корінці спинного мозку. Які розлади будуть спостерігатися в зоні іннервації цих корінців?

Порушення функції гладеньких м’язів

Порушення функції посмугованих скелетних м’язів

Втрата температурної та вібраційної чутливості

Втрата больової чутливості

Втрата всіх видів чутливості

2049 / 6854
До клініки госпіталізована пацієнтка із скаргами на кровохаркання, пітливість. Рентгенологічно виявлено вогнище туберкульозу у верхній частці правої легені. Показана операція. Яку кількість сегментів можна видалити в складі верхньої частки правої легені?

1

4

3

5

2

2050 / 6854
У товстій кишці під час колоноскопії виявлено дефект слизової оболонки діаметром 3,5 см з нерівним горбистим дном, нерівними краями, що підвищуються над дном на 1,7 см; межа цього підвищення нечітка. Тканина дна і країв дефекту щільна, білувата, шари стінки кишки у цій ділянці не розрізняються. Встановіть макроскопічну форму пухлини:

Виразка

Інфільтрат

Кіста

Вузол

Інфільтративно-виразкова форма

2051 / 6854
У препараті в одній з судин мікроциркуляторного русла середня оболонка утворена 1-2 шарами гладеньких міоцитів, які розташовані поодинці і мають спіралеподібний напрямок. Зовнішня оболонка представлена тонким шаром пухкої волокнистої сполучної тканини. Вкажіть вид судини:

Венула

Артеріоловенулярний анастомоз

Капіляр

Артеріола

Посткапіляр

2052 / 6854
У хворої людини посилений рух води з кровоносних капілярів до тканин, що викликало їх позаклітинний набряк (збільшені розміри м’яких тканин кінцівок, печінки тощо). Зменшення якого параметру гомеостазу є найбільш імовірною причиною розвитку набряку?

Осмотичний тиск плазми крові

В’язкість крові

Онкотичний тиск плазми крові

рН крові

Гематокрит

2053 / 6854
У постраждалого в дорожній пригоді лікар виявив перелом лівої ключиці та порушення кровообігу в кінцівці (немає пульсації променевої артерії). Стиснення якої судини є причиною порушення кровообігу в кінцівці?

Пахвова артерія

Хребтова артерія

Пахвова вена

Підключична вена

Підключична артерія

2054 / 6854
Хворого доставлено у медичний заклад в коматозному стані. Зі слів супроводжуючих вдалося з’ясувати, що він знепритомнів під час тренування на завершальному етапі марафонської дистанції. Який вид коми найімовірніше можна запідозрити у даного пацієнта?

Печінкова

Гіперглікемічна

Ацидотична

Гіпотиреоїдна

Гіпоглікемічна

2055 / 6854
У хворого коса пахвинна грижа. Яке анатомічне утворення стало слабким місцем передньої черевної стінки?

Стегнова ямка

Надміхурова ямка

Латеральна пахвинна ямка

Пахвинний трикутник

Медіальна пахвинна ямка

2056 / 6854
У хворих на тиреотоксикоз спостерігаються гіпертермія, булімія, зменшення маси тіла, що пов’язане з порушенням:

Спряження окислення і фосфорилю-вання

Синтезу жирів

Розпаду АТФ

в-окиснення жирних кислот

Циклу лимонної кислоти

2057 / 6854
Людині внутрішньовенно ввели 0,5 л ізотонічного розчину лікарської речовини. Які з рецепторів насамперед прореагують на зміни водно-сольового балансу організму?

Волюморецептори порожнистих вен і передсердь

Осморецептори печінки

Натрієві рецептори гіпоталамуса

Осморецептори гіпоталамусу

Барорецептори дуги аорти

2058 / 6854
Пацієнту 33 роки. Хворіє 10 років. Періодично звертається до лікаря зі скаргами на гострий біль у животі, судоми, порушення зору. У його родичів спостерігаються подібні симптоми. Сеча червоного кольору. Госпіталізований з діагнозом - гостра переміжна порфирія. Причиною захворювання може бути порушення біосинтезу такої речовини:

Простагландини

Колаген

Жовчні кислоти

Гем

Інсулін

2059 / 6854
У хворої 45-ти років при електрокардіографічному обстеженні виявлено такі зміни: інтервал P — Q подовжений, при цьому випадає кожен другий або третій комплекс QRST. Яке саме порушення провідності серця спостерігається?

Атріовентрикулярна блокада повна

Атріовентрикулярна блокада І ступеня

Атріовентрикулярна блокада ІІІ ступеня

Синоаурікулярна блокада

Внутрішлуночкова блокада

2060 / 6854
У вагітної жінки визначили групову належність крові. Реакцію аглютинації еритроцитів викликали стандартні сироватки I, III груп, та не викликала -сироватка II групи. Якою є група досліджуваної крові за системою АВ0?

B(III), α

O(I), α, β

АВ (IV)

A(II), β

2061 / 6854
При розтині тіла померлого чоловіка 65-ти років, який страждав на захворювання легень, патологічний процес переважно був локалізований у бронхах, де при гістологічному дослідженні були чітко видні залози, хрящові острівці та багаторядний циліндричний миготливий епітелій. В яких бронхах відбулися зміни?

Малі бронхи

Великі бронхи

Термінальні бронхіоли

Середні бронхи

Головні бронхи

2062 / 6854
У юнака 18-ти років діагностована м’язова дистрофія. Підвищення в сироватці крові вмісту якої речовини найбільш імовірне при цій патології?

Міоглобін

Міозин

Аланін

Креатин

Лактат

2063 / 6854
У хворого в крові збільшена концентрація пірувату. Значна його кількість екскретується з сечею. Дефіцит якого вітаміну має місце у хворого?

B6

B2

E

B1

B3

2064 / 6854
У 70-ті роки вчені встановили, що причиною важкої жовтяниці новонароджених є порушення зв’язування білірубіну в гепатоцитах. Яка речовина використовується для утворення кон’югату?

Сірчана кислота

Молочна кислота

Сечова кислота

Глюкуронова кислота

Піровиноградна кислота

2065 / 6854
Хворому, що страждає на хронічний гепатит, для оцінки знешкоджуючої функції печінки було проведене навантаження бензоатом натрію. За виділенням якої речовини з сечею судять про знешкоджуючу функцію печінки?

Лимонна кислота

Валеріанова кислота

Фенілоцтова кислота

Щавелєва кислота

Гіпурова кислота

2066 / 6854
У хворого, який проходить курс лікувального голодування, нормальний рівень глюкози у крові підтримується головним чином за рахунок глюконеогенезу. З якої амінокислоти при цьому у печінці людини найбільш активно синтезується глюкоза?

Аланін

Лейцин

Лізин

Валін

Глутамінова кислота

2067 / 6854
В сечі новонародженого визначається цитрулін та високий рівень аміаку. Вкажіть, утворення якої речовини, наймовірніше, порушене у цього малюка:

Креатин

Сечова кислота

Аміак

Креатинін

Сечовина

2068 / 6854
У здорової дорослої людини проводять зондування порожнин серця і великих судин. Де знаходиться зонд, якщо впродовж серцевого циклу зареєстровані зміни тиску від 0 до 120 мм рт.ст.?

Легенева артерія

Аорта

Передсердя

Лівий шлуночок

Правий шлуночок

2069 / 6854
При дослідженні гнійних виділень з шийки матки бактеріоскопічно виявлено присутність грамнегативних бобоподібних диплококів, які знаходилися як в середині, так і поза лейкоцитами. Назвіть збудника гнійного запалення шийки матки:

Chlamidia trachomatis

Neisseria gonorroeae

Calymmatobacterium granulomatis

Haemophilus vaginalis

Trichomonas vaginalis

2070 / 6854
Дитина 1-го року відстає в розумовому розвитку від своїх однолітків. Ранком відзначаються блювання, судоми, непритомність. У крові - гіпоглікемія натще. З дефектом якого ферменту це пов’язане?

Сахараза

Аргіназа

Лактаза

Фосфорилаза

Глікогенсинтетаза

2071 / 6854
Внаслідок черепно-мозкової травми у хворого розвинулись наступні симптоми: інтенційний тремор, дисметрія, адіадохокінез, дизартрія. Яка структура головного мозку ушкоджена?

Чорна речовина

Рухова кора

Бліда куля

Стріатум

Мозочок

2072 / 6854
У гістологічному препараті визначаються рецепторна зона сенсоепітеліального органа чуттів. Клітини даної зони лежать на базальній мембрані і включають наступні види: зовнішні та внутрішні сенсорні, зовнішні та внутрішні фалангові, стовбурові, зовнішні межові і зовнішні підтримуючі. Вкажіть, якому органу чуттів належить зазначена рецепторна зона:

Слуху

Зору

Рівноваги

Смаку

Нюху

2073 / 6854
Хвора скаржиться на набряк ніг, посиніння шкіри, невеличкі виразки збоку латерального виростку. При обстеженні відмічено: припухлість, збільшення розмірів вен, утворення вузлів. З боку якої вени відмічається патологія?

V saphena parva

Vprofunda femoris

V iliaca externa

V saphena magna

V femoralis

2074 / 6854
Хворому встановлений діагноз ураження голівки стегна ішемічного походження. Яка артерія ушкоджена?

Arteria umbilicalis

Arteria illiaca externa

Arteria femoralis

Ramus acetabularum A.obturatoriae

Arteria profunda femoris

2075 / 6854
Чоловік 44-х років з інфарктом міокарда, помер від лівошлуночкової недостатності. На аутопсії: набряк легень, дрібнокраплинні крововиливи у серозних та слизових оболонках. Мікроскопічно: дистрофічні та некробіотичні зміни епітелію проксимальних канальців нирок, у печінці - центролобулярні крововиливи та осередки некрозу. Який з видів порушення кровообігу найбільш імовірний?

Хронічне загальне венозне повнокров’я

Хронічне недокрів’я

Гостре недокрів’я

Артеріальна гіперемія

Гостре загальне венозне повнокров’я

2076 / 6854
У хворого гнійне запалення клиноподібної пазухи. В яку частину носової порожнини витікає гній?

Meatus nasi medius

Meatus nasi communis

Meatus nasi superior

Meatus nasi inferior

2077 / 6854
Для підвищення результатів спортсмену рекомендували застосовувати препарат, який містить у собі карнітин. Який процес в найбільшому ступені активується карнітином?

Транспорт жирних кислот до мітохондрій

Синтез кетонових тіл

Синтез стероїдних гормонів

Синтез ліпідів

Тканинне дихання

2078 / 6854
На ЕКГ пацієнта мають місце такі зміни: зубець P - нормальний, інтервал P — Q - вкорочений, шлуночковий комплекс QRST - розширений, зубець R -двогорбий або двофазний. Яка із форм аритмії має місце у даного пацієнта?

Миготіння шлуночків

Миготлива аритмія

Синдром Фредеріка (тріпотіння передсердь)

Атріовентрикулярна блокада

Синдром WPW (Вольфа-Паркінсона-Уайта)

2079 / 6854
У хворого з жовтяницею встановлено: підвищення у плазмі крові вмісту загального білірубіну за рахунок непрямого (вільного), в калі та сечі - високий вміст стеркобіліну, рівень прямого (зв’язаного) білірубіну в плазмі крові в межах норми. Про який вид жовтяниці можна думати?

Жовтяниця немовлят

Механічна

Гемолітична

Паренхіматозна (печінкова)

Хвороба Жильбера

2080 / 6854
Хвора 36-ти років страждає на колагеноз. Збільшення вмісту якого метаболіту найбільш імовірно буде встановлено у сечі?

Індикан

Сечовина

Оксипролін

Уробіліноген

Креатинін

2081 / 6854
У хворого на гіпертонічну хворобу виявлено високий рівень реніну в крові. Якому з гіпотензивних засобів слід віддати перевагу в цьому випадку?

Дихлотіазид

Ніфедипін

Празозин

Анаприлін

Лізиноприл

2082 / 6854
Причиною захворювання на пелагру може бути переважне харчування кукурудзою і зниження у раціоні продуктів тваринного походження. Відсутність у раціоні якої амінокислоти призводить до даної патології?

Гістидин

Триптофан

Метіонін

Ізолейцин

Фенілаланін

2083 / 6854
У людини збільшена вентиляція легень внаслідок фізичного навантаження. Який з наведених показників зовнішнього дихання у неї значно більший, ніж у стані спокою?

Дихальний об’єм

Резервний об’єм вдиху

Загальна ємність легень

Резервний об’єм видиху

Життєва ємність легень

2084 / 6854
Людина зробила максимально глибокий видих. Як називається об’єм повітря, що знаходиться в її легенях після цього?

Альвеолярний об’єм

Функціональна залишкова ємність легень

Ємність вдиху

Залишковий об’єм

Резервний об’єм видиху

2085 / 6854
Внаслідок активації іонних каналів зовнішньої мембрани збудливої клітини значно збільшився її потенціал спокою. Які канали були активовані?

Швидкі кальцієві

Калієві

Натрієві та кальцієві

Натрієві

Повільні кальцієві

2086 / 6854
Робітник тваринницької ферми гостро захворів і при наростаючих явищах інтоксикації помер. При розтині тіла встановлено: селезінка збільшена, в’яла, на розрізі темновишневого кольору, зішкріб пульпи рясний. М’які мозкові оболонки на склепінні та основі мозку набряклі, просякнуті кров’ю, мають темно-червоний колір ('шапочка кардинала'). Мікроскопічно: серозно-геморагічне запалення оболонок і тканин головного мозку з руйнуванням стінок дрібних судин. Який найбільш імовірний діагноз?

Бруцельоз

Сибірка

Чума

Туляремія

Холера

2087 / 6854
При медичному обстеженні у військкоматі у юнака 18-ти років виявлено опускання головки надп’ясткової кістки, що призвело до плоскостопості. Зі слабкістю якої зв’язки стопи це пов’язано?

Надп’ятково-човноподібна

Підошвова клино-кубоподібна

Роздвоєна

Підошвова п’ятково-човноподібна

Міжкісткова клино-кубоподібна

2088 / 6854
Після видалення у пацієнта 2/3 шлунка у крові зменшився вміст гемоглобіну, кількість еритроцитів, збільшилися розміри цих клітин крові. Дефіцит якого вітаміну призводить до таких змін у крові?

P

В12

В6

C

PP

2089 / 6854
У хворого 40-ка років при прогресуванні стафілококового гнійного періодонтиту виникло гнійне запалення кістково-мозкових просторів альвеолярного відростка, а потім тіла нижньої щелепи. Мікроскопічно кісткові балки витончені, вогнища некрозу, кісткові секвестри, оточені сполучнотканинною капсулою. Який найбільш імовірний діагноз?

Хронічний остеомієліт

Гострий остеомієліт

Гнійний періостит

Хронічний фіброзний періостит

Пародонтома

2090 / 6854
При розтині тіла дитини, померлої від серцевої недостатності, виявлено: розширення порожнин шлуночків серця. Мікроскопічно в стромі міокарда повнокров’я, набряк, розповсюджені інфільтрати з гістіоцитів, лімфоцитів, нейтрофілів, еозинофілів. Який найбільш імовірний діагноз?

Проміжний продуктивний мюкардит

Альтернативний мюкардит

Вузликовий продуктивний міокардит

Осередковий проміжний ексудативний міокардит

Дифузнопроміжний ексудативний міокардит

2091 / 6854
Хвора 75-ти років доставлена до офтальмологічного відділення лікарні зі скаргами на погіршення зору. При об’єктивному дослідженні встановлена наявність пухлини мозку, що розташована в ділянці лівого зорового тракту. При цьому у хворої спостерігається випадіння поля зору в:

Правих і лівих половинах сітківки правого ока

Правих і лівих половинах сітківки лівого ока

Лівих половинах сітківки обох очей

Правих половинах сітківки обох очей

Правих і лівих половинах сітківок обох очей

2092 / 6854
У хворого на ішемічну хворобу серця відзначається гіпертрофія міокарда, тахікардія, зниження ХОК. Який з механізмів є провідним в ушкодженні кардіоміоцитів у даному випадку?

Збільшення числа а та β-адренорецепторів

Пошкодження специфічних мембранних насосів

Дегідратація кардіоміоцитів

Втрата Ca2+ кардіоміоцитами

Втрата Mg2+ кардіоміоцитами

2093 / 6854
Тварині внутрішньовенно ввели концентрований розчин хлориду натрію, що зумовило зниження реабсорбції іонів натрію у канальцях нирок. Внаслідок яких змін секреції гормонів це відбувається?

Збільшення вазопресину

Збільшення альдостерону

Зменшення натрійуретичного фактора

Зменшення альдостерону

Зменшення вазопресину

2094 / 6854
У хворої 49-ти років відзначається обмеження довільних рухів у лівих кінцівках. Тонус м’язів у лівих руці та нозі підвищений за спастичним типом, посилені місцеві сухожилкові рефлекси, виявляються патологічні рефлекси. Який найбільш імовірний механізм призвів до розвитку м’язової гіпертонії та гіперрефлексії?

Зниження гальмівних низхідних впливів

Гальмування мотонейронів кори головного мозку

Активація синаптичної передачі імпульсів

Активація збуджуючих впливів з вогнища інсульту

Активація мотонейронів внаслідок інсульту

2095 / 6854
При виконуванні вправ на колоді гімнастка втратила рівновагу і впала. Із збудження, перш за все, яких рецепторів розпочнуться рефлекси, що забезпечать відновлення порушеної пози?

Ампулярні вестибулорецептори

Рецептори завитки

Пропріорецептори

Отолітові вестибулорецептори

Вестибулорецептори

2096 / 6854
В анотації до препарату вказано, що він містить антигени збудника черевного тифу, адсорбовані на стабілізованих еритроцитах барана. З якою метою використовують цей препарат?

Для виявлення антитіл в реакції зв’язування комплементу

Для виявлення антитіл в реакції Відаля

Для виявлення антитіл в реакції затримки гемаглютинації

Для виявлення антитіл в реакції непрямої гемаглютинації

Для серологічної ідентифікації збудника черевного тифу

2097 / 6854
На гістологічному препараті видно позазародковий орган, який являє собою міхурець, зв’язаний з кишковою трубкою. Стінка його зсередини вистелена епітелієм, зовні утворена сполучною тканиною. На ранніх етапах ембріогенезу він виконує функцію кровотворного органа. Назвіть цей орган:

Пуповина

Плацента

Жовтковий мішок

Алантоїс

Амніон

2098 / 6854
Хворий був доставлений до лікарні в коматозному стані. В анамнезі - цукровий діабет. Об’єктивно: дихання Кусмауля, зниження артеріального тиску, у видихуваному повітрі запах ацетону. Після проведеної невідкладної терапії стан покращився. Який препарат було введено хворому?

Букаркам

Адреналін

Інсулін

Ізадрин

Глібенкламід

2099 / 6854
У приймально-дiагностичне відділення доставили жінку 38-ми років з шлунковою кровотечею. Які зміни найбільш імовірні з боку крові через добу?

Еритроцитоз

Збільшення гематокритного числа

Зменшення гематокритного числа

Лейкопенія

Лейкоцитоз

2100 / 6854
При розтині трупа чоловіка 50-ти років виявлено наступні зміни: права легеня у всіх відділах помірно щільна, на розрізі тканина безповітряна, дрібнозерниста, сухувата. Вісцеральна плевра з нашаруванням фібрину сіро-коричневого кольору. Який найбільш імовірний діагноз?

Бронхопневмонія

Крупозна пневмонія

Туберкульоз

Пневмофіброз

Інтерстиціальна пневмонія

2101 / 6854
У людини звужені зіниці. Чим це зумовлено?

Зростання тонусу симпатичних центрів

Дія норадреналіну

Дія адреналіну

Зростання тонусу парасимпатичних центрів

Збільшення активності симпато-адреналової системи

2102 / 6854
При розтині тіла жінки 50-ти років, померлої від серцевої недостатності, знайдено дифузне збільшення щитоподібної залози. Мікроскопічно: перетворення призматичного епітелію фолікулів у циліндричний, проліферація епітелію з утворенням сосочків, розрідження колоїду та лімфоплазмоцитарна інфільтрація строми; в серці - гіпертрофія лівого шлуночка, серозний набряк та лімфоїдна інфільтрація строми, в печінці - серозний набряк. Який з перелічених діагнозів найбільш імовірний?

Зоб Хашімото

Гострий тиреоїдит

Ендемічний зоб

Дифузний токсичний зоб

Зоб Риделя

2103 / 6854
Пацієнт через 15 діб після повернення з багатомісячного плавання в районах Середземномор’я та Західної Африки відчув слабкість, головний біль, періодичні підвищення температури. Лікар запідозрив у хворого малярію. Який із перерахованих методів є найбільш адекватним в діагностиці даного захворювання?

Серологічний

Алергічний

Мікроскопічний

Біологічний

Мікробіологічний

2104 / 6854
У вагітної жінки взяли кров для підтвердження клінічного діагнозу 'токсоплазмоз'. Яка із перерахованих серологічних реакцій має діагностичне значення?

Реакція нейтралізації

Реакція зв’язування комплементу

Реакція гемадсорбції

Реакція аглютинації

Реакція гальмування гемаглютинації

2105 / 6854
У здорових батьків, спадковість яких не обтяжена, народилась дитина з чисельними вадами розвитку. Цитогенетичний аналіз виявив в соматичних клітинах дитини трисомію за 13-ю хромосомою (синдром Патау). З яким явищем пов’язане народження такої дитини?

Рецесивна мутація

Порушення гаметогенезу

Хромосомна мутація

Соматична мутація

Домінантна мутація

2106 / 6854
У хворого, який скаржиться на поліурію і полідипсію, знайдено цукор в сечі. Вміст цукру в плазмі крові у нормі. З чим пов’язаний механізм глюкозурії у хворого?

Iнсулiнорезистентнiсть рецепторів клітин

Порушення реабсорбції глюкози в канальцях нефрону

Гіперпродукція глюкокортикоїдів наднирниками

Порушення фільтрації глюкози в клубочковому відділі нефрону

Недостатня продукція інсуліну підшлунковою залозою

2107 / 6854
Під час об’єктивного обстеження хворого з діагнозом: атеросклеротичний міокардіосклероз, лікар встановив феномен дефіциту пульсу. При якій формі порушення серцевого ритму спостерігається такий феномен?

Миготлива аритмія

Брадикардія

Синусова екстрасистолія

Ідіовентрикулярний ритм

Передсердно-шлуночковий ритм

2108 / 6854
На препараті представлено орган, покритий сполучнотканинною капсулою, від якої відходять трабекули. В органі можна розрізнити кіркову речовину, де містяться лімфатичні вузлики та мозкову речовину, представлену тяжами лімфоїдних клітин. Який орган представлений на препараті?

Мигдалики

Червоний кістковий мозок

Тимус

Лімфатичний вузол

Селезінка

2109 / 6854
До лікаря звернулася жінка 25-ти років зі скаргами на дисменорею та безпліддя. При обстеженні виявлено: зріст жінки 145 см, недорозвинені вторинні статеві ознаки, на шиї крилоподібні складки. При цитологічному дослідженні в соматичних клітинах не виявлено тілець Барра. Який діагноз встановив лікар?

Синдром Клайнфельтера

Синдром Шерешевського-Тернера

Синдром трисомії Х

Синдром Морріса

2110 / 6854
У хворої внаслідок запалення порушена ендокринна функція фолікулярних клітин фолікулів яєчника. Синтез яких гормонів буде пригнічений?

Фолістатин

Фолікулостимулюючий гормон

Естрогени

Прогестерон

Лютропін

2111 / 6854
Хворому на гострий інфаркт міокарда у комплексній терапії було призначено гепарин. Через деякий час після введення даного препарату з’явилась гематурія. Який антагоніст гепарину необхідно ввести хворому для усунення даного ускладнення?

Вікасол

Неодикумарин

Фібриноген

Протаміну сульфат

Амінокапронова кислота

2112 / 6854
У хворого на хронічну серцеву недостатність, незважаючи на терапію кардіотонічними засобами і тіазидовим діуретиком, зберігаються набряки і виникла загроза асциту. Який препарат слід призначити для підсилення діуретичного ефекту застосованих ліків?

Амілорид

Спіронолактон

Фуросемід

Клопамід

Манітол

2113 / 6854
Пацієнта турбують поліурія (7 л на добу) і полідипсія. При обстеженні не виявлено ніяких розладів вуглеводного обміну. Дисфункція якої ендокринної залози може бути причиною даних порушень?

Аденогіпофіз

Острівці підшлункової залози

Кора наднирників

Мозкова речовина наднирників

Нейрогіпофіз

2114 / 6854
У жінки 20-ти років під час медичного огляду при пальпації в молочній залозі виявлено щільний інкапсульований вузол діаметром 1,0 см. Результат післяопераційного біопсійного дослідження: розростання сполучної тканини навколо протоків молочної залози та залозисті елементи різного діаметру, що не утворюють часточок, без ознак клітинного атипізму. Яка відповідь патологоанатома?

Аденома

Метастаз раку

Фіброаденома

Фіброма

Фіброзний рак

2115 / 6854
У хворого із швидко наростаючою внутрішньомозковою гіпертензією діагностована пухлина мозку. Під час операції видалена пухлина тім’яно-скроневої частки, м’якої консистенції, на розрізі строкатого вигляду. Гістологічно пухлина побудована з поліморфних гіперхромних клітин з утворенням псевдорозеток та великої кількості судин, ділянками некрозів і крововиливами. Який найбільш імовірний діагноз?

Арахноїдендотеліома

Олігодендрогліома

Астроцитома

Менінгіома

Гліобластома

2116 / 6854
У хворого з кровотечею розвинулась гостра ниркова недостатність, що спричинила смерть. На аутопсії макроскопічно: нирки збільшені з широким блідо-рожевим кірковим шаром, різко відмежованим від темно-червоних пірамід. Мікроскопічно: відсутність ядер епітелію звивистих канальців, тубулорексіс, венозний застій, ядра клітин судинних клубочків та прямих канальців збережені. Яка патологія нирок розвинулась у хворого?

Інфаркт

Некронефроз

Пієлонефрит

Нефроз

Гломерулонефрит

2117 / 6854
З метою масового обстеження студентів на носійство S.aureus перед виробничою практикою у дитячому відділенні клінічної лікарні було використано елективне середовище з метою отримання чистої культури цього збудника. Яке з перерахованих середовищ було використано?

Жовтково-сольовий агар

Середовище Вільсона-Блера

Середовище Ендо

Кров’яний телуритовий агар

М’ясо-пептонний агар

2118 / 6854
В крові хворого виявлено високий вміст галактози, концентрація глюкози знижена. Відмічена розумова відсталість, помутніння кришталика. Яке захворювання має місце?

Стероїдний діабет

Фруктоземія

Галактоземія

Цукровий діабет

Лактоземія

2119 / 6854
Під час бійки у чоловіка виникла зупинка серця внаслідок сильного удару у верхню ділянку передньої черевної стінки. Який із зазначених механізмів спричинив зупинку серця?

Парасимпатичні умовні рефлекси

Симпатичні безумовні рефлекси

Симпатичні умовні рефлекси

Периферичні рефлекси

Парасимпатичні безумовні рефлекси

2120 / 6854
У хворого, що надійшов до хірургічного відділення з ознаками гострого апендициту, виявлені наступні зміни білої крові: загальна кількість лейкоцитів - 16 • 109 /л. Лейкоцитарна формула: б.- 0, е.- 2%, ю.- 2%, п.- 8%, с.- 59%, л.-25%, м.- 4%. Як класифікуються зазначені зміни?

Нейтрофілія з регенеративним зсувом вліво

Нейтрофілія з дегенеративним зсувом вліво

Нейтрофілія з зсувом вправо

Нейтрофілія з гіперрегенеративним зсувом вліво

Лейкемоїдна реакція за нейтрофільним типом

2121 / 6854
У людини визначили частоту серцевих скорочень за пульсом. Вона дорівнює 120/хв. Якою при цьому є тривалість серцевого циклу?

1,0 с

0,8 с

0,9 с

0,7 с

0,5 с

2122 / 6854
У хворого із вираженим пневмосклерозом після перенесеного інфільтративного туберкульозу легень розвинулась дихальна недостатність. До якого патогенетичного типу вона відноситься?

Обструктивний

Дисрегуляційний

Рефлекторний

Апнеїстичний

Рестриктивний

2123 / 6854
Після перенесеної стрептококової інфекції у чоловіка діагностовано гострий гломерулонефрит. Найбільш імовірно, що ураження базальної мембрани ниркових тілець виникає внаслідок алергічної реакції такого типу:

Стимулююча

Анафілактична

Імунокомплексна

Цитотоксична

Сповільнена

2124 / 6854
До лікаря звернулась мати з приводу поганого самопочуття дитини - відсутність апетиту, поганий сон, дратівливість. При біохімічному дослідженні в крові виявлено відсутність ферменту глюкоцереброзидази. Для якої патології це характерно?

Хвороба Помпе

Хвороба Гірке

Хвороба Гоше

Хвороба Німана-Піка

Хвороба Тея-Сакса

2125 / 6854
Дитина 9-ми місяців харчується штучними сумішами, які не збалансовані за вмістом вітаміну B6. У дитини спостерігається пелагроподібний дерматит, судоми, анемія. Розвиток судом може бути пов’язаний з порушенням утворення:

Серотоніну

ДОФА

Дофаміну

Гістаміну

ГАМК

2126 / 6854
Внаслідок отруєння неякісною їжею хворому було промито шлунок розчином калію перманганату. Який механізм дії цього засобу?

Вивільнення атомарного кисню

Порушення синтезу ферментів дихального ланцюга

Вивільнення йоду

Вивільнення хлору

Порушення мембран бактерій

2127 / 6854
Чоловік протягом 3-х років працював в одній із африканських країн. Через місяць після переїзду до України звернувся до офтальмолога зі скаргами на біль в очах, набряки повік, сльозоточивість і тимчасове послаблення зору. Під кон’юнктивою ока були виявлені гельмінти розмірами 30-50 мм, які мали видовжене ниткоподібне тіло. Який найбільш імовірний діагноз?

Філяріоз

Дифілоботріоз

Трихоцефальоз

Ентеробіоз

Аскаридоз

2128 / 6854
У пацієнта з бронхіальною астмою за допомогою шкірних алергічних проб встановлено сенсибілізацію алергеном тополиного пуху. Який фактор імунної системи відіграє вирішальну роль в розвитку цього імунопатологічного стану?

IgD

IgE

IgG

IgM

Т-лімфоцити

2129 / 6854
У пацієнта діагностовано компресійний перелом поперекового хребця. При цьому різко збільшилась кривизна лордозу поперекового відділу хребта. Ушкодженням якої зв’язки може супроводжуватись така зміна кривизни хребта?

Задня поздовжня

Передня поздовжня

Жовта

Міжостиста

Клубово-поперекова

2130 / 6854
При дослідженні підозрілих м’ясних продуктів (сосиски), що мали характерних гнилісний запах, було виділено рухливі грамнегативні паличковидні мікроорганізми, що добре росли на МПА з ефектом 'роїння'. При посіві в конденсаційну воду мікроорганізми на поверхні середовища утворювали наліт димчасто-блакитного кольору. Який мікроорганізм міг спричинити гнилісний розпад даного продукту?

Сальмонели

Шигели дизентерії

Протей

Кишкова паличка

Холерний вібріон

2131 / 6854
До лабораторії доставлено кров хворого з підозрою на черевний тиф для проведення серологічного дослідження. Реакцію Відаля поставив недостатньо кваліфікований лаборант, який обмежився використанням тільки Ота Н-діагностикумів із сальмонел тифу. Які ще діагностикуми слід було використати для правильної постановки реакції Відаля?

Висипного та поворотного тифів

Еритроцитарні О- та Н-діагностикуми

Холери та дизентерії

К та Vi діагностикуми сальмонел тифу

Паратифів А та В

2132 / 6854
У дитини виявлена схильність до ожиріння, яка є результатом діатезу Назвіть вид діатезу, при якому найчастіше може розвинутись ожиріння:

Нервово-артритичний

Ексудативно-катаральний

Астенічний

Лімфатико-гіпопластичний

2133 / 6854
Інозитолтрифосфати в тканинах організму утворюються в результаті гідролізу фосфатидилінозитолдифосфатів і відіграють роль вторинних посередників (месенджерів) в механізмі дії гормонів. Їхній вплив у клітині спрямований на:

Гальмування фосфодіестерази

Активацію аденілатциклази

Вивільнення іонів кальцію з клітинних депо

Активацію протеїнкінази А

Гальмування протеїнкінази С

2134 / 6854
У хворого спостерігаються геморагії, в крові знижена концентрація протромбіну. Недостатність якого вітаміну призвела до порушення синтезу цього фактору згортання крові?

K

A

E

C

D

2135 / 6854
При отруєнні невідомим препаратом у пацієнта спостерігались сухість слизової оболонки рота та розширення зіниць. З яким впливом пов’язана дія цього препарату?

Стимуляція H-холінорецепторів

Стимуляція M-холінорецепторів

Стимуляція адренорецепторів

Блокада M-холінорецепторів

Блокада адренорецепторів

2136 / 6854
У відпочиваючого в санаторії у результаті сонячного опіку на шкірі спини утворилися міхурці, заповнені світлою рідиною, оточені зоною гіперемії, болісні. Який з перерахованих механізмів лежить в основі формування ексудації у вогнищі запалення?

Еміграція лейкоцитів з судин

Зменшення рівня кейлонів у тканині

Збільшення колоїдно-осмотичного тиску в тканині

Збільшення кількості лізосомальних ферментів

Зменшення виведення рідини з тканини

2137 / 6854
У лабораторії особливо небезпечних інфекцій проводиться мікроскопічне дослідження патологічного матеріалу від хворого з підозрою на чуму, забарвленого за Гінсом-Буррі. Яку властивість збудника дозволяє визначити даний метод?

Спороутворення

Капсулоутворення

Наявність зерен волютину

Лугостійкість

Кислотостійкість

2138 / 6854
Внаслідок вираженого зниження концентрації кальцію в плазмі крові у дитини 2-х років виникли тетанічні скорочення дихальних і глоткових м’язів. Зниження секреції якого гормону може бути причиною цього?

Кортизол

Соматотропін

Альдостерон

Паратгормон

Тиреокальцитонін

2139 / 6854
У хворого з лихоманкою та висипкою на шкірі після обстеження за допомогою серологічних реакцій поставлений діагноз фасціольоз. Було встановлено, що хворий заразився шляхом споживання сирої води з річки. Яка стадія життєвого циклу фасціоли інвазійна для людини?

Метацеркарій

Фіна

Мірацидій

Адолескарій

Яйце

2140 / 6854
До лікаря-педіатра звернулись батьки новонародженої дитини зі скаргами на виділення рідини (сечі) в ділянці пупка. Яка вроджена вада у дитини?

Незрощена сечова протока

Пахвинна грижа

Розщеплення сечівника

Пупкова грижа

Дивертикул Меккеля

2141 / 6854
При харчовому отруєнні виділена культура анаеробних грампозитивних спороутворюючих паличок. До якого виду, найбільш імовірно, відноситься виділений збудник?

Esherichia coli

Proteus vulgaris

C. perfringens

P.mirabilis

Vibrio parahemolyticus

2142 / 6854
До бактеріологічної лабораторії доставлені блювотні маси хворого з підозрою на холеру. З матеріалу приготований препарат 'висяча крапля'. Який метод мікроскопії буде використаний для виявлення збудника по його рухливості?

Люмінесцентна

Фазово-контрастна

Шерстна

Імунна електронна

Електронна

2143 / 6854
У результаті радіаційного випромінювання ушкоджені стовбурові гемопоетичні клітини. Утворення яких клітин сполучної тканини буде порушено?

Фібробласти

Меланоцити

Макрофаги

Адіпоцити

Періцити

2144 / 6854
У потерпілого з травмою голови у скроневій ділянці діагностовано епідуральну гематому. Яка з артерій найімовірніше пошкоджена?

Середня мозкова

Задня вушна

Поверхнева скронева

Передня оболонкова

Середня оболонкова

2145 / 6854
У дитини спостерігається затримка фізичного та розумового розвитку, глибокі порушення з боку сполучної тканини внутрішніх органів, у сечі виявлено кератансульфати. Обмін яких речовин порушений?

Еластин

Колаген

Глікозаміноглікани

Фібронектин

Гіалуронова кислота

2146 / 6854
У хворої 45-ти років невроз, що проявляється дратівливістю, безсонням, немотивованою тривогою. Який лікарський засіб усуне всі симптоми?

Діазепам

Леводопа

Екстракт валеріани

Кофеїн-бензоат натрію

Пірацетам

2147 / 6854
У хворого після вживання недоброякісної їжі розвинулася діарея. На наступний день у нього знизився артеріальний тиск, з’явились тахікардія, екстрасистолія. pH крові складає 7,18. Ці порушення є наслідком розвитку:

Негазового алкалозу

Газового алкалозу

Газового ацидозу

Негазового ацидозу

Метаболічного алкалозу

2148 / 6854
При ендоскопії шлунка виявлена атрофія слизової оболонки. Мікроскопічно в біоптаті виявлена пухлина, побудована з ланцюжків атипових епітеліальних клітин, розташованих серед тяжів грубоволокнистої сполучної тканини; строма значно переважає над паренхімою. Який найбільш імовірний діагноз?

Медулярний рак

Солідний рак

Скирозний рак

Дрібноклітинний рак

Аденокарцинома

2149 / 6854
Слизова оболонка трубчастого органу вкрита перехідним епітелієм, утворює поздовжні складки. М’язова оболонка складається з двох шарів у верхній половині і трьох у нижній. Який це орган?

Сечовий міхур

Маткова труба

Стравохід

Пряма кишка

Сечовід

2150 / 6854
До лікарні доставлений хворий з отруєнням інсектицидом - ротеноном. Яка ділянка мітохондріального ланцюга переносу електронів блокується цією речовиною?

НАДН-коензим Q-редуктаза

Сукцинат-коензим Q-редуктаза

Коензим Q-цитохром С-редуктаза

Цитохром С-оксидаза

АТФ -синтетаза

2151 / 6854
У хворого, який довготривало приймав преднізолон, в результаті відміни препарату виникло загострення захворювання, зниження артеріального тиску, слабкість. З чим можна зв’язати ці прояви?

Звикання до препарату

Кумуляція препарату

Сенсибілізація до препарату

Виникнення недостатності кори наднирників

Гіперпродукція АКТГ

2152 / 6854
У дитини 7-ми років, хворої на ангіну, був взятий матеріал (мазок з поверхні мигдаликів) і засіяний на кров’яний агар. Через добу виросли колонії стрептококів, навколо яких середовище стало прозорим. Наявність якого фактору патогенності у збудника виявило це дослідження?

Ендотоксин

Гемолізин

Нейрамінідаза

Лейкоцидин

Бета -лактамаза

2153 / 6854
У хворого на алкоголізм почався алкогольний психоз із вираженим психомоторним збудженням. Який препарат з групи нейролептиків слід призначити для швидкої допомоги?

Натрію бромід

Резерпін

Аміназин

Діазепам

Фторотан

2154 / 6854
При огляді дитини 4-х років зі скаргами на загальну слабкість, біль у горлі та утруднене ковтання лікар запідозрив дифтерію та направив матеріал до бактеріологічної лабораторії. На яке диференціально-діагностичне поживне середовище слід засіяти матеріал для виділення збудника дифтерії?

Середовище Левенштейна-Йєнсена

Середовище Плоскірєва

Середовище Сабуро

Кров’яно-телуритовий агар

Середовище Ендо

2155 / 6854
Серед антиатеросклеротичних препаратів, що застосовуються з метою профілактики та лікування атеросклерозу, є левостатин. Він діє шляхом:

Стимулювання екскреції холестерину з організму

Усіма наведеними шляхами

Гальмування біосинтезу холестерину

Активації метаболізму холестерину

Пригнічення всмоктування холестерину в кишечнику

2156 / 6854
У хворого під час ендоскопічного обстеження на слизовій оболонці шлунка виявлено кілька ерозій. Який із зазначених лікарських засобів міг спричинити таке ускладнення?

Діазепам

Глауцин

Фамотидин

Диклофенак-натрій

Дигоксин

2157 / 6854
При лабораторному дослідженні у хворого виявили стеаторею. Вкажіть фермент, недостатність дії якого призвела до виникнення цього симптому:

Ліпаза

Лактаза

Амілаза

Пепсин

Хімотрипсин

2158 / 6854
Людина впродовж тривалого часу вживала їжу, бідну на метіонін, внаслідок чого у неї спостерігалися розлади функції нервової та ендокринної систем. Це може бути наслідком порушення синтезу:

Глюкагону

Пірувату

Тироніну

Адреналіну

Жирних кислот

2159 / 6854
Жінка 45-ти років декілька років хворіє на системний червоний вовчак у легкій формі. При прогресуванні захворювання (з’явився міокардит) їй призначили преднізолон як імуносупресор. Через 2 місяці прийому у хворої виникла шлункова кровотеча. Яка найбільш імовірна її причина?

Підвищення артеріального тиску

Подальше прогресування захворювання

Зменшення згортання крові

Збудження ЦНС

Ульцерогенна дія

2160 / 6854
У новонародженої дитини виявлено вроджені вади розвитку травної системи, що пов’язано з дією тератогенних факторів на початку вагітності. На якій з зародкових листків подіяв тератоген?

Ендодерма і мезодерма

Ендодерма

Мезодерма

Ектодерма

Усі листки

2161 / 6854
Дитина під час гри порізала ногу осколком скла і була направлена у поліклініку для введення протиправцевої сироватки. З метою попередження розвитку анафілактичного шоку лікувальну сироватку вводили за Безредкою. Який механізм лежить в основі подібного способу гіпосенсибілізації організму?

Блокування синтезу медіаторів у тучних клітинах

Зв’язування фіксованих на тучних клітинах IgE

Стимуляція імунологічної толерантності до антигену

Стимуляція синтезу антиген-специфічних IgG

Зв’язування рецепторів до IgE на тучних клітинах

2162 / 6854
До токсикологічного відділення надійшов хворий з симптомами гострого отруєння сполуками ртуті. Який препарат необхідно призначити у якості антидоту?

Нейромедин

Плантаглюцин

Баррол

Трифтазин

Унітіол

2163 / 6854
Недостатність в організмі лінолевої та ліноленової кислот призводить до ушкоджень шкіри, випадіння волосся, сповільненого загоювання ран, тромбоцитопенії, зниження опірності до інфекційних захворювань. Порушення синтезу яких речовин найімовірніше зумовлює вказані симптоми?

Ейкозаноїди

Кортикостероїди

Катехоламіни

Інтерлейкіни

Інтерферони

2164 / 6854
У жінки, що тривало приймала антибіотики з приводу кишкової інфекції, розвинулось ускладнення з боку слизової порожнини рота у вигляді запального процесу і білого нальоту, у якому під час бактеріологічного дослідження були виявлені дріжджеподібні грибки Candida albicans. Який з перерахованих препаратів показаний для лікування цього ускладнення?

Флуконазол

Поліміксин

Фуразолідон

Бісептол

Тетрациклін

2165 / 6854
У хворого в лейкограмі: лейкоцити - 14 • 109 /л; мієлобласти - 71%, промієлоцити, мієлоцити, метамієлоцити - 0%, паличкоядерні нейтрофіли - 6%, сегментоядерні -13%; лімфоцити - 7%, моноцити - 3%. Яка патологія у хворого?

Лімфобластний лейкоз

Нейтрофільний лейкоцитоз

Хронічний мієлолейкоз

Хронічний лімфолейкоз

Мієлобластний лейкоз

2166 / 6854
Під час розтину тіла померлої жінки 54-х років з діагнозом 'вузликовий періартеріїт', мікроскопічно були виявлені такі змін в артеріолах: мукоїдне і фібриноїдне набухання, фібриноїдний некроз стінок, дифузна лімфогістіоцитарна інфільтрація, зміни ендотеліоцитів. Назвіть вид запалення в артеріолах:

Дифтеритичне запалення

Дифузне ексудативне запалення

Гостре не імунне запалення

Фібринозне запалення

Гостре імунне запалення

2167 / 6854
При дослідженні гостроти слуху в коваля виявили втрату слуху на 50% у діапазоні низьких частот і майже нормальну гостроту слуху в діапазоні високих частот. Порушення яких структур слухової системи призвело до такого стану?

Кортієв орган - ближче до овального віконця

Барабанна перетинка

Кортієв орган - ближче до гелікотреми

М’язи середнього вуха

Середня частина кортієвого органу

2168 / 6854
При відборі для ревакцинації вакциною БЦЖ у школяра поставлено пробу Манту, яка виявилася негативною. Результат проби свідчить про такі особливості імунітету до туберкульозу:

Відсутність антитоксичного імунітету

Наявність клітинного імунітету

Відсутність гуморального імунітету

Відсутність клітинного імунітету

Наявність гуморального імунітету

2169 / 6854
Людина вийшла з кондиціонованого приміщення назовні, де немає вітру, температура повітря +38oC, вологість 64%. За рахунок якого механізму буде здійснюватися віддача тепла організмом за цих умов?

Кондукція

Випаровування поту

Конвекція

Теплопроведення

Теплорадіація

2170 / 6854
Робітниця хімічного підприємства внаслідок порушення правил безпечної роботи зазнала токсичної дії азотистої кислоти та нітритів, які викликають дезамінування цитозину в молекулі ДНК. Який фермент ініціює ланцюг репараційних процесів?

Цитидинтрифосфатсинтетаза

Уридин-ДНК-глікозидаза

ДНК-залежна-РНК-полімераза

Тимідилатсинтаза

Оротидилмонофосфат-декарбоксилаза

2171 / 6854
У пацієнта з передозуванням наркотичної речовини відсутня свідомість, гіпотермія, гіпотензія, стійкий міоз, порушення дихання по типу Чейн-Стокса. Який функціональний антагоніст необхідно призначити для забезпечення виживання пацієнта?

Налоксон

Омепразол

Етимізол

Нітразепам

Мезатон

2172 / 6854
Хворий надійшов до клініки зі скаргами на загальну слабкість, порушення сну. Шкіра має жовтий колір. У крові: збільшена кількість прямого білірубіну, жовчних кислот. Кал ахолічний. Для якого стану характерні ці зміни?

Надпечінкова жовтяниця

Механічна жовтяниця

Синдром Жільбера

Хронічний холецистит

Гемолітична жовтяниця

2173 / 6854
У жінки 22-х років через 5 годин після вживання морепродуктів на шкірі тулуба та дистальних відділів кінцівок з’явились маленькі сверблячі папули, які частиною зливаються між собою. Через добу висипка самовільно зникла. Назвіть механізм гіперчутливості, що полягає в основі даних змін:

Антитілоопосередкований клітинний цитоліз

Клітинна цитотоксичність

Імунокомплексна гіперчутливість

Атопія (місцева анафілаксія)

Системна анафілаксія

2174 / 6854
Хворий на гіпертонічну хворобу разом з безсольовою дієтою та з антигіпертензивними засобами, довгий час приймав гідрохлортіазид, що зумовило порушення електролітного балансу. Яке порушення внутрішнього середовища виникло у хворого?

Гіперкаліємія

Збільшення об’єму циркулюючої крові

Метаболічний ацидоз

Гіпохлоремічний алкалоз

Гіпермагніємія

2175 / 6854
При авторадіографічному дослідженні епітелію тонкої кишки було виявлено, що його повне оновлення відбувається протягом 3-х діб за рахунок проліферації малодиференційованих клітин. Вкажіть їх локалізацію:

Власна пластинка слизової оболонки

Дно крипт

Основа ворсинок

Бічна поверхня ворсинок

Верхівка ворсинок

2176 / 6854
Пасажири автобуса у спекотну погоду попросили відкрити люки. Який шлях тепловіддачі при цьому зростає найбільше?

Теплопроведення

Випромінювання

Випромінювання та теплопроведення

Конвекція

Випаровування поту

2177 / 6854
У хворого через добу після апендектомії при аналiзi крові виявили нейтрофільний лейкоцитоз з регенеративним зсувом вліво. Який найбільш імовірний механізм розвитку абсолютного лейкоцитозу у периферичній крові хворого?

Зменшення руйнування лейкоцитів

Посилення лейкопоезу

Перерозподіл лейкоцитів в організмі

Уповільнення еміграції лейкоцитів у тканині

Активація імунітету

2178 / 6854
При загостренні ревматоїдного артриту хворому, в анамнезі якого супутній хронічний гастрит, призначений целекоксиб. Чим обумовлено зменшення побічної дії препарату на травний тракт?

Пригнічення фосфодіестерази

Переважаюче пригнічення циклооксигенази-1

Пригнічення фосфоліпази А2

Переважаюче пригнічення циклооксигенази-2

Переважаюча стимуляція аденіла-тциклази

2179 / 6854
В експерименті на кролі встановлено, що об’єм кисню, який споживається головним мозком за 1 хвилину, дорівнює об’єму CO2 , який виділяється клітинами мозку в кров. Це свідчить, що у клітинах головного мозку має місце:

Окислення жирів

Окислення білків

Окислення вуглеводів

Гіпоксія

Гіпокапнія

2180 / 6854
Хворому поставлено діагноз газова гангрена. Після ідентифікації збудника досліджуваний матеріал необхідно знищити. Який метод слід використати?

Пастеризація

Стерилізація парою під тиском

Кип’ятіння

Стерилізація текучою парою

Тиндалізація

2181 / 6854
Фенілкетонурія - це захворювання, яке зумовлено рецесивним геном, що локалізується в аутосомі. Батьки є гетерозиготами за цим геном. Вони вже мають двох хворих синів і одну здорову доньку. Яка імовірність, що четверта дитина, яку вони очікують, народиться теж хворою?

100%

50%

75%

0%

25%

2182 / 6854
До косметолога звернулася пацієнтка зі скаргами на появу чорних цяток на обличчі. Після обстеження було встановлено, що поява цяток пов’язана з порушенням виділення секрету сальних залоз. Який тип секреції характерний для цих залоз?

Мікроапокриновий

Макроапокриновий

Голокриновий

Мерокриновий та мікроапокриновий

Мерокриновий

2183 / 6854
У хворого 40-ка років ознаки гірської хвороби: запаморочення, задишка, тахікардія, рН крові - 7,50, pCO2 -30 мм рт.ст., зсув буферних основ +4 ммоль/л. Яке порушення кислотноосновного стану має місце?

Негазовий алкалоз

Видільний ацидоз

Газовий ацидоз

Газовий алкалоз

Негазовий ацидоз

2184 / 6854
Після ремонту автомобіля в закритому приміщенні при працюючому двигуні у чоловіка з’явилися задишка, запаморочення, акроціаноз, частота дихання 24-26/хв. Газовий склад крові: pO2 - 60 мм рт.ст., pCO2 - 30 мм рт.ст.; у крові наявний карбоксигемоглобін. Про який вид гіпоксії можна думати?

Респіраторна

Гіпоксична

Тканинна

Циркуляторна

Гемічна

2185 / 6854
У клітині в гранулярній ЕПС відбувається етап трансляції, при якому спостерігається просування і-РНК щодо рибосоми. Амінокислоти з’єднуються пептидними зв’язками в певній послідовності - відбувається біосинтез поліпептиду. Послідовність амінокислот у поліпептиді буде відповідати послідовності:

Кодонів і-РНК

Нуклеотидів р-РНК

Антикодонів т-РНК

Нуклеотидів т-РНК

Антикодонів р-РНК

2186 / 6854
При обстеженні у хворого виявлене порушення чутливості шкіри в ділянці передньої поверхні шиї. Який нерв уражений?

Шийна петля

Поперечний нерв шиї

Надключичні

Великий вушний

Малий потиличний

2187 / 6854
Хворий на хронічну серцеву недостатність тривалий час приймав препарат з групи серцевих глікозидів. У нього з’явились нудота, слабкість, екстрасистолія. Яке явище обумовило розвиток цих симптомів?

Матеріальна кумуляція

Функціональна кумуляція

Ідіосинкразія

Лікарська залежність

Звикання

2188 / 6854
При ультразвуковому обстеженні дитини був виявлений дивертикул Меккеля. Аномалію розвитку якої кишки діагностовано?

Сліпа

Клубова

Порожня

Ободова

Сигмоподібна

2189 / 6854
До реанімаційного відділення в тяжкому стані, без свідомості надійшов пацієнт. Діагностовано передозування барбітуратів, які спричинили феномен тканинної гіпоксії. На якому рівні відбулося блокування електронного транспорту?

НАДН-коензимQ-редуктаза

Цитохром b - цитохром c1

Цитохромоксидаза

Убіхінон

АТФ -синтаза

2190 / 6854
Встановлено, що в клітинах організмів відсутні мембранні органели та їх спадковий матеріал не має нуклеосомної організації. Що це за організми?

Аскоміцети

Еукаріоти

Найпростіші

Прокаріоти

Віруси

2191 / 6854
У пацієнта діагностований первинний туберкульоз легень. Призначення якого протитуберкульозного засобу з групи антибіотиків, що порушують синтез РНК мікобактерій, є бажаним?

Етамбутол

Рифампіцин

ПАСК

Піразинамід

Канаміцину сульфат

2192 / 6854
Хлопчик на другому році життя став часто хворіти на респіраторні захворювання, стоматити, гнійничкові ураження шкіри. Навіть невеликі пошкодження ясен і слизової ускладнюються запаленням, що протікає тривало. Встановлено, що у крові дитини практично відсутні імуноглобуліни усіх класів. Зниження функціональної активності якої клітинної популяції лежить в основі описаного синдрому?

NK-лімфоцити

Нейтрофіли

В-лімфоцити

Т-лімфоцити

Макрофаги

2193 / 6854
Секреція грудного молока у жінок обумовлена полімерними генами, причому кількість молока зростає із збільшенням числа домінантних алелів цих генів у генотипі жінки. Який генотип може мати породілля з відсутністю молока?

M1m1m2m2

M1m1M2m2

m1m1m2m2

M1 M1m2m2

m1m1M2m2

2194 / 6854
У хворого при томографічному обстеженні було виявлено пухлину відділу мозку з ушкодженням ядер XI та XII пар черепних нервів. Який це відділ мозку?

Telencephalon

Metencephalon

Diencephalon

Mesencephalon

Myelencephalon

2195 / 6854
У хворого 69-ти років на шкірі в ділянці нижньої повіки з’явилося невелике бляшкоподібне утворення з наступним виразкуванням, яке було оперативно видалене. При мікроскопічному дослідженні утворення: в дермі шкіри комплекси з атипових епітеліальних клітин; периферії клітини розташовані перпендикулярно до базальної мембрани. Клітини темні, призматичної полігональної форми з гіперхромними ядрами з частими мітозами. Іноді зустрічаються утворення, подібні до волосяного фолікула. Яка гістологічна форма рака у хворого?

Базально-клітинний рак

Плоскоклітинний рак без ороговіння

Аденокарцинома

Плоскоклітинний рак з ороговінням

Недиференційований рак

2196 / 6854
До пологового відділення госпіталізували жінку зі слабкістю пологової діяльності. Який засіб необхідно використати для стимуляції скорочень матки?

Соматостатин

Кортиколіберин

Окситоцин

Іонадоліберин

Вазопресин

2197 / 6854
У стоматолога на прийомі дуже неспокійний пацієнт, який ніяк не може зручно влаштуватися в кріслі, норовить схопити доктора за руку, заглядає на маніпуляційний стіл, цікавиться в медсестри, чи стерильні інструменти. Який темперамент у цього пацієнта?

Сангвінік

Холерик

Меланхолік

Флегматик

2198 / 6854
У хворого на слизовій оболонці ясен виразка овальної форми з припіднятими краями хрящоподібної щільності. Дно виразки м’ясисто-червоного забарвлення з нашаруваннями сірого кольору. При мікроскопічному дослідженні - проліферація ендотелію дрібних судин, периваскулярна лімфоплазмоцитарна інфільтрація. Про яке захворювання йдеться?

Виразка-рак

Травматична виразка

Ерозивно-виразкова лейкоплакія

Виразково-некротичний гінгівіт

Сифіліс

2199 / 6854
Визначте пульсовий і середньодинамічний артеріальний тиск (мм рт.ст.) у обстежуваного, якщо виміряний у нього артеріальний тиск становить 130/70 мм рт.ст.:

60,80

50,90

50, 70

60, 100

60, 90

2200 / 6854
У людини подразнення шкіри - свербіж, висипання, депігментація, збільшені лімфовузли. В оці знайдені філярії. Встановлений діагноз - онхоцеркоз. Які компоненти гнусу могли стати переносниками філярій p.Onchocerca?

Комарі

Ґедзі

Мошки

Москіти

Мокрець

2201 / 6854
Метильні групи (—CH3) використовуються в органiзмi для синтезу таких важливих сполук, як креатин, холін, адреналін, інші. Джерелом цих груп є одна з незамінних амінокислот, а саме:

Метіонін

Триптофан

Валін

Лейцин

Ізолейцин

2202 / 6854
Хвора 46-ти років скаржиться на сухість в роті, спрагу, почащений сечопуск, загальну слабкість. У крові: гіперглікемія, гіперкетонемія. У сечі: глюкоза, кетонові тіла. На ЕКГ: дифузні зміни в міокарді. Який найбільш імовірний діагноз?

Аліментарна гіперглікемія

Цукровий діабет

Гострий панкреатит

Немічна хвороба серця

Нецукровий діабет

2203 / 6854
В шкірі виявлена щільна, рухома, чітко відмежована від оточуючих тканин пухлина. На розрізі вона білого кольору, представлена волокнистою тканиною. Мікроскопічно: хаотично переплетені колагенові волокна, клітин мало. Що це за пухлина?

Дерматофіброма

Міома

Фіброма

Десмоїд

Гістіоцитома

2204 / 6854
У постраждалого виявлено рану верхньої частини передньої ділянки плеча. При обстеженні встановлена втрата активного згинання в ліктьовому суглобі і чутливості шкіри передньобічної поверхні передпліччя. Порушення функції якого нерва має місце?

Пахвовий

Серединний

М’язово-шкірний

Ліктьовий

Променевий

2205 / 6854
При підозрі на туберкульоз хворій дитині зробили пробу Манту. Через 24 години у місці введення алергену з’явились припухлість, гіперемія і болісність. Які основні компоненти визначають цю реакцію організму?

Гранулоцити, Т-лімфоцити і IgG

В-лімфоцити, IgМ

Мононуклеари, Т-лімфоцити і лімфокіни

Макрофаги, В-лімфоцити і моноцити

Плазматичні клітини, Т-лімфоцити і лімфокіни

2206 / 6854
На прийом до терапевта прийшов чоловік 37-ми років зі скаргами на періодичні інтенсивні больові напади у суглобах великого пальця стопи та їх припухлість. У сечі: різко кисла реакція і рожеве забарвлення. З наявністю яких речовин можуть бути пов’язані такі зміни?

Сульфат магнію

Хлориди

Фосфат кальцію

Солі сечової кислоти

Амонієві солі

2207 / 6854
При санітарно-бактеріологічному дослідженні водопровідної води отримані наступні результати: загальна кількість бактерій в 1,0 мл - 80, колі-індекс - 3. Як розцінити результат дослідження?

Вода є забрудненою

Вода є сумнівною

Вода є дуже сумнівною

Вода придатна для споживання

Вода є дуже забрудненою

2208 / 6854
в крові: ер.- 3,0 • 1012 8. /л; Hb-90г/л; ретикул.- 0,5%. В мазку: пойкілоцити, гіпохромні еритроцити. Залізо сироватки крові - 80 мкмоль/л. Для якої патології це характерно?

Залізорефрактерна анемія

B12-дефіцитна анемія

Серпоподібноклітинна анемія

Хвороба Мінковського-Шоффара

Залізодефіцитна анемія

2209 / 6854
Хворий 40-ка років впродовж тривалого часу страждає на бронхіальну астму і тахікардію. Вкажіть найдоцільніший в даній ситуації препарат для усунення бронхоспазму:

Атропіну сульфат

Ефедрину гідрохлорид

Адреналіну гідрохлорид

Сальбутамол

Ізадрин

2210 / 6854
Хворому на гіпертонічну хворобу був призначений препарат для зниження артеріального тиску з групи адренотропних засобів. Через деякий час у хворого тиск нормалізувався, але розвинулися брадикардія до 50/хв. та атріовентрикулярна блокада II ступеня. Який препарат було призначено?

Клофелін

Верапаміл

Мезатон

Празозин

Анаприлін

2211 / 6854
Хлопчик 12-ти років знаходиться у лікарні з підозрою на харчову токсикоінфекцію. При посіві фекалій хворого на середовище Ендо виросла велика кількість безбарвних колоній. Який мікроорганізм можна з найбільшою імовірністю ВИКЛЮЧИТИ з числа можливих збудників захворювання?

Escherichia coli

Pseudomonas aeruginosa

Proteus vulgaris

Yersinia enterocolitica

Salmonella enteritidis

2212 / 6854
Хворій 65-ти років, що страждає на інсулінонезалежний цукровий діабет, призначили всередину глібенкламід. Вкажіть механізм гіпоглікемічної дії цього препарату:

Пригнічує всмоктування глюкози у кишечнику

Стимулює виділення ендогенного інсуліну бета-клітинами

Пригнічує глюконеогенез у печінці

Посилює утилізацію глюкози периферичними тканинами

Пригнічує альфа-глюкозидазу і розпад полісахаридів

2213 / 6854
У тварини через 2 тижні після експериментального звуження ниркової артерії підвищився артеріальний тиск. Зі збільшенням дії на судини якого фактора гуморальної регуляції це пов’язано?

Дофамін

Вазопресин

Кортизол

Альдостерон

Ангіотензин II

2214 / 6854
У лабораторному експерименті на собаці вивчали будову центральних відділів слухової сенсорної системи. Була зруйнована одна з структур середнього мозку. Собака втратив орієнтувальний рефлекс на звукові сигнали. Яка структура була зруйнована?

Червоне ядро

Верхні горбики чотиригорбикового тіла

Чорна речовина

Ядра ретикулярної формації

Нижні горбики чотиригорбикового тіла

2215 / 6854
Хворий 23-х років надійшов до лікарні із черепно-мозковою травмою у важкому стані. Дихання характеризується судомним тривалим вдихом, який переривається коротким видихом. Для якого типу дихання це характерно?

Куссмауля

Апнейстичне

Чейн-Стокса

Біота

Гаспінг-дихання

2216 / 6854
При токсичному ушкодженні клітин печінки з порушенням її функцій у хворого з’явилися набряки. Які зміни складу плазми крові є провідною причиною розвитку набряків?

Зменшення вмісту глобулінів

Зменшення вмісту фібриногену

Зниження вмісту альбумінів

Збільшення вмісту альбумінів

Збільшення вмісту глобулінів

2217 / 6854
Під час операції в печінці хворого виявлені дрібні міхурці малих розмірів з незначною кількістю рідини, які щільно прилягають один до одного. Який гельмінтоз виявився у хворого?

Клонорхоз

Опісторхоз

Альвеококоз

Фасціольоз

Дікроцеліоз

2218 / 6854
Тварині, сенсибілізованій туберкуліном, внутрішньоочеревенно введений туберкулін. Через 24 години при лапаротомії виявлено венозну гіперемію та набряк очеревини. У мазках-відбитках з очеревини велика кількість лімфоцитів та моноцитів. Який патологічний процес у тварини?

Фібринозне запалення

Гнійне запалення

Серозне запалення

Алергічне запалення

Асептичне запалення

2219 / 6854
У хлопчика 3-х років з вираженим геморагічним синдромом відсутній антигемофільний глобулін А (фактор VIII) у плазмі крові. Яка фаза гемостазу первинно порушена у цього хворого?

Зовнішній механізм активації протромбінази

Внутрішній механізм активації протромбінази

Перетворення фібриногену в фібрин

Ретракція кров’яного згустку

Перетворення протромбіну в тромбін

2220 / 6854
У пацієнта з бронхіальною астмою за допомогою шкірних алергічних проб встановлено сенсибілізацію алергеном тополиного пуху. Який фактор імунної системи відіграє вирішальну роль в розвитку цього імунопатологічного стану?

IgM

Сенсибілізовані Т-лімфоцити

IgG

IgD

IgE

2221 / 6854
При недостатності кровообігу у період інтенсивної м’язової роботи у м’язі в результаті анаеробного гліколізу накопичується молочна кислота. Яка її подальша доля?

Використовується тканинами для синтезу кетонових тіл

Використовується у тканинах для синтезу жирних кислот

Використовується у м’язі для синтезу амінокислот

Видаляється через нирки з сечею

Включається в глюконеогенез у печінці

2222 / 6854
Дитина 10-ти років страждає на стафілококовий дерматит. Лікування бензилпеніциліном не дало результатів. Призначення комбінованого препарату пеніциліну з клавулановою кислотою дало швидке одужання. Яка причина позитивної дії цього препарату?

Гальмування аденозиндезамінази

Блокада транслокази

Активація фосфодіестерази

Інактивація бета-лактамази

Гальмування транспептидази

2223 / 6854
Хворий 18-ти років звернувся до лікаря зі скаргами на безсоння, що проявляється у важкому засинанні. В результаті цього він не висипається і на наступний день почуває втому, важко засвоює навчальний матеріал. Лікар встановив, що безсоння пов’язане з неврозоподібним станом. Зробіть раціональний вибір снодійного:

Хлоралгідрат

Етамінал-натрій

Бромізовал

Нітразепам

Фенобарбітал

2224 / 6854
У хворого, який страждає на вугрі та на запальні зміни шкіри обличчя, при мікроскопії матеріалу з осередків ураження виявлені живі членистоногі, довгастої форми, з 4 парами дуже редукованих кінцівок. Встановіть попередній діагноз:

Ураження шкіри блохами

Алергія

Ураження шкіри коростяним свербуном

Демодекоз

Педикульоз

2225 / 6854
Подразнення правого блукаючого нерва спричинило різке сповільнення атріовентрикулярного проведення. На ЕКГ при цьому буде подовжений:

Зубець P

Інтервал R — R

Інтервал P — Q

Комплекс QRST

Зубець T

2226 / 6854
У людини після укусу москітом виникли виразки шкіри. Аналіз вмісту виразки виявив всередині клітин людини безджгутикові одноклітинні організми. Який попередній діагноз?

Лейшманіоз вісцеральний

Токсоплазмоз

Лейшманіоз дерматотропний

Балантидіоз

Трипаносомоз

2227 / 6854
У чоловіка 63-х років рак стравоходу, метастази у лімфатичні вузли середостіння, ракова кахексія. Яка патогенетична стадія пухлинного процесу має місце?

Ініціації

Трансформації

Прогресії

Промоції

2228 / 6854
Жінку 44-х років вжалила оса, внаслідок чого розвинувся шок. В анамнезі - тяжка алергічна реакція на жалення оси. Об’єктивно: Ps- 179/хв, слабкий, АТ- 80/40 мм рт.ст., ЧД- 26/хв. Яка провідна ланка патогенезу анафілактичного шоку?

Зменшення об’єму циркулюючої крові

Тахікардія

Зниження периферійного опору судин

Зменшення ударного об’єму серця

Біль

2229 / 6854
Хворий надійшов до інфекційного відділення з підозрою на холеру. Який основний метод дослідження необхідно використати для підтвердження діагнозу?

Бактеріологічний

Імунологічний

Біологічний

Серологічний

Алергічний

2230 / 6854
При розтині тіла померлого виявлена гіперплазія кісткового мозку плоских і трубчастих кісток (піоїдний кістковий мозок), спленомегалія (6 кг), гепатомегалія (5 кг), збільшення всіх груп лімфатичних вузлів. Якому захворюванню відповідають виявлені зміни?

Лімфогрануломатоз

Справжня поліцитемія

Мієломна хвороба

Хронічний мієлолейкоз

Хронічний лімфолейкоз

2231 / 6854
Електрофоретичне дослідження сироватки крові хворого на пневмонію показало збільшення однієї з білкових фракцій. Вкажіть її:

α1-глобуліни

β-глобуліни

α2-глобуліни

γ-глобуліни

Альбуміни

2232 / 6854
У хворого коса пахвинна грижа. Яке анатомічне утворення стало слабким місцем передньої черевної стінки?

Латеральна пахвинна ямка

Пахвинний трикутник

Надміхурова ямка

Медіальна пахвинна ямка

Стегнова ямка

2233 / 6854
У хворого запалення легень ускладнилось ексудативним плевритом. В якому з перелічених анатомічних утворень переважно може накопичуватися рідина?

Sinus phrenicomediastinalis pleurae

Sinus obliquus pericardii

Sinus transversus pericardii

Sinus costodiaphragmaticus pleurae

Sinus costomediastinalis pleurae

2234 / 6854
Хворий не може відвести від тулуба верхню кінцівку. Який м’яз не виконує свою функцію?

Великий круглий м’яз

Підосний м’яз

Найширший м’яз спини

Дельтоподібний

Малий круглий м’яз

2235 / 6854
У хворої 45-ти років при електрокардіографічному обстеженні виявлено такі зміни: інтервал P — Q подовжений, при цьому випадає кожен другий або третій комплекс QRST. Яке саме порушення провідності серця спостерігається?

Атріовентрикулярна блокада повна

Атріовентрикулярна блокада III ступеня

Внутрішлуночкова блокада

Синоаурікулярна блокада

Атріовентрикулярна блокада I ступеня

2236 / 6854
У хворого порушена моторна функція язика. З патологією якого нерва це пов’язано?

Блукаючий

Язикоглотковий

Додатковий

Лицевий

Під’язиковий

2237 / 6854
При розтині тіла померлого чоловіка 65-ти років, який страждав на захворювання легень, патологічний процес переважно був локалізований у бронхах, де при гістологічному дослідженні були чітко видні залози, хрящові острівці та багаторядний циліндричний миготливий епітелій. В яких бронхах відбулися зміни?

Термінальні бронхіоли

Середні бронхи

Великі бронхи

Головні бронхи

Малі бронхи

2238 / 6854
До лікарні надійшов 9-річний хлопчик розумово і фізично відсталий. При біохімічному дослідженні крові: підвищена кількість фенілаланіну. Блокування якого ферменту може призве- сти до такого стану?

Оксидаза гомогентизинової кислоти

Фенілаланін-4-монооксигеназа

Аспартатамінотрансфераза

Глутаматдекарбоксилаза

Глутамінтрансаміназа

2239 / 6854
За клінічними показами хворому призначено піридоксальфосфат. Для корекції яких процесів рекомендований цей препарат?

Трансамінування і декарбоксилюван-ня амінокислот

Дезамінування амінокислот

Синтез білка

Синтез пуринових і піримідинових основ

Окисне декарбоксилювання кетокислот

2240 / 6854
До реанімаційного відділення надійшов хворий з ознаками гострого отруєння морфіном. Який засіб необхідно використати у даному випадку для промивання шлунку?

Розчин натрію хлориду

Натрію гідрокарбонат

Борна кислота

Фурацилін

Калію перманганат

2241 / 6854
У хворого відзначаються болі у ділянці кореня язика, зіву, піднебінних мигдаликів, у верхньому відділі глотки, вусі, втрачений смак у ділянці задньої третини язика. Ураженням якого нерва викликані ці порушення?

Блукаючий

Великий кам’янистий

Язиковий

Барабанна струна

Язикоглотковий

2242 / 6854
Для лікування деяких інфекційних захворювань, викликаних бактеріями, застосовуються сульфаніламідні препарати, що блокують синтез фактора росту бактерій. Назвіть механізм їх дії:

Інгібують всмоктування фолієвої кислоти

Беруть участь в окисно-відновних процесах

Є алостеричними інгібіторами ферментів

Є антивітамінами параамінобензойної кислоти

Є алостеричними ферментами

2243 / 6854
В сечі новонародженого визначається цитрулін та високий рівень амоніаку. Вкажіть, утворення якої речовини, найімовірніше, порушене у цього малюка:

Амоніак

Сечова кислота

Креатинін

Сечовина

Креатин

2244 / 6854
Жінка 45 -ти років, перукар, скаржиться на болі у ногах, що з’являються після роботи, ввечері та вночі. При огляді хворої виявлено варикозне розширені вени на присередній поверхні гомілки та стегна. Яка вена та її безпосередні протоки розширені?

Мала підшкірна

Передня великогомілкова вена

Стегнова

Глибока вена стегна

Велика підшкірна

2245 / 6854
У людини зменшений діурез, гіпернатріємія, гіпокаліємія. Гіперсекреція якого гормону може бути причиною таких змін?

Вазопресин

Альдостерон

Передсердний натрійуретичний фактор

Паратгормон

Адреналін

2246 / 6854
Після травми передньої поверхні верхньої третини передпліччя у хворого утруднення пронації, послаблення долонного згинання кисті та порушення чутливості шкіри 1-3 пальців на долоні. Який нерв ушкоджено?

n. radialis

n. ulnaris

n. musculocutaneus

n. cutaneus antebrachii medialis

n. medianus

2247 / 6854
У хворого 45-ти років при аналізі ЕКГ встановлено: ритм синусовий, число передсердних комплексів більше числа шлуночкових комплексів; прогресуюче подовження інтервалу P — Q від комплексу до комплексу; випадіння окремих шлуночкових комплексів; зубці P та комплекси QRST без змін. Назвіть тип порушення серцевого ритму:

Атріовентрикулярна блокада II ступеня

Повна атріовентрикулярна блокада

Синоаурікулярна блокада

Внутрішньопередсердна блокада

Атріовентрикулярна блокада I ступеня

2248 / 6854
В родині зростає дочка 14-ти років, у якої спостерігаються деякі відхилення від норми: зріст нижче, ніж у однолітків, відсутні ознаки статевого дозрівання, шия дуже коротка, плечі широкі. Інтелект в нормі. Яке захворювання можна припустити?

Синдром Едвардса

Синдром Клайнфельтера

Синдром Шерешевського-Тернера

Синдром Патау

Синдром Дауна

2249 / 6854
Після обстеження хворому на сечокам’яну хворобу призначили алопурінол - конкурентний інгібітор ксантиноксидази. Підставою для цього був хімічний аналіз ниркових каменів, переважною складовою яких є:

Фосфат кальцію

Урат натрію

Моногідрат оксалату кальцію

Дигідрат оксалату кальцію

Сульфат кальцію

2250 / 6854
У хворого 60-ти років, що багато років страждає на атеросклероз і переніс раніше інфаркт міокарда, розвинувся напад загрудинного болю. Хворий госпіталізований через 3 дні; на фоні прогресуючої серцево-судинної недостатності помер. Під час розтину тіла у ділянці задньої стінки лівого шлуночка і міжшлуночкової перегородки виявлена ділянка білого кольору близько 3 см у діаметрі, волокниста, западаюча, з чіткою межею. Прозектор трактував ці зміни як:

Вогнищевий кардіосклероз

Міокардит

Інфаркт міокарда

Ішемія міокарда

Дистрофія міокарда

2251 / 6854
Дослідник при мікроскопічному і електронно-мікроскопічному вивченні печінки звернув увагу, що деякі окремо розташовані клітини розпалися на дрібні фрагменти, оточені мембраною. У деяких з них наявні органели, інші включають фрагменти ядра, що розпалося. Запальна реакція навколо відсутня. Дослідник розцінив ці зміни, як:

Атрофія

Гіпоплазія

Дистрофія

Некроз

Апоптоз

2252 / 6854
У хворого на підгострий септичний ендокардит при огляді лікар відзначив загальну слабкість і іктеричність шкіри, склер і видимих слизових оболонок. У крові виявлена збільшена кількість непрямого білірубіну. Що зумовлює жовтяничність шкіри і слизових?

Печінкова жовтяниця

Надпечінкова жовтяниця

Жирова дистрофія

Гемосидероз

Підпечінкова жовтяниця

2253 / 6854
У хворого в анамнезі: з дитинства відмічався знижений рівень гемоглобіну. Лікування препаратами заліза не дає ефекту. У крові: ер.- 3,1 • 1012/л, ретик.-16%, Hb- 85 г/л, КП0,75; в мазку крові анізоцити, пойкілоцити, мішенеподібні еритроцити, еритроцити з базофільною зернистістю, рівень заліза у сироватці 30 мкмоль/л. Для якої патології системи крові характерні такі дані?

Фолієводефіцитна анемія

B12-дефіцитна анемія

Залізодефіцитна анемія

Гіпопластична анемія

Таласемія

2254 / 6854
До приймального відділення лікарні доставлено хворого з вираженими явищами гострої серцевої недостатності. Який препарат, із зазначених нижче, слід використати в першу чергу?

Фізостигмін

Дигітоксин

Етазол

Корглікон

Дитилін

2255 / 6854
Чоловік 30-ти років звернувся до стоматолога зі скаргою на розлади жування, у нього виникає біль при відтягуванні щелепи назад. Запалення якого з жувальних м’язів найімовірніше встановить лікар?

M. pterygoideus lateralis

M. temporalis (передш волокна)

M. Мasseter

M. pterygoideus medialis

M. temporalis (задні волокна)

2256 / 6854
Під час оперативного втручання на фоні використання гігронію різко знизився артеріальний тиск. Представники яких груп лікарських препаратів можуть нормалізувати артеріальний тиск?

α-адреноміметики

Н-холіноміметики

α-адреноблокатори

M-холіноміметики

Гангліоблокатори

2257 / 6854
з дихальною недостатністю рН крові 7,35. Визначення pCO2 57. показало наявність гіперкапнії. При дослідженні рН сечі відзначається підвищення її кислотності. Яка форма порушення кислотно-основного стану в даному випадку?

Алкалоз газовий, компенсований

Ацидоз метаболічний, декомпенсований

Ацидоз газовий, компенсований

Ацидоз метаболічний, компенсований

Алкалоз газовий, декомпенсований

2258 / 6854
У хворого, що страждає на порушення мозкового кровообігу, встановлено порушення функції лімбічної системи. Порушення кровопостачання у якій артерії мозку викликало ці симптоми?

Передня ворсинчаста

Передня мозкова

Задня мозкова

Хребтова

Середня мозкова

2259 / 6854
Хворому встановлений діагноз ураження голівки стегна ішемічного походження. Яка артерія ушкоджена?

Arteria profunda femoris

Arteria femoralis

Arteria umbilicalis

Arteria illiaca externa

Ramus acetabularum A.obturatoriae

2260 / 6854
В психіатричну клініку доставлений хворий 40-ка років у стані збудження, агресії, марення. Який препарат слід ввести хворому?

Натрію бромід

Аміназин

Резерпін

Седуксен

Настоянку валеріани

2261 / 6854
У нейрохірургічне відділення надійшов 54-річний чоловік із скаргами на відсутність чутливості шкіри нижньої повіки, латеральної поверхні носа, верхньої губи. Лікар при огляді встановив запалення другої гілки трійчастого нерва. Через який отвір виходить із черепа ця гілка?

Круглий отвір

Верхня очноямкова щілина

Рваний отвір

Остистий отвір

Овальний отвір

2262 / 6854
При бактеріологічному дослідженні проб сметани виділені ізольовані культури S.aureus. Як довести етіологічне значення ізольованої культури S.aureus як збудника харчового отруєння, яке виникло серед групи споживачів сметани?

Визначення гемотоксинів

Визначення лецитиназної активності

Виявлення ентеротоксину

Визначення цукролітичних властивостей

Визначення плазмокоагулазної активності

2263 / 6854
У дорослої людини за добу виділяється 20 л сечі з низькою відносною щільністю. Найбільш імовірною причиною цього є дефіцит в організмі:

Вазопресину

Альдостерону

Реніну

Натрійуретичного фактора

Паратгормону

2264 / 6854
У чоловіка 28-ми років при гістологічному дослідженні шийного лімфовузла виявлено: порушення малюнка внаслідок розростання епітеліоїдних, лімфоїдних клітин і макрофагів з ядрами у вигляді підкови, в центрі деяких скупчень клітин - безструктурні ділянки блідо-рожевого кольору з уламками ядер. Для якого захворювання характерні такі зміни?

Лімфогрануломатоз

Туберкульоз

Метастаз пухлини

Актиномікоз

Сифіліс

2265 / 6854
У хворого з жовтяницею встановлено: підвищення у плазмi крові вмісту загального білірубіну за рахунок непрямого (вільного), в калі та сечі - високий вміст стеркобіліну, рівень прямого (зв’язаного) білірубіну в плазмі крові в межах норми. Про який вид жовтяниці можна думати?

Хвороба Жильбера

Жовтяниця немовлят

Механічна

Гемолітична

Паренхіматозна (печінкова)

2266 / 6854
При штовханні штанги спортсмен закидає голову назад для максимального підвищення тонусу м’язів-розгиначів верхніх кінцівок. Де розташовані центри рефлексів, які при цьому виникають?

Базальні ганглії

Спинний мозок

Ядра Дейтерса

Рухова кора

Червоні ядра

2267 / 6854
Знешкодження ксенобіотиків (лікарських засобів, епоксидів, ареноксидів, альдегідів, нітропохідних тощо) та ендогенних метаболітів (естрадіолу, простагландинів, лейкотрієнів) відбувається в печінці шляхом їх кон’югації з:

Фосфоаденозином

Гліцином

Аспарагіновою кислотою

S-Аденозилметіоніном

Глутатіоном

2268 / 6854
При вивченні родоводу сім’ї, в якій спостерігається гіпертрихоз (надмірне оволосіння вушних раковин), виявлена ознака трапляється в усіх поколіннях тільки у чоловіків і успадковується від батька до сина. Визначте тип успадкування гіпертрихозу:

Зчеплений з Y-хромосомою

Аутосомно-домінантний

Аутосомно-рецесивний

Зчеплений з Х-хромосомою домінантний

Зчеплений з Х-хромосомою рецесивний

2269 / 6854
У спортсмена після інтенсивного тренування відзначається значне зниження тонусу судин у ділянці працюючих м’язів. Причиною розвитку такого ефекту є накопичення у працюючих тканинах:

Серотоніну

Ренін-ангіотензину

Натрійуретичного гормону

Метаболітів

Гістаміну

2270 / 6854
Внаслідок активації іонних каналів зовнішньої мембрани збудливої клітини значно збільшився її потенціал спокою. Які канали були активовані?

Калієві

Натрієві

Натрієві та кальцієві

Повільні кальцієві

Швидкі кальцієві

2271 / 6854
Чоловіку 40-ка років за вимогою діагностичних тестів зробили лімфографію органів грудної порожнини. Хірург встановив, що пухлина вразила орган, з лімфатичних судин якого лімфа безпосередньо переходить в грудну протоку. Який це орган?

Лівий головний бронх

Стравохід

Осердя

Серце

Трахея

2272 / 6854
У хірургічне відділення лікарні надійшла хвора з явищами гострого панкреатиту: блювання, пронос, сильний оперізуючий біль, слабкість, гіпотензія, зневодненням організму. Який препарат з антиферментною активністю показаний хворому?

Натрію гідрокарбонат

Атропіну сульфат

Анальгін

Контрикал

Адреналін

2273 / 6854
На препараті нирки розрізняються нефрони, які лежать на межі між кірковою та мозковою речовиною, мають однаковий діаметр приносних і виносних артеріол. Назвіть, яка функція буде порушена при їхньому пошкодженні?

Синтез еритропоетину

Активність натрієвого рецептора

Шунтування крові при інтенсивному кровообігу

Синтез реніну

Синтез простагландинів

2274 / 6854
Під час гістологічного дослідження стулок мітрального клапана серця жінки 30-ти років було встановлено, що ендотеліальні клітини вогнищево десквамовані, в цих ділянках на поверхні стулки розташовані дрібні тромботичні нашарування, сполучна тканина стулки з явищами мукоїдного набухання з ділянками склерозу та васкуляризації. Діагностуйте вид клапанного ендокардиту:

Дифузний

Фібропластичний

Поворотньо-бородавчастий

Гострий бородавчастий

Поліпозно-виразковий

2275 / 6854
Хворий переніс інсульт. Який з наведених препаратів слід включити до комплексної терапії з метою покращення кровообігу та метаболізму головного мозку?

Камфора

Амітриптилін

Феназепам

Седуксен

Пірацетам

2276 / 6854
Відзначте концентрацію етилового спирту, що має найбільш активну протимікробну дію за наявністю білку у середовищі:

60%

96%

40%

15%

70%

2277 / 6854
У хворого 40-ка років при прогресуванні стафілококового гнійного періодонтиту виникло гнійне запалення кістково-мозкових просторів альвеолярного відростка, а потім тіла нижньої щелепи. Мікроскопічно кісткові балки витончені, вогнища некрозу, кісткові секвестри, оточені сполучнотканинною капсулою. Який найбільш імовірний діагноз?

Хронічний остеомієліт

Хронічний фіброзний періостит

Гнійний періостит

Пародонтома

Гострий остеомієліт

2278 / 6854
Група чоловіків звернулася до лікаря зі скаргами на підвищення температури, головний біль, набряки повік та обличчя, біль у м’язах. З анамнезу: всі вони мисливці і часто вживають в їжу м’ясо диких тварин. Який найбільш імовірний діагноз?

Цистицеркоз

Теніарінхоз

Теніоз

Філяріатоз

Трихінельоз

2279 / 6854
Під час операції видалення матки з яєчниками і матковими трубами лікар перев’язує зв’язку, що підвішує яєчник. Які судини перев’язав лікар в цій зв’язці?

Маткові артерія і вена

Внутрішня клубова вена

Трубні артерія і вена

Внутрішня клубова артерія

Яєчникові артерія і вена

2280 / 6854
При проведенні операції на тонкій кишці лікар виявив ділянку слизової оболонки, де на фоні колових складок була присутня поздовжня складка. Який відділ тонкої кишки має таку будову?

Початковий eiddrn jejunum

Дистальний вiддiл ileum

Pars ascendens duodeni

Pars horizontalis duodeni

Pars descendens duodeni

2281 / 6854
Під час гістологічного дослідження легень хворого, що помер від серцевої недостатності, виявлені вогнища запалення з заповненням альвеол рідиною, забарвленою у блідорожевий колір, місцями з наявністю тонких рожевих ниток, що утворюють дрібнопетлисту сітку з невеликою кількістю лімфоцитів. Який характер ексудату у легенях?

Серозно-фібринозний

Фібринозний

Геморагічний

Серозний

Гнійний

2282 / 6854
У хворого нормально забарвлений кал, у складі якого з находиться велика кількість вільних жирних кислот. Причиною цього є порушення наступного процесу:

Жовчовиділення

Жовчоутворення

Гідроліз жирів

Всмоктування жирів

Секреція ліпаз

2283 / 6854
При термометрії встановлено, що температура відкритих ділянок шкіри на 1-1,5° нижче за температуру поруч розташованих ділянок, закритих одягом з натуральних тканин. Причиною цього є те, що одяг, перш за все, зменшує тепловіддачу таким шляхом:

Випаровування

Проведення

Радіація

Конвекція

2284 / 6854
У хворого діагностовано алкаптонурію. Вкажіть фермент, дефект якого є причиною цієї патології:

Оксидаза гомогентизинової кислоти

Піруватдегідрогеназа

ДОФА-декарбоксилаза

Фенілаланінгідроксилаза

Глутаматдегідрогеназа

2285 / 6854
На препараті м’якої мозкової оболонки виявляється судина, у стінці якої відсутня середня оболонка, зовнішня оболонка зрощена з оточуючою тканиною, внутрішня оболонка побудована із базальної мембрани та ендотелію. Що це за судина?

Артерія м’язового типу

Вена волокнистого типу

Артеріола

Артерія змішаного типу

Вена м’язового типу зі слабким розвитком м’язових елементів

2286 / 6854
На розтині тіла померлого від інтоксикації в тонкій кишці знайдено набряк групових лімфатичних фолікулів, виступаючих над поверхнею слизової оболонки у вигляді м’якоеластичних бляшок з нерівною поверхнею у вигляді борозен і звивин, що нагадують поверхню мозку. Який діагноз найбільш імовірний?

Гострий ентерит

Черевний тиф

Сальмонельоз

Холера

Дизентерія

2287 / 6854
При виконуванні вправ на колоді гімнастка втратила рівновагу і впала. Із збудження, перш за все, яких рецепторів розпочнуться рефлекси, що забезпечать відновлення порушеної пози?

Пропріорецептори

Ампулярні вестибулорецептори

Отолітові вестибулорецептори

Вестибулорецептори

Рецептори завитки

2288 / 6854
Під час обстеження хворої виникла підозра на наявність гнійного випоту в прямокишково-матковому заглибленні. Через яке анатомічне утворення найкраще пропунктувати дане заглиблення?

Ампула прямої кишки

Діафрагма тазу

Передня стінка піхви

Заднє склепіння піхви

Переднє склепіння піхви

2289 / 6854
Хворий 35-ти років звернувся зі скаргами на біль та набряк в ділянці дна ротової порожнини. Діагностовано запальний процес у ділянці вивідної протоки піднижньощелепної слинної залози. Куди відкривається ця протока?

Recesus gingivalis

Vestibulum oris

Caruncula sublingualis

Plica fimbriata

Foramen caecum linguae

2290 / 6854
До хірурга звернувся чоловік 60-ти років, що тривалий час хворіє на цукровий діабет. Об’єктивно: тканини правої стопи чорного кольору, щільні, з чіткими краями. Який діагноз поставив хірург?

Пролежень

Суха гангрена

Волога гангрена

Газова гангрена

Трофічна виразка

2291 / 6854
Запалення характеризується розширенням кровоносних судин на ділянці пошкодження, зменшенням кровообігу, підвищенням проникливості стінки судин. Яким з нижче наведених клітин належить головна роль в цьому?

Макрофаги

Тканинні базофіли

Еозинофіли

Фібробласти

Плазмоцити

2292 / 6854
Хворий після перенесеного епідемічного паротиту схуднув, постійно відчуває спрагу, п’є багато води, відмічає часте сечовиділення, підвищений апетит, шкірний свербіж, слабкість, фурункульоз. У крові: глюкоза - 16 ммоль/л, кетонових тіл - 100 мкмоль/л; глюкозурія. Яке захворювання розвинулось у пацієнта?

Стероїдний діабет

Нецукровий діабет

Інсулінонезалежний цукровий діабет

Інсулінозалежний цукровий діабет

Цукровий діабет недостатнього харчування

2293 / 6854
На гістологічне дослідження надіслано видалений червоподібний відросток. Розміри його збільшені, серозна оболонка тьмяна, повнокровна, вкрита плівками фібрину, стінки стовщені, на розрізі із просвіту виділяється гній. При мікроскопічному дослідженні спостерігається повнокров’я судин, набряк всіх шарів і дифузна інфільтрація їх лейкоцитами. Назвіть форму гострого апендициту:

Флегмонозний

Гангренозний

Апостематозний

Поверховий

Простий

2294 / 6854
У хворого 23-х років в результаті черепно-мозкової травми виник набряк мозку. Який механізм пошкодження клітин безпосередньо призвів до набряку мозку?

Ацидотичний

Протеїновий

Електролітно-осмотичний

Ліпідний

Кальцієвий

2295 / 6854
У хворого, який скаржився на біль у ділянці лівої лопатки, був діагностований інфаркт міокарду. Назвіть вид болю у хворого?

Другий (епікритичний)

Іррадіюючий (відбитий)

Перший (протопатичний)

Фантомний

Вісцеральний

2296 / 6854
Хвора 38-ми років надійшла до реанімаційного відділення в несвідомому стані. Рефлекси відсутні. Цукор крові -2,1 ммоль/л. В анамнезі - цукровий діабет з 18-ти років. Яка кома має місце у хворої?

Гіпоглікемічна

Кетоацидотична

Гіперосмолярна

Лактацидемічна

Гіперглікемічна

2297 / 6854
До фібрилярних елементів сполучної тканини належать колаген, еластин та ретикулін. Вкажіть амінокислоту, яка входить тільки до складу колагену і визначення якої в біологічних рідинах використовується для діагностики захворювань сполучної тканини:

Фенілаланін

Лізин

Гліцин

Пролін

Гідроксипролін

2298 / 6854
Пацієнт через 15 діб після повернення з багатомісячного плавання в районах Середземномор’я та Західної Африки відчув слабкість, головний біль, періодичні підвищення температури. Лікар запідозрив у хворого малярію. Який із перерахованих методів є найбільш адекватним в діагностиці даного захворювання?

Серологічний

Мікробіологічний

Біологічний

Алергічний

Мікроскопічний

2299 / 6854
У вагітної жінки взяли кров для підтвердження клінічного діагнозу 'токсоплазмоз'. Яка із перерахованих серологічних реакцій має діагностичне значення?

Реакція аглютинації

Реакція гемадсорбції

Реакція нейтралізації

Реакція гальмування гемаглютинації

Реакція зв’язування комплементу

2300 / 6854
У здорових батьків, спадковість яких не обтяжена, народилась дитина з чисельними вадами розвитку. Цитогенетичний аналіз виявив в соматичних клітинах дитини трисомію за 13-ю хромосомою (синдром Патау). З яким явищем пов’язане народження такої дитини?

Рецесивна мутація

Порушення гаметогенезу

Хромосомна мутація

Домінантна мутація

Соматична мутація

2301 / 6854
Для запобігання нападів гострого панкреатиту лікар призначив трасілол (контрікал, гордокс), який є інгібітором:

Еластази

Трипсину

Карбоксипептидази

Хімотрипсину

Гастриксину

2302 / 6854
Після травми на рентгенограмі постраждалого визначається перелом плеча у ділянці гребеня великого горбика. Функція якого м’яза буде у цьому випадку порушена?

Великий грудний

Дельтоподібний

Передній зубчастий

Малий грудний

Підключичний

2303 / 6854
Хворий помер від прогресуючої серцевої недостатності. На розтині серце розширене у поперечнику, мляве, м’яз на розрізі нерівномірного кровонаповнення, пістрявий, при гістологічному дослідженні у міокарді повнокров’я, у стромі лімфогістіоцитарні інфільтрати, що розсувають кардіоміоцити. Виявлені морфологічні зміни свідчать про:

Негнійний проміжний міокардит

Кардіосклероз

Інфаркт міокарда

Жирову дистрофії міокарда

Венозне повнокрів’я

2304 / 6854
В експерименті показано, що при саркомі Ієнсена споживання глюкози з привідної до пухлини артерії значно збільшується, має місце також приріст вмісту молочної кислоти у відвідній вені. Про що свідчить дане явище?

Зменшення анаеробного гліколізу

Зменшення окисних процесів

Посилення окисних процесів

Посилення анаеробного гліколізу

Посилення окиснення білків

2305 / 6854
У хворого 23-х років після перенесеної ангіни розвинувся сечовий синдром (гематурія, протеїнурія, лейкоцитурія). У пункційній біопсії нирок виявлена картина інтракапілярного проліферативного гломерулонефриту, а електронномікроскопічно виявлені великі субепітеліальні депозити. Який патогенез цього захворювання?

Атопія

Грануломатоз

Цитотоксична, цитолітична дія антитіл

Імунокомплексний механізм

Клітинно обумовлений цитоліз

2306 / 6854
У 29-річної породіллі на 3-й день після пологів виник дифузний набряк правої грудної залози, болючість при пальпації, гіперемія шкіри у вказаній ділянці, підвищення температури тіла до 38oC. При гістологічному дослідженні тканини залози виявлено: в стромі - дифузний клітинний інфільтрат, який складається з великої кількості нейтрофільних лейкоцитів, інтерстиційний набряк, гіперемія судин. Діагностуйте захворювання:

Хронічний продуктивний мастит

Гострий серозний мастит

Гострий апостематозний мастит

Хронічний гнійний мастит

Гострий флегмонозний мастит

2307 / 6854
У хворого на хронічну серцеву недостатність, незважаючи на терапію кардіотонічними засобами і тіазидовим діуретиком, зберігаються набряки і виникла загроза асциту. Який препарат слід призначити для підсилення діуретичного ефекту застосованих ліків?

Фуросемід

Клопамід

Манітол

Спіронолактон

Амілорид

2308 / 6854
До лікарні госпіталізовано хворого з підозрою на черевний тиф. Який матеріал необхідно взяти у нього з метою ранньої діагностики цього захворювання?

Жовч

Кістковий мозок

Фекалії

Сеча

Кров

2309 / 6854
У чоловіка 30-ти років перед операцією визначили групову належність крові. Кров резуспозитивна. Реакцію аглютинації еритроцитів не викликали стандартні сироватки груп 0αβ (I), Аβ (II), Вα (III). Досліджувана кров належить до групи:

Аβ (II)

Вα (III)

АВ (IV)

0αβ (I)

2310 / 6854
У людини збільшений вміст іонів кальцію в плазмі крові, зменшений - у кістках. Надмірна секреція якого гормону може спричинити такі зміни?

Альдостерон

Тироксин

Тиреокальцитонін

Паратгормон

Трийодтиронін

2311 / 6854
У студента, який складає іспит, вміст глюкози у плазмі крові складає 8 ммоль/л. Збільшена секреція якого з наведених гормонів сприяє розвитку гіперглікемії у студента?

Трийодтиронін

Глюкагон

Альдостерон

Тироксин

Інсулін

2312 / 6854
У хворого з неврологічними порушеннями діагностована пухлина головного мозку. Під час операції видалена пухлина, що має вид щільного вузла, пов’язаного з твердою мозковою оболонкою. Гістологічно пухлина побудована з ендотеліоподібних клітин, тісно прилеглих одна до одної. Який найбільш імовірний діагноз?

Астроцитома

Нейробластома

Менінгеома

Гліобластома

Менінгеальна саркома

2313 / 6854
На аутопсії жінки, що хворіла на хронічну дизентерію, при гістологічному дослідженні внутрішніх органів у стромі та паренхімі міокарда, нирок, у слизовій оболонці шлунка, у сполучній тканині легень виявлені аморфні відкладання фіолетового кольору, що дають позитивну реакцію за Коссом. Яке ускладнення розвинулось у хворої?

Гіаліноз

Дистрофічне звапніння

Метастатичне звапніння

Метаболічне звапніння

Амілоїдоз

2314 / 6854
Медсестра зі стажем роботи 10 років захворіла на контактний дерматит верхніх кінцівок. До якого типу імунної патології відноситься це захворювання?

Первинний імунодефіцит

Т-клітинний імунодефіцит

Алергічна реакція негайного типу

В-клітинний імунодефіцит

Алергічна реакція сповільненого типу

2315 / 6854
У хворого, прооперованого з приводу 'гострого живота', сеча коричневого кольору, кількість індикану в сечі вище 93 ммоль/добу. Про що це свідчить?

Збільшення швидкості окисного дезамінування ароматичних амінокислот

Зниження інтенсивності знезараження амоніаку

Збільшення інтенсивності гниття білків у кишечнику

Зниження активності ферментів орнітинового циклу

Порушення фільтраційної здатності нирок

2316 / 6854
У реанімаційному відділенні знаходиться хворий у коматозному стані. При дослідженні крові відзначено збільшення концентрації іонів K+ і зменшення - Ca++, ацидоз, збільшення рівнів сечовини, сечової кислоти. Який вид коми за етіологією найбільш імовірний?

Печінкова

Діабетична

Гіпоглікемічна

Ниркова

Нейрогенна

2317 / 6854
Жінка в період вагітності тривалий час безконтрольно приймала хіміотерапевтичний препарат. Через деякий час у неї погіршився апетит, з’явились нудота, пронос. З часом виникла жовтяниця. У новонародженого відмічено порушення росту кісток. Який препарат з групи тетрацикліну приймала жінка?

Азитроміцин

Ципрофлоксацин

Доксицикліну гідрохлорид

Бісєптол

Бензилпеніциліну натрiєва сіль

2318 / 6854
Під час розтину тіла дитини, яка померла при ознаках асфіксії, були виявлені в трахеї і головних бронхах сіруватого кольору плівки, які вільно лежали у просвіті дихальних шляхів, нагадуючи їх зліпки. Вкажіть вид запалення:

Серозне

Крупозне

Катаральне

Гнійне

Дифтеритичне

2319 / 6854
Чоловік 60-ти років скаржиться на біль у суглобах. У сироватці крові пацієнта виявлено підвищення концентрації С-реактивного білку та оксипроліну. Для якого захворювання характерні ці симптоми?

Ревматизм

Гепатит

Подагра

Жовтяниця

Цукровий діабет

2320 / 6854
До офтальмолога звернувся пацієнт зі скаргами на різі в очах. При обстеженні встановлена ерозія рогівки - відсутність поверхневого і шипуватого шарів епітелію. Які клітини будуть забезпечувати регенерацію ушкодженого епітелію?

Клітини рогового шару

Клітини блискучого шару

Клітини поверхневого шару

Базальні

Клітини зернистого шару

2321 / 6854
До інфекційного відділення госпіталізували хворого з ознаками загальної слабкості, сильними головними і м’язовими болями, високою температурою, гіперемією обличчя. Встановлено, що тиждень тому хворий відпочивав біля озера. Лікар запідозрив лептоспіроз. Яким чином лептоспіри могли потрапити до організму хворого?

З їжею

З повітрям

З ґрунтом

З водою

Через предмети вжитку

2322 / 6854
У дорослої людини системний артеріальний тиск знизився з 120/70 до 90/50 мм рт.ст., що викликало рефлекторне звуження судин. У якому з зазначених органів звуження судин буде найменшим?

Кишечник

Серце

Шкіра

Печінка

Скелетні м’язи

2323 / 6854
Під час хірургічного втручання на тонкій кишці у людини можлива рефлекторна зупинка серця. Які рецептори в міокарді необхідно заблокувати, щоб попередити зупинку?

β-адренорецептори

H-холінорецептори

M-холінорецептори

Пуринові рецептори

α-адренорецептори

2324 / 6854
У лікарню наприкінці робочого дня доставлений робітник 'гарячого' цеху, який скаржиться на головний біль, запаморочення, нудоту, загальну слабкість. Об’єктивно: свідомість збережена, шкірні покриви гіперемовані, сухі, гарячі на дотик. ЧСС- 130/хв. Дихання часте, поверхневе. Яке порушення процесів терморегуляції, найбільш імовірно, виникло у людини в даній ситуації?

Зниження теплопродукції без змін тепловіддачі

Посилення тепловіддачі і зниження теплопродукції

Посилення теплопродукції без змін тепловіддачі

Посилення тепловіддачі і теплопродукції

Зниження тепловіддачі

2325 / 6854
Експериментальне вивчення нового медичного препарату виявило блокуючий ефект на збирання білків-тубулінів, які є основою веретена поділу в клітинах, що діляться. Який етап клітинного циклу порушується цим препаратом?

Синтетичний період

Телофаза мітозу

Анафаза мітозу

Постмітотичний період інтерфази

Премітотичний період інтерфази

2326 / 6854
У чоловіка 35-ти років під час тривалого бігу виникла гостра серцева недостатність. Які зміни іонного складу спостерігаються у серцевому м’язі при цьому стані?

Зменшення в позаклітинному просторі іонів K+ і Mg2+

Накопичення в клітинах міокарда іонів K+ і Mg2+

Зменшення в клітинах міокарда іонів Na+ і Ca2+

Збільшення в позаклітинному просторі іонів Na+ і Ca2+

Накопичення в клітинах міокарда іонів Na+ і Ca2+

2327 / 6854
Під час аналізу ЕКГ людини з’ясовано, що у другому стандартному відведенні від кінцівок зубці P позитивні, їхня амплітуда 0,1 mV (норма - 0,05-0,25 mV), тривалість - 0,1 с (норма - 0,07-0,10 с). Вірним є висновок, що у передсердях нормально відбувається процес:

Збудження

Розслаблення

Деполяризації

Реполяризації

Скорочення

2328 / 6854
У юнака 18-ти років діагностовано хворобу Марфана. При дослідженні встановлено: порушення розвитку сполучної тканини, будови кришталика ока, аномалії серцевосудинної системи, арахнодактилія. Яке генетичне явище зумовило розвиток цієї хвороби?

Комплементарність

Неповне домінування

Плейотропія

Кодомінування

Множинний алелізм

2329 / 6854
Інозитолтрифосфати в тканинах організму утворюються в результаті гідролізу фосфатидилінозитолдифосфатів і відіграють роль вторинних посередників (месенджерів) в механізмі дії гормонів. Їхній вплив у клітині спрямований на:

Гальмування фосфодіестерази

Активацію протеїнкінази А

Вивільнення іонів кальцію з клітинних депо

Гальмування протеїнкінази С

Активацію аденілатциклази

2330 / 6854
Батьки дитини 3-х років звернули увагу на потемніння кольору його сечі при відстоюванні. Об’єктивно: температура у нормі, шкірні покриви чисті, рожеві, печінка не збільшена. Назвіть імовірну причину даного стану:

Фенілкетонурія

Алкаптонурія

Гемоліз

Синдром Іценка-Кушінга

Подагра

2331 / 6854
Хворий звернувся до лікаря зі скаргами на гнійничкові висипання на шкірі кінцівок. Який антисептик необхідно призначити хворому?

Гепарин

Сибазон

Преднізолон

Інсулін

Розчин йоду спиртовий

2332 / 6854
Після тривалого фізичного навантаження під час заняття з фізичної культури у студентів розвинулась м’язова крепатура. Причиною її виникнення стало накопичення у скелетних м’язах молочної кислоти. Вона утворилась після активації в організмі студентів:

Гліколізу

Ліполізу

Пентозофосфатного циклу

Глікогенезу

Глюконеогенезу

2333 / 6854
Під дією негативних чинників довкілля порушена функція міосателітоцитів. Зміну якої функції всього м’язового волокна слід очікувати в даному випадку?

Трофіка

Скорочення

Скоротливий термогенез

Регенерація

Розслаблення

2334 / 6854
У хворого сечокам’яна хвороба. При видаленні конкременту з правого сечоводу хірург розрізав стінку сечоводу. В яке анатомічне утворення потрапить сеча?

Правий брижовий синус

Прямокишково-міхурове заглиблення

Лівий бічний канал

Заочеревинний простір

Правий бічний канал

2335 / 6854
У дитячому колективі проведено планову вакцинацію проти кору. Яким методом можна перевірити ефективність проведеної вакцинації?

Вірусологічний

Серологічний

Алергопроба

Біологічний

Вірусоскопічний

2336 / 6854
У тварини збільшений тонус м’язів-розгиначів. Це є наслідком посиленої передачі інформації до мотонейронів спинного мозку такими низхідними шляхами:

Медіальні кортикоспінальні

Латеральні кортикоспінальні

Руброспінальні

Вестибулоспінальні

Ретикулоспінальні

2337 / 6854
Хвора 28-ми років потрапила до інфекційної лікарні з приводу пожовтіння шкіри, склер, слизових оболонок. Лабораторно встановлене підвищення рівня прямого білірубіну у крові. В сечі виявлений уробіліноген і білірубін. Для якого з перелічених захворювань характерні такі зміни?

Туберкульоз нирки

Паренхіматозна жовтяниця

Механічна жовтяниця

Інфаркт нирки

Гемолітична жовтяниця

2338 / 6854
У новонародженої дитини спостерігаються: судоми, блювання, жовтяниця, специфічний запах сечі. Лікар-генетик висловив підозру про спадкову хворобу обміну речовин. Який метод дослідження необхідно використати для постановки точного діагнозу?

Популяційно-статистичний

Дерматогліфіка

Цитогенетичний

Близнюковий

Біохімічний

2339 / 6854
У дитини спостерігається затримка фізичного та розумового розвитку, глибокі порушення з боку сполучної тканини внутрішніх органів; у сечі виявлено кератансульфати. Обмін яких речовин порушений?

Гіалуронова кислота

Глікозаміноглікани

Еластин

Фібронектин

Колаген

2340 / 6854
У деяких анаеробних бактерій піруват, що утворюється внаслідок гліколізу, перетворюється на етиловий спирт (спиртове бродіння). У чому біологічний сенс цього процесу?

Утворення АТФ

Забезпечення клітини НАДФН

Утворення лактату

Поповнення фонду НАД+

Утворення АДФ

2341 / 6854
У хворого, який довготривало приймав преднізолон, в результаті відміни препарату виникло загострення захворювання, зниження артеріального тиску, слабкість. З чим можна пов’язати ці прояви?

Кумуляція препарату

Виникнення недостатності кори наднирників

Звикання до препарату

Гіперпродукція АКТГ

Сенсибілізація до препарату

2342 / 6854
В експерименті необхідно оцінити рівень збудливості тканини. Для цього доцільно визначити:

Критичний рівень деполяризації

Тривалість ПД

Амплітуду ПД

Поріг деполяризації

Потенціал спокою

2343 / 6854
Працівник соціальної служби після тривалого спілкування з людиною без визначеного місця проживання захворів на туберкульоз. До складу комплексного лікування було включено напівсинтетичний антибіотик широкого спектру дії. Вкажіть препарат:

Рифампіцин

Цефотаксим

Ампіцилін

Еритроміцин

Лінкоміцин

2344 / 6854
При хворобі Вільсона-Коновалова порушується транспорт міді, що призводить до накопичення цього металу в клітинах мозку та печінки. З порушенням синтезу якого білку це пов’язано?

Транскобаламін

Сидерофілін

Гаптоглобін

Металотіонеїн

Церулоплазмін

2345 / 6854
Хворий впродовж трьох років безрезультатно лікувався з приводу значного зниження кислотності шлункового соку. Його пригнічувала поява на білизні, постелі члеників, що рухались і самостійно виповзали з анального отвору. Який найбільш імовірний діагноз?

Цистицеркоз

Гіменолепідоз

Теніаринхоз

Теніоз

Опісторхоз

2346 / 6854
При нестачі біотину спостерігається порушення синтезу вищих жирних кислот. Утворення якого із зазначених метаболітів може бути порушено при цьому?

Малоніл КоА

Аланін

Серотонін

Сукциніл КоА

Піруват

2347 / 6854
У чоловіка 60-ти років після інсульту настав тривалий сон. Ураження яких структур ЦНС найбільш імовірно призвело до цього стану?

V-IX пари черепних нервів

Чорна субстанція

Висхідна частина РФ

Мозочок

Прецентральна звивина

2348 / 6854
У чоловіка виявлене захворювання, яке зумовлене домінантним геном, локалізованим у Ххромосомі. У кого із дітей буде це захворювання, якщо дружина здорова?

Тільки у синів

У половини дочок

У всіх дітей

У половини синів

Тільки у дочок

2349 / 6854
Недостатність в організмі мікроелементу селену проявляється кардіоміопатією. Імовірною причиною такого стану є зниження активності такого селенвмісного ферменту:

Цитохромоксидаза

Глутатіонпероксидаза

Каталаза

Лактатдегідрогеназа

Сукцинатдегідрогеназа

2350 / 6854
Недостатність в організмі лінолевої та ліноленової кислот призводить до ушкоджень шкіри, випадіння волосся, сповільненого загоювання ран, тромбоцитопенії, зниження опірності до інфекційних захворювань. Порушення синтезу яких речовин найімовірніше зумовлює вказані симптоми?

Катехоламіни

Інтерлейкіни

Інтерферони

Ейкозаноїди

Кортикостероїди

2351 / 6854
Людина потрапила у крижану воду й швидко загинула в результаті різкого переохолодження. Це відбулося тому, що в даному випадку значно збільшилась віддача тепла організмом таким шляхом:

Теплопроведення і радіація

Радіація

Теплопроведення

Конвекція

2352 / 6854
Хворий 39-ти років з алкогольним цирозом печінки скаржиться на задишку, загальну слабкість. Встановлено зниження артеріального тиску, розширення поверхневих вен передньої стінки живота, спленомегалію. Яке порушення гемодинаміки спостерігається у хворого?

Тотальна серцева недостатність

Колапс

Недостатність лівого шлуночка серця

Синдром портальної гіпертензії

Недостатність правого шлуночка серця

2353 / 6854
У жінки, що тривало приймала антибіотики з приводу кишкової інфекції, розвинулось ускладнення з боку слизової порожнини рота у вигляді запального процесу і білого нальоту, у якому під час бактеріологічного дослідження були виявлені дріжджеподібні грибки Candida albicans. Який з перерахованих препаратів показаний для лікування цього ускладнення?

Бісептол

Тетрациклін

Поліміксин

Фуразолідон

Флуконазол

2354 / 6854
До лікаря акушера-гінеколога звернулась вагітна жінка, у якої діагностували мегалобластну анемію. Який з нижченаведених засобів доцільно призначити?

Стрептокіназа

Пентоксил

Ілауцин

Метилурацил

Ціанокобаламін

2355 / 6854
Хвора 45-ти років звернулась із скаргами на облисіння. При огляді: шкіра голови плямисто-коричневого кольору, малорухлива, тоненька, щільна, вкрита роговими лусочками. При гістологічному дослідженні - гіперкератоз, в дермі склероз, периваскулярні лімфомакрофагальні інфільтрати, атрофія потових та сальних залоз. У крові виявлені LE-клітини. Який найбільш імовірний діагноз?

Вузликовий періартеріїт

Системний червоний вовчак

Ревматизм

Ревматоїдний артрит

Системна склеродермія

2356 / 6854
При дослідженні гостроти слуху в коваля виявили втрату слуху на 50% у діапазоні низьких частот і майже нормальну гостроту слуху в діапазоні високих частот. Порушення яких структур слухової системи призвело до такого стану?

М’язи середнього вуха

Барабанна перетинка

Середня частина кортієвого органу

Кортієв орган - ближче до гелікотреми

Кортієв орган - ближче до овального віконця

2357 / 6854
При відборі для ревакцинації вакциною БЦЖ у школяра поставлено пробу Манту, яка виявилася негативною. Результат проби свідчить про такі особливості імунітету до туберкульозу:

Наявність клітинного імунітету

Відсутність клітинного імунітету

Відсутність гуморального імунітету

Відсутність антитоксичного імунітету

Наявність гуморального імунітету

2358 / 6854
Чоловік звернувся до лікаря з приводу безпліддя. Має високий зріст, зниження інтелекту, недорозвинення статевих залоз. У епітелії слизової оболонки порожнини рота виявлений статевий хроматин (1 тільце Барра). Про яку патологію можна думати?

Синдром Іценка-Кушинга

Синдром Ді-Джорджи

Синдром Клайнфельтера

Адреногенітальний синдром

Акромегалія

2359 / 6854
Який стан може розвинутися через 15-30 хвилин після повторного введення антигену внаслідок підвищеного рівня антитіл, переважно IgE, які адсорбуються на поверхні клітин-мішеней - тканинних базофілів (тучних клітин) та базофілів крові?

Анафілаксія

Антитіло-залежна цитотоксичність

Сироваткова хвороба

Гіперчутливість уповільненого типу

Імунно-комплексна гіперчутливість

2360 / 6854
У жінки 22-х років через 5 годин після вживання морепродуктів на шкірі тулуба та дистальних відділів кінцівок з’явились маленькі сверблячі папули, які частиною зливаються між собою. Через добу висипка самовільно зникла. Назвіть механізм гіперчутливості, що полягає в основі даних змін:

Атопія (місцева анафілаксія)

Імунокомплексна гіперчутливість

Клітинна цитотоксичність

Системна анафілаксія

Антитілоопосередкований клітинний цитоліз

2361 / 6854
Хворий на гіпертонічну хворобу разом з безсольовою дієтою та з антигіпертензивними засобами, довгий час приймав гідрохлортіазид, що зумовило порушення електролітного балансу. Яке порушення внутрішнього середовища виникло у хворого?

Гiперкалiємiя

Метаболічний ацидоз

Гiпермагнiємiя

Гіпохлоремічний алкалоз

Збільшення об’єму циркулюючої крові

2362 / 6854
У 49-річної жінки після видалення лімфатичних вузлів правої аксилярної ділянки з приводу раку грудної залози через півроку виявлено збільшення правої верхньої кінцівки в об’ємі та значне її ущільнення, гладка та напружена шкіра, через щілиноподібні дефекти якої на поверхню витікає прозора рідина. Діагностуйте вид порушення лімфообігу:

Гостра загальна лімфедема

Хронічна вроджена місцева лімфедема

Хронічна набута місцева лімфедема

Гостра місцева лімфедема

Хронічна загальна лімфедема

2363 / 6854
У сироватці хворого виявлений імуноферментним методом HBsAg. При якому захворюванні виявлення даного ангигену має діагностичне значення?

Вірусний гепатит В

Натуральна віспа

Сказ

Кір

ВІЛ

2364 / 6854
При повторному введенні алергену починається виділення гістаміну тучними клітинами крові. До якого рівня реактивності відноситься така відповідь організму?

Органний

Молекулярний

Системний

Клітинний

Субклітинний

2365 / 6854
В експерименті на кролику введення пірогеналу призвело до підвищення у тварини температури тіла. Яка з перерахованих речовин відіграє роль вторинного пірогену, що бере участь у механізмі виникнення лихоманкової реакції?

Піромен

Імуноглобулін

Брадикінін

Інтерлейкін-1

Гістамін

2366 / 6854
При авторадіографічному дослідженні епітелію тонкої кишки було виявлено, що його повне оновлення відбувається протягом 3-х діб за рахунок проліферації малодиференційованих клітин. Вкажіть їх локалізацію:

Дно крипт

Власна пластинка слизової оболонки

Бічна поверхня ворсинок

Основа ворсинок

Верхівка ворсинок

2367 / 6854
Лікар при дослідженні мазку крові у пацієнта з анемією встановив діагноз - спадкова гемолітична анемія Мінковського-Шофара. Виявлення у крові яких характерних клітин надало можливість лікарю встановити діагноз?

Мікросфероцити

Пойкілоцити

Анізоцити

Поліхроматофіли

Мегалоцити

2368 / 6854
У хворого з клінічними симптомами гіпотиреозу, щитоподібна залоза збільшена удвічі, при пальпації щільна, з горбистою поверхнею. При гістологічному дослідженні - поряд з атрофією фолікулів залози відмічається дифузна інфільтрація паренхіми лімфоцитами, плазматичними клітинами з утворенням фолікулів і посилене розростання сполучної тканини. Вкажіть найбільш імовірний діагноз:

Дифузний токсичний зоб

Ендемічний зоб

Зоб Хашімото

Фіброзний зоб

Спорадичний зоб

2369 / 6854
При загостренні ревматоїдного артриту хворому, в анамнезі якого супутній хронічний гастрит, призначений целекоксиб. Чим обумовлено зменшення побічної дії препарату на травний тракт?

Переважаюча стимуляція аденіла-тциклази

Пригнічення фосфоліпази А2

Переважаюче пригнічення циклооксигенази-1

Переважаюче пригнічення циклооксигенази-2

Пригнічення фосфодіестерази

2370 / 6854
В експерименті на кролі встановлено, що об’єм кисню, який споживається головним мозком за 1 хвилину, дорівнює об’єму CO2 , який виділяється клітинами мозку в кров. Це свідчить, що у клітинах головного мозку має місце:

Окиснення вуглеводів

Окиснення жирів

Окиснення білків

Гіпоксія

Гіпокапнія

2371 / 6854
При активації запального процесу, деяких аутоімунних та інфекційних захворюваннях у плазмі крові різко зростає рівень білків гострої фази. Який із наведених нижче білків здатний утворювати гель при охолодженні сироватки?

Кріоглобулін

Гаптоглобін

С-реактивний білок

а2-макроглобін

Церулоплазмін

2372 / 6854
Хворому поставлено діагноз газова гангрена. Після ідентифікації збудника досліджуваний матеріал необхідно знищити. Який метод слід використати?

Тиндалізація

Пастеризація

Кип’ятіння

Стерилізація текучою парою

Стерилізація парою під тиском

2373 / 6854
Фенілкетонурія - це захворювання, яке зумовлено рецесивним геном, що локалізується в аутосомі. Батьки є гетерозиготами за цим геном. Вони вже мають двох хворих синів і одну здорову доньку. Яка імовірність, що четверта дитина, яку вони очікують, народиться теж хворою?

50%

75%

0%

100%

25%

2374 / 6854
У пацієнта з підвищеним артеріальним тиском, тремором, тахікардією, була діагностовано доброякісна пухлина мозкової речовини наднирників. Гіперсекреція якого гормону викликає таку симптоматику?

Інсулін

Тироксин

Адреналін

Соматотропін

Глюкагон

2375 / 6854
В пробірку, що містить розчин NaCl 0,9%, додали краплю крові. Що відбудеться з еритроцитами?

Набухання

Зморшкування

Залишаться без змін

Осмотичний гемоліз

Біологічний гемоліз

2376 / 6854
До косметолога звернулася пацієнтка зі скаргами на появу чорних цяток на обличчі. Після обстеження було встановлено, що поява цяток пов’язана з порушенням виділення секрету сальних залоз. Який тип секреції характерний для цих залоз?

Макроапокриновий

Мерокриновий

Мікроапокриновий

Голокриновий

Мерокриновий та мікроапокриновий

2377 / 6854
У пацієнта з хронічним захворюванням нирок розвинулась ниркова недостатність. Який з показників найбільш імовірно свідчить про порушення реабсорбції в канальцях в даному випадку?

Зниження кліренсу

Лейкоцитурія

Гематурія

Гіперазотемія

Гіпо- та ізостенурія

2378 / 6854
У людей, які постійно проживають в гірській місцевості, адаптація до 'кисневого голодування' здійсню ється шляхом полегшеної віддачі кисню гемоглобіном внаслідок:

Зниження температури крові

Зростання парціального тиску CO2

Підвищеного утворення 2,3- дифосфогліцерату в еритроцитах

Підвищення pH крові

Зниженого утворення 2,3- дифосфогліцерату в еритроцитах

2379 / 6854
У хворого 40-ка років ознаки гірської хвороби: запаморочення, задишка, тахікардія, рН крові - 7,50, pCO2 -30 мм рт.ст., зсув буферних основ +4 ммоль/л. Яке порушення кислотноосновного стану має місце?

Негазовий алкалоз

Газовий алкалоз

Негазовий ацидоз

Газовий ацидоз

Видільний ацидоз

2380 / 6854
Після ремонту автомобіля в закритому приміщенні при працюючому двигуні у чоловіка з’явилися задишка, запаморочення, акроціаноз, частота дихання 24-26/хв. Газовий склад крові: pO2 - 60 мм рт.ст., pCO2 - 30 мм рт.ст.; у крові наявний карбоксигемоглобін. Про який вид гіпоксії можна думати?

Респіраторна

Циркуляторна

Гемічна

Гіпоксична

Тканинна

2381 / 6854
При обстеженні у хворого виявлене порушення чутливості шкіри в ділянці передньої поверхні шиї. Який нерв уражений?

Малий потиличний

Великий вушний

Шийна петля

Поперечний нерв шиї

Надключичні

2382 / 6854
У вагітної жінки 26-ти років після тривалого блювання було зареєстровано зниження об’єму циркулюючої крові. Про яку зміну загальної кількості крові може йти мова?

Проста гіповолемія

Поліцитемічна гіповолемія

Поліцитемічна гіперволемія

Олігоцитемічна гіповолемія

Олігоцитемічна гіперволемія

2383 / 6854
У хворої дитини гінгівіт, спричинений анаеробною інфекцією. Яку групу протимікробних засобів потрібно призначити для лікування?

Нітрофурани

Поліміксини

Аміноглікозиди

Сульфаніламіди

Нітроімідазоли

2384 / 6854
Пацієнту, який знаходився в клініці з приводу пневмонії, ускладненої плевритом, у складі комплексної терапії вводили преднізолон. Протизапальна дія цього синтетичного глюкокортикоїда пов’язана з блокуванням вивільнення арахідонової кислоти шляхом гальмування такого ферменту:

Пероксидаза

Циклооксигеназа

Фосфоліпаза А2

Ліпоксигеназа

Фосфоліпаза C

2385 / 6854
Встановлено, що в клітинах організмів відсутні мембранні органели та їх спадковий матеріал не має нуклеосомної організації. Що це за організми?

Віруси

Прокаріоти

Найпростіші

Аскоміцети

Еукаріоти

2386 / 6854
У постраждалого в автомобільній аварії припинилося грудне дихання при збереженні діафрагмального. На якому рівні найбільш імовірно пошкоджено спинний мозок?

VI-VII шийні сегменти

XI-XII грудні сегменти

I-II поперекові сегменти

I-II шийні сегменти

I-II крижові сегменти

2387 / 6854
Хлопчик на другому році життя став часто хворіти на респіраторні захворювання, стоматити, гнійничкові ураження шкіри. Навіть невеликі пошкодження ясен і слизової ускладнюються запаленням, що протікає тривало. Встановлено, що у крові дитини практично відсутні імуноглобуліни усіх класів. Зниження функціональної активності якої клітинної популяції лежить в основі описаного синдрому?

Макрофаги

Т-лімфоцити

NK-лімфоцити

В-лімфоцити

Нейтрофіли

2388 / 6854
На електронній фотографії представлена органела, що являє собою великий поліпротеазний комплекс, що складається з трубкоподібної та двох регуляторних частин, які розташовані на обох кінцях органели. Остання виконує функцію протеолізу. Назвіть цю органелу:

Комплекс Гольджі

Рибосома

Центріоль

Включення

Протеасома

2389 / 6854
У хворого після перенесеної черепно-мозкової травми порушений акт ковтання. Який відділ мозку постраждав?

Кінцевий мозок

Таламус

Довгастий мозок

Середній мозок

Проміжний мозок

2390 / 6854
Дитина 6-ти років знаходиться на стаціонарному лікуванні з діагнозом алергічного риніту. В крові: зміни в лейкоцитарній формулі. Кількість яких клітин лейкоцитарного ряду може бути збільшена?

Базофіли

В-лімфоцити

Т-лімфоцити

Еозинофіли

Нейтрофіли

2391 / 6854
У хворого 69-ти років на шкірі в ділянці нижньої повіки з’явилося невелике бляшкоподібне утворення з наступним виразкуванням, яке було оперативно видалене. При мікроскопічному дослідженні утворення: в дермі шкіри комплекси з атипових епітеліальних клітин; периферії клітини розташовані перпендикулярно до базальної мембрани. Клітини темні, призматичної полігональної форми з гіперхромними ядрами з частими мітозами. Іноді зустрічаються утворення, подібні до волосяного фолікула. Яка гістологічна форма рака у хворого?

Базально-клітинний рак

Аденокарцинома

Плоскоклітинний рак без ороговіння

Недиференційований рак

Плоскоклітинний рак з ороговінням

2392 / 6854
Анатомічний мертвий простір - це частина повітря, яка залишається в повітроносних шляхах після видиху. В якій із наведених нижче ситуацій відбудеться зменшення анатомічного мертвого простору?

Накладання трахеостоми

Дихання через рот

Поворот лежачого пацієнта на лівий бік

Нахил голови вперед

Поворот лежачого пацієнта на правий бік

2393 / 6854
До кардіологічного відділення надійшов хворий з інтенсивним болем, який зумовлений інфарктом міокарда. Для купірування болю було вирішено потенціювати дію анальгетика нейролептиком. Який з перерахованих нейролептиків найбільш придатний у даному випадку?

Галоперидол

Сульпірид

Дроперидол

Аміназин

Трифтазин

2394 / 6854
У 12-річної дитини непереносимість ряду харчових продуктів. Їх вживання викликає алергічну реакцію у вигляді висипань на шкірі, що сверблять. Який протигістамінний засіб слід призначити, щоб не заважати шкільним заняттям дитини?

Диклофенак

Еуфілін

Димедрол

Ефедрин

Лоратадин

2395 / 6854
У хворого на слизовій оболонці ясен виразка овальної форми з припіднятими краями хрящоподібної щільності. Дно виразки м’ясисто-червоного забарвлення з нашаруваннями сірого кольору. При мікроскопічному дослідженні - проліферація ендотелію дрібних судин, периваскулярна лімфоплазмоцитарна інфільтрація. Про яке захворювання йдеться?

Ерозивно-виразкова лейкоплакія

Травматична виразка

Сифіліс

Виразково-некротичний гінгівіт

Виразка-рак

2396 / 6854
Стоматолог призначив пацієнту препарати кальцію для профілактики карієсу. Лікар не знав, що пацієнт хворіє на хронічну серцеву недостатність і приймає серцеві глікозиди. Що відбудеться в результаті поєднання препаратів кальцію і серцевих глікозидів?

Прискориться виведення серцевих глікозидів

Збільшиться активність і токсичність серцевих глікозидів

Уповільниться метаболізм серцевих глікозидів

Знизиться активність і токсичність серцевих глікозидів

Препарати не взаємодіють

2397 / 6854
Тривале лікування гіпофункції щитоподібної залози спричинило загальну дистрофію, карієс зубів, тахікардію, тремор кінцівок. Який лікарський засіб викликав зазначені побічні ефекти?

Паратиреоїдин

Тирокальцитонін

Хумулін

Преднізолон

L-тироксин

2398 / 6854
У жінки встановлено діагноз - рак шийки матки. З яким вірусом може бути асоційована ця патологія?

Varicella-Zoster вірус

Папілома вірус

Аренавірус

Вірус простого герпеса тип 2

Цитомегаловірус

2399 / 6854
У хворого хлопчика 12-ти років вміст холестерину в сироватці крові до 25 ммоль/л. В анамнезі - спадкова сімейна гіперхолестеринемія, причиною якої є порушення синтезу білків-рецепторів до:

Ліпопротеїнів дуже низької щільності

Ліпопротеїнів проміжної щільності

Ліпопротеїнів низької щільності

Хіломікронів

Ліпопротеїнів високої щільності

2400 / 6854
При дослідженні зовнішнього дихання лікар попросив пацієнта здійснити максимально глибокий видих після максимально глибокого вдиху для визначення такого показника:

Резервний об’єм видиху

Функціональна залишкова ємність

Загальна ємність легень

Життєва ємність легень

Киснева ємність крові

2401 / 6854
Хворий 47-ми років з діагнозом вогнищевий туберкульоз верхньої долі правої лєгені, в скиді комбінованої терапії одержує ізоніазид. Через деякий час пацієнт почав пред’являти скарги на м’язову слабкість, зниження шкірної чутливості, порушення зору, координації рухів. Який вітамінний препарат доцільно використати для усунення даних явищ?

Вітамін D

Вітамін В6

Вітамін В12

Вітамін C

Вітамін A

2402 / 6854
Хворий 58-ми років впродовж 9-ти років хворіє на цукровий діабет, отримує для корекції гіперглікемії інсулін-семіленте. 10 днів тому почав лікування гіпертонічної хвороби анаприліном. Через годину після прийому антигіпертензивного препарату розвинулась гіпоглікемічна кома. Який механізм виникнення гіпоглікемії за умови прийому анаприліну?

Зменшення періоду напіввиведення глюкагону

Збільшення біодоступності інсуліну-семіленте

Зменшення всмоктування глюкози

Збільшення періоду напіввиведення інсуліну-семіленте

Пригнічення глікогенолізу

2403 / 6854
Похідні птерину (аміноптерин і метотрексат) - є конкурентними інгібіторами дигідрофолатредуктази, внаслідок чого вони пригнічують регенерацію тетрагідрофолієвої кислоти з дигідрофолату. Ці лікарські засоби призводять до гальмування міжмолекулярного транспорту одновуглецевих груп. Біосинтез якого полімеру при цьому пригнічується?

Гомополісахариди

ДНК

Гангліозиди

Глікозаміноглікани

Білок

2404 / 6854
При підозрі на туберкульоз хворій дитині зробили пробу Манту Через 24 години у місці введення алергену з’явились припухлість, гіперемія і болісність. Які основні компоненти визначають цю реакцію організму?

Макрофаги, В-лімфоцити і моноцити

Мононуклеари, Т-лімфоцити і лімфокіни

Гранулоцити, Т-лімфоцити і IgG

В-лімфоцити, IgM

Плазматичні клітини, Т-лімфоцити і лімфокіни

2405 / 6854
Біохімічний аналіз сироватки крові пацієнта з гепатолентикулярною дегенерацією (хвороба Вільсона-Коновалова) виявив зниження вмісту церулоплазміну. У цього пацієнта в сироватці крові буде підвищена концентрація таких іонів:

Фосфор

Мідь

Кальцій

Натрій

Калій

2406 / 6854
В процесі метаболізму в організмі людини виникають активні форми кисню, у тому числі супероксидний аніон-радикал О2 . Цей аніон інактивується за допомогою ферменту:

Супероксиддисмутаза

Глутатіонпероксидаза

Пероксидаза

Каталаза

Глутатіонредуктаза

2407 / 6854
Хлопчик 10-ти років знаходиться у лікарні з підозрою на харчову токсикоінфекцію. При посіві фекалій хворого на середовище Ендо виросла велика кількість безбарвних колоній. Який мікроорганізм можна з найбільшою імовірністю ВИКЛЮЧИТИ з числа можливих збудників захворювання?

Proteus vulgaris

Salmonella enteritidis

Escherichia coli

Yersinia enterocolitica

Pseudomonas aeruginosa

2408 / 6854
Чоловік 23-х років після ДТП надійшов до лікарні у важкому стані із черепно-мозковою травмою. Дихання характеризується судомним тривалим вдихом, який переривається коротким видихом. Для якого типу дихання це характерно?

Апнейстичне

Кусмауля

Біота

Чейн-Стокса

Гаспінг-дихання

2409 / 6854
У хлопчика 2-х років з вираженим геморагічним синдромом відсутній антигемофільний глобулін А (фактор VIII) у плазмi крові. Яка фаза гемостазу первинно порушена у цього хворого?

Внутрішній механізм активації протромбінази

Перетворення протромбіну в тромбін

Ретракція кров’яного згустку

Зовнішній механізм активації протромбінази

Перетворення фібриногену в фібрин

2410 / 6854
Щуру в плевральну порожнину введено 0,5 мл повітря. Який тип недостатності дихання виникає в даному випадку?

Обструктивне порушення альвеолярної вентиляції

Дисрегуляторне порушення альвеолярної вентиляції

Рестриктивне порушення альвеолярної вентиляції

Перфузійний

Дифузійний

2411 / 6854
У хворого виявлено протозойне захворювання, при якому вражений головний мозок і спостерігається втрата зору. У крові знайдені одноклітинні півмісяцевої форми з загостреним кінцем. Збудником цього захворювання є:

Амеба

Лейшманія

Трихомонада

Токсоплазма

Лямблія

2412 / 6854
Юнак 17-ти років звернувся до медико-генетичної консультації з приводу відхилень у фізичному і статевому розвитку. При мікроскопії клітин слизової оболонки рота виявлене одне тільце Барра. Вкажіть найбільш імовірний каріотип юнака:

47,18+

47, XYY

47, XXY

45, Х0

47,21+

2413 / 6854
У спортсмена внаслідок довільної затримки дихання на 40 секунд зросли частота серцевих скорочень та системний артеріальний тиск. Реалізація яких механізмів регуляції зумовлює зміни показників?

Безумовні парасимпатичні рефлекси

Умовні симпатичні рефлекси

Умовні парасимпатичні рефлекси

Безумовні симпатичні рефлекси

2414 / 6854
Жінці 54-х років поставили попередній діагноз: інфаркт міокарда. Характерною ознакою даного захворювання є суттєве підвищення в крові активності такого ферменту:

Креатинфосфокіназа

Каталаза

Альфа-амілаза

Аргіназа

Г-6-ФДГ

2415 / 6854
Дитина попросила батька надути гумову кульку якомога більше за один видих. Яким з перелічених об’ємів повітря скористується батько?

Загальна ємність легень

Функціональна залишкова ємність

Життєва ємність легень

Ємність вдиху

Резервний об’єм вдиху

2416 / 6854
На розтині тіла хворого 43-х років, померлого від ревматизму, поверхня епікарду ворсиста, покрита плівками сірого кольору, що легко відділяються. Після їх відділення визначається набрякла повнокровна поверхня епікарду. Який найбільш імовірний діагноз?

Катаральний перикардит

Геморагічний перикардит

Проліферативний перикардит

Гнійний перикардит

Фібринозний перикардит

2417 / 6854
При розтині тіла померлого виявлена гіперплазія кісткового мозку плоских і трубчастих кісток (піоїдний кістковий мозок), спленомегалія (6 кг), гепатомегалія (5 кг), збільшення всіх груп лімфатичних вузлів. Якому захворюванню відповідають виявлені зміни?

Мієломна хвороба

Хронічний лімфолейкоз

Лімфогрануломатоз

Хронічний мієлолейкоз

Справжня поліцитемія

2418 / 6854
Хворий 61-го року страждає на цироз печінки. Варикозні розширення яких вен з системи порто-кавальних анастомозів можуть мати місце у хворого?

V subcostalis

V femoralis

V. circumflexa ilium profunda

V epigastrica superficialis

Vv. intercostales posteriores

2419 / 6854
Електрофоретичне дослідження сироватки крові хворого на пневмонію показало збільшення однієї з білкових фракцій. Вкажіть її:

β-глобуліни

α2 -глобуліни

α1 -глобуліни

Альбуміни

γ-глобуліни

2420 / 6854
Хворий 34-х років страждає на туберкульоз легень з дитинства. Йому призначена операція. Під час операції у хворого була видалена середня доля правої легені. Які сегменти були видалені?

Латеральний і медіальний

Верхній і передній

Задній і передній

Медіальний базальний і латеральний базальний

Верхній язичковий і нижній язичковий

2421 / 6854
У потерпілого 35-ти років встановлено гнійне запалення жовчного міхура. В який відділ очеревинної порожнини потрапить гній під час розриву жовчного міхура при його типовому положенні?

Печінкова сумка

Верхній дванадцятипалий закуток

Чепцева сумка

Передшлункова сумка

Лівий бічний канал

2422 / 6854
При мікроскопії мазка фекалій школяра виявлені жовто-коричневого кольору яйця з горбкуватою оболонкою. Якому гельмінту вони належать?

Аскарида

Гострик

Волосоголовець

Ціп’як карликовий

Стьожак широкий

2423 / 6854
В експерименті на тварині досліджують серцевий цикл. Закриті усі клапани серця. Якій фазі циклу відповідає такий стан?

Асинхронного скорочення

Швидкого наповнення

Ізометричного скорочення

Протодіастолічний період

Повільного наповнення

2424 / 6854
На основі лабораторного аналізу у хворого підтверджено діагноз - подагра. Для встановлення діагнозу було проведено визначення вмісту:

Сечовини в крові та сечі

Аміаку в сечі

Сечової кислоти в крові та сечі

Креатиніну в сечі

Залишкового азоту в крові

2425 / 6854
Ціаністий калій є отрутою, від якої смерть організму наступає миттєво. На які ферменти в мітохондріях діє ціанистий калій:

Цитохром В5

Флавінові ферменти

НАД+ - залежні дегідрогенази

Цитохромоксидаза (аа3)

Цитохром Р-450

2426 / 6854
Для лікування деяких інфекційних захворювань, викликаних бактеріями, застосовуються сульфаніламідні препарати, що блокують синтез фактора росту бактерій. Назвіть механізм їх дії:

Є алостеричними ферментами

Інгібують всмоктування фолієвої кислоти

Є алостеричними інгібіторами ферментів

Беруть участь в окисно-відновних процесах

Є антивітамінами параамінобензойної кислоти

2427 / 6854
У різнороба 55-ти років, який 28 років пропрацював на хімічному заводі в цеху зі шкідливими умовами виробництва, спостерігаються часті кровотечі із слизової оболонки носової порожнини. За рахунок яких артерій це відбувається?

Війчасті артерії

Передня мозкова артерія

Очна артерія

Передня і задня решітчасті артерії

Надочноямкова артерія

2428 / 6854
Після травми передньої поверхні верхньої третини передпліччя у хворого утруднення пронації, послаблення долонного згинання кисті та порушення чутливості шкіри 1-3 пальців на долоні. Який нерв ушкоджено?

n. medianus

n. ulnaris

n. radialis

n. cutaneus antebrachii medialis

n. musculocutaneus

2429 / 6854
Юнак 15-ти років доставлений до приймального відділєння у непритомному стань Об’єктивно: на зовнішні подразники хворий не реагує, дихання періодичне за типом Чейн-Стокса, зіниці звужені, зіничний рефлекс відсутній. Було встановлено, що дані симптоми обумовлені використанням морфіну. Назвіть засіб для антидотної терапії:

Унітіол

Апоморфіну гідрохлорид

Протаміну сульфат

Кальцію хлорид

Налоксон

2430 / 6854
При гістологічному дослідженні вузла у видаленій молочній залозі серед рясної строми виявлені різних розмірів і форми комплекси атипових поліморфних епітеліальних клітин з наявністю просвітів у центрі комплексів. Клітини з великими ядрами, збільшеним числом ядерець і ядерцевих організаторів, наявністю атипових мітозів. Який попередній діагноз?

Солідний рак

Плоскоклітинний незроговілий рак

Аденокарцинома

Недиференційований поліморфно-клітинний рак

Фіброаденома молочної залози

2431 / 6854
У чоловіка 63-х років, що багато років страждав на атеросклероз і переніс раніше інфаркт міокарда, розвинувся напад загрудинного болю. Хворий госпіталізований, через 3 дні, на фоні прогресуючої серцево-судинної недостатності, помер. Під час розтину тіла у ділянці задньої стінки лівого шлуночка і міжшлуночкової перегородки виявлена ділянка білого кольору близько 3 см у діаметрі, волокниста, западаюча, з чіткою межею. Прозектор трактував ці зміни як:

Вогнищевий кардіосклероз

Інфаркт міокарда

Ішемія міокарда

Міокардит

Дистрофія міокарда

2432 / 6854
23-х років вдень раптово підвищилася температура до 39,5°C і через 6 годин повернулася до норми. На другу добу напад повторився і температура досягла 41,5°C. Період апірексії настав через 8 годин. Який тип температурної кривої?

Виснажуючий

Постійний

Септичний

Переміжний

Послаблюючий

2433 / 6854
Під час оперативного втручання на фоні використання гігронію різко знизився артеріальний тиск. Представники яких груп лікарських препаратів можуть нормалізувати артеріальний тиск?

α-адреноблокатори

Н-холіноміметики

Гангліоблокатори

α-адреноміметики

M-холіноміметики

2434 / 6854
При глікогенозі - хворобі Гірке - порушується перетворення глюкозо-6-фосфату на глюкозу, що призводить до накопичення глікогену в печінці та нирках. Дефіцит якого ферменту є причиною захворювання?

Глікогенсинтетаза

Альдолаза

Глюкозо-6-фосфатаза

Гексокіназа

Фосфорилаза

2435 / 6854
При проведені наукового експерименту дослідник зруйнував структуру однієї з частин клітини, що порушило здатність клітини до поділу. Яка структура була зруйнована найбільш імовірно?

Центросома

Мікрофібрили

Глікокалікс

Пластичний комплекс

Мітохондрії

2436 / 6854
У хворого з дихальною недостатністю рН крові 7,35. Визначення рСO2 показало наявність гіперкапнії. При дослідженні рН сечі відзначається підвищення її кислотності. Яка форма порушення кислотноосновного стану в даному випадку?

Ацидоз метаболічний, компенсований

Алкалоз газовий, компенсований

Ацидоз газовий, компенсований

Ацидоз метаболiчний, декомпенсований

Алкалоз газовий, декомпенсований

2437 / 6854
При аутопсії тіла чоловіка середніх років, що тривало страждав на бронхоектатичну хворобу, виявлено, що обидва наднирника різко з6ільшені в об’ємі за рахунок кіркового шару. Наднирники щільні, бліді, сального вигляду. Мікроскопічно по ходу ретикулярної строми, у стінках судин - відкладання безструктурних, гомогенних, еозинофільних мас і конго-рот - позитивних мас. Який процес зумовив ці зміни?

Гіаліноз

Амілоїдоз

Ліпідоз

Фібриноїдне набухання

Мукоїдне набухання

2438 / 6854
При бактеріологічному дослідженні сечі хворого на пієлонефрит виділені мікроорганізми, що утворюють на м’ясо-пептонному агарі жовто-зелений пігмент і характерний запах. Як вони називаються?

Ешеріхії

Псевдомонади

Клебсієли

Протеї

Азотобактерії

2439 / 6854
Дитина 4-х років вдихнула ґудзик, який за допомогою бронхоскопу був видалений з правого головного бронху. Який епітелій бронху найбільш імовірно ушкоджений стороннім предметом?

Одношаровий багаторядний війчастий

Одношаровий плоский

Перехідний

Багатошаровий незроговілий

Одношаровий низькопризматичний

2440 / 6854
Жінка 53-х років хвора на гіпертонічну хворобу тривалий час лікувалася гіпотензивними засобами. Останнім часом стан її погіршився: з’явились нудота, набряк повік та губ, уртикарні висипання на шкірі, блювання, брадикардія, пронос. Який препарат викликає ці явища?

Клофелін

Бензогексоній

Дибазол

Анаприлін

Резерпін

2441 / 6854
У хворого 48-ми років, на тлі хронічного гломерулонефриту, розвинулася стійка артеріальна гіпертензія. Вкажіть групу найбільш ефективних лікарських засобів для лікування цього пацієнта:

а-адреноблокатори

Міотропні спазмолітики

Гангліоблокатори

Інгібітори ангіотензинперетворюючого ферменту

Антагоністи кальцію

2442 / 6854
Чоловік 54-х років госпіталізований до нейрохірургічного відділення зі скаргами на відсутність чутливості шкіри нижньої повіки, латеральної поверхні носа, верхньої губи. Лікар при огляді встановив запалення другої гілки трійчастого нерва. Через який отвір виходить із черепа ця гілка?

Овальний отвір

Рваний отвір

Остистий отвір

Верхня очноямкова щілина

Круглий отвір

2443 / 6854
Чоловік 64-х років із хронічним закрепом приймав щодня одне драже 'Бісакоділ'. Через деякий час для отримання ефекту він змушений був приймати по 2 драже. Як називається такий тип зміни дії лікарських речовин?

Ідіосинкразія

Звикання

Кумуляція

Лікарська залежність

Сенсибілізація

2444 / 6854
У чоловіка 22-х років високого росту та астенічної будови тіла з ознаками гіпогонадизму, гінекомастією та зменшеною продукцією сперми (азооспермія) виявлено каріотип 47 XXY. Який спадковий синдром супроводжується такою хромосомною аномалією?

Клайнфельтера

Віскотта-Олдрича

Тернера

Луї-Барра

Дауна

2445 / 6854
У товщі шкіри макроскопічно знайдена і видалена щільна пухлина, рухлива. При мікроскопії вона представлена хаотично розташованими пучками колагенових волокон з невеликою кількістю веретеноподі6них клітин. Яка пухлина видалена?

Ліпома

Іломус-ангіома

Меланома

Щільна фіброма

Лейоміома

2446 / 6854
Хвора 26-ти років звернулася до лікаря зі скаргами на появу у випорожненнях білих плоских рухливих утворів, які нагадують локшину. При лабораторному дослідженні виявлені членики з такою характеристикою: довгі, вузькі, з розміщеним поздовжньо каналом матки, яка має 17-35 бічних відгалужень з кожного боку. Який вид гельмінтів паразитує у кишечнику жінки?

Hymenolepis nana

Taenia solium

Echinococcus granulosus

Taeniarhynchus saginatus

Diphyllobothrium latum

2447 / 6854
У жінки 30-ти років при гістологічному дослідженні шийного лімфовузла виявлено порушення малюнка внаслідок розростання епітеліоїдних, лімфоїдних клітин і макрофагів з ядрами у вигляді підкови, в центрі деяких скупчень клітин - безструктурні ділянки блідо-рожевого кольору з уламками ядер. Для якого захворювання характерні такі зміни?

Сифіліс

Лімфогрануломатоз

Актиномікоз

Метастаз пухлини

Туберкульоз

2448 / 6854
В експерименті у тварини в результаті проведеного перетинання депресорного нерва та руйнування каротидних клубочків розвинулась стійка гіпертензія. З порушенням якої функції нервової системи пов’язане це явище?

Вища нервова діяльність

Сенсорна

Вегетативна

Рухова

Трофічна

2449 / 6854
У хворого 20-ти років видалена пухлина лобної частки правої півкулі головного мозку діаметром 5 см, яка була нечітко відмежована від довколишньої тканини. На розрізі - однорідного вигляду, гістологічно - складається із зіркоподібних клітин, численні відростки яких утворюють густі сплетіння. Яка пухлина була у хворого?

Олігодендрогліома

Епендімома

Астроцитома

Гангліоневрома

Хоріоїдпапілома

2450 / 6854
У хворого 20-ти років з жовтяницею встановлено: підвищення у плазмі крові вмісту загального білірубіну за рахунок непрямого (вільного), в калі та сечі - високий вміст стеркобіліну, рівень прямого (зв’язаного) білірубіну в плазмі крові в межах норми. Про який вид жовтяниці можна думати?

Паренхіматозна (печінкова)

Механічна

Хвороба Жильбера

Гемолітична

Жовтяниця немовлят

2451 / 6854
У хлопчика зі спадково обумовленими вадами зразу ж після народження спостерігався характерний синдром, який називають 'крик кішки'. У ранньому дитинстві малюк мав 'нявкаючий' тембр голосу. Під час дослідження каріотипу цієї дитини було виявлено:

Нестачу Х-хромосоми

Додаткову 21-у хромосому

Додаткову Y-хромосому

Додаткову Х-хромосому

Делецію короткого плеча 5-ї хромосоми

2452 / 6854
При визначенні групи крові за системою АВ0 за допомогою стандартних сироваток були отримані наступні результати: аглютинацію еритроцитів викликали сироватки I, II та III груп. Яка група досліджуваної крові?

Неможливо визначити

O(I)

AB(IV)

B(III)

A(II)

2453 / 6854
Жінка 68-ми років, що страждає на атеросклероз, госпіталізована до хірургічного відділення з приводу розлитого гнійного перитоніту. Під час операції діагностовано тромбоз брижових артерій. Яка найбільш імовірна причина перитоніту?

Немічний інфаркт

Геморагічний інфаркт

Ішемія ангіоспастична

Стаз

Ішемія компресійна

2454 / 6854
Для утворення транспортної форми амінокислот для синтезу білка необхідно:

Аміноацил-тРНК-синтетаза

м-РНК

ГТФ

Ревертаза

Рибосома

2455 / 6854
Перебуваючи у робочому відрядженні в одній із країн тропічної Африки, лікар зіткнувся зі скаргами місцевого населення з приводу хвороби дітей 10-14 років, що супроводжується стійкими лихоманками, які не мають правильного чергування, виснаженням, анемією, збільшенням печінки і селезінки. Враховуючи місцеві умови, що пов’язані з великою кількістю москітів, можна передбачити що це:

Токсоплазмоз

Вісцеральний лейшманіоз

Сонна хвороба

Хвороба Чагаса

Балантидіаз

2456 / 6854
Під час хірургічного втручання на органах черевної порожнини сталася рефлекторна зупинка серця. Де знаходиться центр рефлексу?

Проміжний мозок

Довгастий мозок

Середній мозок

Спинний мозок

Кора великих півкуль

2457 / 6854
У чоловіка 36-ти років розвинулася гарячка, яка супроводжувалася зміщенням установчої точки терморегуляційного центру на більш високий рівень, з послідовним чергуванням наступних стадій: Incrementi, fastigii, decrementi. При якому захворюванні можуть спостерігатися подібні зміни?

Цукровий діабет

Гостра пневмонія

Гіпертрофія міокарда

Ренальний діабет

Акромегалія

2458 / 6854
Одна з форм вродженої патології супроводжується гальмуванням перетворення фенілаланіну в тирозин. Біохімічною ознакою хвороби є накопичення в організмі деяких органічних кислот, зокрема:

Фенілпіровиноградна

Лимонна

Піровиноградна

Молочна

Глутамінова

2459 / 6854
У чоловіка 41-го року відзначаються періодичні напади серцебиття (пароксизми), сильне потовиділення, напади головного болю. При обстеженні виявлена гіпертензія, гіперглікемія, підвищення основного обміну, тахікардія. При якій патології наднирників спостерігається подібна картина?

Гіпофункція кори наднирників

Первинний альдостеронізм

Гіпофункція мозкового шару

Гіперфункція кори наднирників

Гіперфункція мозкового шару

2460 / 6854
У дитини 10-ти років через 2 тижні після перенесеної ангіни розвинувся нефритичний синдром (протеїнурія, гематурія, циліндрурія), що свідчить про ураження базальної мембрани клубочків нирок. Який найбільш імовірний механізм лежить в основі ушкодження базальної мембрани?

Реагіновий

Гранулематозний

Імунокомплексний

Антитільний

Цитотоксичний

2461 / 6854
Жінка 24-х років звернулася до медико-генетичної консультації з приводу оцінки ризику захворювання на гемофілію у її дітей. Її чоловік страждає на гемофілію. Під час збору анамнезу виявилося, що у сім’ї жінки не було випадків гемофілії. Вкажіть ризик народження хворої дитини:

25%

50%

75%

100%

Відсутній

2462 / 6854
При тиреотоксикозі підвищується продукція тиреоїдних гормонів Т3 та Т4, розвиваються схуднення, тахікардія, психічне збудження та інше. Як саме впливають тиреоїдні гормони на енергетичний обмін в мітохондріях клітин?

Роз‘єднують окислення та окисне фосфорилювання

Активують окисне фосфорилювання

Блокують дихальний ланцюг

Активують субстратне фосфорилювання

Блокують субстратне фосфорилювання

2463 / 6854
У людини внаслідок тривалого перебування у горах на висоті 3000 м над рівнем моря збільшилась киснева ємкість крові. Безпосередньою причиною цього є посилене утворення в організмі:

2,3-дифосфогліцерату

Катехоламінів

Карбгемоглобіну

Еритропоетинів

Лейкопоетинів

2464 / 6854
У хворого 45-ти років при рентгенологічному обстеженні була виявлена кила міжхребцевого диску грудного відділу хребетного стовпа. Який вид з’єднання між хребцями зазнав патологічних змін?

Геміартроз

Синсаркоз

Синостоз

Синхондроз

Діартроз

2465 / 6854
Чоловіку 44-х років за вимогою діагностичних тестів зробили лімфографію органів грудної порожнини. Хірург встановив, що пухлина вразила орган, з лімфатичних судин якого лімфа безпосередньо переходить у грудну протоку. Який це орган?

Лівий головний бронх

Трахея

Серце

Стравохід

Осердя

2466 / 6854
Виникнення нижчеперерахованих захворювань пов’язане із генетичними факторами. Назвіть патологію із спадковою схильністю:

Дальтонізм

Цукровий діабет

Фенілкетонурія

Серпоподібноклітинна анемія

Хорея Гентінгтона

2467 / 6854
Серологічна діагностика інфекційних захворювань заснована на специфічній взаємодії антитіл з антигенами. Як називається серологічна реакція, при якій високодисперсні антигени адсорбовані на еритроцитах?

Реакція гемадсорбції

Реакція преципітації

Реакція зв’язування комплементу

Реакція непрямої (пасивної) гемаглютинації

Реакція нейтралізації

2468 / 6854
Чоловік 42-х років з ураженням ліктьового нерва не може звести до серединної лінії II і V пальці. Функція яких м’язів при цьому порушена?

Тильні міжкісткові м’язи

Червоподібні м’язи

Короткий долонний м’яз

Відвідний м’яз мізинця

Долонні міжкісткові м’язи

2469 / 6854
На електронній мікрофотографії епідермісу шкіри серед клітин кубічної форми виділяються паросткові клітини, в цитоплазмі яких добре розвинутий апарат Гольджі, багато рибосом і меланосом. Назвіть ці клітини:

Клітини Меркеля

Меланоцити

Тканинні базофіли

Кератиноцити

Клітини Лангеганса

2470 / 6854
Потужність, що розвиває м’яз, недостатня для піднімання вантажу. Який вид скорочення м’яза має місце у даному випадку?

Концентричне

Ізотонічне

Тетанічне

Ексцентричне

Ізометричне

2471 / 6854
Через рік після субтотальної резекції шлунка з приводу виразки малої кривизни виявлені зміни в аналізі крові - анемія, лейко- і тромбоцитопенія, КП=1.3, наявність мегалобластів та мегалоцитів. Дефіцит якого фактору обумовив розвиток цієї анемії?

Іастрин

Фактор Касла

Муцин

Пепсин

Хлороводнева кислота

2472 / 6854
У мазку з нальоту на мигдаликах хворого з підозрою на дифтерію виявлено палички синього кольору з потовщеннями на полюсах. Який метод фарбування мазків було використано?

Грама

Пнса

Леффлера

Буррі

Нейссера

2473 / 6854
Хвора 66-ти років скаржиться на 6іль у гомілці, який підсилюється при ходьбі. Об’єктивно: набряк та почервоніння по ходу вени. Лікар призначив антикоагулянт прямої дії для місцевого вживання. Який препарат можна використати з цією метою?

Мазь троксевазинова

Мазь саліцилова

Мазь гепаринова

Тромбін

Мазь бутадіонова

2474 / 6854
У хлопчика 6-ти років відзначається змішана інвазія аскаридами та гостриками. Який протиглисний препарат слід призначити для одноразового прийому?

Насіння гарбуза

Аміноакрихін

Мебендазол

Фенасал

Піперазину адипінат

2475 / 6854
Дитина народилася в стані асфіксії. Який препарат необхідно ввести новонародженому для стимуляції дихання?

Лобелін

Прозерин

Етимізол

Атропін

Празозин

2476 / 6854
У пацієнта 38-ми років при прогресуванні стафілококового гнійного періодонтиту виникло гнійне запалення кістково-мозкових просторів альвеолярного відростка, а потім тіла нижньої щелепи. Мікроскопічно кісткові балки витончені, вогнища некрозу, кісткові секвестри, оточені сполучнотканинною капсулою. Який найбільш імовірний діагноз?

Пародонтома

Хронічний остеомієліт

Гострий остеомієліт

Хронічний фіброзний періостит

Гнійний періостит

2477 / 6854
Еритроцити людини не містять мітохондрій. Який основний шлях утворення АТФ в цих клітинах?

Окислювальне фосфорилювання

Креатинкіназна реакція

Аденілаткіназна реакція

Аеробний гліколіз

Анаеробний гліколіз

2478 / 6854
У чоловіка 50-ти років запалення яєчка ускладнилось його водянкою. Необхідне оперативне втручання. Яку з оболонок яєчка останньою розтинає хірург під час операції?

М ’ясиста оболонка

М’яз-підіймач яєчка

Внутрішня сім’яна фасція

Зовнішня сім’яна фасція

Парієтальний листок піхвової оболонки яєчка

2479 / 6854
У жінки 52-х років при обстеженні було виявлено зниження кількості еритроцитів у крові та підвищення рівня вільного гемоглобіну в плазмі крові (гемоглобінемія). КП- 0,85. Який вид анемії спостерігається у хворої?

Набута гемолітична

Гостра постгеморагічна

Анемія внаслідок порушення еритропоезу

Хронічна постгеморагічна

Спадкова гемолітична

2480 / 6854
У хворого 28-ми років тривале блювання призвело до зневоднення організму. Підвищена секреція якого гормону перш за все сприятиме збереженню води в організмі?

Кальцитонін

Вазопресин

Тироксин

Альдостерон

Соматостатин

2481 / 6854
При проведенні операції на тонкій кишці лікар виявив ділянку слизової оболонки, де на фоні колових складок була присутня поздовжня складка. Який відділ тонкої кишки має таку будову?

Pars descendens duodeni

Дистальний вiддiл ileum

Pars horizontalis duodeni

Початковий eiddrn jejunum

Pars ascendens duodeni

2482 / 6854
У хворого нормально забарвлений кал, у складі якого знаходиться велика кількість вільних жирних кислот. Причиною цього є порушення наступного процесу:

Жовчоутворення

Всмоктування жирів

Гідроліз жирів

Жовчовиділення

Секреція ліпаз

2483 / 6854
При термометрії встановлено, що температура відкритих ділянок шкіри на 1-1,5° нижче за температуру поруч розташованих ділянок, закритих одягом з натуральних тканин. Причиною цього є те, що одяг, перш за все, зменшує тепловіддачу таким шляхом:

Випаровування

Проведення

Радіація

Конвекція

2484 / 6854
На препараті м’якої мозкової оболонки виявляється судина, у стінці якої відсутня середня оболонка, зовнішня оболонка зрощена з оточуючою тканиною, внутрішня оболонка побудована із базальної мембрани та ендотелію. Що це за судина?

Вена волокнистого типу

Вена м’язового типу зі слабким розвитком м’язових елементів

Артеріола

Артерія змішаного типу

Артерія м’язового типу

2485 / 6854
В основі розвитку імунних і алергічних реакцій організмом застосовуються однакові механізми відповіді імунної системи на антиген. Визначте основну відмінність алергічних реакцій від імунних?

Шляхи потрапляння в організм

Кількість антигену, що потрапляє

Особливість будови антигенів

Розвиток пошкодження тканин

Спадкова схильність

2486 / 6854
Чоловік 38-ми років поступив до неврологічного відділення зі скаргами на погіршення пам’яті та розумової працездатності після перенесеної травми голови. Запропонуйте лікарський засіб для покращення метаболізму головного мозку:

Меридил

Анальгін

Кофеїн

Пірацетам (ноотропіл)

Сиднокарб

2487 / 6854
У хлопчика 8-ми років виник біль у горлі, підвищилась температура тіла. На 2-й день від початку захворювання виявлені висипання червоного кольору у вигляді дрібних, густо розташованих плям, величиною з макове зерно. Вони покривають все тіло, за винятком носо-губного трикутника. При огляді порожнини рота - в зіві яскраве почервоніння, мигдалики збільшені, язик малиново-червоний. Який найбільш імовірний діагноз?

Стрептококова ангіна

Дифтерія зіва

Скарлатина

Аденовірусна інфекція

Кір

2488 / 6854
Юнака 15-ти років вжалила бджола. На місці укусу визначається набряк, гіперемія, підвищення температури. Назвіть ініціальний патогенетичний фактор запального набряку:

Підвищення осмотичного тиску у вогнищі запалення

Підвищення проникності мікросудин

Порушення лімфовідтоку

Підвищення кров’яного тиску в капілярах

Зниження онкотичного тиску крові

2489 / 6854
Хворий 49-ти років був доставлений до лікарні в коматозному стані. В анамнезі - цукровий діабет. Об’єктивно: дихання Кусмауля, зниження артеріального тиску, у видихуваному повітрі запах ацетону. Після проведеної невідкладної терапії стан покращився. Який препарат було введено хворому?

Інсулін

Адреналін

Глібенкламід

Ізадрин

Букаркам

2490 / 6854
Катіонні глікопротеїни є основними компонентами слини привушних залоз. Які амінокислоти обумовлюють їх позитивний заряд?

Аспартат, глутамат, гліцин

Глутамат, валін, лейцин

Цистеїн, гліцин, пролін

Лізин, аргінін, гістидин

Аспартат, аргінін, глутамат

2491 / 6854
На мікропрепараті очного яблука плода спостерігається пошкодження рогівки. Частина якого зародкового листка була уражена в процесі ембріонального розвитку?

Ектодерма

Дерматом

Ентодерма

Мезодерма

Нефротом

2492 / 6854
Жінці 55-ти років для лікування ІХС був призначений β-адреноблокатор. Через деякий час у неї з’явився кашель, бронхоспазм. Якому з перелічених засобів притаманна така побічна дія?

Атенолол

Метопролол

Фенігідин

Анаприлін

Талінол

2493 / 6854
Студентка 22-х років звернулася до лікаря із скаргами на підвищення температури тіла до 38°C, слабкість, біль у горлі. Об’єктивно: язик вкритий білим нальотом. Які гістологічні структури язика беруть участь в утворенні цього нальоту?

Сполучнотканинна основа всіх сосочків язика

Епітелій листоподібних сосочків

Епітелій ниткоподібних сосочків

Епітелій жолобкуватих сосочків

Епітелій грибоподібних сосочків

2494 / 6854
Досліджуються рецептори, інформація від яких прямує до кори без участі таламусу. Які це рецептори?

Дотикові

Зорові

Слухові

Нюхові

Смакові

2495 / 6854
У здорових батьків, спадковість яких не обтяжена, народилась дитина з чисельними вадами розвитку. Цитогенетичний аналіз виявив у соматичних клітинах дитини трисомію за 13-ю хромосомою (синдром Патау). З яким явищем пов’язане народження такої дитини?

Соматична мутація

Хромосомна мутація

Порушення гаметогенезу

Домінантна мутація

Рецесивна мутація

2496 / 6854
На препараті представлено орган, вкритий сполучнотканинною капсулою, від якої відходять трабекули. В органі можна розрізнити кіркову речовину, де містяться лімфатичні вузлики та мозкову речовину, представлену тяжами лімфоїдних клітин. Який орган представлений на препараті?

Селезінка

Тимус

Червоний кістковий мозок

Мигдалики

Лімфатичний вузол

2497 / 6854
До біорегуляторів клітинних функцій ліпідної природи належать тромбоксани. Джерелом для синтезу цих сполук є:

Пальмітинова кислота

Пальмітоолеїнова кислота

Стеаринова кислота

Фосфатидна кислота

Арахідонова кислота

2498 / 6854
Жінка 25-ти років звернулася зі скаргами на дисменорею та безпліддя. При обстеженні виявлено: зріст жінки 145 см, недорозвинені вторинні статеві ознаки, на шиї крилоподібні складки. При цитологічному дослідженні в соматичних клітинах не виявлено тілець Барра. Який діагноз встановив лікар?

Синдром Шерешевського-Тернера

Синдром трисомії Х

Синдром Морріса

Синдром Клайнфельтера

2499 / 6854
У чоловіка 36-ти років хворого на туберкульоз легень непереносимість аміноглікозидного антибіотика амікацину. Який протитуберкульозний антибіотик можна включити до складу комплексної терапії у даному випадку?

Бензилпеніцилін

Стрептоміцин

Канаміцин

Рифампіцин

Амоксицилін

2500 / 6854
У жінки 60-ти років після емоційної реакції, яка була викликана гнівом, виник напад загрудинного болю. На ЕКГ були встановлені ознаки порушення коронарного кровообігу. Який вид порушень міг спричинити це явище?

Венозна гіперемія

Ангіоспастична ішемія

Артерiальна гіпєрємія

Справжній стаз

Венозний стаз

2501 / 6854
Для запобігання нападів гострого панкреатиту лікар призначив трасілол (контрікал, гордокс), який є інгібітором:

Еластази

Карбоксипептидази

Хімотрипсину

Гастриксину

Трипсину

2502 / 6854
Хворий 58-ми років помер від прогресуючої серцевої недостатності. На розтині: серце розширене у поперечнику, мляве, м’яз на розрізі нерівномірного кровонаповнення, пістрявий. При гістологічному дослідженні: у міокарді повнокров’я, у стромі лімфогістіоцитарні інфільтрати, що розсувають кардіоміоцити. Виявлені морфологічні зміни свідчать про:

Інфаркт міокарда

Кардіосклероз

Негнійний проміжний міокардит

Венозне повнокров’я

Жирову дистрофії міокарда

2503 / 6854
Хвора 39-ти років, з цукровим діабетом в анамнезі, госпіталізована до клініки у прекоматозному стані кетоацидотичного типу. Збільшення вмісту якого метаболіту до цього призвело?

Аспартат

Альфа-кетоглутарат

Ацетоацетат

Малонат

Цитрат

2504 / 6854
У хворих із синдромом набутого імунодефіциту (СНІД) різко знижується імунологічна реактивність, що проявляється розвитком хронічних запальних процесів, інфекційних захворювань, пухлинного росту. Клітини якого типу ушкоджує ВІЛ-інфекція, внаслідок чого знижується імунний захист?

Т8-ефектори

Т-супресори

В-лімфоцити

Природні кілери (NK)

Т4-хелпери

2505 / 6854
Під час аналізу електрокардіограми встановлено збільшення тривалості і амплітуди зубця S. Деполяризація якої ділянки серця порушена у хворого?

Базальні відділи шлуночків

Передсердя

Верхівка серця

Середня і нижня третина міжшлуно-чкової перегородки

Бокові стінки шлуночків

2506 / 6854
У хворого 42-х років з хронічною серцевою недостатністю, незважаючи на терапію кардіотонічними засобами і тіазидовим діуретиком, зберігаються набряки і виникла загроза асциту. Який препарат слід призначити для підсилення діуретичного ефекту застосованих ліків?

Амілорид

Фуросемід

Клопамід

Спіронолактон

Манітол

2507 / 6854
У синтезі пуринових нуклеотидів беруть участь деякі амінокислоти, похідні вітамінів, фосфорні ефіри рибози. Коферментна форма якого вітаміну є переносником одновуглецевих фрагментів в синтезі пуринових нуклеотидів?

Нікотинова кислота

Фолієва кислота

Пантотенова кислота

Піридоксин

Рибофлавін

2508 / 6854
На дослідження в бактеріологічну лабораторію було відправлено випорожнення хворої дитини грудного віку, з яких виділена культура ентеропатогенних кишкових паличок О55К59. На основі яких критеріїв виділена культура віднесена до ЕПКП О55?

Морфологічні ознаки

Біохімічні властивості

Культуральні ознаки

Визначення фаговару

Антигенні властивості

2509 / 6854
У померлої дитини 3-х років за життя мала місце менінгіальна симптоматика, На розтині в м’якій мозковій оболонці макроскопічно виявлені просоподібні вузлики, які мікроскопічно представлені осередком казеозного некрозу з валами епітеліоїдних, лімфоїдних клітин, між якими зустрічаються великі клітини з ядрами на периферії у вигляді півмісяця. Який найбільш імовірний менінгіт у дитини?

Грипозний

Сифілітичний

Менінгококовий

Туберкульозний

Бруцельозний

2510 / 6854
З метою оцінки адаптації до фізичного навантаження лікар провів обстеження робітників після виконання важкої праці. Які зміни в загальному аналізі крові можна виявити?

Лейкопенія

Гіпоальбумінемія

Анемія

Перерозподільчий лейкоцитоз

Зсув лейкоцитарної формули вліво

2511 / 6854
У чоловіка 34-х років під час бійки виникла зупинка серця внаслідок сильного удару у верхню ділянку передньої черевної стінки. Який із зазначених механізмів регуляції спричинив зупинку серця?

Парасимпатичні умовні рефлекси

Симпатичні умовні рефлекси

Периферичні рефлекси

Симпатичні безумовні рефлекси

Парасимпатичні безумовні рефлекси

2512 / 6854
Після фармакологічної блокади іонних каналів мембрани нервового волокна потенціал спокою зменшився з -90 до -80 мВ. Які канали було заблоковано?

Хлорні

Натрієві

Магнієві

Кальцієві

Калієві

2513 / 6854
У хворого 49-ти років на гострий панкреатит виникала загроза некрозу підшлункової залози, що супроводжувалось надходженням у кров і тканини активних панкреатичних протеїназ і розщеплення тканинних білків. Які захисні фактори організму можуть інгібувати ці процеси?

Кріоглобулін, інтерферон

Гемоплексин, гаптоглобін

Імуноглобуліни

α2-макроглобулін, α1 -антитрипсин

Церулоплазмін, трансферин

2514 / 6854
У чоловіка 36-ти років після перенесеної стрептококової інфекції діагностовано гострий гломерулонефрит. Найбільш імовірно, що ураження базальної мембрани ниркових тілець виникає внаслідок алергічної реакції такого типу:

Стимулююча

Анафілактична

Цитотоксична

Імунокомплексна

Сповільнена

2515 / 6854
Хворому 46-ти років на ревматоїдний поліартрит призначили нестероїдний протизапальний засіб. Але, з причини супутнього захворювання, через деякий час препарат відмінили. Яке захворювання є протипоказанням до призначення цієї групи препаратів?

Мігрень

Виразкова хвороба

Бронхіт

Радикуліт

Пневмонія

2516 / 6854
Чоловік 53-х років звернувся зі скаргами на гострий біль у правому підребер’ї. При огляді лікар звернув увагу на пожовтілі склери хворого. Лабораторні аналізи показали підвищену активність АЛТ та негативну реакцію на стеркобілін у калі. Для якого захворювання характерні такі симптоми?

Хронічний коліт

Гепатит

Хронічний гастрит

Жовчнокам’яна хвороба

Гемолітична жовтяниця

2517 / 6854
До інфекційної лікарні доставлено літнього чоловіка, безпритульного. Скарги на високу температуру, запаморочення, висип на шкірі. Враховуючи, що хворий страждає також на педикульоз, лікар запідозрив висипний тиф. Який метод діагностики найдоцільніше використати для підтвердження діагнозу?

Бактеріологічний

Алергологічний

Серологічний

Вірусологічний

Мікроскопічний

2518 / 6854
У ході хірургічної операції на нирці із заднього доступу (з боку спини) виникла необхідність перетиснути ниркову артерію. У якій послідовності розташовані елементи ниркової ніжки у її воротах ззаду наперед?

Артерія, сечовід, вена

Вена, сечовід, артерія

Сечовід, артерія, вена

Вена, артерія, сечовід

Артерія, вена, сечовід

2519 / 6854
У лікарню наприкінці робочого дня доставлений робітник 'гарячого' цеху, який скаржиться на головний біль, запаморочення, нудоту, загальну слабкість. Об’єктивно: свідомість збережена, шкірні покриви гіперемовані, сухі, гарячі на дотик. ЧСС- 130/хв. Дихання часте, поверхневе. Яке порушення процесів терморегуляції, найбільш імовірно, виникло у людини в даній ситуації?

Посилення тепловіддачі та зниження теплопродукції

Посилення теплопродукції без змін тепловіддачі

Зниження тепловіддачі

Посилення тепловіддачі та теплопродукції

Зниження теплопродукції без змін тепловіддачі

2520 / 6854
В експерименті подразнюють гілочки симпатичного нерва, які іннервують серце. Це призвело до збільшення сили серцевих скорочень, тому що через мембрану типових кардіоміоцитів збільшився:

Вихід іонів кальцію

Вхід іонів кальцію та калію

Вхід іонів кальцію

Вхід іонів калію

Вихід іонів калію

2521 / 6854
Під час тривалого бігу у чоловіка 35-ти років виникла гостра серцева недостатність. Які зміни іонного складу спостерігаються у серцевому м’язі при цьому стані?

Накопичення в клітинах міокарда іонів Na+ і Ca2+

Зменшення в клітинах міокарда іонів Na+ і Ca2+

Збільшення в позаклітинному просторі іонів Na+ і Ca2+

Зменшення в позаклітинному просторі іонів K + і Mg2+

Накопичення в клітинах міокарда іонів K + і Mg2+

2522 / 6854
Дитині 14-ти років поставили діагноз: бруцельоз. В контакті з хворими тваринами вона не була. Як дитина могла заразитись?

Через сире молоко

Через воду

Через брудні руки

Через немиті овочі і фрукти

Під час ін’єкції

2523 / 6854
При гострому інфаркті в міокарді виникає декілька зон в осередку інфаркту: зона некрозу, зона ішемічного пошкодження і зона ішемії. Зоні пошкодження на ЕКГ відповідає:

Зміщення сегменту RS — T вище ізолінії

Негативний зубець T

Глибокий зубець Q

Зниження зубця R

Комплекс QRS типу QS

2524 / 6854
Жінка 38-ми років звернулася до ендокринологічної клініки з виразним тремором кінцівок. Гіперпродукція, якого гормону здатна викликати такі порушення?

Соматостатин

Адреналін

Тироксин

АКТГ

Інсулін

2525 / 6854
Чоловік 60-ти років був госпіталізований до стаціонару з діагнозом отруєння грибами, де і помер на 12-й день з ознаками гострої печінкової недостатності. На секції - макроскопічно: печінка дрябла, в’яла, жовто-сірого кольору, глинястого вигляду як на поверхні, так і на розтині; мікроскопічно: ділянки некрозу з аутолітичним розпадом і утворенням жиро-білкового детриту в центрі та вузькою смужкою гепатоцитів у стані жирової дистрофії на периферії печінкових часточок. Який найбільш імовірний діагноз?

Хронічна токсична дистрофія печінки

Спадковий пігментний гепатоз

Токсична дистрофія печінки в стадії жовтої дистрофії

Токсична дистрофія печінки в стадії червоної дистрофії

Хвороба Вільсона-Коновалова

2526 / 6854
У хворого 32-х років у заочеревинному просторі виявлено пухлиноподібне утворення розміром 17х15 см з проростанням його в брижу, яке хірург цілком вилучити не зміг. Макроскопічно: тканина утворення на розрізі жовтувата, м’яка, з осередками некрозу і ослизнення. При мікроскопічному дослідженні виявлено: клітини з вираженим поліморфізмом ядер, з наявністю патологічних мітозів, цитоплазма клітин світла, дрібновакуолізована, при фарбуванні суданом вакуолі дають позитивну реакцію. Визначте пухлину:

Фіброма

Ліпосаркома

Фібросаркома

Гібернома

Ліпома

2527 / 6854
У юнака 16-ти років діагностовано хворобу Марфана. При дослідженні встановлено: порушення розвитку сполучної тканини, будови кришталика ока, аномалії серцевосудинної системи, арахнодактилія. Яке генетичне явище зумовило розвиток цієї хвороби?

Множинний алелізм

Комплементарність

Плейотропія

Кодомінування

Неповне домінування

2528 / 6854
При алкаптонурії відбувається надмірне виділення гомогентизинової кислоти із сечею. З порушенням метаболізму якої амінокислоти пов’язано виникнення цього захворювання?

Метіонін

Фенілаланін

Аланін

Тирозин

Аспарагін

2529 / 6854
У кішки з децеребраційною ригідністю потрібно знизити тонус м’язів. Цього можна досягти шляхом:

Руйнування вестибулярних ядер Дейтерса

Подразнення отолітових вестибуло-рецепторів

Подразнення вестибулярних ядер Дейтерса

Подразнення вестибулослухового нерва

Подразнення ампулярних вестибуло-рецепторів

2530 / 6854
Під дією негативних чинників довкілля порушена функція міосателітоцитів. Зміну якої функції всього м’язового волокна слід очікувати в даному випадку?

Трофіка

Скоротливий термогенез

Скорочення

Розслаблення

Регенерація

2531 / 6854
У жінки 23-х років після аборту з’явилась маткова кровотеча. Мікроскопічно у вишкребі з порожнини матки знайдені ворсини хоріона, що нагадують грона винограду. Мікроскопічно: набряк строми ворсин з утворенням безліч кіст, що супроводжуються проліферацією епітелію та синцитію ворсин, залишки плоду і плодових оболонок. Який найбільш імовірний діагноз?

Плацентарний поліп

Вагітність

Ендометрит

Міхуровий занесок

Хоріонепітеліома

2532 / 6854
У лабораторії особливо небезпечних інфекцій проводиться мікроскопічне дослідження патологічного матеріалу від хворого з підозрою на чуму, забарвленого за Гінсом-Буррі. Яку властивість збудника дозволяє визначити даний метод?

Капсулоутворення

Лугостійкість

Наявність зерен волютину

Спороутворення

Кислотостійкість

2533 / 6854
Фільтрат бульйонної культури збудників газової анаеробної інфекції розлили по пробірках, додали видові антитоксичні сироватки, витримали впродовж 40 хвилин при кімнатній температурі. Для визначення виду анаероба тепер необхідно:

Ввести тваринам вміст пробірок

Додати у пробірки еритроцитарний діагностикум

Вміст про6ірок посіяти на щільні поживні середовища

Додати у пробірки преципітуючу діагностичну сироватку

Додати у пробірки аглютинуючу діагностичну сироватку

2534 / 6854
Для постійного внутрішньовенного ведення ліків хворому краще встановити підключичний катетер. В якому анатомічному утворенні на шиї потрібно шукати підключичну вену?

Fossa retromandibularis

Spatium interscalenum

Spatium interaponeuroticum suprasternale

Spatium antescalenum

2535 / 6854
У тварини збільшений тонус м’язів-розгиначів. Це є наслідком посиленої передачі інформації до мотонейронів спинного мозку такими низхідними шляхами:

Медіальні кортикоспінальні

Руброспінальні

Вестибулоспінальні

Латеральні кортикоспінальні

Ретикулоспінальні

2536 / 6854
У пацієнта 21-го року при очікуванні екстракції зуба виникло сильне відчуття страху. Який з препаратів йому слід призначити для усунення цього відчуття?

Діазепам

Карбамазепін

Аміназин

Етимізол

Анальгін

2537 / 6854
У потерпілого 35-ти років з травмою голови у скроневій ділянці діагностовано епідуральну гематому. Яка з артерій найімовірніше пошкоджена?

Поверхнева скронева

Задня вушна

Середня мозкова

Передня оболонкова

Середня оболонкова

2538 / 6854
В результаті травми порушено цілісність переднього корінця спинного мозку. Які відростки яких нейронів при цьому пошкоджені?

Дендрити чутливих нейронів

Аксони чутливих нейронів

Дендрити рухових нейронів

Дендрити вставних нейронів

Аксони рухових нейронів

2539 / 6854
У чоловіка відмічається випадіння функції медіальних половин сітківки. Який відділ провідного шляху зорового аналізатора уражений?

Правий зоровий тракт

Лівий зоровий нерв

Правий зоровий нерв

Лівий зоровий тракт

Зорове перехрестя

2540 / 6854
В експерименті необхідно оцінити рівень збудливості тканини. Для цього доцільно визначити:

Тривалість ПД

Критичний рівень деполяризації

Поріг деполяризації

Амплітуду ПД

Потенціал спокою

2541 / 6854
Хворий 33-х років звернувся до стоматолога зі скаргами на те, що після лікування солями важких металів у нього з’явились сірі плями на слизовій оболонці ротової порожнини. Який засіб слід застосувати для усунення побічних ефектів?

Налоксон

Діамантовий зелений

Метиленовий синій

Унітіол

Калію перманганат

2542 / 6854
При визначенні повітряної та кісткової провідності звуку було встановлено, що у пацієнта ліве вухо краще сприймає звук при кістковому його проведенні, що могло бути пов’язане з захворюванням:

Середнього вуха зліва

Зовнішнього вуха справа

Середнього вуха справа

Внутрішнього вуха зліва

Внутрішнього вуха справа

2543 / 6854
В хронічному експерименті на щурах стимулювали електричним струмом паравентрикулярні та супраоптичні ядра гіпоталамуса. Яку поведінкову реакцію це спричинило у тварин?

Зменшення споживання їжі

Відмова від їжі та рідини

Зменшення споживання води

Збільшення споживання їжі

Збільшення споживання води

2544 / 6854
Чоловік 55-ти років доставлений до реанімаційного відділення без свідомості. Зі слів родичів стало відомо, що хворий помилково випив спиртовий розчин невідомого виробника. За даними проведеного обстеження діагностовано отруєння метиловим спиртом. Який антидот необхідно використати в даному випадку?

Налоксон

Етанол

Тетурам

Протаміну сульфат

Ацетилцистеїн

2545 / 6854
Внаслідок ДТП у потерпілої 37-ми років виникло неутримання сечі. Які сегменти спинного мозку пошкоджені?

Th2 — Th5

L1 — L2

S2 - S4

Th1 — L1

Th1 — Th5

2546 / 6854
При обстеженні жінки 56-ти років, що хвора на цукровий діабет 1-го типу, виявлене порушення білкового обміну, що при лабораторному дослідженні крові проявляється аміноацидемією а клінічно - уповільненням загоєння ран і зменшенням синтезу антитіл. Який з перерахованих механізмів викликає розвиток аміноацидемії?

Гіперпротеїнемія

Підвищення протеолізу

Підвищення онкотичного тиску в плазмі крові

Збільшення ліпопротеїдів високої щільності

Зменшення концентрації амінокислот у крові

2547 / 6854
Дитина 6-ти років під час гри порізала ногу осколком скла і була направлена у поліклініку для введення протиправцевої сироватки. З метою попередження розвитку анафілактичного шоку лікувальну сироватку вводили за методом Безредка. Який механізм лежить в основі подібного способу гіпосенсибілізації організму?

Стимуляція імунологічної толерантності до антигену

Стимуляція синтезу антиген-специфічних IgG

Зв’язування рецепторів до IgE на тучних клітинах

Блокування синтезу медіаторів у тучних клітинах

Зв’язування фіксованих на тучних клітинах IgE

2548 / 6854
У дитини 6-ти років гостро виникло захворювання з високою температурою тіла, вираженою інтоксикацією і геморагічною висипкою на шкірі. Смерть настала від гострої надниркової недостатності. На розтині морфологічні зміни визначалися головним чином важкістю ДВЗ-синдрому і ендотоксичного шоку. У шкірі виявлені вогнища некрозу, діапедезні крововиливи, у судинах дерми фібринові тромби. У наднирниках масивні крововиливи. Який найбільш імовірний діагноз?

Грип

Кір

Скарлатина

Висипний тиф

Менінгококцемія

2549 / 6854
Хворому на контактний дерматит необхідно призначити антигістамінний лікарський засіб, що не викликає сонливості. Назвіть цей препарат:

Ранітидин

Лоратадин

Дипразин

Димедрол

Супрастин

2550 / 6854
Недостатність в організмі лінолевої та ліноленової кислот призводить до ушкоджень шкіри, випадіння волосся, сповільненого загоювання ран, тромбоцитопенії, зниження опірності до інфекційних захворювань. Порушення синтезу яких речовин найімовірніше зумовлює вказані симптоми?

Інтерферони

Інтерлейкіни

Кортикостероїди

Ейкозаноїди

Катехоламіни

2551 / 6854
Прикладом специфічних паразитів людини є малярійний плазмодій, гострик дитячий і деякі інші. Джерелом інвазії таких паразитів завжди є людина. Такі специфічні паразити людини викликають захворювання, які називаються:

Антропозоонозні

Інфекційні

Зоонозні

Антропонозні

Мультифакторіальні

2552 / 6854
Хворому 63-х років з атонією сечового міхура лікар призначив препарат, дозу якого хворий самостійно збільшив. З’явились підвищене потовиділення, салівація, діарея, м’язові спазми. Препарат якої групи був призначений?

Токолітики

Холіноміметики

Адреноблокатори

Іангліоблокатори

Реактиватори холінестерази

2553 / 6854
Для лікування піодермії лікар призначив вакцину, яка виготовлена зі штаму бактерій, виділеного від хворого. До якого типу вакцин належить даний препарат?

Атенуйована вакцина

Асоційована вакцина

Генно-інженерна вакцина

Хімічна вакцина

Автовакцина

2554 / 6854
У біоптаті нирки 45-річного чоловіка, що має хронічну хворобу нирок, виявлено: склероз, лімфоплазмоцитарна інфільтрація стінок мисок та чашок, дистрофія та атрофія канальців. Збережені канальці розширені, розтягнені колоїдоподібними масами, епітелій сплющений ('щитоподібна' нирка). Який діагноз найбільш імовірний?

Тубуло -інтерстиційний нефрит

Хронічний пієлонефрит

Іломерулонефрит

Іострий пієлонефрит

Нефросклероз

2555 / 6854
У водія 60-ти років довготривала хронічна патологія легень призвела до недостатності клапанів легеневої артерії і трикуспідального клапану, недостатності кровообігу за правошлуночко-вим типом. Який тип артеріальної гіпертензії є причиною перевантаження серця об’ємом?

Рефлексогенна гіпертензія

Легенева гіпертензія

Центрально-ішемічна гіпертензія

Ессенціальна гіпертензія

Сольова гіпертензія

2556 / 6854
Під час футбольного матчу між вболівальниками виникла сутичка. На фоні негативних емоцій в одного учасника сутички були розширені зіниці й підвищене серцебиття. Активація якої системи організму забезпечує такі вегетативні зміни при негативних емоціях?

Симпато-адреналова

Гіпоталамо-гіпофізарно-тиреоїдна

Парасимпатична нервова

Соматична нервова

Метасимпатична нервова

2557 / 6854
При дослідженні гостроти слуху в коваля виявили втрату слуху на 50% у діапазоні низьких частот і майже нормальну гостроту слуху в діапазоні високих частот. Порушення яких структур слухової системи призвело до такого стану?

Барабанна перетинка

Середня частина кортієвого органу

Кортієв орган - ближче до овального віконця

М’язи середнього вуха

Кортієв орган - ближче до гелікотреми

2558 / 6854
У хворого 26-ти лет, що приймав високоефективний протитуберкульозний засіб, наприкінці курсу лікування розвинувся периферичний неврит. Який препарат викликав ускладнення?

Ізоніазид

Етамбутол

Ципрофлоксацин

Флориміцину сульфат

Рифампіцин

2559 / 6854
У людини після довільної тривалої затримки дихання збільшилися частота й глибина дихання. Які зміни у складі крові стали причиною цього?

Підвищення pC02

Підвищення p02

Зниження p02

Зниження pC02

Підвищення pH

2560 / 6854
Стеатоз виникає внаслідок накопичення триацилгліцеролів у гепатоцитах. Одним з механізмів розвитку цього захворювання є зменшення утилізації нейтрального жиру ЛПДНЩ. Які ліпотропні речовини попереджують розвиток стеатозу?

Валін, B3 , B2

Аланін, B1 , PP

Метіонін, B6 , B12

Iзолейцин, B1 , B2

Аргінін, B2 , B3

2561 / 6854
Який стан може розвинутися через 15-30 хвилин після повторного введення антигену внаслідок підвищеного рівня антитіл, переважно IgE, які адсорбуються на поверхні клітин-мішеней - тканинних базофілів (тучних клітин) та базофілів крові?

Антитіло-залежна цитотоксичність

Анафілаксія

Сироваткова хвороба

Імунно-комплексна гіперчутливість

Гіперчутливість уповільненого типу

2562 / 6854
У дівчинки 18-ти років через 5 годин після вживання морепродуктів на шкірі тулуба та дистальних відділів кінцівок з’явились маленькі сверблячі папули, які частиною зливаються між собою. Через добу висипка самовільно зникла. Назвіть механізм гіперчутливості, що полягає в основі даних змін:

Системна анафілаксія

Імунокомплексна гіперчутливість

Атопія (місцева анафілаксія)

Антитілоопосередкований клітинний цитоліз

Клітинна цитотоксичність

2563 / 6854
Чоловік 46-ти років, хворий на гіпертонічну хворобу, разом з безсольовою дієтою та з антигіпертензивними засобами довгий час приймав гідрохлортіазид, що зумовило порушення електролітного балансу. Яке порушення внутрішнього середовища виникло у хворого?

Збільшення об’єму циркулюючої крові

Гіпермагніємія

Гіперкаліємія

Метаболічний ацидоз

Гіпохлоремічний алкалоз

2564 / 6854
Синтез і-РНК проходить на матриці ДНК з урахуванням принципу комплементарності. Якщо триплети у ДНК наступні - АТГ-ЦГТ, то відповідні кодони і-РНК будуть:

ТАГ-УГУ

АУГ-ЦГУ

АТГ-ЦГТ

УАГ-ЦГУ

УАЦ-ГЦА

2565 / 6854
При декарбоксилюванні глутамату утворюється нейромедіатор гамма-аміномасляна кислота (ГАМК). При розпаді ГАМК перетворюється у метаболіт циклу лимонної кислоти, яким є:

Сукцинат

Оксалоацетат

Малат

Лимонна кислота

Фумарат

2566 / 6854
У дитячому дошкільному закладі напередодні новорічних свят було зареєстровано спалах кишкової інфекції. При бактеріологічному дослідженні випорожнень хворих патогенних бактерій не було виділено. При електронній мікроскопії виявлено утворення округлої форми з чітким обідком і товстою втулкою, які нагадують колесо. Вкажіть найбільш імовірний збудник даної інфекції:

Rotavirus

P.vulgaris

Adenovirus

E.coli

Coxsacki-virus

2567 / 6854
Жінка 28-ми років доставлена до приймального відділення зі скаргами на різкий біль у нижній ділянці живота. Гінеколог при огляді припустив позаматкову вагітність. Яке анатомічне утворення потрібно пропунктувати для підтвердження діагнозу?

Передміхуровий простір

Міхурово-прямокишкова заглибина

Прямокишково-маткова заглибина

Міжсигмоподібний закуток

Матково-міхурова заглибина

2568 / 6854
При гістологічному дослідженні біоптатів, взятих з потовщених країв виразки шлунка, виявлені невеликі гніздові скупчення різко атипових гіперхромних невеликих епітеліальних клітин, які розташовані серед дуже розвиненої строми. Визначте пухлину:

Недиференційована саркома

Аденома

Медулярний рак

Аденокарцинома

Скіррозний недиференційований рак

2569 / 6854
Жінка 26-ти років поступила у пологове відділєння в терміні вагітності 40 тижнів. Шийка матки розкрита, але скорочення матки відсутнє. Лікар дав засіб гормональної природи для посилення пологової діяльності. Назвіть засіб:

Гідрокортизон

Тестостерон

Окситоцин

Естрон

АКТГ

2570 / 6854
У хворого 47-ми років виникла кишкова коліка на фоні гіпертонічної хвороби. Засоби якої з перерахованих груп найбільш доцільно використати для її купірування у даній ситуації?

Адреноміметики

Симпатоміметики

М-холиноміметики

Міотропні спазмолітики

Антихолінестеразні засоби

2571 / 6854
Віддачу тепла яким шляхом збільшують люди, які приймають прохолодний душ у спекотні дні?

Теплопроведення

Конвекція

Випаровування поту

Випромінювання

2572 / 6854
Пацієнт доставлений в інфекційне відділення з попереднім діагнозом ботулізм. У бактеріологічній лабораторії провели одну з імунних реакцій на вміст ботулінічного токсину в досліджуваному матеріалі. Як називається ця реакція?

Зв’язування комплементу

Іммобілізації

Аглютинації

Нейтралізації

Подвійної імунної дифузії

2573 / 6854
У недоношеного немовляти спостерігається жовтяниця. З нестачею у нього якого ферменту це пов’язано?

Кисла фосфатаза

Каталаза

Лужна фосфатаза

НАД+ -дегідрогеназа

УДФ-трансглюкуронідаза

2574 / 6854
У студента 19-ти років запальний процес у крилопіднебінній ямці. Інфекція поширилась в середню черепну ямку. Через яке анатомічне утворення розповсюдилась інфекція?

Canalis palatinus minor

Canalis ptherygoideus

Foramen rotundum

Foramen sphenopalatinum

Canalis palatinus major

2575 / 6854
У жінки 36-ти років має місце гіповітаміноз B2. Причиною виникнення специфічних симптомів (ураження епітелію, слизових, шкіри, рогівки ока) імовірно є дефіцит:

Флавінових коферментів

Цитохрому С

Цитохрому В

Цитохрому А1

Цитохромоксидази

2576 / 6854
Пацієнт 22-х років скаржиться на підвищену больову чутливість задньої третини язика і на порушення смаку в цій ділянці. Ураження якого нерва має місце?

Під’язиковий

Трійчастий

Лицевий

Додатковий

Язикоглотковий

2577 / 6854
На електронній мікрофотографії представлена клітина нейрального походження. Термінальна частина дендрита клітини має циліндричну форму і складається з 1000 замкнутих мембранних дисків. Яка клітина зображена на мікрофотографії?

Паличкова нейросенсорна

Нейрон кори великих півкуль

Колбочкова нейросенсорна

Нейрон передніх рогів спинного мозку

Нейрон спинномозкового вузла

2578 / 6854
Порушення процесів мієлінізації нервових волокон призводить до неврологічних розладів і розумової відсталості. Такі симптоми характерні для спадкових і набутих порушень обміну:

Холестерину

Сфінголіпідів

Вищих жирних кислот

Фосфатидної кислоти

Нейтральних жирів

2579 / 6854
Хворому хірург видалив порожнинне утворення печінки діаметром 2 см. Встановлено, що стінка порожнини утворена щільною волокнистою сполучною тканиною, вміст являє собою каламутну, густу, жовтувато-зеленуватого кольору рідину з неприємним запахом, яка мікроскопічно складається переважно з поліморфноядерних лейкоцитів. Якому патологічному процесу відповідають такі морфологічні зміни?

Емпієма

Гострий абсцес

Флегмона

Хронічний абсцес

2580 / 6854
У клітині в гранулярній ЕПС відбувається етап трансляції, при якому спостерігається просування і-РНК щодо рибосоми. Амінокислоти з’єднуються пептидними зв’язками в певній послідовності - відбувається біосинтез поліпептиду. Послідовність амінокислот у поліпептиді буде відповідати послідовності:

Кодонів і-РНК

Антикодонів т-РНК

Антикодонів р-РНК

Нуклеотидів т-РНК

Нуклеотидів р-РНК

2581 / 6854
При травмі периферичних нервів виникає м’язова атрофія, кістки стають порозними і ламкими, на шкірі і слизових виникають виразки. Яка функція нервових системи уражується у даному випадку?

Вища нервова діяльність

Трофічна

Чутлива

Вегетативна

Рухова

2582 / 6854
У вагітної жінки 24-х років після тривалого блювання було зареєстровано зниження об’єму циркулюючої крові. Про яку зміну загальної кількості крові може йти мова?

Олігоцитемічна гіповолемія

Олігоцитемічна гіперволемія

Проста гіповолемія

Поліцитемічна гіповолемія

Поліцитемічна гіперволемія

2583 / 6854
Робочий комунальної служби спустився в каналізаційний колодязь без засобів захисту і через деякий час знепритомнів. Лікарями швидкої допомоги діагностовано отруєння сірководнем. Який вид гіпоксії при цьому розвинувся?

Гемічний

Респіраторний

Тканинний

Циркуляторний

Перевантажувальний

2584 / 6854
У хворого 48-ми років на хронічний гломерулонефрит наявні набряки, АТ-210/100 мм рт.ст., ЧСС- 85/хв., межі серця розширені. Який механізм розвитку артеріальної гіпертензії є головним?

Підвищення активності симпатичного відділу нервової системи

Підвищення продукції вазопресину

Активація ренин-ангіотензин-альдостеронової системи

Підвищення ОЦК

Гіперфункція серця

2585 / 6854
Обстежуваний знаходиться у фазі повільнохвильового глибокого сну. Про це свідчить реєстрація на ЕЕГ таких хвиль:

Альфа-веретена

Дельта-хвилі

Бета-хвилі

Тета -хвилі

Альфа-хвилі

2586 / 6854
Потерпілий 19-ти років доставлений до травматологічного відділення з різаною раною трапецієподібного м’яза. Яка з фасцій шиї формує піхву для даного м’яза?

Вісцеральна частина передтрахейної пластинки шийної фасції

Сонна піхва шийної фасції

Передхребтова пластинка шийної фасції

Поверхнева пластинка шийної фасції

М ’язова частина передтрахейної пластинки шийної фасції

2587 / 6854
У хворого діагностовано ГРВІ. У сироватці крові знайдено імуноглобуліни класу М. Який період інфекційного процесу в даному випадку?

Інкубаційний

Гострий

Реконвалесценція

Продромальний

Мікро6оносійство

2588 / 6854
При огляді дівчинки 5-ти років лікар помітив на мигдаликах сірувату плівку. Мікроскопія мазків, пофарбованих за Нейссером показала наявність коринебактерій дифтерії. Яка морфологічна особливість була найбільш суттєвою для встановлення виду збудника?

Наявність спор, діаметр яких перевищує діаметр клітини

Полярно розташовані гранули волютину

Розташування клітин збудника у вигляді штахетника

Наявність капсули

Локалізація збудника всередині макрофагів

2589 / 6854
На розтині тіла хворого, що помер від серцевої недостатності, в інтимі аорти і коронарних судин виявлені жовтого кольору плями і смуги, що вибухають над поверхнею інтими сірувато-жовтуваті бляшки, а також вогнищеве виразкування даних бляшок з наявністю крововиливів, тромбозів і вогнищ звапнування. Для якого захворювання характерні вказані зміни судин?

Атеросклероз

Гіпертонічна хвороба

Вузликовий періартеріїт

Сифілітичний мезаортит

2590 / 6854
У результаті побутової травми у пацієнта виникла значна крововтрата, що супроводжувалося зниженням артеріального тиску. Дія яких гормонів забезпечує швидке відновлення кров’яного тиску, викликаного крововтратою?

Адреналін, вазопресин

Кортизол

Статеві

Альдостерон

Окситоцин

2591 / 6854
Жінка 31-го року хворіє на ВІЛ-інфекцію на стадії СНІД. На шкірі нижніх кінцівок, слизової оболонки піднебіння з’явились рудувато-червоні плями, яскраво-червоні вузлики різних розмірів. Один з вузликів взято на гістологічне дослідження. Виявлено багато хаотично розташованих тонкостінних судин, вистелених ендотелієм; пучки веретеноподібних клітин з наявністю гемосидерину. Яка пухлина розвинулась у хворої?

Лімфома Беркіта

Саркома Капоші

Фібросаркома

Гемангіома

Лімфангіома

2592 / 6854
Анатомічний мертвий простір - це частина повітря, яка залишається в повітроносних шляхах після видиху. В якій із наведених нижче ситуацій відбудеться зменшення анатомічного мертвого простору?

Поворот лежачого пацієнта на лівий бік

Накладання трахеостоми

Дихання через рот

Нахил голови вперед

Поворот лежачого пацієнта на правий бік

2593 / 6854
У чоловіка 29-ти років з ножовим пораненням шиї визначається кровотеча. При первинній обробці рани встановлено, що пошкоджена судина, розташована вздовж латерального краю груднинно-ключично-соскоподібного м’яза. Визначте цю судину:

V jugularis anterior

V jugularis externa

V. jugularis interna

carotis externa

2594 / 6854
У хворого переливання крові ускладнилося розвитком гемотрансфузійного шоку. Назвіть тип алергічної реакції, що лежить в основі даної патології.

Цитотоксичний

Імунокомплексний

Гіперчутливість сповільненого типу

Анафілактичний

Рецептороопосередкований

2595 / 6854
Лікарі-інфекціоністи широко застосовують антибіотики, які інгібують синтез нуклеїнових кислот. Який етап біосинтезу гальмує рифампіцин?

Транскрипція в прокаріотах і еукаріотах

Реплікація в прокаріотах

Ініціація транскрипції в прокаріотах

Сплайсинг у прокаріотах і еукаріотах

Термінація транскрипції в прокаріотах і еукаріотах

2596 / 6854
Підшлункова залоза - орган змішаної секреції. Ендокринно продукує бета-клітинами гормон інсулін, який впливає на обмін вуглеводів. Як він впливає на активність глікогенфосфорилази (ГФ) і глікогенсинтетази (ГС)?

Не впливає на активність ГФ і ГС

Активує ГФ і Гс

Пригнічує ГФ, активує ГС

Пригнічує ГФ і ГС

Активує ГФ, пригнічує ГС

2597 / 6854
Відомо, що введення в організм людини лікарського препарату дикумаролу викликає різке зниження в крові вмісту протромбіну і ряду інших білкових факторів згортання крові. Антивітаміном якого вітаміну є дикумарол?

Вітамін P

Вітамін H

Вітамін E

Вітамін K

Вітамін C

2598 / 6854
До лікарні звернулися робітники з приводу того, що на тілі ними були знайдені паразити сірого кольору, довжиною близько 3 мм. Вони викликали неприємне подразнення шкіри, в місцях укусів виникали свербіж, біль, папули синього кольору, крововиливи. У окремих робітників піднялася температура. На шкірі були крововиливи. Яке захворювання найбільш імовірно?

Фтіріоз

Саркоптоз

Короста

Міаз шкірний

Педикульоз

2599 / 6854
Пацієнту 40-ка років з ознаками емоційної лабільності та пов’язаним з ними порушенням сну призначено нітразепам. З чим пов’язаний снодійний ефект цього засобу?

Інгібування ефектів збуджуючих амінокислот

Стимуляція H1 -гістамінових рецепторів

Активація ГАМК-ергічної системи

Пригнічення серотонінергічної нейромедіації

Блокада опіатних рецепторів

2600 / 6854
У хлопчика 11-ти років вміст холестерину в сироватці крові до 25 ммоль/л. В анамнезі - спадкова сімейна гіперхолестеринемія, причиною якої є порушення синтезу білків-рецепторів до:

Хіломікронів

Ліпопротеїнів проміжної щільності

Ліпопротеїнів дуже низької щільності

Ліпопротеїнів високої щільності

Ліпопротеїнів низької щільності

2601 / 6854
У сироватці крові пацієнта встановлено підвищення активності гіалуронідази. Визначення якого біохімічного показника сироватки крові дозволить підтвердити припущення про патологію сполучної тканини?

Галактоза

Глюкоза

Сіалові кислоти

Білірубін

Сечова кислота

2602 / 6854
При мікроскопії мікропрепарату з виділень хворої хронічним кольповагінітом лікар виявив округлої форми та еліпсоподібні клітини, що брунькуються, розміром 3-6 мкм. Про збудника якої грибкової хвороби може йти мова в даному випадку?

Кандидоз

Епідермофітія

Кокцидіоз

Криптококоз

Мікроспорія

2603 / 6854
У пацієнта з ознаками коліту виділена чиста культура бактерій, яка за морфологічними, культуральними та біохімічними властивостями віднесена до роду шигел. Яку з названих реакцій доцільно застосувати для серологічної ідентифікації культури?

Аглютинації з діагностичними сироватками

Преципітації

Затримки гемаглютинації

Зв’язування комплементу

Непрямої гемаглютинації

2604 / 6854
У пацієнтки 20-ти років встановлено діагноз - СНІД. Які популяції клітин найбільш чутливі до вірусу імунодефіциту людини?

Епітеліоцити

Гепатоцити

Ендотеліоцити

В-лімфоцити

Т-хелпери

2605 / 6854
Після загоєння рани на її місці утворився рубець. Яка речовина є основним компонентом цього різновиду сполучної тканини?

Гіалуронова кислота

Кератансульфат

Хондроітин-сульфат

Еластин

Колаген

2606 / 6854
У спортсмена внаслідок довільної затримки дихання на 40 секунд зросли частота серцевих скорочень та системний артеріальний тиск. Реалізація яких механізмів регуляції зумовлює зміни показників?

Безумовні симпатичні рефлекси

Безумовні парасимпатичні рефлекси

Умовні симпатичні рефлекси

Умовні парасимпатичні рефлекси

2607 / 6854
У малярійного плазмодія - збудника триденної малярії, розрізняють два штами: південний та північний. Вони відрізняються тривалістю інкубаційного періоду: у південного він короткий, а у північного - довгий. В цьому проявляється виражена дія такого добору:

Статевий

Рушійний

Штучний

Стабілізуючий

Дизруптивний

2608 / 6854
Струми надвисокої частоти (НВЧ), що застосовуються у фізіотерапії, не викликають збудження, а зумовлюють лише тепловий ефект на тканини. Як можна пояснити це явище?

Інтенсивність стимулу менше порога

Тривалість стимулу менше порога

Розвивається акомодація

Стимул поступає у фазу відносної рефрактерності

Стимул поступає у фазу абсолютної рефрактерності

2609 / 6854
У чоловіка 35-ти років феохромоцитома. В крові виявляється підвищений рівень адреналіну та норадреналіну, концентрація вільних жирних кислот зросла в 11 разів. Активація якого ферменту під впливом адреналіну підвищує ліполіз?

Фосфоліпаза С

ТАГ-ліпаза

Ліпопротеїдліпаза

Фосфоліпаза А2

Холестеролестераза

2610 / 6854
В результаті травми пошкоджений спинний мозок (з повним розривом) на рівні першого шийного хребця. Що відбудеться з диханням?

Зростає глибина

Зменшується частота

Не змінюється

Припиняється

Зростає частота

2611 / 6854
У людини з хронічним захворюванням нирок порушена їх видільна функція. рН венозної крові становить 7,33. Для корекції кислотно-лужного стану пацієнту доцільно внутрішньовенно ввести розчин:

Глюкози

Хлориду калію

Бікарбонату натрію

Хлориду кальцію

Хлориду натрію

2612 / 6854
При обтураційній жовтяниці і жовчних норицях часто спостерігається протромбінова недостатність. З дефіцитом в організмі якого вітаміну це пов’язано?

В6

K

E

A

C

2613 / 6854
При розтині тіла померлого виявлена гіперплазія кісткового мозку плоских і трубчастих кісток (піоїдний кістковий мозок), спленомегалія (6 кг), гепатомегалія (5 кг), збільшення всіх груп лімфатичних вузлів. Якому захворюванню відповідають виявлені зміни?

Справжня поліцитемія

Хронічний мієлолейкоз

Хронічний лімфолейкоз

Мієломна хвороба

Лімфогрануломатоз

2614 / 6854
У молодої жінки в зв’язку з гострими болями в клубовій ділянці, вилучена маткова труба з локальним розширенням її середньої третини, що заповнено кров’ю. При гістологічному дослідженні в отворі труби знайдені хоріальні ворсини, великі поля еритроцитів з домішком лейкоцитів. Який найбільш імовірний діагноз?

Геморагічний сальпінгіт

Гострий гнійний сальпінгіт

Гнійний сальпінгіт

Трубна вагітність

Крововилив в маткову трубу

2615 / 6854
При дослідженні крові хворого виявлено значне збільшення активності МВ-форм КФК (креатинфосфокінази) та ЛДГ-1. Яку патологію можна припустити?

Ревматизм

Інфаркт міокарда

Гепатит

Холецистит

Панкреатит

2616 / 6854
До лікарні надійшла дитина з діагнозом 'стафілококовий сепсис'. На яке живильне середовище потрібно посіяти кров хворого з метою виділення збудника?

Цукрово-пептонний бульйон

Середовище Бучіна

Середовище Плоскірьова

Жовчно-сольовий агар

М ’ясо-пептонний агар

2617 / 6854
У чоловіка 62-х років видалено нирку, у якій при макроскопічному дослідженні виявлено пухлину у вигляді вузла діаметром до 8 см. Тканина пухлини на розрізі строката, з множинними крововиливами, некрозами. Гістологічно: пухлина складається із світлих клітин, які утворюють альвеолярні і сосочкові структури, помірно виражений інвазивний ріст. У багатьох клітинах пухлини визначаються патологічні мітози, гіперхромні ядра. Діагностуйте виявлену пухлину нирки:

Нефробластома

Світлоклітинний рак

Аденокарцинома

Ацидофільна аденома з малігнізацією

Світлоклітинна аденома

2618 / 6854
Безпосередньо після переходу з горизонтального положення у вертикальне у чоловіка частота серцевих скорочень збільшилась на 15 скорочень за хвилину. Які механізми регуляції переважно зумовлюють цю зміну?

Умовні симпатичт рефлекси

Симпатичні рефлекси і катехоламіни

Умовні та безумовні симпатичні рефлекси

Безумовні симпатичні рефлекси

Катехоламіни

2619 / 6854
У людини з масою 80 кг після тривалого фізичного навантаження об’єм циркулюючої крові зменшився, гематокрит - 50%, загальний білок крові - 80 г/л. Такі показники крові є наслідком, перш за все:

Збільшення кількості еритроцитів

Збільшення онкотичного тиску плазми

Збільшення діурезу

Втрати води з потом

Збільшення вмісту білків у плазмі

2620 / 6854
Після перенесеного запального процесу хворий став помічати слабкість при згинанні кисті в ділянці 1,2, 3 і 4 пальців, зменшення об’єму м’яза підвищення великого пальця. При обстеженні виявлено порушення больової і температурної чутливості в ділянці поверхні долоні 1, 2, 3 і променевої поверхні четвертого пальців. Який із нервів уражений?

Променевий

Присередній шкірний нерв передпліччя

М’язово-шкірний

Ліктьовий

Серединний

2621 / 6854
У чоловіка 53-х років діагностовано сечокам’яну хворобу з утворенням уратів. Цьому пацієнту призначено аллопурінол, який є конкурентним інгібітором ферменту:

Уратоксидаза

Ксантиноксидаза

Уриділтрансфераза

Дигідроурацилдегідрогеназа

Уреаза

2622 / 6854
В експерименті на тварині досліджують серцевий цикл. Закриті усі клапани серця. Якій фазі циклу відповідає такий стан?

Протодіастолічний період

Швидкого наповнення

Повільного наповнення

Ізометричного скорочення

Асинхронного скорочення

2623 / 6854
Крива дисоціації оксигемоглобіну зміщена вправо. Які зміни в організмі людини можуть бути причиною цього?

Гіпокапнія

Гіпертермія

Алкалоз

Гіпоксемія

Збільшення концентрації 2,3-дифосфогліцерату в еритроцитах

2624 / 6854
При операції правосторонньої лобектомії хірург підійшов до кореня правої легені з метою окремого виділення і обробки його складових. Вкажіть порядок розміщення елементів кореня правої легені з якими лікар зустрінеться при виділенні та обробці в напрямку зверху вниз?

Бронх, легенева артерія, легеневі вени

Легенева артерія, бронх, легеневі вени

Легенева вена, легеневі артерії, бронх

Бронх, легенева артерія, діафрагмальний нерв

Діафрагмальний нерв, бронх, бронхіальні артерія і вена

2625 / 6854
Ціаністий калій є отрутою, від якої смерть організму наступає миттєво. На які ферменти в мітохондріях діє ціанистий калій?

Цитохром Р-450

Цитохромоксидаза (аа3)

Цитохром В5

Флавінові ферменти

НАД+ - залежні дегідрогенази

2626 / 6854
При бактеріоскопічному дослідженні носоглоткового слизу дитини 2,5 років, хворої на назофарингіт, виявлені грамнегативні диплококи, схожі за формою на кавові зерна. Які структури організму дитини найбільш імовірно будуть уражені, якщо ці мікроорганізми проникнуть у кров?

Ниркові гломерули

Сєрцєві клапани

Лімфатичні вузли

Сечо-статеві шляхи

Оболонки мозку

2627 / 6854
У хворого під час трахеотомії виникла виражена кровотеча. Яка артерія була травмована під час операції?

A. thyroidea ima

A. thyroidea inferior

A. laryngea inferior

A. thyroidea superior

A. laryngea superior

2628 / 6854
При різноманітних захворюваннях рівень активних форм кисню різко зростає, що призводить до руйнування клітинних мембран. Для запобігання цьому використовують антиоксиданти. Найпотужнішим природнім антиоксидантом є:

Жирні кислоти

Вітамін D

Гліцерол

Глюкоза

Альфа-токоферол

2629 / 6854
При обстеженні чоловіка 40-ка років було встановлено діагноз: гіпохромна анемія. Який препарат треба призначити для лікування?

Гепарин

Пентоксил

Вікасол

Ферковен

Ціанокобаламін

2630 / 6854
У неврологічне відділення з приводу мозкового крововиливу поступив хворий 62-х років. Об’єктивно: стан важкий. Спостерігається наростання глибини і частоти дихання, а потім його зменшення до апное, після чого цикл дихальних рухів відновлюється. Який тип дихання у хворого?

Гаспінг-дихання

Кусмауля

Апнейстичне

Біота

Чейна-Стокса

2631 / 6854
Хворому проведено субтотальну субфасціальну резекцію щитоподібної залози. У післяопераційному періоді тривалий час зберігається охриплість голосу. Який нерв ушкоджено в ході операції?

Верхній гортанний нерв

Язиковий нерв

Нижньощелепний нерв

Зворотній гортанний нерв

Під’язиковий нерв

2632 / 6854
Чоловік 53-х років доставлений у стаціонар у непритомному стані. Об’єктивно: шкіра суха, дихання часте поверхневе, запах ацетону відсутній, Ps- 126/хв., АТ- 70/40 мм рт.ст. Вміст глюкози у крові 48 ммоль/л, реакція сечі на ацетон негативна. Для якого із перелічених станів найбільш характерні симптоми у хворого?

Гіперосмолярна кома

Гіперкетонемічна кома

Токсична кома

Колапс

Лактацидемічна кома

2633 / 6854
У гістопрепараті представлений паренхіматозний орган, що має кіркову і мозкову речовину. Кіркова утворена тяжами епітеліоцитів, між якими проходять кровоносні капіляри. Тяжі формують три зони. Мозкова речовина складається з хромафіноцитів і венозних синусоїдів. Який орган має дані морфологічні ознаки?

Нирки

Тимус

Лімфатичний вузол

Щитоподібна залоза

Наднирник

2634 / 6854
На спеціальному живильному середовищі після посіву виділень гною з уретри виросли ніжні блакитні колонії. При мікроскопії препаратів з них виявлені грамнегативні бобоподібні диплококи. Збудником якої хвороби вони є?

Туляремія

Хламідіоз

Меліоїдоз

Сифіліс

Гонорея

2635 / 6854
Жінка 68-ми років скаржиться на відсутність рухів у правих руці і нозі. Чотири місяці тому перенесла інсульт. Об’єктивно: рухи в правих кінцівках відсутні, тонус м’язів їх підвищений. Який стан спостерігається у хворої?

Моноплегія

Параплегія

Геміплегія

Тетраплегія

2636 / 6854
У пораненого перев’язали плечову артерію в нижній третині плеча. За рахунок яких артерій можливе відновлення кровопостачання передпліччя і кисті?

Передні і задні огинаючі артерії плеча

Грудонадплечова, задня огинаюча артерія плеча

Підлопаткова, передня міжкісткова артерія

М’язові гілки плечової артерії

Глибока артерія плеча, ліктьовіколатеральні артерії

2637 / 6854
У стінці бронху при гістологічному дослідженні чітко визначаються залози, хрящові острівці та багаторядний циліндричний миготливий епітелій. Які бронхи досліджують?

Головні

Середні

Великі

Термінальні бронхіоли

Малі

2638 / 6854
Під час обстеження у хворої встановлене ураження дорсальної частини мосту, порушена функція жування. Ядро якого нерва уражене?

Рухове ядро трійчастого нерва

Мостове ядро трійчастого нерва

Подвійне ядро блукаючого нерва

Ядро під’язикового нерва

Рухове ядро лицевого нерва

2639 / 6854
Глікоген, що надійшов з їжею, гідролізувався у шлунково-кишковому тракті. Який кінцевий продукт утворився в результаті цього процесу?

Галактоза

Глюкоза

Лактоза

Лактат

Фруктоза

2640 / 6854
При бактеріологічному дослідженні сечі хворого на пієлонефрит виділені мікроорганізми, що утворюють на м’ясо-пептонному агарі жовто-зелений пігмент і характерний запах. Як вони називаються?

Клебсієли

Ешеріхії

Протеї

Азотобактерії

Псевдомонади

2641 / 6854
У хворої 28-ми років виявлена пухлина яєчника і показана операція по його видаленню. При здійсненні її необхідно розтинати зв’язку, що з’єднує яєчник з маткою. Яку зв’язку повинен перерізати хірург?

Lig. Suspensorium ovarii

Lig. umbilicale laterale

Lig. Cardinali

Lig. Latum uteri

Lig. Ovarii proprium

2642 / 6854
Для підвищення результатів спортсмену рекомендували застосовувати препарат, який містить у собі карнітин. Який процес в найбільшому ступені активується карнітином?

Синтез кетонових тіл

Синтез стероїдних гормонів

Тканинне дихання

Синтез ліпідів

Транспорт жирних кислот у мітохондрії

2643 / 6854
Хвора 26-ти років звернулася до лікаря зі скаргами на появу у випорожненнях білих плоских рухливих утворів, які нагадують локшину. При лабораторному дослідженні виявлені членики з такою характеристикою: довгі, вузькі, з розміщеним поздовжньо каналом матки, яка має 17-35 бічних відгалужень з кожного боку. Який вид гельмінтів паразитує у кишечнику жінки?

Echinococcus granulosus

Hymenolepis nana

Taenia solium

Taeniarhynchus saginatus

Diphyllobothrium latum

2644 / 6854
У жінки 30-ти років при гістологічному дослідженні шийного лімфовузла виявлено порушення малюнка внаслідок розростання епітеліоїдних, лімфоїдних клітин і макрофагів з ядрами у вигляді підкови, в центрі деяких скупчень клітин - безструктурні ділянки блідо-рожевого кольору з уламками ядер. Для якого захворювання характерні такі зміни?

Метастаз пухлини

Туберкульоз

Актиномікоз

Лімфогрануломатоз

Сифіліс

2645 / 6854
Хворому з прогресуючою м’язовою дистрофією було проведено біохімічне дослідження сечі. Поява якої речовини у великій кількості в сечі може підтвердити захворювання м’язів у даного хворого?

Гіпурова кислота

Креатин

Креатинін

Сечовина

Порфірини

2646 / 6854
При пункційній біопсії в трансплантованій нирці виявлена дифузна інфільтрація строми лімфоцитами, плазмоцитами, лімфобластами, плазмобластами, а також некротичний артеріїт. Який патологічний процес розвинувся у трансплантаті?

Тубулонекроз

Імунне відторгнення

Гломерулонефрит

Ішемічне пошкодження нирки

Пієлонефрит

2647 / 6854
Який механізм тепловіддачі найбільш ефективно спрацьовує при перебуванні людини в умовах 80% вологості повітря та температурі навколишнього середовища +35oC?

Теплопровідність

Конвекція

Радіація

Потовиділення

2648 / 6854
При обстеженні чоловіка 45-ти років, що тривалий час перебував на рослинній дієті, виявлено негативний азотистий баланс. Яка особливість раціону стала причиною цього явища?

Недостатня кількість жирів

Недостатня кількість жирів і білків

Недостатня кількість білків

Надмірна кількість вуглеводів

Надмірна кількість води

2649 / 6854
До клініки поступив чоловік 40-ка років, якого укусила гадюка. Де переважно буде проходити гемоліз еритроцитів у цьому випадку?

У клітинах печінки

У клітинах селезінки

У кровоносному руслі

У паренхімі нирок

У кістковому мозку

2650 / 6854
При клінічному обстеженні пацієнта 70-ти років виявлено порушення рухових функцій, що пов’язано з віковими змінами у гіаліновому хрящі. Які вікові зміни викликали обмеження рухів у суглобах?

Збільшення кількості ізогенних груп

Відкладання солей кальцію в міжклітинній речовині

Потовщення охрястя

Збільшення кількості хрящових клітин

Збільшення гідрофільності основної речовини

2651 / 6854
Хворому 42-х років для лікування бактеріальної пневмонії призначено амоксицилін. Вкажіть, який механізм бактерицидної дії препарату?

Порушення проникливості цитоплазматичної мембрани

Пригнічення SH-груп ферментів мікроорганізмів

Пригнічення синтезу клітинної стінки мікроорганізмів

Антагонізм із параамінобензойною кислотою

Пригнічення внутрішньоклітинного синтезу білка

2652 / 6854
У жінки обмежений кровотік у нирках, підвищений артеріальний тиск. Гіперсекреція якого гормону зумовила підвищення тиску?

Еритропоетин

Норадреналін

Вазопресин

Адреналін

Ренін

2653 / 6854
У чоловіка 41-го року відзначаються періодичні напади серцебиття (пароксизми), сильне потовиділення, напади головного болю. При обстеженні виявлена гіпертензія, гіперглікемія, підвищення основного обміну, тахікардія. При якій патології наднирників спостерігається подібна картина?

Гіперфункція кори наднирників

Гіпофункція кори наднирників

Гіпофункція мозкового шару

Первинний альдостеронізм

Гіперфункція мозкового шару

2654 / 6854
Пацієнт 16-ти років, що страждає на хворобу Іценко-Кушінга, консультований з приводу надмірної ваги тіла. При опитуванні з’ясувалося, що енергетична цінність спожитої їжі складає 1700-1900 ккал/добу. Яка провідна причина ожиріння у даному випадку?

Нестача глюкокортикоїдів

Нестача інсуліну

Гіподинамія

Надлишок глюкокортикоїдів

Надлишок інсуліну

2655 / 6854
У людини з нападом бронхоспазму необхідно зменшити вплив блукаючого нерва на гладеньку мускулатуру бронхів. Які мембранні циторецептори доцільно заблокувати для цього?

α- та β-адренорецептори

М-холінорецептори

β-адренорецептори

α-адренорецептори

Н-холінорецептори

2656 / 6854
У новонародженої дитини виявлено наступну патологію: аномалія розвитку нижньої щелепи та гортані, що супроводжується характерними змінами голосу, а також мікроцефалія, вада серця, чотирьохпалість. Найбільш імовірною причиною таких аномалій є делеція:

Короткого плеча 11-ої хромосоми

Короткого плеча 7-ої хромосоми

Короткого плеча 9-ої хромосоми

21-ої хромосоми

Короткого плеча 5-ої хромосоми

2657 / 6854
У жінки з важкою інтоксикацією, зумовленою сепсисом, який і послужив безпосередньою причиною смерті, на розтині виявлене 'тигрове серце'. Мікроскопічно в цитоплазмі кардіоміоцитів виявлені ліпіди. Який морфогенетичний механізм розвитку переважно лежить в основі даної дистрофії?

Спотворений синтез

Декомпозиція

Трансформація

Інфільтрація

2658 / 6854
Чоловік 42-х років з ураженням ліктьового нерва не може звести до серединної лінії II і V пальці. Функція яких м’язів при цьому порушена?

Відвідний м’яз мізинця

Долонні міжкісткові м’язи

Короткий долонний м’яз

Тильні міжкісткові м’язи

Червоподібні м’язи

2659 / 6854
Хворому з гострою недостатністю надниркових залоз був призначений лікарський препарат, після застосування якого у нього з’явилися скарги на біль у кістках (двічі були переломи), часті простудні хвороби, набряки, повільне загоювання ран. Який препарат міг спричинити такі явища?

Естріол

Ретаболіл

Спіронолактон

Тестостерон

Преднізолон

2660 / 6854
У подружжя народився син, хворий на гемофілію. Батьки здорові, а дідусь за материнською лінією також хворий на гемофілію. Визначте тип успадкування ознаки.

Неповне домінування

Домінантний, зчеплений зі статтю

Аутосомно-домінантний

Аутосомно-рецесивний

Рецесивний, зчеплений зі статтю

2661 / 6854
Хворому з діагнозом цукровий діабет II типу ендокринолог призначив глібенкламід. Вкажіть основний механізм дії цього засобу:

Підсилює захоплення глюкози периферичними тканинами

Підсилює метаболізм глюкози

Пригнічує глюконеогенез

Активує транспорт глюкози в клітину

Стимулює секрецію інсуліну бета-клітинами острівців Лангерганса

2662 / 6854
У мазку з нальоту на мигдаликах хворого з підозрою на дифтерію виявлено палички синього кольору з потовщеннями на полюсах. Який метод фарбування мазків було використано?

Грама

Леффлера

Нейссера

Буррі

Гінса

2663 / 6854
Дитина народилася в стані асфіксії. Який препарат необхідно ввести новонародженому для стимуляції дихання?

Прозерин

Атропін

Лобелін

Етимізол

Празозин

2664 / 6854
Хворий похилого віку страждає на хронічний закреп, в основі якого лежить гіпотонія товстого кишечнику. Який препарат слід призначити хворому?

Ацеклідин

Натрію сульфат

Прозерин

Касторова олія

Бісакодил

2665 / 6854
У пацієнта 38-ми років при прогресуванні стафілококового гнійного періодонтиту виникло гнійне запалення кістково-мозкових просторів альвеолярного відростка, а потім тіла нижньої щелепи. Мікроскопічно кісткові балки витончені, вогнища некрозу, кісткові секвестри, оточені сполучнотканинною капсулою. Який найбільш імовірний діагноз?

Хронічний остеомієліт

Гнійний періостит

Хронічний фіброзний періостит

Гострий остеомієліт

Пародонтома

2666 / 6854
У немовляти виявлено мікроцефалію. Лікарі вважають, що це пов’язано з застосуванням жінкою під час вагітності актиноміцину Д. На які зародкові листки в першу чергу подіяв цей тератоген?

Ентодерма

Ектодерма

Мезодерма

Ентодерма та мезодерма

Усі листки

2667 / 6854
У чоловіка 65-ти років впродовж 15-ти років була виражена артеріальна гіпертензія. Останнім часом систолічний тиск почав знижуватися, а діастолічний залишився підвищеним. Який гемодинамічний тип артеріальної гіпертензії у хворого?

Гіпокінетичний

Еукінетичний

Гіперкінетичний

Нормокінетичний

2668 / 6854
У жінки 52-х років при обстеженні було виявлено зниження кількості еритроцитів у крові та підвищення рівня вільного гемоглобіну в плазмі крові (гемоглобінемія). КП- 0,85. Який вид анемії спостерігається у хворої?

Набута гемолітична

Хронічна постгеморагічна

Спадкова гемолітична

Анемія внаслідок порушення ери-тропоезу

Гостра постгеморагічна

2669 / 6854
У бактеріологічній лабораторії проводиться дослідження якості питної води. Її мікробне число виявилося близько 100. Які мікроорганізми враховувалися при цьому?

Умовно-патогенні мікроорганізми

Всі бактерії, що виросли на живильному середовищі

Ентеропатогенні бактерії та вїруси

Бактерії, патогенні для людей та тварин

Бактерії групи кишкової палички

2670 / 6854
В гістологічному препараті визначається орган, стінка якого утворена трьома оболонками. Внутрішня оболонка складається з ендотелію і тонкого підендотеліального шару. Зовнішня оболонка найтовстіша. Який орган представлений у препараті?

Серце

Сечовід

Вена

Матка

Артерія

2671 / 6854
Лімфоцит уражений ретровірусом ВІЛ (СНІД). В цьому випадку напрямок потоку інформації в клітині буде:

РНК -> ДНК -> і-РНК -> поліпептид

ДНК -> поліпептид -> і-РНК

і-РНК -> поліпептид -> ДНК

ДНК -> і-РНК -> поліпептид -> ДНК

Поліпептид -> РНК -> ДНК -> і-РНК

2672 / 6854
Під час судово-медичної експертизи жінки, яка загинула у автокатастрофі, знайдено ембріон на стадії ранньої гаструли. Назвіть місце його локалізації при умові нормального розвитку:

Яєчник

Маткова частина яйцепроводу

Черевна порожнина

Ампульна частина яйцепроводу

Стінка матки

2673 / 6854
В основі розвитку імунних і алергічних реакцій організмом застосовуються однакові механізми відповіді імунної системи на антиген. Визначте основну відмінність алергічних реакцій від імунних?

Шляхи потрапляння в організм

Спадкова схильність

Кількість антигену, що потрапляє

Розвиток пошкодження тканин

Особливість будови антигенів

2674 / 6854
У пацієнта внаслідок черепно-мозкової травми знижена шкірна чутливість. Яка ділянка кори великого мозку може бути ураженою?

Задня центральна звивина

Потилична ділянка

Передня центральна звивина

Поясна звивина

Лобна ділянка кори

2675 / 6854
При розтині трупа чоловіка 47-ми років, померлого раптово, в інтимі черевного відділу аорти знайдені осередки жовтого кольору у вигляді плям та смуг, що не вибухають над поверхнею інтими. При фарбуванні суданом ІІІ спостерігається жовтогаряче забарвлення. Для якої стадії атеросклерозу характерні такі зміни?

Стадія утворення атероматозної виразки

Ліпоїдозу

Атерокальцинозу

Ліпосклерозу

Атероматозу

2676 / 6854
При огляді ротової порожнини стоматолог виявив появу у дитини перших великих нижніх кутніх зубів. Який вік дитини?

12-13 років

8-9 років

6-7 років

4-5 років

10-11 років

2677 / 6854
У батьків, хворих на гемоглобінопатію (аутосомно-домінантний тип успадкування), народилася здорова дівчинка. Які генотипи батьків?

Обоє гетерозиготні за геном гемоглобінопатії

Обоє гомозиготні за геном гемоглобінопатії

Мати гетерозиготна за геном гемоглобінопатії, у батька цей ген відсутній

Батько гетерозиготний за геном гемоглобінопатії, у матері цей ген відсутній

У обох батьків ген гемоглобінопатії відсутній

2678 / 6854
Хворому з артеріальною гіпертензією було призначено один з антигіпертензивних засобів. Артеріальний тиск нормалізувався, однак хворого почав турбувати постійний сухий кашель. Який з перерахованих препаратів має таку по6ічну дію?

Анаприлін

Резерпін

Лізиноприл

Ніфедипін

Клофелін

2679 / 6854
У 30-річного померлого наркомана, який страждав на ВІЛ-інфекцію, при патоморфологічному дослідженні виявлено, що обидві легені ущільнені, темно-бордовосірі, мало повітряні, міжальвеолярні перетинки густо інфільтровані лімфоцитами, частина альвеолоцитів трансформовані у великі клітини з центрально розташованим круглим ядром зі світлим обідцем, які нагадують 'совине око'. Яка опортуністична інфекція викликала пневмонію у даного хворого?

Токсоплазма

Пневмоциста карінії

Герпес-вірус

Атипова мікобактерія

Цитомегаловірус

2680 / 6854
У пацієнта у результаті тривалого блювання відбувається значна втрата шлункового соку, що є причиною порушення кислотно-лужного стану в організмі. Яка з перерахованих форм порушення кислотно-лужного стану має місце?

Метаболічний ацидоз

Негазовий алкалоз

Негазовий ацидоз

Газовий алкалоз

Газовий ацидоз

2681 / 6854
Хвора звернулася зі скаргами на болі у правій латеральній ділянці живота. Під час пальпації визначається щільне, нерухоме, пухлиноподібне утворення. У ділянці якого відділу травної трубки можлива наявність пухлини?

Colon ascendens

Colon sigmoideum

Colon transversum

Colon descendens

Caecum

2682 / 6854
На практичному занятті з мікробіології студентам запропоновано пофарбувати суміш бактерій за методикою Грама та пояснити механізм фарбування. Які морфологічні структури бактерій зумовлюють грамнегативне та грампозитивне фарбування бактерій?

Клітинна стінка

Цитоплазма

Капсула

ЦПМ

Джгутики

2683 / 6854
Студентка 22-х років звернулася до лікаря із скаргами на підвищення температури тіла до 38oC, слабкість, біль у горлі. Об’єктивно: язик вкритий білим нальотом. Які гістологічні структури язика беруть участь в утворенні цього нальоту?

Сполучнотканинна основа всіх сосочків язика

Епітелій ниткоподібних сосочків

Епітелій листоподібних сосочків

Епітелій грибоподібних сосочків

Епітелій жолобкуватих сосочків

2684 / 6854
Досліджуються рецептори, інформація від яких прямує до кори без участі таламусу. Які це рецептори?

Смакові

Нюхові

Слухові

Дотикові

Зорові

2685 / 6854
Препарат 'Гептрал', який використовують при хворобах печінки, містить Sаденозилметіонін. Ця активна амінокислота бере участь у синтезі:

Гему

Жовчних кислот

Триацилгліцеролів

Фосфоліпідів

Холестерину

2686 / 6854
Хвора 39-ти років, з цукровим діабетом в анамнезі, госпіталізована до клініки у прекоматозному стані кетоацидотичного типу. Збільшення вмісту якого метаболіту до цього призвело?

Цитрат

Малонат

Аспартат

Ацетоацетат

Альфа-кетоглутарат

2687 / 6854
В хірургічне відділення доставлено пацієнта з рiзаною раною медіального краю передпліччя. При обстеженні виявлено, що в хворого перерізано ліктьовий м’яз-згинач зап’ястка і ліктьовий м’яз-розгинач зап’ястка. Які з рухів кисті будуть порушені у хворого?

Розгинання і відведення

Відведення

Приведення

Згинання

Розгинання

2688 / 6854
Для діагностування деяких хромосомних хвороб використовують визначення статевого хроматину. Назвіть хворобу, при якій потрібне це визначення:

Трисомія Е

Хвороба Брутона

Синдром Шерешевського-Тернера

Хвороба Дауна

Гемофілія

2689 / 6854
У хворої після видалення матки розвинулась гостра анурія (немає виділення сечі). Які анатомічні структури найімовірніше було пошкоджено при операції?

Цибулинно-губчастий м’яз

Сечоводи

Зовнішній сфінктер сечівника

Сечівник

Внутрішній сфінктер сечівника

2690 / 6854
Дихальний коефіцієнт у хворого складає 0,7. Це свідчить, що у клітинах людини переважає:

Окислення вуглеводів

Окислення жирів

Змішане окислення жирів та вуглеводів

Окислення білків

Змішане окислення жирів та білків

2691 / 6854
У чоловіка при обстеженні виявлено порушення кровообігу міокарда лівого передсердя. У басейні якої артерії відбулись порушення кровообігу?

Права та ліва вінцеві

Ліва вінцева

Передня міжшлуночкова гілка лівої вінцевої артерії

Права вінцева

2692 / 6854
Лікар-цитогенетик при виготовленні метафазної пластинки обробив культуру лейкоцитів гіпотонічним (0,56%) розчином хлориду калію. Після цього відбулося набухання клітин і розрив клітинної мембрани за рахунок надходження води до клітини. Який механізм транспорту має місце в цьому випадку?

Ендоосмос

Піноцитоз

Фагоцитоз

Полегшена дифузія

Дифузія

2693 / 6854
При зовнішньому дослідженні трупа чоловіка 69-ти років, який помер 4 години тому, патологоанатом відмітив, що м’язи померлого мають дуже щільну консистенцію, суглоби згинаються та розгинаються важко. Як називається ця патологоанатомічна ознака смерті?

Трупне заклякання

Трупні гіпостази

Трупне охолодження

Трупне розкладання

Трупне висихання

2694 / 6854
Швидкість проведення збудження нервовими волокнами становить 120 м/сек. Який з наведених чинників, перш за все, забезпечує таку швидкість?

Великий фактор надійності

Велика амплітуда потенціалу дії

Малий поріг деполяризації

Великий потенціал спокою

Наявність мієлінової оболонки

2695 / 6854
У померлої дитини 3-х років за життя мала місце менінгіальна симптоматика, На розтині в м’якій мозковій оболонці макроскопічно виявлені просоподібні вузлики, які мікроскопічно представлені осередком казеозного некрозу з валами епітеліоїдних, лімфоїдних клітин, між якими зустрічаються великі клітини з ядрами на периферії у вигляді півмісяця. Який найбільш імовірний менінгіт у дитини?

Сифілітичний

Грипозний

Менінгококовий

Туберкульозний

Бруцельозний

2696 / 6854
Лікар записав в історії хвороби, що у хворого дихання поверхневе (знижена глибина дихання). Це означає, що зменшеним є такий показник зовнішнього дихання:

Життєва ємність легень

Хвилинний об’єм дихання

Дихальний об’єм

Ємність вдиху

Функціональна залишкова ємність

2697 / 6854
При проведенні амніоцентезу в клітинах плоду виявлено по 2 тільця статевого хроматину (тільця Барра). Для якого захворювання характерна дана ознака?

Синдром Патау

Трисомія X

Синдром Клайнфельтера

Синдром Дауна

Синдром Шерешевського-Тернера

2698 / 6854
Відомо, що типові патологічні процеси розвиваються за однаковими закономірностями в різних органах і тканинах та у різних видів тварин. Яке з перерахованих явищ можна віднести до типового патологічного процесу?

Гіпертонічна хвороба

Туберкульоз

Пухлина

Інфаркт міокарда

Непрохідність кишківника

2699 / 6854
У хворого 49-ти років на гострий панкреатит виникала загроза некрозу підшлункової залози, що супроводжувалось надходженням у кров і тканини активних панкреатичних протеїназ і розщеплення тканинних білків. Які захисні фактори організму можуть інгібувати ці процеси?

α2-макроглобулін, α1-антитрипсин

Церулоплазмін, трансферин

Гемоплексин, гаптоглобін

Кріоглобулін, інтерферон

Імуноглобуліни

2700 / 6854
У 19-місячної дитини із затримкою розвитку та проявами самоагресії, вміст сечової кислоти в крові -1,96 ммоль/л. При якому метаболічному порушенні це спостерігається?

Хвороба Гірке

Синдром Леша-Ніхана

Подагра

Синдром набутого імунодефіциту

Хвороба Іценко-Кушінга

2701 / 6854
Чоловік 53-х років звернувся зі скаргами на гострий біль у правому підребер’ї. При огляді лікар звернув увагу на пожовтілі склери хворого. Лабораторні аналізи показали підвищену активність АЛТ та негативну реакцію на стеркобілін у калі. Для якого захворювання характерні такі симптоми?

Хронічний гастрит

Гепатит

Гемолітична жовтяниця

Хронічний коліт

Жовчнокам’яна хвороба

2702 / 6854
Основними тригерами, що включають ефекторні системи клітини у відповідь на дію гормонів, є протеїнкінази, які змінюють каталітичну активність певних регуляторних ферментів шляхом АТФ-залежного фосфорилювання. Який із наведених ферментів є активним у фосфорильованій формі?

Піруваткіназа

Глікогенфосфорилаза

Глікогенсинтаза

ГОМГ-КоА-редуктаза

Ацетил-КоА-карбоксилаза

2703 / 6854
У пацієнта встановлено порушення синтезу та виділення вазопресину. В якому відділі нефрона найбільше порушиться процес сечоутворення?

Проксимальний звивистий каналець

Товста частина петлі Генле

Клубочок

Збірна трубочка

Тонка частина петлі Генле

2704 / 6854
При обстеженні 2-х місячної дитини педіатр звернула увагу, що плач дитини нагадує котячий крик. Діагностовані мікроцефалія і вада серця. За допомогою цитогенетичного метода з’ясований каріотип дитини 46, XX, 5р-. Дане захворювання є наслідком такого процесу:

Дуплікація

Інверсія

Делеція

Плейотропія

Транслокація

2705 / 6854
В експерименті подразнюють гілочки симпатичного нерва, які іннервують серце. Це призвело до збільшення сили серцевих скорочень, тому що через мембрану типових кардіоміоцитів збільшився:

Вхід іонів кальцію та калію

Вихід іонів кальцію

Вихід іонів калію

Вхід іонів калію

Вхід іонів кальцію

2706 / 6854
У пацієнта 60-ти років виявлено збільшення порогу сприймання звуків високої частоти. Зміна функцій яких структур слухового аналізатора зумовлює виникнення цього порушення?

Органу Корті ближче до гелікотре-ми

Барабанної перетинки

М’язів середнього вуха

Органу Корті ближче до овального віконця

Євстахієвої труби

2707 / 6854
У пацієнта виявлено кишкову непрохідність, знижений апетит, нудоту, блювання. На основі проведеної лабораторної діагностики встановлено дифілоботріоз. Зараження відбулось через вживання:

Яєць

Яловичини

Риби

Свинини

Крабів та раків

2708 / 6854
В інфекційну клініку доставлено хворого із проявами лихоманки, що повторюється вдруге з інтервалом 2 дні. В краплі крові, зафарбованій за Романовським-Гімзою, виявлено звивисті клітини синьо-фіолетового кольору. Який мікроорганізм викликав захворювання?

Ricketsia typhi

Borrelia recurentis

Leptospira interrogans

Treponema pallidum

Plasmodium vivax

2709 / 6854
У юнака 18-ти років діагностовано хворобу Марфана. При дослідженні встановлено: порушення розвитку сполучної тканини, будови кришталика ока, аномалії серцево- судинної системи, арахнодактилія. Яке генетичне явище зумовило розвиток цієї хвороби?

Кодомінування

Плейотропія

Множинний алелізм

Неповне домінування

Комплементарність

2710 / 6854
При алкаптонурії відбувається надмірне виділення гомогентизинової кислоти із сечею. З порушенням метаболізму якої амінокислоти пов’язано виникнення цього захворювання?

Аспарагін

Фенілаланін

Тирозин

Метіонін

Аланін

2711 / 6854
Чоловік 30-ти років, водій за професією, страждає на алергічний риніт із загостренням у весняний період. Лікар призначив хворому антигістамінний засіб з незначним седативним ефектом та тривалістю дії близько 24 годин. Який із перерахованих засобів було призначено?

Гепарин

Вікасол

Димедрол

Окситоцин

Лоратадин

2712 / 6854
Хворий госпіталізований з небезпекою поширення запального процесу з потиличної ділянки в порожнину черепа. Крізь яке анатомiчне утворення можливе це поширення?

Остистий отвір

Тім ’яний отвір

Овальний отвір

Виростковий канал

Круглий отвір

2713 / 6854
Після введення пеніцилину в хворого розвився набряк Квінке. Який препарат екстреної терапії необхідно ввести хворому?

Сульфацил-натрій

Аскорбінова кислота

Преднізолон

Рифампіцин

Но-шпа

2714 / 6854
Під час електронномікроскопічного дослідження біоптату гепатоцитів на біліарному полюсі виявлено велику кількість плоских цистерн, сплющених у центральній частині й розширених на периферії, та дрібних міхурців із секреторними гранулами. Назвіть цю структуру:

Піноцитозні міхурці

Комплекс Гольджі

Мікротрубочки

Ендоплазматична сітка

Лізосома

2715 / 6854
У кішки з децеребраційною ригідністю потрібно знизити тонус м’язів. Цього можна досягти шляхом:

Руйнування вестибулярних ядер Дейтерса

Подразнення вестибулярних ядер Дейтерса

Подразнення ампулярних вестибулорецепторів

Подразнення отолітових вестибулорецепторів

Подразнення вестибулослухового нерва

2716 / 6854
При гастробіопсії у хворого встановлена метаплазія поверхневого епітелію слизової оболонки, який замість циліндричного набув вигляду кишкового. Разом з тим спостерігається склероз на місці залоз слизової оболонки та лімфогістіоцитарна інфільтрація. Про яке захворювання шлунка можна думати?

Корозивний гастрит

Поверхневий хронічний гастрит

Хронічний гастрит з ураженням залоз без атрофії

Хронічний атрофічний гастрит

Ерозивний гастрит

2717 / 6854
При дослідженні імунного статусу людини обов’язково визначають кількість імуноглобулінів різних класів. Яка з перерахованих реакцій використовується для цього?

Радіальної імунодифузії

Бласттрансформації

Подвійної імунодифузії

Оберненої непрямої гемаглютинації

Ланцюгово-полімеразна

2718 / 6854
У хворого, що страждає на спадкову хворобою Хартнупа, спостерігаються пелагроподібні ураження шкіри, порушення розумового розвитку в результаті нестачі нікотинової кислоти. Причиною цього захворювання є порушення такого процесу:

Всмоктування і реабсорбція в нирках метіоніну

Трансамінування фенілаланіну

Всмоктування і реабсорбція цистеїну

Всмоктування і реабсорбція в нирках триптофану

Декарбоксилювання триптофану

2719 / 6854
Хворий помилково прийняв надмірну дозу тироксину. До яких змін секреції тиреоліберину та тиреотропіну це призведе?

Секреція гормонів зменшиться

Змін секреції гормонів не буде

Секреція гормонів збільшиться

Секреція тиреотропіну збільшиться, тиреоліберину - зменшиться

Секреція тиреоліберину збільшиться, тиреотропіну - зменшиться

2720 / 6854
Проводять дуоденальне зондування. Що із наведеного доцільно ввести людині під шкіру, щоб суттєво збільшити надходження до дванадцятипалої кишки жовчі?

Секретин

Гастрин

Холецистокінін-панкреозимін

Нейротензин

Соматостатин

2721 / 6854
У чоловіка 32-х років, хворого на пневмонію, спостерігається закупорка харкотинням дихальних шляхів. В організмі хворого при цьому буде розвиватися така зміна кислотнолужної рівноваги:

Змін не буде

Респіраторний алкалоз

Метаболічний ацидоз

Респіраторний ацидоз

Метаболічний алкалоз

2722 / 6854
При аналізі ЕКГ виявлено випадіння деяких серцевих циклів PQRST. Наявні зубці і комплекси не змінені. Назвіть вид аритмії:

Синоатріальна блокада

Передсердна екстрасистола

Внутрішньопередсердна блокада

Миготлива аритмія

Атріовентрикулярна блокада

2723 / 6854
У хворої встановлено порушення виділення тиреотропного гормону гіпофіза. Зі зниженням функцій якої частки гіпофіза це пов’язано?

Infundibulum

Lobus anterior

Lobus posterior

Pars intermedia

2724 / 6854
У пацієнтки 23-х років після використання нової губної помади з’явилися набряк і свербіння губ, а через 2 дні - кірочки на червоній облямівці губ. Який тип алергічної реакції найбільш імовірний?

Анафілактичний

Стимулюючий

Сповільнений

Цитотоксичний

Імунокомплексний

2725 / 6854
У хворого лікар виявив накопичення рідини в плевральній порожнині справа над куполом діафрагми. У якому анатомічному утворі накопичилась рідина?

Діафрагмально-середостінний синус

Реброво-діафрагмальний лівий синус

Реброво-середостінний синус

Реброво-діафрагмальний правий синус

2726 / 6854
При регенерації епітелію слизової оболонки порожнини рота (розмноження клітин) відбулася реплікація (авторепродукція) ДНК за напівконсервативним механізмом. При цьому нуклеотиди нової нитки ДНК є комплементарними до:

Материнської нитки

Ферменту ДНК-полімерази

Ферменту РНК-полімерази

Інтронних ділянок гену

Змістовних кодонів

2727 / 6854
В результаті травми порушено цілісність переднього корінця спинного мозку. Які відростки яких нейронів при цьому пошкоджені?

Аксони чутливих нейронів

Дендрити чутливих нейронів

Дендрити рухових нейронів

Дендрити вставних нейронів

Аксони рухових нейронів

2728 / 6854
При визначенні повітряної та кісткової провідності звуку було встановлено, що у пацієнта ліве вухо краще сприймає звук при кістковому його проведенні, що могло бути пов’язане з захворюванням:

Середнього вуха справа

Внутрішнього вуха зліва

Внутрішнього вуха справа

Середнього вуха зліва

Зовнішнього вуха справа

2729 / 6854
Чоловік 55-ти років доставлений до реанімаційного відділення без свідомості. Зі слів родичів стало відомо, що хворий помилково випив метиловий спирт. Який антидот необхідно використати в даному випадку?

Протаміну сульфат

Етанол

Ацетилцистеїн

Налоксон

Тетурам

2730 / 6854
Хвора на ревматоїдний артрит після трьохтижневого лікування преднізолоном почала скаржитись на перебої в ро6оті серця. З чим пов’язаний розвиток даного небажаного ефекту препарату?

Гіперглікемія

Гіпоглікемія

Гіперкаліємія

Гіперурікемія

Гіпокаліємія

2731 / 6854
Для профілактики атеросклерозу, ішемічної хвороби серця, порушень мозкового кровообігу рекомендується споживання жирів із високим вмістом поліненасичених жирних кислот. Однією з таких жирних кислот є:

Лінолева

Стеаринова

Пальмітоолеїнова

Лауринова

Олеїнова

2732 / 6854
Обстежуваний знаходиться у фазі швидкохвильового сну. При цьому на ЕЕГ реєструється:

δ-хвиля

α-хвиля

β-хвиля

α-веретено

θ-хвиля

2733 / 6854
Внаслідок ДТП у потерпілої 37-ми років виникло неутримання сечі. Які сегменти спинного мозку пошкоджені?

Th1 — Th5

Th1 — L1

S2 - S4

Th2 — Th5

L1 — L2

2734 / 6854
Жінці 26-ти років, хворій на бронхіт, призначили засіб етіотропної терапії - антибіотик широкого спектру дії. Який це препарат?

Доксициклін

Амброксол

Дексаметазон

БЦЖ-вакцина

Інтерферон

2735 / 6854
При обстеженні жінки 56-ти років, що хвора на цукровий діабет 1-го типу, виявлене порушення білкового обміну, що при лабораторному дослідженні крові проявляється аміноацидемією а клінічно - уповільненням загоєння ран і зменшенням синтезу антитіл. Який з перерахованих механізмів викликає розвиток аміноацидемії?

Підвищення протеолізу

Підвищення онкотичного тиску в плазмі крові

Гіперпротеїнемія

Збільшення ліпопротеїдів високої щільності

Зменшення концентрації амінокислот у крові

2736 / 6854
Хворому на туберкульоз легень було призначено препарат з групи антибіотиків, що відноситься до високоефективних протитуберкульозних засобів. Який препарат був призначений?

Рифампіцин

Цефалексин

ПАСК

Циклосерин

Тетрациклін

2737 / 6854
При дослідженні тимуса дитини 5-ти років, що померла від гострої деструктивної стафілококової пневмонії, виявлено зменшення маси залози до 3,0 г. При гістологічному дослідженні в тимусі знайдено: зменшення часточок залози, значне зменшення кількості лімфоцитів, з колапсом строми часточок, інверсія шарів, кистоподібне збільшення тілець Гассаля. Який з перелічених діагнозів найбільш імовірний?

Гіпоплазія тимусу

Тимомегалія

Акцидентальна інволюція тимусу

Агенезія тимусу

Дисплазія тимусу

2738 / 6854
При обстеженні хворого окуліст виявив збільшення часу адаптації ока до темряви. Нестача якого вітаміну може бути причиною такого симптому?

C

B2

K

A

B6

2739 / 6854
Хворому 63-х років з атонією сечового мiхура лікар призначив препарат, дозу якого хворий самостійно збільшив. З’явились підвищене потовиділення, салівація, діарея, м’язові спазми. Препарат якої групи був призначений?

Гангліоблокатори

Холіноміметики

Адреноблокатори

Реактиватори холінестерази

Токолітики

2740 / 6854
На мікропрепараті плівки пухкої сполучної тканини видно клітину овальної форми, зі світлою цитоплазмою і великим ядром що містить специфічний малюнок гетерохроматину у вигляді циферблату годинника (або спиць колеса). Яка клітина у полі зору?

Тканинний базофіл

Адипоцит

Плазмоцит

Фібробласт

Макрофаг

2741 / 6854
Для лікування піодермії лікар призначив вакцину, яка виготовлена зі штаму бактерій, виділених від хворого. До якого типу вакцин належить даний препарат?

Хімічна вакцина

Генноінженерна вакцина

Асоційована вакцина

Атенуйована вакцина

Аутовакцина

2742 / 6854
У хворого скарги на загальну слабкість, підвищену втому, зниження апетиту і маси тіла. В анамнезі часті пневмонії. На підставі клінічних даних та результатів дослідження периферійної крові у нього діагностовано хронічний лімфолейкоз. Які дегенеративні зміни лейкоцитів характерні для даного захворювання?

Палички Ауера

Зерна Амато

Тільця Князькові-Деле

Токсична зернистість

Тіні Боткіна-Гумпрехта

2743 / 6854
У чоловіка 72-х років довготривала хронічна патологія легень призвела до недостатності клапанів легеневої артерії і трикуспідального клапану, недостатності кровообігу за правошлуночковим типом. Який тип артеріальної гіпертензії є причиною перевантаження серця об’ємом?

Есенціальна гіпертензія

Центрально-ішемічна гіпертензія

Сольова гіпертензія

Легенева гіпертензія

Рефлексогенна гіпертензія

2744 / 6854
Хворому для лікування серцевої недостатності було призначено серцевий глікозид. Яка супутня патологія може сприяти кумуляції серцевих глікозидів?

Гіпоацидний гастрит

Ниркова недостатність

Анорексія

Гіперацидний гастрит

Гіпертонічна хвороба

2745 / 6854
Чоловік 40-ка років хворіє на гіперацидний гастрит з нічними голодними болями. Призначте хворому лікарський засіб - блокатор гістамінових Н2 -рецепторів III покоління, який знизить виділення хлористоводневої кислоти (особливо вночі) та збільшить утворення захисного слизу:

Фамотидин

Атропіну сульфат

Пірензепін

Платифіліну гідротартрат

Метацин

2746 / 6854
П’ятирічна дитина-правша після черепно-мозкової травми на деякий час втратила здатність розмовляти, але через тривалий час ця здатність у неї відновилась. Яка півкуля була травмована й за рахунок якої властивості ЦНС дітей відновлення мови стало можливим?

Ліва півкуля, інертність

Ліва півкуля, пластичність

Обидві півкулі, інертність

Права півкуля, рухливість

Права півкуля, пластичність

2747 / 6854
Чоловік 35-ти років помер у приймальному відділенні лікарні, куди був доставлений у шоковому стані. На розтині тіла виявлено аневризму дуги аорти з її розривом та масивною кровотечею. Мікроскопічно: в медії аорти скупчення лімфоцитів та плазмоцитів, поодинокі гігантські клітини Пирогова-Лангханса, фібробласти. Спостерігаються також дрібні вогнища некрозу та руйнування еластичних структур стінки аорти. Яке захворювання зумовило такі зміни?

Атеросклероз

Висипний тиф

Туберкульоз

Сифіліс

2748 / 6854
У дитини 5-ти років розвинулось гостре респіраторне захворювання, яке супроводжувалось кашлем, виділенням значної кількості слизу із носа. Який тип запалення у хворої дитини?

Геморагічне

Фібриноїдне

Гнійне

Гнилісне

Катаральне

2749 / 6854
Чоловікові 58-ми років зроблено операцію з приводу раку простати. Через 3 місяці йому проведено курс променевої та хіміотерапії. До комплексу лікарських препаратів входив 5- фтордезоксиуридин - інгібітор тимідилатсинтази. Синтез якої речовини блокується цим препаратом?

ДНК

і-РНК

р-РНК

т-РНК

2750 / 6854
Стеатоз виникає внаслідок накопичення триацилгліцеролів у гепа-тоцитах. Одним з механізмів розвитку цього захворювання є зменшення утилізації нейтрального жиру ЛПДНЩ. Які ліпотропні речовини попереджують розвиток стеатозу?

Аргінін, B2 , B3

Метіонін, B6 , B12

Аланін, B1 , PP

Валін, B3 , B2

Ізолейцин, B1 , B2

2751 / 6854
У пацієнта, що прибув з ендемічного за малярією району, підвищилася температура тіла, відзначається головний біль, озноб, загальне нездужання - симптоми, що характерні й для звичайної застуди. Які лабораторні дослідження необхідно провести, щоб підтвердити або спростувати діагноз 'малярія'?

Дослідження спинномозкової рідини

Мікроскопія пунктату червоного кісткового мозку

Мікроскопія мазків крові

Дослідження пунктату лімфовузлів

Аналіз сечі

2752 / 6854
У чоловіка 40-ка років внаслідок посиленого гемолізу еритроцитів підвищився вміст заліза в плазмі крові. Який білок забезпечує його депонування в тканинах?

Альбумін

Феритин

Гаптоглобін

Транскортин

Трансферин

2753 / 6854
У дитячому дошкільному закладі напередодні новорічних свят було зареєстровано спалах кишкової інфекції. При бактеріологічному дослідженні випорожнень хворих патогенних бактерій не було виділено. При електронній мікроскопії виявлено утворення округлої форми з чітким обідком і товстою втулкою, які нагадують колесо. Вкажіть найбільш імовірний збудник даної інфекції:

Pvulgaris

Adenovirus

Rotavirus

E.coli

Coxsacki-virus

2754 / 6854
У біоптаті щитоподібної залози виявлено атрофію паренхіматозних елементів, дифузну інфільтрацію тканини залози лімфоцитами та плазматичними клітинами з утворенням в ній лімфоїдних фолікулів. Для якого захворювання є характерними наведені ознаки?

Хвороба Базедова

Ендемічний зоб

Аденома щитоподібної залози

Тиреоїдит Ріделя

Тиреоїдит Хасімото

2755 / 6854
При гістологічному дослідженні біоптатів, взятих з потовщених країв виразки шлунка, виявлені невеликі гніздові скупчення різко атипових гіперхромних невеликих епітеліальних клітин, які розташовані серед дуже розвиненої строми. Визначте пухлину:

Недиференційована саркома

Аденома

Медулярний рак

Скіррозний недиференційований рак

Аденокарцинома

2756 / 6854
При розтині тіла померлого у інфекційному відділенні виявлено: фібринозногнійний перитоніт; в слизовій оболонці клубової кишки багаточислені виразкові дефекти овальної форми до 3-5 см, які розташовані вздовж кишки і повторюють форму пейєрової бляшки, краї виразок рівні, заокруглені, дно чисте, представлене м’язовою або серозною оболонкою. В дні двох виразок знайдені отвори до 0,3 см в діаметрі. Для якого захворювання найбільш характерні ці зміни?

Дизентерія

Неспецифічний виразковий коліт

Хвороба Крона

Черевний тиф

Паратиф

2757 / 6854
Хворий 78-ми років з хворобою Паркінсона приймає препарати левадопи (наком). Яка антипаркінсонічна дія у цього засобу?

Блокуюча гістамінергічна дія

М-холіноміметична

Адреноблокуюча

М-холіноблокуюча

Допамінергічна

2758 / 6854
У хворого 47-ми років виникла кишкова коліка на фоні гіпертонічної хвороби. Засоби якої з перерахованих груп найбільш доцільно використати для її купірування у даній ситуації?

Симпатоміметики

М-холиноміметики

Адреноміметики

Міотропні спазмолітики

Антихолінестеразні засоби

2759 / 6854
Під час ректороманоскопії хворого зі скаргами на діарею виявлено, що слизова оболонка прямої і сигмоподібної кишок різко гіперемована, набрякла, вкрита великою кількістю слизу, а у деяких ділянках вкрита плівчастими накладаннями зеленуватого кольору. Про яке захворювання можна думати?

Сальмонельоз

Холера

Амебіаз

Черевний тиф

Дизентерія

2760 / 6854
У хворого на ЕКГ виявлено збільшення тривалості комплексу QRS. Наслідком чого це може бути?

Збільшення часу охоплення збудженням передсердь

Збільшення збудливості передсердь

Порушення провідності у атріовентрикулярному вузлі

Збільшення збудливості шлуночків та передсердь

Збільшення часу охоплення збудженням шлуночків

2761 / 6854
Хвороба Андерсена належить до групи спадкових хвороб, що розвиваються внаслідок уродженої недостатності синтезу певних ферментів глікогенолізу. Недостатність якого ферменту є молекулярною основою цього глікогенозу?

Аміло(1,4-1,6)трансглікозидаза

Глікогенсинтаза

Фосфофруктокіназа

Лізосомальні глікозидази

Глюкозо-6-фосфатази

2762 / 6854
У хворого з варикозним розширенням вен під час огляду нижніх кінцівок відзначається: ціаноз, пастозність, зниження температури шкіри, поодинокі петехії. Який розлад гемодинаміки має місце у хворого?

Венозна гіперемія

Артеріальна гіперемія

Компресійна ішемія

Тромбоемболія

Обтураційна ішемія

2763 / 6854
Після прийому сульфаніламідів у хворого виникли лихоманка, блювання і стул з кров’ю. У крові: лейк.-0,9 • 109 /л (гранул.- 0,7 • 109 /л), лейкоаглютиніни. Який з термінів найбільш точно характеризує виявлені зміни у крові?

Лейкоз

Агранулоцитоз

Гемодилюція

Лейкопенія

2764 / 6854
При гістологічному дослідженні органів і тканин померлої від ниркової недостатності молодої жінки, у якої прижиттєво виявлявся високий титр антинуклеарних антитіл, виявлені поширені фібриноїдні зміни в стінках судин. Відмічається ядерна патологія з вакуолізацією ядер, каріоре-ксисом, утворенням гематоксилінових тілець. Який найбільш імовірний діагноз?

Гіпертонічна хвороба

Вузликовий періартеріїт

Атеросклероз

Облітеруючий ендартеріїт

Системний червоний вовчак

2765 / 6854
У недоношеного немовляти спостерігається жовтяниця. З нестачею у нього якого ферменту це пов’язано?

Кисла фосфатаза

Лужна фосфатаза

НАД+- дегідрогеназа

Каталаза

УДФ-трансглюкуронідаза

2766 / 6854
У хворого пухлина грудного відділу стравоходу. Куди можуть безпосередньо метастазувати пухлинні клітини?

Ductus thoracicus

Nodi intercostales

Nodi mediastinales

Nodi gastrici

Nodi hepatici

2767 / 6854
При дослідженні вмісту дванадцятипалої кишки людини знайдені найпростіші грушоподібної форми з парними ядрами, чотирма парами джгутиків. Між ядрами - дві опірні нитки, з вентрального боку розташований присмоктувальний диск. Який представник найпростіших виявлений у хворого?

Токсоплазма

Лямблія

Трихомонада кишкова

Трипаносома

Лейшманія

2768 / 6854
36-ти років має місце гіповітаміноз B2 168. . Причиною виникнення специфічних симптомів (ураження епітелію, слизових, шкіри, рогівки ока) імовірно є дефіцит:

Цитохрому В

Флавінових коферментів

Цитохромоксидази

Цитохрому С

Цитохрому А1

2769 / 6854
При диспансерному обстеженні у хворого знайдено цукор в сечі. Який найбільш імовірний механізм виявлених змін, якщо вміст цукру в крові нормальний?

Порушення фільтрації глюкози в клубочковому відділі нефрона

Порушення реабсорбції глюкози в канальцях нефрона

Гіперпродукція глюкокортикоїдів наднирниками

Інсулінорезистентність рецепторів клітин

Недостатня продукція інсуліну підшлунковою залозою

2770 / 6854
На розтині в серці виявлено наступні зміни: великий осередок некрозу білого кольору з червоною облямівкою, який захоплює всю товщу серцевого м’яза. На зовнішній оболонці серця - ознаки фібринозного перикардиту. Який найбільш імовірний діагноз?

Міокардит

Субендокардіальний інфаркт міокарда

Субепікардіальний інфаркт міокарда

Трансмуральний інфаркт міокарда

Інтрамуральний інфаркт міокарда

2771 / 6854
На електронній мікрофотографії представлена клітина нейрального походження. Термінальна частина дендрита клітини має циліндричну форму і складається з 1000 замкнутих мембранних дисків. Яка клітина зображена на мікрофотографії?

Колбочкова нейросенсорна

Нейрон кори великих півкуль

Паличкова нейросенсорна

Нейрон спинномозкового вузла

Нейрон передніх рогів спинного мозку

2772 / 6854
Порушення процесів мієлінізації нервових волокон призводить до неврологічних розладів і розумової відсталості. Такі симптоми характерні для спадкових і набутих порушень обміну:

Нейтральних жирів

Сфінголіпідів

Вищих жирних кислот

Фосфатидної кислоти

Холестерину

2773 / 6854
У хворого 15-ти років концентрація глюкози натще 4,8 ммоль/л, через годину після цукрового навантаження - 9,0 ммоль/л, через 2 години - 7,0 ммоль/л, через 3 години - 4,8 ммоль/л. Ці показники характерні для такого захворювання:

Цукровий діабет II типу

Цукровий діабет I типу

Прихований цукровий діабет

Хвороба Іценко-Кушінга

2774 / 6854
У чоловіка 48-ми років виявлено порушення периферичного кровообігу з обмеженням припливу артеріальної крові, при цьому має місце збліднення даної ділянки, зниження місцевої температури. Це порушення називається:

Ішемія

Стаз

Реперфузійний синдром

Сладж

Венозна гіперемія

2775 / 6854
Хворому хірург видалив порожнинне утворення печінки діаметром 2 см. Встановлено, що стінка порожнини утворена щільною волокнистою сполучною тканиною, вміст являє собою каламутну, густу, жовтувато-зеленуватого кольору рідину з неприємним запахом, яка мікроскопічно складається переважно з поліморфноядерних лейкоцитів. Якому патологічному процесу відповідають такі морфологічні зміни?

Хронічний абсцес

Флегмона

Гострий абсцес

Емпієма

2776 / 6854
Лікар призначив пацієнту з хронічним бронхітом відхаркувальний засіб, який діє шляхом розщеплення дисульфідних зв’язків глікозаміногліканів харкотиння, зменшуючи цим його в’язкість, проте попередив хворого про можливий бронхоспазм при його використанні. Який засіб був призначений?

Натрію гідрокарбонат

Трава термопсису

Бромгексин

Лібексин

Ацетилцистеїн

2777 / 6854
Лікар призначив хворому з гострою серцевою недостатністю не-глікозидний кардіотонічний засіб, який безпосередньо стимулює β1 -адренорецептори міокарда, що збільшує кровообіг, діурез. Застосовується лише внутрішньовенно крапельно внаслідок швидкої інактивації в організмі. Який препарат призначив лікар?

Адреналін

Дигоксин

Анаприлін

Добутамін

Корглікон

2778 / 6854
Чоловік 40-ка років перебував у пульмонологічному відділенні з приводу рецидивуючої правосторонньої пневмонії. Помер від легенево-серцевої недостатності. На розтині в правій лєгєні визначається ділянка круглої форми 3х4 см. Вона являє собою порожнину з нерівними шорсткими краями, заповнену каламутною вершкоподібною жовто-зеленою рідиною. Мікроскопічно: стінка порожнини утворена тканиною легені з дифузною інфільтрацією лейкоцитами. Визначте патологічний процес у легені:

Гострий абсцес

Емпієма

Інфаркт

Гангрена

Хронічний абсцес

2779 / 6854
Робочий комунальної служби спустився в каналізаційний колодязь без засобів захисту і через деякий час знепритомнів. Лікарями швидкої допомоги діагностовано отруєння сірководнем. Який вид гіпоксії при цьому розвинувся?

Гемічний

Респіраторний

Перевантажувальний

Тканинний

Циркуляторний

2780 / 6854
Під час розтину тіла жінки 52-х років, яка тривалий час хворіла на жовчно-кам’яну хворобу, було знайдено: макроскопічно - печінка помірно збільшена, деформована, поверхня органу горбиста, тканина щільна, на розрізі тканина коричнева з зеленим відтінком, складається з множинних вузликів діаметром 8-10 мм. Мікроскопічно - гепатоцелюлярні вузлики оточені прошарками сполучної тканини, яка містить збільшену кількість дрібних жовчних протоків з холестазом. Діагностуйте захворювання печінки:

Токсична дистрофія печінки

Біліарний цироз печінки

Портальний цироз печінки

Холелітіаз

Постнекротичний цироз печінки

2781 / 6854
При гістологічному дослідженні біоптату, отриманого із нижньої третини стравоходу 57- річного чоловіка із симптомами тривалого рефлюксу шлункового вмісту, виявлено наступні зміни: у слизовій оболонці на місці багатошарового плоского епітелію визначається одношаровий залозистий призматичний епітелій, з ознаками продукції слизу. Вкажіть патологічний процес, який виник у слизовій оболонці:

Метаплазія

Організація

Гіпертрофія

Регенерація

Гіперплазія

2782 / 6854
Хворій жінці із захворюванням нирок, що супроводжується вираженими набряками, призначили діуретичний препарат, що пригнічує реабсорбцію в нирках іонів Na+ і води, посилює виведення нирками іонів K+ і Mg++, викликає гіперурикемію, зумовлює потужний діуретичний ефект. Назвіть цей препарат:

Діакарб

Аллопуринол

Фуросемід

Спіронолактон

Тріамтерен

2783 / 6854
Лікар-стоматолог для лікування гінгівіту призначив пацієнту препарат з протипротозойною та антибактеріальною діями, який може викликати відразу до алкоголю. Вкажіть препарат, який призначив лікар:

Левоміцетин

Тетрациклін

Метронідазол

Лінкоміцину гідрохлорид

Цефтріаксон

2784 / 6854
Аналіз ЕКГ хворого виявив відсутність зубця Р. Тривалість та амплітуда QRS комплексу та зубця Т відповідають нормі. Що є водієм ритму серця даного пацієнта?

Передсердно-шлуночковий вузол

Волокна Пуркін’є

Синусовий вузол

Міокард шлуночків

Пучок Гіса

2785 / 6854
Спеціальний режим харчування призвів до зменшення іонів Ca2+ в крові. До збільшення секреції якого гормону це призведе?

Вазопресин

Паратгормон

Соматотропін

Тироксин

Тирокальцитонін

2786 / 6854
Хворій 43-х років для лікування бронхопневмонії призначена бензилпеніциліну натрієва сіль. Який з вказаних побічних ефектів найбільш характерний для даного засобу?

Неврит слухового нерва

Алергічні реакції

Агранулоцитоз

Ураження печінки

Анемія

2787 / 6854
Пацієнт із захворюванням першого верхнього різця зліва скаржиться на сильні болі шкіри в ділянці надбрівної дуги з того ж боку. Реалізація якого виду рефлексів спричиняє вказані реакції?

Вісцеро-вісцеральні

Вісцеро-соматичні

Вісцеро-дермальні

Пропріоцептивні

Сомато-вісцеральні

2788 / 6854
Потерпілий 19-ти років доставлений до травматологічного відділення з різаною раною трапецієподібного м’яза. Яка з фасцій шиї формує піхву для даного м’яза?

Передхребтова пластинка шийної фасції

Сонна піхва шийної фасції

Вісцеральна частина передтрахейної пластинки шийної фасції

Поверхнева пластинка шийної фасції

М’ язова частина передтрахейної пластинки шийної фасції

2789 / 6854
У хворого діагностовано ГРВІ. У сироватці крові знайдено імуноглобуліни класу М. Який період інфекційного процесу в даному випадку?

Гострий

Інкубаційний

Мікробоносійство

Реконвалесценція

Продромальний

2790 / 6854
У хворого 34-х років після перенесеної кишкової інфекції, викликаної сальмонелами, стали згасати симптоми захворювання. Імуноглобуліни якого класу будуть виявлені в крові хворого в період реконвалесценції?

IgG

IgM

IgA

IgD

IgE

2791 / 6854
До шпиталю було доставлено юнака 16-ти років, хворого на інсулінозалежний цукровий діабет. Рівень глюкози у крові пацієнта складав 18 ммоль/л. Хворому було введено інсулін. Дві години потому рівень глюкози зменшився до 8,2 ммоль/л, тому що інсулін:

Стимулює розщеплення глікогену у м’язах

Стимулює транспорт глюкози через плазматичні мембрани в головному мозку та печінці

Стимулює розщеплення глікогену в печінці

Стимулює перетворення глюкози в печінціу глікоген та ТАГ

Гальмує синтез кетонових тіл із глюкози

2792 / 6854
Хворий на трансмуральний інфаркт міокарда лівого шлуночка переведений до відділення реанімації у важкому стані. АТ- 70/50 мм рт.ст., ЧСС- 56/хв., ЧД- 32/хв. Зазначте головну ланку в патогенезі кардіоген-ного шоку:

Падіння периферичного судинного опору

Падіння серцевого викиду

Втрата води

Втрата електролітів

Крововтрата

2793 / 6854
Обстеження пацієнта з високим артеріальним тиском показало в нього вторинну артеріальну гіпертензію. Причиною такого стану є ренін-продукуюча пухлина нирки. Що є головною ланкою в патогенезі вторинної артеріальної гіпертензії в хворого?

Недостатня продукція катехоламінів

Гіперпродукція ангіотензину 2, альдостерону

Недостатня продукція вазопресину

Гіперпродукція кортизолу

Гіпєрпродукція інсуліну

2794 / 6854
У чоловіка 29-ти років з ножовим пораненням шиї визначається кровотеча. При первинній обробці рани встановлено, що пошкоджена судина, розташована вздовж латерального краю груднинно-ключично-соскоподібного м’яза. Визначте цю судину:

A. carotis externa

V. jugularis interna

V jugularis externa

V jugularis anterior

A. carotis interna

2795 / 6854
У хворого переливання крові ускладнилося розвитком гемотрансфузійного шоку. Назвіть тип алергічної реакції, що лежить в основі даної патології:

Гіперчутливість сповільненого типу

Анафілактичний

Рецептороопосередкований

Імунокомплексний

Цитотоксичний

2796 / 6854
Підшлункова залоза - орган змішаної секреції. Ендокринно продукує бета-клітинами гормон інсулін, який впливає на обмін вуглеводів. Як він впливає на активність глікогенфо-сфорилази (ГФ) і глікогенсинтетази (ГС)?

Активує ГФ, пригнічує ГС

Не впливає на активність ГФ і ГС

Пригнічує ГФ, активує ГС

Активує ГФ і ГС

Пригнічує ГФ і ГС

2797 / 6854
Призначення доксицикліну гідрохлориду викликало порушення симбіозу мікробної флори в кишечнику. Визначити тип порушень при антибіотикотерапії:

Ідіосинкразія

Дисбактеріоз

Сенсибілізація

Бактеріоз

Суперінфекція

2798 / 6854
Для проведення анальгезії наркотичний анальгетик застосували з препаратом бензодіазепінового ряду. Який засіб використали для потенціювання анальгезії?

Карбамазепін

Діазепам

Трифтазин

Хлорпротіксен

Імізин

2799 / 6854
У хворого стенокардія. Який антиангінальний засіб протипоказаний для лікування, якщо в пацієнта алергія на йод?

Нітрогліцерин

Дротаверин

Верапаміл

Аміодарон

Нітросорбід

2800 / 6854
Молода людина 25-ти років споживає надмірну кількість вуглеводів (600 г на добу), що перевищує її енергетичні потреби. Який процес буде активуватися в організмі людини у даному випадку?

Ліпогенез

Окиснення жирних кислот

Ілюконеогенез

Гліколіз

Ліполіз

2801 / 6854
В шкірі виявлена щільна, рухома, чітко відмежована від оточуючих тканин пухлина. На розрізі вона білого кольору, представлена волокнистою тканиною. Мікроскопічно: хаотично переплетені колагенові волокна, клітин мало. Що це за пухлина?

Десмоїд

Міома

Дерматофіброма

Фіброма

Гістіоцитома

2802 / 6854
Для корекції артеріального тиску при колаптоїдному стані хворому було введено мезатон. Який механізм дії даного препарату?

Блокує β-адренорецептори

Стимулює α-адренорецептори

Стимулює β-адренорецептори

Стимулює α- і β-адренорецептори

Блокує α-адренорецептори

2803 / 6854
Після переходу до змішаного харчування у новонародженої дитини виникла диспепсія з діареєю, метеоризмом, відставанням у розвитку. Біохімічна основа даної патології полягає у недостатності:

Ліпази та креатинкінази

Трипсину та хімотрипсину

Целюлази

Лактази та целобіази

Сахарази та ізомальтази

2804 / 6854
У чоловіка 52-х років діагностовано системний амебіаз з ураженням кишківника, печінки, легенів. Який препарат слід призначити?

Ентеросептол

Метронідазол

Хінгамін

Тетрациклін

Хініофон

2805 / 6854
У пацієнта з ознаками коліту виділена чиста культура бактерій, яка за морфологічними, культуральними та біохімічними властивостями належить до роду шигел. Яку з названих реакцій доцільно застосувати для серологічної ідентифікації культури?

Зв’язування комплементу

Аглютинації з діагностичними сироватками

Непрямої гемаглютинації

Затримки гемаглютинації

Преципітації

2806 / 6854
Для лікування урогенітальних інфекцій використовують хінолони - інгібітори ферменту ДНК-гірази. Який процес порушується під дією хінолонів у першу чергу?

Зворотна транскрипція

Репарація ДНК

Реплікація ДНК

Ампліфікація генів

Рекомбінація генів

2807 / 6854
У хворого з аневризмою правої підключичної артерії спостерігається захриплість голосу. З подразненням якого нерву це може бути пов’язано?

N.laringeus superior sinister

N.laringeus superior dexter

N.laringeus inferior sinister

N.laringeus reccurens dexter

N.laringeus reccurens sinister

2808 / 6854
Під час операції в печінці хворого виявлені дрібні пухирці малих розмірів з незначною кількістю рідини, які щільно прилягають один до одного. Який гельмінтоз виявився у хворого?

Фасціольоз

Дікроцеліоз

Опісторхоз

Альвеококоз

Клонорхоз

2809 / 6854
У хлопчика 3-х років з вираженим геморагічним синдромом відсутній антигемофільний глобулін А (фактор VIII) у плазмі крові. Яка фаза гемостазу первинно порушена у цього хворого?

Внутрішній механізм активації протромбінази

Зовнішній механізм активації протромбінази

Ретракція кров’яного згустку

Перетворення фібриногену в фібрин

Перетворення протромбіну в тромбін

2810 / 6854
До лікаря звернувся чоловік 27-ми років. При огляді було виявлено збільшення кистей, стоп та нижньої щелепи. Крім того спостерігалися деформація суглобів (kiphosis), гормональні порушення (імпотенція, атрофія яєчок). Функція якої залози порушена?

Щитоподібна залоза

Передня частка гіпофізу

Прищитоподібні залози

Надниркові залози

Шишкоподібне тіло

2811 / 6854
У чоловіка 25-ти років діагностований гострий дифузний гломерулонефрит. З анамнезу: за 18 днів до прояву хвороби переніс ангіну. Який механізм ураження ниркових клубочків буде спостерігатися у цьому випадку?

Ішемічний

Нефротоксичний

Імунний

Медикаментозний

2812 / 6854
У чоловіка 35-ти років феохромоцитома. В крові спостерігається підвищений рівень адреналіну та норадреналіну, концентрація вільних жирних кислот зросла в 11 разів. Активація якого ферменту під впливом адреналіну підвищує ліполіз?

ТАГ -ліпаза

Фосфоліпаза А2

Холестеролестераза

Фосфоліпаза С

Ліпопротеїдліпаза

2813 / 6854
Жінку 44-х років втяла оса, внаслідок чого розвинувся шок. В анамнезі - тяжка алергічна реакція на укус оси. Об’єктивно: РS- 179/хв, слабкий, АТ- 80/40 мм рт.ст., ЧД-26/хв. Яка провідна ланка патогенезу анафілактичного шоку?

Зменшення ударного об’єму серця

Тахікардія

Зменшення об’єму циркулюючої крові

Зниження периферійного опору судин

Біль

2814 / 6854
Фекалії дитини, що хворіє на ентерит, емульгують в фізіологічному розчині і краплю емульсії наносять на елективне середовище: 10% молочно-сольовий, або жовтковосольовий агар. Який мікроорганізм передбачається виділити?

Клебсієла

Ентерокок

Стафілокок

Стрептокок

Кишкова паличка

2815 / 6854
У хворої 63-х років діагностований інсулінонезалежний цукровий діабет. Ендокринолог почав лікування з призначення глібенкламіду. Вкажіть механізм дії цього засобу:

Підсилює руйнування білків

Стимулює гіпоталамічні центри

Активує в-клітини острівців Лангерганса

Зменшує всмоктування глюкози в кишківнику

Гальмує транспорт глюкози до клітин

2816 / 6854
У хворого коса пахвинна грижа. Яке анатомічне утворення стало слабким місцем передньої черевної стінки?

Пахвинний трикутник

Латеральна пахвинна ямка

Надміхурова ямка

Стегнова ямка

Медіальна пахвинна ямка

2817 / 6854
Людині внутрішньовенно ввели 0,5 л ізотонічного розчину лікарської речовини. Які з рецепторів насамперед відреагують на зміну водно-сольового балансу організму?

Осморецептори гіпоталамусу

Барорецептори дуги аорти

Осморецептори печінки

Натрієві рецептори гіпоталамусу

Волюморецептори порожнистих вен і передсердь

2818 / 6854
До лікарні надійшла дитина з діагнозом 'стафілококовий сепсис'. На яке живильне середовище потрібно посіяти кров хворого з метою виділення збудника?

Середовище Бучіна

Жовчно-сольовий агар

Цукрово-пептонний бульйон

Середовище Плоскірева

М ’ясо-пептонний агар

2819 / 6854
При бактеріоскопічному дослідженні носоглоткового слизу дитини 2,5 років, хворої на назофарингіт, виявлені грамнегативні диплококи, схожі за формою на кавові зерна. Які структури організму дитини найбільш імовірно будуть уражені, якщо ці мікроорганізми проникнуть у кров?

Серцеві клапани

Ниркові гломерули

Оболонки мозку

Сечостатеві шляхи

Лімфатичні вузли

2820 / 6854
У хлопчика 2-х років спостерігається збільшення в розмірах печінки та селезінки, катаракта. В крові підвищена концентрація цукру, але тест толерантності до глюкози в нормі. Спадкове порушення обміну якої речовини є причиною цього стану?

Сахароза

Мальтоза

Фруктоза

Галактоза

Глюкоза

2821 / 6854
В сечі новонародженого визначається цитрулін та високий рівень аміаку. Вкажіть, утворення якої речовини, найімовірніше, порушене у цього малюка:

Креатинін

Креатин

Сечовина

Аміак

Сечова кислота

2822 / 6854
У хворого, що звернувся до лікарню зі скаргами на пронос, діагностували амебну дизентерію. До комплексного лікування був включений доксациклін. Назвіть вид дії призначеного препарату:

Етіотропна дія

Незворотня дія

Основна дія

Пряма дія

Рефлекторна дія

2823 / 6854
Під час обстеження у хворої встановлене ураження дорсальної частини мосту, порушена функція жування. Ядро якого нерва уражене?

Рухове ядро лицевого нерва

Ядро під’язикового нерва

Подвійне ядро блукаючого нерва

Рухове ядро трійчастого нерва

Мостове ядро трійчастого нерва

2824 / 6854
з дихальною недостатністю рН крові 7,35. Визначення рСO2 24. показало наявність гіперкапнії. При дослідженні рН сечі відзначається підвищення її кислотності. Яка форма порушення кислотноосновного стану в даному випадку?

Алкалоз газовий, компенсований

Алкалоз газовий, декомпенсований

Ацидоз метаболічний, компенсований

Ацидоз газовий, компенсований

Ацидоз метаболічний, декомпенсований

2825 / 6854
При бактеріологічному дослідженні сечі хворого на пієлонефрит виділені мікроорганізми, що утворюють на м’ясо-пептонному агарі жовто-зелений пігмент і мають характерний запах. Як вони називаються?

Псевдомонади

Клебсієли

Ешеріхії

Азотобактерії

Протеї

2826 / 6854
У хворого впродовж 10-ти днів має місце підвищена температура, напади характерного кашлю. Лікар призначив посів слизу з носоглотки на середовище КВА. Який мікроорганізм передбачається виявити?

Клебсієла

Паличка коклюшу

Стафілокок

Лістерія

Палочка інфлуенци

2827 / 6854
У нейрохірургічне відділення надійшов 54-річний чоловік зі скаргами на відсутність чутливості шкіри нижньої повіки, латеральної поверхні носа, верхньої губи. Лікар при огляді встановив запалення другої гілки трійчастого нерва. Через який отвір виходить з черепа ця гілка?

Верхня очноямкова щілина

Овальний отвір

Круглий отвір

Рваний отвір

Остистий отвір

2828 / 6854
Пацієнт 46-ти років звернувся до лікаря зі скаргами на болі в дрібних суглобах ніг та рук. Суглоби збільшені, мають вигляд потовщених вузлів. У сироватці встановлено підвищений вміст уратів. Це може бути спричинене:

Порушенням обміну вуглеводів

Порушенням обміну ліпідів

Порушенням обміну піримідинів

Порушенням обміну пуринів

Порушенням обміну амінокислот

2829 / 6854
У хворого з жовтяницею встановлено: підвищення у плазмі крові вмісту загального білірубіну за рахунок непрямого (вільного), в калі та сечі - високий вміст стеркобіліну, рівень прямого (зв’язаного) білірубіну в плазмі крові в межах норми. Про який вид жовтяниці можна думати?

Механічна

Жовтяниця немовлят

Паренхіматозна (печінкова)

Гемолітична

Хвороба Жильбера

2830 / 6854
У хворого з тромбофлебітом нижніх кінцівок раптово після навантаження виникли задишка, різкий біль у грудях, ціаноз, набухання шийних вен. Яке найбільш імовірне порушення кровообігу виникло у хворого?

Тромбоемболія судин головного мозку

Тромбоемболія ворітної вени

Тромбоемболія легеневої артерії

Тромбоемболія мезентеріальних судин

Тромбоемболія вінцевих судин

2831 / 6854
При штовханні штанги спортсмен закидає голову назад для максимального підвищення тонусу м’язів-розгиначів верхніх кінцівок. Де розташовані центри рефлексів, які при цьому виникають?

Рухова кора

Ядра Дейтерса

Базальні ганглії

Спинний мозок

Червоні ядра

2832 / 6854
Хворий 65-ти років, що страждає на атеросклероз, госпіталізований до хірургічного відділення з приводу розлитого гнійного перитоніту. Під час операції діагностовано тромбоз брижових артерій. Яка найбільш імовірна причина перитоніту?

Iшемiчний інфаркт

Геморагічний інфаркт

Стаз

Ішемія компресійна

Ішемія ангіоспастична

2833 / 6854
У крові чоловіка 26-ти років виявлено 18% еритроцитів сферичної, сплощеної, кулястої та остистої форм. Інші еритроцити були у формі двоввігнутих дисків. Як називається таке явище?

Патологічний пойкілоцитоз

Фізіологічний анізоцитоз

Фізіологічний пойкілоцитоз

Патологічний анізоцитоз

Еритроцитоз

2834 / 6854
Хвора страждає від болю в ногах та набряків. В ході обстеження хворої на медіальній поверхні стегна спостерігається набряк, збільшення розміру вен, утворення вузлів. З боку якої вени спостерігається патологія?

V saphena parva

V saphena magna

V profunda femoris

V tibialis

V femoralis

2835 / 6854
Знешкодження ксенобіотиків (лікарських засобів, епоксидів, ареноксидів, альдегідів, нітропохідних тощо) та ендогенних метаболітів (естрадіолу, простагландинів, лейкотрієнів) відбувається в печінці шляхом їх кон’югації з:

Аспарагіновою кислотою

Фосфоаденозином

Глутатіоном

S-Аденозилметіоніном

Гліцином

2836 / 6854
У хворого на гіпертонічну хворобу виявлено високий рівень реніну в крові. Якому з гіпотензивних засобів слід надати перевагу в цьому випадку?

Дихлотіазид

Празозин

Анаприлін

Лізиноприл

Ніфедипін

2837 / 6854
На останньому місяці вагітності вміст фібриногену в плазмі крові в 2 рази вище за норму. Яку швидкість осідання еритроцитів слід при цьому очікувати?

40-50 мм/годину

10-15 мм/годину

0-5 мм/годину

3-12 мм/годину

5-10 мм/годину

2838 / 6854
На розтині у дитини знайдена некротична ангіна, флегмона шиї, гнійний отит, гнійний менінгіт. Ці зміни найбільш характерні для:

Отогенного сепсису

Менінгококової інфекції

Дифтерії зіву

Септичної скарлатини

Токсичної скарлатини

2839 / 6854
У хворого відзначаються періодичні напади серцебиття (пароксизми), сильне потовиділення, напади головного болю. При обстеженні виявлена гіпертензія, гіперглікемія, підвищення основного обміну, тахікардія. При якій патології наднирників спостерігається подібна картина?

Первинний альдостеронізм

Гіпофункція мозкового шару

Гіпофункція кори наднирників

Гіперфункція кори наднирників

Гіперфункція мозкового шару

2840 / 6854
На енцефалограмі людини зареєстровано дельта-ритм. У якому стані вона перебуває?

Активної бадьорості

Парадоксального сну

Засинання

Пасивної бадьорості

Повільного сну

2841 / 6854
У людини з нападом бронхоспазму необхідно зменшити вплив блукаючого нерва на гладеньку мускулатуру бронхів. Які мембранні циторецептори доцільно заблокувати для цього?

М-холінорецептори

Н-холінорецептори

α- та β-адренорецептори

β-адренорецептори

α-адренорецептори

2842 / 6854
Необхідно оцінити рівень збудливості нерва у хворого. Для цього доцільно визначити для нерва наступну величину:

Потенціал спокою

Амплітуда потенціалу дії

Тривалість потенціалу дії

Порогова сила подразника

Критичний рівень деполяризації

2843 / 6854
Хворому з кардіогенним шоком, гіпотензією, ядухою і набряками ввели неглі-козидний кардіотонік. Який саме препарат був введений хворому?

Кофеїн-бензоат натрію

Бемегрид

Кордіамін

Етимізол

Добутамін

2844 / 6854
Біосинтез пуринового кільця відбувається на рибозо-5-фосфаті шляхом поступового нарощення атомів азоту і вуглецю та замикання кілець. Джерелом рибозофосфату є наступний процес:

Гліконеогенез

Гліколіз

Глюконеогенез

Глікогеноліз

Пентозофосфатний цикл

2845 / 6854
У жінки з важкою інтоксикацією, спричиненою сепсисом, який і послужив безпосередньою причиною смерті, на розтині виявлене 'тигрове серце'. Який морфогенетичний механізм розвитку переважно лежить в основі даної дистрофії?

Інфільтрація

Трансформація

Спотворений синтез

Декомпозиція

2846 / 6854
Хворий 32-х років з ураженням ліктьового нерва не може звести до серединної лінії ІІ і V пальці. Функція яких м’язів при цьому порушена?

Тильні міжкісткові м’язи

Червоподібні м’язи

Відвідний м’яз мізинця

Короткий долонний м’яз

Долонні міжкісткові м’язи

2847 / 6854
Який з нижченаведених сечогінних засобів слід призначити хворому з первинним гіперальдостеронізмом?

Спіронолактон

Гіпотіазид

Фуросемід

Маніт

Триамтерен

2848 / 6854
У мазку з нальоту на мигдаликах хворого з імовірною дифтерією виявлено палички синього кольору з потовщеннями на полюсах. Який метод фарбування мазків було використано?

Леффлера

Нейссера

Грама

Буррі

Гінса

2849 / 6854
При розтині померлого, який хворів на туберкульоз, у верхній частці правої легені знайдено порожнину розмірами 3х2 см, яка сполучається з бронхом. Стінка порожнини щільна, має три шари: внутрішній -піогенний, середній - шар туберкульозної грануляційної тканини, зовнішній - сполучнотканинний. Який діагноз найбільш імовірний?

Фіброзно-осередковий туберкульоз

Гострий осередковий туберкульоз

Фіброзно-кавернозний туберкульоз

Туберкульома

Гострий кавернозний тубеокульоз

2850 / 6854
Дитина народилася в стані асфіксії. Який препарат необхідно ввести новонародженому для стимуляції дихання?

Лобелін

Празозин

Етимізол

Атропін

Прозерин

2851 / 6854
Хворий похилого віку страждає на хронічний закреп, в основі якого лежить гіпотонія товстого кишківника. Який препарат слід призначити хворому?

Прозерин

Ацеклідин

Бісакодил

Касторова олія

Натрію сульфат

2852 / 6854
При обстеженні в аналізі крові пацієнта виявлено лейкоцитоз, лімфоцитоз, клітини Боткіна-Гумпрехта на тлі анемії. Про яку хворобу слід думати лікарю?

Гострий мієлолейкоз

Лімфогранулематоз

Мієломна хвороба

Хронічний лімфолейкоз

Інфекційний мононуклеоз

2853 / 6854
У жінки 37-ми років протягом року періодично виникали інфекційні захворювання бактеріального генезу, їх перебіг був вкрай тривалим, ремісії - короткочасними. При обстеженні виявлена гіпогамаглобулінемія. Порушення функції яких клітин може бути прямою її причиною?

Лімфоцити

Макрофаги

Нейтрофіли

Плазматичні клітини

Фагоцити

2854 / 6854
При гістологічному дослідженні щитоподібної залози визначається значна інфільтрація лімфоцитами з утворенням лімфоїдних фолікулів, руйнування паренхіматозних елементів, розростання волокон сполучної тканини. Для якого захворювання характерна така картина?

Ендемічний зоб

Дифузний токсичний зоб

Колоїдний зоб

Паренхіматозний зоб

Зоб Хашимото

2855 / 6854
У чоловіка 50-ти років при обстеженні було виявлено зниження кількості еритроцитів у крові та підвищення рівня вільного гемоглобіну в плазмі крові (гемоглобінемія). КП становив 0,85. Який вид анемії спостерігається у хворого?

Набута гемолітична

Хронічна постгеморагічна

Гостра постгеморагічна

Спадкова гемолітична

Анемія внаслідок порушення еритропоезу

2856 / 6854
У хворого 68-ми років, який страждає на серцеву недостатність та впродовж тривалого часу приймає препарати наперстянки, з’явилися явища інтоксикації, які швидко нівелювалися застосуванням донатора сульфгідрильних груп унітіолу. Який механізм терапевтичної дії цього засобу?

Підвищує енергозабезпечення міокарду

Реактивує натрій-калієву-АТФ-азу мембран міокардіоцитів

Гальмує вивільнення калію з міокардіоцитів

Зменшує накопичення іонізованого кальцію

Сповільнює надходження натрію до міокардіоцитів

2857 / 6854
При визначенні мікробного числа повітря у лікарняній палаті виявилося, що воно становить 1500 клітин/м3 . Які групи мікроорганізмів враховувалися при цьому?

Всі патогенні та умовно-патогенні бактерії

Бактерії та віруси - збудники респіраторних інфекцій

Збудники госпітальних інфекцій

Всі бактерії, що виросли на живильному середовищі

Стафілококи та гемолітичні стрептококи

2858 / 6854
При проведенні операції на тонкій кишці лікар виявив ділянку слизової оболонки, де на фоні колових складок була присутня поздовжня складка. Який відділ тонкої кишки має таку будову?

Дистальний відділ ileum

Pars horizontalis duodeni

Pars descendens duodeni

Початковий відділ jejunum

Pars ascendens duodeni

2859 / 6854
В гістологічному препараті визначається орган, стінку якого утворюють три оболонки. Внутрішня оболонка складається з ендотелія та тонкого підендотеліального шару. Зовнішня оболонка найтовща. Вкажіть, який орган представлено у препараті?

Матка

Вена

Артерія

Сечовід

Серце

2860 / 6854
У хворого нормально забарвлений кал, у складі якого є велика кількість вільних жирних кислот. Причиною цього є порушення:

Всмоктування жирів

Гідролізу жирів

Секреції ліпаз

Жовчовиділення

Жовчоутворення

2861 / 6854
У новонародженого малюка педіатр виявив, що отвір крайньої плоті за величиною не перевищує діаметр сечовивідного каналу і голівка статевого члена не може виходити через такий отвір. Як називається цей стан?

Гіпоспадія

Парафімоз

Гермафродитизм

Епіспадія

Фімоз

2862 / 6854
Під час судово-медичної експертизи жінки, яка загинула у автокатастрофі, знайдено ембріон на стадії ранньої гаструли. Назвіть місце його локалізації за умови нормального розвитку:

Яєчник

Стінка матки

Маткова частина яйцепроводу

Ампульна частина яйцепроводу

Черевна порожнина

2863 / 6854
В основі розвитку імунних і алергічних реакцій організмом застосовуються однакові механізми відповіді імунної системи на антиген. Визначте основну відмінність алергічних реакцій від імунних:

Спадкова схильність

Особливість будови антигенів

Шляхи потрапляння антигенів до організму

Кількість антигену, що потрапляє

Розвиток пошкодження тканин

2864 / 6854
У хворої 49-ти років відзначається обмеження довільних рухів у лівих кінцівках. Тонус м’язів у лівих руці та нозі підвищений за спастичним типом, посилені місцеві сухожилкові рефлекси, виявляються патологічні рефлекси. Який найбільш імовірний механізм призвів до розвитку м’язової гіпертонії та гіперрефлексії?

Активація мотонейронів внаслідок інсульту

Зниження гальмівних низхідних впливів

Активація збуджувальних впливів з вогнища інсульту

Гальмування мотонейронів кори головного мозку

Активація синаптичної передачі імпульсів

2865 / 6854
Чоловік 50-ти років хворіє на хронічний бронхіт, скаржиться на задишку під час фізичного навантаження, постійний кашель з відходженням харкотиння. При обстеженні діагностовано ускладнення - емфізема легень. Чим вона зумовлена?

Зниження еластичних властивостей легень

Зменшення розтяжності легень

Порушення вентиляційно-перфузійного співвідношення в легенях

Зменшення перфузії легень

Зменшення альвеолярної вентиляції

2866 / 6854
У чоловіка 58-ми років, померлого при наростаючих явищах хронічної серцевої недостатності, діагностовано ревматичний гранульоматозний міокардит. Мікроскопічно в міокарді спостерігаються гранульоми, що складаються з макрофагів з гіперхромними ядрами та світлою цитоплазмою, в центрі - осередок некрозу. Який характер має некроз в середині гранульоми?

Ценкеровський

Фібриноїдний

Колікваційний

Жировий

Казеозний

2867 / 6854
У хворого з довготривалим запаленням слизової оболонки носової порожнини з’явилися симптоми ураження слизової оболонки верхньощелепної пазухи (гайморит). Через яке утворення носової порожнини стало можливим розповсюдження інфекції?

Решітчасті комірки

Верхньощелепний отвір

Решітчаста лійка

Клино-піднебінний отвір

Клино-решітчаста заглибина

2868 / 6854
Хворому з артеріальною гіпертензією було призначено один з антигіпертензивних засобів. Артеріальний тиск нормалізувався, однак хворого почав турбувати постійний сухий кашель. Який з перерахованих препаратів має таку побічну дію?

Ніфедипін

Лізиноприл

Клофелін

Анаприлін

Резерпін

2869 / 6854
В анотації до препарату вказано, що він містить антигени збудника черевного тифу, адсорбовані на стабілізованих еритроцитах барана. З якою метою використовують цей препарат?

Для серологічної ідентифікації збудника черевного тифу

Для виявлення антитіл в реакції непрямої гемаглютинації

Для виявлення антитіл в реакції зв’язування комплементу

Для виявлення антитіл в реакції Відаля

Для виявлення антитіл в реакції гальмування гемаглютинації

2870 / 6854
До дерматолога звернувся хворий зі скаргами на появу гнійничків на шкірі обличчя та шиї. При лабораторному аналізі вмісту гнійних фолікул було виявлено рухомі червоподібні паразити. Вкажіть збудника, який викликав це захворювання:

Блоха людська

Залозниця вугрова

Коростяний свербун

Блощиця ліжкова

Личинка мухи

2871 / 6854
На розтині тіла хлопчика 8-ми років, що хворів на дифтерію зіва та мигдаликів і помер на другий тиждень від початку захворювання, виявлено зміни в міокарді у вигляді дрібновогнищевих некрозів міокардиоцитів, набряку строми з незначною лімфоцитарною інфільтрацією. Діагностуйте вид міокардиту:

Гранулематозний

Інтерстиційний

Вогнищевий проміжний ексудативний

Альтеративний

Септичний

2872 / 6854
У хворого діагностовано септичний ендокардит. Температура тіла протягом 5-ти днів коливалася в межах 39,5°С - 40,2°С. На 6-й день на тлі різкого зниження температури до 35,2°С розвинувся колапс. Який головний механізм колапсу?

Гіпервентиляція

Тахікардія

Вазодилатація

Поліурія

Посилене потовиділення

2873 / 6854
У пацієнта у результаті тривалого блювання відбувається значна втрата шлункового соку, що є причиною порушення кислотно-лужного стану в організмі. Яка з перерахованих форм порушення кислотно-лужного стану має місце?

Метаболічний ацидоз

Негазовий алкалоз

Негазовий ацидоз

Газовий алкалоз

Газовий ацидоз

2874 / 6854
Хворий 55-ти років хворіє на хронічний гломерулонефрит протягом 15-ти років. Які зміни складу крові або сечі найбільш характерно свідчать про обмеження секреторної функції нирок?

Гіперглікемія

Протеїнурія

Гіперазотемія

Гіпо-, ізостенурія

Гіпопротеїнемія

2875 / 6854
У хворого в обох щелепах рентгенологічно виявлено численні дефекти у вигляді гладкостінних округлих отворів. При гістологічному дослідженні - явища остеолізису і остеопорозу при явищах слабкого кісткоутворення. В сечі хворого знайдено білок БенсДжонса. Назвіть захворювання:

Хронічний еритромієлоз

Мієломна хвороба

Гострий мієлолейкоз

Гострий недиференційований лейкоз

Хронічний мієлолейкоз

2876 / 6854
Хворий 30-ти років звернувся до лікаря зі скаргами на підвищення температури тіла до 38°С, слабкість, біль у горлі. Об’єктивно: язик вкритий білим нальотом. Які гістологічні структури язика беруть участь в утворенні цього нальоту?

Епітелій листоподібних сосочків

Епітелій грибоподібних сосочків

Епітелій ниткоподібних сосочків

Епітелій жолобкуватих сосочків

Сполучнотканинна основа всіх сосочків язика

2877 / 6854
Досліджуються рецептори, інформація від яких прямує до кори без участі таламусу. Які це рецептори?

Зорові

Нюхові

Смакові

Слухові

Дотикові

2878 / 6854
На розтині тіла чоловіка похилого віку, який протягом останніх 2-х тижнів страждав на гострий розлад кишківника, виявлені зміни у прямій та сигмоподібній кишках: на поверхні слизової оболонки спостерігається коричнево-зелена плівка. Стінка кишки потовщена, порожнина різко звужена. Мікроскопічно виявляється проникаючий на різну глибину некроз слизової оболонки, некротичні маси пронизані нитками фібрину, з лейкоцитарною інфільтрацією. Який з перерахованих діагнозів найбільш імовірний?

Виразковий коліт

Фібринозний коліт

Фолікулярний коліт

Катаральний коліт

2879 / 6854
У вагітної жінки взяли кров для підтвердження клінічного діагнозу 'токсоплазмоз'. Яка з перерахованих серологічних реакцій має діагностичне значення?

Реакція гальмування гемаглютинації

Реакція гемадсорбції

Реакція аглютинації

Реакція нейтралізації

Реакція зв’язування комплементу

2880 / 6854
У здорових батьків, спадковість яких не обтяжена, народилася дитина з множинними вадами розвитку. Цитогенетичний аналіз виявив у соматичних клітинах дитини трисомію за 18-ю хромосомою (синдром Едвардса). З яким явищем пов’язане народження такої дитини?

Хромосомною мутацією - дуплікацією

Соматичною мутацією у ембріона

Впливом тератогенних факторів

Домінантною мутацією

Нерозходженням пари хромосом під час гаметогенезу

2881 / 6854
Під час об’єктивного обстеження хворого з діагнозом: атеросклеротичний міокардіосклероз, лікар встановив феномен дефіциту пульсу. При якій формі порушення серцевого ритму спостерігається такий феномен?

Синусова екстрасистолія

Ідіовентрикулярний ритм

Миготлива аритмія

Брадикардія

Передсердно-шлуночковий ритм

2882 / 6854
При аналізі родоводу пробанда виявлено, що ознака проявляється з однаковою частотою у представників обох статей і хворі наявні у всіх поколіннях (по вертикалі), а по горизонталі - у сибсів (братів і сестер пробанда) з відносно великих родин. Який тип успадкування досліджуваної ознаки?

Зчеплений з Х-хромосомою, домінантний

Автосомно-рецесивний

Зчеплений з Х-хромосомою, рецесивний

Автосомно-домінантний

Зчеплений з У-хромосомою

2883 / 6854
До клініки доставлено хвору на цукровий діабет, госпіталізовано у прекоматозному стані кетоацидотичного типу. Збільшення вмісту якого метаболіту до цього призвело?

Ацетоацетат

α-кетоглутарат

Малонат

Аспартат

Цитрат

2884 / 6854
В хірургічне відділення доставлено пацієнта з різаною раною медіального краю передпліччя. При обстеженні виявлено, що в хворого перерізано ліктьовий м’яз-згинач зап’ястка і ліктьовий м’яз-розгинач зап’ястка. Які з рухів кисті будуть порушені у хворого?

Згинання

Приведення

Розгинання і відведення

Відведення

Розгинання

2885 / 6854
При огляді хворого хірург встановив поранення в області двох верхніх третин правої нирки. Цілісність якого органу слід перевірити при цьому, беручи до уваги син-топію правої нирки?

Низхідна ободова кишка

Шлунок

Печінка

Підшлункова залоза

Тонка кишка

2886 / 6854
Хворому на гострий інфаркт міокарда у комплексній терапії було призначено гепарин. Через деякий час після введення даного препарату з’явилася гематурія. Який антагоніст гепарину необхідно ввести хворому для усунення даного ускладнення?

Неодикумарин

Вікасол

Протаміну сульфат

Фібриноген

Амінокапронова кислота

2887 / 6854
У синтезі пуринових нуклеотидів беруть участь деякі амінокислоти, похідні вітамінів, фосфорні ефіри рибози. Коферментна форма якого вітаміну є переносником одновуглецевих фрагментів в цьому синтезі?

Фолієва кислота

Нікотинова кислота

Піридоксин

Пантотенова кислота

Рибофлавін

2888 / 6854
У жінки на шкірі шиї спостерігається новоутворення на ніжці, кулястої форми, зморшкувате. У біоптаті з нього виявлена надмірна проліферація покривного епітелію та строми шкіри у вигляді сосочків, збережена полярність клітин, їх комплексність та базальна мембрана, характерний тканинний атипізм. Який найбільш імовірний діагноз?

Фіброма

Рак

Папілома

Невус

Саркома

2889 / 6854
У хворого 75-ти років, який довгий час страждав на атеросклероз церебральних судин, на аутопсії виявлені: тромбоз правої середньої мозкової артерії, великий осередок неправильної форми сірого кашоподібного розм’якшення мозкової тканини. Який патологічний процес розвинувся в спинному мозку?

Ішемічний інфаркт

Геморагічний інфаркт

Гума мозку

Коагуляційний некроз

2890 / 6854
У померлої дитини 4-х років при житті була наявна менінгіальна симптоматика, На розтині в м’якій мозковій оболонці макроскопічно виявлені просоподібні вузлики, які мікроскопічно представлені осередком казеозного некрозу з валами епітеліоїдних, лімфоїдних клітин, між якими зустрічаються великі клітини з ядрами на периферії у вигляді півмісяця. Який найбільш імовірний менінгіт у дитини?

Менінгококовий

Бруцельозний

Сифілітичний

Туберкульозний

Грипозний

2891 / 6854
На препараті представлено орган ендокринної системи, зовні вкритий сполучнотканинною капсулою, від якої всередину органа відходять перегородки, що ділять його на часточки. Кожна часточка складається з двох видів клітин - нейросекреторних пінеалоцитів - полігональних клітин з відростками, локалізованих центрально, та гліоцитів (астроцитів) - на периферії. Що за орган представлено на препараті?

Мозкова речовина наднирників

Щитоподібна залоза

Епіфіз

Гіпофіз

Гіпоталамус

2892 / 6854
У здорових батьків народилася дитина з синдромом Патау. Який метод медичної генетики дасть змогу віддиференціювати дану спадкову хворобу від її фенокопії?

Визначення статевого хроматину

Дерматогліфічний

Близнюковий

Біохімічний

Цитогенетичний

2893 / 6854
Лікар записав в історії хвороби, що у хворого дихання поверхневе (знижена глибина дихання). Це означає, що зменшеним є такий показник зовнішнього дихання:

Хвилинний об’єм дихання

Дихальний об’єм

Ємність вдиху

Функціональна залишкова ємність

Життєва ємність легень

2894 / 6854
У хворого 65-ти років діагностовано хворобу Паркінсона. Який засіб, що підвищує вміст дофаміну, слід йому призначити?

Леводопа

Циклодол

Скополаміну гідробромід

Атропину сульфат

Амізил

2895 / 6854
Після фармакологічної блокади іонних каналів мембрани нервового волокна потенціал спокою зменшився з -90 до -80 м Які канали було заблоковано?

Магнієві

Натрієві

Кальцієві

Калієві

Хлорні

2896 / 6854
У студента, який раптово зустрів кохану дівчину, збільшився системний артеріальний тиск. Посилена реалізація яких рефлексів спричинила таку зміну тиску?

Безумовні парасимпатичні

Умовні симпатичні

Безумовні симпатичні

Умовні симпатичні та парасимпатичні

Умовні парасимпатичні

2897 / 6854
Вагітна жінка під час пологів втратила близько 800 мл крові. Спостерігається тахікардія, артеріальний тиск 100/70 мм рт.ст., тахіпное до 28/хв. Який тип гіпоксії розвивається первинно в такій клінічній ситуації ?

Змішана

Серцево-судинна

Дихальна

Кров’яна

Тканинна

2898 / 6854
Основними тригерами, що вмикають ефекторні системи клітини у відповідь на дію гормонів, є протеїнкінази, які змінюють каталітичну активність певних регуляторних ферментів шляхом АТФ-залежного фосфорилювання. Який з наведених ферментів є активним у фосфори-льованій формі?

ГОМГ-КоА-редуктаза

Ілікогенсинтаза

Глікогенфосфорилаза

Піруваткіназа

Ацетил-КоА-карбоксилаза

2899 / 6854
У пацієнта виявлено: поганий апетит, нудота, блювання, анемія. На основі проведеної лабораторної діагностики встановлено дифілоботріоз. Зараження відбулося через вживання:

Яєць

Свинини

Риби

Крабів та раків

Яловичини

2900 / 6854
Очищення слизової оболонки дихальних шляхів від пилу і мікроорганізмів відбувається завдяки мукоциліарному транспорту - переміщенню слизу поверхнею епітелію. Які клітини забезпечують цей механізм очищення?

Ендокринні та базальні

Щіточкові

Бронхіолярні екзокриноцити

Дендритні

Війчасті та келихоподібні

2901 / 6854
На електронномікроскопічній фотографії поперечного зрізу волокна чітко візуалізуються декілька осьових циліндрів з мезаксонами. Яке це волокно?

Нервове мієлінове

Еластичне

Колагенове

Ретикулярне

Нервове безмієлінове

2902 / 6854
Інозитолтрифосфати в тканинах організму утворюються в результаті гідролізу фосфатидилінозитолдифосфатів і відіграють роль вторинних посередників (месенджерів) в механізмі дії гормонів. Їхній вплив у клітині спрямований на:

Гальмування фосфодіестерази

Вивільнення іонів кальцію з клітинних депо

Активацію аденілатциклази

Гальмування протеїнкінази С

Активацію протеїнкінази А

2903 / 6854
В ході експерименту було продемонстровано підвищення активності β-галактозидази після внесення лактози до культурального середовища з Е.соlі. Яка ділянка лактозного оперону стає розблокованою від репресору за цих умов?

Оператор

Праймер

Структурний ген

Регуляторний ген

Промотор

2904 / 6854
Під час електронномікроскопічного дослідження біоптату гепатоцитів на біліарному полюсі виявлено велику кількість плоских цистерн, сплющених у центральній частині й розширених на периферії, та дрібних міхурців із секреторними гранулами. Назвіть цю структуру:

Піноцитозні міхурці

Мікротрубочки

Ендоплазматична сітка

Комплекс Гольджі

Лізосома

2905 / 6854
При гастробіопсії у хворого встановлена метаплазія поверхневого епітелію слизової оболонки, який замість циліндричного набув вигляду кишкового. Разом з тим спостерігається склероз на місці залоз слизової оболонки та лімфогістіоцитарна інфільтрація. Про яке захворювання шлунка можна думати?

Корозивний гастрит

Хронічний атрофічний гастрит

Ерозивний гастрит

Поверхневий хронічний гастрит

Хронічний гастрит з ураженням залоз без атрофії

2906 / 6854
У жінки 23-х років після аборту з’явилася маткова кровотеча. Мікроскопічно у зіскобі з порожнини матки знайдені ворсини хоріона, що нагадують грона винограду. Мікроскопічно: набряк строми ворсин з утворенням безліч кіст, що супроводжуються проліферацією епітелію та синцитію ворсин, залишки плоду і плодових оболонок. Який найбільш імовірний діагноз?

Вагітність

Плацентарний поліп

Хоріонепітеліома

Міхуровий занесок

Ендометрит

2907 / 6854
У дитячому колективі проведено планову вакцинацію проти кору. Яким методом можна перевірити ефективність проведеної вакцинації?

Серологічний

Вірусологічний

Алергопроба

Вірусоскопічний

Біологічний

2908 / 6854
Хворому з лихоманкою та висипкою на шкірі після обстеження за допомогою серологічних реакцій поставлено діагноз - фасціольоз. Було встановлено, що хворий заразився шляхом споживання сирої води з річки. Яка стадія життєвого циклу фасціоли інвазійна для людини?

Яйце

Адолескарій

Фіна

Метацеркарій

Мірацидій

2909 / 6854
Хворий помилково прийняв надмірну дозу тироксину. До яких змін секреції тиреоліберину та тиреотропіну це призведе?

Секреція гормонів збільшиться

Секреція тиреотропіну збільшиться, тиреоліберину - зменшиться

Секреція гормонів зменшиться

Змін секреції гормонів не буде

Секреція тиреоліберину збільшиться, тиреотропіну - зменшиться

2910 / 6854
Турист нещодавно повернувся з країн Середньої Азії, де є багато москітів. У нього на шкірі з’явилися невеликі виразки з нерівними краями. В цьому випадку можна припустити наступне захворювання:

Токсоплазмоз

Демодекоз

Скабіес

Специфічний міаз

Дерматотропний лейшманіоз

2911 / 6854
У потерпілого з травмою голови у скроневій ділянці діагностовано епідуральну гематому. Яка з артерій найімовірніше пошкоджена?

Поверхнева скронева

Передня оболонкова

Середня мозкова

Задня вушна

Середня оболонкова

2912 / 6854
У дитини спостерігається затримка фізичного та розумового розвитку, глибокі порушення з боку сполучної тканини внутрішніх органів, у сечі виявлено кератансульфати. Обмін яких речовин порушений?

Гіалуронова кислота

Фібронектин

Глікозаміноглікани

Колаген

Еластин

2913 / 6854
При аналізі ЕКГ виявлено випадіння деяких серцевих циклів PQRST. Наявні зубці та комплекси не змінені. Назвіть вид аритмії:

Синоатріальна блокада

Атріовентрикулярна блокада

Миготлива аритмія

Внутрішньопередсердна блокада

Передсердна екстрасистола

2914 / 6854
До лікарні доставлено хворого з отруєнням інсектицидом - ротеноном. Яка ділянка мітохондріального ланцюга переносу електронів блокується цією речовиною?

АТФ -синтетаза

Сукцинат-коензим О-редуктаза

Цитохром С-оксидаза

НАДН-коензим О-редуктаза

Коензим О-цитохром С-редуктаза

2915 / 6854
В ході регенерації епітелію слизової оболонки порожнини рота (розмноження клітин) відбулася реплікація (авторепродукція) ДНК за напівконсервативним механізмом. При цьому нуклеотиди нової нитки ДНК є комплементарними до:

Змістовних кодонів

Ферменту ДНК-полімерази

Материнської нитки

Ферменту РНК-полімерази

Інтронних ділянок гену

2916 / 6854
Серед антиатеросклеротичних препаратів, що застосовуються з метою профілактики та лікування атеросклерозу, є левостатин. Він діє шляхом:

Стимулювання екскреції холестерину з організму

Пригнічення всмоктування холестерину в кишківнику

Гальмування біосинтезу холестерину

Усіма наведеними шляхами

Активації метаболізму холестерину

2917 / 6854
При визначенні повітряної та кісткової провідності звуку було встановлено, що у пацієнта ліве вухо краще сприймає звук при кістковому його проведенні, що могло бути пов’язано з захворюванням:

Внутрішнього вуха справа

Зовнішнього вуха справа

Внутрішнього вуха зліва

Середнього вуха зліва

Середнього вуха справа

2918 / 6854
В хронічному експерименті на щурах стимулювали електричним струмом паравентрикулярні та супраоптичні ядра гіпоталамуса. Яка поведінкова реакція спостерігалася у тварин?

Відмова від їжі та рідини

Зменшення споживання води

Збільшення споживання їжі

Збільшення споживання води

Зменшення споживання їжі

2919 / 6854
У хворого 45-ти років на тлі трансмурального інфаркту міокарда розвинулася гостра лівошлуночкова недостатність. Який лікарський засіб доцільно застосувати у даній ситуації для покращення помпової функції серця?

Еуфілін

Добутамін

Ефедрин

Промедол

Ізадрин

2920 / 6854
При зниженні активності ферментів антиоксидантного захисту посилюються процеси перекисного окиснення ліпідів клітинних мембран. При нестачі якого мікроелементу знижується активність глутатіонпероксидази?

Мідь

Селен

Кобальт

Марганець

Молібден

2921 / 6854
У хворого з нагноєнням рани при бактеріологічному дослідженні ранового вмісту виявлено грамнегативну паличку, яка на МПА утворює напівпрозорі слизові колонії синьо-зеленого кольору з перламутровим відтінком. Культура має специфічний запах фіалок або жасмину. Який вид збудника виділений з рани хворого?

P aeruginosa

S. pyogenes

S. faecalis

S. aureus

P vulgaris

2922 / 6854
У хворого під час комп’ютерної томографії грудної клітки діагностовано пухлину заднього нижнього середостіння. Яка з перерахованих структур стиснута пухлиною?

Trachea

Arcus aortae

Aorta thoracica

Vena cava superior

N. phrenicus

2923 / 6854
Під час операції холецистектомії у хірурга виникла необхідність визначити топографію загальної жовчної протоки. Злиттям яких проток утворюється дана анатомічна структура?

Правої та лівої печінкових проток

Загальної печінкової і міхурової проток

Лівої печінкової і міхурової проток

Загальної печінкової і правої печінкової проток

Загальної печінкової і лівої печінкової проток

2924 / 6854
Внаслідок ДТП у потерпілої 37-ми років виникло неутримання сечі. Які сегменти спинного мозку пошкоджені?

Th2 — Th5

L1 — L2

S2 - S4

Th1 — L1

Th1 — Th5

2925 / 6854
При обстеженні жінки 56-ти років, що хвора на цукровий діабет 1-го типу, виявлене порушення білкового обміну, що при лабораторному дослідженні крові проявляється аміноацидемією, а клінічно - уповільненням загоєння ран і зменшенням синтезу антитіл. Який з перерахованих механізмів викликає розвиток аміноацидемії?

Зменшення концентрації амінокислот у крові

Гіперпротеїнемія

Збільшення ліпопротеїдів високої щільності

Підвищення онкотичного тиску в плазмі крові

Підвищення протеолізу

2926 / 6854
У людини в результаті патологічного процесу збільшена товщина гематоальвеолярного бар’єру. Безпосереднім наслідком цього буде зменшення:

Хвилинного об’єму дихання

Резервного об’єму видиху

Дифузійної здатності легень

Альвеолярної вентиляції легень

Кисневої ємності крові

2927 / 6854
Недостатність в організмі мікроелементу селену проявляється кардіоміопатією. Імовірною причиною такого стану є зниження активності такого селенвмісного ферменту:

Каталаза

Сукцинатдегідрогеназа

Цитохромоксидаза

Глутатіонпероксидаза

Лактатдегідрогеназа

2928 / 6854
При дослідженні тимуса дитини 5-ти років, що померла від гострої деструктивної стафілококової пневмонії, виявлено зменшення маси залози до 3,0 г. При гістологічному дослідженні в тимусі знайдено: зменшення часточок залози, значне зменшення кількості лімфоцитів з колапсом строми часточок, інверсія шарів, кистоподібне збільшення тілець Гассаля. Який з перерахованих діагнозів найбільш імовірний?

Дисплазія тимусу

Акцидентальна інволюція тимусу

Тимомегалія

Гіпоплазія тимусу

Агенезія тимусу

2929 / 6854
У хворого внаслідок хронічного обструктивного бронхіту на тлі задишки, тахікардії та ціанозу під час дослідження газового складу крові виявлено розвиток гіпоксемії та гіперкапнії. Яке порушення зовнішнього дихання спостерігається у хворого?

Гіповентиляція

Гіпервентиляція

Гіпердифузія

Гіпоперфузія

Гіперперфузія

2930 / 6854
Хлопчику 5-ти років був встановлений діагноз - міастенія. Оберіть препарат з групи антихолінестеразних засобів, який покращує нервово-м’язову передачу:

Алоксим

Армін

Прозерин

Галантаміну гідробромід

Ацеклідин

2931 / 6854
Хворому для лікування серцевої недостатності було призначено серцевий глікозид. Яка супутня патологія може сприяти кумуляції серцевих глікозидів?

Гіпертонічна хвороба

Анорексія

Гіпоацидний гастрит

Ниркова недостатність

Гіперацидний гастрит

2932 / 6854
П’ятирічна дитина-правша після черепно-мозкової травми на деякий час втратила здатність розмовляти, але через тривалий час ця здатність у неї відновилась. Яка півкуля була травмована й за рахунок якої властивості ЦНС дітей відновлення мови стало можливим?

Ліва півкуля, пластичність

Ліва півкуля, інертність

Права півкуля, рухливість

Обидві півкулі, інертність

Права півкуля, пластичність

2933 / 6854
При дослідженні гостроти слуху в коваля виявили втрату слуху на 50% у діапазоні низьких частот і майже нормальну гостроту слуху в діапазоні високих частот. Порушення яких структур слухової системи призвело до такого стану?

М’язи середнього вуха

Барабанна перетинка

Кортієв орган - ближче до гелікотреми

Середня частина кортієвого органу

Кортієв орган - ближче до овального віконця

2934 / 6854
В підводному човні під час занурення порушилася система подачі кисню. У підводників збільшилися частота дихання і серцевих скорочень. Який вид гіпоксії розвинувся у підводників?

Серцево-судинна

Кров’яна

Гіпоксична

Дихальна

Тканинна

2935 / 6854
Сучасні антиатеросклеротичні препарати застосовуються з метою профілактики та лікування атеросклерозу. Такі препарати як гемфіброзил та фенфібрат гальмують біосинтез холестерину шляхом інгібування ферменту:

в-ГОМК-редуктаза

Ацилтрансфераза

Ацил-КоА-холестеринацилтрансфераза

Ілюкозо-6-фосфатаза

Гексокиназа

2936 / 6854
Хворий на атеросклероз приймає антисклеротичний засіб - фенофібрат. Який механізм дії має цей засіб?

Поновлює негативний електричний заряд ендотелію судин

Поліпшує мікроциркуляцію крові

Підвищує захоплення ліпопротеїдів низької щільності та блокує біосинтез ендогенного холестерину

Знижує рівень хіломікронів

Інгібує абсорбцію холестерину в ШКТ

2937 / 6854
Хворий на гіпертонічну хворобу разом з безсольовою дієтою та з антигіпертензивними засобами, довгий час приймав гідрохлортіазид, що зумовило порушення електролітного балансу. Яке порушення внутрішнього середовища виникло у хворого?

Гіпохлоремічний алкалоз

Метаболічний ацидоз

Гіпермагніємія

Гіперкаліємія

Збільшення об’єму циркулюючої крові

2938 / 6854
При лабораторному дослідженні крові пацієнта виявлено, що вміст білків у плазмі становить 40 г/л. Як це впливає на транскапілярний обмін води в мікроциркуляторному руслі?

Обмін не змінюється

Збільшується фільтрація, зменшується реабсорбція

Зменшується фільтрація, збільшується реабсорбція

Зменшуються фільтрація і реабсорбція

Збільшуються фільтрація і реабсорбція

2939 / 6854
При гістологічному дослідженні біоптатів, взятих з потовщених країв виразки шлунка, виявлені невеликі гніздові скупчення різко атипових гіперхромних невеликих епітеліальних клітин, які розташовані серед дуже розвиненої строми. Визначте пухлину:

Медулярний рак

Аденома

Скіррозний недиференційований рак

Недиференційована саркома

Аденокарцинома

2940 / 6854
У хворого на ЕКГ виявлено збільшення тривалості комплексу QRS. Наслідком чого це може бути?

Збільшення часу охоплення збудженням передсердь

Збільшення часу охоплення збудженням шлуночків

Порушення провідності у атріовентрикулярному вузлі

Збільшення збудливості передсердь

Збільшення збудливості шлуночків та передсердь

2941 / 6854
В ході авторадіографічного дослідження епітелію тонкої кишки було виявлено, що його повне оновлення відбувається протягом 3-х діб за рахунок проліферації малодиференційованих клітин. Вкажіть їх локалізацію:

Дно крипт

Основа ворсинок

Власна пластинка слизової оболонки

Бічна поверхня ворсинок

Верхівка ворсинок

2942 / 6854
У хворого пухлина грудного відділу стравоходу. Куди можуть безпосередньо метастазувати пухлинні клітини?

Nodi hepatici

Nodi intercostales

Ductus thoracicus

Nodi mediastinales

Nodi gastrici

2943 / 6854
При загостренні ревматоїдного артриту хворому, в анамнезі якого супутній хронічний гастрит, призначений целекоксиб. Чим обумовлено зменшення побічної дії препарату на травний тракт?

Переважне пригнічення циклооксигенази-1

Пригнічення фосфоліпази А2

Пригнічення фосфодіестерази

Переважне пригнічення циклооксигенази-2

Переважна стимуляція аденілатциклази

2944 / 6854
Дитина 4-х років госпіталізована в ортопедичне відділення з переломом гомілки зі зсувом. Перед репозицією уламків необхідна анальгезія. Який препарат слід обрати?

Панадол

Морфіну гідрохлорид

Промедол

Омнопон

Анальгін

2945 / 6854
При напруженій фізичній роботі у м’язовій тканині накопичується молочна кислота, яка дифундує в кров і підхоплюється печінкою та серцем. Який процес забезпечує відновлення запасів глікогену в м’язах?

Цикл лимонної кислоти

Цикл Корі

Цикл трикарбонових кислот

Пентозофосфатний шлях

Орнітиновий цикл

2946 / 6854
Фенілкетонурія - це захворювання, яке зумовлено рецесивним геном, що локалізується в аутосомі. Батьки є гетерозиготами за цим геном. Вони вже мають двох хворих синів і одну здорову доньку. Яка імовірність, що четверта дитина, яку вони очікують, народиться теж хворою?

0%

50%

100%

75%

25%

2947 / 6854
Визначення Х-хроматину в соматичних клітинах використовується для експресдіагностики спадкових захворювань, пов’язаних зі зміною кількості статевих хромосом. Який каріотип чоловіка, переважна більшість клітин якого містять одну грудочку Ххроматину:

46, ХУ

47, ХХУ

49, ХХХХУ

48, ХХХУ

45, Х0

2948 / 6854
36-ти років має місце гіповітаміноз В2 148. . Причиною виникнення специфічних симптомів (ураження епітелію, слизових оболонок, шкіри, рогівки ока) імовірно є дефіцит:

Цитохрому С

Цитохромоксидази

Цитохрому В

Цитохрому А1

Флавінових коферментів

2949 / 6854
На розтині в серці виявлено наступні зміни: великий осередок некрозу білого кольору з червоною облямівкою, який захоплює всю товщу серцевого м’яза. На зовнішній оболонці серця - ознаки фібринозного перикардиту. Який найбільш імовірний діагноз?

Інтрамуральний інфаркт міокарда

Трансмуральний інфаркт міокарда

Субендокардіальний інфаркт міокарда

Субепікардіальний інфаркт міокарда

Міокардит

2950 / 6854
В пробірку, що містить розчин NaCl 0,9%, додали краплю крові. Що відбудеться з еритроцитами?

Осмотичний гемоліз

Зморшкування

Залишаться без змін

Набухання

Біологічний гемоліз

2951 / 6854
До косметолога звернулася пацієнтка зі скаргами на появу чорних цяток на обличчі. Після обстеження було встановлено, що поява цяток пов’язана з порушенням виділення секрету сальних залоз. Який тип секреції характерний для цих залоз?

Мерокриновий та мікроапокриновий

Голокриновий

Макроапокриновий

Мерокриновий

Мікроапокриновий

2952 / 6854
У хворого 40-ка років ознаки гірської хвороби: запаморочення, задишка, тахікардія, рН крові - 7,50, pCO2 - 30 мм рт.ст., зсув буферних основ +4 ммоль/л. Яке порушення кислотно-основного стану має місце?

Негазовий ацидоз

Видільний ацидоз

Негазовий алкалоз

Газовий ацидоз

Газовий алкалоз

2953 / 6854
У чоловіка 48-ми років виявлено порушення периферичного кровообігу з обмеженням припливу артеріальної крові, при цьому має місце зблідніння даної ділянки, зниження місцевої температури. Це порушення називається:

Сладж

Венозна гіперемія

Ішемія

Стаз

Реперфузійний синдром

2954 / 6854
Хворому хірург видалив порожнинне утворення печінки діаметром 2 см. Встановлено, що стінка порожнини утворена щільною волокнистою сполучною тканиною, вміст являє собою каламутну, густу, жовтувато-зеленого кольору рідину з неприємним запахом, яка мікроскопічно складається переважно з поліморфноядерних лейкоцитів. Якому патологічному процесу відповідають такі морфологічні зміни?

Гострий абсцес

Хронічний абсцес

Емпієма

Флегмона

2955 / 6854
Лікар призначив пацієнту з хронічним бронхітом відхаркувальний засіб, який діє шляхом розщеплення дисульфідних зв’язків глікозаміногліканів харкотиння, зменшуючи цим його в’язкість, проте попередив хворого про можливий бронхоспазм при його використанні. Який засіб був призначений?

Трава термопсису

Бромгексин

Лібексин

Ацетилцистеїн

Натрію гідрокарбонат

2956 / 6854
Лікар призначив хворому з гострою серцевою недостатністю неглікозидний кардіотонічний засіб, який безпосередньо стимулює β1 -адренорецептори міокарда, що збільшує кровообіг, діурез. Застосовується лише внутрішньовенно крапельно внаслідок швидкої інактивації в організмі. Який препарат призначив лікар?

Адреналін

Корглікон

Анаприлін

Дигоксин

Добутамін

2957 / 6854
У молодого подружжя народилася дитина з енцефалопатією. Лікар встановив, що хвороба пов’язана з порушенням мітохондріальної ДНК. Як успадковуються мітохондріальні патології?

Від батька тільки синами

Від батька тільки дочками

Від матері тільки синами

Від обох батьків усіма дітьми

Від матері всіма дітьми

2958 / 6854
При розтині тіла жінки, померлої від хронічної ниркової недостатності, в слизовій оболонці товстої кишки виявлені сіро-жовті плівки, що щільно з’єднані з підлеглим шаром, при відокремленні яких утворюються виразки. Мікроскопічно: глибока ділянка некрозу пронизана нитками фібрину. Визначте вид запалення:

Катаральне

Гнійне

Крупозне

Гнильне

Дифтеритичне

2959 / 6854
Чоловік 40-ка років перебував у пульмонологічному відділенні з приводу рецидивуючої правосторонньої пневмонії. Помер від легенево-серцевої недостатності. На розтині в правій легені визначається ділянка круглої форми 3х4 см. Вона являє собою порожнину з нерівними шорсткими краями, заповнену каламутною вершкоподібною жовто-зеленою рідиною. Мікроскопічно: стінка порожнини утворена тканиною легені з дифузною інфільтрацією лейкоцитами. Визначте патологічний процес у легені:

Хронічний абсцес

Емпієма

Гангрена

Гострий абсцес

Інфаркт

2960 / 6854
Під час розтину тіла жінки 52-х років, яка тривалий час хворіла на жовчнокам’яну хворобу, було знайдено: макроскопічно - печінка помірно збільшена, деформована, поверхня органу горбиста, тканина щільна, на розрізі тканина коричнева з зеленим відтінком, складається з множинних вузликів діаметром 8-10 мм. Мікроскопічно - гепатоцелюлярні вузлики оточені прошарками сполучної тканини, яка містить збільшену кількість дрібних жовчних протоків з холестазом. Діагностуйте захворювання печінки:

Постнекротичний цироз печінки

Біліарний цироз печінки

Токсична дистрофія печінки

Холелітіаз

Портальний цироз печінки

2961 / 6854
Встановлено, що в клітинах організмів відсутні мембранні органели та їх спадковий матеріал не має нуклеосомної організації. Що це за організми?

Віруси

Найпростіші

Аскоміцети

Еукаріоти

Прокаріоти

2962 / 6854
Аналіз ЕКГ хворого виявив відсутність зубця P. Тривалість та амплітуда QRS комплексу та зубця Т відповідають нормі. Що є водієм ритму серця даного пацієнта?

Передсердно-шлуночковий вузол

Синусовий вузол

Волокна Пуркіньє

Міокард шлуночків

Пучок Гіса

2963 / 6854
Під час виконання оперативного втручання на гомілці хірург виділяє задню стінку canalis cruropopliteus. Яка анатомічна структура її утворює?

M.tibialis anterior

M.soleus

M.tibialis posterior

M.gastrocnemius

M.plantaris

2964 / 6854
Хворій 43-х років для лікування бронхопневмонії призначена бензилпеніциліну натрієва сіль. Який з вказаних побічних ефектів найбільш характерний для даного засобу?

Неврит слухового нерва

Ураження печінки

Алергічні реакції

Агранулоцитоз

Анемія

2965 / 6854
Після лікування високоефективним протитуберкульозним засобом у жінки 48ми років виникли явища невриту зорового нерва, порушення пам’яті, судоми. Який із зазначених протитуберкульозних препаратів приймала хвора?

ПАСК

Канаміцину сульфат

Етамбутол

Рифампіцин

Ізоніазид

2966 / 6854
Хлопчик на другому році життя став часто хворіти на респіраторні захворювання, стоматити, гнійничкові ураження шкіри. Навіть невеликі пошкодження ясен і слизової оболонки ускладнюються запаленням, що протікає тривало. Встановлено, що у крові дитини практично відсутні імуноглобуліни всіх класів. Зниження функціональної активності якої клітинної популяції лежить в основі описаного синдрому?

Т-лімфоцити

Нейтрофіли

Макрофаги

NK-лімфоцити

В-лімфоцити

2967 / 6854
Пацієнт із захворюванням першого верхнього різця зліва скаржиться на сильний біль шкіри в ділянці надбрівної дуги з того ж боку. Реалізація якого виду рефлексів спричиняє вказані реакції?

Вісцеро-соматичні

Пропріоцептивні

Вісцеро-вісцеральні

Вісцеро-дермальні

Сомато-вісцеральні

2968 / 6854
Під час роботи лікарю - стоматологу доводиться довго стояти на ногах, що може призвести до застою крові у венах нижніх кінцівок та їх варикозного розширення. З порушенням якого механізму венозного припливу крові до серця це пов’язано?

Градієнт тиску

Відсутність скорочення скелетних м’язів

Присмоктувальний ефект грудної клітки

Залишкова рушійна сила серця

Присмоктувально-тисковий помповий ефект діафрагми на органи черевної

2969 / 6854
. У хворого 34-х років після перенесеної кишкової інфекції, викликаної сальмонелами, стали згасати симптоми захворювання. Імуноглобуліни якого класу будуть виявлені в крові хворого в період реконвалесценції?

IgE

IgG

IgA

IgM

IgD

2970 / 6854
На плановий прийом до педіатра батьки привели дитину віком 13 місяців. Під час повного огляду лікар перевірив розвиток II сигнальної системи дитини. Назвіть період, коли у людини вперше з’являються ознаки розвитку II сигнальної системи:

3-5 років

2-2,5 роки

6-12 місяців

1,5-2 роки

2,5-3 роки

2971 / 6854
У хворого спостерігається пухлина тканин орбіти позаду очного яблука. Зазначено порушення акомодації та звуження зіниці ока. Яке анатомічне утворення ушкоджено?

Ganglion ciliare

N. opticus

N. nasociliaris

N. lacrimalis

N. trochlearis

2972 / 6854
До серцево-судинного відділення надійшов хворий зі скаргами на постійний головний біль у потиличній ділянці, шум у вухах, запаморочення. При обстеженні: АТ-180/110 мм рт.ст., ЧСС- 95/хв. Рентгенологічно визначено звуження однієї з ниркових артерій. Активація якої з перерахованих систем викликала гіпертензивний стан хворого?

Імунна

Ренін-ангіотензинова

Гемостатична

Симпатоадреналова

Кінінова

2973 / 6854
Хворого на трансмуральний інфаркт міокарда лівого шлуночка переведено до відділення реанімації у важкому стані. АТ-70/50 мм рт.ст., ЧСС- 56/хв., ЧД- 32/хв. Зазначте головну ланку в патогенезі кардіогенного шоку:

Крововтрата

Втрата електролітів

Падіння периферичного судинного опору

Падіння серцевого викиду

Втрата води

2974 / 6854
У чоловіка 29-ти років з ножовим пораненням шиї спостерігається кровотеча. При первинній обробці рани встановлено, що пошкоджена судина, розташована вздовж латерального краю груднинно-ключично-соскоподібного м’яза. Визначте цю судину:

A. carotis interna

A. carotis externa

V jugularis externa

V jugularis anterior

V jugularis interna

2975 / 6854
Після перелому нижньої щелепи постраждалий відзначає втрату чутливості шкіри у ділянці підборіддя і нижньої губи. Який нерв був пошкоджений?

Infraorbitalis

Maxillaris

Facialis

Mentalis

Buccalis

2976 / 6854
У хворого переливання крові ускладнилося розвитком гемотрансфузійного шоку. Назвіть тип алергічної реакції, що лежить в основі даної патології:

Анафілактичний

Цитотоксичний

Гіперчутливість сповільненого типу

Імунокомплексний

Рецептороопосередкований

2977 / 6854
Підшлункова залоза - орган змішаної секреції. Ендокринно продукує бета-клітинами гормон інсулін, який впливає на обмін вуглеводів. Як він впливає на активність глікогенфосфорилази (ГФ) і гліко-генсинтетази (ГС)?

Пригнічує ГФ і ГС

Активує ГФ, пригнічує ГС

Пригнічує ГФ, активує ГС

Активує ГФ і ГС

Не впливає на активність ГФ і ГС

2978 / 6854
До приймального відділення інфекційної лікарні надійшов чоловік 25-ти років. Діагноз: СНІД. Ураження яких клітин зумовлює стан імунодефіциту?

Т-хелпери

Т-супресори

Т-кіллери

Плазмоцити

Гладкі клітини (тканинні базофіли)

2979 / 6854
Тривале лікування гіпофункції щитоподібної залози спричинило загальну дистрофію, карієс зубів, тахікардію, тремор кінцівок. Який лікарський засіб викликав зазначені побічні ефекти?

L-тироксин

Преднізолон

Тирокальцитонін

Хумулін

Паратиреоїдин

2980 / 6854
Молода людина 25-ти років споживає надмірну кількість вуглеводів (600 г на добу), що перевищує її енергетичні потреби. Який процес буде активуватися в організмі людини у даному випадку?

Гліколіз

Ліполіз

Окиснення жирних кислот

Ілюконеогенез

Ліпогенез

2981 / 6854
У пацієнта встановлено гіповітаміноз фолієвої кислоти, що може призвести до порушення синтезу:

Пуринових нуклеотидів та холестерину

Гема та креатину

Тимідилових нуклеотидів та жирних кислот

Цитрату та кетонових тіл

Пуринових та тимідилових нуклеотидів

2982 / 6854
У людини виявлена пухлина одного з відділів головного мозку, внаслідок чого в неї порушена здатність підтримувати нормальну температуру тіла. Яка структура головного мозку пошкоджена?

Таламус

Мозочок

Гіпоталамус

Стріатум

Чорна субстанція

2983 / 6854
До складу харчових раціонів обов’язково входять продукти, в яких є клітковина. Відомо, що вона не перетравлюється ферментами травного тракту й не засвоюється організмом. Яку роль відіграє ця речовина?

Гальмує секреторну функцію травного каналу

Стимулює моторну функцію травного каналу

Гальмує процеси виділення ферментів травних соків

Гальмує всмоктувальну функцію травного каналу

Гальмує моторну функцію травного каналу

2984 / 6854
До лікарні звернувся чоловік 50-ти років з розладами пам’яті, болісними відчуттями по ходу нервових стовбурів, зниженням інтелектуальних функцій, порушеннями з боку серцево-судинної системи і явищами диспепсії. В анамнезі хронічний алкоголізм. Дефіцит якого вітаміну може викликати ці симптоми?

Рибофлавін

Тіамін

Ретинол

Кальциферол

Ніацин

2985 / 6854
По приїзді групи експертів на місце злочину виявлено тіло без ознак життя. В ході дослідження крові загиблого виявлена велика концентрація іонів ціанової кислоти. Інгібування якого комплексу дихального ланцюга мітохондрій стало причиною смерті?

II

V

III

IV

І

2986 / 6854
Вивчення організму мешканця Паміру виявило високий рівень основного обміну, розширення грудної клітки, зростання кисневої ємності крові за рахунок збільшення еритроцитів, високий вміст гемоглобіну. До якого адаптивного екологічного типу слід віднести цього чоловіка?

Пустельний

Субтропічний

Арктичний

Гірський

Тропічний

2987 / 6854
Пацієнт звернувся зі скаргами на гострий біль у правому підребер’ї. При огляді лікар звернув увагу на пожовтіння склер хворого. Лабораторно: підвищена активність АлАТ та негативна реакція на стеркобілін в калі. Для якого захворювання характерні такі симптоми?

Гепатит

Хронічний гастрит

Гемолітична жовтяниця

Хронічний гастродуоденіт

Хронічний коліт

2988 / 6854
Жінка 30-ти років народила хлопчика з розщепленням верхньої губи ('заяча губа' 'вовча паща'). При додатковому обстеженні виявлені значні порушення нервової, серцевосудинної систем та зору. При дослідженні каріотипу діагностована трисомія за 13-ю хромосомою. Який синдром наявний у хлопчика?

Патау

Едвардса

Шерешевського-Тернера

Клайнфельтера

Дауна

2989 / 6854
До медико-генетичної консультації звернулася жінка 30-ти років у якої в ядрах більшості клітин епітелію слизової оболонки щоки було виявлено по два тільця Барра. Який попередній діагноз можна встановити?

Трисомія за 18-ю хромосомою

Трисомія за 21-ю хромосомою

Трисомія за 13-ю хромосомою

Трисомія за X-хромосомою

Моносомія за X-хромосомою

2990 / 6854
При розтині хворої 28-ми років, що померла від уремії, виявлені збільшені строкаті нирки з осередками крововиливів. Патогістологічно в судинних клубочках виявлені гематоксилінові тільця, капілярні мембрани клубочків у вигляді дротяних петель, гіалінові тромби та осередки фібриноїдного некрозу. За патогенезом гіперчутливість якого типу лежить в основі описаної хвороби?

Гіперчутливість V типу (гранулематоз)

Гіперчутливість IV типу (клітинна цитoтоксичність)

Гіперчутливість II типу (антитілозалежна)

Гіперчутливість III типу (імунокомплексна)

Гіперчутливість I типу (анафілактична)

2991 / 6854
На розтині померлого було виявлено наявність крові в тонкій та товстій кишках (1,5-2 л). В нижніх відрізках клубової кишки стінка мала 'брудні' виразки на місці групових лімфатичних фолікулів. Решта пейєрових бляшок була зеленуватого кольору з демаркаційним запаленням навколо. Було діагностовано черевний тиф. Для якого періоду черевного тифу найбільш характерні описані морфологічні зміни кишки й ускладнення?

Загоювання виразок

Мозкоподібного набухання

Утворення виразок

Чистих виразок

Некроз

2992 / 6854
Ефективна діагностика носійства збудників кишкових інфекцій ґрунтується на виявленні антитіл до певних антигенів бактерій в реакції непрямої гемаглютинації. Який стандартний препарат слід застосувати у цій реакції?

Еритроцити барана й гемолітичну сироватку

Антитіла проти імуноглобулінів основних класів

Еритроцитарні діагностикуми з адсорбованими антигенами бактерій

Моноклональні антитіла

Монорецепторні діагностичні сироватки

2993 / 6854
У туриста під час тривалого перебування на спекоті відбулася значна втрата води, що супроводжувалося різким зниженням діурезу. Посилення секреції яких гормонів відбувається при цьому?

Ілюкокортикоїди й інсулін

Адреналін і норадреналін

Тироксин і трийодтиронін

Вазопресин й альдостерон

Серотонін і дофамін

2994 / 6854
Після травми на рентгенограмі тазу виявлена тріщина кістки, яка має затульну борозну. Яка це кістка?

Сіднична

Лобкова

Крижова

Куприкова

Клубова

2995 / 6854
У пацієнта стоматологічного відділення виявлено хворобу Педжета, що супроводжується деградацією колагену. Вирішальним фактом для постановки діагнозу було виявлення у сечі хворого підвищеного рівня:

Триптофану

Аланіну

Оксипроліну

Аргініну

Серину

2996 / 6854
Школяр 8-ми років звернувся до стоматолога з герпетичним висипанням на нижній губі. Який найбільш ефективний засіб слід призначити?

Фурадонін

Ампіцилін

Кетоконазол

Ацикловір

Оксацилін

2997 / 6854
У пацієнта перед кардіологічною операцією зареєстровано тиск у всіх відділах серця. Який тиск в лівому шлуночку під час діастоли?

100 мм рт.ст.

40 мм рт.ст.

0 мм рт.ст.

120 мм рт.ст.

80 мм рт.ст.

2998 / 6854
Хворий 47-ми років впродовж останніх 3-х років хворіє на туберкульоз легень, скаржиться на задишку, важкість в області правого боку грудної стінки, температуру тіла 37,7°С. Виявлено правобічний ексудативний плеврит. Який тип клітин передбачається у плевральному пунктаті?

Еритроцити

Еозинофіли

Нейтрофіли

Лімфоцити

Атипові клітини

2999 / 6854
У п’ятирічного хлопчика спостерігалися малий зріст, розумове відставання, обмежені рухи, грубі риси обличчя. ЦІ особливості стали помітними з 18-місячного віку. У нього виявили дефіцит L-ідуронідази. Обмін яких сполук порушено?

Фосфоліпіди

Вітаміни

Глікозаміноглікани

Білки

Нуклеотиди

3000 / 6854
Дитячий стоматолог мав контакт з хворим на дифтерію підлітком. Час останньої імунізації лікаря проти дифтерії - 12 років. Який препарат необхідно ввести стоматологу?

Жива вакцина

Антитоксична протидифтерійна сироватка

Рекомбінантна вакцина

Інтерферон

Хімічна вакцина

3001 / 6854
У хворого після перелому верхньої третини плечової кістки розвинувся параліч задньої групи м’язів плеча i передпліччя. Який нерв пошкоджено?

Пахвовий

М’язово-шкірний

Променевий

Ліктьовий

Серединний

3002 / 6854
Хворому з ревматоїдним артритом тривалий час вводили гідрокортизон. У нього з’явилися гіперглікемія, поліурія, глюкозурія, спрага. Ці ускладнення лікування є наслідком активації процесу:

Глікогеноліз

Глікогенез

Гліколіз

Глюконеогенез

Ліполіз

3003 / 6854
Для корекції артеріального тиску при колаптоїдному стані хворому було введено мезатон. Який механізм дії даного препарату?

Блокує β-адренорецептори

Стимулює β-адренорецептори

Стимулює α- і β-адренорецептори

Стимулює α-адренорецептори

Блокує α-адренорецептори

3004 / 6854
На електронній мікрофотографії представлені структури у вигляді відкритих міхурців, внутрішня поверхня яких вистелена одношаровим епітелієм, який утворений респіраторними та секреторними клітинами. Які це структури?

Ацинуси

Термінальні бронхіоли

Бронхіоли

Альвеолярні ходи

Альвеоли

3005 / 6854
В гістопрепараті представлений паренхіматозний орган, поверхневий шар кіркової речовини якого формують клубочки, утворені ендокриноцитами. Якому органу належить дана морфологічна ознака?

Селезінка

Лімфатичний вузол

Яєчник

Щитовидна залоза

Наднирник

3006 / 6854
Після переходу до змішаного харчування у новонародженої дитини виникла диспепсія з діареєю, метеоризмом, відставанням у розвитку. Біохімічна основа даної патології полягає у недостатності:

Ліпази та креатинкінази

Трипсину та хімотрипсину

Лактази та целобіази

Целюлази

Сахарази та ізомальтази

3007 / 6854
Хворому перед операцією було введено дитилін (лістенон) і проведено інтубацію. Дефіцит якого ферменту в організмі хворого подовжує дію м’язового релаксанту?

Карбангідраза

N-ацетилтрансфераза

Сукцинатдегідрогеназа

K-Na-АТФ-аза

Псевдохолінестераза

3008 / 6854
У хворого із захворюванням печінки виявлено зниження вмісту протромбіну в крові. Це призведе, перш за все, до порушення:

Антикоагулянтних властивостей крові

Першої фази коагуляційного гемостазу

Судинно-тромбоцитарного гемостазу

Другої фази коагуляційного гемостазу

Фібринолізу

3009 / 6854
У хворого із сечокам’яною хворобою виникли нестерпні спастичні болі. Для попередження больового шоку йому ввели разом з атропіном наркотичний анальгетик, що не має спазмогенного ефекту. Який це був препарат?

Пірітрамід

Трамадол

Промедол

Морфіну гідрохлорид

Етилморфіну гідрохлорид

3010 / 6854
Яким буде скорочення м’язів верхньої кінцівки при намаганні підняти непосильний вантаж?

Одиночне

Ізотонічне

Ізометричние

Ауксотонічне

Фазичне

3011 / 6854
Для лікування урогенітальних інфекцій використовують хінолони - інгібітори ферменту ДнК-гірази. Який процес порушується під дією хінолонів у першу чергу?

Реплікація ДНК

Ампліфікація генів

Зворотна транскрипція

Репарація ДНК

Рекомбінація генів

3012 / 6854
Жінку госпіталізовано в клініку з симптомами гострого живота. При обстеженні виникла підозра на позаматкову вагітність. Яке з анатомічних утворень таза необхідно пропунктувати для підтвердження діагнозу?

Excavatio vesicouterina

Excavatio rectovesicalis

Excavatio rectouterina

Fossa ischiorectalis

Processus vaginalis peritonei

3013 / 6854
Щуру в плевральну порожнину введено 0,5 мл повітря. Який тип недостатності дихання виникає в даному випадку?

Обструктивне порушення альвеолярної вентиляції

Дисрегуляторне порушення альвеолярної вентиляції

Рестриктивне порушення альвеолярної вентиляції

Перфузійний

Дифузійний

3014 / 6854
Дитина 10-ти років страждає на стафілококовий дерматит. Лікування бензилпеніциліном не дало результатів. Призначення комбінованого препарату пеніциліну з клавулановою кислотою дало швидке одужання. Яка причина позитивної дії цього препарату?

Інактивація β-лактамази

Гальмування аденозиндезамінази

Гальмування транспептидази

Активація фосфодіестерази

Блокада транслокази

3015 / 6854
У хворого спостерігається типова для нападу малярії клінічна картина: озноб, жар, проливний піт. Яка стадія малярійного плазмодію найімовірніше буде виявлена в крові хворого в цей час?

Мікро- або макрогамети

Спороциста

Спорозоїт

Мерозоїт

Оокінета

3016 / 6854
У немовляти спостерігаються епілептиформні судоми, викликані дефіцитом вітаміну В6. Це спричинено зменшенням у нервовій тканині гальмівного медіатора - γ-аміномасляної кислоти. Активність якого ферменту знижена при цьому?

Глутаматдекарбоксилаза

Глутаматдегідрогеназа

Піридоксалькіназа

Аланінамінотрансфераза

Глутаматсинтетаза

3017 / 6854
Жінку 44-х років вжалила оса внаслідок чого розвинувся шок. В анамнезі вже була важка алергічна реакція на жалення оси. Об’єктивно: пульс -179/хв., слабкий, АТ- 80/40 ммрт.ст., ЧД-26/хв. Яка провідна ланка патогенезу анафілактичного шоку?

Тахікардія

Зменшення об’єму циркулюючої крові

Зменшення ударного об’єму серця

Біль

Зниження периферійного опору судин

3018 / 6854
У препараті в одній з судин мікроциркуляторного русла середня оболонка утворена 1-2 шарами гладеньких міоцитів, які розташовані поодинці і мають спіралеподібний напрямок. Зовнішня оболонка представлена тонким шаром пухкої волокнистої сполучної тканини. Вкажіть вид судини:

Капіляр

Посткапіляр

Венула

Артеріоловенулярний анастомоз

Артеріола

3019 / 6854
Фекалії дитини, що хворіє на ентерит, емульгують в фізіологічному розчині і краплю емульсії наносять на елективне середовище: 10% молочно-сольовий, або жовтково-сольовий агар. Який мікроорганізм передбачається виділити?

Ентерокок

Стафілокок

Стрептокок

Кишкова паличка

Клебсієла

3020 / 6854
При обстеженні хворого виявлена характерна клініка колагенозу Вкажіть, збільшення якого показника сечі характерне для цієї патології:

Аргінін

Солі амонію

Гідроксипролін

Мінеральні солі

Глюкоза

3021 / 6854
При патологічних процесах, які супроводжуються гіпоксією, відбувається неповне відновлення молекули кисню в дихальному ланцюзі і накопичення пероксиду водню. Вкажіть фермент, який забезпечує його руйнування:

Цитохромоксидаза

Каталаза

Аконітаза

Кетоглутаратдегідрогеназа

Сукцинатдегідрогеназа

3022 / 6854
Електрофоретичне дослідження сироватки крові хворого пневмонією показало збільшення одної з білкових фракцій. Вкажіть її:

Альбуміни

β-глобуліни

α1 -глобуліни

α2 -глобуліни

γ-глобуліни

3023 / 6854
При травмі в області тазу у хворого на рентгенологічному знімку виявлено некроз головки стегнової кістки. Під час травми кульшового суглоба було пошкоджено зв’язку:

Клубово-стегнова

Головки стегнової кістки

Лобкова-стегнова

Сіднично-стегнова

3024 / 6854
Людині внутрішньовенно ввели 0,5 л ізотонічного розчину лікарської речовини. Які з рецепторів насамперед прореагують на зміни водно-сольового балансу організму?

Волюморецептори порожнистих вен і передсердь

Натрієві рецептори гіпоталамуса

Осморецептори гіпоталамусу

Осморецептори печінки

Барорецептори дуги аорти

3025 / 6854
В судово-медичній експертизі широко використовується метод дактилоскопії, який оснований на тому, що сосочковий шар дерми визначає строго індивідуальний малюнок на поверхні шкіри. Яка тканина утворює цей шар дерми?

Щільна оформлена сполучна тканина

Пухка волокниста неоформлена сполучна частина

Ретикулярна тканина

Щільна неоформлена сполучна тканина

Жирова тканина

3026 / 6854
Важливою складовою частиною ниркового фільтраційного бар’єру є тришарова базальна мембрана, яка має спеціальну сітчасту будову її середнього електроннощільного шару. Де міститься ця базальна мембрана?

Тонкі канальці

Капіляри перитубулярної капілярної сітки

Ниркове тільце

Дистальні прямі канальці

Проксимальні канальці

3027 / 6854
Чоловік 42-х років помер при явищах вираженої інтоксикації і дихальної недостатності. На розтині: тканина легень у всіх відділах строката, з множинними дрібновогнищевими крововиливами та вогнищами емфіземи. Гістологічно у легенях: геморагічна бронхопневмонія з абсцедуванням, у цитоплазмі клітин епітелію бронхів еозинофільні і базофільні включення. Діагностуйте виявлене на секції захворювання:

Часткова пневмонія

Грип

Стафілококова бронхопневмонія

Плевропневмонія

Крупозна пневмонія

3028 / 6854
В експерименті подразнюють скелетний м’яз серією електричних імпульсів. Який вид м’язового скорочення виникне, якщо кожний наступний імпульс припадає на період вкорочення поодинокого м’язового скорочення?

Серія поодиноких скорочень

Асинхронний тетанус

Зубчастий тетанус

Контрактура м’яза

Суцільний тетанус

3029 / 6854
У чоловіка 53-х років діагностовано сечокам’яну хворобу з утворенням уратів. Цьому пацієнту призначено аллопурінол, який є конкурентним інгібітором ферменту:

Дигідроурацилдегідрогеназа

Уратоксидаза

Уреаза

Ксантиноксидаза

Уриділтрансфераза

3030 / 6854
Мати зауважила занадто темну сечу у її 5-річної дитини. Дитина скарг не висловлює. Жовчних пігментів у сєчі не виявлено. Поставлено діагноз алкаптонурія. Дефіцит якого ферменту має місце у дитини?

Тирозиназа

Фенілаланінгідроксилаза

Оксидаза гомогентизинової кислоти

Оксидаза оксифенілпірувату

Декарбоксилаза фенілпірувату

3031 / 6854
До дерматолога звернулася пацієнтка із скаргами на екзематозне ураження шкіри рук, що з’являється після контакту з миючим засобом ”Лотос” Використання гумових рукавичок запобігає цьому. Патологічна реакція шкіри зумовлена активацією:

Т-лімфоцитів

В-лімфоцитів

Базофілів

Нейтрофілів

Моноцитів

3032 / 6854
У хворої 36-ти років, яка лікувалася сульфаніламідами з приводу респіраторної вірусної інфекції, в крові гіпорегенераторна нормохромна анемія, лейкопенія, тромбоцитопенія. В кістковому мозку - зменшення кількості мієлокаріоцитів. Яка це анемія?

Залізодефіцитна

Постгеморагічна

Гемолітична

Гіпопластична

В12-фолієводефіцитна

3033 / 6854
Чоловік 38-ми років раптово помер. На розтині: у задній стінці лівого шлуночка серця виявлено інфаркт міокарда. Які найбільш імовірні зміни у будові міокардіоцитів можна побачити у вогнищі інфаркту мікроскопічно?

Жирова дистрофія

Звапнування

Білкова дистрофія

Вуглеводна дистрофія

Каріолізис

3034 / 6854
У відділення реанімації надійшов чоловік 47-ми років з діагнозом інфаркт міокарда. Яка з фракцій лактатдегідрогенази (ЛДГ) буде переважати в сироватці крові впродовж перших двох діб захворювання?

ЛДГ1

ЛДГ4

ЛДГ2

ЛДГ3

ЛДГ5

3035 / 6854
У хлопчика 2-х років спостерігається збільшення в розмірах печінки та селезінки, катаракта. В крові підвищена концентрація цукру, але тест толерантності до глюкози в нормі. Спадкове порушення обміну якої речовини є причиною цього стану?

Сахароза

Галактоза

Мальтоза

Глюкоза

Фруктоза

3036 / 6854
У неврологічне відділення з приводу мозкового крововиливу поступив хворий 62-х років. Об’єктивно: стан важкий. Спостерігається наростання глибини і частоти дихання, а потім його зменшення до апное, після чого цикл дихальних рухів відновлюється. Який тип дихання у хворого?

Гаспінг-дихання

Чейна-Стокса

Біота

Кусмауля

Апнейстичне

3037 / 6854
Глікоген, що надійшов з їжею, гідролізується у шлунково-кишковому тракті. Який кінцевий продукт утворюється в результаті цього процесу?

Фруктоза

Глюкоза

Галактоза

Лактат

Лактоза

3038 / 6854
Після перенесеного геморагічного інсульту у хворого розвинулася кіста головного мозку. Через 2 роки помер від післягрипозної пневмонії. На секції трупа виявлено в мозку кісту із стінками біло-іржавого відтінку, реакція Перлса позитивна. Який з процесів найбільш імовірний у стінці кісти?

Загальний гемосидероз

Місцевий гемосидероз

Первинний гемохроматоз

Інфільтрація білірубіну

Місцевий гемомеланоз

3039 / 6854
При розтині тіла померлого чоловіка 73-х років, який довго страждав на ішемічну хворобу серця з серцевою недостатністю, знайдено: ’’мускатна” печінка, бура індурація легень, ціанотична індурація нирок та селезінки. Який з видів порушення кровообігу найбільш імовірний?

Гостре загальне венозне повнокрів’я

Хронічне малокрів’я

Артеріальна гіперемія

Хронічне загальне венозне повнокрів’я

Гостре малокрів’я

3040 / 6854
У нейрохірургічне відділення поступив 54-річний чоловік із скаргами на відсутність чутливості шкіри нижньої повіки, латеральної поверхні зовнішнього носа, верхньої губи. Лікар при огляді встановлює запалення другої гілки трійчастого нерва. Через який отвір виходить із черепа ця гілка?

Рваний отвір

Верхня очноямкова щілина

Овальний отвір

Остистий отвір

Круглий отвір

3041 / 6854
Чоловіку 18-ти років з приводу флегмони плеча було зроблено внутрішньом’язову ін’єкцію пеніциліну. Після цього у нього з’явилися тахікардія, ниткоподібний пульс, АТ знизився до 80/60 мм рт.ст. Який вид фармакологічної реакції розвинувся?

Периферична дія

Потенціювання

Анафілаксія

Центральна дія

Рефлекторна дія

3042 / 6854
Після побутової травми у пацієнта 18-ти років з’явились постійні запаморочення, ністагм очей, скандована мова, невпевнена хода. Це свідчить про порушення функції:

Мозочка

Рухової кори

Вестибулярних ядер

Чорної субстанції

Базальних гангліїв

3043 / 6854
Хворий 65-ти років, що страждає на атеросклероз, госпіталізований до хірургічного відділення з приводу розлитого гнійного перитоніту. Під час операції діагностовано тромбоз брижових артерій. Яка найбільш імовірна причина перитоніту?

Геморагічний інфаркт

Стаз

Ішемія ангіоспастична

Ішемічний інфаркт

Ішемія компресійна

3044 / 6854
Хворому з прогресуючою м’язовою дистрофією було проведено біохімічне дослідження сечі. Поява якої речовини у великій кількості в сечі може підтвердити захворювання м’язів у даного хворого?

Креатин

Креатинін

Сечовина

Порфірини

Гіпурова кислота

3045 / 6854
У крові чоловіка 26-ти років виявлено 18% еритроцитів сферичної, сплощеної, кулястої та остистої форм. Інші еритроцити були у формі двоввігнутих дисків. Як називається таке явище?

Патологічний анізоцитоз

Фізіологічний пойкілоцитоз

Еритроцитоз

Патологічний пойкілоцитоз

Фізіологічний анізоцитоз

3046 / 6854
Хвора страждає від болю в ногах та набряків. В ході обстеження хворої на медіальній поверхні стегна спостерігається набряк, збільшення розміру вен, утворення вузлів. З боку якої вени спостерігається патологія?

V femoralis

V saphena parva

V tibialis

V profunda femoris

V saphena magna

3047 / 6854
Під час підготовки пацієнта до операції на серці проведено вимірювання тиску в камерах серця. В одній з них тиск протягом серцевого циклу змінювався від 0 до 120 мм рт.ст. Назвіть цю камеру серця:

Ліве передсердя

Лівий шлуночок

Правий шлуночок

Праве передсердя

3048 / 6854
У жінки обмежений кровотік у нирках, підвищений артеріальний тиск. Гіперсекреція якого гормону зумовила підвищений тиск?

Вазопресин

Норадреналін

Адреналін

Ренін

Еритропоетин

3049 / 6854
На останньому місяці вагітності вміст фібриногену в плазмі крові в 2 рази вище за норму. Яку швидкість осідання еритроцитів слід при цьому очікувати?

10-15 мм/годину

40-50 мм/годину

0-5 мм/годину

5-10 мм/годину

3-12 мм/годину

3050 / 6854
Пацієнт 16-ти років, що страждає на хворобу Іценко-Кушінга, консультований з приводу надмірної ваги тіла. При опитуванні з’ясувалося, що енергетична цінність спожитої їжі складає 1700-1900 ккал/добу. Яка провідна причина ожиріння у даному випадку?

Нестача інсуліну

Надлишок глюкокортикоїдів

Надлишок інсуліну

Нестача глюкокортикоїдів

Гіподинамія

3051 / 6854
Людина зробила спокійний видих. Як називається об’єм повітря, який міститься у неї в легенях при цьому?

Дихальний об’єм

Функціональна залишкова ємкість легень

Залишковий об’єм

Резервний об’єм видиху

Життєва ємність легень

3052 / 6854
Людина зробила максимально глибокий видих. Як називається об’єм повітря, що знаходиться в її легенях після цього?

Функціональна залишкова ємність легень

Залишковий об’єм

Альвеолярний об’єм

Резервний об’єм видиху

Ємність вдиху

3053 / 6854
В експерименті на ссавці зруйнували певну структуру серця, що призвело до припинення проведення збудження від передсердь до шлуночків. Що саме зруйнували?

Синоатріальний вузол

Волокна Пуркін’є

Пучок Гіса

Ніжки пучка Гіса

Атріовентрикулярний вузол

3054 / 6854
Необхідно оцінити рівень збудливості нерва у хворого. Для цього доцільно визначити для нерва наступну величину:

Критичний рівень деполяризації

Тривалість потенціалу дії

Амплітуда потенціалу дії

Потенціал спокою

Порогова сила подразника

3055 / 6854
Хворому з гострою недостатністю надниркових залоз був призначений лікарський препарат після якого у нього з’явилися скарги на біль в кістках (двічі були переломи), часті простудні хвороби, набряки, повільне загоювання ран. Який препарат міг спричинити такі явища?

Тестостерон

Естріол

Спіронолактон

Ретаболіл

Преднізолон

3056 / 6854
Під час розтину тіла мертвонародженої дитини виявлено аномалію розвитку серця: шлуночки не розмежовані, з правої частини виходить суцільний артеріальний стовбур. Для яких хребетних характерна подібна будова серця?

Риби

Птахи

Амфібії

Ссавці

Рептилії

3057 / 6854
Який з нижченаведених сечогінних засобів слід призначити хворому з первинним гіперальдостеронізмом?

Фуросемід

Маніт

Гіпотіазид

Спіронолактон

Триамтерен

3058 / 6854
В результаті травми хворий не може підняти руку до горизонтального рівня. Який м’яз постраждав?

Трапецієподібний

Триголовий

Дельтоподібний

Найширший м’яз спини

Двоголовий

3059 / 6854
В експерименті збільшили проникність мембрани збудливої клітини для іонів калію. Які зміни електричного стану мембрани при цьому виникнуть?

Потенціал дії

Локальна відповідь

Гіперполяризація

Змін не буде

Деполяризація

3060 / 6854
При обстеженні в аналізі крові пацієнта виявлено лейкоцитоз, лімфоцитоз, клітини Боткіна- Гумпрехта на тлі анемії. Про яку хворобу слід думати лікарю?

Інфекційний мононуклеоз

Лімфогранулематоз

Хронічний лімфолейкоз

Мієломна хвороба

Гострий мієлолейкоз

3061 / 6854
У жінки 37-ми років протягом року періодично виникали інфекційні захворювання бактеріального генезу, їх перебіг був вкрай тривалим, ремісії - короткочасними. При обстеженні виявлена гіпогамаглобулінемія. Порушення функції яких клітин може бути прямою її причиною?

Макрофаги

Фагоцити

Нейтрофіли

Лімфоцити

Плазматичні клітини

3062 / 6854
У хворого 37-ми років на фоні тривалого застосування антибіотиків спостерігається підвищена кровоточивість при невеликих пошкодженнях. У крові - зниження активності факторів згортання крові ІІ, VII, IX, X, подовження часу згортання крові. Недостатністю якого вітаміну обумовлені зазначені зміни?

Вітамін D

Вітамін К

Вітамін Е

Вітамін А

Вітамін С

3063 / 6854
Хворий 18-ти років звернувся до лікарні із скаргами на шум та больові відчуття у вусі. Об’єктивно - у хворого гостре респіраторне захворювання, риніт. Крізь який отвір глотки інфекція потрапила до барабанної порожнини та викликала її запалення?

Зів

Глотковий отвір слухової труби

Вхід до гортані

Барабанний отвір слухової труби

Хоани

3064 / 6854
У хворого 68-ми років, який страждає на серцеву недостатність та впродовж тривалого часу приймає препарати наперстянки, з’явилися явища інтоксикації, які швидко нівелювалися застосуванням донатора сульфгідрильних груп унітіолу. Який механізм терапевтичної дії цього засобу?

Іальмує вивільнення калію з міокардіоцитів

Сповільнює надходження натрію до міокардіоцитів

Реактивує натрій-калієву-АТФ-азу мембран міокардіоцитів

Підвищує енергозабезпечення міокарду

Зменшує накопичення іонізованого кальцію

3065 / 6854
Охолодження тіла людини у воді виникає значно швидше, ніж на повітрі, тому, що у воді значно ефективнішою є віддача тепла шляхом:

Випаровування поту

Тепловипромінювання

Конвекції

Теплопроведення

3066 / 6854
У бактеріологічній лабораторії проводиться дослідження якості питної води. Її мікробне число виявилося близько 100. Які мікроорганізми враховувалися при цьому?

Бактерії, патогенні для людей та тварин

Ентеропатогенні бактерії та віруси

Бактерії групи кишкової палички

Умовно-патогенні мікроорганізми

Всі бактерії, що виросли на живильному середовищі

3067 / 6854
У юнака 20-ти років діагностовано спадковий дефіцит УДФ-глюкуронілтрансферази. Підвищення якого показника крові підтверджує діагноз?

Прямий (кон’югований) білірубін

Тваринний індикан

Стеркобіліноген

Непрямий (некон’югований) білірубін

Уробілін

3068 / 6854
Хворому 50-ти років з хронічною серцевою недостатністю і тахиаритмією призначили кардіотонічний препарат. Який з препаратів призначили хворому?

Дофамін

Мілдронат

Аміодарон

Дигоксин

Добутамін

3069 / 6854
У людини порушено всмоктування продуктів гідролізу жирів. Причиною цього може бути дефіцит у порожнині тонкої кишки:

Жовчних кислот

Іонів натрію

Жиророзчинних вітамінів

Ліполітичних ферментів

Жовчних пігментів

3070 / 6854
У відповідь на сильне швидке скорочення м’яза спостерігається його рефлекторне розслаблення. З подразнення яких рецепторів починається ця рефлекторна реакція?

Сухожилкові рецептори Гольджі

Суглобові рецептори

Больові рецептори

М’язові веретена

Дотикові рецептори

3071 / 6854
У хворого, що переніс 5 років тому субтотальну резекцію шлунка, розвинулась В12-фолієводефіцитна анемія. Який механізм є провідним у розвитку такої анемії?

Відсутність внутрішнього фактора Касла

Порушення всмоктування вітаміну В в тонкій кишці

Відсутність зовнішнього фактора Касла

Дефіцит фолієвої кислоти

Дефіцит транскобаламіну

3072 / 6854
Чоловік 50-ти років хворіє на хронічний бронхіт, скаржиться на задишку під час фізичного навантаження, постійний кашель з відходженням харкотиння. При обстеженні діагностовано ускладнення - емфізема легень. Чим вона зумовлена?

Зменшення розтяжності легень

Порушення вентиляційно-перфузійного співвідношення в легенях

Зменшення альвеолярної вентиляції

Зменшення перфузії легень

Зниження еластичних властивостей легень

3073 / 6854
При розтині трупа чоловіка 47-ми років, померлого раптово, в інтимі черевного відділу аорти знайдені осередки жовтого кольору у вигляді плям та смуг, що не вибухають над поверхнею інтими. При фарбуванні суданом III спостерігається жовтогаряче забарвлення. Для якої стадії атеросклерозу характерні такі зміни?

Ліпосклерозу

Атероматозу

Стадія утворення атероматозної виразки

Атерокальцинозу

Ліпоїдозу

3074 / 6854
У клініку госпіталізовано хворого з діагнозом карциноїду кишечника. Аналіз виявив підвищену продукцію серотоніну, який утворюється з амінокислоти триптофан. Який біохімічний механізм лежить в основі даного процесу?

Утворення парних сполук

Мікросомальне окиснення

Трансамінування

Декарбоксилювання

Дезамінування

3075 / 6854
У батьків, хворих на гемоглобінопатію (аутосомно-домінантний тип успадкування), народилася здорова дівчинка. Які генотипи батьків?

Батько гетерозиготний за геном гемоглобінопатії, у матері цей ген відсутній

Обоє гетерозиготні за геном гемоглобінопатії

Обоє гомозиготні за геном гемоглобінопатії

Мати гетерозиготна за геном гемоглобінопатії, у батька цей ген відсутній

У обох батьків ген гемоглобінопатії відсутній

3076 / 6854
Хворому з артеріальною гіпертензією було призначено один з антигіпертензивних засобів. Артеріальний тиск нормалізувався, однак хворого почав турбувати постійний сухий кашель. Який з перерахованих препаратів має таку побічну дію?

Анаприлін

Ніфедипін

Резерпін

Клофелін

Лізиноприл

3077 / 6854
В анотації до препарату вказано, що він містить антигени збудника черевного тифу, адсорбовані на стабілізованих еритроцитах барана. З якою метою використовують цей препарат?

Для серологічної ідентифікації збудника черевного тифу

Для виявлення антитіл в реакції Відаля

Для виявлення антитіл в реакції непрямої гемаглютинації

Для виявлення антитіл в реакції зв’язування комплементу

Для виявлення антитіл в реакції гальмування гемаглютинації

3078 / 6854
При розтині тіла померлого чоловіка 48-ми років в ділянці 1-го сегменту правої легені виявлено круглий утвір діаметром 5 см з чіткими контурами, оточений тонким прошарком сполучної тканини, виповнений білими крихкими масами. Діагностуйте форму вторинного туберкульозу:

Туберкулома

Гострий вогнищевий туберкульоз

Фіброзно-кавернозний туберкульоз

Гострий кавернозний туберкульоз

Казеозна пневмонія

3079 / 6854
У хворого діагностовано септичний ендокардит. Температура тіла протягом 5-ти днів коливалася в межах 39,5oC -40,2oC. На 6-й день на тлі різкого зниження температури до35,2oC розвинувся колапс. Який головний механізм колапсу?

Вазодилатація

Поліурія

Тахікардія

Гіпервентиляція

Посилене потовиділення

3080 / 6854
Хвора звернулася зі скаргами на болі у правій латеральній ділянці живота. Під час пальпації визначається щільне, нерухоме, пухлиноподібне утворення. У ділянці якого відділу травної трубки можлива наявність пухлини?

Colon descendens

Caecum

Colon transversum

Colon sigmoideum

Colon ascendens

3081 / 6854
У хворого струс головного мозку, що супроводжується повторним блюванням і задишкою. При обстеженні відзначено: рН - 7,62; pCO - 40 мм рт.ст. Яке порушення кислотно-основного стану є у хворого?

Газовий алкалоз

Негазовий ацидоз

Негазовий алкалоз

Газовий ацидоз

3082 / 6854
У хворого в обох щелепах рентгенологічно виявлено численні дефекти у вигляді гладкостінних округлих отворів. При гістологічному дослідженні - явища остеолізису і остеопорозу при явищах слабкого кісткоутворення. В сечі хворого знайдено білок Бенс-Джонса. Назвіть захворювання:

Гострий недиференційований лейкоз

Хронічний еритромієлоз

Мієломна хвороба

Хронічний мієлолейкоз

Гострий мієлолейкоз

3083 / 6854
Катіонні глікопротеїни є основними компонентами слини привушних залоз. Які амінокислоти обумовлюють їх позитивний заряд?

Аспартат, глутамат, гліцин

Аспартат, аргінін, глутамат

Цистеїн, гліцин, пролін

Лізин, аргінін, гістидин

Глутамат, валін, лейцин

3084 / 6854
На мікропрепараті очного яблука плода спостерігається пошкодження рогівки. Частина якого зародкового листка була уражена в процесі ембріонального розвитку?

Дерматом

Ектодерма

Ентодерма

Мезодерма

Нефротом

3085 / 6854
На практичному занятті з мікробіології студентам запропоновано пофарбувати суміш бактерій за методикою Грама та пояснити механізм фарбування. Які морфологічні структури бактерій зумовлюють грамнегативне та грампозитивне фарбування бактерій?

Джгутики

Цитоплазма

Капсула

Клітинна стінка

ЦПМ

3086 / 6854
У вагітної жінки взяли кров для підтвердження клінічного діагнозу 'токсоплазмоз' Яка з перерахованих серологічних реакцій має діагностичне значення?

Реакція гемадсорбції

Реакція гальмування гемаглютинації

Реакція зв’язування комплементу

Реакція аглютинації

Реакція нейтралізації

3087 / 6854
У здорових батьків, спадковість яких не обтяжена, народилася дитина з множинними вадами розвитку. Цитогенетичний аналіз виявив у соматичних клітинах дитини трисомію за 18-ю хромосомою (синдром Едвардса). З яким явищем пов’язане народження такої дитини?

Домінантною мутацією

Соматичною мутацією у ембріона

Нерозходженням пари хромосом під час гаметогенезу

Хромосомною мутацією - дуплікацією

Впливом тератогенних факторів

3088 / 6854
На препараті представлено орган, вкритий сполучнотканинною капсулою, від якої відходять трабекули. В органі можна розрізнити кіркову речовину, де містяться лімфатичні вузлики та мозкову речовину, представлену тяжами лімфоїдних клітин. Який орган представлений на препараті?

Червоний кістковий мозок

Лімфатичний вузол

Селезінка

Тимус

Мигдалики

3089 / 6854
При аналізі родоводу пробанда виявлено, що ознака проявляється з однаковою частотою у представників обох статей і хворі наявні у всіх поколіннях (по вертикалі), а по горизонталі - у сибсів (братів і сестер пробанда) з відносно великих родин. Який тип успадкування досліджуваної ознаки?

Автосомно-домінантний

Автосомно-рецесивний

Зчеплений з Y-хромосомою

Зчеплений з Х-хромосомою, рецесивний

Зчеплений з Х-хромосомою, домінантний

3090 / 6854
У 60-річного пацієнта було виявлено гіперглікемію і глюкозурію. Для лікування хворого лікар призначив препарат для приймання всередину. Який це препарат?

Корглікон

Фуросемід

Глібенкламід

Окситоцин

Панкреатин

3091 / 6854
Хворий 58-ми років помер від прогресуючої серцевої недостатності. На розтині: серце розширене у поперечнику, мляве, м’яз на розрізі нерівномірного кровонаповнення, пістрявий. При гістологічному дослідженні: у міокарді повнокров’я, у стромі лімфогістіоцитарні інфільтрати, що розсувають кардіоміоцити. Виявлені морфологічні зміни свідчать про:

Інфаркт міокарда

Негнійний проміжний міокардит

Венозне повнокров’я

Жирову дистрофію міокарда

Кардіосклероз

3092 / 6854
У важкоатлета при підйомі штанги відбувся розрив грудної лімфатичної протоки. Вкажіть найбільш імовірне місце ушкодження:

Ділянка шиї

Місце впадіння у венозний кут

Ділянка попереково-крижового сполучення

Заднє середостіння

Ділянка аортального отвору діафрагми

3093 / 6854
При пошкодженні клітини іонізуючим випромінюванням вмикаються механізми захисту і адаптації. Який механізм відновлення порушеного внутрішньоклітинного гомеостазу реалізується при цьому?

Активація Са-опосередкованих клітинних функцій

Гіпертрофія мітохондрій

Накопичення Na+ в клітинах

Активація антиоксидантної системи

Пригнічення аденілатциклази

3094 / 6854
У хворої після видалення матки розвинулась гостра анурiя (немає виділення сечі). Які анатомiчнi структури найімовірніше було пошкоджено при операції?

Зовнішній сфінктер сечівника

Внутрішній сфінктер сечівника

Сечоводи

Сечівник

Цибулинно-губчастий м’яз

3095 / 6854
У хворої внаслідок запалення порушена ендокринна функція фолікулярних клітин фолікулів яєчника. Синтез яких гормонів буде пригнічений?

Прогестерон

Лютропін

Фолікулостимулюючий гормон

Естрогени

Фолістатин

3096 / 6854
У чоловіка при обстеженні виявлено порушення кровообігу міокарда лівого передсердя. У басейні якої артерії відбулись порушення кровообігу?

Права вінцева

Ліва вінцева

Права та ліва вінцеві

Передня міжшлуночкова гілка лівої вінцевої артерії

3097 / 6854
Чоловік 36-ти років, лісник за фахом, через тиждень після тривалого перебування у весняному лісі гостро захворів - гарячка, головний біль, порушення свідомості,епілептиформні напади, смерть розвинулася на 3-й день хвороби. На розтині тіла: набрякголовного мозку, множинні точкові геморагії; під час мікроскопічного дослідження - периваскулярний та перицелюлярний набряк, множинні периваскулярні, переважно лімфоцитарні, інфільтрати. Діагностуйте основне захворювання:

Поліомієліт

Церебро-васкулярна хвороба

Гнійний енцефаліт

Кліщовий енцефаліт

Менінгококова інфекція

3098 / 6854
При зовнішньому дослідженні трупа чоловіка 69-ти років, який помер 4 години тому, патологоанатом відмітив, що м’язи померлого мають дуже щільну консистенцію, суглоби згинаються та розгинаються важко. Як називається ця патологоанатомічна ознака смерті?

Трупне розкладання

Трупне висихання

Трупне охолодження

Трупні гіпостази

Трупне заклякання

3099 / 6854
Хворому з метою попередження жирової дистрофії печінки лікар призначив ліпотропний препарат - донор метильних груп. Це імовірно:

Холестерин

Глюкоза

S-Аденозилметіонін

Білірубін

Валін

3100 / 6854
Хворому на гострий інфаркт міокарда у комплексній терапії було призначено гепарин. Через деякий час після введення даного препарату з’явилась гематурія. Який антагоніст гепарину необхідно ввести хворому для усунення даного ускладнення?

Фібриноген

Амінокапронова кислота

Протаміну сульфат

Неодикумарин

Вікасол

3101 / 6854
У хворого виявили злоякісну анемію. Терапія внутрішньо-м’язовим введення вітаміну Б12 давала нетривалий нестійкий ефект поліпшення складу крові. Пацієнт - завзятий рибаль і часто вживає самостійно виловлену і недостатньо термічно оброблену рибу. Який діагноз можна припустити?

Парагонімоз

Ентеробіоз

Анкілостомоз

Трихоцефальоз

Дифілоботріоз

3102 / 6854
У синтезі пуринових нуклеотидів беруть участь деякі амінокислоти, похідні вітамінів, фосфорні ефіри рибози. Коферментна форма якого вітаміну є переносником одновуглецевих фрагментів в цьому синтезі?

Фолієва кислота

Нікотинова кислота

Рибофлавін

Піридоксин

Пантотенова кислота

3103 / 6854
У тварини зруйнували отолітові вестибулорецептори. Які з наведених рефлексів зникнуть внаслідок цього у тварини?

Статокінетичні при рухах з кутовим прискоренням

Випрямлення тулуба

Первинні орієнтувальні

Міотатичні

Статокінетичні при рухах з лінійним прискоренням

3104 / 6854
Лікар записав в історії хвороби, що у хворого дихання поверхневе (знижена глибина дихання). Це означає, що зменшеним є такий показник зовнішнього дихання:

Хвилинний об’єм дихання

Функціональна залишкова ємність

Дихальний об’єм

Життєва ємність легень

Ємність вдиху

3105 / 6854
До лікаря звернувся студент з проханням призначити препарат для лікування алергічного риніту, який виник у нього під час цвітіння липи. Який засіб можна застосувати?

Лоратадин

Норадреналіну гідротартрат

Лозартан

Амброксол

Анаприлін

3106 / 6854
Хворий похилого віку страждає на хронічний закреп, в основі якого лежить гіпотонія товстої кишки. Який препарат слід призначити хворому?

Атропіну сульфат

Натрію сульфат

Бісакодил

Новокаїнамід

Касторова олія

3107 / 6854
При лабораторному дослідженні дитини виявлено підвищений вміст у крові та сечі лейцину, валіну, ізолейцину та їх кетопохідних. Сеча має характерний запах кленового сиропу. Недостатність якого ферменту характерно для цього захворювання?

Дегідрогеназа розгалужених амінокислот

Амінотрансфераза

Глюкозо-6-фосфатаза

Фосфофруктокіназа

Фосфофруктомутаза

3108 / 6854
При обстеженні 2-х місячної дитини педіатр звернула увагу, що плач дитини нагадує котячий крик. Діагностовані мікроцефалія і вада серця. За допомогою цитогенетичного метода з’ясований каріотип дитини: 46, XX, 5р. Дане захворювання є наслідком такого процесу:

Делеція

Дуплікація

Інверсія

Транслокація

Плейотропія

3109 / 6854
Під час аутопсії тіла хворого, котрий помер від тяжкого ексикозу, на ґрунті профузної діареї, виявлені такі зміни: слизова оболонка прямої та сигмоподібної кишок на всьому протязі вкрита сіро-білими плівчастими нашаруваннями, що міцно з’єднані з підлеглими тканинами, між плівками розташовані множинні великі й поверхневі виразки, вкриті згортками крові. Мікроскопічно діагностовано фібринозно-виразковий коліт. Яке захворювання проявляється такими змінами?

Дизентерія

Сальмонельози

Стафілококова інфекція

Колі-інфекція

Ієрсініоз

3110 / 6854
У лікарню до кінця робочого дня доставлений працівник 'гарячого' цеху, який скаржиться на головний біль, запаморочення, нудоту, загальну слабкість. Свідомість збережена, шкірні покриви гіперемовані, сухі, гарячі на дотик. ЧСС- 130/хв. Дихання часте, поверхневе. Яке порушення процесів регуляції тепла найімовірніше виникло у людини у даній ситуації?

Посилення тепловіддачі і теплопродукції

Зниження тепловіддачі

Посилення теплопродукції без змін тепловіддачі

Зниження теплопродукції без зміни тепловіддачі

Посилення тепловіддачі і зниження теплопродукції

3111 / 6854
В експерименті подразнюють гілочки симпатичного нерва, які іннервують серце. Це призвело до збільшення сили серцевих скорочень, тому що через мембрану типових кардіоміоцитів збільшився:

Вхід іонів калію

Вхід іонів кальцію та калію

Вихід іонів кальцію

Вхід іонів кальцію

Вихід іонів калію

3112 / 6854
У пацієнта 60-ти років виявлено збільшення порогу сприймання звуків високої частоти. Зміна функцій яких структур слухового аналізатора зумовлює виникнення цього порушення?

М’язів середнього вуха

Євстахієвої труби

Органу Корті ближче до овального віконця

Органу Корті ближче до гелікотреми

Барабанної перетинки

3113 / 6854
У 3-річної дитини тривале підвищення температури, збільшені лімфовузли, у крові - значне підвищення кількості лімфоцитів. Методом ІФА виявлено антиген віруса Епштейна-Бара. Який діагноз можна поставити на основі вказаного?

Іенералізована інфекція, викликана herpes-zoster

Цитомегаловірусна інфекція

Інфекційний мононуклеоз

Іерпетична аденопатія

Лімфома Беркета

3114 / 6854
У дитини виявлена схильність до ожиріння, яка є результатом діатезу. Назвіть вид діатезу, при якому частіше може розвинутись ожиріння:

Нервово-артритичний

Астенічний

Ексудативно-катаральний

Лімфатико-гіпопластичний

3115 / 6854
У юнака 18-ти років діагностовано хворобу Марфана. При дослідженні встановлено порушення розвитку сполучної тканини, будови кришталика ока, аномалії серцево-судинної системи, арахнодактилію. Прикладом якого генетичного явища є ця хвороба?

Кодомінування

Неповне домінування

Комплементарність

Множинний алелізм

Плейотропія

3116 / 6854
Ураження хворого одноразовою дозою іонізуючого випромінювання спричинило розвиток кістково-мозкової форми променевої хвороби. Які патологічні прояви з боку крові будуть характерними в період удаваного благополуччя?

Наростаюча лімфопенія, лейкопенія

Тромбоцитопенія, анемія

Перерозподільчий лейкоцитоз, лім-фоцитоз

Анемія, лейкопенія

Тромбоцитопенія, лейкоцитоз

3117 / 6854
У хворого на черевний тиф при проведенні серологічного дослідження (реакція Відаля) виявлено О- і Н-аглютитиніни у титрі 1:800 і 1:200 відповідно. Це свідчить про:

Проведене щеплення

Початок захворювання

Неможливість підтвердити діагноз

Раніше перенесене захворювання

Період реконвалесценції

3118 / 6854
У дитячому колективі проведено планову вакцинацію проти кору. Яким методом можна перевірити ефективність проведеної вакцинації?

Алергопроба

Біологічний

Серологічний

Вірусологічний

Вірусоскопічний

3119 / 6854
У клінічній практиці застосовують для лікування туберкульозу препарат ізоніазид - антивітамін, який здатний проникати у туберкульозну палочку. Туберкулостатичний ефект обумовлений порушенням процесів реплікації, окисно-відновних реакцій завдяки утворенню несправжнього коферменту з:

ФАД

ФМН

ТДФ

КоО

НАД

3120 / 6854
При огляді хворої лікар-гінеколог відмітив симптоми запалення статевих шляхів, у мазку взятому із піхви, виявлено грушоподібні найпростіші з шипом, з передньої частини відходять джгутики, наявна ундулююча мембрана. Яке захворювання підозрює лікар у хворої?

Лямбліоз

Кишковий трихомоноз

Урогенітальний трихомоноз

Балантидіоз

Токсоплазмоз

3121 / 6854
У чоловіка 32-х років, хворого на пневмонію, спостерігається закупорка харкотинням дихальних шляхів. В організмі хворого при цьому буде розвиватися така зміна кислотно-лужної рівноваги:

Метаболічний ацидоз

Змін не буде

Респіраторний ацидоз

Респіраторний алкалоз

Метаболічний алкалоз

3122 / 6854
При аналiзi ЕКГ виявлено випадіння деяких серцевих циклів PQRST. Наявні зубці та комплекси не змінені. Назвать вид аритмії:

Атріовентрикулярна блокада

Внутрішньопередсердна блокада

Передсердна екстрасистола

Миготлива аритмія

Синоатріальна блокада

3123 / 6854
У хворої встановлено порушення виділення тиреотропного гормону гіпофіза. Зі зниженням функцій якої частки гіпофіза це пов’язано?

Pars intermedia

Lobus anterior

Infundibulum

Lobus posterior

3124 / 6854
Стресовий стан і больове відчуття у пацієнта перед візитом до стоматолога супроводжуються анурією (відсутністю сечовиділення). Це явище зумовлене збільшенням:

Секреції вазопресину та зменшенням адреналіну

Активності парасимпатичної нервової системи

Активності антиноціцептивної системи

Секреції адреналіну та зменшенням вазопресину

Секреції вазопресину та адреналіну

3125 / 6854
Введення знеболюючого пацієнту перед екстракцією зуба призвело до розвитку анафілактичного шоку, який супроводжувався розвитком олігурії. Який патогенетичний механізм зумовив зменшення діурезу в даній клінічній ситуації?

Зниження гідростатичного тиску в капілярах клубочків

Зменшення кількості функціонуючих нефронів

Підвищення гідростатичного тиску в капсулі Шумлянського-Боумена

Збільшення онкотичного тиску крові

Пошкодження клубочкового фільтру

3126 / 6854
Хвора на ревматоїдний артрит після трьохтижневого лікування преднізолоном почала скаржитись на перебої в роботі серця. З чим пов’язаний розвиток даного небажаного ефекту препарату?

Гіпоглікемія

Гіперглікемія

Гіпокаліємія

Гіперкаліємія

Гіперурікемія

3127 / 6854
У хворого 45-ти років на тлі трансмурального інфаркту міокарда розвинулася гостра лівошлуночкова недостатність. Який лікарський засіб доцільно застосувати у даній ситуації для покращення помпової функції серця?

Еуфілін

Добутамін

Ефедрин

Промедол

Ізадрин

3128 / 6854
Для профілактики атеросклерозу, ішемічної хвороби серця, порушень мозкового кровообігу рекомендується споживання жирів із високим вмістом поліненасичених жирних кислот. Однією з таких жирних кислот є:

Стеаринова

Лінолева

Пальмітоолеїнова

Лауринова

Олеїнова

3129 / 6854
У хворого з нагноєнням рани при бактеріологічному дослідженні ранового вмісту виявлено грамнегативну паличку, яка на МПА утворює напівпрозорі слизові колонії синьо-зеленого кольору з перламутровим відтінком. Культура має специфічний запах фіалок або жасмину. Який вид збудника виділений з рани хворого?

S. faecalis

P vulgaris

S. pyogenes

P aeruginosa

S. aureus

3130 / 6854
Після обстеження пацієнта в клініці нервових хвороб встановлена відсутність звуження зіниці при дії світла. З ураженням яких структур головного мозку це пов’язано?

Ретикулярні ядра середнього мозку

Ретикулярш ядра довгастого мозку

Ядра гіпоталамуса

Вегетативні ядра 3 пари черепно-мозкових нервів

Червоні ядра середнього мозку

3131 / 6854
У хворого під час комп’ютерної томографії грудної клітки діагностовано пухлину заднього нижнього середостіння. Яка з перерахованих структур стиснута пухлиною?

Trachea

Arcus aortae

N. phrenicus

Aorta thoracica

Vena cava superior

3132 / 6854
Під час операції холецистектомії у хірурга виникла необхідність визначити топографію загальної жовчної протоки. Злиттям яких проток утворюється дана анатомічна структура?

Загальної печінкової і міхурової проток

Загальної печінкової і лівої печінкової проток

Лівої печінкової і міхурової проток

Загальної печінкової і правої печінкової проток

Правої та лівої печінкових проток

3133 / 6854
Дитина 6-ти років під час гри порізала ногу осколком скла і була направлена у поліклініку для введення протиправцевої сироватки. З метою попередження розвитку анафілактичного шоку лікувальну сироватку вводили за методом Безредка. Який механізм лежить в основі подібного способу гіпосенсибілізації організму?

Зв’язування фіксованих на тучних клітинах IgE

Стимуляція імунологічної толерантності до антигену

Зв’язування рецепторів до IgE на тучних клітинах

Стимуляція синтезу антиген-специфічних IgG

Блокування синтезу медіаторів у тучних клітинах

3134 / 6854
Хворому лікар призначив протикашльовий препарат центральної дії, який є алкалоїдом мачку жовтого. Діє на кашльовий центр вибірково, не пригнічуючи дихання, не затримує виділення харкотиння. Не викликає обстипації і лікарської залежності. Можна призначати дітям. Визначте препарат:

Бромгексин

Лібексин

Глауцину гідрохлорид

Кодеїну фосфат

Окселадин

3135 / 6854
Хлопчику 5-ти років був встановлений діагноз - міастенія. Оберіть препарат з групи антихолінестеразних засобів, який покращує нервово-м’язову передачу:

Армін

Прозерин

Алоксим

Іалантаміну гідробромід

Ацеклідин

3136 / 6854
Хворому 63-х років з атонією сечового міхура лікар призначив препарат, дозу якого хворий самостійно збільшив. З’явились підвищене потовиділення, салівація, діарея, м’язові спазми. Препарат якої групи був призначений?

Гангліоблокатори

Холіноміметики

Токолітики

Реактиватори холінестерази

Адреноблокатори

3137 / 6854
Під час розтину тіла жінки 28-ми років, яка померла від геморагічного шоку, виявлено: гемоперитонеум, права маткова труба збільшена у розмірах, багряна, з наскрізним дефектом стінки, її дилятований просвіт виповнений темно-червоними згортками крові. Гістологічно у слизовій оболонці труби та серед м’язових клітин визначаються пласти великих світлих децидуальних клітин, у м’язовій оболонці та серед згортків крові у просвіті труби - ворсинки хоріона. Діагностуйте патологію вагітності:

Порушена трубна вагітність

Порушена маткова вагітність

Деструюючий міхурцевий занесок

Черевна вагітність

Інтерлігаментарна вагітність

3138 / 6854
У біоптаті нирки 45-річного чоловіка, що має хронічну хворобу нирок, виявлено: склероз, лімфо-плазмоцитарна інфільтрація стінок мисок та чашок, дистрофія та атрофія канальців. Збережені канальці розширені, розтягнені колоїдоподібними масами, епітелій сплющений (’’щитоподібна” нирка). Який діагноз найбільш імовірний?

Тубуло-інтерстиційний нефрит

Гломерулонефрит

Гострий пієлонефрит

Нефросклероз

Хронічний пієлонефрит

3139 / 6854
Чоловік 40 років хворіє на гіперацидний гастрит з нічними голодними болями. Призначте хворому лікарський засіб - блокатор гістамінових H2-рецепторів III покоління, який знизить виділення хлористоводневої кислоти (особливо вночі) та збільшить утворення захисного слизу:

Атропіну сульфат

Пірензепін

Метацин

Платифіліну гідротартрат

Фамотидин

3140 / 6854
Під час футбольного матчу між вболівальниками різних команд виникла сутичка. На фоні негативних емоцій в одного учасника сутички були розширені зіниці й підвищене серцебиття. Активація якої системи регуляції функцій організму забезпечує такі вегетативні зміни при негативних емоціях?

Симпато-адреналова

Парасимпатична нервова

Метасимпатична нервова

Гіпоталамо-гіпофізарно-тиреоїдна

Соматична нервова

3141 / 6854
У хворого 32-х років після оперативного втручання розвинувся гнійний процес. Із гною рани виділено культуру S. aureus. Який з перерахованих тестів найдоцільніше використати для диференціації S. aureus від S. epidermidis?

Ферментація арабінози

Колір колонії

Оксидазний тест

Гемоліз на кров’яному агарі

Плазмокоагулазна активність

3142 / 6854
Чоловікові 58-ми років зроблено операцію з приводу раку простати. Через 3 місяці йому проведено курс променевої та хіміотерапії. До комплексу лікарських препаратів входив 5-фтордезоксиуридин - інгібітор тимідилатсинтази. Синтез якої речовини блокується цим препаратом?

р-РНК

і-РНК

ДНК

т-РНК

3143 / 6854
В підводному човні під час занурення порушилася система подачі кисню. У підводників збільшилися частота дихання і серцевих скорочень. Який вид гіпоксії розвинувся у підводників?

Кров’яна

Серцево-судинна

Тканинна

Гіпоксична

Дихальна

3144 / 6854
Синтез і-РНК проходить на матриці ДНК з урахуванням принципу комплементарності. Якщо триплети у ДНК наступні - АТГ-ЦГТ, то відповідні кодони і-РНК будуть:

УАЦ-ГЦА

АТГ-ЦГТ

АУГ-ЦГУ

УАГ-ЦГУ

ТАГ-УГУ

3145 / 6854
У процесі фібринолізу кров’яний тромб розсмоктується. Розщеплення нерозчинного фібрину відбувається шляхом його гідролізу під дією протеолітичного ферменту плазміну, який наявний у крові в неактивній формі плазміногену. Активується плазміноген шляхом обмеженого протеолізу за участю фермента:

Хімотрипсин

Ентерокіназа

Пепсин

Урокіназа

Трипсин

3146 / 6854
У біоптаті щитоподібної залози виявлено атрофію паренхіматозних елементів, дифузну інфільтрацію тканини залози лімфоцитами та плазматичними клітинами з утворенням в ній лімфоїдних фолікулів. Для якого захворювання є характерними наведені ознаки?

Ендемічний зоб

Аденома щитоподібної залози

Тиреоїдит Хасімото

Тиреоїдит Ріделя

Хвороба Базедова

3147 / 6854
У хворого з жовтяницею встановлено: підвищення у плазмі крові вмісту загального білірубіну за рахунок непрямого (вільного), в калі і сечі - високий вміст стеркобіліну, рівень прямого (зв’язаного) білірубіну у плазмі крові в межах норми. Який вид жовтяниці має місце у хворого?

Хвороба Жильбера

Гемолітична

Паренхіматозна

Механічна

3148 / 6854
При гістологічному дослідженні біоптатів, взятих з потовщених країв виразки шлунка, виявлені невеликі гніздові скупчення різко атипових гіперхромних невеликих епітеліальних клітин, які розташовані серед дуже розвиненої строми. Визначте пухлину:

Медулярний рак

Недиференційована саркома

Скіррозний недиференційований рак

Аденома

Аденокарцинома

3149 / 6854
Жінка 69-ти років довго хворіла на атеросклероз. Поступила в хірургічне відділення з симптомами гострого живота. При лапаротомії виявлені: тромбоз мезентеріальної артерії, петлі тонкої кишки набряклі, багряно-чорного кольору, на їх серозній оболонці фібринозні нашарування. Який патологічний процес розвинувся у кишці хворої?

Коагуляційний некроз

Немічний інфаркт

Волога гангрена

Суха гангрена

Секвестр

3150 / 6854
Під час ректороманоскопії хворого зі скаргами на діарею виявлено, що слизова оболонка прямої і сигмоподібної кишок різко гіперемована, набрякла, вкрита великою кількістю слизу, а у деяких ділянках вкрита плівчастими накладаннями зеленуватого кольору. Про яке захворювання можна думати?

Холера

Сальмонельоз

Амебіаз

Черевний тиф

Дизентерія

3151 / 6854
При копрологічному дослідженні у працівників кав’ярні лікарями санітарно- епідеміологічної станції були виявлені округлі цисти, характерною ознакою яких є наявність чотирьох ядер. Імовірніше за все у цих працівників безсимптомно паразитує:

Амеба кишкова

Дизентерійна амеба

Лямблія

Кишкова трихомонада

Балантидій

3152 / 6854
У хворого з варикозним розширенням вен під час огляду нижніх кінцівок відзначається: ціаноз, пастозність, зниження температури шкіри, поодинокі петехії. Який розлад гемодинаміки має місце у хворого?

Компресійна ішемія

Венозна гіперемія

Тромбоемболія

Обтураційна ішемія

Артеріальна гіперемія

3153 / 6854
При гістологічному дослідженні органів і тканин померлої від ниркової недостатності молодої жінки, у якої прижиттєво виявлявся високий титр антинуклеарних антитіл, виявлені поширені фібриноїдні зміни в стінках судин. Відмічається ядерна патологія з вакуолізацією ядер, каріорексисом, утворенням гематоксилінових тілець. Який найбільш імовірний діагноз?

Гіпертонічна хвороба

Системний червоний вовчак

Атеросклероз

Вузликовий періартеріїт

Облітеруючий ендартеріїт

3154 / 6854
Мікоплазми є своєрідною групою мікроорганізмів, що відносяться до родини Mycoplasmataceae і що мають властивості як бактерій, так і вірусів. Назвіть одну особливість мікоплазм, яка відрізняє їх від бактерій і вірусів:

Внутрішньоклітинний паразитизм

Спосіб розмноження

Відсутність клітинної будови

Відсутність клітинної стінки

Висока ферментативна активність

3155 / 6854
Після довготривалого вживання антибіотиків у хворого на слизовій ротової порожнини появилися округлі білі плями, на язику білий наліт. Який мікроорганізм імовірно спричинив дані симптоми?

Кишкова паличка

Лактобацили

Гриби роду Candida

Стрептокок

Ентерокок

3156 / 6854
Фенілкетонурія - це захворювання, яке зумовлено рецесивним геном, що локалізується в аутосомі. Батьки є гетерозиготами за цим геном. Вони вже мають двох хворих синів і одну здорову дочку. Яка імовірність, що четверта дитина, яку вони очікують народиться теж хворою?

0%

25%

100%

75%

50%

3157 / 6854
При диспансерному обстеженні у хворого знайдено цукор в сечі. Який найбільш імовірний механізм виявлених змін, якщо вміст цукру в крові нормальний?

Недостатня продукція інсуліну підшлунковою залозою

Порушення реабсорбції глюкози в канальцях нефрона

!нсулінорезистентність рецепторів клітин

Гіперпродукція глюкокортикоїдів наднирниками

Порушення фільтрації глюкози в клубочковому відділі нефрона

3158 / 6854
В пробірку, що містить розчин NaCl 0,9%, додана крапля крові. Що відбудеться з еритроцитами?

Набухання

Осмотичний гемоліз

Зморшкування

Залишаться без змін

Біологічний гемоліз

3159 / 6854
У хворого 40-ка років ознаки гірської хвороби: запаморочення, задишка, тахікардія, рН крові - 7,50, pCO2 - 30 мм рт.ст., зсув буферних основ +4 ммоль/л. Яке порушення кислотно-основного стану має місце?

Негазовий ацидоз

Газовий ацидоз

Негазовий алкалоз

Видільний ацидоз

Газовий алкалоз

3160 / 6854
У хворого 15-ти років концентрація глюкози натще - 4,8 ммоль/л, через годину після цукрового навантаження - 9,0 ммоль/л, через 2 години - 7,0 ммоль/л, через 3 години - 4,8 ммоль/л. Ці показники характерні для такого захворювання:

Цукровий діабет I типу

Прихований цукровий діабет

Цукровий діабет II типу

Хвороба !ценко-Кутінга.

3161 / 6854
Лікар призначив хворому з гострою серцевою недостатністю не-глікозидний кардіотонічний засіб, який безпосередньо стимулює в1-адренорецептори міокарда, що збільшує кровообіг, діурез. Застосовується лише внутрішньовенно крапельно внаслідок швидкої інактивації в організмі. Який препарат призначив лікар?

Добутамін

Корглікон

Адреналін

Дигоксин

Анаприлін

3162 / 6854
Чоловік 40-ка років перебував у пульмонологічному відділенні з приводу рецидивуючої правосторонньої пневмонії. Помер від легенево-серцевої недостатності. На розтині в правій легені визначається ділянка круглої форми 3х4 см. Вона являє собою порожнину з нерівними шорсткими краями, заповнену каламутною вершкоподібною жовто-зеленою рідиною. Мікроскопічно: стінка порожнини утворена тканиною легені з дифузною інфільтрацією лейкоцитами. Визначте патологічний процес у легені:

Гангрена

Емпієма

Хронічний абсцес

Гострий абсцес

!нфаркт

3163 / 6854
У вагітної жінки 24-х років після тривалого блювання було зареєстровано зниження об’єму циркулюючої крові. Про яку зміну загальної кількості крові може йти мова?

Поліцитемічна гіперволемія

Олігоцитемічна гіперволемія

Поліцитемічна гіповолемія

Проста гіповолемія

Олігоцитемічна гіповолемія

3164 / 6854
Пацієнту, який знаходився в клініці з приводу пневмонії, ускладненої плевритом, у складі комплексної терапії вводили преднізолон. Протизапальна дія цього синтетичного глюкокортикоїда пов’язана з блокуванням вивільнення арахідонової кислоти шляхом гальмування:

Фосфоліпази С

Пероксидази

Циклооксигенази

Фосфоліпази A2

Ліпоксигенази

3165 / 6854
У чоловіка 64-х років, який тривалий час курив і вживав міцні спиртні напої, на боковій поверхні язика виявили подібний на виразку утвір із білої, помірно щільної тканини розмірами 5х3 см. При гістологічному дослідженні біоптату виявили, що утвір побудований із клітини, які формують солідні структури і тяжі, що нагадують за будовою багатошаровий плоский епітелій, в якому клітини з вираженим поліморфізмом, з великими атиповими ядрами з патологічними мітозами. Діагностуйте виявлене у чоловіка захворювання:

Лейкоплакія

Плоскоклітинний зроговілий рак

Рак на місці

Еритроплакія

Плоскоклітинний незроговілий рак

3166 / 6854
Обстежуваний знаходиться у фазі повільнохвильового глибокого сну. Про це свідчить реєстрація на ЕЕГ таких хвиль:

Дельта-хвилі

Альфа-веретена

Тета -хвилі

Альфа-хвилі

Бета-хвилі

3167 / 6854
Лікар-стоматолог для лікування гінгівіту призначив пацієнту препарат з протипротозойною та антибактеріальною діями, який може викликати відразу до алкоголю. Вкажіть препарат, який призначив лікар:

Лінкоміцину гідрохлорид

Тетрациклін

Цефтріаксон

Метронідазол

Левоміцетин

3168 / 6854
Спеціальний режим харчування призвів до зменшення іонів Ca2+ в крові. До збільшення секреції якого гормону це призведе?

Тирокальцитонін

Паратгормон

Соматотропін

Вазопресин

Тироксин

3169 / 6854
У пацієнта після переохолодження на губах з’явились герпетичні висипання. Для лікування призначений крем ацикловіру, терапевтичний ефект якого пояснюється здатністю:

Порушувати функцію зворотної транскриптази

Гальмувати активність протеази

Уповільнювати реплікацію РНК- і ДНК-геномних вірусів

Інгібувати нейрамінідазу

Гальмувати активність ДНК-полімерази

3170 / 6854
На плановий прийом до педіатра батьки привели дитину віком 13 місяців. Під час повного огляду лікар перевірив розвиток II сигнальної системи дитини. Назвіть період, коли у людини вперше з’являються ознаки розвитку II сигнальної системи:

2-2,5 роки

6-12 місяців

3-5 років

1,5-2 роки

2,5-3 роки

3171 / 6854
У сироватці крові новонародженого виявлено антитіла до вірусу кору. Про наявність якого імунітету це може свідчити?

Природний пасивний

Штучний активний

Штучний пасивний

Спадковий, видовий

Природний активний

3172 / 6854
У хворого спостерігається пухлина тканин орбіти позаду очного яблука. Зазначено порушення акомодації та звуження зіниці ока. Яке анатомічне утворення ушкоджено?

Ganglion ciliare

N. lacrimalis

N. nasociliaris

N. trochlearis

N. opticus

3173 / 6854
У хворого виявлено зміну функції привушної слинної залози. Який з вузлів вегетативної нервової системи віддає післявузлові симпатичні волокна для неї?

Ganglion pterygopalatinum

Ganglion cervicothoracicum

Ganglion oticum

Ganglion cervicale superius

Ganglion submandibulare

3174 / 6854
Жінка 31-го року хворіє на В!Л- інфекцію на стадії СНІД. На шкірі нижніх кінцівок, слизової оболонки піднебіння з’явились рудувато-червоні плями, яскраво-червоні вузлики різних розмірів. Один з вузликів взято на гістологічне дослідження. Виявлено багато хаотично розташованих тонкостінних судин, вистелених ендотелієм, пучки веретеноподібних клітин з наявністю ге-мосидерину. Яка пухлина розвинулась у хворої?

Лімфома Беркіта

Лімфангіома

Фібросаркома

Гемангіома

Саркома Капоші

3175 / 6854
У альпініста, що піднявся на висоту 5200 м, розвинувся газовий алкалоз. Що є причиною його розвитку?

Гіпервентиляція легенів

Зниження температури навколишнього середовища

Гіповентиляція легенів

Гіпероксемія

Гіпоксемія

3176 / 6854
У хворого хронічна нежить. Набряк слизової оболонки носової порожнини призводить до порушення функції рецепторів нюхового нерва, які розташовані в нюховій ділянці носової порожнини. Через яке утворення волокна цього нерва потрапляють в передню черепну ямку?

Foramen incisivum

Lamina cribrosa os ethmoidale

Foramen sphenopalatinum

Foramen ethmoidale anterior

Foramen ethmoidale posterior

3177 / 6854
На гістологічному препараті в складі видовженої структури, обмеженої плазмолемою, по периферії розташовані численні ядра, а в цитоплазмі наявна поперечна посмугованість. Яка це структура?

Синцитіотрофобласт

Колагенове волокно

Міосимпласт

Кардіоміоцит

Гладкий міоцит

3178 / 6854
У спортсмена легкоатлета (бігуна на довгі дистанції) під час змагань розвинулась гостра серцева недостатність. В результаті чого виникла ця патологія?

Перевантаження серця об’ємом

Прямого пошкодження міокарда

Порушення вінцевого кровообігу

Патологія перикарда

Перевантаження серця опором

3179 / 6854
У пацієнта встановлено гіповітаміноз фолієвої кислоти, що може призвести до порушення синтезу:

Пуринових та тимідилових нуклеотидів

Цитрату та кетонових тіл

Тимідилових нуклеотидів та жирних кислот

Пуринових нуклеотидів та холестерину

Гема та креатину

3180 / 6854
Спадкова гіперліпопротеїнемія І типу обумовлена недостатністю ліпопротеїнліпази. Підвищення рівня яких транспортних форм ліпідів в плазмі навіть натщесерце є характерним?

Ліпопротеїни низької густини

Ліпопротеїни високої густини

Хіломікрони

Модифіковані ліпопротеїни

Ліпопротеїни дуже низької густини

3181 / 6854
У людини виявлена пухлина одного з відділів головного мозку, внаслідок чого в неї порушена здатність підтримувати нормальну температуру тіла. Яка структура головного мозку пошкоджена?

Гіпоталамус

Чорна субстанція

Таламус

Стріатум

Мозочок

3182 / 6854
До лікарні звернувся чоловік 50-ти років з розладами пам’яті, болісними відчуттями по ходу нервових стовбурів, зниженням інтелектуальних функцій, порушеннями з боку серцево-судинної системи і явищами диспепсії. В анамнезі хронічний алкоголізм. Дефіцит якого вітаміну може викликати ці симптоми?

Кальциферол

Рибофлавін

Тіамін

Ніацин

Ретинол

3183 / 6854
Пацієнт звернувся зі скаргами на гострий біль у правому підребер’ї. При огляді лікар звернув увагу на пожовтіння склер хворого. Лабораторно: підвищена активність АлАТ та негативна реакція на стеркобілін в калі. Для якого захворювання характерні такі симптоми?

Хронічний гастрит

Гепатит

Хронічний гастродуоденіт

Хронічний коліт

Гемолітична жовтяниця

3184 / 6854
У хворого після тривалого психоемоційного напруження спостерігається підвищення артеріального тиску, що супроводжується серцебиттям, кардіалгіями, головним болем, запамороченням. Домінуючим у формуванні артеріальної гіпертензії у даному випадку є збільшення:

Тонусу венул

Частоти серцевих скорочень

Об’єму циркулюючої крові

Тонусу артеріол

Серцевого викиду

3185 / 6854
Незалежно від расової чи етнічної належності у людини розвивається комплекс морфофункціональних, біохімічних, імунологічних ознак, які обумовлюють кращу біологічну пристосованість людини до відповідного фізичного середовища. Який тип біологічної реакції представлений у людини?

Гірський тип

Тип зони помірного клімату

Адаптивний тип

Арктичний тип

Тропічний тип

3186 / 6854
Для знеболювання використовують новокаїн, під дією якого нервове волокно втрачає здатність проводити збудження. Який мембранно-іонний механізм дії цього препарату?

Блокування кальцієвих іоноселектив-них каналів

Блокування калій-натрієвого насоса

Блокування натрієвих іоноселективних каналів

Блокування натрій-протонного насоса

Блокування калієвих іоноселективних каналів

3187 / 6854
У хворого 41-го року відзначається гіпонатріємія, гіперкаліємія, дегідратація, зниження артеріального тиску, м’язова слабкість, брадикардія, аритмія. З порушенням функцій яких гормонів це пов’язано?

Гормони мозкової речовини наднирників

Гормони підшлункової залози

Статеві гормони

Тиреоїдні

Кортикостероїди

3188 / 6854
При розтині хворої 28-ми років, що померла від уремії, виявлені збільшені строкаті нирки з осередками крововиливів. Патогістологічно в судинних клубочках виявлені гематоксилінові тільця, капілярні мембрани клубочків у вигляді дротяних петель, гіалінові тромби та осередки фібриноїдного некрозу. За патогенезом гіперчутливість якого типу лежить в основі описаної хвороби?

Гіперчутливість III типу (імунокомплексна)

Гіперчутливість V типу (гранулематоз)

Гіперчутливість II типу (антитілозалежна)

Гіперчутливість IV типу (клітинна цитотоксичність)

Гіперчутливість I типу (анафілактична)

3189 / 6854
Хвора 37-ми років померла під час нападу експіраторної задухи, що був спричинений контактом з екзогенним алергеном (пилок амброзії). При гістологічному дослідженні в просвіті бронхів спостерігаються скупчення слизу, в стінці бронхів багато тучних клітин (лаброцитів), більшість з яких у стані де-грануляції, багато еозинофілів. До патогенезуякого типу реакцій гіперчутливості можна віднести описані зміни?

I типу (анафілактична)

IV типу (клітинна цитотоксичність)

V типу (гранулематоз)

III типу (імунокомплексна)

II типу (антитілозалежна)

3190 / 6854
Оперуючи на наднирниковій залозі, хірург зупиняє кровотечу з артерії, що відходить до наднирникової залози від черевної аорти. Назвіть цю артерію:

A. phrenica inferior

A. uprarenalis inferior

A. suprarenalis media

A. renalis

A. suprarenalis superior

3191 / 6854
Хворому на паратиф А інфекціоніст на 3-му тижні захворювання призначив повторне бактеріологічне дослідження. Який матеріал слід взяти для виділення збудника?

Кров

Ліквор

Харкотиння

Випорожнення

Блювотні маси

3192 / 6854
Пацієнтка тривалий час приймала снодійний засіб нітразепам. Після відміни препарату у неї розвинулись безсоння, зниження апетиту, агресивність. Як називається такий стан?

Абстиненція

Кумуляція

Тахіфілаксія

Сенсибілізація

Ейфорія

3193 / 6854
Хворому на шизофренію призначено аміназин для купіювання психічного стану. Вкажіть механізм дії препарату:

Стимуляція серотонінових рецепторів

Блокада ГАМК-рецепторів

Стимуляція опіоїдних рецепторів

Блокада Д2-дофамінових рецепторів

3194 / 6854
Хворому на пневмонію призначений антибіотик із групи макролідів. Вкажіть цей препарат:

Тетрациклін

Ампіцилін

Азитроміцин

Стрептоміцин

Гентаміцин

3195 / 6854
В клініку доставлено чоловіка з травмою спини. Під час обстеження виявлено перелом хребців грудного відділу. Під час об’єктивного огляду нейрохірургом виявлено: нижче рівня перелому з правого боку відсутня глибока чутливість, з лівого боку – порушена температурна та тактильна чутливість. Яке ураження з боку спинного мозку є у хворого?

Судомний синдром

Парастезія

Анестезія

Хвороба Паркінсона

Синдром Броун Секара

3196 / 6854
Через 8 днів після хірургічної операції у пацієнта розвинувся правець. Лікар запідозрив, що причиною став контамінований збудником правця шовний матеріал, який був доставлений в бактеріологічну лабораторію. Яке живильне середовище необхідно використовувати для первинного посіву шовного матеріалу?

Ендо

ЖСА

Кітт-Тароцці

Гіса

Сабуро

3197 / 6854
Після проведеної операції на шиї хворий втратив чутливість в її передній ділянці. Яка гілка шийного сплетення була пошкоджена під час операції?

Малий потиличний нерв

Великий вушний нерв

Шийна петля

Надключичні нерви

Поперечний нерв шиї

3198 / 6854
У пацієнта перед кардіологічною операцією зареєстровано тиск у всіх відділах серця. Який тиск в лівому шлуночку під час діастоли?

0 мм рт.ст.

80 мм рт.ст.

120 мм рт.ст.

40 мм рт.ст.

100 мм рт.ст.

3199 / 6854
При обстеженні жінки виявили перелом однієї з кісток черепа, ускладнений кровотечею з поперечної пазухи твердої оболонки мозку. Назвіть кістку, на якій розташовується борозна цієї пазухи:

Тім’яна

Лобова

Потилична

Клиноподібна

Скронева

3200 / 6854
Батьки - глухонімі, але глухота у дружини залежить від аутосомно-рецесивного гена, а у чоловіка виникла внаслідок тривалого прийому антибіотиків у дитинстві. Яка імовірність народження глухої дитини в родині, якщо батько гомозиготний за аллелю нормального слуху?

75%

25%

10%

0%

100%

3201 / 6854
Тривале перебування в умовах спеки викликало у людини спрагу. Сигналізація від яких рецепторів, перш за все, зумовила її розвиток?

Глюкорецептори гіпоталамусу

Барорецептори дуги аорти

Осморецептори гіпоталамусу

Осморецептори печінки

Натрiєвi рецептори гіпоталамусу

3202 / 6854
Хворому з ревматоїдним артритом тривалий час вводили гідрокортизон. У нього з’явилися гіперглікемія, поліурія, глюкозурія, спрага. Ці ускладнення лікування є наслідком активації такого процесу:

Гліколіз

Глікогеноліз

Глюконеогенез

Ліполіз

Глікогенез

3203 / 6854
Для зняття марення і галюцинацій у хворої на шизофренію лікар використав аміназин. Який механізм антипсихотичної дії препарату?

Стимуляція холінергічних процесів в ЦНС

Блокада холінергічних процесів в ЦНС

Стимуляція адренергічних і дофамінергічних процесів в ЦНС

Блокада адренергічних і дофамінергічних процесів в ЦНС

Інгібування зворотнього нейронального захоплення МАО

3204 / 6854
При підозрі на туберкульоз хворій дитині зробили пробу Манту. Через 24 години у місці введення алергену з’явились припухлість, гіперемія і болісність. Які основні компоненти визначають цю реакцію організму?

Мононуклеари, Т-лімфоцити і лімфокіни

Гранулоцити, Т-лімфоцити і IgG

Плазматичні клітини, Т-лімфоцити і лімфокіни

Макрофаги, В-лімфоцити і моноцити

В-лімфоцити, IgM

3205 / 6854
Хворому перед операцією було введено дитилін (лістенон) і проведено інтубацію. Дефіцит якого ферменту в організмі хворого подовжує дію м’язового релаксанту?

K-Аа-АтФ-аза

Псевдохолінестераза

Сукцинатдегідрогеназа

Карбангідраза

N-ацетилтрансфераза

3206 / 6854
Хлопчик 12-ти років знаходиться у лікарні з підозрою на харчову токсикоінфекцію. При посіві фекалій хворого на середовище Ендо виросла велика кількість безбарвних колоній. Який мікроорганізм можна з найбільшою імовірністю ВИКЛЮЧИТИ з числа можливих збудників захворювання?

Proteus vulgaris

Pseudomonas aeruginosa

Yersinia enterocolitica

Salmonella enteritidis

Escherichia coli

3207 / 6854
У хворого внаслідок хронічного захворювання органів дихання, що супроводжується задишкою, тахікардією і ціанозом, при дослідженні газового складу крові виявлений розвиток гіпоксемії і гіперкапнії. Внаслідок якого з порушень зовнішнього дихання виникли ці зміни?

Гіперперфузія

Гіпервентиляція

Гіпоперфузія

Гіповентиляція

Гіпердифузія

3208 / 6854
У тварини через 2 тижні після експериментального звуження ниркової артерії підвищився артеріальний тиск. З посиленням дії на судини якого чинника гуморальної регуляції це пов’язане?

Кортизол

Ангіотензин II

Вазопресин

Альдостерон

Дофамін

3209 / 6854
У чоловіка 43-х років, що помер у хірургічному відділенні при явищах розлитого гнійного перитоніту, на розтині у дистальному відділі тонкого кишечника виявлені пейєрові бляшки, що виступають у просвіт, поверхня деяких з них вкрита струпом, коричнево-зеленуватого кольору, у центрі деяких бляшок наявні глибокі дефекти, що сягають серозного шару. Для якого захворювання характерні зміни, виявлені у тонкому кишечнику?

Хвороба Крона

Туберкульоз кишечника

Стафілококовий ентерит

Дизентерія

Черевний тиф

3210 / 6854
У хворого виявлено ожиріння, гірсутизм, «місяцеподібне» обличчя, рубці багряного кольору на шкірі стегон. Артеріальний тиск -180/110 мм рт. ст., глюкоза крові - 17,2ммоль/л. При якій зміні продукції гормонів наднирників можлива така картина?

Гіпопродукція глюкокортикоїдів

Гіперпродукція глюкокортикоїдів

Гіпопродукція адреналіну

Гіперпродукція мінералокортикоїдів

Гіпопродукція мінералокортикоїдів

3211 / 6854
У хворого, що тривало приймав глюкокортикоїди, в результаті відміни препарату виникло загострення наявного захворювання, зниження артеріального тиску, слабкість. Ці явища можна пов’язати з розвитком:

Гіперпродукцією АКТГ

Звикання до препарату

Недостатності наднирників

Кумуляцією

Сенсибілізацією

3212 / 6854
Тварині, сенсибілізованій туберкуліном, внутрішньоочеревинно введений туберкулін. Через 24 години при лапаротомії виявлено венозну гіперемію та набряк очеревини. Умазках-відбитках з очеревини велика кількість лімфоцитів та моноцитів. Яке запалення має місце у тварини?

Фібринозне

Алергічне

Гнійне

Асептичне

Серозне

3213 / 6854
У хворого спостерігаються біль голови , висока температура, озноб, кашель. З харкотиння виділили палички овоїдної форми з біполярним забарвленням, грам-негативні, у мазку з бульйонної культури розташовані ланцюжками, на агарі утворюються колонії R-форми. Це характерне для збудника такого захворювання:

Туберкульоз

Дифтерія

Чума

Менінгококовий назофарингіт

Стрептококова ангіна

3214 / 6854
До лікарні потрапили пацієнти зі скаргами: слабкість, болі в кишечнику, розлади травлення. Після дослідження фекалій були виявлені кулясті цисти з чотирма ядрами. Для якого найпростішого характерні такі цисти?

Балантидій

Амеба ротова

Амеба кишкова

Кишкова трихомонада

Амеба дизентерійна

3215 / 6854
У доношеного новонародженого спостерігається жовте забарвлення шкіри та слизових оболонок. Імовірною причиною цього стану може бути тимчасова нестача такого ферменту:

Гемсинтетаза

Гемоксигеназа

Уридинтрансфераза

Білівердинредуктаза

УДФ-глюкуронілтрансфераза

3216 / 6854
У немовляти спостерігаються епілептиформні судоми, викликані дефіцитом вітаміну В6. Це спричинено зменшенням у нервовій тканині гальмівного медіатора - гамма-аміномасляної кислоти. Активність якого ферменту знижена:

Глутаматдегідрогеназа

Глутаматсинтетаза

Аланінамінотрансфераза

Піридоксалькіназа

Глутаматдекарбоксилаза

3217 / 6854
У молодої особи впродовж року прогресувала ниркова недостатність зі смертельним наслідком. При розтині тіла виявлені великі строкаті нирки з червоними дрібними краплинами в жовто-сірому корковому шарі. Гістологічно в клубочках виявлені «півмісяці» з проліферуючого нефротелія. Який найбільш імовірний діагноз?

Гнійний нефрит

Рак нирки

Швидкопрогресуючий гломерулонефрит

Крововилив в нирки

Амілоїдоз

3218 / 6854
У хворого відсутній зір, але зіничний рефлекс реалізується нормально. Де може знаходитись зона пошкодження?

Зорова кора

Зоровий перехрест

Нижні горбики чотиригорбкового тіла

Верхні горбики чотиригорбкового тіла

Соматосенсорна кора

3219 / 6854
Хворий з 30-річним стажем роботи у шахті помер від легенево-серцевої недостатності, що наростала. На розтині легені збільшені у розмірах, щільної консистенції, на розрізі у них велика кількість вузликів розмірами з просяні зерна і більше, щільних, сіруватого і сірувато-чорного кольору, місцями вузлики зливаються у більш великі ділянки. Назвіть найбільш імовірне захворювання:

Антрако-силікоз

Бериліоз

Сидероз

Азбестоз

Алюміноз

3220 / 6854
Для серодіагностики черевного тифу ставлять реакцію, при якій до різних розведень сироватки хворого добавляють дiагностикуми трьох видів мікроорганізмів і результат якої оцінюють за наявності осаду із склеєних бактерій. Ця реакція відома під назвою:

Райта

Асколі

Вассермана

Відаля

Борде-Жангу

3221 / 6854
У хворого напад тахікардії. Які мембранні циторецептори кардіоміоцитів доцільно заблокувати, щоб припинити напад?

Альфа-адренорецептори

Н-холінорецептори

М- та Н-холінорецептори

М-холінорецептори

Бета-адренорецептори

3222 / 6854
У хворого на ЕКГ виявлено, що тривалість інтервалу RR дорівнює 1,5 с, частота серцевих скорочень - 40/хв. Що є водієм ритму серця?

Синусовий вузол

Ліва ножка Гіса

Атріовентрикулярний вузол

Права ножка Гіса

Пучок Гіса

3223 / 6854
У хворого за лабораторними даними визначили аскаридоз. Який засіб треба призначити?

Піперазин

Гентаміцин

Фенасал

Фуразолідон

Мебендазол

3224 / 6854
Хворий не може відвести від тулуба верхню кінцівку. Який м’яз НЕ ВИКОНУЄ свою функцію?

Малий коловий м’яз

Підосний м’яз

Найширший м’яз спини

Великий коловий м’яз

Дельтоподібний м’яз

3225 / 6854
Пункцію сечового міхура через передню черевну стінку, не займаючи очеревину, можливо виконати:

Тільки у жінок

При пустому сечовому міхурі

Тільки у дітей

При наповненому сечовому міхурі

Тільки у чоловіків

3226 / 6854
В судово-медичній експертизі широко використовується метод дактилоскопії, який оснований на тому, що сосочковий шар дерми визначає строго індивідуальний малюнок на поверхні шкіри. Яка тканина утворює цей шар дерми?

Щільна оформлена сполучна тканина

Ретикулярна тканина

Жирова тканина

Щільна неоформлена сполучна тканина

Пухка волокниста неоформлена сполучна тканина

3227 / 6854
Важливою складовою частиною ниркового фільтраційного бар’єру є тришарова базальна мембрана, яка має спеціальну сітчасту будову її середнього електроннощільного шару. Де міститься ця базальна мембрана?

Проксимальні канальці

Капіляри перитубулярної капілярної сітки

Ниркове тільце

Тонкі канальці

Дистальні прямі канальці

3228 / 6854
Чоловік 42-х років помер при явищах вираженої інтоксикації і дихальної недостатності. На розтині: тканина легень у всіх відділах строката, з множинними дрібновогнищевими крововиливами та вогнищами емфіземи. Гістологічно у легенях: геморагічна бронхопневмонія з абсцедуванням, у цитоплазмі клітин епітелію бронхів еозинофільні і базофільні включення. Діагностуйте виявлене на секції захворювання:

Стафілококова бронхопневмонія

Крупозна пневмонія

Грип

Часткова пневмонія

Плевропневмонія

3229 / 6854
В експерименті подразнюють скелетний м’яз серією електричних імпульсів. Який вид м’язового скорочення виникне, якщо кожний наступний імпульс припадає на період вкорочення поодинокого м’язового скорочення?

Асинхронний тетанус

Суцільний тетанус

Контрактура м’яза

Зубчастий тетанус

Серія поодиноких скорочень

3230 / 6854
У хворого, який тривалий час страждав на переміжну кульгавість, тканини пальців стопи сухі, чорного кольору, нагадують мумію. На невеликій відстані від почорнілої ділянки розташована двоколірна лінія (червоний колір прилягає до практично незмінених тканин, а біло-жовтий колір - до змінених тканин). Який вид некрозу у даного хворого?

Гангрена

Секвестр

Пролежень

Мацерація

!нфаркт

3231 / 6854
У пацієнта, що звернувся до лікаря, спостерігається жовте забарвлення шкіри, сеча - темна, кал темно-жовтого кольору. Підвищення концентрації якої речовини буде спостерігатися в сироватці крові?

Вільний білірубін

Кон’югований білірубін

Вердоглобін

Мезобілірубін

Білівердин

3232 / 6854
До дерматолога звернулася пацієнтка із скаргами на екзематозне ураження шкіри рук, що з’являється після контакту з миючим засобом «Лотос» Використання гумових рукавичок запобігає цьому. Патологічна реакція шкіри зумовлена активацією:

Моноцитів

Базофілів

В -лімфоцитів

Т-лімфоцитів

Нейтрофілів

3233 / 6854
У хворого на тимому (пухлину вилочкової залози) спостерігається ціаноз, розширення підшкірної венозної сітки і набряк м’яких тканин обличчя, шиї, верхньої половини тулуба і верхніх кінцівок. Який венозний стовбур перетиснено пухлиною?

Підключична вена

Внутрішня яремна вена

Передня яремна вена

Верхня порожниста вена

Зовнішня яремна вена

3234 / 6854
В результаті травми у чоловіка 47-ми років пошкоджені передні корінці спинного мозку. Відростки яких нейронів пошкоджені?

Дендрити і аксони чутливих псевдоуні-полярних

Дендрити рухових і аксони ядер бокових стовпів

Аксони чутливих псевдоуніполярних

Дендрити чутливих псевдоуніполярних

Аксони нейронів рухових соматичних та вегетативних ядер

3235 / 6854
Під час розтину померлого 43-х років, що страждав на ІХС з розвитком інфаркту міокарда, патологоанатом виявив набряк легень. Які патологічні зміни могли зумовити набряк легень?

Гостра правошлуночкова недостатність

Ішемія малого кола

Стаз крові

Гостра лівошлуночкова недостатність

Гостре загальне малокрів’я

3236 / 6854
У чоловіка 40-ка років ушкоджені міжпальцеві проміжки на ногах: шкіра мокне, відшаровується, з’явилися тріщини. При посіві зі шкрібу шкіри на середовище Сабуро виросли пухнасті колонії, білі зверху та зеленувато-жовті внизу. У мазках з верхньої частини колоній видно конідії у вигляді «дубинок» з 1-5 клітинами. Які ще органи найбільш імовірно може уразити цей збудник?

Слизова статевих шляхів

Волосся

Нігті

Лімфатичні судини

Підшкірна клітковина

3237 / 6854
При ендоскопічному дослідженні у хворого з хронічним ентероколітом (запалення кишки) спостерігається відсутність специфічних структур рельєфа тонкої кишки. Які компоненти визначають особливості рельєфа слизової оболонки цього органу?

Косо-спіральні складки

Циркулярні складки, ворсинки та крипти

Поля, складки, ямки

Поля, ворсинки

Гаустри, ворсинки, крипти

3238 / 6854
Під час операції на головному мозку відмічено, що подразнення певних зон кори великих півкуль викликало у хворого і тактильні і температурні відчуття. На яку звивину діяли подразники?

Прецентральна

Постцентральна

Поясна

Парагіпокампова

Верхня латеральна

3239 / 6854
Хворому проведено субтотальну субфасціальну резекцію щитоподібної залози. У післяопераційному періоді тривалий час зберігається охриплість голосу. Який нерв ушкоджено в ході операції?

Під’язиковий нерв

Нижньощелепний нерв

Язиковий нерв

Верхній гортанний нерв

Зворотній гортанний нерв

3240 / 6854
В результат виснажуючої м’язової роботи у робочого значно зменшилась буферна ємність крові. Надходженням якої речовини у кров можна пояснити це явище?

Піруват

Лактат

1,3-бісфосфогліцерат

Альфа-кетоглутарат

3-фосфоглицерат

3241 / 6854
В біоптаті бронха хворого, який зловживає палінням, в потовщеній слизовій оболонці виявлено хронічний обструктивний бронхіт з ознаками трансформації одношарового війчастого епітелія в багатошаровий плоский епітелій. Який з процесів найбільш імовірний?

Лейкоплакія

Плоскоклітинний рак

Метаплазія

Гіперплазія епітелію

Гіпертрофія епітелію

3242 / 6854
У хворого 45-ти років при аналізі ЕКГ встановлено: ритм синусовий, число передсердних комплексів більше числа шлуночкових комплексів; прогресуюче подовження інтервалу P-Q від комплексу до комплексу; випадіння окремих шлуночкових комплексів; зубці P та комплекси QRST без змін. Назвіть тип порушення серцевого ритму:

Повна атріовентрикулярна блокада

Атріовентрикулярна блокада I ступеня

Внутрішньопередсердна блокада

Атріовентрикулярна блокада II ступеня

Синоаурікулярна блокада

3243 / 6854
Хворий з діагнозом цукровий діабет, вранці натще отримав призначену дозу інсуліну пролонгованої дії. Пропустив черговий прийом їжі і невдовзі відчув слабкість, біль голови ,запаморочення, пітливість, тремтіння тіла, судоми, відчуття голоду, явища гіпоглікемії. Застосування глюкози стан не полегшило. Який препарат необхідно ввести для купірування даного стану?

Норадреналін

Адреналін

Гідрокортизон

Пренізолон

Тріамцінолон

3244 / 6854
У чоловіка 25-ти років з переломом основи черепа виділяється великий об’єм сечі з низькою відносною щільністю. Причиною змін сечоутворення є порушення синтезу такого гормону:

Окситоцин

Соматотропний гормон

Тиреотропний гормон

Вазопресин

Адренокортикотропний гормон

3245 / 6854
Розпочинається імплантація бластоцисти людини. Як називається період ембріогенезу, що розпочинається одночасно з імплантацією?

Диференціювання

Дроблення

Інвагінація

Гаструляція

Гістогенез

3246 / 6854
Дитина 8-ми років готувалася до тонзилектомії. Аналіз крові показав, що час згортання збільшено (до 7 хвилин). Який препарат за 5 днів до операції слід включити до комплексу лікарських засобів підготовчого періоду в першу чергу?

Вікасол

Дицинон

Амінокапронова кислота

Фібриноген

Кальцію хлорид

3247 / 6854
У дитини, що страждає на пілоростеноз, що супроводжується частим блюванням, розвинулись ознаки зневоднення організму. Яка форма порушення кислотно-основного стану може розвиватися у даному випадку?

Метаболічний ацидоз

Негазовий алкалоз

Газовий алкалоз

Негазовий ацидоз

Газовий ацидоз

3248 / 6854
У хворого після автомобільної травми артеріальний тиск - 70/40 мм рт. ст. Хворий у непритомному стані. За добу виділяє близько 550 мл сечі. Яке порушення функції нирок спостерігається у хворого?

Хронічна ниркова недостатність

Гострий дифузний гломерулонефрит

Пієлонефрит

Тубулопатія

Гостра ниркова недостатність

3249 / 6854
У гістопрепараті яєчника жінки визначаються структури, що мають велику порожнину. Овоцит І порядку в них оточений прозорою оболонкою, променистим вінцем і розташований у яйценосному горбику, стінка утворена шаром фолікулярних клітин i текою. Вкажіть, якій структурі яєчника належать дані морфологічні ознаки:

Зрілий (третинний) фолікул

Примордіальний фолікул

Атретичне тіло

Жовте тіло

Первинний фолікул

3250 / 6854
Хворий переніс повторний інтрамуральний інфаркт міокарда. Після лікування та реабілітації виписаний у задовільному стані під нагляд дільничного терапевта. Через 2роки загинув у автомобільній катастрофі. Який характер патологічного процесу в міокарді було встановлено на розтині?

Крупновогнищевий кардіосклероз

Некроз

Гіперплазія

Дрібновогнищевий кардіосклероз

Атрофія

3251 / 6854
При бактеріологічному дослідженні гною з післяопераційної рани виділені мікроорганізми, які дали ріст на цукрово-кров’яному агарі через 7-10 днів в анаеробних умовах: колонії S-форми, блискучі, чорного кольору з неприємним запахом. При мікроскопії виявлені поліморфні грамнегативні палички. Які мікроорганізми могли викликати цей нагнійний процес?

Бактероїди

Кишкова паличка

Клостридії

Велонели

Фузобактерії

3252 / 6854
У 12-річного хлопчика в сечі виявлено високий вміст усіх амінокислот аліфатичного ряду. При цьому відмічена найбільш висока екскреція цистину та цистеїну. Крім того, УЗД нирок показало наявність каменів у них. Виберіть можливу патологію:

Алкаптонурія

Хвороба Хартнупа

Фенілкетонурія

Цистит

Цистинурія

3253 / 6854
Цикл Кребса відіграє важливу роль у реалізації глюкопластичного ефекту амінокислот. Це зумовлено обов’язковим перетворенням безазотистого залишку амінокислот у:

Сукцинат

Оксалоацетат

Малат

Фумарат

Цитрат

3254 / 6854
Після прийому препарату у хворого з серцевою недостатністю зменшилася частота скорочень серця, пульс став кращого наповнення, зменшилися набряки, збільшився діурез. Вкажіть, який препарат приймав хворий:

Дилтіазем

Анаприлін

Верапаміл

Резерпін

Дигоксин

3255 / 6854
У пацієнта 65-ти років з тривалими скаргами, характерними для хронічного гастриту, у периферичній крові виявлені мегалоцити, у кістковому мозку мегалобластичний еритропоез. Який найбільш імовірний діагноз?

Гіпопластична анемія

Апластична анемія

Залізодефіцитна анемія

B12-фолієводефіцитна анемія

Гемолітична анемія

3256 / 6854
З калу та блювотних мас від хворого з підозрою на холеру були виділені культури вібріонів. Проведення якої реакції дозволить визначити вид мікроба, що викликав це захворювання?

Преципітації

Аглютинації з сироватками, що містять Н-антитіла

Аглютинації з сироватками, що містять О-антитіла

Пасивної гемаглютинації з еритроци-тарним антигенним діагностикумом

Аглютинації Відаля

3257 / 6854
У дитини на слизовій оболонці щік та язика виявлені білуваті плями, які нагадують молоко, що скипілося. У виготовлених препаратах-мазках знайдені грампозитивні овальні дріжджоподібні клітини. Які це збудники?

Гриби роду Кандіда

Актиноміцети

Дифтерійна паличка

Стафілококи

Фузобактерії

3258 / 6854
При обстеженні юнака з розумовою відсталістю виявлено євнухоїдну будову тіла, недорозвиненість статевих органів. В клітинах порожнини рота - статевий хроматин. Який метод генетичного дослідження слід застосувати для уточнення діагнозу?

Клініко-генеалогічний

Популяційно-статистичний

Біохімічний

Цитологічний

Дерматогліфіка

3259 / 6854
У хворого з жовтяницею встановлено: підвищення у плазмі крові вмісту загального білірубіну за рахунок непрямого (вільного), в калі та сечі - високий вміст стеркобіліну, рівень прямого (зв’язаного) білірубіну в плазмі крові в межах норми. Про який вид жовтяниці можна думати?

Гемолітична

Хвороба Жильбера

Жовтяниця немовлят

Механічна

Паренхіматозна (печінкова)

3260 / 6854
Кухар в результаті необачності обпік руку парою. Підвищення концентрації якої речовини викликало почервоніння, набряклість та болючість ураженої ділянки шкіри?

Гістамін

Глутамін

Галактозамін

Лізин

Тіамін

3261 / 6854
Дитина доставлена в санпропускник в стані асфіксії. При огляді в гортані виявлені білуваті плівки, що обтурують просвіт та легко видаляються. Лікар запідозрив дифтерію. Про яке запалення гортані йдеться?

Серозне

Дифтеритичне

Катаральне

Гнійне

Крупозне

3262 / 6854
У хворого із запаленням легень спостерігається підвищення температури тіла. Яка біологічно активна речовина відіграє провідну роль у виникненні цього прояву хвороби?

Серотонін

Інтерлейкін-I

Гістамін

Брадикінін

Лейкотрієни

3263 / 6854
Хворий 12-ти років поступив в клініку з гемартрозом колінного суглоба, з раннього дитинства страждає кровоточивістю. Яка хвороба у хлопчика?

Геморагічний васкуліт

Гемофілія

Гемолітична анемія

Тромбоцитопенічна пурпура

B12 фолієво-дефіцитна анемія

3264 / 6854
У хворого видалено 12-палу кишку. Це призведе до зменшення секреції, перш за все, такого гормону:

Соматостатин

Нейротензин

Гістамін

Гастрин

Холецистокінін-секретин

3265 / 6854
В клітині, яка мітотично ділиться, спостерігається розходження дочірніх хроматид до полюсів клітини. На якій стадії мітотичного циклу знаходиться клітина?

Анафаза

Телофаза

Інтерфаза

Метафаза

Профаза

3266 / 6854
Знешкодження ксенобіотиків (лікарських засобів, епоксидів, ареноксидів, альдегідів, нітропохідних тощо) та ендогенних метаболітів (естрадіолу, простагландинів, лейкотрієнів) відбувається в печінці шляхом їх кон’югації з:

Аспарагіновою кислотою

Гліцином

S-Аденозилметіоніном

Фосфоаденозином

Глутатіоном

3267 / 6854
Вітамін A у комплексі зі специфічними циторецепторами проникає через ядерні мембрани, індукує процеси транскрипції, що стимулює ріст та диференціювання клітин. Ця біологічна функція реалізується наступною формою вітаміну A:

Транс -ретиналь

Транс-ретиноєва кислота

Цис-ретиналь

Каротин

Ретинол

3268 / 6854
Одна з форм вродженої патології супроводжується гальмуванням перетворення фенілаланіну в тирозин. Біохімічною ознакою хвороби є накопичення в організмі деяких органічних кислот, у тому числі такої кислоти:

Лимонна

Ілутамінова

Піровиноградна

Молочна

Фенілпіровиноградна

3269 / 6854
Хворому встановлено діагноз - активний вогнищевий туберкульоз легень. Вкажіть, який із препаратів йому найбільш доцільно призначити в першу чергу

Етіонамід

Етоксид

Ізоніазид

Сульфален

Циклосерин

3270 / 6854
В експерименті на спинному мозку при збудженні альфа-мотонейронів згиначів встановлено гальмування альфа-мотонейронів м’язів-розгиначів. Який вид гальмування лежить в основі цього явища?

Латеральне

Пресинаптичне

Зворотнє

Деполяризаційне

Реципрокне

3271 / 6854
У хворого, що страждає на важку форму порушення водно-сольового обміну, настала зупинка серця у діастолі. Який найбільш імовірний механізм зупинки серця у діастолі?

Гіперкаліємія

Гіпонатріємія

Гіпокаліємія

Гіпернатріємія

Дегідратація організма

3272 / 6854
Встановлено ураження ВІЛ Т-лімфоцитів. При цьому фермент вірусу зворотня транскриптаза (РНК-залежна ДНК-полімераза) каталізує синтез:

і-РНК на матриці вірусного білку

ДНК на матриці вірусної і-РНК

Вірусної РНК на матриці ДНК

Вірусної і-РНК на матриці ДНК

ДНК на вірусній р-РНК

3273 / 6854
Людина зробила спокійний видих. Як називається об’єм повітря, який міститься у неї в легенях при цьому?

Залишковий об’єм

Резервний об’єм видиху

Життєва ємність легень

Дихальний об’єм

Функціональна залишкова ємкість легень

3274 / 6854
Людина зробила максимально глибокий видих. Як називається об’єм повітря, що знаходиться в її легенях після цього?

Резервний об’єм видиху

Альвеолярний об’єм

Ємність вдиху

Функціональна залишкова ємність легень

Залишковий об’єм

3275 / 6854
У людини необхідно оцінити стан клапанів серця. Яким з інструментальних методів дослідження доцільно скористатися для цього?

Електрокардіографія

Сфігмографія

Флебографія

Фонокардіографія

Зондування судин

3276 / 6854
Чоловіку 40-ка років за вимогою діагностичних тестів зробили лімфографію органів грудної порожнини. Хірург встановив, що пухлина вразила орган, з лімфатичних судин якого лімфа безпосередньо переходить в грудну протоку. Який це орган?

Стравохід

Осердя

Серце

Трахея

Лівий головний бронх

3277 / 6854
При гістологічному дослідженні біоптату шкіри виявлені гранульоми, які складаються з макрофагальних вузликів з наявністю лімфоцитів та плазматичних клітин. Крім того, зустрічаються великі макрофаги з жировими вакуолями, які містять запакованих у вигляді куль збудників захворювання (клітини Вірхова). Грануляційна тканина добре васкуляризована. Для якого захворювання характерна описана гранульома?

Лепра

Сифіліс

Риносклерома

Туберкульоз

Сап

3278 / 6854
При мікроскопічному дослідженні біоптату шкіри виявляються гранульоми, які складаються з епітеліоїдних клітин, оточених в основному Т-лімфоцитами. Серед епітеліоїдних клітин розташовуються поодинокі гігантські багатоядерні клітини типу Пирогова-Лангханса. В центрі гранульом виявляються ділянки казеозного некрозу. Кровоносні судини відсутні. Для якого захворювання характерні описані гранульоми?

Сап

Сифіліс

Туберкульоз

Лепра

Риносклерома

3279 / 6854
Ліквідатору наслідків аварії на Чорнобильські АЕС, що отримав велику дозу опромінення, проведено трансплантацію кісткового мозку. Через деякий час після проведеної операції у пацієнта діагностовано розвиток реакції трансплантат проти хазяїна. Які антигени послужили пусковим механізмом виникнення цієї реакції?

Антигени HBs, HBc, Hbe

Антигенами системи ABO еритроцитів ліквідатора

Антигени системи HLA клітин організму донора

Антигени системи Rh еритроцитів ліквідатора

Антигени системи HLA клітин організму ліквідатора

3280 / 6854
Під час гістологічного дослідження стулок мітрального клапана серця жінки 30-ти років було встановлено, що ендотеліальні клітини вогнищево десквамовані, в цих ділянках на поверхні стулки розташовані дрібні тромботичні нашарування, сполучна тканина стулки з явищами мукоїдного набухання з ділянками склерозу та васкуляризації. Діагностуйте вид клапанного ендокардиту:

Фібропластичний

Дифузний

Поворотньо-бородавчастий

Поліпозно-виразковий

Гострий бородавчастий

3281 / 6854
При органічних пошкодженнях головного мозку пам’ять може покращити:

Пірацетам

Діазепам

Нітразепам

Мезапам

Кофеїн

3282 / 6854
У хворої під час травми стався розрив лобкового симфізу. Який тип з’єднання постраждав?

Іеміартроз

Синхондроз

Синдесмоз

Синостоз

Діартроз

3283 / 6854
У хворого діагностовано інфаркт задньої частини міжшлуночкової перегородки. В ділянці якої кровоносної судини виникло порушення кровообігу?

R. marginalis dexter

R. atrialis intermedius

R. marginalis sinister

R. interventricularis posterior

R. circumflexus

3284 / 6854
У дитячому відділенні інфекційної клініки хлопчику поставлено діагноз «дифтерія» Який препарат потрібно ввести хворому в першу чергу?

Протидифтерійна антитоксична сироватка

АДП

TABte

АКДП

Дифтерійний анатоксин

3285 / 6854
У студента перед екзаменом виникла тахікардія. Які зміни на ЕКГ будуть свідчити про її наявність?

Розширення комплексу QRS

Подовження інтервалу R-R

Подовження інтервалу P-Q

Подовження сегменту Q-T

Укорочення інтервалу R-R

3286 / 6854
При розтині тіла чоловіка, померлого від опікової хвороби, знайдено набряк головного мозку, збільшення печінки, а також нирок, кірковий шар яких широкий, блідо-сірий, мозковий - повнокровний. Мікроскопічно: некроз епітелію канальців головних відділів з деструкцією базальних мембран, набряк інтерстицію з лейкоцитарною інфільтрацією та крововиливами. Який з перелічених діагнозів найбільш імовірний?

Пієлонефрит

Мієломна нирка

Тубулоінтерстиціальний нефрит

Некротичний нефроз

Подагрична нирка

3287 / 6854
У хворої 59-ти років з нейроциркуляторною (первинною) артеріальною гіпотензією купіровано гіпотонічний криз підшкірним введенням 1 мл 20% розчину кофеїну бензоатунатрію. Механізм дії цього препарату полягає у тому, що він блокує:

Пуринові рецептори

Альфа-адренорецептори

Моноаміноксідазу

Аденозинові рецептори

Бета-адренорецептори

3288 / 6854
Хворий похилого віку страждає на хронічний закреп, в основі якого лежить гіпотонія товстого кишечнику. Який препарат слід призначити хворому?

Бісакодил

Ацеклідин

Прозерин

Касторова олія

Натрію сульфат

3289 / 6854
У жінки через 6 місяців після пологів розвинулася маткова кровотеча. При гінекологічному огляді у порожнині матки виявлена тканина темно-червоного кольору змножинними порожнинами, що нагадує «губку» При мікроскопічному дослідженні пухлини у лакунах крові виявлені атипові світлі епітеліальні клітини Лангханса і гігантські клітини синцитіотрофобласту. Назвіть пухлину:

Міхурний занесок

Хоріонепітеліома

Фіброміома

Плоскоклітинний незроговілий рак

Аденокарцінома

3290 / 6854
У хворого спостерігається гемералопія (куряча сліпота). Яка з перерахованих речовин володітиме лікувальною дією?

Карнозин

Каротин

Карнітин

Креатин

Кератин

3291 / 6854
Хворий 18-ти років звернувся до лікарні із скаргами на шум та больові відчуття у вусі. Об’єктивно: у хворого гостре респіраторне захворювання, риніт. Крізь який отвір глотки інфекція потрапила до барабанної порожнини та викликала її запалення?

Вхід до гортані

Глотковий отвір слухової труби

Зів

Хоани

Барабанний отвір слухової труби

3292 / 6854
Хворий 50-ти років страждає на гіпертонічну хворобу. Під час фізичного навантаження у нього з’явилося відчуття слабкості, нестачі повітря, синюшність слизової оболонки губ, шкіри обличчя. Дихання супроводжувалося відчутними на відстані вологими хрипами. Який механізм лежить в основі виникнення такого синдрому?

Хронічна правошлуночкова недостатність

Хронічна лівошлуночкова недостатність

Колапс

Гостра лівошлуночкова недостатність

Тампонада серця

3293 / 6854
Внаслідок тривалого голодування в організмі людини швидко зникають резерви вуглеводів. Який з процесів метаболізму за цих умов поновлює вміст глюкози в крові?

Анаеробний гліколіз

Пентозофосфатний шлях

Глюконеогенез

Аеробний гліколіз

Глікогеноліз

3294 / 6854
У чоловіка 40-ка років було встановлено діагноз: серпоподібноклітинна анемія. Який механізм приводить до зменшення кількості еритроцитів в крові у цього хворого?

Нестача вітаміну B12 і фолієвої кислоти

Внутрішньосудинний гемоліз

Позасудинний гемоліз

Нестача заліза в організмі

Нестача білка

3295 / 6854
У жінки 52-х років при обстеженні було виявлено зниження кількості еритроцитів у крові та підвищення рівня вільного гемоглобіну в плазмі крові (гемоглобінемія). КП- 0,85. Який вид анемії спостерігається у хворої?

Анемія внаслідок порушення еритропоезу

Хронічна постгеморагічна

Іостра постгеморагічна

Набута гемолітична

Спадкова гемолітична

3296 / 6854
Лікування туберкульозу здійснюється за допомогою комбінованої хіміотерапії, що поєднує речовини різного механізму дії. Який з протитуберкульозних засобів пригнічує транскрипцію ДНК в РНК мікобактерій?

Стрептоміцин

Ізоніазид

Рифампіцин

ПАСК

Етіонамід

3297 / 6854
На тканину діють електричним імпульсом катодного напрямку, амплітуда якого дорівнює 70% порогу. Які зміни мембранного потенціалу клітин це викличе?

Потенціал дії

Змін не буде

Часткова деполяризація

Гіперполяризація

3298 / 6854
У людини визначили величину енерговитрат. У якому стані знаходилась людина, якщо її енерговитрати виявилися меншими за основний обмін?

Легка робота

Нервове напруження

Спокій

Відпочинок

Сон

3299 / 6854
При визначенні основного обміну з’ясовано, що його величина у досліджуваного менша за належну величину на 7%. Це означає, що інтенсивність процесів енергетичного метаболізму у досліджуваного:

Нормальна

Помірно знижена

Суттєво підвищена

Суттєво знижена

Помірно підвищена

3300 / 6854
Внаслідок руйнування певних структур стовбуру мозку тварина втратила орієнтувальні рефлекси. Які структури було зруйновано?

Чорна речовина

Вестибулярні ядра

Чотиригорбкові тіла

Медіальні ядра ретикулярної формації

Червоні ядра

3301 / 6854
У новонародженого малюка педіатр виявив, що отвір крайньої плоті за величиною не перевищує діаметр сечовивідного каналу і голівка статевого члена не може виходити через такий отвір. Як називається цей стан?

Парафімоз

Гермафродитизм

Епіспадія

Фімоз

Гіпоспадія

3302 / 6854
Психологічне дослідження встановило: у людини добра здатність швидко пристосовуватися до нового оточення, добра пам’ять, емоційна стійкість, висока працездатність. Найімовірніше, ця людина:

Флегматик з елементами меланхоліка

Холерик

Меланхолік

Сангвінік

Флегматик

3303 / 6854
Хворий на ішемічну хворобу серця не повідомив лікаря, що в нього бувають напади бронхоспазму. Лікар призначив препарат, після прийому якого напади стенокардії стали рідше, але напади бронхоспазму стали частіше. Який препарат був призначений?

Дилтіазем

Верапаміл

Анаприлін

Нітросорбід

Нітрогліцерин

3304 / 6854
До ендокринолога звернулась хвора 45-ти років із скаргами на підвищення апетиту, сухість слизових оболонок ротової порожнини, збільшення діурезу. При обстеженні вперше виявлено інсулін незалежний діабет. Який з названих препаратів доцільно призначити хворій?

Окситоцин

Адіурекрин

Інсулін

Глібенкламід

Вазопресин

3305 / 6854
Дитина 7-ми років поступила в інфекційне відділення зі скаргами на різкий біль у горлі, утруднення під час ковтання, підвищення температури тіла до 39oC, набряк шиї. Об’єктивно: мигдалики збільшені, їх слизова оболонка повнокровна, вкрита великою кількістю плівок білувато-жовтого кольору, які щільно прилягають до слизової оболонки. При спробі зняти плівку залишається глибокий дефект, який кровоточить. Який вид запалення має місце?

Серозне

Крупозне

Дифтеритичне

Геморагічне

Гнійне

3306 / 6854
У хворого на хронічний дифузний гломерулонефрит розвинулася хронічна недостатність нирок. В термінальній стадії ХНН розвивається оліго- та анурія, що спричиняється:

Ішемією коркової речовини нирок внаслідок спазму судин

Зниженням маси діючих нефронів

Зменшенням фільтраційного тиску та фільтрації

Дисемінованим внутрішньосудинним зсіданням крові

Збільшенням реабсорбції води в дистальних канальцях

3307 / 6854
При розтині тіла померлого чоловіка 48-ми років в ділянці 1-го сегменту правої легені виявлено круглий утвір діаметром 5 см з чіткими контурами, оточений тонким прошарком сполучної тканини, виповнений білими крихкими масами. Діагностуйте форму вторинного туберкульозу:

Гострий вогнищевий туберкульоз

Фіброзно-кавернозний туберкульоз

Туберкулома

Казеозна пневмонія

Гострий кавернозний туберкульоз

3308 / 6854
На препарат представлено орган, вкритий сполучнотканинною капсулою, від якої відходять трабекули. В органі можна розрізнити кіркову речовину, де містяться лiмфатичнi вузлики та мозкову речовину, представлену тяжами лімфоїдних клітин. Який орган представлений на препараті?

Мигдалики

Тимус

Лімфатичний вузол

Селезінка

Червоний кістковий мозок

3309 / 6854
В регуляції фізіологічних функцій беруть участь іони металів. Один із них отримав назву «король месенджерів» Таким біоелементом посередником є:

Fe+++

Са++

Na+

Zn++

K+

3310 / 6854
Хворий звернувся до лікаря зі скаргами на болі в кульшовому суглобі та у всіх привідних м’язах і шкірі над ними при рухах. Який нерв ушкоджено?

Затульний нерв

Задній шкірний нерв стегна

Стегновий нерв

Латеральний шкірний нерв стегна

Клубово-пахвинний нерв

3311 / 6854
Хворому 35-ти років для обстеження очного дна був призначений атропіну сульфат у вигляді очних крапель. Для відновлення акомодації йому закрапали пілокарпіну гідрохлорид, але це не дало бажаного ефекту. Що є причиною відсутності ефекту?

Тахіфілаксія

Синергізм

Двосторонній антагонізм

Односторонній антагонізм

Звикання

3312 / 6854
В поліклініку до лікаря звернулася жінка 32-х років зі скаргами на відсутність в неї лактації після народження дитини. Дефіцитом якого гормону, найбільш імовірно, можна пояснити дане порушення?

Вазопресин

Тиреокальцитонін

Пролактин

Глюкагон

Соматотропін

3313 / 6854
У хворого пухлина черевної порожнини, що стискає нижню порожнисту вену. Який кава- кавальний анастомоз на передній стінці живота забезпечить відтік венозної крові?

Між верхніми і нижніми прямокишковими венами

Між верхньою і нижньою надчеревними венами

Між пупковою і верхньою надчеревною венами

Між хребетними і пупковими венами

Між непарною і напівнепарною венами

3314 / 6854
У людини під дією мутагенного фактору з’явилась велика кількість мутантних клітин. Але більшість з них були розпізнані і знищені клітинами:

В -лімфоцитами

Плазмобластами

Т-лімфоцитами кілерами

Стовбуровими

Т-лімфоцитами супресорами

3315 / 6854
При пошкодженні клітини іонізуючим випромінюванням вмикаються механізми захисту і адаптації. Який механізм відновлення порушеного внутрішньоклітинного гомеостазу реалізується при цьому?

Активація Са-опосередкованих клітинних функцій

Активація антиоксидантної системи

Гіпертрофія мітохондрій

Накопичення Na+ в клітинах

Пригнічення аденілатциклази

3316 / 6854
На гістологічному препараті легень видно структуру діаметром близько 0,5 мм, слизова оболонка якої вкрита одношаровим кубічним війковим епітелієм, у якому зустрічаються секреторні клітини Клара, війчасті клітини, мікроворсинчасті. Вкажіть структуру:

Середній бронх

Альвеолярний хід

Термінальна бронхіола

Малий бронх

Альвеола

3317 / 6854
Чоловік 36-ти років, лісник за фахом, через тиждень після тривалого перебування у весняному лісі гостро захворів -гарячка, головний біль, порушення свідомості, епілептиформні напади, смерть розвинулася на 3-й день хвороби. На роз- тині тіла: набряк головного мозку, множинні точкові геморагії; під час мікроскопічного дослідження - периваскулярний та перицелюлярний набряк, множинні периваскулярні,переважно лімфоцитарні, інфільтрати. Діагностуйте основне захворювання:

Кліщовий енцефаліт

Поліомієліт

Гнійний енцефаліт

Менінгококова інфекція

Церебро-васкулярна хвороба

3318 / 6854
У хворого з флегмоною передпліччя при мікробіологічному аналізі ексудату в зоні запалення визначена присутність стрептококів. Які клітини будуть переважати в ексудаті?

Еозинофільні гранулоцити

Моноцити

Нейтрофільні гранулоцити

Лімфоцити

Базофільні гранулоцити

3319 / 6854
У п’ятимісячної дівчинки виявлено застійні явища у легенях. При обстеженні виявлено зв’язок між висхідною аортою та легеневою артерією, що в нормі спостерігається у деяких земноводних і плазунів. Назвіть цю природжену ваду розвитку:

Незрощення боталової протоки

Транспозиція магістральних судин

Дефект міжпередсердної перегородки

Дефект міжшлуночкової перегородки

Розвиток правої дуги аорти

3320 / 6854
Хворому на гострий інфаркт міокарда у комплексній терапії було призначено гепарин. Через деякий час після введення даного препарату з’явилася гематурія. Який антагоніст гепарину необхідно ввести хворому для усунення даного ускладнення?

Вікасол

Фібриноген

Амінокапронова кислота

Неодикумарин

Протаміну сульфат

3321 / 6854
До лікарні звернулася жінка з дитиною, у якої на голові була гангренозна рана. Лікар при огляді виявив у рані білих червоподібних личинок комах. Яка комаха могла їх відкласти?

Москіти

Комарі

Вольфартова муха

Блохи

Муха-жигалка

3322 / 6854
На дослідження в бактеріологічну лабораторію було відправлено випорожнення хворої дитини грудного віку, з яких виділена культура ентеропатогенних кишкових паличок О55К59. На основі яких критеріїв виділена культура віднесена до ЕПКП О55?

Морфологічні ознаки

Антигенні властивості

Культуральні ознаки

Визначення фаговару

Біохімічні властивості

3323 / 6854
У чоловіка 30-ти років перед операцією визначили групову належність крові. Кров резус- позитивна. Реакцію аглютинації еритроцитів не викликали стандартні сироватки груп 0а/3 (I), А/3 (II), Ва (III). Досліджувана кров належить до групи:

Ва (III)

0ав (I)

Ар (II)

АВ (IV)

3324 / 6854
Швидкість проведення збудження нервовими волокнами становить 120 м/с. Який з наведених чинників, перш за все, забезпечує таку швидкість?

Великий фактор надійності

Великий потенціал спокою

Велика амплітуда потенціалу дії

Наявність мієлінової оболонки

Малий поріг деполяризації

3325 / 6854
У тварини зруйнували отолітові вестибулорецептори. Які з наведених рефлексів зникнуть внаслідок цього у тварини?

Випрямлення тулуба

Статокінетичні при рухах з кутовим прискоренням

Первинні орієнтувальні

Статокінетичні при рухах з лінійним прискоренням

Міотатичні

3326 / 6854
У хворого з ВШ-інфекцією визначена ураження шкіри нижніх кінцівок у вигляді множинних пухлинних вузликів синюшно-червоного кольору, що зливаються і створюють поверхневі виразки. При дослідженні біоптату шкіри виявлено новоутворення кровоносних судин, що створюють порожнини різної форми і величини та побудоване з ендотелію, багато пучків веретеноподібних клітин. Діагностуйте найімовірніший характер шкірної патології:

Дерматомікоз

Запальний дерматит

Саркома Капоші

Базаліома

Лімфома шкіри

3327 / 6854
У хворого з патологією серцево-судинної системи розвинулись набряки на нижніх кінцівках. Який механізм розвитку серцевого набряку?

Підвищення гідростатичного тиску в артеріолах

Зниження осмотичного тиску плазми крові

Порушення лімфовідтоку

Підвищення гідростатичного тиску в венулах

Підвищення онкотичного тиску плазми крові

3328 / 6854
У хворої при лікуванні гіпертонічної хвороби на фоні прийому препарату виник сухий кашель. Для якого препарату характерна така побічна дія?

Дротаверин

Резерпін

Лізиноприл

Гідрохлортіазид

Но-шпа

3329 / 6854
У фібробластах шкіри дитини із хворобою Дауна виявлено 47 хромосом. Визначте тип аномалії:

Трисомія 18

Трисомія 21

Трисомія Х

Трисомія 13

Полісомія Y

3330 / 6854
При операції у хлопчика 12-ти років видалений апендикс, який надіслано патологу на дослідження. Макроскопічно: апендикс в дистальному відділі з булавоподібним стовщенням діаметром 3 см, при розрізі якого вилилася прозора жовтувата рідина, стінка апендикса стоншена. Мікроскопічно: атрофія всіх шарів апендикса, ознак запалення немає. Який найбільш імовірний діагноз?

Хронічний апендицит

Водянка апендикса

Емпієма апендикса

Флегмонозний апендицит

Міксоглобульоз апендикса

3331 / 6854
У лікарню наприкінці робочого дня доставлений робітник «гарячого» цеху, який скаржиться на головний біль, запаморочення, нудоту, загальну слабкість. Об’єктивно: свідомість збережена, шкірні покриви гіперемовані, сухі, гарячі на дотик. Частота серцевих скорочень - 130/хв. Дихання часте, поверхневе. Яке порушення процесів терморегуляції, найбільш імовірно, виникло у людини в даній ситуації"?

Посилення тепловіддачі і теплопродукції

Зниження тепловіддачі

Зниження теплопродукції без змін тепловіддачі

Посилення тепловіддачі і зниження теплопродукції

Посилення теплопродукції без змін тепловіддачі

3332 / 6854
При розтині трупа чоловіка 48-ми років виявлено, що кістковий мозок пласких кісток, діафізів та епіфізів трубчастих кісток соковитий, сіро-червоний або сіро-жовтий гноєподібний (піоїдний кістковий мозок). Селезінка масою - 7 кг. На розрізі вона темно-червоного кольору, з ішемічними інфарктами. Всі лімфатичні вузли збільшені, м’які, сіро-червоного кольору. В печінці жирова дистрофія і лейкемічні інфільтрати. Який найбільш імовірний діагноз?

Лімфогранулематоз

Гострий мієлоїдний лейкоз

Мієломна хвороба

Гострий лімфоїдний лейкоз

Хронічний мієлоїдний лейкоз

3333 / 6854
У 3-річної дитини тривале підвищення температури, збільшені лімфовузли, у крові - значне підвищення кількості лімфоцитів. Методом ІФА виявлено антиген віруса Епштейна-Бара. Який діагноз можна поставити на основі вказаного?

Генералізована інфекція, викликана herpes-zoster

Інфекційний мононуклеоз

Лімфома Беркета

Цитомегаловірусна інфекція

Герпетична аденопатія

3334 / 6854
В бактеріологічну лабораторію поступив досліджуваний матеріал (промивні води, в’ялена риба домашнього приготування), взятий у хворого з підозрою на ботулізм. На яке поживне середовище слід зробити первинний посів матеріалу?

Цукровий м’ясо-пептонний бульйон

Сироватковий агар

Середовище Кітта-Тароцці

Цукровий м’ясо-пептонний агар

Цукрово-кров’яний агар

3335 / 6854
У юнака 18-ти років діагностовано хворобу Марфана. При дослідженні встановлено: порушення розвитку сполучної тканини, будови кришталика ока, аномалії серцево-судинної системи, арахнодактилія. Яке генетичне явище зумовило розвиток цієї хвороби?

Кодомінування

Множинний алелiзм

Комплементарність

Неповне домiнування

Плейотропія

3336 / 6854
У бактерій встановлений процес кон’югації, при якому між бактерiями утворюється цитоплазматичний місток, по якому з клітини-донора до клітини-реціпієнта надходять плазміди і фрагменти молекули ДНК. Яке значення цього процесу?

Ліквідує небажані мутації

Підвищує гетерозиготність

Забезпечує обмін генетичного матеріалу

Сприяє активізації мутаційного процесу

Забезпечує обмін речовинами між клітинами

3337 / 6854
Дослідженнями останніх десятиліть встановлено, що безпосередніми «виконавцями» апоптозу в клітині є особливі ферменти - каспази. В утворенні одного з них бере участь цитохром С. Вкажіть його функцію в нормальній клітині:

Компонент піруватдегідрогеназної системи

Фермент бета-окислювання жирних кислот

Фермент дихального ланцюга переносу електронів

Компонент H + - АТФ-азной системи

Фермент ЦТК

3338 / 6854
У відділення реанімації був доставлений хворий з інфарктом міокарда. Який препарат необхідно ввести хворому для лікування больового шоку?

Промедол

Налоксон

Парацетамол

Целекоксиб

Анальгін

3339 / 6854
Жінка 52-х років, хвора на рак молочної залози, пройшла курс променевої терапії. Розмір пухлини зменшився. Який з наведених механізмів ушкодження клітини найбільше обумовлює ефективність променевої терапії?

Гіпертермія

Лізис NK-клітинами

Тромбоз судин

Утворення вільних радикалів

Мутагенез

3340 / 6854
Після операції на кишечнику у хворого з’явились симптоми отруєння аміаком за типом печінкової коми. Який механізм дії аміаку на енергозабезпечення ЦНС?

Гальмування бета-окиснення жирних кислот

Гальмування гліколізу

Інактивація ферментів дихального ланцюга

Гальмування ЦТК в результаті зв’язування альфа-кетоглутарату

Роз’єднування окисного фосфорилювання

3341 / 6854
У пацієнта перфоративна виразка передньої стінки шлунка. В яке похідне очеревини попаде вміст шлунка?

Правий брижовий синус

Чепцева сумка

Печінкова сумка

Лівий брижовий синус

Передшлункова сумка

3342 / 6854
При огляді хворої лікар-гінеколог відмітив симптоми запалення статевих шляхів, у мазку взятому із піхви, виявлено грушоподібні найпростіші з шипом, з передньої частини відходять джгутики, наявна ундулююча мембрана. Яке захворювання підозрює лікар у хворої?

Лямбліоз

Токсоплазмоз

Балантидіоз

Урогенітальний трихомоноз

Кишковий трихомоноз

3343 / 6854
Жінка звернулася до лікаря зі скаргами на утруднення рухів язика. Обстеження головного мозку за допомогою ЯМР показало, що у хворої крововилив в нижньому відділі довгастого мозку. Про пошкодження якого ядра довгастого мозку у хворої можна думати?

Одиноке ядро

Ядро під’язикового нерва

Нижнє слиновидільне ядро

Ядро додаткового нерва

Подвійне ядро

3344 / 6854
При мікроскопічному дослідженні виявляється паренхіматозний орган, в якому епітеліальні тяжі формують клубочкову, пучкову та сітчасту зони. Центральна частина органу представлена скупченнями хроматофінних клітин. Визначте орган:

Епіфіз

Печінка

Надниркова залоза

Щитоподібна залоза

Гіпофіз

3345 / 6854
У відділення реанімації поступив хворий після ДТП з однобічним пневмотораксом. Який вид дихання спостерігається у даному випадку?

Глибоке часте

Поверхневе

Асфіктичне

Поверхневе рідке

Поверхневе часте

3346 / 6854
У хворої виявлена ракова пухлина шийки матки. У які регіонарні лімфатичні вузли можливе розповсюдження метастазів?

Навколоміхурні і поперекові

Пахвинні і здухвинні

Верхні і нижні брижові

Поперекові і ниркові

3347 / 6854
При огляді порожнини рота дитини педіатр виявила наявність нижніх медіальних різців. Дитина розвивається нормально. Визначте її вік:

6-7 місяців

13-14 місяців

8-9 місяців

10-12 місяців

3348 / 6854
Серед антиатеросклеротичних препаратів, що застосовуються з метою профілактики та лікування атеросклерозу, є левостатин. Він діє шляхом:

Гальмування біосинтезу холестерину

Активації метаболізму холестерину

Стимулювання екскреції холестерину з організму

Усіма наведеними шляхами

Пригнічення всмоктування холестерину в кишківнику

3349 / 6854
При зниженні активності ферментів антиоксидантного захисту посилюються процеси перекисного окиснення ліпідів клітинних мембран. При нестачі якого мікроелементу знижується активність глутатіонпероксидази?

Селен

Марганець

Молібден

Кобальт

Мідь

3350 / 6854
Хворий 75-ти років був оперований з приводу рака передміхурової залози, помер раптово на 4-ту добу після оперативного втручання. При розтині тіла померлого у просвітах головного стовбура і біфуркації легеневої артерії були виявлені і легко видалені крихкі маси темно-червоного кольору з тьмяною поверхнею. Такі ж маси знаходились у порожнині правого шлуночка серця. Який різновид порушення кровообігу призвів до раптової смерті хворого?

Тромбоемболія легеневої артерії

Тканинна емболія

Парадоксальна емболія

!нфаркт міокарда

Тромбоз легеневої артерії

3351 / 6854
При обстеженні жінки 56-ти років, що хвора на цукровий діабет 1-го типу, виявлене порушення білкового обміну, що при лабораторному дослідженні крові проявляється аміноацидемією а клінічно - уповільненням загоєння ран і зменшенням синтезу антитіл. Який з перерахованих механізмів викликає розвиток аміноацидемії?

Підвищення протеолізу

Підвищення онкотичного тиску в плазмі крові

Гіперпротеїнемія

Збільшення ліпопротеїдів високої щільності

Зменшення концентрації амінокислот у крові

3352 / 6854
При недостатності вітаміну C спостерігається порушення структури колагенових волокон. Яка стадія їх синтезу порушується при цьому?

Агрегація тропоколагену і утворення фібрил

Приєднання глюкозних і галактозних залишків

Відщеплення сигнальних олігопептидів

Утворення гідроксипроліну і гідроксилізину

Утворення гама-карбоксиглутамату

3353 / 6854
У людини в результаті патологічного процесу збільшена товщина гематоальвеолярного бар’єру. Безпосереднім наслідком цього буде зменшення:

Кисневої ємності крові

Хвилинного об’єму дихання

Дифузійної здатності легень

Резервного об’єму видиху

Альвеолярної вентиляції легень

3354 / 6854
Недостатність в організмі мікроелементу селену проявляється кардіоміопатією. Імовірною причиною такого стану є зниження активності такого селенвмісного ферменту:

Каталаза

Лактатдегідрогеназа

Цитохромоксидаза

Глутатіонпероксидаза

Сукцинатдегідрогеназа

3355 / 6854
У хворого після перенесеного ішемічного інсульту стали неможливими довільні рухи у правих кінцівках, спостерігається гіперрефлексія. Під час пальпації визначається підвищений тонус м’язів кінцівок. Яка форма порушення рухової функції має місце у хворого?

Центральний параліч

Периферичний парез

Мозочкова атаксія

Тетанія

Периферичний параліч

3356 / 6854
У жінки, що тривало приймала антибіотики з приводу кишкової інфекції, розвинулось ускладнення з боку слизової порожнини рота у вигляді запального процесу і білого нальоту, у якому під час бактеріологічного дослідження були виявлені дріжджеподібні грибки Candida albicans. Який з перерахованих препаратів показаний для лікування цього ускладнення?

Флуконазол

Бісептол

Поліміксин

Тетрациклін

Фуразолідон

3357 / 6854
Досліджуються клітини червоного кісткового мозку людини, які належать до клітинного комплексу, що постійно діляться. Який процес забезпечує генетичну ідентичність цих клітин:

Мейоз

Трансплантація

Репарація

Мутація

Мітоз

3358 / 6854
Хвора 57-ми років для лікування гіпертонічної хвороби тривалий час приймала анаприлін. Побічні ефекти спонукали пацієнтку відмовитись від прийому препарату, що призвело до розвитку гіпертонічного кризу і нападу стенокардії. Як називається ускладнення, яке виникло?

Тахіфілаксія

Лікарська залежність

Звикання

Сенсибілізація

Синдром відміни

3359 / 6854
При лабораторному дослідженні крові пацієнта виявлено, що вміст білків у плазмі становить 40 г/л. Як це впливає на транскапілярний обмін води в мікроциркуляторному руслі?

Зменшуються фільтрація і реабсорбція

Зменшується фільтрація, збільшується реабсорбція

Обмін не змінюється

Збільшуються фільтрація і реабсорбція

Збільшується фільтрація, зменшується реабсорбція

3360 / 6854
У хворого спостерігається тремтіння рук, що пов’язане з хворобою Паркінсона. Дефіцит якого медіатора в стріопалідарних структурах призводить до таких симптомів?

Серотонін

Субстанція Р

ГАМК

Норадреналін

Дофамін

3361 / 6854
У хворого на жовчно-кам’яну хворобу спостерігаються ознаки холемічного синдрому. Який симптом з перерахованих зумовлений відсутністю надходження жовчі у кишечник?

Астенія

Шкірний свербіж

Брадикардія

Гіпотонія

Стеаторея

3362 / 6854
У людини після гострої крововтрати виникло відчуття спраги. Зміна якого гомеостатичного параметру викликала це відчуття?

Підвищення онкотичного тиску рідин організму

Зниження онкотичного тиску рідин організму

Підвищення осмотичного тиску рідин організму

Зменшення об’єму позаклітинної рідини

Зниження осмотичного тиску рідин організму

3363 / 6854
Студент використав консервовану донорську кров для визначення часу її зсідання. Однак, будь-якого позитивного результату він отримати не зміг. Причиною цього є відсутність в крові:

Іонізованого кальцію

Вітаміну K

Тромбопластину

Фі6риногєну

Фактора Хагемана

3364 / 6854
Введення тварині екстракту тканини передсердя посилює виділення натрію з сечею. Дія якої біологічно активної речовини стала причиною такого стану?

Ілюкокортикоїд

Калійкреїн

Серотонін

Адреналін

Натрійуретичний гормон

3365 / 6854
Хворій жінці із захворюванням нирок, що супроводжується вираженими набряками, призначили діуретичний препарат, що пригнічує реабсорбцію в нирках іонів Na+ і води, посилює виведення нирками іонів K+ і Mg++, викликає гіперурикемію, зумовлює потужний діуретичний ефект. Назвіть цей препарат:

Спіронолактон

Аллопуринол

Тріамтерен

Діакарб

Фуросемід

3366 / 6854
Хворому призначили антибіотик хлорамфенікол (левомецитин), який порушує у мікроорганізмів синтез білку шляхом гальмування процесу:

Процесінг

Транскрипція

Ампліфікація генів

Утворення полірибосом

Елонгація трансляції

3367 / 6854
До офтальмолога звернулася жінка зі скаргами на шкірний свербіж і набряклість повік. При обстеженні виявлено членистоноге червоподібної форми величиною 0,4 мм. Суцільний щиток вкриває передню частину тіла, тіло має поперечну смугастість. Ноги короткі, лапки з двома кігтиками. Який діагноз може встановити лікар?

Демодекоз

Педікульоз

Фтиріоз

Факультативний міаз

Короста

3368 / 6854
У хворого 69-ти років на шкірі в ділянці нижньої повіки з’явилося невелике бляшкоподібне утворення з наступним виразкуванням, яке було оперативно видалене. При мікроскопічному дослідженні утворення: в дермі шкіри комплекси з атипових епітеліальних клітин; периферії клітини розташовані перпендикулярно до базальної мембрани. Клітини темні, призматичної полігональної форми з гіперхромними ядрами з частими мітозами. Іноді зустрічаються утворення, подібні до волосяного фолікула. Яка гістологічна форма раку у хворого?

Базально-клітинний рак

Недиференційований рак

Плоскоклітинний рак без ороговіння

Плоскоклітинний рак з ороговінням

Аденокарцинома

3369 / 6854
Після перелому нижньої щелепи постраждалий відзначає втрату чутливості шкіри у ділянці підборіддя і нижньої губи. Який нерв був пошкоджений?

Maxillaris

Facialis

Buccalis

Infraorbitalis

Mentalis

3370 / 6854
У молодого чоловіка внаслідок подразнення сонячного сплетення запальним процесом (солярит) підвищена функціональна активність залоз шлунка, що виражається, зокрема, у збільшенні продукції хлоридної кислоти. Яка з вказаних нижче речовин викликає гіперхлоргідрію у даному випадку?

Калікреїн

Глюкагон

Гастрин

Урогастрон

Гастроінгібуючий пептид

3371 / 6854
У альпініста, що піднявся на висоту 5200 м, розвинувся газовий алкалоз. Що є причиною його розвитку?

Зниження температури навколишнього середовища

Гіповентиляція легенів

Гіпероксемія

Гіпоксемія

Гіпервентиляція легенів

3372 / 6854
Реакції міжмолекулярного транспорту одновуглецевих радикалів є необхідними для синтезу білків та нуклеїнових кислот. З якого з наведених нижче вітамінів утворюється кофермент, потрібний для вказаних вище реакцій?

Рибофлавін

Фолієва кислота

Аскорбінова кислота

Пантотенова кислота

Тіамін

3373 / 6854
Прокаріотичні та еукаріотичні клітини характеризуються здатністю до по- ділу. Поділ прокаріотичних клітин відрізняється від поділу еукаріотичних, але існує молекулярний процес, який лежить в основі цих поділів. Який це процес?

Репарація

Ампліфікація генів

Транскрипція

Реплікація ДНК

Трансляція

3374 / 6854
Для проведення анальгезії наркотичний анальгетик застосували з препаратом бензодіазепінового ряду. Який засіб використали для потенціювання анальгезії?

Трифтазин

Карбамазепін

Імізин

Хлорпротіксен

Діазепам

3375 / 6854
Молода людина 25-ти років споживає надмірну кількість вуглеводів (600 г на добу), що перевищує її енергетичні потреби. Який процес буде активуватися в організмі людини уданому випадку?

Окиснення жирних кислот

Гліколіз

Ліпогенез

Ліполіз

Глюконеогенез

3376 / 6854
Юнак 15-ти років скаржиться на загальну слабкість, запаморочення, швидку стомлюваність. При обстеженні виявлено еритроцити зміненої форми, кількість їх знижена. Попередній діагноз: серпоподібно-клітинна анемія. Який тип мутації зумовлює розвиток цього патологічного стану?

Хромосомна аберація

Інверсія

Делеція

Мутація зсуву рамки зчитування

Точкова мутація

3377 / 6854
Хворому із больовим синдромом в суглобах постійно призначають аспірин. Який з перерахованих ферментів він пригнічує?

Фосфоліпаза Д

Фосфоліпаза А2

Циклооксигеназа

Фосфоліпаза С

Ліпооксигеназа

3378 / 6854
Препарати групи вітаміну В2 призначають при захворюваннях шкіри. Завдяки наявності якої структури в його складі визначається здатність до окиснення-відновлення?

Залишок фосфорної кислоти

Рибітол

Ізоалоксазин

Рибозофосфат

Аденін

3379 / 6854
Під час автомобільної аварії людина отримала сильний удар в епігастральну ділянку, внаслідок чого виникла зупинка серця. Що могло стати причиною таких змін серцевої діяльності?

Підвищення тонусу симпатичної нервової системи

Збільшене виділення адреналіну

Збільшене виділення альдостерону

Збільшене виділення кортизолу

Підвищення тонусу блукаючого нерва

3380 / 6854
У хворого з синдромом Іценко-Кушинга спостерігаються стійка гіперглікемія та глюкозурія. Синтез та секреція якого гормону збільшені у цього хворого?

Альдостерон

Кортизол

Тироксин

Глюкагон

Адреналін

3381 / 6854
У хворого має місце хронічний запальний процес мигдаликів. За рахунок якого біохімічного процесу у вогнищі запалення підтримується концентрація НАДФН, необхідного для реалізації механізму фагоцитозу?

Цикл Кребса

Орнітиновий цикл

Цикл Корі

Пентозо-фосфатний шлях

Гліколіз

3382 / 6854
Щоденно в організмі людини 0,5% всього гемоглобіну перетворюється на метгемоглобін. Який фермент, що міститься в еритроцитах, каталізує відновлення метгемоглобіну до гемоглобіну?

Глюкуронілтрансфераза

Метгемоглобінредуктаза

Метгемоглобінтрансфераза

Гемоксигеназа

Білівердинредуктаза

3383 / 6854
У хворого з гемолітичною анемією виявлено дефіцит піруваткінази в еритроцитах. За цих умов причиною розвитку гемолізу еритроцитів є:

Генетичні дефекти глікофорину А

Дефіцит спектрину

Нестача Na+ в еритроцитах

Зменшення активності Na+, К+ -АТФ-ази

Надлишок К + в еритроцитах

3384 / 6854
Хвора 37-ми років померла під час нападу експіраторної задухи, що був спричинений контактом з екзогенним алергеном (пилок амброзії). При гістологічному дослідженні в просвіті бронхів спостерігаються скупчення слизу, в стінці бронхів багато тучних клітин (лаброцитів), більшість з яких у стані дегрануляції, багато еозинофілів. До патогенезу якого типу реакцій гіперчутливості можна віднести описані зміни?

V типу (гранулематоз)

IV типу (клітинна цитотоксичність)

II типу (антитілозалежна)

I типу (анафілактична)

III типу (імунокомплексна)

3385 / 6854
У хворого запалення присереднього надвиростка плечової кістки (епіконділіт). Який нерв залучений у процес?

N. axillaris

N. musculocutaneus

N. medianus

N. radialis

N. ulnaris

3386 / 6854
Який вітамінний препарат доцільно призначити пацієнту зі скаргами на зниження гостроти зору в сутінках, сухість шкіри, часті застудні захворювання, ламкість волосся?

Нікотинова кислота

Пантотенова кислота

Ергокальциферол

Тіаміну хлорид

Ретинолу ацетат

3387 / 6854
43-х річний капітан корабля скаржиться на прояви сезонної алергії. Який лікарський засіб, що не має снодійного ефекту, можна йому призначити?

Ванкоміцин

Лоратадин

Супрастин

Дипразин

Мікосептин

3388 / 6854
У жінки 52-х років артеріальна гіпертензія ускладнилась правобічною геміплегією і втратою мови. Яка зона головного мозку є імовірно найбільш ураженою?

Права передня звивина

Ліва передня звивина

Ліва скронева частка

Ліва передня звивина і ліва скронева частка

Потилична частка

3389 / 6854
Хворому на хронічний бронхіт призначили муколітичний препарат, що підвищує синтез сурфактанту. Який препарат був використаний?

Кодеїн

Фенотерол

Амброксол

Лібексин

Мукалтин

3390 / 6854
Ефективна діагностика носійства збудників кишкових інфекцій ґрунтується на виявленні антитіл до певних антигенів бактерій в реакції непрямої гемаглютинації. Який стандартний препарат слід застосувати у цій реакції'?

Еритроцитарні діагностикуми з адсорбованими антигенами бактерій

Еритроцити барана й гемолітичну сироватку

Моноклональні антитіла

Монорецепторні діагностичні сироватки

Антитіла проти імуноглобулінів основних класів

3391 / 6854
На розтині тіла померлого, який страждав на гіпертонічну хворобу, у лівій гемісфері мозку виявлена порожнина округлої форми 4х5 см із іржавою стінкою, заповнена жовтуватою прозорою рідиною. Назвіть патологію, яка розвинулась у головному мозку хворого:

Немічний інфаркт

Абсцес

Кіста

Геморагічне просякнення

Гематома

3392 / 6854
Клінічні ознаки хвороби у пацієнта почалися через 24 години після вживання рибних консервів і включали нудоту, слабкість, сухість у роті. Потім приєдналися двоїння зображення, афагія, проблеми з диханням. Чим обумовлена така симптоматика при ботулізмі?

Вплив ентеротоксину

Активація цАМФ

Розмноження збудника у ШКТ

Вплив нейротоксину

Вплив ендотоксину

3393 / 6854
У хворого з підозрою на «озену» з носоглотки були виділені грамнегативні палички, які утворювали капсулу на поживному середовищі. Які мікроорганізми спричинили хворобу?

Хламідії

Шигели

Мікоплазми

Сальмонели

Клебсієли

3394 / 6854
До травматологічного пункту доставлено хворого з пошкодженням м’язів нижніх кінцівок. За рахунок яких клітин можлива репаративна регенерація м’язових волокон і відновлення функції м’язів?

Плазмоцити

Ендотеліоцити

Міосателітоцити

Адипоцити

Фібробласти

3395 / 6854
У чоловіка 25-ти років на 4-й день після крововтрати в крові зросла кількість поліхроматофільних еритроцитів, ретикулоцитів і навіть з’явились поодинокі нормобласти. Про що свідчить такі гематологічні зміни?

Процеси регенерації випереджають дозрівання клітин

Зменшено об’єм плазми крові

Підвищена спорідність гемоглобіну до кисню

Знижена продукція еритропоезу в нирках

Ослаблені процеси еритропоезу

3396 / 6854
Хворому на туберкульоз легень призначений препарат першого ряду, який спричинив розвиток невриту лицьового нерва та порушення рівноваги. Вкажіть цей лікарський засіб:

Левоміцетин

Бісептол

Фуразолідон

Цефазолін

Ізоніазид

3397 / 6854
Хірург під час операції на щитоподібній залозі перев’язав верхню щитоподібну артерію. Гілку якої судини перев’язав лікар?

A. carotis externa

A. pharyngea ascendens

A. carotis interna

A. lingualis

A. facialis

3398 / 6854
Батьки - глухонімі, але глухота у дружини залежить від аутосомно-рецесивного гена, а у чоловіка виникла внаслідок тривалого прийому антибіотиків у дитинстві. Яка імовірність народження глухої дитини в родині, якщо батько гомозиготний за аллелю нормального слуху?

100%

75%

25%

0%

10%

3399 / 6854
До лікарні потрапив футболіст з ушкодженням поверхневого пахвинного кільця та розривом двох ніжок, що його обмежують. Похідним якої анатомічної структури вони є?

Апоневроз поперечного м’яза живота

Апоневроз зовнішнього косого м’яза живота

Апоневроз внутрішнього косого м’яза живота

Власна фасція живота

Міжножкові волокна

3400 / 6854
У потерпілого перелом у ділянці бічної поверхні променево-зап’ясткового суглоба. Де найімовірніше відбувся перелом?

Головчаста кістка

Шилоподібний відросток ліктьової кістки

Шилоподібний відросток променевої кістки

Нижня третина плечової кістки

Гачкувата кістка

3401 / 6854
Під час гіпертонічного кризу хворому ввели магнію сульфат, в результат чого відбулося різке зниження артеріального тиску. Введенням якого препарату можна запобiгти по6ічному ефекту сульфату магнію?

Кальцію хлорид

Натрію сульфат

Трилон Б

Натрію бромід

Калію хлорид

3402 / 6854
Паренхіма аденогіпофізу представлена трабекулами, утвореними залозистими клітинами. Серед аденоцитів є клітини з гранулами, які забарвлюються основними барвниками і містять глікопротеїди. Які це клітини?

Хромофобні клітини

Мамотропоцити

Гонадотропоцити, тиротропоцити

Меланотропоцити

Соматотропоцити

3403 / 6854
У хворого із сечокам’яною хворобою виник нестерпний спастичний біль. Для попередження больового шоку йому ввели разом з атропіном наркотичний анальгетик, що не має спазмогенного ефекту. Який це препарат?

Промедол

Піритрамід

Трамадол

Морфіну гідрохлорид

Етилморфіну гідрохлорид

3404 / 6854
До травматологічного пункту доставлено постраждалого після ДТП з діагнозом: закритий перелом середньої третини стегна зі зміщенням. З метою репозиції кісткових уламків хворому введено 10 мл 2% розчину дитиліну в/в, внаслідок чого розвинулося тривале апное та міорелаксація. Дефіцитом якого ферменту зумовлена вказана фармакогенетична ферментопатія?

Уридіндифосфоглюкуронова трансфераза

Глюкозо-6-фосфатдегідрогеназа

Псевдохолінестераза

N-ацетилтрансфераза

Метгемоглобінредуктаза

3405 / 6854
Підлітку, що перебував у стані тяжкого алкогольного сп’яніння, лікар швидкої допомоги серед інших заходів здійснив внутрішньом’язове введення розчину кофеїну. Поясніть, який принцип взаємодії між алкоголем та кофеїном пояснює доцільність даної маніпуляції:

Потенціація

Фізіологічний антагонізм

Конкурентний антагонізм

Сумація ефектів

Синергізм

3406 / 6854
Захворювання бері-бері - це класична форма недостатності вітаміну тіаміну. Активна форма його синтезується за допомогою ферменту з класу:

Ізомераз

Гідролаз

Трансфераз

Ліаз

Оксидоредуктаз

3407 / 6854
Після введення пірогену у хворого підвищилася температура тіла, шкірні покриви стали блідими, холодними на дотик, з’явилася остуда, збільшилося споживання кисню. Яка зміна у процесах терморегуляції буде спостерігатися, насамперед, під час описаного періоду лихоманки?

Зниження теплопродукції

Зниження тепловіддачі

Тепловіддача та теплопродукція перебувають у рівновазі

Збільшення теплопродукції

3408 / 6854
Епідеміологічне дослідження поширення пухлин виявило високу кореляцію розвитку пухлин легень з тютюнопалінням. З дією якого хімічного канцерогену найімовірніше пов’язаний розвиток даного виду патології?

Метилхолантрен

Ортоаміноазотолуол

Афлатоксин

Діетилнітрозамін

3,4-бензопірен

3409 / 6854
В ході експерименту у білого щура моделювався набряк легені шляхом введення адреналіну. Який патогенетичний механізм розвитку набряку є провідним в даному випадку?

Гідродинамічний

Токсичний

Лімфогенний

Колоїдно-осмотичний

Мембраногенний

3410 / 6854
У хворого інфаркт міокарда. Активність якого ферменту буде значно підвищена в сироватці крові хворого в перші години?

Креатинфосфокіназа МВ

ЛДГ4

АЛТ

ЛДГ5

ACT

3411 / 6854
У хворого спостерігається атонія м’язів. Назвіть фермент м’язової тканини, активність якого може бути знижена при такому стані:

Креатинфосфокіназа

Амілаза

Транскетолаза

Каталаза

Глутамінтрансфераза

3412 / 6854
Пацієнтові, що звернувся до травмпункту у зв’язку з травмою, отриманою під час роботи на присадибній ділянці, лікар призначив введення правцевого анатоксину. Який імунітет сформується у даного пацієнта після введення препарату?

Антитоксичний активний

Антитоксичний пасивний

Антимікробний активний

Антимікробний пасивний

Нестерильний

3413 / 6854
При яких групах крові батьків за резус-фактором можливий розвиток резус-конфлікту під час вагітності?

Жінка Rh(+), чоловік Rh(+) (гомозигота)

Жінка Rh(+) (гетерозигота), чоловік Rh(+) (гомозигота)

Жінка Rh(-), чоловік Rh(-)

Жінка Rh(-), чоловік Rh(+) (гомозигота)

Жінка Rh(+), чоловік Rh(+) (гетерозигота)

3414 / 6854
Здорова людина перебуває в небезпечному за захворюванням на малярію районі. Який із зазначених препаратів необхідно призначити з метою особистої хіміопрофілактики малярії?

Бісептол

Сульфален

Хінгамін

Метронідазол

Тетрациклін

3415 / 6854
У хворого спостерігається типова для нападу малярії клінічна картина: остуда, жар, проливний піт. Яка стадія малярійного плазмодію найімовірніше буде виявлена в крові хворого в цей час?

Мерозоїт

Оокінета

Спороциста

Спорозоїт

Мікро- або макрогамети

3416 / 6854
Юнак 15-ти років скаржиться на загальну слабкість, запаморочення, швидку стомлюваність. В ході обстеження виявлено еритроцити зміненої форми, кількість їх знижена. Попередній діагноз: серпоподібноклітинна анемія. Який тип мутації зумовлює розвиток цього патологічного стану?

Хромосомна аберація

Точкова мутація

Інверсія

Делеція

Мутація зсуву рамки зчитування

3417 / 6854
Хворій виконують операцію на щитоподібній залозі. Гілки яких артерій повинен перев’язати хірург під час операції?

Верхня щитоподібна та висхідна шийна

Верхня та нижня гортанна

Верхня щитоподібна та щитошийний стовбур

Висхідна шийна та глибока шийна

Верхня та нижня щитоподібна

3418 / 6854
У хворого 75-ти років з позачеревної клітковини видалена пухлина розмірами 16х8х6 см. Мікроскопічно: анаплазовані жирові клітини з ознаками клітинного атипізму, поліморфізму. Зустрічаються величезні спотворені клітини, що мають у цитоплазмі жирові крапельки. Найімовірнішим є такий діагноз:

Фібросаркома

Мезотеліома

Ліпома

Ліпосаркома

Міосаркома

3419 / 6854
Внаслідок травми у людини ушкоджений отолітовий апарат внутрішнього вуха. На які подразники НЕ ЗМОЖЕ реагувати ця людина?

Рух з кутовим прискоренням

Дотикові

Звукові

Світлові

Рух з лінійним прискоренням

3420 / 6854
У хворого напад тахікардії. Які мембранні циторецептори кардіоміоцитів доцільно заблокувати, щоб припинити напад?

α-адренорецептори

М-холінорецептори

Н-холінорецептори

β-адренорецептори

М- та Н-холінорецептори

3421 / 6854
Під час патологічних процесів, які супроводжуються гіпоксією, відбувається неповне відновлення молекули кисню в дихальному ланцюзі і накопичення пероксиду водню. Вкажіть фермент, який забезпечує його руйнування:

Цитохромоксидаза

Каталаза

Аконітаза

Сукцинатдегідрогеназа

Кетоглутаратдегідрогеназа

3422 / 6854
У хворого після травми виникла необхідність введення протиправцевої сироватки, але проба на чутливість до сироватки виявилася позитивною. Специфічну гіпосенсибілізацію у хворого слід виконати за допомогою введення:

Фізіологічних доз глюкокортикоїдів

Роздільної дози специфічного алергену

Мінімальних доз специфічного алергену

Лікувальних доз антигістамінних препаратів

Наркотичних речовин, що знижують чутливість

3423 / 6854
Водій автомобіля дістав травму грудної клітки внаслідок удару об рульове колесо. Яка з перерахованих артерій найімовірніше може бути ушкодженою?

А. suprascapularis

А. subscapularis

А. vertebralis

A. thoracica interna

А. thyroidea superior

3424 / 6854
У вагітної жінки розвинувся токсикоз з тяжким повторним блюванням шлунковим вмістом протягом доби. Наприкінці доби почали проявлятися тетанічні судоми та зневоднення організму. Який розлад кислотно-лужної рівноваги викликав дані зміни?

Негазовий видільний алкалоз

Газовий алкалоз

Газовий ацидоз

Негазовий видільний ацидоз

Негазовий метаболічний ацидоз

3425 / 6854
На ізольованому серці шляхом охолодження припиняють функціонування окремих структур. Яку структуру охолодили, якщо серце внаслідок цього спочатку припинило скорочення, а далі відновило їх із частотою у 2 рази меншою за вихідну?

Волокна Пуркіньє

Синоатріальний вузол

Ніжки пучка Гіса

Пучок Гіса

Атріовентрикулярний вузол

3426 / 6854
У жінки 30-ти років хвилинний об’єм крові у стані спокою становить 5 л/хв. Який об’єм крові проходить у неї за 1 хвилину крізь судини легень?

1,5 л

5 л

2,5 л

2,0 л

3,75 л

3427 / 6854
Після перенесеного запального захворювання у хворого виникло неповне відведення очного яблука в латеральну сторону. Який нерв у хворого пошкоджений?

Зоровий

Окоруховий

Блоковий

Відвідний

Лицевий

3428 / 6854
У дитини, що померла від дифтерії з явищами серцевої недостатності, на розтині виявлено, що порожнини серця розширені в поперечнику; м’яз серця тьмяний, в’ялий, на розрізі строкатий, з жовтуватими ділянками. У цитоплазмі деяких кардіоміоцитів зі збереженою цитоплазмою спостерігаються дрібні вакуолі. На заморожених зрізах вакуолі забарвлюються у оранжевий колір суданом 3. Який вид дистрофії виявлений у кардіоміоцитах?

Балонна

Гідропічна

Гіаліново-крапельна

Вуглеводна

Жирова

3429 / 6854
У хворого на тимому (пухлину загрудинної залози) спостерігаються ціаноз, розширення підшкірної венозної сітки і набряк м’яких тканин обличчя, шиї, верхньої половини тулуба і верхніх кінцівок. Який венозний стовбур перетиснено пухлиною?

Підключична вена

Передня яремна вена

Верхня порожниста вена

Внутрішня яремна вена

Зовнішня яремна вена

3430 / 6854
Хворій 20-ти років в зв’язку з ревматизмом призначено тривалий прийом аспірину. Який структурний компонент слизової оболонки шлунку найбільшою мірою забезпечить її захист від ушкодження?

Багатошаровий плаский незроговілий епітелій

М’язова тканина

Одношаровий призматичний залозистий епітелій

Сполучна тканина

Багатошаровий війчастий епітелій

3431 / 6854
У хворої 36-ти років, яка лікувалася сульфаніламідами з приводу респіраторної вірусної інфекції, в крові спостерігаються гіпорегенераторна нормохромна анемія, лейкопенія, тромбоцитопенія. В кістковому мозку - зменшення кількості мiєлокарiоцитiв. Яка це анемія?

Постгеморагічна

Гіпопластична

Залізодефіцитна

Гемолітична

В 12-фолієводефіцитна

3432 / 6854
Чоловік 38-ми років раптово помер. На розтині: у задній стінці лівого шлуночка серця виявлено інфаркт міокарда. Які найбільш імовірні зміни у будові міокардіоцитів можна побачити у вогнищі інфаркту мікроскопічно?

Білкова дистрофія

Звапнування

Вуглеводна дистрофія

Каріолізис

Жирова дистрофія

3433 / 6854
У лікарню надійшла робітниця хімічного підприємства з ознаками отруєння. У волоссі цієї жінки знайдено підвищену концентрацію арсенату, який блокує ліпоєву кислоту. Вкажіть, порушення якого процесу є найімовірнішою причиною отруєння:

Знешкодження супероксидних іонів

Відновлення органічних перекисів

Відновлення метгемоглобіну

Окислювальне декарбоксилювання ПВК

Мікросомальне окиснення

3434 / 6854
Під час операції на головному мозку помічено, що подразнення певних зон кори великих півкуль викликало у хворого і тактильні і температурні відчуття. На яку звивину діяли подразники?

Постцентральна

Поясна

Прецентральна

Парагіпокампова

Верхня латеральна

3435 / 6854
З метою аналгезії можуть бути використані речовини, що імітують ефекти морфіну, але виробляються в ЦНС. Вкажіть таку речовину:

β-ендорфін

Вазопресин

Кальцитонін

Окситоцин

Соматоліберин

3436 / 6854
У хворого через 12 годин після гострого нападу загрудинного болю спостерігається різке підвищення активності АсАТ в сироватці крові. Вкажіть патологію, для якої характерне це зміщення:

Нецукровий діабет

Вірусний гепатит

Інфаркт міокарду

Колагеноз

Цукровий діабет

3437 / 6854
У дитини, що народилася від пізнього шлюбу, спостерігаються малий зріст, відставання у розумовому розвитку, товстий 'географічний' язик, вузькі очні щілини, пласке обличчя з широкими вилицями. Що найімовірніше спричинило розвиток вказаного синдрому?

Внутрішньоутробна інтоксикація

Пологова травма

Внутрішньоутробний імунний конфлікт

Спадкова хромосомна патологія

Внутрішньоутробна інфекція

3438 / 6854
Людина стоїть у кімнаті в легкому одязі; температура повітря +14°C. Вікна і двері зачинені. Яким шляхом організм людини віддає найбільше тепла?

Конвекція

Теплорадіація

Теплопроведення

Випаровування

Перспірація

3439 / 6854
До медико-генетичної консультації звернулася жінка за рекомендацією гінеколога з приводу відхилень фізичного і статевого розвитку. В ході мікроскопії клітин слизової оболонки ротової порожнини не знайдено статевого хроматину. Який найбільш імовірний діагноз?

Хвороба Реклінгаузена

Синдром Шерешевського-Тернера

Трисомія за Х-хромосомою

Синдром Клайнфельтера

Хвороба Дауна

3440 / 6854
35-річному хворому на атопічний дерматит був призначений лоратадин. Визначте механізм дії цього препарату:

Стимулює дофамінові рецептори

Стимулює М-холінорецептори

Блокує ГАМК-рецептори

Блокує Н1-гістамінові рецептори

Блокує β-адренорецептори

3441 / 6854
Розпочинається імплантація бластоцисти людини. Як називається період ембріогенезу, що розпочинається одночасно з імплантацією?

Дроблення

Інвагінація

Гаструляція

Диференціювання

Гістогенез

3442 / 6854
Лікар швидкої допомоги був викликаний до жінки 40-ка років з приводу нападу бронхіальної астми з явищами стенокардії, лікар ввів хворій належний препарат. Який із наведених препаратів найбільш ефективний для невідкладної допомоги?

Адреналін

Ефедрин

Платифілін

Сальбутамол

Атропін

3443 / 6854
Який препарат слід призначити хворому, у якого з’явилося безсоння внаслідок появи висипань алергічного характеру з почервонінням, набряком та сильним свербінням?

Нітразепам

Натрію оксибутират

Фенобарбітал

Хлоралгідрат

Димедрол

3444 / 6854
В дитячому садку проведені планові щеплення вакциною проти кору. Яким методом можна перевірити формування післявакцинального імунітету?

Серологічний

Алергічний

Бактеріологічний

Вірусологічний

Бактеріоскопічний

3445 / 6854
Після курсу терапії хворому на виразку дванадцятипалої кишки лікар пропонує вживати соки з капусти та картоплі. Вміст яких речовин в цих продуктах сприяє профілактиці та загоєнню виразок?

Вітамін K

Вітамін C

Вітамін Б

Пантотенова кислота

Вітамін U

3446 / 6854
У дитини з білявим волоссям, блідою шкірою відмічається збільшений тонус м’язів, судоми та розумова відсталість. В крові підвищений рівень фенілаланіну. Який з перелічених методів необхідно застосувати для встановлення діагнозу цієї ензимопатії?

Біохімічний

Популяційно-статистичний

Електрофізіологічний

Цитогенетичниий

Генеалогічний

3447 / 6854
Хворому з больовим синдромом в суглобах постійно призначають аспірин. Який з перерахованих ферментів він пригнічує?

Фосфоліпаза D

Фосфоліпаза С

Фосфоліпаза А2

Циклооксигеназа

Ліпооксигеназа

3448 / 6854
Робота шахтарів у забої часто спричинює антракоз. Який вид дихальної недостатності може розвинутися при цьому?

Діафрагмальний

Обструктивний

Дисрегуляторний

Рестриктивний

Торакальний

3449 / 6854
До гінекологічного відділення госпіталізована жінка з клінікою гострого живота, припускається позаматкова вагітність. Крізь яке анатомічне утворення або його частину гінеколог буде виконувати пункцію черевної порожнини?

Rectum

Cervix uteri

Fornix vaginae posterior

Fornix vaginae anterior

3450 / 6854
У дитини 6-ти місяців спостерігається різке відставання в психомоторному розвитку, бліда шкіра з екзематозними змінами, біляве волосся, блакитні очі, напади судом. Найточніше встановити діагноз у цієї дитини дозволить визначення в крові і сечі концентрації такої речовини:

Валін

Лейцин

Триптофан

Гістидин

Фенілпіруват

3451 / 6854
У хворого діагностували легеневу форму туберкульозу. Який з наведених лікарський засіб використовують для лікування туберкульозу?

Норсульфазол

Ізоніазид

Тетрациклін

Пеніцилін

Фуразолідон

3452 / 6854
Дитина доставлена в санпропускник в стані асфіксії. Під час огляду в гортані виявлені білуваті плівки, що обтурують просвіт та легко видаляються. Лікар припускає дифтерію. Про яке запалення гортані йдеться?

Гнійне

Крупозне

Дифтеритичне

Серозне

Катаральне

3453 / 6854
В бактеріологічній лабораторії досліджувалися консерви, які спричинили тяжку токсикоінфекцію. В результаті мікроскопії культури з середовища Кітта-Тароцці виділені грампозитивні палички, схожі на тенісну ракетку. Який діагноз встановить лікар?

Ботулізм

Хламідіоз

Черевний тиф

Дизентерія

Туляремія

3454 / 6854
Хворому на глаукому призначили пілокарпіну гідрохлорид в очних краплях. До якої фармакологічної групи належить цей препарат?

α-адреноблокатори

Міорелаксанти

М-холіноблокатори

Гангліоблокатори

М-холіноміметики

3455 / 6854
Під час автомобільної аварії людина зазнала сильного удару в епігастральну ділянку, внаслідок чого виникла зупинка серця. Що могло стати причиною таких змін серцевої діяльності?

Підвищене виділення адреналіну

Підвищене виділення кортизолу

Підвищення тонусу блукаючого нерва

Підвищення тонусу симпатичної нервової системи

Підвищене виділення альдостерону

3456 / 6854
До лікаря-інфекціоніста на прийом прийшов хворий зі скаргами на лихоманку, яка триває три дні, загальну слабкість, безсоння, погіршення апетиту. Лікар припускає черевний тиф. Для підтвердження діагнозу найдоцільніше призначити виділення:

Уринокультури

Білікультури

Мієлокультури

Копрокультури

Гемокультури

3457 / 6854
У здорової дорослої людини швидкість проведення збудження через атріовентрикулярний вузол дорівнює 0,02-0,05 м/с. Атріовентрикулярна затримка забезпечує:

Одночасність скорочення обох шлуночків

Послідовність скорочення передсердь та шлуночків

Достатню силу скорочення шлуночків

Достатню силу скорочення передсердь

Одночасність скорочення обох передсердь

3458 / 6854
В клітині, яка мітотично ділиться, спостерігається розходження дочірніх хроматид до полюсів клітини. На якій стадії мітотичного циклу перебуває клітина?

Анафаза

Метафаза

Профаза

Телофаза

Інтерфаза

3459 / 6854
На розтині тіла померлого виявлено, що вся права легеня збільшена, щільна, на плеврі нашарування фібрину, на розрізі тканина сірого кольору, з якої стікає каламутна рідина. Для якого захворювання легенів характерна така картина?

Гангрена легені

Інтерстиціальна пневмонія

Крупозна пневмонія

Фіброзувальний альвеоліт

Вогнищева пневмонія

3460 / 6854
Вкажіть антибіотик, що утворює хелатні сполуки з іонами кальцію, заліза, алюмінію, які не абсорбуються з тонкого кишківника:

Ампіциліну натрієва сіль

Гентаміцину сульфат

Доксицикліну гідрохлорид

Амоксиклав

Хіноксидин

3461 / 6854
Аналіз крові хворого на цукровий діабет показав наявність молочної кислоти у концентрації 2,5 ммоль/л. Яка кома розвинулася у хворого?

Гіперкетонемічна

Гіпоглікемічна

Гіперглікемічна

Гіперосмолярна

Лактацидемічна

3462 / 6854
Підвищення внутрішньочерепного тиску у хворого з церебральною гематомою обумовило надмірну активність блукаючого нерва (ваготонію) та зміну частоти серцевих скорочень. Який вид аритмії серця виникає при цьому?

Пароксизмальна тахікардія

Шлуночкова екстрасистолія

Синусова брадикардія

Синусова тахікардія

Передсердно-шлуночкова блокада

3463 / 6854
В ході обстеження людини необхідно визначити, яка частка альвеолярного повітря оновлюється під час кожного вдиху. Який з наведених показників необхідно розрахувати для цього?

Життєва ємність легень

Функціональна залишкова ємність легень

Хвилинний об’єм дихання

Коефіцієнт легеневої вентиляції

Хвилинна альвеолярна вентиляція

3464 / 6854
У людини необхідно оцінити стан клапанів серця. Яким з інструментальних методів дослідження доцільно скористатися для цього?

Сфігмографія

Фонокардіографія

Електрокардіографія

Флебографія

Зондування судин

3465 / 6854
Швидкою допомогою в приймальне відділення доставлений хворий з кривавим блюванням. В анамнезі цироз печінки. Пошкодження яких вен найбільш ймовірне вданому випадку?

Печінкові

Верхні брижові

Нижні брижові

Селезінкові

Стравохідні

3466 / 6854
Ліквідатору наслідків аварії на Чорнобильській АЕС, що отримав велику дозу опромінення, проведено трансплантацію кісткового мозку. Через деякий час після проведеної операції у пацієнта діагностовано розвиток реакції 'трансплантат проти хазяїна'. Які антигени стали пусковим механізмом виникнення цієї реакції?

Антигени системи HLA-клітин організму донора

Антигени системи Rh еритроцитів ліквідатора

Антигенами системи AB0 еритроцитів ліквідатора

Антигени системи HLA-клітин організму ліквідатора

Антигени HBs, HBc, Hbe

3467 / 6854
В експерименті ізольований м’яз жаби ритмічно подразнюють електричними імпульсами. Кожний наступний імпульс припадає на період розслаблення попереднього скорочення. Яке скорочення виникає?

Зубчастий тетанус

Одиночне

Тонічне

Суцільний тетанус

Асинхронне

3468 / 6854
Під час розтину тіла мертвонародженої дитини виявлено аномалію розвитку серця: шлуночки не розмежовані, з правої частини виходить суцільний артеріальний стовбур. Для яких хребетних характерна подібна будова серця?

Риби

Амфібії

Рептилії

Ссавці

Птахи

3469 / 6854
Хворому на сепсис призначили антимікробний препарат з групи фторхінолонів. Оберіть його серед наведених препаратів:

Метронідазол

Цефалексин

Цефпіром

Ампіцилін

Ципрофлоксацин

3470 / 6854
Під дією медіатора на постсинаптичну мембрану нервової клітини розвинулася гіперполяризація. Збільшення проникності мембрани для яких іонів може викликати такі зміни?

Натрію

Магнію

Натрію і калію

Калію

Кальцію

3471 / 6854
Щоденно в організмі людини 0,5% всього гемоглобіну перетворюється на метгемоглобін. Який фермент, що міститься в еритроцитах, каталізує відновлення метгемоглобіну до гемоглобіну?

Білівердинредуктаза

Метгемоглобінтрансфераза

Ілюкуронілтрансфераза

Гемоксигеназа

Метгемоглобінредуктаза

3472 / 6854
У хворого спостерігається гемералопія (куряча сліпота). Яка з перерахованих речовин матиме лікувальну дію?

Кератин

Креатин

Карнітин

Каротин

Карнозин

3473 / 6854
У хворого з серцевою недостатністю виникла аритмія у вигляді генерації позачергових імпульсів в пучку Гіса. Порушення якої функції серцевого м’язу спостерігається в даному випадку?

Збудливість

Скоротливість

Збудливість та провідність

Автоматизм

Провідність

3474 / 6854
Хворий 50-ти років страждає на гіпертонічну хворобу. Під час фізичного навантаження у нього з’явилося відчуття слабкості, нестачі повітря, синюшність слизової оболонки губ, шкіри обличчя. Дихання супроводжувалося чутними на відстані вологими хрипами. Який механізм лежить в основі виникнення такого синдрому?

Хронічна лівошлуночкова недостатність

Хронічна правошлуночкова недостатність

Тампонада серця

Гостра лівошлуночкова недостатність

Колапс

3475 / 6854
На слизовій оболонці правого піднебінного мигдалика спостерігається безболісна виразка з гладеньким лакованим дном та рівними краями хрящеподібної консистенції. Мікроскопічно: запальний інфільтрат, що складається з лімфоцитів, плазмоцитів, невеликої кількості нейтрофілів та епітеліоїдних клітин, наявність ендо- та периваскуліту. Про яке захворювання йдеться?

Скарлатина

Сифіліс

Туберкульоз

Актиномікоз

Дифтерія зіву

3476 / 6854
У юнака 20-ти років діагностовано спадковий дефіцит УДФ-глюкуронілтрансферази. Підвищення якого показника крові підтверджує цей діагноз?

Непрямий (некон’югований) білірубін

Тваринний індикан

Прямий (кон’югований) білірубін

Уробілін

Стеркобіліноген

3477 / 6854
Хворому 50-ти років з хронічною серцевою недостатністю і тахіаритмією призначили кардіотонічний препарат. Який з препаратів призначили хворому?

Дофамін

Добутамін

Дигоксин

Мілдронат

Аміодарон

3478 / 6854
Офтальмолог з діагностичною метою (розширення зіниць для огляду очного дна) використав 1% розчин мезатону. Мідріаз, викликаний препаратом, обумовлений:

Блокадою α1-адренорецепторів

Активацією β1-адренорецепторів

Активацією α1-адренорецепторів

Активацією М-холінорецепторів

Активацією α2-адренорецепторів

3479 / 6854
Чоловік 55-ти років, що скаржиться на біль в ділянці нирок, надійшов в лікарню. В ході ультразвукового обстеження пацієнта виявлено наявність ниркових каменів. Наявність в сечі якої з наведених речовин є найбільш імовірною причиною утворення каменів уданого пацієнта?

Білірубін

Креатинін

Сечова кислота

Уробілін

Білівердин

3480 / 6854
В ході визначення енерговитрат організму людини встановлено, що дихальний коефіцієнт дорівнює 1,0. Це означає, що у клітинах досліджуваного переважно окиснюються:

Вуглеводи

Білки

Білки і вуглеводи

Жири

Вуглеводи та жири

3481 / 6854
Чоловік 43-х років доставлений у лікарню з ознаками черевного тифу. Хворий нещодавно був у місцевості, епідемічній за даним захворюванням, де пив некип’ячену воду з колодязя. З моменту появи перших симптомів пройшло 3 дні. У цей період збудник може бути виявлений у:

Лімфоїдній тканині кишківника

Шлунку

Тонкій кишці

Крові

Жовчному міхурі

3482 / 6854
У хворого запалення присереднього надвиростка плечової кістки (епікондиліт). Який нерв залучений у процес?

N. medianus

N. musculocutaneus

N. ulnaris

N. axillaris

N. radialis

3483 / 6854
У холодну погоду з вітром люди мерзнуть швидше, ніж за відсутності вітру. Причиною цього є те, що вітер збільшує, насамперед, віддачу тепла таким шляхом:

Конвекція

Радіація

Випаровування

Теплопроведення

3484 / 6854
Хворий помер від інтоксикації на 4-у добу після вживання сирих яєць. На розтині: слизова оболонка шлунка і тонкої кишки запалена, вкрита слизовим ексудатом; в легенях, головному мозку і печінці знайдені абсцеси. Який діагноз найбільш імовірний?

Сальмонельоз (інтестинальна форма)

Дизентерія

Сальмонельоз (черевнотифозна форма)

Черевний тиф

Сальмонельоз (септична форма)

3485 / 6854
Під час визначення групової належності крові за системою АВ0 аглютинацію еритроцитів досліджуваної крові викликали стандартні сироватки I та II груп та не викликала сироватка III групи. Якою є група крові?

А (II) бета

0 (I) альфа, бета

В (III) альфа

АВ (IV)

Неможливо визначити

3486 / 6854
На мікропрепараті серця розрізняються клітини, які розташовані у вигляді світлих тяжів, мають невелику кількість міофібрил, включення глікогену та ексцентрично локалізоване ядро. Які це клітини?

Скоротливі

Волокна Пуркіньє

Провідні пейсмекерні

Ендокринні

Провідні перехідні

3487 / 6854
Чоловік 40-ка років скаржиться на загальну слабкість, головний біль, кашель з виділенням мокротиння, задишку. Після клінічного огляду й обстеження поставлено діагноз: пневмонія. Який тип гіпоксії має місце у хворого?

Ішемічна

Респіраторна

Циркуляторна

Гіпоксична

Тканинна

3488 / 6854
До хірурга звернувся чоловік 60-ти років, що тривалий час хворів на цукровий діабет. Тканини правої стопи були чорного кольору, щільні, з чіткими краями. Який діагноз поставив хірург?

Пролежень

Іазова гангрена

Волога гангрена

Трофічна виразка

Суха гангрена

3489 / 6854
У жінки 52-х років артеріальна гіпертензія ускладнилася правобічною геміплегією і втратою мови. Яка зона головного мозку є імовірно найбільш ураженою?

Ліва передня звивина

Права передня звивина

Ліва передня звивина і ліва скронева частка

Ліва скронева частка

Потилична частка

3490 / 6854
Хворий 62-х років блідий, всі групи лімфовузлів збільшені. В крові: Hb- 60 г/л, еритроцити - 1,9 Т/л, лейкоцити - 29 Г/л, тромбоцити - 110 Г/л. Лейкоцитарна формула: сегментоядерні лейкоцити - 10%, лімфоцити - 8%, моноцити - 2%, бластних клітин - 80%. Цитохімічні дослідження бластних клітин: позитивна реакція на глікоген, негативна – на ліпіди і пероксидазу. Дайте заключення про патологію:

Гострий монобластний лейкоз

Гострий мегакаріоцитарний лейкоз

Гострий мієлобластний лейкоз

Гострий промієлоцитарний лейкоз

Гострий лімфобластний лейкоз

3491 / 6854
До лікарні надійшов пацієнт з перфоративною виразкою задньої стінки шлунка. Який елемент очеревини хірург під час операції повинен ретельно обстежити?

Права брижова пазуха

Передшлункова сумка

Чепцева сумка

Лівий бічний канал

Печінкова сумка

3492 / 6854
У померлого від інтоксикації чоловіка під час розтину тіла виявлено збільшення стегна в об’ємі; на поверхні шкіри нориці з виділенням в’язкої жовто-зеленої рідини; стегнова кістка потовщена та деформована, має місце утворення секвестрів. При мікроскопічному дослідженні: секвестральні порожнини оточені грануляційною та сполучною тканиною з наявністю нейтрофілів, кістково-мозкові канали облітеровані, компактна пластинка потовщена. Який з перерахованих діагнозів найбільш імовірний?

Остеопороз

Гострий гематогенний остеомієліт

Хронічний гнійний остеомієліт

Паратиреоїдна остеодистрофія

Туберкульозний остеомієліт

3493 / 6854
Хворому на хронічний бронхіт призначили муколітичний препарат, що підвищує синтез сурфактанту. Який препарат був використаний?

Лібексин

Кодеїн

Фенотерол

Амброксол

Мукалтин

3494 / 6854
Внаслідок травми низу передньої стінки живота у хворої ушкоджена зв’язка, що розташована у пахвинному каналі. Яка це зв’язка?

Ligamentum inguinale

Ligamentum ovarium proprium

Ligamentum lacunare

Ligamentum teres uteri

Ligamentum latum uteri

3495 / 6854
Після перенесеного ГРЗ у хворої спостерігається утруднене носове дихання, підвищена температура, головний біль, сльозовиділення, болючість під час пальпації в ділянці fossa canina справа. Запаленням якої пазухи ускладнилося захворювання?

Sinus frontalis dexter

Sinus sphenoidalis dexter

Cellulae ethmoidales posteriores

Cellulae ethmoidales anteriores

Sinus maxillaris dexter

3496 / 6854
Під час профілактичного огляду у деяких робітників, що працюють на виробництві кам’яновугільних смол, у ротовій порожнині виявлені ділянки стовщення і зроговіння слизової оболонки, переважно щік, білястого кольору, з шорсткою поверхнею, безболісні. Про яку патологію йдеться?

Глосит

Папіломатоз

Лейкоплакія

Стоматит

3497 / 6854
У хворого на рентгенограмі легень виявлено затемнення. В ході діагностичної експрес-біопсії лімфатичного вузла бронха виявлено казеозний некроз, навколо якого розташовані епітеліоїдні та лімфоїдні пласти з домішками багатоядерних гігантських клітин. Вкажіть причину лімфаденіту:

Туберкульоз

Пневмонія

Сифіліс

Метастази раку

Аденовірусна інфекція

3498 / 6854
На розтині тіла померлого в підкоркових ядрах правої півкулі головного мозку спостерігається порожнина неправильної форми 5х3,5 см, заповнена червоними згустками крові та розм’якшеною тканиною мозку. Назвіть патологію, яка розвинулася у головному мозку:

Абсцес

Гематома

Ішемічний інфаркт

Геморагічне просякнення

Астроцитома

3499 / 6854
У хворого діагностовано пухлину мозку, яка розміщена в ділянці 'пташиної шпори. Порушення якої функції розвинеться у хворого, якщо пухлина буде активно розвиватися?

Зір

Нюх

Слух

Дотикова чутливість

Смак

3500 / 6854
Хворому 35-ти років для обстеження очного дна був призначений атропіну сульфат у вигляді очних крапель. Для відновлення акомодації йому закрапали пілокарпіну гідрохлорид, але це не дало бажаного ефекту. Що є причиною відсутності ефекту?

Тахіфілаксія

Синергізм

Звикання

Односторонній антагонізм

Двосторонній антагонізм

3501 / 6854
Хвора 38-ми років померла під час нападу бронхіальної астми, який не вдалося зняти. В ході гістологічного дослідження у просвіті бронхів виявлені накопичення слизу, в стінці бронхів численні лаброцити, багато з них у стані дегрануляції, а також велика кількість еозинофілів. Який патогенез (механізм розвитку) цих змін у бронхах?

Атопія

Імунокомплексний механізм

Клітинно обумовлений цитоліз

Гранулематоз

Цитотоксична, цитолітична дія антитіл

3502 / 6854
У хворого 23-х років після перенесеної ангіни розвинувся сечовий синдром (гематурія, протеїнурія, лейкоцитурія). В пункційній біопсії нирок виявлена картина інтракапілярного проліферативного гломерулонефриту, а електронномікроскопічно виявлені великі субепітеліальні депозити. Який патогенез цього захворювання?

Імунокомплексний механізм

Гранулематоз

Клітинно обумовлений цитоліз

Цитотоксична, цитолітична дія антитіл

Атопія

3503 / 6854
У хворого, який на тлі атеросклерозу переніс ішемічний інсульт, спостерігається порушення рухової функції у вигляді геміплегії. Яка з перерахованих ознак є характерною для уражених кінцівок при даній патології?

Гіпертонус м’язів

Трофічні розлади

Гіпотонус м’язів

Гіпорефлексія

3504 / 6854
Через три тижні після пересадки серця у пацієнта різко погіршився стан. Смерть настала від гострої серцевої недостатності. Гістологічно в міокарді виявлено васкуліт, некрози та лімфоцитарну інфільтрацію. Назвіть причину таких змін:

Найгостріше відторгнення трансплантату

Некротизуючий васкуліт

Хронічне відторгнення трансплантату

Інфаркт міокарда

Гостре відторгнення трансплантату

3505 / 6854
У хворого з флегмоною передпліччя в ході мікробіологічного аналізу ексудату в зоні запалення виявлена присутність стрептококів. Які клітини будуть переважати в ексудаті?

Моноцити

Базофільні гранулоцити

Еозинофільні гранулоцити

Нейтрофільні гранулоцити

Лімфоцити

3506 / 6854
Важливим джерелом утворення аміаку в головному мозку є дезамінування АМФ. Яка амінокислота відіграє основну роль у зв’язуванні аміаку в нервовій тканині?

Аланін

Глутамат

Аргінін

Ізолейцин

Лізин

3507 / 6854
У хворої 19-ти років з дитинства спостерігалося зниження гемоглобіну до 9095 г/л. Аналіз крові під час госпіталізації: еритроцити - 3,2 • 1012/л, гемоглобін - 85 г/л, КП- 0,78;лейкоцити - 5,6 • 109/л, тромбоцити - 210 • 109/л. В мазку: анізоцитоз, пойкілоцитоз, мішенеподібні еритроцити. Ретикулоцити - 6%. Лікування препаратами заліза не ефективне. Яку патологію системи крові можна припустити в даному випадку?

Фавізм

Серпоподібноклітинна анемія

Ферментопатія

Мембранопатія

Таласемія

3508 / 6854
Жінці 50-ти років, яка хворіє на тромбофлебіт, ввели гепарин, що спровокував кишкову кровотечу. Який препарат потрібно призначити?

Вікасол

Стрептокіназу

Натрію цитрат

Кислоту амінокапронову

Протаміну сульфат

3509 / 6854
До хірургічного відділення ЦРЛ надійшов хворий з колотою раною стопи, яку він отримав під час косовиці. Який специфічний препарат необхідно застосувати з метою екстреної пасивної імунопрофілактики правця?

Анатоксин

Антитоксична сироватка

Антибіотики

Протиправцева вакцина

Вакцина АКДП

3510 / 6854
Під час серцевого нападу чоловік в автобусі втратив свідомість, з’явилися судоми. Лікар швидкої допомоги виявив на ЕКГ, що частота скорочення передсердь перевищує частоту скорочення шлуночків. Що може бути причиною даного стану?

Порушення автоматії СА-вузла

Повна поперечна блокада проведення збудження

Порушення автоматії АВ-вузла

Порушення проведення збудження між передсердями

Виникнення гетеротропних вогнищ збудження

3511 / 6854
На розтині тіла померлого, який страждав на гіпертонічну хворобу, у лівій півкулі мозку виявлена порожнина округлої форми 4х5 см з іржавою стінкою, заповнена жовтуватою прозорою рідиною. Назвіть патологію, яка розвинулася у головному мозку хворого:

Кіста

Гематома

Абсцес

Геморагічне просякнення

Ішемічний інфаркт

3512 / 6854
Під час постсинтетичного періоду мітотичного циклу було порушено синтез білків тубулінів. До яких наслідків це може призвести?

Порушення формування веретена поділу

Порушення цитокінезу

Скорочення тривалості мітозу

Порушення репарації ДНК

Порушення спіралізації хромосом

3513 / 6854
У хворого 40-ка років, доставленого в травматологічне відділення, виявлено поранення правої кисті на долонній поверхні. Під час обстеження травмований не може самостійно звести розведені II, IV та V пальці. Які м’язи пошкоджені?

Долонні міжкісткові

Короткий долонний

Довгий долонний

Червоподібні

Тильні міжкісткові

3514 / 6854
На аутопсії тіла жінки, що хворіла на хронічну дизентерію, в ході гістологічного дослідження внутрішніх органів в стромі та паренхімі міокарда, нирок, в слизовій оболонці шлунка та в сполучній тканині легень виявлені аморфні відкладення фіолетового кольору, що дають позитивну реакцію за Коссом. Яке ускладнення розвинулося у хворої?

Амілоїдоз

Дистрофічне звапніння

Метаболічне звапніння

Метастатичне звапніння

Гіаліноз

3515 / 6854
У хворого з патологією серцево-судинної системи розвинулися набряки на нижніх кінцівках. Який механізм розвитку серцевого набряку?

Зниження осмотичного тиску плазми крові

Порушення лімфовідтоку

Підвищення гідростатичного тиску в венулах

Підвищення онкотичного тиску плазми крові

Підвищення гідростатичного тиску в артеріолах

3516 / 6854
Під час проведення хірургічних маніпуляцій було використано новокаїн з метою знеболення. Через 10 хвилин у хворого з’явилися блідість шкірних покривів, задишка, гіпотензія. Яку алергічну реакцію можна припустити?

Анафілактична

Цитотоксична

Стимулююча

Імунокомплексна

Клітинно-опосередкована

3517 / 6854
У 19-місячної дитини із затримкою розвитку та проявами самоагресії, вміст сечової кислоти в крові - 1,96 ммоль/л. При якому метаболічному порушенні це спостерігається?

Синдром Леша-Ніхана

Подагра

Хвороба Іценко-Кушинга

Хвороба Гірке

Синдром набутого імунодефіциту

3518 / 6854
Дитина 9-ти місяців харчується сумішами, незбалансованими за вмістом вітаміну В6. У дитини спостерігається пелагроподібний дерматит, судоми, анемія. Розвиток судом може бути пов’язаний з дефіцитом утворення:

Гістаміну

ГАМК

Серотоніну

ДОФА

Дофаміну

3519 / 6854
У хлопчика 12-ти років видалено апендикс, який надіслано патологу на дослідження. Макроскопічно: апендикс в дистальному відділі з булавоподібним потовщенням діаметром 3 см, при розрізі якого вилилася прозора жовтувата рідина, стінка апендикса стоншена. Мікроскопічно: атрофія всіх шарів апендикса, ознак запалення немає. Який найбільш імовірний діагноз?

Водянка апендикса

Флегмонозний апендицит

Емпієма апендикса

Хронічний апендицит

Міксоглобульоз апендикса

3520 / 6854
У дитини 7-ми років, яка неодноразово хворіла на стрептококову ангіну, лікар припускає розвиток ревматизму. Призначено серологічне дослідження. Наявність антитіл до якого з стрептококових антигенів найімовірніше підтвердить передбачуваний діагноз?

М-білок

С-вуглевод

Капсульний полісахарид

Еритрогенний токсин

О-стрептолізин

3521 / 6854
У хворого з попереднім діагнозом 'озена' з носоглотки були виділені грам-негативні палички, які утворювали капсулу на поживному середовищі. Які мікроорганізми спричинили хворобу?

Хламідії

Клебсієли

Сальмонели

Шигели

Мікоплазми

3522 / 6854
У зв’язку з крововтратою пацієнту введено 1 л розчину хлориду натрію з концентрацією 150 ммоль/л. Внаслідок цього, насамперед, зменшиться:

Онкотичний тиск крові

Онкотичний тиск міжклітинної рідини

Осмотичний тиск внутрішньоклітинний

Осмотичний тиск крові

Осмотичний тиск міжклітинної рідини

3523 / 6854
У хворого кровотеча з поперечної ободової кишки. Чим кровопостачається цей відділ товстої кишки?

Arteria lienalis

Arteria mesenterica inferior

Arteria ileocolica

Arteria colica media

Arteria sigmoidea

3524 / 6854
Чоловік протягом 3-х років працював в одній з африканських країн. Після переїзду до України звернувся до офтальмолога зі скаргами на біль в очах, набряки повік, сльозоточивість і тимчасове послаблення зору. Під кон’юнктивою ока були виявлені гельмінти розмірами 30-50 мм, які мали видовжене ниткоподібне тіло. Який діагноз може поставити лікар?

Філяріоз

Трихоцефальоз

Аскаридоз

Дифілоботріоз

Ентеробіоз

3525 / 6854
Хворому призначена ендоскопія 12-палої кишки. В результаті виявлено запалення великого дуоденального сосочка і порушення виділення жовчі в просвіт кишки. У якому відділі 12-палої кишки виявлені порушення?

Висхідна частина

Верхня частина

Цибулина

Низхідна частина

Горизонтальна частина

3526 / 6854
Чоловік з гострим міокардитом помер від серцево-судинної недостатності. В ході мікроскопічного дослідження внутрішніх органів виявлені: плазморагія, набряк, стази в капілярах, численні крововиливи, а також дистрофічні зміни в паренхімі. Наслідком чого є дані зміни?

ДВС-синдром

Місцеве артеріальне повнокрів’я

Гострий загальний венозний застій

Загальне артеріальне повнокрів’я

Хронічний загальний венозний застій

3527 / 6854
Хворому на гепатит для попередження уражень печінки призначили вітаміноподібну речовину холін. Його лікувальний ефект пов’язаний з:

Пригніченням синтезу холестерину

Активацією глікогенсинтази

Активацією глікогенфосфорилази

Пригніченням синтезу ацетонових тіл

Ліпотропною дією

3528 / 6854
Дослідженнями останніх десятиліть встановлено, що безпосередніми 'виконавцями' апоптозу в клітині є особливі ферменти - каспази. В утворенні одного з них бере участь цитохром С. Вкажіть його функцію в нормальній клітині:

Фермент в-окиснення жирних кислот

Компонент піруватдегідрогеназної системи

Компонент H + - АТФ-азної системи

Фермент дихального ланцюга переносу електронів

Фермент ЦТК

3529 / 6854
Процес біосинтезу білка є енергозалежним. Вкажіть, який макроергічний субстрат безпосередньо використовується в цьому процесі на стадії елонгації:

АДФ

АТФ

ГТФ

УТФ

ЦТФ

3530 / 6854
З метою профілактики гепатиту В групі стоматологів ввели вакцину, яка являє собою генно-інженерний HBs-антиген. Від якого ще інфекційного агента захищає таке щеплення?

Вірус імунодефіциту людини

Вірус грипу, тип В

Вірус Коксакі, група В

Вірус гепатиту С

Вірус дельта

3531 / 6854
У хворого з ознаками інтоксикації та ниркової недостатності виявлені у сечі рухливі мікроорганізми з численними дрібними завитками, які забарвилися за Романовським-Гімзою у рожевий колір. З анамнезу відомо, що хворий кілька днів тому купався у відкритій водоймі. Яке захворювання можна припустити?

Туберкульоз

Псевдотуберкульоз

Сифіліс

Бруцельоз

Лептоспіроз

3532 / 6854
У жінки під час мейозу відбулося порушення розходження аутосом. Утворилася яйцеклітина з зайвою 18-ю хромосомою. Яйцеклітина запліднюється нормальним сперматозооном. У майбутньої дитини буде синдром:

Патау

Шерешевського-Тернера

Едвардса

Клайнфельтера

Дауна

3533 / 6854
Для виявлення джерела інфікування хворих у хірургічному відділенні патогенним стафілококом було проведено дослідження з застосуванням стандартних стафілококових бактеріофагів. Яке дослідження чистих культур, виділених із різних джерел, було проведено?

Фагоідентифікація

Фагоіндикація

Фаготипування

Фагодіагностика

Фаготерапія

3534 / 6854
У новонародженої дитини спостерігається зниження інтенсивності смоктання, часте блювання, гіпотонія. У сечі та крові значно підвищена концентрація цитруліну. Який метаболічний процес порушений?

Цикл Корі

Орнітиновий цикл

Гліколіз

ЦТК

Глюконеогенез

3535 / 6854
Після споживання солоної їжі у людини значно зменшилася кількість сечі. Підвищена секреція якого гормону призвела до зменшення діурезу?

Вазопресин

Ренін

Альдостерон

Ангіотензин-II

Натрійуретичний

3536 / 6854
У результаті радіаційного випромінювання були ушкоджені стовбурові гемопоетичні клітини. Утворення яких клітин сполучної тканини буде порушено?

Макрофаги

Перицити

Фібробласти

Меланоцити

Адипоцити

3537 / 6854
Через 8 днів після хірургічної операції у пацієнта розвинувся правець. Лікар припускає, що причиною став контамінований збудником правця шовний матеріал, який був доставлений в бактеріологічну лабораторію. Яке поживне середовище необхідно використовувати для первинного посіву шовного матеріалу?

ЖСА

Сабуро

Ендо

Кітта-Тароцці

Гіса

3538 / 6854
У пацієнта має місце пошкодження волокон дев’ятої пари черепних нервів (язикоглотковий нерв). Формування якого відчуття буде порушено?

Усі смакові відчуття

Солодке

Кисле

Солоне

Гірке

3539 / 6854
Чоловік 70-ти років хворіє на атеросклероз судин нижніх кінцівок та ішемічну хворобу серця. Під час обстеження виявлено порушення ліпідного складу крові. Надлишок яких ліпопротеїнів є головною ланкою в патогенезі атеросклерозу?

Проміжної щільності

Низької щільності

Високої щільності

Холестерину

Хіломікронів

3540 / 6854
У хворого на есенціальну артеріальну гіпертензію розвинувся гіпертонічний криз, що призвело до нападу серцевої астми. Який механізм серцевої недостатності є провідним вданому випадку?

Абсолютна коронарна недостатність

Перевантаження серця збільшеним об’ємом крові

Порушення надходження крові до серця

Перевантаження серця підвищеним опором

Пошкодження міокарда

3541 / 6854
Стресовий стан і больове відчуття у пацієнта перед візитом до стоматолога супроводжуються анурією (відсутністю сечовиділення). Це явище зумовлене збільшенням:

Секреції вазопресину та зменшенням адреналіну

Активності парасимпатичної нервової системи

Секреції адреналіну та зменшенням вазопресину

Активності антиноціцептивної системи

Секреції вазопресину та адреналіну

3542 / 6854
У хворого під час прийому їжі виникла асфіксія внаслідок закупорки трахеї стороннім тілом. Яка форма порушення зовнішнього дихання спостерігається у хворого?

Дифузно-пневмонозна

Дифузно-рестриктивна

Первинно дискінетична

Обструктивна

Вентиляційно-рестриктивна

3543 / 6854
Під час дослідження епітелію шкіри з’ясувалося, що він складається з кількох шарів клітин. Епітеліоцити зовнішнього шару не мають ядер. Який це епітелій?

Багатошаровий плаский зроговілий

Багаторядний війчастий

Багатошаровий плаский незроговілий

Багатошаровий кубічний

Перехідний

3544 / 6854
Введення пацієнту знеболювального перед екстракцією зуба призвело до розвитку анафілактичного шоку, який супроводжувався розвитком олігурії. Який патогенетичний механізм зумовив зменшення діурезу в даній клінічній ситуації?

Пошкодження клубочкового фільтру

Збільшення онкотичного тиску крові

Зниження гідростатичного тиску в капілярах клубочків

Зменшення кількості функціонуючих нефронів

Підвищення гідростатичного тиску в капсулі Шумлянського-Боумена

3545 / 6854
Після видалення зуба у пацієнта виникла кровотеча. Аналіз крові виявив зниження протромбінового індексу. Дефіцит якого вітаміну може бути причиною такого стану?

K

D

B

C

A

3546 / 6854
У пацієнтів для оцінки ефективності дихання використовують показник функціональної залишкової ємності. З яких наступних об’ємів вона складається?

Резервний об’єм вдиху, дихальний, залишковий

Резервний об’єм видиху та дихальний

Резервний об’єм вдиху та залишковий

Резервний об’єм вдиху та дихальний

Резервний об’єм видиху та залишковий

3547 / 6854
В хронічному експерименті на щурах стимулювали електричним струмом паравентрикулярні та супраоптичні ядра гіпоталамуса. Яка поведінкова реакція спостерігалася у тварин?

Збільшення споживання води

Збільшення споживання їжі

Зменшення споживання їжі

Відмова від їжі та рідини

Зменшення споживання води

3548 / 6854
Після обстеження пацієнта в клініці нервових хвороб встановлена відсутність звуження зіниці під дією світла. З ураженням яких структур головного мозку це пов’язано?

Вегетативні ядра 3 пари черепно-мозкових нервів

Ядра гіпоталамуса

Ретикулярні ядра середнього мозку

Червоні ядра середнього мозку

Ретикулярні ядра довгастого мозку

3549 / 6854
Аспірин інгібує синтез простагландинів, завдяки блокуванню активності циклооксигенази. Яка жирна кислота необхідна для цього синтезу?

Пальмітинова

Арахідонова

Лінолева

Стеаринова

Ліноленова

3550 / 6854
Лікар встановив у хворого наявність ураження парасимпатичного ядра IX пари черепних нервів. Яке ядро уражене в даному випадку?

Nucleus salivatorius inferior

Nucleus thoracicus

Nucleus salivatorius superior

Nucleus ambiguus

Nucleus tractus solitarii

3551 / 6854
Хворому лікар призначив протикашльовий препарат центральної дії, який є алкалоїдом мачку жовтого. Діє на кашльовий центр вибірково, не пригнічує дихання, не затримує виділення харкотиння. Не викликає обстипації і лікарської залежності. Можна призначати дітям. Визначте цей препарат:

Бромгексин

Глауцину гідрохлорид

Лібексин

Кодеїну фосфат

Окселадин

3552 / 6854
У хворого після резекції шлунка з приводу виразкової хвороби перистальтика кишківника не відновилася. Який лікарський засіб доцільно призначити хворому для відновлення моторики?

Атенолол

Гігроній

Метилурацил

Прозерин

Резерпін

3553 / 6854
Коли людина проходить повз їдальню та чує дзвін посуду, у неї виділяється слина. Реалізація якого рефлексу зумовлює цю реакцію?

Умовний природний

Умовний інструментальний

Умовний штучний

Безумовний орієнтовний

Умовний орієнтовний

3554 / 6854
До медико-генетичної консультації звернувся юнак з приводу відхилень у фізичному і статевому розвитку. В ході мікроскопії клітин слизової оболонки рота виявлені тільця Барра. Вкажіть найбільш імовірний каріотип юнака:

47, 21+

47, ХХY

45, ХО

47,18+

47, ХYY

3555 / 6854
У хворого з яскраво вираженою жовтушністю шкіри, склер та слизових оболонок, сеча має колір темного пива, кал світлий. У крові підвищений вміст прямого білірубіну, в сечі визначається білірубін. Який тип жовтяниці у хворого?

Паренхіматозна

Екскреційна

Обтураційна

Гемолітична

Кон’югаційна

3556 / 6854
У реанімаційне відділення надійшов хворий з гострим інфарктом міокарда, якому для зменшення болю ввели морфіну гідрохлорид. Механізм дії морфіну гідрохлориду обумовлений:

Блокадою фосфодіестерази

Стимуляцією аденілатциклази

Стимуляцією опіатних рецепторів

Блокадою холінестерази

Блокадою гістамінових рецепторів

3557 / 6854
Хворому у ЛОР-відділенні проведено біопсію слизової оболонки носа. Гістологічно виявлено продуктивне запалення з формуванням гранульом, у складі яких переважають лімфоцити і плазмоцити, спостерігаються еозинофільні гіаліноподібні кулі та великі макрофаги із світлою цитоплазмою (клітини Мікуліча). Який збудник міг викликати описані зміни?

Бліда трепонема

Золотистий стафілокок

Мікобактерія лепри

Респіраторно-синцитіальний вірус

Паличка Волковича-Фріша

3558 / 6854
У працівників хімічних комбінатів, де виробляють органічні розчинники, які здатні розчиняти фосфоліпіди, часто розвиваються захворювання легень. Який компонент аерогематичного бар’єру при цьому пошкоджується в першу чергу?

Секреторні альвеолоцити

Сурфактант

Респіраторні альвеолоцити

Альвеолярні макрофаги

Септальні клітини

3559 / 6854
У людини після довільної тривалої затримки дихання збільшилися частота й глибина дихання. Які зміни в крові, насамперед, стали причиною цього?

Зниження рСО2

Зниження рО2

Підвищення рО2

Підвищення рН

Підвищення рСО2

3560 / 6854
У відділення надійшла дитина з носовою кровотечею та меленою в калі. Зі слів матері, відбулося отруєння дитини кумаринами, які застосовувалися для боротьби з щурами. Введення якого засобу припинить кровотечу у дитини?

Вікасол

Адреналін

Фраксипарин

Тромбін

Фепранон

3561 / 6854
Внаслідок дефіциту УФО-ендонуклеази порушується репарація ДНК і виникає таке захворювання:

Серпоподібноклітинна анемія

Подагра

Альбінізм

Фенілкетонурія

Пігментна ксеродермія

3562 / 6854
В експериментальної тварини зроблено двобічну перерізку блукаючих нервів. Що відбудеться з диханням?

Зупиниться в фазі вдиху

Не зміниться

Стане рідким і глибоким

Зупиниться в фазі видиху

Стане частим і поверхневим

3563 / 6854
Жінка 69-ти років довго хворіла на атеросклероз. Надійшла до хірургічного відділення з симптомами гострого живота. В ході лапаротомії виявлені: тромбоз мезентеріальної артерії, петлі тонкої кишки набряклі, багряно-чорного кольору, на їх серозній оболонці фібринозні нашарування. Який патологічний процес розвинувся у кишці хворої?

Секвестр

Коагуляційний некроз

Суха гангрена

Волога гангрена

Ішемічний інфаркт

3564 / 6854
До лікарні потрапив футболіст з ушкодженням поверхневого пахвинного кільця та розривом двох ніжок, що його обмежують. Похідним якої анатомічної структури вони є?

Апоневроз внутрішнього косого м’яза живота

Міжніжкові волокна

Апоневроз зовнішнього косого м’яза живота

Власна фасція живота

Апоневроз поперечного м’яза живота

3565 / 6854
В ході копрологічного дослідження у працівників кав’ярні лікарями санітарно- епідеміологічної станції були виявлені округлі цисти, характерною ознакою яких є наявність чотирьох ядер. Імовірніше за все у цих працівників безсимптомно паразитує:

Кишкова трихомонада

Балантидій

Дизентерійна амеба

Амеба кишкова

Лямблія

3566 / 6854
При деяких гельмінтозах людина може сама виявити гельмінта, оскільки зрілі членики збудника можуть активно виповзати з ануса людини. Це характерно для:

Теніаринхозу

Гіменолепідозу

Теніозу

Дифілоботріозу

Ехінококозу

3567 / 6854
Хворий на хронічну серцеву недостатність протягом декількох місяців приймав дигітоксин; в процесі дигіталізації з’явилися такі симптоми: головний біль, нудота, діарея, втрата апетиту, порушення кольорового сприйняття, брадикардія. Який антидот доцільно призначити для зменшення симптомів інтоксикації?

Налоксон

Атропіну сульфат

Унітіол

Преднізолон

Адреналіну гідрохлорид

3568 / 6854
У сироватці крові новонародженого виявлено антитіла до вірусу кору. Про наявність якого імунітету це свідчить?

Спадковий, видовий

Природний активний

Штучний пасивний

Природний пасивний

Штучний активний

3569 / 6854
Під час реплікації ДНК один із її ланцюгів синтезується із запізненням. Що визначає дану особливість синтезу?

Компліментарність ланцюгів

Антипаралельність ланцюгів

Необхідність репарації

Великі розмірами ДНК-полімерази

Відсутність трифосфонуклеотидів

3570 / 6854
Фермент оксидаза D-амінокислот каталізує дезамінування тільки D-амінокислот. Яка властивість ферментів виявляється при цьому?

Абсолютна специфічність

Стереохімічна специфічність

Відносна специфічність

Термолабільність

Залежність від рН

3571 / 6854
У хворого хронічний нежить. Набряк слизової оболонки носової порожнини призводить до порушення функції рецепторів нюхового нерва, які розташовані в нюховій ділянці носової порожнини. Через яке утворення волокна цього нерва потрапляють до передньої черепної ямки?

Foramen incisivum

Foramen ethmoidale anterior

Foramen ethmoidale posterior

Lamina cribrosa os ethmoidale

Foramen sphenopalatinum

3572 / 6854
На гістологічному препараті в складі видовженої структури, обмеженої плазмолемою, по периферії розташовані численні ядра, а в цитоплазмі наявна поперечна посмугованість. Яка це структура?

Кардіоміоцит

Колагенове волокно

Гладенький міоцит

Синцитіотрофобласт

Міосимпласт

3573 / 6854
A woman complains of раіп іп the right іlіас region. Palpation detects there a soft, movie, and painful intestine. What intestine is being palpated?

Cecum

Rectum

Jejunum

Ascendmg colon

Sigmoid colon

3574 / 6854
An experiment has demonstrated that after exposure to ultrav radiation the dermal cells of the patients with xeroderma pigmentosum are slower to restore the native DNA structure than they are in the healthy individuals due to deficiency of the DNA repair enzyme. What enzyme takes part in the repair process?

Endonuclease

RNA ligase

DNA polymerase III holoenzyme

DNA gyrase

Primase

3575 / 6854
Blood serum analysis of the patient with acute hepatitis shows increased levels of alanine aminotransferase (ALT) and aspartate aminotransferase (AST). What changes on the cellular level can result in such developments?

Disturbed energy supply to the cells

Disturbed intercellular interactions

Cell destruction

Disturbed cellular enzyme systems

Damage to the genetic apparatus of the cells

3576 / 6854
The disease onset occurred 3 days ago. The patient complains of body temperature up to 38°C, stomachache, and frequent loose bloody stools. Bacillary dysentery was clinically diagnosed in the patient. What method of microbiological diagnostics would be advisable in this case and what samples should be obtained from the patient to confirm this diagnosis?

Bacterioscopy, blood

Serology, blood

Bacterioscopy, feces

Bacteriology, feces

Bacteriology, urine

3577 / 6854
In the morning a man diagnosed with diabetes mellitus received a prescribed dose of long- acting insulin on an empty stomach. He missed his regular meal and soon after that he developed weakness, headache, and vertigo, body tremors, convulsions, feeling of hunger, and signs of hypoglycemia. Glucose administration did not improve the patient’s condition. What medicine should be administered to provide quick relief to the patient?

Hydrocortisone

Prednisolone

Noradrenaline

Triamcinolone

Adrenaline

3578 / 6854
A man complaining of memory deterioration, reduced mental capabilities, sleep disorders, and vertigo was brought to the neurologic department. The patient explains these symptoms as consequences of the brain concussion received in a traffic accident 2 years ago. Choose the drug that can improve the patient’s brain metabolism and would be the most advisable in this case:

Cordiamin (Nikethamide)

Sydnocarb (Mesocarb)

Caffeine

Sodium oxybutirate

Piracetam

3579 / 6854
Autopsy of the body shows that the soft meninges of the deceased individual are plethoric, thickened, opaque, and yellow-green colored. What type of exudative inflammation can be characterized by such changes in the soft meninges?

Hemorrhagic

Suppurative

Serous

Fibrinous

Catarrhal

3580 / 6854
Mass mortality of rodents was observed in one of the mountain villages. Simultaneously there occurred a disease outbreak in the local population. The disease manifested by rapidly progressive fever up to 40°C, marked intoxication, and enlargement of inguinal lymph nodes. Smear preparations made from autopsy specimens contained gram-negative ovoid bacilli with bipolar staining. What microorganism is the causative agent of this disease?

Francisella tularensis

Staphylococcus

Bacillus anthracis

Clostridia

Yersinia pestis

3581 / 6854
A 36-year-old woman suffers from a connective tissue disease (collagenosis). What metabolite is the most likely to be increased in her urine?

Urea

Indican

Urobilinogen

Oxyproline

Creatinine

3582 / 6854
A 34-year-old woman was diagnosed with hereditary spherocytosis (hereditary microspherocytic hemolytic anemia, Minkowski-Chauffard syndrome). What mechanism leads to erythrocyte hemolysis in the patient?

Bone marrow hypoplasia

Membranopathy

Hemoglobinopathy

Autoimmune disorder

Enzymopathy

3583 / 6854
A patient has an incised wound on the posterior surface of his thigh and is unable to flex his lower leg. What muscles were damaged?

Semitendinosus, adductor, gracilis

Semimembranosus, semitendinosus, gracilis

Biceps, gracilis, adductor

Biceps, adductor, gracilis

Semitendinosus, semimembranosus, biceps

3584 / 6854
After intensive training session an athlete presents with significant loss off vascular tone in the working muscles. These changes are caused by accumulation of the following in the vessels:

Histamine

Natriuretic hormone

Metabolites

Serotonin

Renin-angiotensin

3585 / 6854
The cell was exposed to mutagenic factor which resulted in DNA molecule losing 2 nucleotide pairs. What type of mutation occurred in the DNA?

Replication

Translocation

Duplication

Inversion

Deletion

3586 / 6854
Permeability of the excitable cell membrane to potassium ions was increased in an experiment. What changes in the membrane potential will occur?

Depolarization

Hyperpolarization

Action potential

Local response

No changes

3587 / 6854
An unconscious patient was brought into the hospital. The smell of acetone can be detected from the patient’s mouth. Blood glucose - 25 mmol/L, ketone bodies -0.57 mmol/L. What hormone deficiency can result in the development of this condition?

Thyroxin

Insulin

Somatotropin

Glucocorticoids

Aldosterone

3588 / 6854
A patient has developed systemic (megaloblastic) anemia despite eating a balanced diet. The day before he underwent a gastric surgical resection. The anemia in this patient is caused by the deficiency of:

Vitamin PP

Vitamin C

Folic acid

Protein

Castle factor

3589 / 6854
A patient was hospitalized with diagnosis of an intestinal carcinoid. Laboratory analysis detects increased synthesis of serotonin from tryptophan. This process is based on the following biological mechanism:

Formation of paired compounds

Decarboxylation

Transamination

Microsomal oxidation

Deaminization

3590 / 6854
A patient with ciliary arrhythmia was prescribed digoxin. What mechanism of action of this medicine results in its antiarrhythmic effect?

Decreased sympathetic influences

Decreased permeability of the cell membrane to sodium

Increased potassium concentration in the cardiomyocytes

Decreased permeability of the cell membrane to calcium

Inhibition of Na, K-ATPase

3591 / 6854
Thoracic duct rupture has occurred in a weightlifter, when he was lifting the bar. The injury to the thoracic duct is the most likely to be localized in the area of:

Posterior mediastinum

Aortic hiatus

Venous angle junction

Lumbosacral junction

Neck

3592 / 6854
A man was brought to the hospital in a severe condition: facial edemas, myalgia, high temperature, and respiratory distress. History-taking revealed that the patient’s family regularly consumes untested pork. What helminth can be the cause of such symptoms?

Strongyloides stercoralis

Taeniarynchus saginatus

Trichinella spiralis

Ancylostoma duodenale

Diphyllobothrium latum

3593 / 6854
Blood sample poured into a test tube has clotted within 6 minutes. The test tube was then put into a thermostat and in 24 hours the blood clot was destroyed due to activation of:

Kallikreins

Heparin

Antithrombins

Plasmins

Kinins

3594 / 6854
Autopsy of the body of a 48-year-old man shows that the bone marrow in the flat bones, as well as in the cylindrical bone diaphyses and epiphyses, is moist, colored gray-red or gray-yellow, and puriform (pyoid bone marrow). The spleen weight is 7 kg; it is dark red on section, with signs of ischemic infarctions. All the lymph nodes are enlarged, soft, and gray-red in color. In the liver there are signs of fatty degeneration and leukemic infiltrates. What is the most likely diagnosis?

Chronic myeloid leukemia

Multiple myeloma

Acute myeloid leukemia

Lymphogranulomatosis

Acute lymphoid leukemia

3595 / 6854
A 52-year-old woman suffering from breast cancer had undergone a course of radiation therapy. As a result the tumor diminished in size. What mechanism of cell damage ensures effectiveness of radiation therapy?

Mutagenesis

Formation of free radicals

NK cell-induced lysis

Hyperthermia

Vascular thrombosis

3596 / 6854
A person has memorized a phone number for a short period of time (a few seconds). After making a call the person was unable to reproduce this sequence of numbers. In this case the process of memorizing was based on the following type of memory:

Iconic memory

Long-term memory

Secondary and tertiary memory

Short-term memory

Medium-term memory (episodic buffer)

3597 / 6854
A 27-year-old woman suffering from rheumatic heart disease since her childhood has developed cardiac decompensation and died. Autopsy shows mitral stenosis; mitral valve cusps are sharply thickened, sclerotic, and fused together along the closure line. Specify the type of endocarditis in this case:

Diffuse endocarditis

Recurrent verrucous endocarditis

Fibroplastic endocarditis

Ulcero-polypoid endocarditis

Acute verrucous endocarditis

3598 / 6854
A patient with ischemic heart disease was prescribed a calcium channel blocker. Name this drug:

Nitroglycerine

Thiotriazolin

Carvedilol

Amlodipine

Altiopril

3599 / 6854
Analysis detects glucose and amino acids in the primary urine. In the residual urine they are absent due to tubular reabsorption of these substances. Where in the nephron does this process occur?

Distal convoluted tubule

Macula densa

Proximal convoluted tubule

Collecting duct

Henle’s loop

3600 / 6854
Histologic specimen of an ovary shows a follicle in its cortical substance. The follicle consists of a primary oocyte, transparent membrane, and one layer of prismatic cells. Specify the type of the follicle:

Secondary

Tertiary

Primordial

Primary

Ovulating

3601 / 6854
При деяких гельмінтозах людина може сама виявити гельмінта, оскільки зрілі членики збудника можуть активно виповзати з ануса людини. Це характерно для такого захворювання:

Теніарінхоз

Гіменолепідоз

Теніоз

Ехінококоз

Дифілоботріоз

3602 / 6854
У хворого з невритом стегнового нерва порушено згинання стегна та розгинання гомілки у колінному суглобі. Функція якого м'яза при цьому порушена?

Напівперетинчастий м'яз

Двоголовий м’яз стегна

Півсухожилковий м'яз

Триголовий м'яз стегна

Чотирьохголовий м'яз стегна

3603 / 6854
У жінки 62-х років розвинулася катаракта (помутніння кришталика) на фоні цукрового діабету. Який тип модифікації білків має місце при діабетичній катаракті?

Метилювання

АДФ-рибозилюзання

Обмежений протеоліз

Фосфорилювзння

Глікозилювання

3604 / 6854
У людини внаслідок травми мозку сталося вимкнення грудного дихання зі збереженням діафрагмального. У разі якої локалізації травми це може відбутися?

На рівні ретикулярної формації стовбура

Між шийними та грудними сегментами спинного мозку

На рівні 8-го ірудного сегмента спинного мозку

На рівні 1-го шийного сегмента спинного мозку

На рівні варолієвого моста

3605 / 6854
У полі зору цистоскопа гладка поверхня слизової оболонки без складок. Яка частина сечового міхура у полі зору?

Шийка.

Дно

Міхуровий трикутник

Верхівка.

Тіло.

3606 / 6854
Хворому на туберкульоз призначено антибіотик олігоміцин. Назвіть процес, який інгібірує цей препарат при розмноженні туберкульозної палички.

Трансамінування.

Реплікація.

Трансляція.

Окислювальне фосфорилювання.

Транскрипція.

3607 / 6854
У хворого, який на тлі атеросклерозу переніс ішемічний інсульт, спостерігається порушення рухової функції у вигляді геміплегії. Яка з перелічених ознак є характерною при даній патології для уражених кінцівок?

Гіпертонус м'язів

Гіпотонус м'язів

Гіпорефлексія

Трофічні розлади

Відсутність трофічних розладів

3608 / 6854
У хворого з ІХС виникли порушення серцевого ритму, збільшився рівень глюкози у крові. Поруч з антиангінальними засобами, лікар призначив вітамінний препарат. Який з вітамінних засобів має кардіотрофічний та гіпоглікемічний ефект?

Ергокальциферол

Тіамін

Ціанокобаламін

Рибофлавін

Ретинол

3609 / 6854
Під час експерименту, внаслідок уведення тварині синтетичного аналога тиреоїдних гормонів, збільшилася частота серцевих скорочень, що опосередковано:

Метасимпатичною нервовою системою

Соматичною нервовою системою

Парасимпатичною нервовою системою

Симпатичною нервовою системою

3610 / 6854
При мікроскопічному дослідженні біоптату шкіри виявляються гранульоми, які складаються з епітеліоїдних клітин, оточених в основному Т-лімфоцитами. Серед епітеліоїдних клітин розташовуються поодинокі гігантські багатоядерні клітини типу Пирогова-Лангханса. В центрі деяких гранульом виявляються ділянки казеозного некрозу. Кровоносні судини відсутні. Для якого захворювання характерні зазначені зміни?

Сифіліс

Туберкульоз

Сап

Лепра

Риносклерома

3611 / 6854
Хворий на хронічний пієлонефрит помер від хронічної ниркової недостатності. При житті аускультативно відмічено "шум тертя перикарду? На розтині виявлено. що епікард тьмяний, шорсткий, ніби покритий волосяним покривом. Який перикардит за характером запалення має місце?

Фібринозний

Гнійний

Катаральний

Серозний

Гнильний

3612 / 6854
Реалізація загального адаптаційного синдрому здійснюється переважно через нейроендокринну систему. Якій з ланок цієї системи належить провідна роль у патогенезі реакції, що розвішається?

Гіпофізарно-адреногенітальна

Гіпофізарно-адреналова

Гіпофізарно-інсулярна

Гіпофізарно-юкстагломерулярна

Гіпофізарно-тиреоїдна

3613 / 6854
При операції у хлопчика 12-ти років видалений апендикс, який надіслано патологу на дослідження. Макроскопічно: апендикс в дистальному відділі з булавоподібним стовщенням діаметром 3 см, при розрізі якого вилилася прозора жовтувата рідина, стінка апендикса стоншена. Мікроскопічно: атрофія всіх шарів апендикса, ознак запалення немає. Який найбільш імовірний діагноз?

Хронічний апендицит

Міксоглобульоз апендикса

Водянка апендикса

Емпієма апендикса

Флегмонозний апендицит

3614 / 6854
При гістологічному дослідженні шийного лімфатичного вузла було виявлено повнокрів'я та набухання коркового шару, в мозковій речовині наявна велика кількість плазматичних клітин, зменшення кількості лімфоцитів, значна макрофагальна реакція. Назвіть характер змін в лімфатичному вузлі:

Лімфома

Гострий лімфаденіт

Лімфогранулематоз

Антигенна стимуляція лімфоїдної тканини

3615 / 6854
Під час розтину тіла дівчики 9 років у верхівці правої легені субплеврально було знайдене вогнище казеозного некрозу діаметром 15 мм, біфуркаційні лімфатичні вузли були збільшені містили дрібні вогнища некрозу коагуляційного типу. Мікроскопічно - у легеневому вогнищі та в лімфатичних вузлах навколо некротичних мас були розташовані епітеліоїдні клітини, лімфоцити та поодинокі багатоядерні гігантські клітини. Діагностуйте захворювання.

Вторинний фіброзно-вогнищевий туберкульоз

Первинний туберкульоз

Вторинний вогнищевий туберкульоз

Гематогенний туберкульоз з переважним ураженням легень

Гематогенний генералізований туберкульоз

3616 / 6854
У хворого з яскраво вираженою жовтяничністю шкіри, склер, слизових оболонок сеча має колір темного пива, кал світлий. У крові підвищений вміст прямого білірубіну, в сечі визначається білірубін. Який тип жовтяниці у хворого?

Кон'югаційна

Гемолітична

Екскреційна

Обтураційна

Паренхіматозна

3617 / 6854
При макро-мікроскопічному дослідженні ділянки великогомілкової кістки та м’яких тканин довкола виявлено дифузне гнійне запалення; що захоплює кістковий мозок, гаверсові канали та періост осередки некрозу Про яке захворювання слід думати?

Хронічний гематогений остеомієліт

Хвороба Педжета

Остеонекроз

Гострий гематогений остеомієліт

Паратиреоїдна остеодистрофія

3618 / 6854
У хворого 65-ти років під час неврологічного обстеження виявлено крововилив у межах верхньої скроневої звивини. У зоні кровопостачання якої артерії воно знаходиться?

Задня мозкова артерія

Передня мозкова артерія

Середня мозкова артерія

Основна артерія

Передня сполучна артерія

3619 / 6854
При анемії в периферичній крові визначаються дегенеративні і регенеративні форми еритроцитів. Назвіть регенеративні форми еритроцитів.

Сфероцити

Пойкіяоцити

Гіперхромні еритроцити

Ретикулоцити

Мікроцити

3620 / 6854
Внаслідок дефіциту УФО-ендонуклеази порушується репарація ДНК і виникає таке захворювання:

Серпоподібноклітинна анемія

Альбінізм

Пігментна ксеродермїя

Подагра

Фенілкетонурія

3621 / 6854
В лікуванні системного захворювання сполучної тканини (склеродермія) необхідно використовувати десенсибілізуючу, протизапальну та імунодепресивну дії ліків. Якій групі засобів притаманні всі ці ефекти?

Антигістамінні засоби

Стероїдні протизапальні

Адреноблокатори

Анаболічні стероїди

Адреноміметичні засоби

3622 / 6854
У відділення реанімації поступив хворий після ДТП з однобічним пневмотораксом. Який вид дихання спостерігається у даному випадку?

Поверхневе рідке

Поверхневе часте

Асфіктичне

Глибоке часте

Поверхневе

3623 / 6854
При патології нирок в сечі з'являються патологічні складові частини. Поява яких патологічних складових частин сечі свідчить про підвищення проникності клубочкової мембрани?

Глюкозурія

Протеїнурія

Аміноацидурія

Алкаптонурія

Піурія

3624 / 6854
На розтині померлого в підкоркових ядрах правої півкулі головного мозку визначається порожнина неправильної форми 5 х 3,5 см, заповнена червоними згортками крові і розм’якшеною тканиною мозку. Назвіть патологію, яка розвинулась у головного мозку?

Абсцес.

Кіста.

Геморагічне просякнення.

Гематома.

Ішемічний інфаркт.

3625 / 6854
У хворої при гастродуоденоскопії виявили множинні виразки шлунка. Діагностовано синдром Золлінгера-Еллісона. Надлишок якого гормону має місце в даному випадку?

Тироксин

Кортизол

Окситоцин

Гастрин

Інсулін

3626 / 6854
Дівчина хвора на цукровий діабет, чекає на донорську нирку. Яке ускладнення діабету в неї є причиною хронічної ниркової недостатності?

Атеросклероз

Ретинопатія

Нейропатія

Макроангіопатія

Мікроангіопатія

3627 / 6854
На розтині тіла чоловіка, яким номер від черевного тифу, в клубовій кишці виявлені дефекти, розташовані по всій довжині кишки, краї їх рівні, дно утворене м’язовим шаром. Яка зі стадій черевного тифу діагностована?

Чистих виразок

Загоєння

Утворення виразок

Некрозу

Мозкоподібного набухання

3628 / 6854
При гастритах, як правило, ушкоджуються залози слизової оболонки шлунку. За рахунок яких клітин можлива їх регенерація?

Головні

Ендокринні

Шийкові мукоцити

Парієтальні

Додаткові

3629 / 6854
На секції тіла жінки 76-ти років були знайдені ознаки хронічного бронхіту та легеневого серця, печінка збільшена в розмірах, щільна, тканина на розрізі строката. Мікроскопічно: просвіти центральних вен та синусоїдних капілярів розширені. повнокровні, а гепатоцити на периферії часточок з жировою дистрофією. Який вид розладу кровообігу викликав зміни печінки?

Загальне малокрів’я

Загальна венозна гіперемія

Місцева венозна гіперемія

Загальна артеріальна гіперемія

Місцева артеріальна гіперемія

3630 / 6854
Через 6 годин після інфаркту міокарда у хворого в крові піднялася активність лактатдегідрогенази. Наявність якого ізоферменту слід чекати у цьому випадку?

ЛДГЗ

ЛДГ2

ЛДГ4

ЛДГ1

ЛДГ3

3631 / 6854
У жінки народилася мертва дитина з множинними вадами розвитку. Яке протозойне захворювання могло спричинити внутрішньоутробну загибель плоду? 1

Токсоплазмоз

Трихомоніаз

Балантидіаз

Лямбліоз

Амебіаз

3632 / 6854
Відповідно до сучасної концепції атерогенезу «Response to injury», атеросклероз є проявом хронічного запалення в інтимі артерій. З якою стадією запалення пов’язано формування фіброзних бляшок при атеросклерозі:

Трансформація

Ексудація

Первинна альтерація

Проліферація

Вторинна альтерація

3633 / 6854
В експерименті на ізольованій збудливій клітині необхідно отримати збільшення мембранного потенціалу спокою (гіперполяризацію). Для цього доцільно викликати активацію таких іонних каналів:

Калієві

Кальцієві

Калієві та натрієві

Натрієві та кальцієві

Натрієві

3634 / 6854
У пацієнтки на поверхні шкіри лівого кута лопатки знайдено утвір з наступними характеристиками: 5-7 мм діаметром, кулястої форми, твердої консистенції, який має широку основу та сосочкову поверхню. Гістологічно: клітини плоского епітелію. що нерівномірно розростаються, кількість шарів збільшена, надлишкова кількість кровоносних судин. Вкажіть патологічний процес:

Аденома

Аденокарцинома

«Рак на місці»

Папілома

Гідроаденома

3635 / 6854
І.М. Сєченов встановив, що втомлена кінцівка відновлює працездатність швидше, якщо в період відпочинку друга кінцівка працює. Це дало можливість розробити вчення про:

Втому

Оптимум

Парабіоз

Активний відпочинок

Песимум

3636 / 6854
Жінка 40-ка років звернулася із скаргами на неможливість розгинати стопу і пальці, що створює труднощі при ходьбі. Об’єктивно: ступня звисає, дещо повернена всередину, пальці зігнуті ('кінська стопа'), чутливість втрачена на зовнішній поверхні гомілки і тильній поверхні стопи. Який нерв уражений?

Великогомілковий

Сідничний

Стегновий

Спільний малогомілковий

Підшкірний

3637 / 6854
Хворий звернувся до лікаря з пораненням проксимальної фаланги пальця, яка ускладнилася флегмоною долоні. Гній заповнив спільну синовіальну піхву для згиначів, в якій лежать сухожилки поверхневого та глибокого згиначів пальців. На якому пальці було ушкодження проксимальної фаланги?

V

I

II

IV

ІІІ

3638 / 6854
При обстеженні лікарем хворої людини на основі клінічних даних був поставлений діагноз парагрип. Яким із перелічених способів може передаватись це захворювання?

Фекально-оральний

Трансмісивний

Через укус тварин

Парентеральний

Повітряно-крапельний

3639 / 6854
Жінка 35-ти років розпочала голодувати. Депо яких поживних речовин використовується у початковий період голодування і як при цьому змінюється дихальний коефіцієнт (ДК)?

Білки, ДК наближається до 0,7

Жири, ДК наближається до 0,72

Жири, ДК наближається до 0,85

Білки, ДК наближається до 1

Вуглеводи, ДК наближається до 1

3640 / 6854
У пацієнтки 26-ти років висипання на шкірі, свербіж після вживання цитрусових. Призначте лікарський засіб з блокаторів Н1-гістамінорецепторів:

Дифенгідрамін

Кислота ацетилсаліцилова

Парацетамол

Метамізол

Менадіону натрію біосульфат

3641 / 6854
Хворому при безсонні, викликаному емоційними розладами, лікар призначив засіб, що викликає сон за рахунок транквілізуючої дії. Який снодійний препарат був призначений пацієнту?

Фенобарбітал

Хлоралгідрат

Бромізовал

Етамінал-натрій

Нітразепам

3642 / 6854
Біологічне окислення та знешкодження ксенобіотиків відбувається за рахунок гемвмісних ферментів. Який метал є обов'язковою складовою цих ферментів?

Мg

Zn

Fe

Мn

Со

3643 / 6854
На гістологічному препараті представлений зріз судини, що характеризується правильною круглою формою; зіяє, стінка складається з 3-х оболонок, де у середній спостерігається наявність 40-50 вікончастих еластичних мембран. Яка судина представлена на препараті?

Артерія змішаного типу

Вена м’язового типу

Артерія еластичного типу

Кровоносний капіляр

Артерія м'язового типу

3644 / 6854
У жінки при профілактичному огляді виявили пухлину молочної залози. Результати біопсії підтвердили наявність злоякісної пухлини. Який основний механізм інфільтративного росту злоякісної пухлини?

Здатність пухлинних клітин до амебоїдного руху

Підвищення активності кейлонів в клітині

Збільшення адгезивності пухлинних клітин

Порушення контактного гальмування

Підвищення ферментативної активності лізосом

3645 / 6854
При обстеженні хворого 70-ти років виявлено цукровий діабет II типу. Яким препарат доцільно призначити хворому?

Інсулін

Кортизон

Глібенкламід

Паратиреоїдин

Мерказоліл

3646 / 6854
Людина стоїть у кімнаті в легкому одязі, температура повітря +14°С, вікна і двері зачинені. Яким шляхом вона віддає найбільше тепла?

Теплорадіація

Теплопроведення

Випаровування

Перспірація

Конвекція

3647 / 6854
Чоловіка 49 років доставили з місця автомобільної аварії в лікарню в непритомному стані. Шкірні покриви бліді, пульс частий і поверхневий. Переломів кісток і пошкодження головного мозку не виявлено. При пункції черевної порожнини отримано значну кількість крові. Первинною причиною тяжкого стану потерпілого є

Еритропенія

Гіпопротеїнемія

Гіповолемія

Гіпоінсулінемія

Гіпокатріємія

3648 / 6854
Під час бігу на довгі дистанції скелетна мускулатура тренованої людини використовує глюкозу з метою отримання енергії АТФ для м'язового скорочення. Вкажіть основний процес утилізації глюкозиза цих умов;

Аеробне окиснення

Глікогенез

Глікогеноліз

Анаеробний гліколіз

Глюконеогенез

3649 / 6854
Центральну роль в обміні амінокислот у нервовій тканині відіграє глутамінова кислота. Це пов'язано з тим, що дана амінокислота:

Використовується для синтезу нейроспецифічних білків

Використовується для синтезу ацетонових тіл

Використовується для синтезу ліків

Зв’язує аміак з утворенням глутаміну

Використовується для синтезу глюкози

3650 / 6854
Після споживання солоної їжі в людини значно зменшилася кількість сечі. Який з указаних гормонів уплинув на функцію?

Антидіуретичний

Окситоцин

Адреналін

Соматостатин

АКТГ

3651 / 6854
Внаслідок руйнування певних структур стовбуру мозку тварина втратила орієнтувальні рефлекси. Які структури було зруйновано?

Медіальні ядра ретикулярної формації

Червоні ядра

Чорна речовина

Чотиригорбкова структура

Вестибулярні ядра

3652 / 6854
При дії окислювачів (перекис водню, оксиди азоту та інші), гемоглобін, до складу якого входить Fе2+, перетворюється на сполуку, що містить FеЗ+. Ця сполука не здатна переносити кисень і має назву?

Метгемоглобін

Карбгемоглобін

Карбоксигемоглобін

Глікозильований гемоглобін

Оксигемоглобін

3653 / 6854
Під час постсинтетичного періоду мітотичного циклу було порушено синтез білків тубулінів. До яких наслідків це може призвести?

Порушення репарації ДНК

Порушення цитокінезу

Порушення спіралізації хромосом

Скорочення тривалості мітозу

Порушення формування веретена поділу

3654 / 6854
Хворому із загальним набряковим синдромом на тлі серцевої недостатності призначено фуросемід. Вплив на який процес забезпечує його терапевтичний ефект в цьому випадку?

Синтез натрієвих каналів в дистальних канальцях

Карбоангідразу

Нирковий кровотік

Активний транспорт іонів крізь базальну мембрану

Транспорт іонів крізь апікальну мембрану

3655 / 6854
Під дією УФ-опромінення та інших факторів можуть відбуватися зміни в структурі ДНК. Репарація молекули ДНК досягається узгодженою дією всіх наступних ферментів, ЗА ВИНЯТКОМ:

ДНК-лігаза

ДНК-полімераза

Ендонуклеаза

ДНК-глікозидаза

Аміноацил-тРНК-синтетаза

3656 / 6854
При мікроскопічному дослідженні нирок померлої від ниркової недостатності жінки 36-ти років, в клубочках виявлено проліферацію нефротелію капсули, подоцитів та макрофагів з утворенням "півмісяців", некроз капілярних петель, фібринові тромби в їх просвітах, а також склероз та гіаліноз клубочків, атрофію канальців та фіброз строми нирок. Який з перелічених діагнозів найбільш вірогідний?

Гострий гломерулонефрит

Мембранозна нефропатія

Хронічний гломерулонефрит

Фокальний сегментарний склероз

Підгострий гломерулонефрит

3657 / 6854
У клітинах мозку лисиці, яка була спіймана в межі міста, виявлені включення у вигляді тілець Бабеша-Негрі. Джерелом якого захворювання є ця тварина?

Інфекційний мононуклеоз

Кліщовий енцефаліт

Вітряна віспа

Сказ

Грип

3658 / 6854
Для підвищення результатів спортсмену рекомендували застосовувати препарат, який містить у собі карнітин. Який процес в найбільшому ступені активується карнітином?

Синтез стероїдних гормонів

Тканинне дихання

Транспорт жирних кислот у мітохондрії

Синтез кетонових тіл

Синтез ліпідів

3659 / 6854
У хворого з хронічним гіперацидним гастритом з'явився біль у суглобах. Для полегшення болю, враховуючи супутню патологію, був призначений целекоксиб. Вибіркова дія цього препарату на певний фермент забезпечує відсутність впливу на слизову шлунка. Назвіть цей фермент:

Фосфоліназа С

Фосфоліпаза А2

Калікреїн

Циклооксигеназа 2

Циклооксигеназа 1

3660 / 6854
На щурах моделювали опіки. Збільшилось утворення гістаміну, серотоніну, кінінів. оксиду азоту. Який тип причинно-наслідкових відносин у патогенезі єднає ці зміни з розвитком артеріальної гіперемії та підвищення проникності судин?

Дивергенція

Саногенез

Конвергенція

Circulus vitiosus

'Пряма лінія'

3661 / 6854
До лікарні поступив пацієнт з перфоративною виразкою задньої стінки шлунка. Який елемент очеревини під час операції хірург повинен ретельно обстежити?

Печінкову сумку

Чепцеву сумку

Передшлункову сумку

Лівий бічний канал

Праву брижозу пазуху

3662 / 6854
Основний спосіб зняття гіпертонічного кризу – швидке і значне зниження судинного опору. Ін’єкційні препарати якої групи адреноблокаторів можуть бути використані для цього?

a-Адреноблокатори

Неселективні В-адреноблокатори

Симпатолітики

Кардіоселективні В-адреноблокатори

3663 / 6854
У хворого збільшений основний обмін, підвищена температура тіла, тахікардія у стані спокою. Причиною цього може бути підвищена функція :

Нейрогіпофізу.

Статевих залоз.

Підшлункової залози.

Щитовидної залози.

Кіркової речовини наднирників.

3664 / 6854
ГІід час гістологічного дослідження видаленого збільшеного шийного лімфатичного вузла встановлено, що структура його стерта, лімфоїдні фолікули відсутні. Картина його одноманітна та представлена великою кількістю лімфобластів, серед яких наявні клітини з патологічним поділом. Інші групи лімфатичних вузлів і кістковий мозок не змінені. Виявлені зміни найбільш характерні для:

Неспецифічного гіперпластичного лімаденіту

Хронічного лімфолейкозу

Лімфогранулематозу

Лімфосаркоми

Саркоїдозу

3665 / 6854
Хворий після прийому жирної їжі відчуває нудоту, млявість. З часом з явилися ознаки стеатореї. В крові, холестерин - 9,2 ммоль/л. Причиною такого стану є нестача:

Фосфоліпідів

Хіломікронів

Жирних кислот

Жовчних кислот

Тригліцеридів

3666 / 6854
Хворому поставлений діагноз: гострий пієлонефрит із запаленням стінок ниркових чашечок і мисок. Який епітелій зазнав пошкодження?

Одношаровий призматичний

Багатошаровий кубічний

Багаторядний війчастим

Багатошаровий плоский зроговілий

Перехідний

3667 / 6854
До отоларинголога звернувся хворий у якого під час огляду виявлено неповне змикання голосових зв’зок при фонації. Голосова щілина при цьому приймає форму овалу.
Функція якого м’яза гортані порушена у хворого?

М. aryttenoideus transversus

М. cricoaryttenoideus posterior

М. cricoaryttenoideus lateralis

М. thyroaryttenoideus

М. vocalis

3668 / 6854
У людини, яка обертається на каруселі виникли збільшення частоти серцевії скорочень, потовиділення, нудота. З подразненням яких рецепторів, перш за все це пов’язано?

Пропріоцептори

Зорові

Дотикові

Слухові

Вестибулярні

3669 / 6854
Для лікування захворювань, збудники яких виділяють екзотоксин, застосовують антитоксичні сироватки. Для лікування якого з перерахованих захворювань антитоксичну сироватку застосувати НЕМОЖЛИВО?

Правець

Туберкульоз

Газова гангрена

Ботулізм

Дифтерія

3670 / 6854
Під час підйому пішки на 5 поверх у людини підвищився артеріальний тиск. Причиною є збільшення:

Вмісту іонів в плазмі крові

Хвилинного об'єму крові

Кількості функціонуючих капілярів

Об’єму циркулюючої крові

В'язкості крові

3671 / 6854
У людини з масою 80 кг після тривалого фізичного навантаження об’єм циркулюючої крові зменшився, гематокрит - 50%, загальний білок крові - 80 г/л. Такі показники крові є наслідком, перш за все:

Збільшення кількості еритроцитів

Збільшення діурезу

Збільшення онкотичного тиску плазми

Втрати води з потом

Збільшення змісту білків в плазмі

3672 / 6854
Лікар-гематолог призначив пацієнту з кровотечею коагулянт, який діє шляхом підвищення синтезу протромбіну та інших факторів зсідання крові переважно в печінці і є синтетичним водорозчинним вітаміном. Який препарат призначив лікар?

Етамзилат

Кальцію хлорид

Гепарин

Тромбін

Менадіону натрію біосульфат

3673 / 6854
Людину 35-ти років вкусила бджола. На місці укусу комахи визначається набряк, гіперемія, підвищення температури. Назвіть ініціальний патогенетичний фактор запального набряку:

Зниження онкотичного тиску крові

Підвищення осмотичного тиску у вогнищі запалення

Підвищення проникності мікросудин

Підвищення кров'яного тиску в капілярах

Порушення лімфовідтоку

3674 / 6854
У підлітка 12-ти років, який хворіє на бронхіальну астму, розвинувся тяжкий напад астми: виражена експіраторна задишка. блідість шкірних покровів. Який вид порушення альвеолярної вентиляції має місце?

Нервово-м’язовий

Торако-діафрагмальний

Рестриктивннй

Обструктивний

Центральний

3675 / 6854
Хірург під час операції на щитоподібній залозі перев’язав верхню щитоподібну артерію. Гілку якої судини перев’язав лікар?

А. carotis externa

A. lingualis

А. carotis interna

A. facialis

A. pharyngеа ascendens

3676 / 6854
У хворого відсутній зір, але зіничний рефлекс реалізується нормально. Де може знаходитись зона пошкодження?

Соматосенсорна кора

Зорова кора

Зоровий перехрест

Верхні горбки чотиригорбкового тіла

Нижні горбики чотиригорбкового тіла

3677 / 6854
У чоловіка і його сина інтенсивно росте волосся по краю вушних раковин. Це явище спостерігалося також у батька дідуся за батьківською лінією. Який тип успадкування зумовлює це?

Зчеплений з У-хромосомою

Аутосомно-домінантний

Аутосомно-рецесивний

Домінантний, зчеплений з Х-хромосомою

Рецесивний, зчеплений з X-хромосомою

3678 / 6854
У жінки 42 років, яка перенесла операцію на нирці, після наркозу розвинулися явища рекураризації і припинилося дихання. Як міорелаксант був застосований дитилін. Яки засіб найбільш доцільно застосовувати для відновлення тонусу м'язів?

Стрихніну нітрат

Плазму крові

Галантоміну гідробромід

Кофеїн

Прозерин

3679 / 6854
До хірургічного відділення ЦРЛ надійшов хворий з колотою раною стопи, яку він отримав під час косовиці. Який специфічний препарат необхідно застосувати з метою екстреної пасивної імунопрофілактики правця?

Протиправцева вакцина

Анатоксин

Антибіотики

Вакцина АКДП

Антитоксична сироватка

3680 / 6854
У хворого на туберкульоз легень непереносимість аміноглікозидного антибіотика амікацину. Який протитуберкульозний антибіотик можна включити до складу комплексної терапії у даному випадку?

Бензилпеніцилін

Рифампіцин

Канзміцин

Амоксицилін

Стрептоміцин

3681 / 6854
У хворого пухлина черевної порожнини, що стискає нижню порожнисту вену. Який кава-кавальний анастомоз на передній стінці живота забезпечить відтік венозної крові?

Між верхньою і нижньою надчеревними венами

Між хребетними і пупковими венами

Між непарною і напівнепарною венами

Між верхніми і нижніми прямокишковими венами

Між пупковою і верхньою надчеревною венами

3682 / 6854
Хворий на сімейну гіперліпідемію, викликану дефіцитом рецепторів ЛПНЩ, вживав інгібітори бета-гідроксиметил-глутарил-КоА-редуктази. Цей препарат сприяє:

Зниженню клітинного вмісту бета-ГМГ-КоА

Зниженню рівня холестеролу крові

Підвищенню рівня клітинної ацилхолестеролацилтрансферази

Підвищенню рівня триацилгліцеролів крові

Підвищенню рівня сквалену в клітинах

3683 / 6854
Людина 28 років споживає надмірну кількість вуглеводів (600 г на добу), що перевищує ії енергетичні потреби. Який процес буде активуватися у даному випадку?

Ліполіз

Глюконеогенез

Окислення жирних кислот

Ліпогенез

Гліколіз

3684 / 6854
Під час розтину померлого 43-х років, що страждав на ІХС з розвитком інфаркту міокарда, патологоанатом виявив набряк легень. Які патологічні зміни могли зумовити набряк легень?

Гостра правошлуночкова недостатність

Ішемія малого кола

Гостре загальне малокрїв'я

Стаз крові

Гостра лівошлуночкова недостатність

3685 / 6854
У 50-х роках у Західній Європі від матерів, які приймали в якості снодійного талідоміл, народилося кілька тисяч дітей з відсутністю або недорозвиненням кінцівок, порушенням будови скелета, іншими вадами. Яка природа даної патології?

Тератогенна дія

Моносомія

Генна мутація

Триплоїдів

Трисомія

3686 / 6854
Для підтвердження діагнозу: гострий атрофічний кандидоз проведено мікроскопію мазків, виготовлених із шкрібку нальоту на слизовій оболонці щоки, який взятий від хворої жінки. Виявлено овальної форми мікроорганізми, що брунькуються. Який метод забарвлення використав бактеріолог для фарбування мазків із досліджуваного матеріалу?

Грама

Морозова

Романовського-Гімзе

Нейсера

Ожешка

3687 / 6854
В ході обстеження людини необхідно визначити, яка частка альвеолярного повітря оновлюється під час кожного вдиху. Який з наведених показників необхідно розрахувати для цього?

Хвилинний об'єм дихання

Хвилинна альвеолярна вентиляція

Коефіцієнт легеневої вентиляції

Життєва ємність легень

Функціональна залишкова ємність легень

3688 / 6854
У медико-генетичну консультацію звернувся чоловік з приводу безпліддя. В ядрах більшості клітин епітелію слизової оболонки щоки у нього виявлено одне тільце Барра. Про який синдром може йти мова?

Патау

Шерешевського-Тернера

Клайнфельтера

Дауна

Едвардса

3689 / 6854
На 8-й день після введення протиправцевої сироватки у пацієнта піднялась температура. він став скаржитися на біль у суглобах та свербіж шкіри. Який механізм цього ускладнення?

Антитілозалежна цитотоксичність

Анафілаксія

Гіперчутливість уповільненого типу

Клітинна цитотоксичність

Імунокомилексна гіперчутливість

3690 / 6854
Хворий 58-ми років надійшов у торакальне відділення з кровохарканням: при бронхоскопії виявлено звуження правого стовбурового бронха внаслідок розростання сірувато-білої тканини. В ході біопсії виявлена пухлина, що складається з дрібних клітин овальної й округлої форми з гіперхромними ядрами та незначною цитоплазмою, клітини справляють враження «голих ядер» Клітини пухлини ростуть пластами, тяжами. Гістологічна будова пухлини свідчить про те, що у хворого:

Недиференційований рак

Пласкоклітинний незроговілий рак

Базальноклітинний рак

Пласкоклітинний зроговілий рак

Перехідноклітинний рак

3691 / 6854
У хворого 40-ка років в результаті щелепно-лицьової травми порушилася функція під'язикової і підщелепної залоз зліва - залози почали декретувати невелику кількість густої слини. Функція якого нерва порушена?

Трійчастий

Язикоглотковий

Блукаючий

Лицьовий

3692 / 6854
Під час проведення морфологічного дослідження периферичної крові хворого було помічено, що у еритроцитів забарвлена лише периферична частина, а в центрі є незабарвлене прояснення. Кольоровий показник – 0,56. Яка анемія найбільш ймовірна у цього пацієнта?

Апластична

В12-фолієводефіцитна

Залізодефіцитна

Сидеробластна

Е. Гемолітична

3693 / 6854
Підвищення внутрішньочерепного тиску у хворого з церебральною гематомою обумовило надмірну активність блукаючого нерва (ваготонію) та зміну частоти серцевих скорочені. Який вид аритмії серця виникає при ньому?

Шлуночкова екстрасистолія

Синусова тахікардія

Синусова брадикардія

Передсердно-шлуночкова блокада

Пароксизмальна тахікардія

3694 / 6854
Хворий на хронічний алкоголізм на вулиці втратив свідомість. Було діагностовано гіпоглікемію внаслідок порушення процесу глюконеогенезу. Які з наступних пар ферментів є необхідними для цього процесу?

Глюкозо-6-фосфатаза і піруватдегідрогеназа

Фосфоенолпіруваткарбоксикіназа і глюкокіназа

Глюкозо-6-фосфатаза та фосфофруктокіназа

Фруктозо-1.6-діфосфатаза і піруваткарбоксилаза

Піруваткіназа і піруваткарбоксилаза

3695 / 6854
У студента через добу після іспиту в аналізі крові виявили лейкоцитоз без істотної зміни в лейкоцитарній формулі Який механізм найімовірніше зумовив розвиток виявленої пиши в периферичній крові?

Посилення лейкопоезу

Перерозподіл лейкоцитів в організмі

Уповільнення еміграції лейкоцитів до тканин

Зменшення руйнування лейкоцитів

3696 / 6854
Юнак 15-ти років після переохолодження був доставлений в лікарню зі скаргами на біль, озноб. При огляді «гусяча шкіра», блідість, температура поступово підвищується. Яка стадія гарячки спостерігається у хворого?

St. absorbi

St. decrementі

St. fastigii

St. incrementi

St. inflamenti

3697 / 6854
На розтині тіла чоловіка, який хворів на фіброзно-кавернозний туберкульоз і помер від ниркової недостатності, виявлені збільшені в розмірах нирки, щільні на дотик, сіруватого кольору, на розрізі мають сальний вигляд. Який діагноз можна припустити?

Хронічний пієлоневрит

Гломерулонефрит

Амілоїдоз нирок

Нефросклероз

Мієломна нирка

3698 / 6854
Знешкодження ксенобіотиків (лікарських засобів, епоксидів, ареноксидів, альдегідів, нітропохідних тощо) та ендогенних метаболітів (естрадіолу, простагландинів, лейкотрієнів) відбувається в печінці шляхом їх кон'югації з:

Аспарагіновою кислотою

Б-Аденозилметіоніном

Гліцином

Фосфоаденозином

Глутатіоном

3699 / 6854
У хворого після травми виникла необхідність введення протиправцевої сироватки, але проба на чутливість до сироватки виявилася позитивною. Специфічну гіпосенсибілізацію у хворого слід виконати за допомогою введення:

Мінімальних доз специфічного алергену

Фізіологічних доз глюкокортикоїдів

Лікувальних доз антигістамінних препаратів

Наркотичних речовин, що знижують чутливість

Роздільної дози специфічного алергену

3700 / 6854
У людини частота серцевих скорочень постійно утримується на рівні 40 разів за хвилину. Що є водієм ритму серця у неї?

Атріовентрикулярний вузол

Ніжки пучка Гіса

Пучок Гіса

Волокна Пуркін'є

Синоатріальний вузол

3701 / 6854
Внаслідок захворювання нирок у пацієнта відмічаються набряки. В аналізах сечі масивна протеїнурія. Який механізм є основним у виникненні набряків у такого пацієнта?

Зниження фільтраційного тиску в нирках

Зниження онкотичного тиску тканин

Підвищення осмотичного тиску плазми крові

Зниження онкотичного тиску плазми крові

Зниження онкотичного тиску лімфи

3702 / 6854
Бактеріологічний метод діагнотики був використаний для підтвердження діагнозу: газова гангрена у хворого. Які живільні середовища необхідно використовувати для культивуваня збудника в цьому випадку?

Вільсона-Блера, Кітта-Тароцці

Лужний агар

МПА, МПБ

Ендо, Левіна, Плоскірєва

ЖСА, кров’яний агар

3703 / 6854
Внаслідок дефіциту вітаміну D у дитини визначається симптом рахіту. Зниження активності якого ферменту крові спостерігається при цьому?

Холінестераза

а-амілаза

Креатинкіназа

Лужна фосфатаза

Кисла фосфатаза

3704 / 6854
Хворому на глаукому призначили пілокарпіну гідрохлорид в очних краплях. До якої фармакологічної групи належить цей препарат?

М-холіноміметики

а-адреноблокатори

Гангліоблокатори

Міорелаксанти

М -холіноблокатори

3705 / 6854
Робота шахтарів у забої часто спричинює антракоз. Який вид дихальної недостатності може розвинутися при цьому?

Рестриктивний

Обструктивний

Діафрагмальний

Торакальний

Дисрегуляторний

3706 / 6854
У хворого з синдромом Іценко-Кушинга спостерігаються стійка гіперглікемія та глюкозурія. Синтез та секреція якого гормону збільшені у цього хворого?

Кортизол

Тироксин

Альдостерон

Глюкагон

Адреналін

3707 / 6854
Хвора 38-ми років померла під час нападу бронхіальної астми, що не вдалося купірувати. Під час гістологічного дослідження в просвіті бронхів виявлені скупчення слизу, в стінці бронхів багато тучних клітин (лаброцитів), багато з них у стані дегрануляції, а також багато еозинофілів. Який патогенез цих змін у бронхах?

Цитотоксична, цитолітична дія антитіл

Атопія

Гранулематоз

Клітинно обумовлений цитоліз

Імунокомплексний механізм

3708 / 6854
У жінки під час пологів в зв'язку з крововтратою визначили групу крові, Реакція аглютинації еритроцитів відбулася зі стандартними сироватками груп О (I), А (II) і не відбулася зі стандартною сироваткою групи В (III). Досліджувана кров належить до групи

В (III)

А (II)

АВ (IV)

О (І)

3709 / 6854
В ділянці хромосоми гени розташовані в такій послідовності: АВСDЕFG. В результаті дії радіоактивного випромінювання відбулася перебудова, після чого ділянка хромосоми має наступний вигляд: АВDЕFG. Яка мутація відбулася?

Дуплікація

Інсерція

Делеція

Інверсія

Мутація

3710 / 6854
Для оцінки придатності води для пиття проведено бактеріологічне дослідження. Який показник характеризує кількість бактерій групи кишкових паличок, що знаходяться в 1 л води?

Мікробне число

Титр колі-фага

Перфрінгенс-титр

Колі-індекс

Колі-титр

3711 / 6854
У хворого з клінічними ознаками імунодефіциту проведено імунологічні дослідження. Виявлено значне зниження кількості клітин, що утворюють розетки з еритроцитами барана. Який висновок слід зробити на основі даних аналізу?

Зниження рівня Т-лімфоцитів

Зниження рівня натуральних кїлерів (УХ-клітин)

Зниження рівня Б-лімфоцитів

Зниження рівня системи комплементу

Недостатність клітин-ефекторів гуморального імунітету

3712 / 6854
На певному етапі онтогенезу людини між кровоносними системами матері і плоду встановлюється фізіологічний зв'язок. Цю функцію виконує провізорний орган:

Жовтковий мішок

Амніон

Алантоїс

Серозна оболонка

Плацента

3713 / 6854
Встановлено ураження вірусом ВІЛ Т-лімфоцитів. При цьому фермент вірусу зворотня траскриптаза (РНК-залежна ДНК-полімераза) каталізує синтез:

ДНК на вірусній р-РНК

Вірусної і-РНК на матриці ДНК

і-РНК на матриці вірусного білка

ДНК на матриці вірусної і-РНК

Вірусної ДНК на матриці ДНК

3714 / 6854
При дослідженні людини у вертикальній позі встановлено, що в альвеолах верхівок легень парціальний тиск кисню складає 140 мм рт. ст. Причиною цього є те, що у даних відділах легень:

Вентиляція відсутня

Перфузія переважає над вентиляцією

Перфузія і вентиляція врівноважені

Вентиляція переважає над перфузією

3715 / 6854
У гістопрепараті яєчника жінки визначаються структури, що мають велику порожнину. Овоцит І порядку в них оточений прозорою оболонкою, променистим вінцем і розташований у яйценосному горбику, стінка утворена шаром фолікулярних клітин і текою. Вкажіть, якій структурі яєчника належать дані морфологічні ознаки:

Первинний фолікул

Примордіальний фолікул

Жовте тіло

Зрілий (третинний) фолікул

Атретичне тіло

3716 / 6854
На заняттях з лікувальної фізкультури лікар-фізіотерапевт запропонував юнакам відхилитись назад і дістати долонями підлогу. Яка зв’язка запобігає надмірному розгинанню хребтового стовпа?

Надостьова

Жовта

Задня повздовжня

Міжпоперечна

Передня повздовжня

3717 / 6854
На розтині жінки 23 років, помершої при нирковій недостатності, виявлено на шкірі лиця “червоний метелик”, на мітральному клапані дрібні до 0.2 см червонувато-рожеві бородавчаті нашарування, в нирках осередки фібриноїдного некрозу в клубочках, потовщення базальних мембран капілярів клубочків у вигляді “дротяних петель”, гематоксилінові тільця, каріорексис. Яке захворювання стало причиною смерті хворої?

Вузликовий периартериит

Ревматизм

Ревматоїдний артрит

Системна склеродермія

Системний червоний вовчак

3718 / 6854
В експерименті у кролика було видалено верхній шийний вузол симпатичного стовбура. На боці видалення спостерігається почервоніння і підвищення температури шкіри голови. Яка форма порушень периферичного кровообігу розвинулась у кроля?

Нейропаралітична артеріальна гіперемія

Нейротонічна артеріальна гіперемія

Метаболічна артеріальна гіперемія

Стаз

Венозна гіперемія

3719 / 6854
Експериментатору необхідно якнайшвидше виробити умовний рефлекс у собаки. На базі якого безумовного рефлекса доцільно виробляти умовний?

Орієнтувального

Травного

Міотатичного

Захисного

Статевого

3720 / 6854
У дитини, що часто хворіє на ангіни і фарингіти, відзначається збільшення лімфовузлів та селезінки. Зовнішній вигляд характеризується пастозністю і блідістю, м’язова тканина розвинена слабко. У крові спостерігається лімфоцитоз. Як називається такий вид діатезу?

Лімфатико-гіпопластичний

Геморагічний

Ексудативно-катаральний

Астенічний

Нервово-артритичний

3721 / 6854
У хворого з підозрою на ботулізм необхідно визначити тип екзотоксину, що циркулює в крові. Яка реакція може бути використана з цією метою?

Реакція преципітації

Реакція зв’язування комплементу

Реакція пасивної гемаглютинації

Реакція гальмування гемаглютинації

Реакція нейтралізації

3722 / 6854
У хворого після травми коліна гомілку у зігнутому під прямим кутом положенні можна зміщувати вперед і назад подібно до висувної шухляди. Які зв'язки розірвані?

Схрещені зв'язки коліна

Поперечна зв'язка коліна

Коса підколінна

Великогомілкова колатеральна

Малогомілкова колатеральна

3723 / 6854
Підлітку, що перебував у стані важкого алкогольного сп'яніння, лікар швидкої допомоги серед інших заходів здійснив внутрішньом'язове введення розчину кофеїну. Поясніть на основі якого принципу дії дана маніпуляція є доцільною:

Синергізм

Потенціація

Конкурентний антагонізм

Сумація ефектів

Фізіологічний антагонізм

3724 / 6854
Чоловік 43-х років доставлений у лікарню з ознаками черевного тифу. Хворий нещодавно був у місцевості, епідемічній за даним захворюванням, де пив некип'ячену воду з колодязя. З моменту появи перших симптомів пройшло З дні. У цей період збудник може бути виявлений у:

Жовчному міхурі

Шлунку

Крові

Лімфоїдній тканині кишківника

Тонкій кишці

3725 / 6854
Офтальмолог з діагностичною метою (розширення зіниць для огляду очного дна) використав 1% розчин мезатону. Мідріаз, викликаний препаратом, обумовлений:

Блокада альфа-1 адренореципторів.

Активація альфа-1 адренорецепторів.

Активація М-холінорецепторів

Активація 6ета-1 адренорецепторів

Активація альфа-2 адренорецепторів.

3726 / 6854
Зменшення тиску в каротидному синусі спричиняє наступні ефекти:

Падіння венозного тиску

Рефлекторне гіперпное

Рефлекторне зростання венозного тиску

Зростання частоти серцевих скорочень

Рефлекторну брадикардію

3727 / 6854
З метою профілактики гепатиту В групі стоматологів ввели вакцину, яка являє собою генно-інженерний НВs-антиген. Від якого ще інфекційного агента захищає таке щеплення?

Вірус Коксакі, група В

Вірус гепатиту С

Вірус грипу, тип В

Вірус імунодефіциту людини

Вірус дельта

3728 / 6854
Після прийому препарату у хворого з серцевою недостатністю зменшилася частота скорочень серця, пул став кращого наповнення, зменшилися набряки, збільшився діурез. Вкажить, який препарат приймав хворий:

Анаприлін

Дигоксин

Резерпін

Верапаміл

Дилтіазем

3729 / 6854
У чоловіка, померлого від внутрішньої кровотечі (гемоперитонеум). в печінці субкапсулярно виявлено губчастий вузол темно-червоного кольору розмірами 15x10 см, добре відмежований від навколишньої тканини. Мікроскопічно: тканина вузла складається з великих судинних тонкостінних порожнин, вистелених ендотеліальними клітинами та заповнених рідкою або згорнутою кров'ю. Встановіть вид пухлини:

Гемангіоперицитома

Венозна гемангіома

Лімфангіома

Капілярна гемангіома

Кавернозна гемангіома

3730 / 6854
Людина вийшла з кондиційованого приміщення на вулицю, де температура повітря дорівнює +40°С, вологість повітря - 60%. Віддача тепла з організму на вулиці буде здійснюватися за рахунок:

Проведення

Випаровування поту

Радіації

Конвекції

3731 / 6854
До приймального відділення доставлений хворий зі скаргами на сухість в роті, світлобоязнь та порушення зору. Об'єктивно: шкіра гіперемована, суха, зіниці розширені, тахікардія. При подальшому обстеженні був встановлений діагноз: отруєння алкалоїдами беладонни. Який лікарський засіб доцільно застосувати?

Ацеклідин

Дипіроксим

Армій

Прозерин

Пілокарпін

3732 / 6854
Хворий 63 років звернувся до невропатолога зі скаргою на те, що протягом трьох місяців не може виконувати столярні роботи, які потребують точності виконання тому що права рука робить багато неціленаправленних рухів. При дослідженні виявлено, що у хворого пошкоджена:

Gyrus angularis.

Gyrus postcentralis.

Gyrus supramarginalis.

Gyrus precentralis.

Gyrus temporalis superior.

3733 / 6854
У новонародженої дитини спостерігаються зниження інтенсивності смоктання, часте блювання, гіпотонія. У сечі та крові значно підвищена концентрація цитруліну. Який метаболічний процес порушений?

Глюконеогенез

Гліколіз

ЦТК

Цикл Корі

Орнітиновий цикл

3734 / 6854
На ізольованому серці шляхом охолодження припиняють функціонування окремих структур. Яку структуру охолодили, якщо серце внаслідок цього спочатку припинило скорочення, а далі відновили її з частотою, у 2 рази меншою за вихідну?

Пучок Гіса

Волокна Пуркін'є

Атріозетрикулярний вузол

Ніжки пучка Гіса

Синоатріальний вузол

3735 / 6854
Жінка літнього віку перенесла сильний стрес. У крові різко збільшилась концентрація адреналіну і норадреналіну. Які ферменти каталізують процес інактивації катехоламінів?

Пептидази

Глікозидази

Карбоксилаза

Тирозиназа

Моноамінооксидази

3736 / 6854
У новонародженої дитини після годування молоком спостерігалися диспептичні розлади (диспепсія, блювота). При годуванні розчином глюкози ці явища зникали. Вкажіть фермент, що бере участь в перетравленні вуглеводів, недостатня активність якого приводить до вказаних розладів.

Сахараза.

Амілаза.

Лактаза.

Мальтаза.

Ізомальтаза.

3737 / 6854
У хворого, що страждає на важку форму порушення водно- сольового обміну, настала зупинка серця в діастолі. Який найбільш вірогідний механізм зупинки серця в діастолі?

Гіпокаліємія

Дегідратація організму

Гіперкаліємія

Гіпернатріемія

Гіпонатріємія

3738 / 6854
Після перенесеного запального захворювання у хворого виникло неповне відведення очного яблука в латеральну сторону. Який нерв у хворого пошкоджений?

Блоковий

Зоровий

Окоруховий

Лицевий

Відвідний

3739 / 6854
В аналізі крові лаборант виявив без’ядерні форменні елементи у вигляді двовгнутих дисків. Назвіть їх:

Нейтрофіли

Еозинофізи

Еритроцити

Моноцити

Лімфоцити

3740 / 6854
Пацієнт, що прийшов на прийом, скаржиться на свербіж між пальцями. Лікар поставив діагноз - скабієз. Які членистоногі можуть спричиняти це захворювання?

Селищний кліщ.

Дермацентор.

Коростяний свербун.

Собачий кліщ.

Тайговий кліщ.

3741 / 6854
Студент дістав завдання розрахувати альвеолярну вентиляцію. Для цього йому необхідні знати наступні показники зовнішнього дихання:

Об’єм мертвого простору, життєва ємність легень, дихальний об’єм

Хвилинний об’єм дихання, частота дихання, дихальний об’єм

Частота дихання, життєва ємність легень, резервний об’єм вдиху

Дихальний об’єм, об’єм мертвого простору, частота дихання

Дихальний об’єм, резервний об’єм вдиху, резервний об’єм видиху

3742 / 6854
Аспірин інгібує синтез простагландинів, завдяки блокуванню активності циклооксигенази. Яка жирна кислота необхідна для цього синтезу?

Стеаринова

Ліноленова

Лінолева

Пальмітинова

Арахідонова

3743 / 6854
Характерними ознаками холери є втрата організмом великої кількості води та іонів натрію. Який механізм лежить в основі виникнення діареї при цьому?

Гальмування синтезу вазопресину з гіпоталамусі

Посилення секреції реніну клітинами ниркових артеріол

Посилення синтезу кортикотропіну

Окиснення альдостерону в корі наднирникїв

Активація аденілатциклази ентероцитів

3744 / 6854
Під час пологової діяльності при важкому прорізуванні голівки плоду, щоб уникнути розриву промежини, виконують розсічення отвору піхви біля основи великої статевої губи. Який м’яз промежини при цьому розсікають?

Глибокий поперечний м’яз

Сіднично-печеристий м'яз

Поверхневий поперечний м’яз

Цибулинно-губчастий м’яз

Зовнішній сфінктер прямої кишки

3745 / 6854
У працівників хімічних комбінатів, де виробляють органічні розчинники, які здатні розчиняти фосфоліпіди, часто розвиваються захворювання легень Який компонент аерогематичного бар'єру при цьому пошкоджується в першу чергу?

Респіраторні альвеолоцити

Септальні клітини

Сурфактант

Альвеолярні макрофаги

Секреторні альвеолоцити

3746 / 6854
У пацієнта після переохолодження у ділянці крил носа та верхньої губи з'явились герпетичні висипання. Для лікування була застосована мазь. Який противірусний засіб містить застосована мазь?

Інтерферон

Ацикловір

Дексаметазон

Індометацин

Азидотимідин

3747 / 6854
Під час розтину порожнин серця на внутрішній стінці були виявлені гребінцеві м'язи. Які відділи порожнин серця розкриті ?

Ліве передсердя і лівий шлуночок

Ліве вушко і лівий шлуночок

Праве передсердя і правий шлуночок

Правий і лівий шлуночок

Праве і ліве вушко

3748 / 6854
Під час ендоскопічного дослідження лікар виявив порушення цілісності стінки шлунка в межах слизової оболонки. Вкажіть. яким типом епітелію в нормі вистелена з середини стінку шлунка:

Багатошаровий плоский незроговілий

Перехідний

Багатошаровий плоский зроговілий

Псевдобагатошаровий

Одношаровий призматичний залозистий

3749 / 6854
Хворому 50-ти років після операції з метою прискорення загоєння рани місцево був призначений препарат, який має репаративну та імуностимулюючу активність. Визначте препарат:

Циклоспорин

Дексаметазон

Меркаптопурин

Діазолін

Метилурацил

3750 / 6854
Хворому на сепсис призначте антимікробний препарат з групи фторхінолонів. Оберіть його серед наведених препаратів

Ампіцилін

Цефалексин

Цефпіром

Ципрофлоксацин

Метронідазол

3751 / 6854
A 24-year-old man undergoes surgery and during the operation, an organ is excised and sent for histological evaluation. A light microscopic examination roved s Ihe organ encased by thin connective tissue capsule that enters the substance oi the lobes to further subdivide the organ into irregular lobular units. Bach lobule contains a cluster of follicles filled with colloid. Follicular epithelium consists of low columnar, cuboidal or squamous cells depending on the level of activity of the follicle. Which of the following organs does this tissue most likely belong to?

Parotid gland

Pancreas

Thyroid gland

Thymus

Parathyroid gland

3752 / 6854
A 23-year-old woman presents to the emergency department complaining of bloody diarrhea, fatigue and confusion. A few days earlier, she went to a fast food restaurant for a birthday party. Her friends are experiencing similar symptoms. Laboratory studies show anemia. Which of the following would you most likely obtain for microbiologic testing?

Cerebrospinal fluid

Urine

Stool

Bile

Blood

3753 / 6854
An 18-year-old girl comes to her physician with concern about her health because she has not achieved menarche. She denies any significant weight loss, changes in mood, or changes in her appetite. She mentions that her mother told her about mild birth defects, but she cannot recall the specifics. Past medical history and family history are benign. On physical examination, the patient is short in stature, has a short and webbed neck and wide chest. Staining of buccal smear reveals absence of Barr bodies in the nucleus of epithelial cells. A urine pregnancy test is negative. Which of the following genetic disorders is the most likely cause of this patient’s condition?

Palau syndrome

Cri du chat ("cat-cry”) syndrome

Edwards syndrome

Klinefelter syndrome

Turner syndrome

3754 / 6854
A 38-year-old woman, who was diagnosed with systemic lupus erythematosus (SLE) 3 years ago, comes to her physician with a complaint of facial swelling and decreased urination that she first noticed 2 weeks ago. She currently takes azathioprine and corticosteroid. Her vital signs show blood pressure 150/90 mm Hg, pulse – 91/min., temperature – 36.8 C and respiratory rate – 15/min. On physical examination, the doctor notices erythematous rash on her face exhibiting a butterfly pattern. The laboratory studies reveal hypertriglyceridemia and proteinuria. Which of the following is the most likely mechanism of SLE`s complication in this patient?

Decrease in renal blood flow (ischemic nephropathy)

Increased plasma oncotic pressure

Immune complex-mediated glomerular disease

Acute infection of the kidney

3755 / 6854
A 37-year-old man is admitted to hospital with mental confusion and disorientation. His wile reports he became more irritable and forgetlul in the past year. In addition, she notes that be became a vegan a year ago and currently, his diet consists of starchy foods like potatoes, corn and leafy vegetables. GI symptoms include anorexia, diarrhea and vomiting. He has glossitis and skin lesions that appear as vesicles over the extremities. Eczemalike lesions around the mouth, as well as desquamation and roughened skin over the hands are also present. Neurologic examination reveals symmetrical hypesthesia for all types of sensation in both upper and lower extremities in a ’’gloves and socks” distribution. Deficiency in diet of which of the following amino acids is the most likely cause of this condition?

Tryptophan

Histidine

Lysine

Arginine

Threonine

3756 / 6854
A 60-year old man with a history of hypertension, diabetes and hyperlipidemia had a sudden onset a right-sided weakness. By the time the ambulance arrived, he had difficulty speaking. Unfortunately, the patient died within the next two hours and an autopsy was performed immediately. The gross examination of the cerebral left hemisphere showed brain swelling, widened, gyri and poorly demarcated gray-white junction. Which of the following is the most likely cause of this patient`s death?

Abscess

Intracerebral hemorrhage

Cyst

Tumor

Ischemic stroke

3757 / 6854
A 34-year-old-man visits dentist complaining of toothache. After a dental procedure that onvolved extraction of several teeth, he develops severe bleeding lasting more than 15 minutes. He has a history of chronic hepatitis. Which of the following is the most likely cause of prolonged bleeding in this patient?

Hypocalcemia

Hypoalbuminemia

Trombocytopenia

Hypofibrinogenemia

3758 / 6854
A 43-year-old man seeks evaluation at emergency department with complaints of fever with chills, melaise, diffuse abdominal pain for over a week, diarrhea and loss of appetite. He says that his symptoms are progressively getting worse. He recalls that the fever began slowly and climbed its way- up stepwise to the current 39.8°C. His blood pressure is 110/70 mm Hg. A physical exam reveals a coated tongue, enlarged spleen and rose spots on the abdomen. Serologic study shows the agglutinin O titre of 1:200 by the Widal test. Which of the following is the most likely causative organism for this patient's condition?

Leptospira interrogans

Enterohemorrhagic E. coli

Mycobacterium tuberculosis

Salmonella typhi

Vibrio cholerae

3759 / 6854
A 64-year-old man presents with a tremor in his legs and arms. He says he has had the tremor for "many years", but it has worsened in the last year. The tremor is more prominent at rest and nearly disappears on movement. His daughter mentions that his movements have become slower. The patient is afebrile and vital signs are within normal limits. On physical examination, the patient is hunched over and his face is expressionless throughout examination. There is a "pillrolling" resting tremor that is accentuated when the patient is asked to clench the contralateral hand and alleviated by finger nose testing. When asked to walk across the room, the patient has difficulty taking the first step, has a stooped posture and takes short rapid shuffling steps. A doctor initiates pharmacotherapy and the drug of first line, levodopa, is prescribed. Which of the following is the most likely mechanism of action of this drug?

Activation of M2-cholinergic receptors

Inhibition of M2-cholinergic receptors

Cholinesterase inhibition

Stimulation of dopamine production

3760 / 6854
A 65-year-old woman presents to the emergency department because of shortness of breath and chest pain that started a few hours ago. She did not have a lever, expectoration, or any accompanying symptoms. She has a history of right leg deep vein thrombosis that occurred 5 years ago. Some time later, she dies of severe respiratory distress. A pulmonary autopsy specimen reveals red loose mass that is lodged in the bifurcation of the pulmonary trunk with extensions into both the left and right main pulmonary arteries. Which of the following is the most likely diagnosis?

Pneumothorax

Pneumonia

Myocardial infarction

Tromboembolism

3761 / 6854
An 11-year-old girl is brought to the doctor’s office by her mother who stales her daughter has been weak with swollen face for 5 days. The mother slates her daughter had always been healthy and active until the initiation of symptoms. Upon inquiry, the girl describes a foamy appearance of her urine but denies blood in urine, urinary frequency at night, or pain during urination. Physical examination reveals generalized swelling of the face and pitting edema on the lower limbs. Laboratory study shows proteinuria and microscopic hematuria. Which of the following is the most likely cause of findings in the laboratory study of urine?

Increased permeability across the glomerular capillary wall

Increased plasma oncotic pressure

Increased glomerular hydrostatic pressure

Increased hydrostatic pressure in Bowman's capsule

3762 / 6854
A group of researchers aimed to study cardiac physiology found that overstretching of atria in the heart leads to decreased sodium reabsorption in the distal convoluted tubule and increase in glomerular filtration rate. Which of the following is the most likely cause ot physiologic effects discovered by researchers?

Renin

Aldosterone

Angiotensin

Natriuretic peptide

Antidiuretic hormone

3763 / 6854
After dehelmintization, a 35-year-old man passed a 3.5 m tapeworm during a bowel movement. A stool examination reveals scolex with four suckers and hooks. Mature proglottids are static with up to 12 primary uterine branches. Which of the following is the most likely diagnosis?

Opisthorchiasis

Diphyllobothriasis

Echinococcosis

Taeniasis

3764 / 6854
A 14-year old girl presents to the emergency department for evaluation of an "infected leg". She states there is no history of trauma but mentions she had a history of sickle cell disease. On physical examination, her upper part of right shin is very painful, red, swollen and hot. Her temperature is 39.2°C. An X-ray shows focal bony lysis and loss of trabecular architecture in the metaphysis of right tibia, increased activity of which of the following cells is the most likely cause of bone reabsorption in this patient?

Osteoblasts

Chondrocytes

Osteoclasis

Chondroblasts

Osteocytes

3765 / 6854
A male neonate born to a 24-year-old primigravida had jaundice at 8 hours f life The neonates red blood cell time was A+, while the mothers RBC type was 0+. Laboratory studies revealed elevated titer of mother's anti-A antibody, normal erythrocyte glucose-6-phosphate and negative sickle cell test. The infant’s hemoglobin was 106 g/L. Which of the following is the most likely cause of infant’s jaundice?

Decrease in hemoglobin level

Glucose-6-phosphate dehydrogenase (G6PD) deficiency

Sickle cell disease

Hyperbilirubinemia

Rh incompatibility

3766 / 6854
A 16-year-old girl concerned about her sexual development comes to the physician. She mentions that she has still not had a menstrual period. However, she is otherwise a healthy girl with no significant medical problems since birth. On physical examination, her vital signs are stake. She does not have pubic hair and her breast fe sliehtlv elevated with areola remaining in contour with surrounding breast. Which of the following is the most likely cause of this abnormal physical development?

Adrenal medulla hyperfunction

Hypothyroidism

Pancreatic islet insufficiency

Ovarian insufficiency

Hyperthyroidism

3767 / 6854
A 46-year-old man presents with fatigue and joint pain in his lingers and wrists lor the last 2 months. The pain is present in both hands and the wrists are swollen. Furthermore, he describes morning stiffness in his joints lasting about 2 hours, which improves with use. His past medical history reveals he has been successfully treated for H. pylori related ulcers last year. He denies smoking and stopped drinking when his gastric symptoms started. Which of the following drugs is the best choice for his joint`s pain management?

Aspirin

Paracetamol

Celecoxib

Morphine

Prednisone

3768 / 6854
A 68-year-old man comes to his physician with complaints of severe fatigue and altered sensations in his extremities. Past medical history is remarkable for chronic gastritis. He drinks alcohol almost every day. His blood pressure is 130/80 mm Hg, heart rate is 95/min., respiratory rate -14/min. and temperature of 37.1 C. His heart has a regular rate and rhythm, his lungs are clear to auscultation bilaterally. Neurologic examination reveals loss of touch and vibration sense in both upper and lower limbs. Laboratory investigation results include a hemoglobin of 80 g/L, Mean Corpuscular Volume (MCV) of 115 fL (the reference range is 80-100 fL) and White Blood Cells (WBC) of 3.0=109/L. Which of the following is the most likely diagnosis?

Vitamin A deficiency

Viitamin B12 deficiency

Vitamin C deficiency

Iron deficiency

3769 / 6854
A 6-year-old boy is brought to the pediatrician by his mother, who complains of low-grade fever, chronic cough and night sweats in her child. She describes the cough as productive, producing white sputum that is sometimes streaked with blood. She also says thet her son has lost some weight in the last month. His vital sings include blood pressure of 115/75 mm Hg, heart rate of 110/min., respiratory rate of 18/min and temperature ot 36.6 C. On physical examination, the patient is ill looking. Pulmonary auscultation reveals some fine crackles 'in the right upper lobe. The pediatrician suspects an active infection and performs Mantoux test. Intradermal injection of which of the following substances has been most likely used by pediatrician for screening test in this clinical case?

Diphtheria-tetanus toxoids-acellular pertussis vaccine (DTaP)

Tetanus and diphtheria toxoids vaccine (Td)

Bacillus Calmcttc-Guerin (BCG) vaccine

Tuberculin

3770 / 6854
A 37-year old female presents to the clinic complaining of severe pain in her left wrist and tingling sensation in her left thumb, index finger, and middle finger, and some part of her ring finger. Hie pain started as an occasional throb and she could ignore it or take ibuprofen but now the pain is much worse and wakes her up at night. She works as a typist and her pain mostly increases alter typing all day. Her right wrist and fingers are line Nerve conduction studies reveal ncive compression. Which of the following nerves is most likely compressed in this patient?

Musculocutaneous nerve

Radial nerve

Median nerve

Axillary nerve

Ulnar nerve

3771 / 6854
A 45-year-old woman comes to her physician with complaints of excessive fatigue and weakness. She says that these symptoms have been present for the past month. On further questioning, she admits having lost 3 kilograms in the last 2 weeks. On physical examination, she is a tired-appearing thin woman. Hyperpigmentation is present over many areas of her body, most prominently over the face, neck and back of hands (areas exposed to light). Increased production of which of the following hormones is the most likely cause of hyperpigmentation in this patient?

Growth hormone (GII)

B-Lipolropin

Melanocytc-stimulating hormone (MSH)

Thyroid-stimulaiing hormone (TSII)

Gonadotropins

3772 / 6854
A 40-year-old woman dies of intracerebral hemorrhage alter hypertensive emergency. During an autopsy, the pathologist reveals severe obesity, excess of body hair and wide purplish stria on the abdomen. Microscopic examination of pituitary gland reveals hyperplastic acini populated by a homogenous cluster of deeply basophilic cells. Which of the following was the most likely underlying disease

Hyperthyroidism

Sheehan's syndrome

Cushing disease

Arterial hypertension

3773 / 6854
A 56-year-old man presents for a checkup. The patient says he has to urinate quite frequently, but denies any dysuria or pain on urination. Past medical history is significant for diabetes mellitus type 2 and hypertension, both managed medically. Current medications are metformin, aspirin, rosuvastaiin. captopril and furosemide Laboratory findings arc significant for the following: Glycated Hemoglobin (Hb A1c) - 8.0%. Fasting Blood Glucose - 12 mmol L. His doctor decides to add glibenclamide to the therapy. Which of the following is the most likely mechanism of this drug's action?

Stimulation of insulin release

Inhibition of insulin release

Stimulation of glucose reuptake by the cell

3774 / 6854
After dehelmintization, a 35-year-old man passed a 3.5 m tapeworm during a bowel movement. A stool examination reveals scolex with four suckers and hooks. Mature proglottids are static with up to 12 primary uterine branches. Which of the following is the most likely diagnosis?

Taeniasis

Opisthorchiasis

Diphyllobothriasis

Echinococcosis

3775 / 6854
A mother of a 4-month-old male infant brought him to pediatrician with complaints of food rejection and weight loss. He started having trouble latching onto his bottle. He has also become extremely lethargic. Examination reveals diminished muscle tone in all four limbs, and hepatosplenomegaly. An ophthalmoscopic exam reveals macular cherry red spots. During the next few weeks, hepatosplenomegaly progresses, the boy fails to thrive, and he continues to reject food. Chest X-ray shows a reticulonodular pattern and calcified nodules. Biopsy of the liver shows foamy histiocytes. A Niemann-Pick disease is suspected. Which of the following is the most likely deficient enzyme in this patient?

Sphingomyelinase

Glucocerebrosidase

Phenylalanine-hydroxylase

Glucose-6-phosphatase

Galactocerebrosidase

3776 / 6854
An unidentified surgical specimen is received for histopathologic analysis. A portion of the specimen is cut and stained with hematoxylin and eosin. Under the microscope, you see an organ encapsulated by dense connective tissue that extends to the deeper areas by way of the trabecular extensions. The organ can be subdivided into two regions: a cortex with lymphoid nodules and medulla with medullary cords populated by plasma cells, B-cells and T-cells. Which of the following structures is most likely the origin of this surgical specimen?

Thymus

Bone marrow

Lymph node

Tonsils

Spleen

3777 / 6854
A team of medical students is performing research on phases of cell cycle. During one of the mitotic phases the cell is nearly done dividing, the chromosomes decondense and two nuclei begin to form around them. Which of the following phases most likely takes place in ihe cell?

Prophase

Metaphase

Anaphase

Telophase

3778 / 6854
A 20-year-old female comes to the clinic after missing her last 2 periods. Her cycles are usually regular, occurring at 28-30 day interval with moderate bleeding and some abdominal discomfort. She also complains of progressively diminishing peripheral vision. Her doctor reveals loss of vision in the lateral halves of both eyes. Involvement of which of the following structures would you most likely expect to be the reason of bitemporal hemianopsia?

Right optic tract

Left optic nerve

Optic chiasm

Left optic tract

Right optic nerve

3779 / 6854
A 54-year-old woman has a total thyroidectomy for papillary thyroid carcinoma. 11 hours after operation she complains of tingling around her mouth. On physical examination, the Trousseau's sign and Chvostek’s sign are present. Her condition rapidly deteriorates with laryngospasm and focal seizures. The suigeon suggests surgical destruction of parathyroid glands. Which of the following is the most likely cause of this patient`s neurologic abnormality?

Hyperkalemia

Hyperchloremia

Hyponatremia

Hypocalcemia

Hypophosphatemia

3780 / 6854
Хворому призначена ендоскопія 12-палої кишки. В результаті виявлено запалення великого дуоденального сосочка і порушення виділення жовчі в просвіт кишки. У якому відділі 12-палої кишки виявлені порушення?

Верхня частина

Цибулина

Висхідна частина

Низхідна частина

Горизонтальна частина

3781 / 6854
При аналізі ЕКГ встановлено: ритм синусовий, число передсердних комплексів перевищує число шлуночкових комплексів; прогресуюче подовження інтервалу Р-Q від комплексу до комплексу; випадання окремих шлуночкових комплексів, через що після зубця Р йде довга пауза; зубці Р та комплекси QRST без змін. Назвіть тип порушення серцевого ритму:

Неповна атріовентрикулярна блокада І ступеня

Повна атріовентрикулярна блокада

Неповна атріовентрикулярна блокада II ступеня

Неповна атріовентрикулярна блокада III ступеня

Синоатріальна блокада

3782 / 6854
Чоловік 55-ти років, що скаржиться на біль в ділянці нирок, надійшов в лікарню. В ході ультразвукового обстеження пацієнта виявлено наявність ниркових каменів. Наявність в сечі якої з наведених речовин є найімовірнішою причиною утворення каменів у цього пацієнта?

Креатинін

Уробілін

Білірубін

Білівердин

Сечова кислота

3783 / 6854
У 70-річної людини швидкість поширення пульсової хвилі виявилася суттєво вище, ніж у 25-річної. Причиною цього є зниження:

Артеріального тиску

Еластичності судинної стінки

Швидкості кровотоку

Частоти серцевих скорочень

Серцевого викиду

3784 / 6854
Чоловік внаслідок транспортної аварії втратив багато крові, свідомість затьмарена, низький кров’яний тиск. При цьому у нього компенсаторно активується ренін-ангіотензинова система, що веде до:

Посилення еритропоезу

Гіперпродукції альдостерону

Посилення серцевих скорочень

Гіперпродукції вазопресинуАпо В48

Підвищення згортання крові

3785 / 6854
При дослідженні людини у вертикальній позі встановлено, що в альвеолах верхівок легень парціальний тиск кисню становить 140 мм рт.ст. Причиною цього є те, що у цих відділах легень:

Вентиляція переважає над перфузією

Перфузія та вентиляція врівноважені

Перфузія переважає над вентиляцією

Вентиляція відсутня

3786 / 6854
Хворий протягом останнього року став відзначати підвищену втомлюваність, загальну слабкість. Аналіз крові: еритроцити -4,1•1012/л, Нb-119 г/л, к.п.- 0,87, лейкоцити - 57•109/л, лейкоформула: Ю- 0, П- 0, С- 9%, Е- 0, Б- 0, лімфобласти - 2%, пролімфоцити - 5%, лімфоцити - 81%, М- 3%, тромбоцити - 160•109/л. В мазку: нормохромія, велика кількість тіней Боткіна-Гумпрехта. Про яку патологію системи крові свідчить дана гемограма?

Хронічний монолейкоз

Хронічний мієлолейкоз

Гострий лімфобластний лейкоз

Гострий мієлобластний лейкоз

Хронічний лімфолейкоз

3787 / 6854
У хворого з синдромом Іценка- Кушинга спостерігаються стійка гіперглікемія та глюкозурія. Синтез та секреція якого гормону підвищені у цього хворого?

Тироксин

Альдостерон

Кортизол

Адреналін

Глюкагон

3788 / 6854
Для лікування стрептодермії лікар призначив хворому мазь, що містить антибіотик групи тетрацикліну, і рекомендував обмежити перебування на сонці. Чим небезпечна інсоляція?

Розвитком стійкості збудника до антибіотика

Зниженням активності тетрацикліну

Генералізацією процесу

Підвищенням токсичності тетрацикліну

Розвитком фотосенсибілізації

3789 / 6854
На аутопсії тіла жінки, що хворіла на хронічну дизентерію, в ході гістологічного дослідження внутрішніх органів в стромі та паренхімі міокарда, нирок, в слизовій оболонці шлунка та в сполучній тканині легень виявлені аморфні відкладення фіолетового кольору, що дають позитивну реакцію за Коссом. Яке ускладнення розвинулося у хворої'?

Метаболічне звапніння

Гіаліноз

Амілоїдоз

Дистрофічне звапніння

Метастатичне звапніння

3790 / 6854
У 12-річного хлопчика в сечі виявлено високий вміст усіх амінокислот аліфатичного ряду. При цьому відзначена найвища екскреція цистину та цистеїну. Крім того, УЗД нирок показало наявність у них каменів. Виберіть можливу патологію:

Фенілкетонурія

Цистит

Цистинурія

Алкаптонурія

Хвороба Хартнупа

3791 / 6854
При мікроскопічному дослідженні біоптату нирки виявлено вогнища, в центрі яких розташовані зернисті еозинофільні маси, оточені інфільтратом з лімфоцитів, епітеліоїдних клітин та поодиноких клітин Пирогова-Лангханса. Виберіть патологічний процес, що найповніше відповідає зазначеним змінам:

Гранулематозне запалення

Казеозний некроз

Коагуляційний некроз

Альтеративне запалення

Проліферація та диференціювання макрофагів

3792 / 6854
У дитини 8-ми років через 5 днів після контакту з хворим на вітряну віспу з’явилося нездужання, підвищилася температура, незабаром з’явилися характерні висипи. Який термін найточніше характеризує період, що передував появі перших ознак захворювання?

Контагіозний

Латентний

Інкубаційний

Неспецифічний

Інфекційний

3793 / 6854
Хворому, після радіоактивного опромінення, лікар рекомендував збільшити в раціоні вміст рослинних олій - джерела полієнових жирних кислот. Назвіть кислоту, що містить три подвійних зв’язки:

Олеїнова

Лауринова

Ліноленова

Арахідонова

Міристинова

3794 / 6854
Хвора 55-ти років з гострим нападом печінкової кольки надійшла в гастроентерологічне відділення. Об’єктивно: температура тіла - 38°С, склери, слизові та шкіра іктеричні, сеча темна, кал безбарвний. Скарги на свербіння шкіри. Яка причина жовтяниці у цієї хворої?

Тривале вживання продуктів, багатих на каротин

Деструкція гепатоцитів

Порушення ліпідного обміну

Обтурація жовчних проток

Посилений розпад еритроцитів

3795 / 6854
Хвора 57-ми років для лікування гіпертонічної хвороби тривалий час приймала анаприлін. Побічні ефекти спонукали пацієнтку відмовитись від прийому препарату, що призвело до розвитку гіпертонічного кризу і нападу стенокардії. Як називається ускладнення, яке виникло?

Тахіфілаксія

Лікарська залежність

Сенсибілізація

Звикання

Синдром відміни

3796 / 6854
У хворої після операції видалення ракової пухлини молочної залози та регіонарних лімфовузлів розвинувся набряк руки, який був пов’язаний з лімфатичною недостатністю. Який це вид лімфатичної недостатності за механізмом виникнення?

Механічна

Спастична

Резорбційна

Динамічна

Акінетична

3797 / 6854
У жінки, що тривало приймала антибіотики з приводу кишкової інфекції, розвинулось ускладнення з боку слизової порожнини рота у вигляді запального процесу і білого нальоту, у якому під час бактеріологічного дослідження були виявлені дріжджеподібні грибки Сапdіda аlbісаns. Який з перерахованих препаратів показаний для лікування цього ускладнення?

Флуконазол

Бісептол

Фуразолідон

Поліміксин

Тетрациклін

3798 / 6854
В результаті експресії окремих компонентів геному клітини зародка набувають характерних для них морфологічних, біохімічних та функціональних особливостей. Яку назву має цей процес?

Рецепція

Капацитація

Диференціювання

Детермінація

Індукція

3799 / 6854
Жінка 54-х років звернулася до лікаря зі скаргами на непереносимість курячих яєць, що з’явилася нещодавно. Антигістамінні препарати, призначені лікарем, дещо покращували стан хворої. Які антитіла могли сприяти розвитку цієї реакції?

IgA

ІgМ

IgE

ІgD

IgG

3800 / 6854
В результаті інфікування рани після видалення зуба у хворого виникло її нагноєння. Для очищення рани, як один із засобів, рекомендовано полоскання рота гіпертонічним розчином NaCl. Який процес лежить в основі цього способу очищення?

Гемостатична дія

Денатурація білка

Бактерицидна дія

Осмотична активність солі

Механічне очищення

3801 / 6854
Під час підйому пішки на 5-й поверх у людини підвищився артеріальний тиск. Причиною є збільшення:

Вмісту іонів в плазмі крові

Кількості функціонуючих капілярів

В’язкості крові

Хвилинного об’єму крові

Об’єму циркулюючої крові

3802 / 6854
На 9-й день після введення протиправцевої сироватки з приводу брудної рани стопи в пацієнта підвищилася температура тіла до 38°С, з’явився біль у суглобах, висип, свербіння шкіри. З боку крові спостерігаються лейкопенія і тромбоцитопенія. Який тип алергічної реакції розвинувся?

Імунокомплексний

Стимулюючий

Цитотоксичний

Анафілактичний

Клітинно-опосередкований

3803 / 6854
Тварині в експерименті перерізали передні корінці п’яти сегментів спинного мозку. Які зміни відбудуться в зоні іннервації?

Гіперчутливість

Втрата пропріоцептивної чутливості

Втрата температурної чутливості

Втрата дотикової чутливості

Втрата рухів

3804 / 6854
У потерпілого травма черепа з порушенням дірчастої пластинки решітчастої кістки. Який нерв може бути ушкоджений?

N. oрthalmicus

N.opticus

N. trосhlіаris

N.abducens

N. оlfactorius

3805 / 6854
У хворого 50-ти років через місяць після перенесеного інфаркту міокарда розвинулася серцева недостатність. Невідповідність між навантаженням на серце та його здатністю виконувати роботу пов’язана в цьому випадку з:

Змішаною формою недостатності серця

Перевантаженням серця

Ушкодженням міокарда

Підвищеним опором вигнанню крові в легеневому стовбурі

Підвищеним опором вигнанню крові в аорті

3806 / 6854
У хворого після резекції шлунка розвинулась мегалобластична анемія. Який препарат необхідно призначити хворому?

Вітамін В6

Ферковен

Аскорбінову кислоту

Вітамін В12

Заліза лактат

3807 / 6854
У жінки 62-х років розвинулася катаракта (помутніння кришталику) на тлі цукрового діабету. Вкажіть, який тип модифікації білків має місце при діабетичній катаракті:

АДФ-рибозилювання

Обмежений протеоліз

Фосфорилювання

Метилювання

Глікозилювання

3808 / 6854
Розділ фармакології, що вивчає всмоктування, розподіл та елімінацію ліків в організмі людини, називається:

Фармакокінетика

Токсикологія

Імунофармакологія

Фармакодинаміка

Фармакогенетика

3809 / 6854
При пульмонологічному обстеженні виникла необхідність визначити частину повітря, яка обмінюється в легенях за один дихальний цикл. Цей показник називається:

Коефіцієнт легеневої вентиляції

Дихальний коефіцієнт

Об’єм мертвого простору

Функціональна остаточна ємність

Хвилинна легенева вентиляція

3810 / 6854
Для кращого огляду дна очного яблука лікар закрапав в кон’юнктиву ока пацієнта розчин атропіну. Це призвело до розширення зіниці через блокаду таких мембранних циторецепторів:

Бета-адренорецепторів

Н2-рецепторів

Альфа-адренорецепторів

Н-холінорецепторів

М-холінорецепторів

3811 / 6854
Жінка 38-ми років надійшла в хірургічне відділення з приводу розлитого гнійного перитоніту. Під час операції виявлена перфорація виразки клубової кишки, виразку було ушито. Черевну порожнину дреновано. Через 4 дні хвора померла. На розтині в клубовій кишці наявні виразки в пеєрових бляшках, розташовані уздовж кишки, дно деяких з них має поперечну смугастість. Для якого захворювання характерні такі виразки?

Амебіазу кишечника

Дизентерії

Черевного тифу

Туберкульозу кишечника

Пухлини тонкої кишки

3812 / 6854
Хворий помер від наростаючої легенево-серцевої недостатності. При гістологічному дослідженні виявлено дифузне ураження легенів з інтерстиціальним набряком та інфільтрацією інтер-стиціальної тканини лімфоцитами, макрофагами, плазмоцитами; наявні пневмофіброз та панацинарна емфізема. Поставте діагноз:

Фіброзивний альвеоліт

Ателектаз легенів

Бронхопневмонія

Бронхіальна астма

Хронічний бронхіт

3813 / 6854
В процесі розвитку у дитини хребет поступово набув два лордози та два кіфози. Це пояснюється розвитком здатності до:

Сидіння

Лежання

Плавання

Повзання

Прямоходіння

3814 / 6854
Судово-медичний експерт при розтині тіла 20-річної дівчини встановив смерть внаслідок отруєння ціанідами. Порушення якого процесу, найімовірніше, стало причиною смерті?

Транспорту амінокислот

Синтезу сечовини

Тканинного дихання

Синтезу гемоглобіну

Транспорту кисню гемоглобіном

3815 / 6854
В лікарню звернувся 65-річний хворий зі скаргами на часте сечовиділення зі слідами крові. При посіві сечі на МПА виросли великі колонії з характерним квітковим запахом, які мали зелено- синій пігмент. В мазку з колоній наявні грамнегативні палички. Який збудник спричинив запальний процес сечостатевої системи?

Pseudomonas aeruginosa

Hafnia alvei

Proteus vulgaris

Escherichia coli

Кlebsiella ozaenae

3816 / 6854
Чоловік з гострим міокардитом помер від серцево-судинної недостатності. В ході мікроскопічного дослідження внутрішніх органів виявлені: плазморагія, набряк, стази в капілярах, численні крововиливи, а також дистрофічні зміни в паренхімі. Наслідком чого є дані зміни?

Місцеве артеріальне повнокрів’я

Гострий загальний венозний застій

Хронічний загальний венозний застій

Загальне артеріальне повнокрів’я

ДВ3-синдром

3817 / 6854
В епідермісі є клітини, що виконують захисну функцію та мають моноцитарний генез. Які це клітини?

Клітини Лангерганса

Кератиноцити остистого шару

Меланоцити

Кератиноцити базального шару

Кератиноцити зернистого шару

3818 / 6854
Чоловік 40-ка років скаржиться на загальну слабкість, головний біль, кашель із виділенням мокротиння, задишку. Після клінічного огляду й обстеження поставлено діагноз пневмонія. Який тип гіпоксії має місце у хворого?

Циркуляторна

Тканинна

Респіраторна

Гіпоксична

Гемічна

3819 / 6854
У підлітка після перенесеного інфекційного захворювання з’явилася різко виражена аритмія з вкороченням інтервалу R-R під час вдиху і подовженням його під час видиху. Що лежить в основі цього виду аритмії?

Порушення скоротливої функції серця

Порушення функції збудливості серця

Порушення функції провідності серця

Коливання тонусу блукаючого нерва під час акту дихання

Рефлекс Бейнбріджа

3820 / 6854
Жінці 58-ми років проведене повне видалення матки з придатками, після чого виділення сечі припинилося. При цистоскопії: міхур сечі не містить, з устів сечоводів сеча не надходить. Який відділ сечовидільної системи було ушкоджено в ході операції?

Pelvis renalis

Vesica urinaria

Rеп

Uretra

Ureter

3821 / 6854
У чоловіка 30-ти років перед операцією визначили групову належність крові. Кров резус-позитивна. Реакцію аглютинації еритроцитів не викликали стандартні сироватки груп 0αβ (І), Аβ (II), Вα (III). Досліджувана кров належить до групи:

Вα (III)

Aβ (II)

0αβ (І)

АВ (IV)

3822 / 6854
У лікарню машиною швидкої допомоги доставлено хворого в стані коми. В анамнезі цукровий діабет. При обстеженні спостерігається шумне прискорене дихання, при якому глибокі вдихи чергуються з посиленими видохами за участю експіраторних м’язів. Яка форма порушення зовнішнього дихання спостерігається?

Апнейстичне дихання

Дихання Біота

Стенотичне дихання

Дихання Чейна-Стокса

Дихання Куссмауля

3823 / 6854
У хворого після черепно-мозкової травми, під час якої була ушкоджена мозочкова ділянка, розвинулися порушення часової та просторової координації рухів. Яка патологія розвинулася у хворого?

Абазія

Парез

Атаксія

Астазія

Дисметрія

3824 / 6854
Пацієнт 55-ти років звернувся до лікаря зі скаргами на часті судоми. Встановлено, що тривалий час він працює у гарячому цеху в умовах високих температур. Порушення якого виду обміну призвело до цього стану?

Білкового

Водно-сольового

Ліпідного

Вітамінного

Вуглеводного

3825 / 6854
У результаті радіаційного випромінювання були ушкоджені стовбурові гемопоетичні клітини. Утворення яких клітин сполучної тканини буде порушено?

Меланоцити

Адипоцити

Фібробласти

Макрофаги

Перицити

3826 / 6854
У потерпілого пошкоджений м’яз, апоневроз якого утворює пахвинну зв’язку. Назвіть цей м’яз:

Внутрішній косий м’яз живота

Пірамідальний м’яз

Прямий м’яз живота

Зовнішній косий м’яз живота

Поперечний м’яз живота

3827 / 6854
При обстеженні хворого похилого віку виявлено моторну афазію. Де локалізований осередок пошкодження головного мозку?

Постцентральна звивина

Кутова звивина

Звивина Гешля

Центр Брока

Прецентральна звивина

3828 / 6854
У сільського жителя на кисті правої руки з’явилася неболюча припухлість, яка набула в центрі чорного кольору. При мікроскопічному дослідженні виявлено великі грампозитивні палички, розташовані ланцюжками. Який мікроорганізм міг спричинити це захворювання?

Bacillus anthracis

Clostridium tetani

Clostridium botulinum

Mycobacterium tuberculosis

Bacillus cereus

3829 / 6854
Симбіотична теорія пояснює походження еукаріотичних клітин переходом до аеробного дихання. Це відбулося внаслідок проникнення в клітину аеробних бактерій, які в процесі еволюції перетворилися на:

Пероксисоми

Мітохондрїї

Рибосоми

Лізосоми

Комплекс Гольджі

3830 / 6854
Оглядаючи дитину 6-ти років, лікар помітив на глоткових мигдаликах сірувату плівку, при спробі видалення якої виникла помірна кровотеча. Бактеріоскопія мазків з мигдаликів показала наявність грампозитивних бактерій булавоподібної форми. Які симптоми можуть виникнути у дитини у найближчі дні, якщо не буде проведене специфічне лікування?

Папульозні висипи на шкірі

Токсичні ураження серцевого м’язу, печінки, нирок

Набряк легенів

Дуже сильний нападоподібний кашель

Хвилеподібна лихоманка

3831 / 6854
Деякі тяжкі захворювання нирок супроводжуються еритропенією. Який механізм цього явища найімовірніший?

Підвищене руйнування еритроцитів в селезінці

Порушення синтезу еритропоетинів

Збільшення діурезу

Порушення функціонування печінки

Аліментарний дефіцит Fе2+

3832 / 6854
У померлого, що понад 20 років працював на шахті з видобутку кам’яного вугілля, при розтині виявлено ущільнені легені сіро-чорного кольору зі значними ділянками новоутвореної сполучної тканини та наявністю великої кількості макрофагів із пігментом чорного кольору в цитоплазмі. Який з перерахованих діагнозів найімовірніший?

Антракоз

Силікоантракоз

Талькоз

Сидероз

Антракосилікоз

3833 / 6854
У крові резус-негативної жінки під час вагітності виявлені специфічні білки, здатні руйнувати резус-позитивні еритроцити плода. Як називається цей захисний компонент організму матері?

Резус-фактор

Сироватка

Гормон

Фактор некрозу пухлини

Антитіло

3834 / 6854
Хворий 50-ти років скаржиться на поліурію, спрагу, протягом доби випиває до 15 літрів рідини. При обстеженні виявлено: вміст глюкози крові - 4,8 ммоль/л, сеча безбарвна, відносна щільність - 1,002-1,004, цукор і білок відсутні. Яка імовірна причина поліурії?

Дефіцит альдостерону

Надлишок тиреоїдних гормонів

Дефіцит вазопресину

Надлишок альдостерону

Дефіцит тиреоїдних гормонів

3835 / 6854
У пацієнта перфоративна виразка передньої стінки шлунка. В яке похідне очеревини потрапить вміст шлунка?

Лівий брижовий синус

Передшлункова сумка

Чепцева сумка

Правий брижовий синус

Печінкова сумка

3836 / 6854
За умов тривалої інтоксикації тварин тетрахлорметаном було визначене суттєве зниження активності аміноацил- тРНК-синтетаз в гепатоцитах. Який метаболічний процес порушується в цьому випадку?

Реплікація ДНК

Посттранскрипційна модифікація РНК

Біосинтез білків

Посттрансляційна модифікація пептидів

Транскрипція РНК

3837 / 6854
У хворого зі швидко наростаючою внутрішньомозковою гіпертензією діагностована пухлина мозку. Під час операції видалена пухлина тім’яно-скроневої частки, м’якої консистенції, строката на розрізі. Гістологічно пухлина побудована з поліморфних гіперхромних клітин з утворенням псевдорозеток та великої кількості судин, ділянками некрозів і крововиливами. Поставте діагноз:

Астроцитома

Менінгіома

Гліобластома

Арахноїдендотеліома

Олігодендрогліома

3838 / 6854
Експериментатору необхідно якнайшвидше виробити умовний рефлекс у собаки. На базі якого безумовного рефлексу доцільно виробляти умовний?

Міотатичний

Орієнтувальний

Статевий

Захисний

Травний

3839 / 6854
У хворого 60-ти років під час об’єктивного обстеження виявлено набряки на ногах, асцит, збільшення печінки, що свідчить про недостатність кровообігу за правошлуночковим типом. В анамнезі перенесений ревматизм. Одним з основних факторів розвитку набряків є активація системи ренін-ангіотензин- альдостерон, яка є наслідком:

Зменшення хвилинного об’єму серця

Розширення посткапілярних вен

Утруднення дифузії речовин

Ацидозу

Поліцитемічної гіперволемїї

3840 / 6854
У п’ятимісячної дівчинки виявлено застійні явища у легенях. При обстеженні виявлено зв’язок між висхідною аортою та легеневою артерією, що в нормі спостерігається у деяких земноводних і плазунів. Назвіть цю природжену ваду розвитку:

Незрощення боталової протоки

Дефект міжшлуночкової перегородки

Транспозиція магістральних судин

Дефект міжпередсердної перегородки

Розвиток правої дуги аорти

3841 / 6854
Під час проведення морфологічного дослідження периферичної крові хворого було помічено, що у еритроцитів забарвлена лише периферична частина, а в центрі є незабарвлене прояснення. Кольоровий показник - 0,56. Яка анемія найімовірніша у цього пацієнта?

Залізодефіцитна

Апластична

Гемолітична

В₁₂ фолієводефіцитна

Сидеробластна

3842 / 6854
Під час хірургічної операції виникла необхідність масивного переливання крові. Група крові потерпілого - III (В) Rh(+). Якого донора треба вибрати?

І(0)Rh(-)

III (В) Rh(+)

III (В) Rh(-)

II (А) Rh(+)

IV (АВ) Rh(-)

3843 / 6854
У людини внаслідок лікування антибіотиками виник дисбактеріоз товстого кишечника. Яких вітамінів, синтезованих бактеріями в товстому кишечнику, буде менше надходити до організму?

Вітамін К та вітаміни групи В

Вітамін В

Вітаміни А та Е

Аскорбінова кислота

Вітаміни Р і С

3844 / 6854
Лікар швидкої допомоги встановив у хворого діагноз гіпертонічний криз. Для усунення цього стану хворому парентерально введено засіб, який крім гіпотензивної має також протисудомну дію, а при ентеральному введенні - проносну і жовчогінну. Назвіть цей препарат:

Пентамін

Натрію нітропрусид

Дибазол

Магнію сульфат

Клофелін

3845 / 6854
Жінка 62-х років скаржиться на частий біль грудного відділу хребта, переломи ребер. Лікар припустив мієломну хворобу (плазмоцитому). Який з перерахованих нижче лабораторних показників буде мати найбільше діагностичне значення?

Парапротеїнемія

Гіпопротеїнемія

Гіперальбумінемія

Протеїнурія

Гіпоглобулінемія

3846 / 6854
У чоловіка, померлого від внутрішньої кровотечі (гемоперитонеум), в печінці субкапсулярно виявлено губчастий вузол темно-червоного кольору розмірами 15x10 см, добре відмежований від навколишньої тканини. Мікроскопічно: тканина вузла складається з великих судинних тонкостінних порожнин, вистелених ендотеліальними клітинами та заповнених рідкою або згорнутою кров’ю. Встановіть вид пухлини:

Кавернозна гемангіома

Гемангіоперицитома

Венозна гемангіома

Капілярна гемангіома

Лімфангіома

3847 / 6854
Після пошкодження мозку у людини порушене сприйняття зорової інформації. В якому відділі кори сталося пошкодження?

Задня центральна звивина

Потилична ділянка кори

Скронева ділянка кори

Передня центральна звивина

Тім’яна ділянка кори

3848 / 6854
Новонароджений, який з’явився на світ у домашніх умовах без кваліфікованої медичної допомоги, і мати якого не проходила у період вагітності необхідних обстежень, на другу добу доставлений в лікарню з гострим гнійним кон’юнктивітом. При мікроскопії виділень з очей дитини знайдені грамнегативні диплококи, розташовані всередині лейкоцитів та поза клітинами. Який мікроорганізм є найімовірнішим збудником захворювання?

Staphylococcus aureus

Neisseria gonorrhoae

Соrynebacterіит dірhtheriае

Pseudomonas aeruginosa

Сhlamydia trachomatis

3849 / 6854
Фізіологи встановили, що кількість еритроцитів у крові залежить від функціонального стану червоного кісткового мозку й тривалості життя еритроцита. Який термін життя еритроцита в периферичній крові в середньому?

220 діб

120 діб

50 діб

70 діб

150 діб

3850 / 6854
Жінка 35-ти років звернулася до лікаря зі скаргами на дратівливість, тривожність, швидку втомлюваність, безсоння. Для усунення неврозу лікар призначив пацієнтці транквілізатор діазепам. Вкажіть фармакодинамічний ефект діазепаму, що дав можливість застосувати його за цих обставин:

Анксіолітичний

Міорелаксантний

Психостимулювальний

Протисудомний

Антипсихотичний

3851 / 6854
В колективі дошкільної установи одночасно виявили декілька дітей з катаральним запаленням слизової оболонки трахеї та бронхів. За клінічними проявами припущено коклюшну інфекцію. Яким чином, як правило, передається коклюш?

Аліментарним

Повітряно-пиловим

Повітряно-краплинним

Контактним

Трансмісивним

3852 / 6854
У хворого на тромбофлебіт нижніх кінцівок з’явилися біль в грудній клітці, кровохаркання, наростаюча дихальна недостатність, при явищах якої він помер. На розтині діагностовані множинні інфаркти легенів. Яка найімовірніша причина їх розвитку в цьому випадку?

Тромбоз гілок легеневої артерії

Тромбоемболія бронхіальних артерій

Тромбоз бронхіальних артерій

Тромбоемболія гілок легеневої артерії

Тромбоз легеневих вен

3853 / 6854
У хворого видалили щитоподібну залозу, яка була значно збільшена в розмірах, щільно-еластичної консистенції, з горбистою поверхнею. При гістологічному дослідженні в паренхімі залози визначається дифузна лімфо-плазмоцитарна інфільтрація з формуванням лімфоїдних фолікулів з гермінативними центрами, атипія та метаплазія фолікулярного епітелію, вогнища склеротичних змін паренхіми. Який діагноз найімовірніший?

Аутоімунний тиреоїдит

Вузловий зоб

Тиреоїдит де Кервена

Дифузний тиреотоксичний зоб

Дифузний еутиреоїдний зоб

3854 / 6854
У патогенезі розвитку II типу гіперліпопротеїнемії (сімейна гіперхолестеролемія) провідну роль відіграє дефіцит рецепторів до апобілка ЛПНЩ. Назвіть його:

Апо СІІ

Апо ВСІ

Апо В48

Апо В100

Апо А1

3855 / 6854
Хвороба Куру характеризується тремором і атаксією; при хворобі Крейтцфельдта-Якоба спостерігаються розвиток атаксії і деменції. Доведено, що ці захворювання викликаються:

Пріонами

Грибами

Бактеріями, позбавленими клітинної стінки

Повільними вірусами

Токсичними продуктами навколишнього середовища

3856 / 6854
У хворої діагностовано пухлину головки підшлункової залози, порушення венозного відтоку з деяких органів черевної порожнини. Яка венозна судина була здавлена пухлиною?

Нижня порожниста вена

Ниркова вена

Ліва шлункова вена

Ворітна вена печінки

Права шлункова вена

3857 / 6854
У дитини спостерігаються невротичні симптоми: запаморочення, слабкість, головний біль, який супроводжується нудотою, болем в правому підребер’ї, частими позивами на дефекацію. При лабораторному дослідженні дуоденального вмісту виявлено грушоподібні най-простіші з двома ядрами, 4-ма парами джгутиків, а у фекаліях овальної форми цисти. Яке захворювання у дитини?

Токсоплазмоз

Амебіаз

Балантидіаз

Кишковий трихомоноз

Лямбліоз

3858 / 6854
Психологічне дослідження встановило: у людини добра здатність швидко пристосовуватися до нового оточення, добра пам’ять, емоційна стійкість, висока працездатність. Найімовірніше, ця людина:

Меланхолік

Холерик

Флегматик

Сангвінік

Флегматик з елементами меланхоліка

3859 / 6854
В організмі людини визначено порушення обміну мелатоніну. Це може бути пов’язано з нестачею амінокислоти, з якої мелатонін синтезується. Яка це амінокислота?

Триптофан

ДОФА

Аланін

Глутамат

Гістидин

3860 / 6854
У хворих із синдромом набутого імунодефіциту (СНІД) різко знижується імунологічна реактивність, що проявляється розвитком хронічних запальних процесів, інфекційних захворювань, пухлинного росту. Клітини якого типу ушкоджує ВІЛ-інфекція, внаслідок чого знижується імунний захист?

Природні кілери (NК)

В-лімфоцити

Т8-ефектори

Т-супресори

Т4-хелпери

3861 / 6854
При глікогенозі (хворобі Гірке) пригнічується перетворення глюкозо-6- фосфату в глюкозу, що супроводжується порушенням розпаду глікогену в печінці. Дефіцит якого ферменту є причиною цього захворювання?

Глюкозо-6-фосфатази

Фосфофруктокінази

Фосфоглюкомутази

Глюкозо-6-фосфатдегідрогенази

Глікогенфосфорилази

3862 / 6854
У хворого шкіра чутлива до сонячного світла. Назвіть це спадкове захворювання, зумовлене дефектами ферментів системи репарації ДНК:

Порфірія

Альбінізм

Вітиліго

Пігментна ксеродермія

Хвороба Леша-Ніхана

3863 / 6854
Ряд антибіотиків є специфічними інгібіторами процесу трансляції в мікроорганізмах. Робота яких органел порушується при цьому?

Мікротрубочок

Лізосом

Пероксисом

Рибосом

Мітохондрій

3864 / 6854
При недостатньому харчуванні часто має місце білкове голодування, яке проявляється зниженням вмісту білка в плазмі крові й розвитком набряків. За рахунок яких білків плазми більшою мірою розвивається зниження онкотичного тиску при цьому?

Гамма-глобулінів

Бета-глобулінів

Фібриногену

Альфа-глобулінів

Альбумінів

3865 / 6854
Для профілактики атеросклерозу, ішемічної хвороби серця та порушень мозкового кровообігу людина повинна одержувати 2-6 г незамінних поліненасичених жирних кислот на добу. Ці кислоти необхідні для синтезу:

Стероїдів

Простагладинів

Адреналіну

Вітамінів групи D

Жовчних кислот

3866 / 6854
Жінка 35-ти років розпочала голодування. Депо яких поживних речовин використовується у початковий період голодування і як при цьому змінюється дихальний коефіцієнт (ДК)?

Вуглеводи, ДК наближається до 1

Білки, ДК наближається до 0,7

Жири, ДК наближається до 0,72

Білки, ДК наближається до 1

Жири, ДК наближається до 0,85

3867 / 6854
На мікропрепараті серця розрізняємо кардіоміоцити зірчастої форми з центрально розташованим ядром, розвиненими гранулярною ендоплазматичною сіткою, апаратом Гольджі та специфічними гранулами. З цими клітинами пов’язана така функція:

Захисна

Проведення імпульсу

Скорочення

Регенераторна

Ендокринна

3868 / 6854
43-річний хворий надійшов в нефрологічне відділення з масивними набряками. Два роки лікувався амбулаторно, при цьому постійно відзначався підвищений артеріальний тиск. Двічі лікувався преднізолоном, з позитивним ефектом. У сечі: відносна щільність - 1017, білок - 4,0 г/л, еритроцити - 15-18 в полі зору (вилужені), лейкоцити - 5-7 в полі зору. Яка переважно функція нирок порушена у хворого?

Концентраційна

Фільтраційна

Секреторна

Реабсорбційна

Інкреторна

3869 / 6854
Під час трьох вагітностей у жінки спостерігалися викидні. З анамнезу відомо, що жінка протягом тривалого часу проживала в сім’ї, де була кішка. Яким одноклітинним паразитом, що міг бути причиною викиднів, могла заразитися жінка?

Балантидій

Трихомонада

Амеба

Токсоплазма

Лямблія

3870 / 6854
Хворий звернувся до лікаря зі скаргами на пронос і біль в животі протягом 5-ти днів, підвищену температуру тіла до 37,5°С. Бактеріологічно встановлений діагноз амебна дизентерія. Вкажіть препарат вибору для лікування цього захворювання:

Фурацилін

Ітраконазол

Хітамін

Ацикловір

Метронідазол

3871 / 6854
Хворій 39-ти років, яка протягом 8-ми років не може завагітніти, порадили звернутися до ендокринолога. При обстеженні у хворої виявлено екзофтальм, тремор повік, тахікардію. Захворювання якої ендокринної залози супроводжується такими симптомами?

Епіфіза

Надниркових

Підшлункової

Статевих

Щитоподібної

3872 / 6854
На заняттях з лікувальної фізкультури лікар-фізіотерапевт запропонував юнакам відхилитися назад і дістати долонями до підлоги. Яка зв’язка запобігає надмірному розгинанню хребтового стовпа?

Надостьова

Жовта

Передня поздовжня

Міжпоперечна

Задня поздовжня

3873 / 6854
Дитина 3-х років померла від гострої пневмонії на тлі хронічної серцевої недостатності. На аутопсії: дефект міжшлуночкової перегородки, стеноз устя легеневої артерії, гіпертрофія правого шлуночка серця, декстрапозиція аорти. Яка вада серця у дитини була встановлена в ході аутопсії?

Пентада Фалло

Синдром Патау

Тетрада Фалло

Синдром Марфана

3874 / 6854
У хворого, який помер від уремії, на розтині виявлена деформація хребетного стовпа з різким обмеженням рухливості. Суглобові хрящі дрібних суглобів хребта зруйновані, є виражені ознаки тривалого поточного хронічного запалення в тканинах суглобів, порожнини суглобів заповнені сполучною тканиною, місцями кістковою з формуванням анкілозів. В аорті, серці та легенях наявні хронічне запалення та вогнищевий склероз. У нирках спостерігається амілоїдоз. Який діагноз у цьому випадку найімовірніший?

Ревматоїдний артрит

Хвороба Педжета (деформівний остоз)

Остеопетроз (мармурова хвороба)

Анкілозивний спондилоартрит (хвороба Бехтєрєва)

Паратиреоїдна остеодистрофія

3875 / 6854
При дефіциті біотину синтез вищих жирних кислот знижується. Внаслідок недостатньої активності якого ферменту це відбувається?

Піруватдегідрогенази

Еноїлредуктази

Ацетил-КоА-карбоксилази

Бета-кетоацилредуктази

Цитратсинтетази

3876 / 6854
Хворому поставили попередній діагноз інфаркт міокарда. Характерною ознакою цього захворювання є суттєве підвищення в крові активності:

Каталази

Кретинфосфокінази

Аргінази

Альфа-амілази

Г-6-ФДГ

3877 / 6854
Хворому на крупозну пневмонію внутрішньом’язово ввели бензилпеніцилін-натрій. Через кілька хвилин у пацієнта розвинувся анафілактичний шок. Який лікарський засіб необхідно ввести хворому?

Мезатон

Кофеїн-бензоат натрію

Ефедрин

Адреналіну гідрохлорид

Норадреналіну гідротартрат

3878 / 6854
Хворий переніс повторний інтрамуральний інфаркт міокарда. Після лікування та реабілітації був виписаний у задовільному стані під нагляд дільничного терапевта. Через 2 роки загинув у автомобільній катастрофі. Який характер патологічного процесу в міокарді було встановлено на розтині?

Дрібновогнищевий кардіосклероз

Некроз

Атрофія

Гіперплазія

Великовогнищевий кардіосклероз

3879 / 6854
У хворого діагностовано пухлину мозку, яка розміщена в ділянці 'пташиної шпори'. Порушення якої функції виникне у хворого, якщо пухлина буде активно розвиватися?

Дотикова чутливість

Зір

Слух

Нюх

Смак

3880 / 6854
У хворого з хронічним гіперацидним гастритом з’явився біль у суглобах. Для полегшення болю, враховуючи супутню патологію, був призначений целекоксиб. Вибіркова дія цього препарату на певний фермент забезпечує відсутність впливу на слизову шлунка. Назвіть цей фермент:

Циклооксигеназа 1

Циклооксигеназа 2

Фосфоліпаза А2

Фосфоліпаза С

Калікреїн

3881 / 6854
Чим пояснити той факт, що для лікування туберкульозу доза ізоніазиду підбирається індивідуально, з обов’язковим контролем після перших прийомів препарату його вмісту в сечі?

Гіперглікемією, що виникає на тлі приймання препарату

Генетично обумовленою швидкістю ацетилювання препарату у різних людей

Подразнювальною дією препарату

Розвитком гемолітичної анемії

Розвитком ниркової недостатності

3882 / 6854
У хворого щорічно навесні та на початку літа в період цвітіння трав і дерев розвивається гостре катаральне запалення кон’юнктиви очей та слизової носової порожнини. Активація та екзоцитоз яких клітинних елементів лежить в основі цього синдрому?

Нейтрофілів

Тромбоцитів

Ендотеліальних клітин

Тканинних базофілів

Макрофагів

3883 / 6854
Хворий 13-ти років скаржиться на загальну слабкість, запаморочення, втомлюваність. Спостерігається відставання у розумовому розвитку. При обстеженні виявлено високу концентрацію валіну, ізолейцину, лейцину в крові та сечі. Сеча має специфічний запах. Що може бути причиною такого стану?

Гістидинемія

Хвороба Аддісона

Хвороба кленового сиропу

Базедова хвороба

Тирозиноз

3884 / 6854
У дитини 6-ти років захворювання почалося гостро з різкого катару в зіві та на мигдаликах, який поширився на слизову оболонку рота, язик ('малиновий язик') та глотку. На поверхні мигдаликів некрози. Місцями внаслідок відторгнення некротичних мас утворюються виразки. Шийні лімфовузли збільшені. На тілі, за винятком носогубного трикутника, спостерігається дрібнокрапковий висип яскраво-червоного кольору. Яке захворювання можна припустити?

Скарлатина

Кір

Дифтерія

Менінгококовий назофарингіт

Ангіна

3885 / 6854
Клінічне обстеження хворого дозволило встановити попередній діагноз рак шлунка. В шлунковому соці виявлено молочну кислоту. Який тип катаболізму глюкози має місце у ракових клітинах?

Анаеробний гліколіз

Аеробний гліколіз

Глюконеогенез

Глюкозо-аланіновий цикл

Пентозофосфатний цикл

3886 / 6854
При вимірюванні артеріального тиску у чоловіка 56-ти років встановлено зростання діастолічного артеріального тиску до 100 мм рт.ст. Від якого з наведених факторів переважно залежить величина діастолічного артеріального тиску?

Периферичного опору судин

Величини кінцево-діастолічного об’єму лівого шлуночка

Величини ударного об’єму лівого шлуночка

Об’єму циркулюючої крові

Швидкості кровотоку

3887 / 6854
Артеріальна гіпертензія у хворої 44-х років обумовлена наявністю феохромоцитоми - пухлини мозкового шару наднирників. Антигіпертензивні засоби якої групи найдоцільніше буде призначити?

Гангліоблокатори

Антагоністи кальцію

Симпатолітики

Бета-адреноблокатори

Альфа-адреноблокатори

3888 / 6854
Бактеріологічний метод діагностики був використаний для підтвердження діагнозу газова гангрена у хворого. Які живильні середовища необхідно використовувати для культивування збудника в цьому випадку?

ЖСА, кров’яний агар

МПА, МПБ

Ендо, Левіна, Плоскірєва

Лужний агар

Вільсона-Блера, Кітта-Тароцці

3889 / 6854
Хворому з діагнозом цукровий діабет II типу ендокринолог призначив глібенкламід. Вкажіть основний механізм дії цього засобу:

Активує транспорт глюкози в клітину

Підсилює захоплення глюкози периферичними тканинами

Стимулює секрецію інсуліну бета- клітинами острівців Лангерганса

Підсилює метаболізм глюкози

Пригнічує глюконеогенез

3890 / 6854
Хворому 35-ти років для обстеження очного дна був призначений атропіну сульфат у вигляді очних крапель. Для відновлення акомодації йому закрапали пілокарпіну гідрохлорид, але це не дало бажаного ефекту. Що є причиною відсутності ефекту?

Синергізм

Односторонній антагонізм

Двосторонній антагонізм

Тахіфілаксія

Звикання

3891 / 6854
До хірургічного відділення ЦРЛ надійшов хворий з колотою раною стопи, яку він отримав під час косовиці. Який специфічний препарат необхідно застосувати з метою екстреної пасивної імунопрофілактики правця?

Антибіотики

Анатоксин

Антитоксична сироватка

Вакцина АКДП

Протиправцева вакцина

3892 / 6854
В медико-генетичну консультацію за рекомендацією андролога звернувся чоловік 35-ти років з приводу відхилень фізичного і психічного розвитку. Об’єктивно встановлено: високий зріст, астенічна будова тіла, гінекомастія, розумова відсталість. При мікроскопії клітин слизової оболонки ротової порожнини знайдено в 30% статевий хроматин (одне тільце Барра). Який діагноз найімовірніший?

Хвороба Реклінгаузена

Хвороба Іценка-Кушинга

Синдром Клайнфельтера

Хвороба Дауна

Синдром Ді Джорджі

3893 / 6854
В психіатрії для лікування ряду захворювань ЦНС використовують біогенні аміни. Вкажіть препарат цієї групи, який є медіатором гальмування:

Гістамін

Дофамін

Таурин

Серотонін

Гамма-аміномасляна кислота

3894 / 6854
В експерименті на жабі вивчали міотатичний рефлекс. Однак при роз-тягненні скелетного м’яза, його рефлекторне скорочення не відбулося. На по-рушення функції яких рецепторів слід звернути увагу?

Тактильних

Сухожильних органів Гольджі

Больових

Суглобових

М’язових веретен

3895 / 6854
У жінки під час мейозу відбулося порушення розходження аутосом. Утворилася яйцеклітина із зайвою 18-ю хромосомою. Яйцеклітина запліднюється нормальним сперматозооном. У майбутньої дитини буде синдром:

Шерешевського-Тернера

Дауна

Едвардса

Патау

Клайнфельтера

3896 / 6854
На розтині звертає на себе увагу наступне: тіло молодого чоловіка високого зросту, виражена блідість та еластичність шкіри, доліхоцефалічна будова голови, подовження і контрактура пальців верхніх і нижніх кінцівок (нагадують кінцівки павука), незначне вдавлення грудини всередину, помірно виражений сколіоз. При дослідженні органів грудної порожнини виявили розрив аневризми висхідної частини аорти. Яке захворювання, найімовірніше, мало місце в цьому випадку?

Атеросклероз аорти

Сифілітичний мезоаортит

Синдром Марфана

Синдром Гудпасчера

3897 / 6854
Пацієнт був доставлений до лікарні з такими симптомами: запаморочення, сухість в роті, зіниці сильно розширені, порушення акомодації, тахікардія, утруднення сечовипускання, атонія кишечника. Передозування якого препарату могло викликати ці симптоми?

Каптоприл

Клофелін

Фуросемід

Празозин

Атропіну сульфат

3898 / 6854
На розтині тіла померлого виявлено, що вся права легеня збільшена, щільна, на плеврі нашарування фібрину, на розрізі тканина сірого кольору, з якої стікає каламутна рідина. Для якого захворювання легенів характерна така картина?

Інтерстиціальна пневмонія

Гангрена легені

Крупозна пневмонія

Вогнищева пневмонія

Фіброзивний альвеоліт

3899 / 6854
При деяких гельмінтозах людина може сама виявити гельмінта, оскільки зрілі членики збудника можуть активно виповзати з ануса людини. Це характерно для:

Дифілоботріозу

Гіменолепідозу

Ехінококозу

Теніаринхозу

Теніозу

3900 / 6854
Чоловік 35-ти років захворів гостро, відзначалося підвищення температури до 39°С, з’явилися нежить, кашель, сльозотеча. При огляді слизова носоглотки набрякла, гіперемована з рясним слизовиділенням. Який вид запалення розвинувся в носоглотці?

Катаральне

Серозне

Геморагічне

Фібринозне

Гнійне

3901 / 6854
У чоловіка 33-х років як наслідок спинномозкової травми порушена больова та температурна чутливість, що обумовлено пошкодженням такого шляху:

Медіального спінокортикального

Заднього спіномозочкового

Переднього спіномозочкового

Латерального спінокортикального

Спіноталамічного

3902 / 6854
Під час бігу на короткі дистанції у нетренованої людини виникає м’язова гіпоксія. До накопичення якого метаболіту в м’язах це призводить?

Оксалоацетату

Глюкозо-6-фосфату

Лактату

Кетонових тіл

Ацетил-КоА

3903 / 6854
Студент перкуторно визначає межу серця, яка проектується на передню грудну клітку на рівні хрящів третього ребра. Яку межу серця визначив студент?

Праву

Верхню

Ліву

Верхівку

Нижню

3904 / 6854
У хворого з підозрою на «озену» з носоглотки були виділені грамнегативні палички, які утворювали капсулу на поживному середовищі. Які мікроорганізми спричинили хворобу?

Мікоплазми

Шигели

Сальмонели

Клебсієли

Хламідії

3905 / 6854
Жінка літнього віку перенесла сильний стрес. У крові різко збільшилась концентрація адреналіну і норадреналіну. Які ферменти каталізують процес інактивації катехоламінів?

Пептидази

Карбоксилази

Глікозидази

Моноамінооксидази

Тирозиназа

3906 / 6854
У пацієнта, який тривалий час вживає препарати, що блокують вироблення ангіотензину II, виникли брадикардія, порушення серцевого ритму. Можливою причиною цих розладів є:

Гіперкаліємія

Гіпернатріємія

Гіпокальціємія

Гіперкальціємія

Гіпокаліємія

3907 / 6854
Хворому 68-ми років в комплекс лікування атеросклерозу, ускладненого ішемічною хворобою серця, лікар включив гіполіпідемічний засіб, який знижує вміст в крові переважно тригліцеридів. Який із вказаних препаратів було призначено хворому?

Інсулін

Анаприлін

Фенофібрат

Глібенкламід

Преднізолон

3908 / 6854
У дитини, яка часто хворіє на ангіни та фарингіти, відзначається збіль-шення лімфовузлів і селезінки. Зовнішній вигляд характеризується пастозністю та блідістю, м’язова тканина розвинена слабко. У крові спостерігається лімфоцитоз. Як називається такий вид діатезу?

Нервово-артритичний

Лімфатико-гіпопластичний

Геморагічний

Ексудативно-катаральний

Астенічний

3909 / 6854
У здорової дорослої людини проводять зондування порожнин серця і великих судин. Де розташований зонд, якщо протягом серцевого циклу зареєстровані зміни тиску від 0 до 120 мм рт.ст.?

Лівий шлуночок

Аорта

Передсердя

Правий шлуночок

Легенева артерія

3910 / 6854
Під час гінекологічного обстеження пацієнтці був поставлений діагноз ендометрит (запалення ендометрію). Яка частина маткової стінки уражена запальним процесом?

Навколоматкова клітковина

Адвентиційна оболонка

Серозна оболонка

М ’язова оболонка

Слизова оболонка

3911 / 6854
Хворому на туберкульоз призначено бактерицидний антибіотик, побічним ефектом якого є забарвлення біологічних рідин у червоний колір. Визначте препарат:

Протіонамід

Етамбутол

Ізоніазид

Рифампіцин

Стрептоміцину сульфат

3912 / 6854
Аміак є дуже отруйною речовиною, особливо для нервової системи. Яка речовина бере особливо активну участь у знешкодженні аміаку в тканинах мозку?

Гістидин

Глутамінова кислота

Пролін

Аланін

Лізин

3913 / 6854
Офтальмолог з діагностичною метою (розширення зіниць для огляду очного дна) використав 1% розчин мезатону. Мідріаз, викликаний препаратом, обумовлений:

Активацією β₁ -адренорецепторів

Блокадою α₁-адренорецепторів

Активацією α₁-адренорецепторів

Активацією М-холінорецепторів

Активацією α₂-адренорецепторів

3914 / 6854
В результаті травми у чоловіка 47- ми років пошкоджені передні корінці спинного мозку. Відростки яких нейронів пошкоджені?

Дендрити рухових нейронів й аксони ядер бокових стовпів

Аксони чутливих псевдоуніполярних нейронів

Дендрити чутливих псевдоуніполярних нейронів

Дендрити й аксони чутливих псевдоуніполярних нейронів

Аксони нейронів рухових соматичних та вегетативних ядер

3915 / 6854
У дитини 7-ми років підвищилася температура тіла до 39°С, з’явилися нежить, кон’юнктивіт і кашель. На шкірі відзначалися великоплямисті висипи, при огляді порожнини рота спостерігаються білуваті висипи на слизовій оболонці щік. Раптово з’явилося важке дихання і настала смерть при явищах асфіксії. Який діагноз найімовірніший?

Дифтерія

Скарлатина

Кір

Грип

Менінгококовий назофарингіт

3916 / 6854
У відділення реанімації надійшов хворий після ДТП з однобічним пневмотораксом. Який вид дихання спостерігається у цьому випадку?

Поверхневе

Поверхневе рідке

Глибоке часте

Асфіктичне

Поверхневе часте

3917 / 6854
Під дією УФ-опромінення та інших факторів можуть відбуватися зміни в структурі ДНК. Репарація молекули ДНК досягається узгодженою дією всіх наступних ферментів, ЗА ВИНЯТКОМ:

ДНК-полімерази

ДНК-глікозидази

Аміноацил-тРНК-синтетази

Ендонуклеази

ДНК-лігази

3918 / 6854
Реалізація загального адаптаційного синдрому здійснюється переважно через нейроендокринну систему. Якій з ланок цієї системи належить провідна роль у патогенезі реакції, що розвивається?

Гіпофізарно-тиреоїдна

Гіпофізарно-адреналова

Гіпофізарно-адреногенітальна

Гіпофізарно-інсулярна

Гіпофізарно-юкстагломерулярна

3919 / 6854
Кросинговер - це обмін ділянками гомологічних хромосом у процесі клітинного поділу, переважно в профазі першого мейотичного поділу, іноді в мітозі. Від чого залежить частота кросинговеру?

Від зовнішніх факторів

Від типу хромосоми

Від кількості хромосом

Від відстані між генами

Від довжини хромосоми

3920 / 6854
У людини вимірюють енерговитрати натщесерце, лежачи, в умовах фізичного і психічного спокою, при температурі комфорту. В який час енерговитрати будуть найбільшими?

3-4 години ранку

14-16 годин дня

17-18 годин вечора

10-12 годин дня

7-8 годин ранку

3921 / 6854
Відомо, що в ході альтерації у вогнищі запалення утворюються біологічно активні речовини-медіатори запалення, які відіграють важливу роль у патогенезі цього патологічного процесу. До утворення яких медіаторів призводить активація ліпооксигенази?

Гістаміну

Простагландинів

Тромбоксану

Лейкотрієнів

Простацикліну

3922 / 6854
Внаслідок захворювання нирок у пацієнта відзначаються набряки. В аналізах сечі визначається масивна протеїнурія. Який механізм є основним у виникненні набряків у такого пацієнта?

Зниження фільтраційного тиску в нирках

Зниження онкотичного тиску лімфи

Підвищення осмотичного тиску плазми крові

Зниження онкотичного тиску тканин

Зниження онкотичного тиску плазми крові

3923 / 6854
У дитини при обстеженні виявлено набряклість обличчя та розширення вен верхньої частини тулуба. На рентгенограмі спостерігається збільшення тіні органів середостіння. Ураження якого органу імуногенезу призвело до виникнення клінічних проявів захворювання?

Передні середостінні лімфатичні вузли

Задні середостінні лімфатичні вузли

Кістковий мозок грудини

Тимус

Діафрагмальні лімфатичні вузли

3924 / 6854
Тривалий вплив на організм людини токсичних речовин призвів до руйнування органел, які відповідають за синтез білків у гепатоцитах печінки. Які органели здійснюють синтез білків у гепатоцитах?

Мітохондрії

Пероксисоми

Лізосоми

Агранулярна ендоплазматична сітка

Рибосоми

3925 / 6854
У хворого після травми в скроневій ділянці виявлена епідуральна гематома. Яка артерія була пошкоджена?

Верхня барабанна артерія

Нижня барабанна артерія

Глибока скронева артерія

Глибока вушна артерія

Середня оболонна артерія

3926 / 6854
У хворого після травми втрачена чутливість шкіри на долонній поверхні 5-го пальця лівої кисті. Який нерв пошкоджений?

N. теdianus

N. musculo-cutaneus

N. ulnaris

N. radialis

N. ахillaris

3927 / 6854
Для прискорення загоєння рани слизової оболонки в ротовій порожнині хворому призначено препарат, який являє собою термостабільний білок, що міститься у людини в сльозах, слині, грудному молоці матері, а також його можна виявити в свіжознесеному курячому яйці. Відомо, що він є фактором природної резистентності організму. Назвіть цей білок:

Комплемент

Iнтерлейкін

Лізоцим

Iнтерферон

Iманін

3928 / 6854
В тубдиспансері у хворого на інфільтративну форму туберкульозу легень, який лікувався ізоніазидом, проявилися симптоми В6-гіповітамінозу. Чому ізоніазид призводить до цього явища?

Прискорюється елімінація

Iзоніазид є антагоністом вітаміну В6

Прискорюється біотрансформація

Утворюється міцний зв’язок з білками плазми крові

Сповільнюється всмоктування вітаміну

3929 / 6854
В аналізі крові 35-річного хворого: Нb = 58 г/л, еритроцити = 1,3•1012/л, колірний показник = 1,3, лейкоцити = 2,8•109/л, тромбоцити = 1,1•109/л, ретикулоцити = 2%, ШОЕ = 35 мм/год. Визначаються полісегментовані нейтрофіли, а також тільця Жоллі, кільця Кебота, мегалоцити. Яка це анемія?

Гіпопластична

Залізодефіцитна

В12-фолієводефіцитна

Гемолітична

Постгеморагічна

3930 / 6854
72-year-old female patient suffers from a long-standing heart failure. She presents to the hospital with a cough and dyspnea on exertion for 1 week. Her symptoms worsen at night and she has noticed that her sputum is pink and frothy. Chest auscultation reveals bilateral line crepitations over the lung bases. The physician prescribes a drug that reduces preload. Which of the following is the most likely drug prescribed by the physician?

Triamterene

Clopamide

Furosemide

Spironolactone

cetazolamide

3931 / 6854
A 12-year-old boy presents with progressive clumsiness and difficulty walking. He walks 'like a drunken-man' and has experienced frequent falls. His muscle tone and strength in all 4 limbs are slightly decreased. When he is asked to stand with his eyes closed and with both feet close together, he sways from side to side, unable to stand still. Which of the following brain regions is most likely affected and caused the symptoms described above?

Substantia nigra

Reticular formation

Red nucleus

Cerebellum

Right hemisphere

3932 / 6854
A doctor refers the patient to a gastroenterologist for a stomach acid test and an upper gastrointestinal endoscopy, which revealed that this patient is a heavy acid producer and has a gastric peptic ulcer. Which of the following is the most likely causative organism for this patient’s condition?

Salmonella

Helicobacter

Shigella

Leptospira

Listeria

3933 / 6854
A 21-year-old woman who is a medical student is undergoing evaluation after sticking herself with a needle while drawing blood from a patient. In this case a medical professional is at high risk of getting a blood-transmitted infection. Which of the following diseases is least likely to be transmitted via blood?

HBV

AIDS

HCV

SLE

HIV

3934 / 6854
A 38-year-old woman, who was diagnosed with systemic lupus erythematosus (SLE) 3 years ago, comes to her physician with a complaint of facial swelling and decreased urination that she first noticed 2 weeks ago. She currently takes azathioprine and a corticosteroid. Her vital signs show blood pressure - 150/90 mm Hg, pulse - 91/min., temperature - 36.8°С and respiratory rate - 15/min. On physical examination, the doctor notices erythematous rash on her face exhibiting a butterfly pattern. The laboratory studies reveal hypercholesterolemia, hypertriglyceridemia and proteinuria. Which of the following is the most likely mechanism of SLE’s complication in this patient?

Decrease in renal blood flow (ischemic nephropathy)

Increased plasma oncotic pressure

Acute infection of the kidney

Immune complex-mediated glomerular disease

3935 / 6854
A 24-year-old man undergoes a surgery and during the operation, an organ is excised and sent for histological evaluation. A light microscopic examination reveals the organ encased by thin connective tissue capsule that enters the substance of the lobes to further subdivide the organ into irregular lobular units. Each lobule contains a cluster of follicles filled with colloid. Follicular epithelium consists of low columnar, cuboidal or squamous cells, depending on the level of activity of the follicle. Which of the following organs does this tissue most likely belong to?

Thymus

Pancreas

Parathyroid gland

Thyroid gland

Parotid gland

3936 / 6854
A researcher is investigating the relationship between inflammatory mediators. He performs an experiment, investigating the effect of nonsteroidal anti-inflammatory drugs (NSAIDs) on patients with high-grade fever. His research indicates that certain NSAIDs act as competitively reversible inhibitors of the cyclooxygenase (COX) enzymes. It is known that COX catalyzes the formation of prostaglandins from a certain molecule that itself is derived from the cellular phospholipid bilayer by phospholipase A2. Which of the following molecules is a precursor of an inflammatory mediators mentioned above?

Cholesterol

Arachidonic acid Proopiomelanocortin

Tyrosine

Palmitic acid

3937 / 6854
A medical student studies a waste disposal system in human epithelial cells. During electronic microscopy he reveals the spherical vesicles, surrounded by a membrane and containing many different hydrolytic enzymes. The main function of these organelles is to provide intracellular digestion and protective reactions of the cell. Which of the following organelles is mentioned above?

Lysosomes

Mitochondria

Ribosomes

Centrosomes

Endoplasmatic reticulum

3938 / 6854
A biology graduate student is performing an experiment in the immunology laboratory. He studies a blood cell count from a patient with acute appendicitis, which shows an increase in the number of cells having a multilobed nucleus and multiple cytoplasmic granules. These cells engulf pathogens or necrotic tissue and help in the degradation of foreign products. Which of the following procescsses is seen in the cell desbribed above?

Passive diffusion

Parietal digestion

Pinocytosis

Osmosis

Phagocytosis

3939 / 6854
A patient suffers from a condition which, is characterized by a restriction in blood supply to tissues which leads to inadequate oxygen delivery to cells and contravention of cell metabolism. It is often caused by partial or total blockage of arteries. Which of the following is developed in this patient?

Inflammation

Hypoxia

Spasm

Embolism

Ischemia

3940 / 6854
A bone marrow biopsy of an 8-year-old girl shows a group of cells which has undergone the process of pyknosis and loss of the nucleus during its differentiation. Which of the following types of hemopoiesis is characterised by the morphological changes described above?

Lymphocytopoesis Thrombocytopoesis

Erytropoesis

Monocytopoesis

Granulocytopoesis

3941 / 6854
The healthcare provider performs a complete blood count to find out if the bone marrow is making the right number of blood cells. He studies a blood cell that has no nucleus and has a function to react to bleeding of an injured blood vessel by clumping, thereby initiating a blood clot. Which of the following is the main object of testing?

Platelets Stem cells

Prothrombine

Leukocytes

Macrophages

3942 / 6854
With total starvation the only source of water for the body is the oxidation process of organic compounds. Which of the following substances under these conditions is the main source of endogenic water?

Lipoproteins

Proteins

Glycoproteins

Carbohydrates

Lipids

3943 / 6854
An 18-year-old girl comes to her physician with concern about her health because she has not achieved menarche. She denies any significant weight loss, changes in mood, or changes in her appetite. She mentions that her mother told her about mild birth defects, but she cannot recall the specifics. Past medical history and family history are benign. On physical examination, the patient is short in stature, has a short and webbed neck and wide chest. Staining of buccal smear reveals absence of Barr bodies in the nucleis of epithelial cells. A urine pregnancy test is negative. Which of the following genetic disorders is the most likely cause of this patient’s condition?

Cri du chat ('cat-cry') syndrome

Turner syndrome

Patau syndrome

Edwards syndrome

Klinefelter syndrome

3944 / 6854
A 6-year-old boy is brought to the pediatrician by his mother, who complains of low-grade fever, chronic cough and night sweats in her child. She describes the cough as productive, producing white sputum that is sometimes streaked with blood. She also says that her son has lost some weight in the last month. His vital signs include blood pressure of 115/75 mm Hg, heart rate of 110/min., respiratory rate of 18/min. and temperature of 36.6°On physical examination, the patient is ill looking. Pulmonary auscultation reveals some fine crackles in the right upper lobe. The pediatrician suspects an active infection and performs Mantoux test. Intradermal injection of which of the following substances has been most likely used by pediatrician for the screening test in this clinical case?

Tuberculin

Bacillus Calmette-Guerin (BCG) vaccine

Diphtheria-tetanus toxoids-acellular pertussis vaccine (DTaP)

Tetanus and diphtheria toxoids vaccine (Td)

3945 / 6854
A research group is investigating a complex of three enzymes. They have created cultures of myocytes derived from high-performance college athletes and simulated starvation conditions. After the experiment they concluded that during starvation the amount of this complex in the muscle tissue was higher. The complex converts pyruvate into acetyl-coenzyme-which enters the citric acid cycle (Krebs cycle) under aerobic conditions. This reaction also involves the further reduction of NAD+ molecules into NADH. An activating effect of which of the following enzymes is described above?

Pyruvate dehydrogenase

Phosphofructokinase

Hexokinase

Lactate dehydrogenase

Phosphorylase

3946 / 6854
This action is an involuntary and nearly instantaneous movement in response to a stimulus. It is made possible by neural pathways which can act on an impulse before that impulse reaches the brain and does not require conscious thought. Which of the following actions is described above?

Initiation

Defense

Neuralgia

Tetanus

Reflex

3947 / 6854
A 16-year-old girl concerned about her sexual development comes to the physician. She mentions that she has still not had a menstrual period. However, she is otherwise a healthy girl with no significant, medical problems since birth. On physical examination, her vital signs are stable. She does not have pubic hair and her breast is slightly elevated with areola remaining in contour with surrounding breast. Which of the following is the most likely cause of this abnormal physical development?

Pancreatic islet insufficiency

Hypothyroidism

Ovarian insufficiency

Hyperthyroidism

Adrenal medulla hyperfunction

3948 / 6854
A 28-year-old male patient presents to the physician with pain and discomfort in his right lower extremity. On physical exam, palpation of the patient’s pulses reveals the absence of pulsation in the dorsalis pedis and tibialis posterior arteries, while the pulsation on the femoral artery is present. The skin of his extremity is also pale and cold/ Which of the following arteries is most likely damaged in this patient?

External iliac artery

Deep femoral artery

Descending genicular artery

Popliteal artery

Tibial artery

3949 / 6854
A pathologist studies a specimen of the small bean-shaped structure which is the part of human immune system. In a cross section it consists of an outer layer (cortex) and inner layer (medulla), and is surrounded by a fibrous capsule and subscapular sinus and is about 1.8 cm long. Which of the following is being studied by the pathologist?

Salivary gland

Parathyroid gland

Thymus

Spleen

Lymph node

3950 / 6854
A 27-year-old female presents with a severe sore throat, hoarseness, painful swallowing and low-grade fever. On intraoral examination, a large grey membrane is noticed on the oropharynx. Removal of the membrane reveals a bleeding oedematous mucosa. Which of the following is the most likely diagnosis?

Streptococcal pharyngitis

Scarlet fever

Meningococcal disease

Diphtheria

Measles

3951 / 6854
The doctor evaluates his patient’s spyrography. One of the evaluation parameters represents the normal amount of air displaced between normal inhalation and exhalation without any extra efforts or appointments. Which of the following is being evaluated in this case?

Residual volume

Vital volume

Total lung capacity

Inspiratory capacity

Tidal volume

3952 / 6854
A 37-year-old man is admitted to a hospital with mental confusion and disorientation. His wife reports he became more irritable and forgetful in the past year. In addition, she notes that he became a vegan a year ago, and currently, his diet consists of starchy foods like potatoes, corn, and leafy vegetables. GI symptoms include anorexia, diarrhea and vomiting. He has glossitis and skin lesions that appear as vesicles over the extremities. Eczema-like lesions around the mouth, as well as desquamation and roughened skin over the hands, are also present. Neurologic examination reveals symmetrical hypesthesia for all types of sensation in both upper and lower extremities in a 'gloves and socks' distribution. Deficiency in the diet the of which of the following amino acids is the most likely cause of this condition?

Threonine

Histidine

Arginine

Lysine

Tryptophan

3953 / 6854
The main function of the human glands are to produce and release substances that perform a specificfunction in the body. According to the classification there are endocrine and exocrine glands. But also there are glands that may be classified as both. Which of the following glands can be endo-and exocrine simultaneously?

Gastrointestinal

Parathyroid

Sebaceous

Lacrimal

Salivary

3954 / 6854
Physiological solution is an isotonic diluent used to maintain cell integrity and viability in procedures that -require preparation of test suspension of organisms. This sterile solution in water provides osmotic protection for microbial cells. Which of the following concentrations of NaCl is considered to be physiological?

0.9%

10%

9%

5%

9%

3955 / 6854
A 60-year-old man with a history of hypertension, diabetes and hyperlipidemia had a sudden onset of right-sided weakness. By the time the ambulance arrived, he had difficulty speaking. Unfortunately, the patient died within the next 2 hours and an autopsy was performed immediately. The gross examination of the cerebral left hemisphere showed brain swelling, widened gyri and poorly demarcated gray-white junction. Which of the following is the most likely cause of this patient’s death?

Cyst

Ischemic stroke

Intracerebral hemorrhage

Abscess

Tumor

3956 / 6854
A 14-year-old girl presents to the emergency department for evaluation of an 'infected leg'. She states there is no history of trauma but mentions she had a history of sickle cell disease/ On physical examination, her upper part of the right shin is very painful, red, swollen, and hot. Her temperature is 39.2°An X-ray shows focal bony lysis and loss of trabecular architecture in the metaphysis of the right tibiIncreased activity of which of the following cells is the most likely cause of bone reabsorption in this patient?

Osteocytes

Chondroblasts

Chondrocytes

Osteoblasts

Osteoclasts

3957 / 6854
A 46-year-old man presents with fatigue and joint pain in his fingers and wrists for the last 2 months. The pain is present in both hands and the wrists are swollen. Furthermore, he describes morning stiffness in his joints lasting about 2 hours, which improves with use. His past medical history reveals he has been successfully treated for H. pylori related ulcers last year. He denies smoking and stopped drinking when his gastric symptoms started. Which of the following drugs is the best choice for his joints’ pain management?

Morphine

Paracetamol

Aspirin

Celecoxib

Prednisone

3958 / 6854
A 20-year-old female comes to the clinic after missing her last 2 periods. Her cycles are usually regular, occurring at 28-30 day interval with moderate bleeding and some abdominal discomfort. She also complains of progressively diminishing peripheral vision. Her doctor reveals loss of vision in the lateral halves of both eyes. Involvement of which of the following structures would you most likely expect to be the reason of bitemporal hemianopsia?

Optic chiasm

Left optic tract

Left optic nerve

Right optic nerve

Right optic tract

3959 / 6854
A 25-year-old male presents to an emergency department with suddenonset right-sided pleuritic chest pain and breathlessness. On examination, he has arterial blood saturation of 90% in air and is afebrile. An X- ray shows a lung edge visible in the right hemithorax, beyond which no lung markings are seen. Which of the following conditions has most likely occured in this patient?

Pleurisy

Aerophagy

Rib fracture

Pneumothorax

Pneumonia

3960 / 6854
На прийом до терапевта прийшов чоловік 37-ми років зі скаргами на періодичні інтенсивні больові напади у суглобах великого пальця стопи та їх припухлість. При аналізі сечі встановлено її різко кислий характер і рожеве забарвлення. З наявністю яких речовин можуть бути пов’язані такі зміни сечі?

Фосфату кальцію

Амонієвих солей

Солей сечової кислоти

Сульфату магнію

Хлоридів

3961 / 6854
У хворого на цукровий діабет з’явився різкий біль у правій стопі. При огляді великий палець стопи чорного кольору, тканини стопи набряклі, осередки відшарування епідермісу, виділення з неприємним запахом. Яка клініко- морфологічна форма некрозу розвинулася в хворого?

Інфаркт

Пролежень

Гангрена суха

Секвестр

Іангрена волога

3962 / 6854
На розтині трупа жінки 63-х років, що страждала на ревматизм і комбіновану мітральну ваду, виявлено, що стулки мітрального клапана різко потовщені, зрощені між собою, кам’янистої щільності, під час розрізання визначається хрускіт. Який патологічний процес зумовив кам’янисту щільність клапана серця?

Дистрофічне звапнення

Метаболічне звапнення

Фібриноїд

Амілоїдоз

Метастатичне звапнення

3963 / 6854
У клітину потрапив вірус грипу. Трансляція при біосинтезі вірусного білка в клітині буде здійснюватися:

На полірибосомах

У лізосомах

На каналах гладенької ендоплазматичної сітки

У ядрі

У клітинному центрі

3964 / 6854
У обстежуваного визначили дихальний об’єм (500 мл), частоту дихання (15 за хвилину), об’єм мертвого простору (100 мл). Скільки повітря пройде у нього за хвилину через альвеоли?

6000 мл

7400 мл

7500 мл

1500 мл

9000 мл

3965 / 6854
Експериментатору необхідно якнайшвидше виробити умовний рефлекс у собаки. На базі якого безумовного рефлексу доцільно виробляти умовний?

Статевий

Міотатичний

Травний

Захисний

Орієнтувальний

3966 / 6854
Подразнення правого блукаючого нерва спричинило різке сповільнення атріовентрикулярного проведення. На ЕКГ про це буде свідчити подовження:

Зубця Р

Інтервалу RR

Комплексу QRST

Інтервалу PQ

Зубця Т

3967 / 6854
Хворому було призначено препарат. Через декілька діб дія препарату значно знизилась і для отримання початкового ефекту потрібно збільшити дозу речовини. Вкажіть назву цього типу зміни дії лікарської речовини:

Ідіосинкразія

Лікарська залежність

Звикання

Кумуляція

Тахіфілаксія

3968 / 6854
У жінки 62-х років розвинулася катаракта (помутніння кришталику) на тлі цукрового діабету. Вкажіть, який тип модифікації білків має місце при діабетичній катаракті:

Метилювання

Фосфорилювання

Обмежений протеоліз

АДФ-рибозилювання

Ілікозилювання

3969 / 6854
При кесаревому розтині у хворої 32-х років, внаслідок сильної кровотечі і не-можливості її зупинити, оператор вимушений був видалити матку. За рахунок яких судин здійснюється кровопостачання матки?

Arteria pudenda interna

Arteria clitoridis

Arteria obturatoria

Arteria uterina

Arteria sacralis lateralis

3970 / 6854
У 6-місячної дитини спостерігались часті та сильні підшкірні кровотечі. Призначення синтетичного аналога вітаміну К (вікасола) дало позитивний ефект. В гамма-карбоксилюванні глута- мінової кислоти якого з перерахованих нижче білків згортальної системи крові бере участь цей вітамін?

Фібриногену

Протромбіну

Антигемофільного глобуліну А

Фактора Розенталя

Фактора Хагемана

3971 / 6854
У хворого виявлена серповидноклі- тинна анемія. Заміна якої амінокислоти в поліпептидному ланцюгу НЬ на валін призводить до цього захворювання?

Аргініну

Ілутамінової кислоти

Аспарагінової кислоти

Лейцину

Треоніну

3972 / 6854
Під час підйому пішки на 5-й поверх у людини підвищився артеріальний тиск. Причиною є збільшення:

Вмісту іонів в плазмі крові

Об’єму циркулюючої крові

Кількості функціонуючих капілярів

Хвилинного об’єму крові

В’язкості крові

3973 / 6854
В приймальне відділення надійшла дитина 1,5 років з ознаками отруєння нітратами: стійкий ціаноз, задишка, судоми. Утворення якої форми гемоглобіну лежить в основі цих симптомів?

Редукованого гемоглобіну

Оксигемоглобіну

Карбгемоглобіну

Метгемоглобіну

Карбоксигемоглобіну

3974 / 6854
Внаслідок росту пухлини в порожнину III шлуночка головного мозку у пацієнта розвиваються вегетативні розлади у вигляді порушення сну, терморегуляції, всіх видів обміну, нецукровий діабет. Подразнення ядер якої ділянки головного мозку викликало ці симптоми?

Гіпотапамуса

Покришки середнього мозку

Ніжок мозку

Довгастого мозку

Моста

3975 / 6854
Хворому призначено препарат дигоксин. Через декілька днів у хворого виявлено ознаки передозування цим препаратом, його вміст у крові значно перевищував верхню межу терапевтичної концентрації. Як називається такий варіант дії лікарських речовин?

Кумуляція

Тахікардія

Звикання

Антагонізм

Потенціювання

3976 / 6854
В результаті травми у чоловіка 47-ми років пошкоджені передні корінці спинного мозку. Відростки яких нейронів пошкоджені?

Аксони чутливих псевдоуніполярних нейронів

Дендрити й аксони чутливих псевдоуніполярних нейронів

Дендрити рухових нейронів й аксони ядер бокових стовпів

Дендрити чутливих псевдоуніполярних нейронів

Аксони нейронів рухових соматичних та вегетативних ядер

3977 / 6854
У зоопарк доставлені антилопи з Африки. В їх крові виявлено Trypanosoma brucei gambiense. Чи становлять ці тварини епідеміологічну небезпеку?

Епідеміологічної небезпеки немає

Небезпека для інших антилоп

Небезпека лише для людини

Небезпека лише для хижаків

Небезпека для домашніх тварин і людини

3978 / 6854
Після травматичного ураження промежини у потерпілого спостерігається нетримання сечі. Який м’яз був травмований?

Глибокий поперечний м’яз промежини

Внутрішній стискач відхідника

Зовнішній стискач сечівника

Сіднично-печеристий м’яз

Поверхневий поперечний м’яз промежини

3979 / 6854
Хворого з явищами енцефалопатії госпіталізували в неврологічний стаціонар і виявили кореляцію між наростанням енцефалопатії і речовинами, які надходять з кишечника в загальний крово- тік. Яка сполука, що утворюється в кишечнику, може бути причиною такого стану?

Біотин

Індол

Орнітин

Ацетоацетат

Бутират

3980 / 6854
До клініки доставили пацієнта 32-х років з масивною крововтратою внаслідок ДТП. Ps- 110/хв., ЧДР- 22 за 2 хв, АТ- 100/60 мм рт.ст. Яка зміна крові з перерахованих буде найбільш характерною через 1 годину після крововтрати?

Гіповолемія

Лейкопенія

Еритропенія

Гіпопротеїнемія

Гіпохромія еритроцитів

3981 / 6854
Людина отримала електротравму При цьому струм пройшов крізь серцевий м’яз. Які небезпечні порушення в роботі серця можуть виникнути у цій ситуації, що вимагають термінового втручання?

Тахікардія

Брадикардія

Фібриляція шлуночків

Атріовентрикулярна блокада

Екстрасистолія

3982 / 6854
Після тривалого вживання одного з лікарських препаратів у хворого розвинулась підвищена чутливість до простудних захворювань. Який з вказаних нижче лікарських засобів міг сприяти зниженню імунітету?

Клофелін

Преднізолон

Резерпін

Сустак-форте

Настойка женьшеню

3983 / 6854
Дитина 5-ти років після невдалого падіння з дивану скаржиться на різкий біль у шиї, що виникає при найменших рухах. Огляд виявив припухлість, порушення цілісності в ділянці правої ключиці, крепітацію. Що слід ввести в першу чергу для швидкого усунення болю і заспокоєння дитини?

Парацетамол

Кодеїн

Анальгін

Промедол

Діазепам

3984 / 6854
Чоловіку 70-ти років, який страждає на хворобу Паркінсона, був призначений препарат леводопа. Через тиждень стан хворого значно покращився. Який механізм дії лікарського засобу?

Іальмування гістамінергічної системи

Іальмування серотонінергічної системи

Активація енкефалінової системи

Активація дофамінової системи

Іальмування холінергічної системи

3985 / 6854
Жінка 37-ми років звернулася до гінеколога з приводу запального процесу піхви, який супроводжується свербінням та пінистими виділеннями. Бакана- ліз встановив наявність трихомонадної інфекції. Який засіб найбільш ефективний у даному випадку?

Метронідазол

Еритроміцин

Нітроксолін

Ністатин

Ампіцилін

3986 / 6854
При обстеженні хворого на гемофілію виявлено зміну деяких показників крові. Яка з перерахованих ознак відповідає цьому захворюванню?

Тромбоцитопенія

Еритроцитоз

Час згортання крові вповільнений

Еозинофілія

Афібриногенемія

3987 / 6854
У хворого, який страждав на хронічний алкоголізм і цироз печінки, розвинулася профузна кровотеча з варикозно розширених вен стравоходу, в результаті чого настала смерть. На аутопсії печінка дрібногорбиста, зменшена в розмірах, щільна, жовтуватого кольору. Під час гістологічного дослідження кріостатних зрізів печінки (забарвлення гематоксиліном і еозином) в гепатоцитах виявляються великі оптично порожні вакуолі, в яких міститься речовина, що забарвлюється в чорний колір при застосуванні осмієвої кислоти. Оптично порожні вакуолі гепатоцитів - це:

Гідропічна дистрофія

Алкогольний гіалін (тільця Меллорі)

Включення гіаліну

Великокрапельна жирова дистрофія

Псевдовакуолі гіалоплазми

3988 / 6854
Дослідник в ході мікроскопічного і електронно-мікроскопічного вивчення печінки звернув увагу, що деякі окремо розташовані клітини розпалися на дрібні фрагменти, оточені мембраною. У деяких з них є органели, інші включають в себе фрагменти ядра, що розпало-ся. Запальна реакція навколо була відсутня. Він розцінив ці зміни як прояв:

Апоптозу

Атрофії

Некрозу

Дистрофії

Гіпоплазії

3989 / 6854
У гістопрепараті визначається орган, що складається з сірої та білої речовини. Сіра речовина розташовується на периферії й має 6 шарів: молекулярний, зовнішній зернистий, пірамідний, внутрішній зернистий, гангліонарний і шар поліморфних клітин. Визначте утворення, якому належать дані морфологічні ознаки:

Спинномозковий вузол

Мозочок

Кора великих півкуль

Спинний мозок

Довгастий мозок

3990 / 6854
Машиною швидкої допомоги доставлено до приймального відділення лікарні людину з гострим отруєнням морфіном. Який із найбільш специфічних антагоністів необхідно застосувати в цьому випадку?

Налоксон

Пентазоцин

Дипіроксим

Омнопон

Метацин

3991 / 6854
Хворий переніс повторний інтраму- ральний інфаркт міокарда. Після лікування та реабілітації був виписаний у задовільному стані під нагляд дільничного терапевта. Через 2 роки загинув у автомобільній катастрофі. Який характер патологічного процесу в міокарді було встановлено на розтині?

Некроз

Дрібновогнищевий кардіосклероз

Атрофія

Великовогнищевий кардіосклероз

Гіперплазія

3992 / 6854
Хвора на бронхіальну астму приймала протягом 3-х місяців преднізолон у таблетках. Внаслідок значного покращення стану раптово припинила його прийом. Розвиток якого ускладнення ви- сокоймовірний у цьому випадку?

Синдрому відміни

Шлункової кровотечі

Гіпотонії

Синдрому Іценка-Кушинга

Ожиріння верхньої половини тулуба

3993 / 6854
У клініку госпіталізований хворий з отруєнням. Встановлено, що в печінці порушені механізми детоксикації. Які з органел гепатоцитів в першу чергу обумовили цей стан?

Комплекс Іольджі

Гранулярна ендоплазматична сітка

Мітохондрїї

Рибосоми

Агранулярна ендоплазматична сітка

3994 / 6854
На земній кулі існують території (біохімічні провінції) частина населення яких страждає на ендемічний зоб. Дефіцит якого біоелемента в грунті, воді та харчових продуктах викликає це захворювання?

Мідь

Йод

Залізо

Кобальт

Цинк

3995 / 6854
У 12-річного хлопчика в сечі виявлено високий вміст усіх амінокислот аліфатичного ряду. При цьому відзначена найвища екскреція цистину та цистеїну. Крім того, УЗД нирок показало наявність у них каменів. Виберіть можливу патологію:

Цистинурія

Фенілкетонурія

Алкаптонурія

Цистит

Хвороба Хартнупа

3996 / 6854
Внаслідок захворювання нирок у пацієнта відзначаються набряки. В аналізах сечі визначається масивна протеїну- рія. Який механізм є основним у виникненні набряків у такого пацієнта?

Зниження онкотичного тиску тканин

Зниження фільтраційного тиску в нирках

Зниження онкотичного тиску плазми крові

Зниження онкотичного тиску лімфи

Підвищення осмотичного тиску плазми крові

3997 / 6854
У пацієнта з гнійничковими ураженнями шкіри виділено збудник, який на кров’яному агарі утворює жовті колонії округлої форми, середніх розмірів, оточені зоною гемолізу. В мазках з колоній містяться коки, розташовані скупченнями неправильної форми, Гр+. Ви-ділена культура є оксидазо- та каталазопозитивною, ферментує маніт, синтезує плазмокоагулазу Якому виду з нижче- перерахованих відповідає виділений збудник?

Streptococcus agalactiae

Staphylococcus epidermidis

Staphylococcus saprophyticus

Streptococcus pyogenes

Staphylococcus aureus

3998 / 6854
У хворого на цукровий діабет виникла значна спрага, дисфагія та порушення психічної діяльності. Який тип розладів водно-електролітного балансу характеризує поява вказаних ознак?

Дегідратація гіпоосмотична

Гідратація гіпоосмотична

Гідратація ізоосмотична

Дегідратація гіперосмотична

Дегідратація ізоосмотична

3999 / 6854
Потерпілий обробляв рослини розчином речовини з інсектицидною дією без індивідуальних засобів захисту. Через деякий час у нього почалося сильне виділення слини, поту, сліз, біль у животі, понос. При огляді виявлено міоз. Речовина, що викликала отруєння, належить до групи:

Антихолінестеразні засоби

Н-холіноміметики

Нітрати

Органічні сполуки хлору

Солі міді

4000 / 6854
Пацієнт 64-х років має гостру серцеву недостатність, артеріальний тиск - 80/60 мм рт.ст., добовий діурез - 530 мл, істотно збільшену концентрацію сечовини та креатиніну в крові. Назвіть патогенетичний механізм розвитку азотемії та олигурії:

Спазм приносних артеріол клубочка

Зменшення об’єму циркулюючої крові

Зменшення фільтраційного тиску в нирках

Збільшення продукції вазопресину

Гіпернатріємія

4001 / 6854
Людина довгий час проживала в умовах високогір’я. Які зміни в кровоносній системі будуть у неї?

Зниження кількості лейкоцитів

Збільшення діаметра кровоносних судин

Вповільнення пульсу

Збільшення кількості лейкоцитів

Збільшення кількості гемоглобіну

4002 / 6854
При підйомі в гори у альпініста розвинулась ейфорія, яка замінилася головним болем, запамороченням, серцебиттям, задишкою, що перейшла в апное. Яке порушення кислотно-лужної рівноваги розвинулось в даному випадку?

Негазовий алкалоз

Видільний алкалоз

Негазовий ацидоз

Газовий алкалоз

Іазовий ацидоз

4003 / 6854
У дівчинки 6-ти років пастозність повік, губ, шиї, слизової оболонки язика виникла після того, як вона з’їла апельсин. Раніше на апельсини виникали висипання на шкірі, свербіння. Який патогенетичний механізм є провідним у розвитку набряку у дівчинки?

Порушення лімфовідтоку

Підвищення проникності капілярів

Підвищення онкотичного тиску тканинної рідини

Зниження онкотичного тиску крові

Підвищення гідростатичного тиску крові в капілярах

4004 / 6854
На прийом до лікаря прийшов пацієнт дуже високого зросту, з довгими товстими пальцями рук, великою нижньою щелепою і відвислою нижньою губою. Підвищену секрецію якого гормону якої залози можна припустити?

Гормонів щитоподібної залози

Гонадотропного гормону передньої частки гіпофіза

Гормонів наднирників із групи глюко- кортикоїдів

Соматотропного гормону передньої частки гіпофіза

Антидіуретичного гормону задньої частки гіпофіза

4005 / 6854
Під час гістологічного дослідження щитоподібної залози, видаленої в ході операції, виявлена деструкція й атрофія фолікулів, дифузна лімфоцитарна інфільтрація з формуванням лімфоїдних фолікулів в стромі. До якої групи захворювань належить такий тиреоїдит?

Бактеріальних

Аутоімунних

Вірусних

Інфекційно-алергічних

Викликаних фізичними факторами

4006 / 6854
Дівчинка 4-х років на 3-ю добу від початку захворювання на дифтерію померла від справжнього крупу На ауто- псії слизова оболонка гортані, трахеї та бронхів потовщена, набрякла, тьмяна, вкрита сіруватою плівкою, що легко відокремлюється. Визначити вид ексудативного запалення гортані:

Фібринозне

Змішане

Гнійне

Катаральне

Серозне

4007 / 6854
У хворого внаслідок отруєння сулемою розвинулася гостра ниркова недостатність, перебіг якої включав 4 стадії: перша - початкова, друга - оліго-, анурії, четверта - одужання. Як називається третя стадія гострої ниркової недостатності?

Метаболічна

Іемодинамічна

Патохімічна

Ішемічна

Поліурична

4008 / 6854
Злоякісна гіперхромна анемія хвороба Бірмера виникає внаслідок нестачі вітаміну В12. Який біоелемент входить до складу цього вітаміну?

Цинк

Кобальт

Магній

Залізо

Молібден

4009 / 6854
У хворого час від часу суттєво знижується вміст гемоглобіну і еритроцитів в крові, з’являється жовтяниця. З анамнезу з’ясовано, що ці напади завжди виникають після вживання кінських бобів. Яка з форм анемії має місце у даного хворого?

Набута гемолітична анемія

Спадкова гемоглобінопатія

Спадкова мембранопатія

Залізодефіцитна анемія

Спадкова ферментопатія

4010 / 6854
На розтині тіла померлого виявлено, що вся права легеня збільшена, щільна, на плеврі нашарування фібрину, на розрізі тканина сірого кольору, з якої стікає каламутна рідина. Для якого захворювання легенів характерна така картина?

Інтерстиціальна пневмонія

Крупозна пневмонія

Фіброзивний альвеоліт

Іангрена легені

Вогнищева пневмонія

4011 / 6854
На розтині хворого, який багато років працював на шахті і помер від хронічної легенево-серцевої недостатності, виявлено, що легені малоповітряні, значно ущільнені, склерозовані, верхівки емфізематозно змінені, поверхня сіро- чорного кольору, на розрізі тканина легенів аспідно-чорного кольору. Від якої хвороби настала смерть?

Талькоз

Асбестоз

Антракоз

Силікоз

Алюміноз

4012 / 6854
У потерпілого травма внаслідок прямого удару по внутрішній поверхні середньої третини гомілки. Перелом якого анатомічного утворення найбільш імовірний?

Діафіз великогомілкової кістки

Проксимальний епіфіз малогомілкової кістки

Дистальний епіфіз великогомілкової кістки

Проксимальний епіфіз великогомілкової кістки

Дистальний епіфіз малогомілкової кістки

4013 / 6854
При мікроскопічному дослідженні легені недоношеної дитини виявлено спадання стінок альвеол через відсутність сурфактанту. Вкажіть, з порушенням розвитку яких клітин стінки альвеоли це пов’язано:

Фібробластів

Секреторних клітин

Альвеолярних макрофагів

Альвеолоцитів II типу

Альвеолоцитів І типу

4014 / 6854
Хворий 70-ти років, який страждав на цукровий діабет та переніс інфаркт міокарда, помер при явищах прогресуючої серцево-судинної недостатності. На розтині ціанотична індурація селезінки та нирок, бура індурація легень та мускатна печінка. Який вид порушення кровообігу обумовив зміни внутрішніх органів? 

Загальна хронічна венозна гіперемія

Загальна гостра венозна гіперемія

Загальна артеріальна гіперемія після анемії

Місцева хронічна венозна гіперемія

Артеріальна ішемія в результаті перерозподілу крові

4015 / 6854
На слизовій оболонці мигдаликів та м’якого піднебіння виявляються білувато-сірого кольору плівки, які щільно з’єднані з підлеглою тканиною. При спробі зняти плівку на її місці виникає глибокий дефект тканини. Визначити патологічний процес, який виник на слизовій оболонці мигдаликів та м’якого піднебіння:

Дифтеритичне запалення

Крупозне запалення

Серозне запалення

Гнійне запалення

Змішане запалення

4016 / 6854
.3 метою ранньої діагностики вагітності досліджується сеча жінки. З’явлення яких гормонів в сечі свідчить про ймовірну вагітність?

Естріол

17-бета-естрадіол

Тестостерон

Хоріонічний гонадотропін

Прогестерон

4017 / 6854
Людина знепритомніла у салоні автомобіля, де тривалий час очікувала приятеля при ввімкненому двигуні. В крові у неї знайдено сполуку гемоглобіну. Яку саме?

Метгемоглобін

Карбгемоглобін

Дезоксигемоглобін

Карбоксигемоглобін

Оксигемоглобін

4018 / 6854
При обстеженні людини встановлено, що хвилинний об’єм серця дорівнює 3500 мл, систолічний об’єм - 50 мл. Якою є у людини частота серцевих скорочень?

50 скорочень за хвилину

90 скорочень за хвилину

60 скорочень за хвилину

70 скорочень за хвилину

80 скорочень за хвилину

4019 / 6854
У людини внаслідок хронічного захворювання печінки суттєво порушена її білковосинтезуюча функція. До зменшення якого параметру гомеостазу це призведе?

Осмотичний тиск

pH

Онкотичний тиск плазми крові

Іематокритний показник

Щільність крові

4020 / 6854
Унітіол є антидотом і застосовується, зокрема, при отруєннях солями важких металів. Як називається такий тип взаємодії лікарських речовин?

Синергоантагонізм

Неконкурентний антагонізм

Фізіологічний антагонізм

Фізичний антагонізм

Хімічний антагонізм

4021 / 6854
Хворому на туберкульоз в комплексній терапії призначено препарат - похідне гідразиду ізонікотинової кислоти. Визначити цей препарат:

Рифампіцин

Стрептоміцину сульфат

Ізоніазид

Канаміцин

Цефалоридин

4022 / 6854
Робітник тваринницької ферми гостро захворів і при наростаючих явищах інтоксикації помер. На розтині встановлено: селезінка збільшена, в’яла, на розрізі темно-вишневого кольору, зішкріб пульпи рясний. М’які мозкові оболонки на склепінні та основі мозку набряклі, просякнуті кров’ю, мають темно-червоний колір ('шапочка кардинала'). Мікроскопічно: серозно- геморагічне запалення оболонок і тканин головного мозку з руйнуванням стінок дрібних судин. Поставте діагноз:

Холера

Сибірка

Туляремія

Бруцельоз

Чума

4023 / 6854
Під час розтину тіла жінки, яка померла внаслідок пухлинної дисемінації муцинозної цистаденокарциноми і тривалий час мала вимушене положення в ліжку, були знайдені великі ділянки некрозу шкіри та підлеглих м’яких тканин крижової ділянки. Назвіть форму некрозу:

Пролежень

Воскоподібний (ценкеровський) некроз

Секвестр

Сирнистий некроз

Інфаркт

4024 / 6854
На розтині чоловіка, який помер на 5-у добу захворювання черевним тифом, виявлені наступні зміни: групові фолікули клубової кишки збільшені, повнокровні і виступають над слизовою оболонкою, на їх поверхні видно борозди та звивини. Гістологічно: повнокрів’я і набряк тканини, наявність гранульом, які складаються з великих клітин зі світлою цитоплазмою і містять черевнотифозні палички. Про який період місцевих змін при черевному тифі можна думати?

Стадія некрозу

Стадія чистих виразок

Стадія мозкоподібного набухання

Стадія утворення виразок

Стадія загоювання виразок

4025 / 6854
У клініці хворому було пересаджено нирку. Які з перерахованих клітин імунної системи можуть безпосередньо впливати на клітини трансплантату?

Т-хелпери

Т-кілери

Тимоцити

Плазмобласти

В-лімфоцити

4026 / 6854
На препараті селезінки виявляється судина, стінка якої складається з базальної мембрани з ендотелієм, середня оболонка відсутня, зовнішня оболонка зрощена зі сполучнотканинними прошарками селезінки. Що це за судина?

Артеріола

Артерія еластичного типу

Вена м’язового типу зі слабким розвитком м’язових елементів

Артерія м’язового типу

Вена безм’язового типу

4027 / 6854
У хворого спостерігається збільшений тонус артеріол за нормальних показників роботи серця. Як це вплине на величину артеріального тиску?

Зменшиться переважно діастолічний

Зросте переважно систолічний

Зросте переважно діастолічний

Тиск не зміниться

Зменшиться переважно систолічний

4028 / 6854
При дослідженні людини у вертикальній позі встановлено, що в альвеолах верхівок легень парціальний тиск кисню становить 140 мм рт.ст. Причиною цього є те, що у цих відділах легень:

Перфузія та вентиляція врівноважені

Вентиляція відсутня

Перфузія переважає над вентиляцією

Е. -

Вентиляція переважає над перфузією

4029 / 6854
Офтальмолог з діагностичною метою (розширення зіниць для огляду очного дна) використав 1 % розчин мезатону. Мідріаз, викликаний препаратом, обумовлений:

Активацією α1-адренорецепторів

Активацією α2-адренорецепторів

Активацією β1-адренорецепторів

Блокадою α1-адренорецепторів

Активацією М-холінорецепторів

4030 / 6854
Чоловік 55-ти років, що скаржиться на біль в ділянці нирок, надійшов в лікарню. В ході ультразвукового обстеження пацієнта виявлено наявність ниркових каменів. Наявність в сечі якої з наведених речовин є найімовірнішою причиною утворення каменів у цього паці-єнта?

Білірубін

Уробілін

Білівердин

Сечова кислота

Креатинін

4031 / 6854
В осередку запалення утворюється біогенний амін, що має судинорозширювальну дію. Назвіть його:

ГАМК

Гістамін

Триптамін

ДОФА

Серотонін

4032 / 6854
При розтині померлого від чуми хворого на фоні геморагічного синдрому знайдено геморагічний некроз шкіри стегна, лімфангіт, пахвинний геморагічний лімфаденіт. Назвіть форму чуми:

Бубонна

Шкірно-бубонна

Первинно-септична

Первинно-легенева

Геморагічна

4033 / 6854
Психологічне дослідження встановило: у людини добра здатність швидко пристосовуватися до нового оточення, добра пам’ять, емоційна стійкість, висока працездатність. Найімовірніше, ця людина:

Сангвінік

Меланхолік

Флегматик

Холерик

Флегматик з елементами меланхоліка

4034 / 6854
Чоловік 40-ка років скаржиться на загальну слабкість, головний біль, кашель із виділенням мокротиння, задишку. Після клінічного огляду й обстеження поставлено діагноз пневмонія. Який тип гіпоксії має місце у хворого?

Циркуляторна

Респіраторна

Тканинна

Гемічна

Гіпоксична

4035 / 6854
Хвора 45-ти років госпіталізована з підозрою на пухлину матки. Після обстеження встановлено діагноз - фіброміома матки. В якому шарі матки розміщується ця пухлина?

Endometrium

Parametrium

Myometrium

Perimetrium

Mesometrium

4036 / 6854
При дослідженні амніотичної рідини, одержаної при амніоцентезі (прокол амніотичної оболонки), виявлені клітини, ядра яких містять статевий хроматин (тільце Барра). Про що з зазначеного це може свідчити?

Трисомія

Розвиток плода жіночої статі

Поліплоїдія

Розвиток плода чоловічої статі

Генетичні порушення в розвитку плода

4037 / 6854
У хворої 43-х років після чергового загострення ревмокардиту з’явилися ознаки декомпенсації серцевої діяльності з виникненням набряків на ногах і асциту. Затримці води в організмі хворої сприяло підвищення продукції:

Кортизолу

Кортикотропіну

Інсуліну

Альдостерону

Тироксину

4038 / 6854
У хворого спостерігається порушення зору - гемералопія ('куряча сліпота'). Який вітамінний препарат треба вживати хворому, щоб відновити зір?

Ретинолу ацетат

Токоферолу ацетат

Тіаміну хлорид

Піридоксин

Вікасол

4039 / 6854
Хвора 38-ми років надійшла в реанімаційне відділення в непритомному стані. Рефлекси відсутні. Цукор крові - 2,1 ммоль/л. В анамнезі: цукровий діабет з 18-ти років. Яка кома має місце у хворої?

Гіпоглікемічна

Гіперглікемічна

Кетоацидотична

Лактацидемічна

Гіперосмолярна

4040 / 6854
У збудливій клітині заблокували іонні канали, внаслідок чого клітина з часом повністю втратила потенціал спокою. Які канали заблокували?

Калієві та натрієві

Натрієві

Кальцієві

Калієві

Хлорні

4041 / 6854
У збудливій клітині заблокували іонні канали. Це не змінило суттєво рівень потенціалу спокою, але клітина втратила здатність до генерації ПД. Які канали заблокували?

Калієві

Натрієві

Натрієві та калієві

Хлорні

Кальцієві

4042 / 6854
У хворого діагностовано пухлину мозку, яка розміщена в ділянці 'пташиної шпори'. Порушення якої функції виникне у хворого, якщо пухлина буде активно розвиватися?

Дотикова чутливість

Зір

Смак

Нюх

Слух

4043 / 6854
У хворого виявлено порушення периферичного кровообігу, основою якого є обмеження припливу артеріальної крові. При цьому має місце збліднення даної ділянки, зниження місцевої температури. Це обумовлене:

Сладж-феноменом

Ішемією

Венозною гіперемією

Лімфостазом

Артеріальною гіперемією

4044 / 6854
Хворому 35-ти років для обстеження очного дна був призначений атропіну сульфат у вигляді очних крапель. Для відновлення акомодації йому закрапали пілокарпіну гідрохлорид, але це не дало бажаного ефекту Що є причиною відсутності ефекту?

Двосторонній антагонізм

Синергізм

Тахіфілаксія

Звикання

Односторонній антагонізм

4045 / 6854
В процесі експерименту на собаці виникла необхідність підвищити збудливість серцевого м’язу За допомогою введення якого іону можливо досягти бажаного ефекту?

С1~

Mg2+

К+ С Fe2+

Са2+

4046 / 6854
Гістони - ядерні білки, які пов’язані з ДНК іонним зв’язком. Які амінокислоти обумовлюють основний характер гісто- нів?

Іліцин та аланін

Пролін і метіонін

Аргінін і лізин

Серин і цистеїн

Лейцин та ізолейцин

4047 / 6854
Для діагностування деяких хромосомних хвороб використовують визначення статевого хроматину. Назвіть хворобу, при якій потрібне це визначення:

Хвороба Дауна

Гемофілія

Синдром Шерешевського-Тернера

Синдром Патау

Хвороба Брутона

4048 / 6854
До лікаря звернулась жінка зі скаргами на те, що після операції видалення щитоподібної залози у неї спостерігається порушення мови, захриплість. Пошкодження якого нерва під час операції могло стати причиною цього явища?

Під’язикового нерва

Язикового нерва

Верхнього гортанного нерва

Язиково-глоткового нерва

Зворотного гортанного нерва

4049 / 6854
У жінки, хворої на злоякісну пухлину молочної залози, лікар виявив збільшення регіонарних лімфатичних вузлів. Яка група вузлів підлягає ураженню при даній патології в першу чергу?

Пахвові вузли

Діафрагмальні вузли

Ліктьові вузли

Пахвинні вузли

Шийні вузли

4050 / 6854
До хірургічного відділення ЦРЛ надійшов хворий з колотою раною стопи, яку він отримав під час косовиці. Які специфічні препарати необхідно застосувати з метою екстреної активно-пасивної імунопрофілактики правця?

Протиправцева вакцина

Інтерферон

Антитоксична сироватка та анатоксин

Вакцина АКДП

Антибіотики

4051 / 6854
У п’ятимісячної дівчинки виявлено застійні явища у легенях. При обстеженні виявлено зв’язок між висхідною аортою та легеневою артерією, що в нормі спостерігається у деяких земноводних і плазунів. Назвіть цю природжену ваду розвитку:

Незрощення боталової протоки

Розвиток правої дуги аорти

Дефект міжпередсердної перегородки

Дефект міжшлуночкової перегородки

Транспозиція магістральних судин

4052 / 6854
Лікар-дослідник у складі альпіністської експедиції піднявся у базовий табір, розташований на висоті 5000 м. На 3-й день перебування у нього з’явилися ознаки гірської хвороби: задишка, головний біль, втрата апетиту, загальна слабкість, ціаноз. Який тип гіпоксії має місце в цьому випадку?

Змішана

Іемічна

Гіпоксична

Циркуляторна

Тканинна

4053 / 6854
У чоловіка 30-ти років перед операцією визначили групову належність крові. Кров резус-позитивна. Реакцію аглютинації еритроцитів не викликали стандартні сироватки груп 0оД (І), АЗ (II), Beu (ПІ). Досліджувана кров належить до групи:

ОоД (І)

Во (III)

АД (II)

AB (IV)

4054 / 6854
Локальне порушення кровопостачання міокарда призвело до порушення у клітинах синтезу АТФ та до припинення роботи натрій-калієвих насосів. Найбільш імовірним наслідком цього буде така зміна потенціалу спокою клітин у зоні пошкодження:

Суттєве збільшення

Зникнення

Несуттєве зменшення

Відсутність змін

Збільшення

4055 / 6854
На аутопсії тіла жінки, що хворіла на хронічну дизентерію, в ході гістологічного дослідження внутрішніх органів в стромі та паренхімі міокарда, нирок, в слизовій оболонці шлунка та в сполучній тканині легень виявлені аморфні відкладення фіолетового кольору, що дають позитивну реакцію за Коссом. Яке ускладнення розвинулося у хворої?

Метаболічне звапніння

Гіаліноз

Дистрофічне звапніння

Метастатичне звапніння

Амілоїдоз

4056 / 6854
Розпад глікогену в печінці стимулюється адреналіном. Який вторинний ме- сенджер (посередник) при цьому утворюється в клітині?

ц-ГМФ

ц-АМФ

Діацилгліцерол

CO D.NO

4057 / 6854
Дитина 3-х років із симптомами стоматиту, гінгівіту, дерматиту відкритих ділянок шкіри була госпіталізована. При обстеженні встановлено спадкове порушення транспорту нейтральних амінокислот у кишечнику. Нестачею якого вітаміну будуть зумовлені дані симптоми?

Ніацину

Кобаламіну

Пантотенової кислоти

Біотину

Вітаміну А

4058 / 6854
Хворий ходить хитаючись, широко розставляючи ноги. У нього знижений тонус м’язів рук і ніг, скандована мова. У якому відділі головного мозку локалізується ураження?

Хвостате ядро

Шкаралупа

Червоне ядро

Мозочок

Моторна кора

4059 / 6854
Встановлено, що токсична дія ціанідів виявляється у гальмуванні клітинного дихання. Який органоїд клітини є найбільш чутливим до цих отрут?

Лізосоми

Комплекс Іольджі

Мітохондрїї

Клітинний центр

Рибосоми

4060 / 6854
Юнак 17-ти років страждає на фурункульоз, викликаний умовно- патогенним Staphylococcus epidermidis. Яке дослідження найдоцільніше провести, щоб правильно вибрати препарат для лікування цього хворого?

Визначити антигенні властивості

Дослідити біохімічні властивості

Визначити фаговар

Скласти антибіотикограму

Виявити фактори патогенності

4061 / 6854
У хворого впродовж 10-ти днів має місце підвищена температура, напади характерного кашлю. Лікар призначив посів слизу з носоглотки на середовище казеїнова-вугільний агар. Який мікроорганізм передбачається виявити?

Стафілокок

Лістерію

Паличку коклюшу

Паличку інфлюенци

Клебсіелу

4062 / 6854
Хворому призначена ендоскопія 12- палої кишки. В результаті виявлено запалення великого дуоденального сосочка і порушення виділення жовчі в просвіт кишки. У якому відділі 12-палої кишки виявлені порушення?

Висхідна частина

Цибулина

Низхідна частина

Іоризонтальна частина

Верхня частина

4063 / 6854
У дитини, яка часто хворіє на ангіни та фарингіти, відзначається збільшення лімфовузлів і селезінки. Зовнішній вигляд характеризується пастозністю та блідістю, м’язова тканина розвинена слабко. У крові спостерігається лімфо- цитоз. Як називається такий вид діатезу?

Геморагічний

Нервово-артритичний

Астенічний

Лімфатико-гіпопластичний

Ексудативно-катаральний

4064 / 6854
У жінки 35-ти років, яка протягом З місяців обмежувала кількість продуктів у харчовому раціоні, спостерігається зменшення маси тіла, погіршення фізичного стану та розумової діяльності, з’явилися набряки. Дефіцит яких харчових речовин міг призвести до таких змін?

Мікроелементів

Жирів

Вуглеводів

Білків

Вітамінів

4065 / 6854
Хворому діагностовано недостатність мітрального клапана. Де даний клапан розміщується?

Між лівим і правим шлуночками

Між правим передсердям і правим шлуночком

Між лівим передсердям і лівим шлуночком

Між лівим і правим передсердями

В місці виходу аорти

4066 / 6854
При проведенні оперативного втручання на шиї хірургу потрібно виділити зовнішню сонну артерію. Що є анатомічним орієнтиром для встановлення місця початкового відділу вказаної судини в ділянці шиї?

Верхній край щитоподібного хряща

Нижній край щитоподібного хряща

Кут нижньої щелепи

Яремна вирізка

Місце початку грудинно-ключично- соскоподібного м’яза

4067 / 6854
У 18-річної дівчини в сільському районі Індії розвився профузний пронос з втратою рідини до 8 літрів на добу. Який з наведених нижче мікроорганізмів може бути збудником захворювання?

Salmonella typhi

Campylobacter jejuni

Ентеропатогенна Escherichia coli

Vibrio cholerae

Shigella dysenteriae

4068 / 6854
Хворий госпіталізований з попереднім діагнозом 'черевний тиф'. Яке живильне середовище з перерахованих можна використати для виділення гемо- культури?

Жовчний бульйон

МПБ

Середовище Левенштейна-Иенсена

Кров ’яний агар

Жовтково-сольовий агар

4069 / 6854
При обстеженні лікар встановив, що дисфункція кори головного мозку пацієнта викликана ураженням мережі нейронів мозкового стовбура, яка підтримувала активність кори великих півкуль. Які структури мозку уражені?

Ядра таламуса

Базальні ядра

Ретикулярна формація

Ядра мозочка

Ядра гіпоталамуса

4070 / 6854
У пацієнта перфоративна виразка передньої стінки шлунка. В яке похідне очеревини потрапить вміст шлунка?

Передшлункова сумка

Печінкова сумка

Лівий брижовий синус

Правий брижовий синус

Чепцева сумка

4071 / 6854
У результаті радіаційного випромінювання були ушкоджені стовбурові гемопоетичні клітини. Утворення яких клітин сполучної тканини буде порушено?

Меланоцити

Адипоцити

Фібробласти

Перицити

Макрофаги

4072 / 6854
Реалізація загального адаптаційного синдрому здійснюється переважно через нейроендокринну систему. Якій з ланок цієї системи належить провідна роль у патогенезі реакції, що розвивається?

Гіпофізарно-адреналова

Гіпофізарно-інсулярна

Гіпофізарно-адреногенітальна

Гіпофізарно-юкстагломерулярна

Гіпофізарно-тиреоїдна

4073 / 6854
У відділення реанімації надійшов хворий після ДТП з однобічним пневмотораксом. Який вид дихання спостерігається у цьому випадку?

Асфіктичне

Глибоке часте

Поверхневе

Поверхневе рідке

Поверхневе часте

4074 / 6854
У хворого зі скаргами на полідипсію, поліфагію, поліурію визначили високий рівень глюкози крові і наявність її у сечі. Яке захворювання можна припустити?

Нецукровий діабет

Хвороба Аддісона

Акромегалія

Цукровий діабет

Інсулінома

4075 / 6854
Хворому на активну форму туберкульозу призначили ізоніазид. Який вітамінний препарат необхідно застосовувати для профілактики побічної дії ізоніазиду?

Ретинолу ацетат

Токоферолу ацетат

Рутин

Ціанокобаламін

Піридоксину гідрохлорид

4076 / 6854
Відомо, що однією з причин виникнення мембранного потенціалу спокою є різниця концентрації іонів по обидві сторони клітинної мембрани. Який механізм забезпечує іонну асиметрію всередині і зовні клітини?

Фільтрування

Полегшена дифузія

Піноцитоз

Активний транспорт

Дифузія

4077 / 6854
Людина застосовувала дихальну методику, яка заснована на проведенні частого та поверхнього дихання, що супроводжується гіповентиляцією. Внаслідок чого при цьому розвивається респіраторний ацидоз?

Нестачі СО2 в крові

Нестачі О2 в крові

Надлишку С*О2 в крові

Надлишку О2 в крові

4078 / 6854
Під дією УФ-опромінення та інших факторів можуть відбуватися зміни в структурі ДНК. Репарація молекули ДНК досягається узгодженою дією всіх наступних ферментів, ЗА ВИНЯТКОМ:

ДНК-глікозид ази

ДНК-лігази

Ендонуклеази

ДНК-полімерази

Аміноацил-тРНК-синтетази

4079 / 6854
Батько і син мають різні мітохондрі- альні геноми. Це пояснюється тим, що ці геноми:

Підлягають комбінативній мінливості

Мають різну експресивність

Успадковуються від матері

Не успадковуються

Мають високий рівень мутабільності

4080 / 6854
У жінки, що тривало приймала антибіотики з приводу кишкової інфекції, розвинулось ускладнення з боку слизової порожнини рота у вигляді запального процесу і білого нальоту, у якому під час бактеріологічного дослідження були виявлені дріжджеподібні грибки Candi-da albicans. Який з перерахованих препаратів показаний для лікування цього ускладнення?

Фуразолідон

Бісептол

Флуконазол

Тетрациклін

Поліміксин

4081 / 6854
Хвора 57-ми років для лікування гіпертонічної хвороби тривалий час приймала анаприлін. Побічні ефекти спонукали пацієнтку відмовитись від прийому препарату, що призвело до розвитку гіпертонічного кризу і нападу стенокардії. Як називається ускладнення, яке виникло?

Сенсибілізація

Синдром відміни

Звикання

Лікарська залежність

Тахіфілаксія

4082 / 6854
При деяких гельмінтозах людина може сама виявити гельмінта, оскільки зрілі членики збудника можуть активно виповзати з ануса людини. Це характерно для:

Теніозу

Теніаринхозу

Дифілоботріозу

Ехінококозу

Гіменолепідозу

4083 / 6854
Жінка літнього віку перенесла сильний стрес. У крові різко збільшилась концентрація адреналіну і норадреналі- ну. Які ферменти каталізують процес інактивації катехоламінів?

Ілікозидази

Моноамінооксидази

Тирозиназа

Карбоксилаза

Пептидази

4084 / 6854
У чоловіка, померлого від внутрішньої кровотечі (гемоперитонеум), в печінці субкапсулярно виявлено губчастий вузол темно-червоного кольору розмірами 15x10 см, добре відмежований від навколишньої тканини. Мікроскопічно: тканина вузла складається з великих судинних тонкостінних порожнин, вистелених ендотеліальними клітинами та заповнених рідкою або згорнутою кров’ю. Встановіть вид пухлини:

Лімфангіома

Венозна гемангіома

Капілярна гемангіома

Кавернозна гемангіома

Іемангіоперицитома

4085 / 6854
На заняттях з лікувальної фізкультури лікар-фізіотерапевт запропонував юнакам відхилитися назад і дістати долонями до підлоги. Яка зв’язка запобігає надмірному розгинанню хребтового стовпа?

Надостьова

Міжпоперечна

Передня поздовжня

Задня поздовжня

Жовта

4086 / 6854
Вивчається мітотичний поділ клітин епітелію ротової порожнини. Встановлено, що в клітині диплоїдний набір хромосом. Кожна хромосома складається з двох максимально спіралізованих хроматид. Хромосоми розташовані у площині екватору клітини. Така картина характерна для стадії мітозу:

Анафаза

Телофаза

Прометафаза

Метафаза

Профаза

4087 / 6854
У хворого з хронічним гіперацидним гастритом з’явився біль у суглобах. Для полегшення болю, враховуючи супутню патологію, був призначений целекоксиб. Вибіркова дія цього препарату на певний фермент забезпечує відсутність впливу на слизову шлунка. Назвіть цей фермент:

Фосфоліпаза С

Циклооксигеназа 1

Циклооксигеназа 2

Фосфоліпаза А2

Калікреїн

4088 / 6854
Після травми печінки у хворого з’явились симптоми отруєння аміаком за типом печінкової коми. Як аміак діє на енергозабезпечення ЦНС?

Блокування ЦТК внаслідок зв’язування альфа-кетоглутарату

Інактивація ферментів дихального ланцюга

Іальмування бета-окислення жирних кислот

Іальмування гліколізу

Інгібування окисного фосфорилюван- ня

4089 / 6854
Бактеріологічний метод діагностики був використаний для підтвердження діагнозу газова гангрена у хворого. Які живильні середовища необхідно використовувати для культивування збудника в цьому випадку?

Ендо, Левіна, Плоскірєва

Вільсона-Блера, Кітта-Тароцці

МПА, МПБ

ЖСА, кров’яний агар

Лужний агар

4090 / 6854
У дівчинки 6-ти років виражені ознаки гемолітичної анемії. При біохімічному аналізі еритроцитів встановлено дефіцит ферменту глюкозо-6- фосфатдегідрогенази. Порушення якого метаболічного процесу відіграє головну роль у розвитку цієї патології?

Окисного фосфорилювання

Глюконеогенезу

Анаеробного гліколізу

Тканинного дихання

Пентозофосфатного шляху

4091 / 6854
Скорочення поперечно-посмугованих м’язів неможливе без кальцію. Яку роль відіграє цей іон в утворенні актино- міозинових містків?

З’єднується з гістаміновими рецепторами

З’єднується з холінорецептором

З’єднується з адренорецепторами

З’єднується з серотоніновими рецепторами

З’єднується з тропоніном

4092 / 6854
У хворого з синдромом Іценка- Кушинга спостерігаються стійка гіперглікемія та глюкозурія. Синтез та секреція якого гормону підвищені у цього хворого?

Тироксин

Глюкагон

Альдостерон

Кортизол

Адреналін

4093 / 6854
Обмеження споживання води призвело до зневоднення організму. Який механізм активується для збереження води в організмі?

Збільшення секреції тироксину

Збільшення секреції соматостатину

Збільшення секреції вазопресину

Зменшення секреції альдостерону

Зменшення секреції кальцитоніну

4094 / 6854
У хворого на подагру виявлено значне підвищення рівня сечової кислоти в крові. Кінцевим продуктом обміну яких речовин є сечова кислота?

Пуринових основ

Жирних кислот

Альбумінів

Глобулінів

Тригліцеридів

4095 / 6854
При тривалому використанні одного з гормональних препаратів у хворого виявлено ознаки остеопорозу, ерозії слизової шлунка, гіперглікемію, зменшення рівня АКТГ в крові. Препарат якої групи гормонів міг викликати ці явища?

Мінералокортикоїдів

Статевих гормонів

Иодвмісних гормонів

Ілюкокортикоїдів

Кальцитоніну

4096 / 6854
Людина потрапила в ситуацію, що пов’язана з емоційним напруженням. У результаті цього в неї в крові підвищився рівень адреналіну і, як наслідок, збільшилась сила серцевих скорочень. Яким чином адреналін збільшує силу серцевих скорочень?

Викликає збудження барорецепторів

Збуджує бета-адренорецептори

Впливає на барорецептори

Знижує тонус блукаючих нервів

Знижує збудливість клітин- пейсмекерів

4097 / 6854
У хворого на малярію після вживання протималярійного препарату при- махіну розвинулася гемолітична анемія. Спадкова недостатність якого ферменту в еритроцитах спостерігається при цьому?

Фосфофруктокінази

Ілюкозо-6-фосфатдегідрогенази

Ліпази

Тріозофосфатізомерази

Фруктозо-1-фосфатальдолази

4098 / 6854
Під час проведення морфологічного дослідження периферичної крові хворого було помічено, що у еритроцитів забарвлена лише периферична частина, а в центрі є незабарвлене прояснення. Кольоровий показник - 0,56. Яка анемія найімовірніша у цього пацієнта?

Сидеробластна

Апластична

Іемолітична

Залізодефіцитна

Ві2 фолієводефіцитна

4099 / 6854
На фельдшерський пункт звернулась жінка з приводу рваної рани правої кисті, що була спричинена власним мисливським собакою, який загинув через 5 днів від сказу. Які препарати потрібно використати для профілактики сказу у вкушеної жінки?

Антирабічна вакцина+антирабічний імуноглобулін

Інтерферон

Вакцина БЦЖ

Антирабічна сироватка-і-антибіотики

Антибіотики

4100 / 6854
У хворого, який звернувся в шкірно- венерологічний диспансер, виявлений твердий шанкр. Яке мікробіологічне дослідження дозволить поставити діагноз сифілісу на даному етапі хвороби?

Серологічний

Темнопольна мікроскопія

Бактеріологічний

Біологічний

Алергологічний

4101 / 6854
У фермера, що має стадо кіз, спостерігається лихоманка неясного походження. Найімовірніше, збудником захворювання є:

Brucella melitensis

Histoplasma capsulatum

S. aureus

Т pallidum

Clostridium novyi

4102 / 6854
Жінка 35-ти років розпочала голодування. Депо яких поживних речовин використовується у початковий період голодування і як при цьому змінюється дихальний коефіцієнт (ДК)?

Жири, ДК наближається до 0,72

Вуглеводи, ДК наближається до 1

Жири, ДК наближається до 0,85

Білки, ДК наближається до 0,7

Білки, ДК наближається до 1

4103 / 6854
У хворого діагностовано паразитарне захворювання, яке супроводжується клінічною картиною В12- фолієводефіцитної анемії. Який гельмінт спричинив дане захворювання?

Fasciola hepatica

Diphyllobothrium latum

Hymenolepis nana

Taenia solium

Taeniarhynchus saginatus

4104 / 6854
У хворого з підозрою на озену з носоглотки були виділені грамнегатив- ні палички, які утворювали капсулу на поживному середовищі. Які мікроорганізми спричинили хворобу?

Хламідії

Клебсієли

Сальмонели

Мікоплазми

Шигели

4105 / 6854
Чоловік з гострим міокардитом помер від серцево-судинної недостатності. В ході мікроскопічного дослідження внутрішніх органів виявлені: плазмора- гія, набряк, стази в капілярах, численні крововиливи, а також дистрофічні зміни в паренхімі. Наслідком чого є дані зміни?

Загальне артеріальне повнокрів’я

Гострий загальний венозний застій

Місцеве артеріальне повнокрів’я

ДВЗ -синдром

Хронічний загальний венозний застій

4106 / 6854
При складних методах фарбування виявляють особливості хімічного складу бактеріальної клітини або наявність певних структур. Який з перерахованих методів фарбування є основним та найчастіше вживаним?

Ожешко

Романовського-Гімзи

Ціля-Нільсена

Грама

Леффлера

4107 / 6854
Пацієнтці 68-ми років з туберкульозом легень призначено антибіотик, прийом якого може викликати забарвлення сечі та сльозної рідини в червоний колір. Назвіть його:

Етамбутол

Тетрациклін

Рифампіцин

Етіонамід

Амоксиклав

4108 / 6854
У жінки під час мейозу відбулося порушення розходження аутосом. Утворилася яйцеклітина із зайвою 18-ю хромосомою. Яйцеклітина запліднюється нормальним сперматозооном. У майбутньої дитини буде синдром:

Дауна

Шерешевського-Тернера

Патау

Едвардса

Клайнфельтера

4109 / 6854
У клініку госпіталізовано хворого з діагнозом карцинома кишечника. Для більшості карцином характерна підвищена продукція і секреція серотоніну. Відомо, що ця речовина утворюється з триптофану в ході реакції:

Декарбоксилювання

Мікросомального окислення

Утворення парних сполук

Дезамінування

Трансамінування

4110 / 6854
After severe emotional strain a 45-year-old man suddenly developed constricting pain in the area of his heart. The pain was irradiating into his left arm, neck, and left shoulder blade. His fase was pale and covered in cold sweat. Nitroglycerine was able to relieve the pain. What condition developed in the patient in this case?

Gastric ulcer perforation

Stroke

Psychogenic shock

Angina pectoris

Myocardial infarction

4111 / 6854
A 23-year-old woman was brought into the emergency department complaining of bloody diarrhea, tiredness, and dizziness. A few days ago she went to a fast food reataurant for a birthday party. Her friends are experiencing similar symptoms. Laboratory studies show anemia. What samples should be obtained for microbiologic testing in this case?

Urine

Stool

Blood

Bile

Cerebrospinal fluid

4112 / 6854
Histological microslide shows a gastrointestinal organ. The wall of this organ consists of 4 layers: mucosal, submucosal, muscular, and serous. The muscular layer has folds and pits. What organ has such appearance?

Appendix

Duodenum

Stomach

Esophagus

Small intestine

4113 / 6854
A 2-year-old child with delayed physical and mental development was brought to the hospital. The child’s parents are the most concerned by frequent profuse vomiting that occurs in their child after eating. Laboratory testing detected phenylpyruvic acid in the child’s urine. What type of metabolism is disturbed, causing this patology?

Lipid metabolism

Carbohydrate metabolism

Water and electrolyte metabolism

Amino acid metabolism

Phosphorus and calcium metabolism

4114 / 6854
The heart is a muscular organ that pumps blood through the body. This function is carried out by valves, muscles, and vessels. What valve is located between the left ventricle and left atrium?

Tricuspid

Bicuspid

Non-return

Pulmonary

Aortic

4115 / 6854
During your physiology class, the professor asks you to explain the effect of various hormones and neurotransmitters on the metabolism of glucose in the human body. You open your report with the statement that the use of glucose by the cells is preceded by its transport from the intercellular substance into the cell. What hormone is most likely responsible for the glucose uptake by the cell?

Thyroxine

Adrenaline

Insulin

Aldosterone

Glucagon

4116 / 6854
A 25-year-old man came to the family doctor complaining of the loss of taste sensation in the front two- thirds of his tongue. The doctor determined that this condition was caused by the damage to a certain nerve. What nerve is likely to be functionally impaired in this case?

Hypoglossal nerve

Chorda tympani

Vagus

Accessory nerve

Glossopharyngeal nerve

4117 / 6854
A 28-year-old man complains of nausea, vomiting, and right-sided subcostal pain. Objectively, his skin and sclerae are icteric, he has elevated body temperature, enlarged liver, dark urine, fecal hypocholia, hyperbilirubinemia (direct and indirect bilirubin), bilirubinuria, urobilinuria, hypoproteinemia, and decreased blood coagulability. What condition can be characterized by these changes?

Hepatocellular parenchymal jaundice

Acute pancreatitis

Pre-hepatic hemolytic jaundice

Acute cholecystisis

Post-hepatic jaundice

4118 / 6854
A soft tissue node obturated the patient’s middle lobe bronchus, leading to the development of right middle lobe atelectasis. In the area of obturation, bronchial biopsy detected proliferations of atypical glandular epithelium with pathological mitoses. Epithelium penetrates into the underlaying tissues and cartilage. What disease is the most likely in this case?

Bronchogetic lung cancer

Bronchial sarcoma

Inflammatory polyp

Bronchitis deformans

Bronchial epithelial dysplasia

4119 / 6854
The main functions of neural tissue can be described as communication and integration. What is the basic unit of neural tissue that ensures its functioning?

Myelin sheath

Dendrite

Axon

Neuron

Nucleus

4120 / 6854
The patient has developed a refractive error, where the focal point of the image is located not precisely on the retina, but anterior to it. This causes distant objects to be blurry, while close objects often appear normal. Among the other signs there are headaches and eye strain. This condition is associated with retinal detachment, cataract, and glaucoma. Name this condition:

Nearsightedness

Astigmatism

Blindness

Farsightedness

Emmetropia

4121 / 6854
A 20-year-old woman came to the clinic after missing her last 2 periods. Her cycles are usually regular, occurring at 28-30 day interval with moderate bleeding and occasional abdominal pain. She also complains of progressively diminishing peripheral vision. Her doctor determined the loss of vision in the lateral halves of both retinas. What structure is likely to be alfected, causing bitemporal hemianopsia in the patient?

Right optic tract

Left optic nerve

Optic chiasm

Left optic tract

Right optic nerve

4122 / 6854
A woman was bitten by a venomous snake (Macrovipera lebetinis). Autopsy of her body shows marked intravascular hemolysis. Her pancreas, bone marrow, and lymph nodes are brown. Microscopy detects abundant brown pigment in the macrophage cytoplasm. What pigment accumulates in the tissues in such cases?

Hematin

Bilirubin

Lipofuscin

Hematoidin

Hemosiderin

4123 / 6854
Histological microslide shows an organ consisting of gray and white matter. The gray matter is located in the center and is made up of bundle neurons (neurocytus funicularis), radicular neurons (neurocytus radiculatus), and interneurons. Such morphology is characteristic of the following organ?

Spinal cord

Spinal ganglion

Cerebral hemispheres

Medulla oblongata

Cerebellum

4124 / 6854
A molecular biologist studies various molecules. One of them is a polymeric molecule essential in various biological roles in coding, decoding, regulation, and expression of genes. It is a helical single­stranded molecule folded onto itself. Which of the following molecules is being studied?

ADP

DNA

HLA

ATP

RNA

4125 / 6854
There is a system that maintains the balanced state of multicellular organisms. Its main functions are to ensure an adequate physiological response and to defend the body against infectious agents and other unintended invasions. What system has such functions?

Immune system

Lymphatic system

Homeostatic system

Nervous system

Endocrine system

4126 / 6854
A team of medical students researches the phases of cell cycle. During one of the mitotic phases the cell has nearly completed its division, the chromosomes decondense, and two nuclei begin to form around them. What phase is likely observed in the cell at this moment?

Telophase

Prophare

Metaphase

Anaphase

4127 / 6854
A 34-yeat-old man visits his dentist complaining of a toothache. After a dental procedure that involved extraction of several teeth, he developed a severe bleeding lasting more than 15 minutes. He has a history of chromic hepatitis C. What is the most likely cause of the prolonged bleeding in this patient?

Hypocalcemia

Hypoalbuminemia

Thrombocytopenia

Hypofibrinogenemia

4128 / 6854
Medical examination in an army recruitment center detected a 15-year-old boy with tall stature, eunuchoid body proportions, gynecomastia, and female pattern of pubic hain growth. The boy has fat deposits on the thighs, no facial fair growth, high-pitched voice, and below average IQ. He was diagnosed with Klinefelter syndrome. What karyotype corresponds with this disease?

47, XYY

47, XXY

45, X0

46, XY

47, XY, +18

4129 / 6854
A man with signs of peritonitis was brought to the admission room. He has a 12-year-long history of peptic ulcer disease with ulcer localization on the posterior gastric wall. He was diagnosed with gasiric ulcer perforation. What anatomical structure is likely to be contaminated with gastric content in this case?

Bursa omentalis

Canalis lateralis sinister

Bursa hepatica

Bursa pregastrica

Canalis lateralis dexter

4130 / 6854
A 46-year-old man complains of tiredness and pain in the joints of his fingers and wrists. These signs are observed for the last 2 months. The pain is present in both hands and the wrists are swollen. Furthermore, he describes morning stiffness in his joints, lasting about 2 hours, which improves with use. His past medical history reveals that he was successfully treated for H. pylori - related ulcer last year. He denies smoking and stopped drinking when his GI symptoms started. What drug is the best choice for his joint pain management?

Prednisolone

Patacelanol

Morphine

Aspirin

Celecoxib

4131 / 6854
A 27-year-old woman complains of insomnia, irritability, hand tremor, acute weight loss despite high appetite, and constant fever with body temperature of 37,2-37,5 Co. What endocrine gland is likely to be functionally impaired in this case?

Neurohypophysis

Pancreas

Thyroid gland

Parathyroid gland

Adrenal glands

4132 / 6854
General practitioner performs physical examination of the patient. In the course of the examination it is necessary to measure the palpable regular expansion of a superficial artery caused by the ejection of blood into the vessels through heart contractions. What sign does the doctor measure?

Pulse

Heartbeat

Blood pressure

Heart electricity

Saturaion

4133 / 6854
A man has signs of acute respiratory viral disease. His physician referred him for microbiological testing. A mucus smear obtained trom the patient’s tonsils contains spherical microorganisins arranged in short chains. The patient was diagnosed with tonsillitis. What microorganisms were detected in the smear from the patient's tonsils?

Tetracocci

Staphylococci

Streptocci

Micrococci

Diplococci

4134 / 6854
Muscle relaxant tubocurarine was administered during a surgery. After the surgery the patient's breathing is not restored. What antidote should he be given to resore his breathing?

Proserin (Neostigmine)

Atropine sulfate

Anaprilin (Propranolol)

Clophelin (Clonidine)

Bemegride

4135 / 6854
A 45-yeat-old woman came to her physician with complaints of extreme tiredness and weakness. She says that these symptoms lasts for a month already. Within the last 2 weeks she losts 3 kilograms. Objectively, she is tired-looking thin woman. Skin hyperpigmentation is observed in many areas of her body, most prominently on the face, neck, and the backs of her hands (the areas exposed to light). What hormone is produccd in an excess in this patient, most likely causing the hyperpigmentation?

Growth hormone (GH)

P-Lipotopin

Melanocyte-stimulating hormone (MSH)

Thyroid-stimulating hormone (TSH)

Gonadotropins

4136 / 6854
A 50-year-old woman complains of constant thist. She drinks large amounts of liquid and has increased diuresis. Her blood glucose is 12 mmol/L. Her urine contains glucose. What endocrine organ is likely to be functionally impaired in this patient?

Parathyroid gland

Neurohypophysis

Adrenal glands

Pancreas

Thyroid gland

4137 / 6854
An unidentified surgical specimen is received fot histopathologic analysis. A portion of the specimen is cut and stained with hematoxylin and eosin. Under the microscope, you see an organ encapaulated by dense connective tissue that extends to the deeper areas by way of the trabecular extensions. The organ can be subdivided into two regions: a cortex with lymphoid nodules and medulla with medullary cords populated by plasma cells, B cells, and Tcells. What anatomical structure is the most likely origin of this surgical specimen?

Spleen

Tonsils

Lymph node

Bone marrow

Thymus

4138 / 6854
A 65-year-old woman was brought into the emergency departmnent because of shortness of breath and chest pain that started a few hours ago. She denied having fever, expectoration, or any accompanying symptoms. She had a 5-year-long history of deep vein thrombosis in her thin. After a time, she died of respiratory distress. Autopsy reveals red loose masses lodged in the bifurcation of the pulmonary trunk with extensions into both the left and the right pulmonary arteries. Whnt is the most likely diagnosis?

Pneumothorax

Myocardial infarction

Pneumonia

Thromboembolism

4139 / 6854
A 54-year-old woman had a total thyroidectomy lor papillary thyroid carcinoma. 11 hours after operation she complained of tingling around her mouth. On physical examination, the Trousseau's sign and Chvostek's sign are present. Her condition rapidly deteriorates with laryngospasm and focal seizures. The surgeon suspects surgical destruction of the parathyroid glands. What is the most likely cause of this patient's neurological condition?

Hyperkalemia

Hyponatremia

Hypophosphatemia

Hyperchloremia

Hypocalcemia

4140 / 6854
Унаслідок обтурації жовчовивідної протоки у хворого зменшилося надходження жовчі в 12-палу кишку, що призвело до порушення всмоктування:

Білків та вуглеводів

Білків

Жирів

Вуглеводів

Мінеральних солей

4141 / 6854
У хворого на шкірі обличчя поступово розвилась бляшка з некрозом і виразкою в центрі. Під час патогістологічного дослідження біоптату виявлено розростання атипових епітеліальних клітин із великою кількістю патологічних мітозів. Який діагноз найімовірніший?

Рак шкіри

Фіброма

Папілома

Саркома

Трофічна виразка

4142 / 6854
Речовини виводяться з клітини внаслідок з’єднання мембранної структури апарату Гольджі з цитолемою. Уміст такої структури викидається за межі клітини. Цей процес має назву:

Ендоцитоз

Екзоцитоз

Полегшена дифузія

Піноцитоз

Осмос

4143 / 6854
У гематологічному відділенні хворому на лейкоз лікар призначив 5-фторурацил, який:

Інгібує транскрипцію

Інгібує трансляцію

Інгібує синтез ДНК

Прискорює реплікацію

Стимулює ДНКазу

4144 / 6854
Під час операції у хворого виявили дивертикул Меккеля. У якому відділі шлунково-кишкового тракту він може розміщуватися?

Порожня кишка

Сліпа кишка

Сигмоподібна кишка

Дванадцятипала кишка

Клубова кишка

4145 / 6854
У хворого з хронічним гіперацидним гастритом з’явився біль у суглобах. Для полегшення болю, ураховуючи супутню патологію, був призначений целекоксиб. Вибіркова дія цього препарату на певний фермент забезпечує відсутність впливу на слизову шлунка. Назвіть цей фермент:

Циклооксигеназа 1

Циклооксигеназа 2

Калікрсїн

Фосфоліпаза С

Фосфоліпаза А2

4146 / 6854
Під час дослідження людини у вертикальній позі встановлено, що в альвео-лах верхівок легень парціальний тиск кисню становить 140 мм рт.ст. Причиною цього є те, що в цих відділах легень:

Перфузія переважає над вентиляцією

Вентиляція переважає над перфузією

Вентиляція відсутня

Перфузія та вентиляція врівноважені

4147 / 6854
У жінки 28 років із гнійно-серозним кон’юнктивітом і скаргами на біль під час сечовипускання взято для дослідження зішкріб із кон’юнктиви. Під час мікроскопії в цитоплазмі епітеліальних клітин виявлено включення. Під час посіву патологічного матеріалу на живильні середовища росту не виявлено, але під час зараження курячих ембріонів у жовтковий мішок вдалося виділити культуру збудника. Який із наведених мікроорганізмів є найімовірнішим збудником?

Аденовіруси

Хламідії

Дифтероїди

Мікоплазми

Гемолітичний стрептокок

4148 / 6854
У 70-ті роки науковці встановили, що причиною важкої жовтяниці новонароджених є порушення зв’язування білірубіну в гепатоцитах. Яка речовина використовується для утворення кон’югату?

Глюкуронова кислота

Піровиноградна кислота

Сечова кислота

Молочна кислота

Сірчана кислота

4149 / 6854
Під час операції на щитоподібній залозі з приводу захворювання на Базедову хворобу помилково були видалені паращитовидні залози. Виникли судоми, тетанія. Обмін якого біоелемента було порушено?

Магнія

Заліза

Кальція

Натрія

Калія

4150 / 6854
Жінка 35 років розпочала голодування. Депо яких поживних речовин використовується у початковий період голодування і як у цьому разі змінюється дихальний коефіцієнт (ДК)?

Жири, ДК наближається до 0,85

Білки, ДК наближається до 1

Білки, ДК наближається до 0,7

Жири, ДК наближається до 0,72

Вуглеводи, ДК наближається до 1

4151 / 6854
На розтині чоловіка 52 років, який тривалий час страждав на туберкульо-зний простатит і помер від менінгоен- цефаліту, у м’яких оболонках основи і бічних поверхонь головного мозку, селезінці, нирках, печінці виявлено вели-ку кількість щільних, сірого кольору вузликів діаметром 0,5 - 1мм. Гістологі-чне дослідження показало, що вузлики складаються з епітеліоїдних, лімфоїдних і нечисленних гігантських клітин з ядрами, розташованими на периферії клітин, що мають вигляд підкови. Виявлені зміни свідчать про:

Міліарний туберкульоз

Септикопіємію

Вторинний туберкульоз

Великовогнищевий дисемінований туберкульоз

Найгостріший туберкульозний сепсис

4152 / 6854
Пацієнту 33 роки. Хворіє 10 років. Періодично звертається до лікаря зі скаргами на гострий біль в животі, судоми, порушення зору. У його родичів спостерігаються подібні симптоми. Сеча червоного кольору. Госпіталізований із діагнозом 'гостра переміжна порфірія'. Причиною захворювання може бути порушення біосинтезу:

Колагену

Інсуліну

Гему

Жовчних кислот

Простагландинів

4153 / 6854
Цукровий діабет у хворої 35 років ускладнився виникненням і розвитком катаракти, ймовірною причиною якої є:

Глікозилювання білків кришталика і накопичення сорбітолу

Зниження спорідненості білків з киснем

Клітинна дегідратація

Дефіцит НАДФ • Н2 та гальмування синтезу жирних кислот

Порушення зв’язування рецепторів з інсуліном

4154 / 6854
В експерименті на щурах електричне подразнення головного мозку викликало у голодних тварин відмову від споживання їжі. Яку структуру стимулювали?

Бліду кулю

Гіпокамп

Мигдалеподібні ядра

Латеральну ділянку гіпоталамуса

Вентромедіальне ядро гіпоталамуса

4155 / 6854
Під час обстеженя пацієнта встановили сильний, урівноважений, інертний тип вищої нервової діяльності за Павловим. До якого виду темпераменту за Гіппократом відноситься пацієнт?

Холеричний

Меланхолічний

Флегматичний

Сангвінічний

4156 / 6854
У бактеріологічній лабораторії досліджувалися консерви, які стали причиною важкої токсикоінфскцїї. Під час мікроскопії культури із середовища Кітта- Тароцці виявлені грампозитивні споро- утворювачі палички, схожі на тенісну ракетку. Який діагноз поставить лікар?

Ботулізм

Дизентерія

Черевний тиф

Хламідіоз

Туляремія

4157 / 6854
У нормальних умовах кровообіг матері і плода не має прямих зв’язків. Кров ембріона протікає по судинах ворсинок хоріона, а материнська циркулює в міжворсинковому просторі ендометрію матки. Укажіть, що розділяє кров плода і матері:

Гемохоріальний бар’єр

Сполучнотканинні перегородки

Фібриноподібна оксифільна маса (фібриноїд Лангханса)

Замикальна пластинка основної відпадаючої оболонки матки

Аморфний фібриноїд Рора

4158 / 6854
У хворого на бронхіальну астму виникла гостра недостатність дихання. Який тип недостатності дихання виникає в цьому разі?

Перфузійний

Дисрегуляторне порушення альвеолярної вентиляції

Дифузний

Обструктивне порушення альвеолярної вентиляції

Рестриктивне порушення альвеолярної вентиляції

4159 / 6854
У 25-річного пацієнта на тлі гострого запалення легень із температурою тіла 40,1°С визначено лейкоцитоз 14,9 г/л із чітким зсувом лейкоформули вліво. Який із наведених чинників безпосередньо підвищує в кістковому мозку і проліферацію, і диференціацію лейкоцитів?

Колонієстимулюючий чинник

Інтерлейкін-1

Пухлинний некротичний чинник

Інтерлейкін-10

Простациклін

4160 / 6854
У хворого на мікросфероцитарну гемолітичну анемію (хворобу Мінковського-Шоффара) внаслідок підвищення проникності мембрани еритроцитів у клітину надходять іони натрію й вода. Еритроцити набувають форми сфероцитів і легко руйнуються. Який провідний механізм пошкодження еритроцитів у цьому разі?

Електролітно-осмотичний

Протеїновий

Кальцієвий

Ацидотичний

Нуклеїновий

4161 / 6854
У дорослої людини системний артеріальний тиск знизився з 120/70 до 90/50 мм рт.ст., що викликало рефлекторне звуження судин. У якому із за-значених органів звуження судин буде найбільшим?

Серце

Наднирники

Кишечник

Головний мозок

Нирки

4162 / 6854
У людини трапляється спадкова хвороба, симптомокомплекс якої поєднує в собі цироз печінки та дистрофічні процеси головного мозку. Вона супроводжується зменшенням вмісту церулоплазміну в плазмі крові та порушенням обміну міді в організмі. Це хвороба:

Німанна-Піка

Тея-Сакса

Вільсона-Коновалова

Марфана

Жильбера

4163 / 6854
У людини внаслідок тривалого голодування швидкість клубочкової фільтрації зросла на 20%. Найімовірніша причина змін фільтрації в таких умовах є:

Збільшення системного артеріального тиску

Збільшення коефіцієнта фільтрації

Зменшення онкотичного тиску плазми крові

Збільшення проникності ниркового фільтру

Збільшення ниркового плазмотоку

4164 / 6854
У відповідь на розтягнення м’яза спостерігається його рефлекторне скоро-чення. З подразнення яких рецепторів починається ця рефлекторна реакція?

Больові рецептори

М’язові веретена

Дотикові рецептори

Суглобові рецептори

Сухожилкові рецептори Гольджі

4165 / 6854
Для розслаблення скелетних м’язів, щоб зіставити відламки стегнової кіс-тки, що утворилися внаслідок перелому, хворому був введений міореалаксант, що призвело до зупинки дихання. Після введення свіжої цитратної крові дихання відновилося. Який міореалаксант ввели хворому?

Тубокурарину хлорид

Панкуронію бромід

Дитилін

Атракурію бесилат

Піпекуронію бромід

4166 / 6854
У чоловіка 55 років, який протягом багатьох років страждав на недоста-тність мітрального клапану, виникла гостра серцева недостатність. Який патофізіологічний варіант недостатності серця спостерігається у цьому разі?

Перевантаження серця тиском

Гостра тампонада серця

Гіпоксичне ушкодження серця

Перевантаження серця об’ємом

Коронарогенне ушкодження серця

4167 / 6854
У крові хворого з лейкопенією виявлено антилейкоцитарні антитіла. Який тип алергічної реакції за Кумбсом і Джеллом виник у цьому разі?

Імунокомплексний

Стимулюючий

Анафілактичний

Гіперчутливість сповільненого типу

Цитотоксичний

4168 / 6854
Для профілактики та лікування тромбозу застосовуються засоби, що знижують згортання крові (антикоагулянти). Укажіть антикоагулянт, у разі передозування якого застосовують як антагоніст протаміну сульфат:

Натрію гідроцитрат

Синкумар

Неодикумарин

Фснілін

Гепарин

4169 / 6854
Людина в стані спокою штучно примушує себе дихати часто і глибоко впродовж 3-4 хвилин. Як це відбивається на кислотно-лужній рівновазі організму?

Виникає метаболічний ацидоз

Виникає метаболічний алкалоз

Виникає дихальний ацидоз

Виникає дихальний алкалоз

Виникає змішаний ацидоз

4170 / 6854
У жінки 38 років розвинувся напад бронхіальної астми. Який із наведених бронхолітиків є ефективним для надання невідкладної допомоги та належить до групи бета-2-адреноміметиків?

Платифілін

Сальметерол

Адреналін

Сальбутамол

Іпратропію бромід

4171 / 6854
Під час УЗД вагітної в серцево- судинній системі плоду порушень не виявлено, артеріальна протока функціонує. Визначте, які судини вона з’єднує:

Пупкову вену з аортою

Легеневий стовбур із нижньою порожнистою веною

Легеневий стовбур із верхньою порожнистою веною

Пупкову вену із пупковою артерією

Легеневий стовбур з аортою

4172 / 6854
Під час огляду у хворого виявилося запалення анатомічного утворення, що врівноважує тиск між барабанною порожниною та глоткою. Назвіть це утворення:

Слухова труба

Внутрішній слуховий прохід

Зовнішній слуховий прохід

Внутрішнє вухо

Соскоподібна печера

4173 / 6854
Парубок звернувся до лікарні зі скаргами на порушення сечовипускання. Під час обстеження зовнішніх статевих органів виявлено, що сечівник розщеплений зверху і сеча витікає крізь цей отвір. Який вид аномалії розвитку зовнішніх статевих органів спостерігається у цьому разі?

Гермафродитизм

Гіпоспадія

Фімоз

Парафімоз

Епіспадія

4174 / 6854
Аналіз родоводу дитини з міотонічною дистрофією дав можливість уста-новити, що захворювання виявляється у кожному поколінні однаково у осіб обох статей, батьки однаковою мірою передають захворювання дітям. Якщо один із батьків хворий (гетерозигота), а другий - здоровий, ризик народження хворої дитини становитиме 50%. Визначте тип успадкування захворювання:

Х-зчеплене домінантне успадкування

Аутосомно-домінантний

Аутосомно-рецесивний

У-зчеплене успадкування

Х-зчеплене рецесивне успадкування

4175 / 6854
Хворому призначена ендоскопія 12- палої кишки. Унаслідок цього виявлено запалення великого дуоденального сосочка і порушення виділення жовчі в просвіт кишки. У якому відділі 12-палої кишки виявлені порушення?

Висхідна частина

Верхня частина

Цибулина

Горизонтальна частина

Низхідна частина

4176 / 6854
Молодому чоловікові було видано позитивну відповідь на наявність у нього антитіл до ВІЛ (використаний ІФА), проте обстежуваний наполягає на дослідженні, яке б достовірно довело наявність у нього антитіл до цього вірусу. Яке дослідження слід провести для підтвердження діагнозу?

РІФ

Імуноблотинг

ПЛР

РПГА

РГГА

4177 / 6854
Хворому проведено трепанацію і вишкрібання осередків відростка скроневої кістки через його гнійне запалення, яке перейшло з середнього вуха. На якому відростку здійснено хірургічне втручання?

Processus zygomaticus

Processus prerygoideus

Processus mastoideus

Processus styloideus

4178 / 6854
Дитина 2 років випила очні краплі з домашньої аптечки. Стан важкий, значне пото- і слиновиділення, астматичне дихання, кашель, зіниці різко звужені, тони серця глухі, брадикардія. Перистальтика кишечника посилена, пронос, артеріальний тиск знижений. Яким препаратом викликане отруєння?

Платифіліну гідротартрат

Анаприлін

Пілокарпіну гідрохлорид

Сульфацил-натрій

Атропін

4179 / 6854
Хворому перед екстракцією зуба була проведена провідникова анестезія лідокаїном, після введення якого з’явилися набряк і гіперемія навколо місця уколу, свербіння шкіри, загальна слабкість, гіпотензія, рухове збудження. Визначте, як називається ускладнення, що виникло:

Лікарська залежність

Толерантність

Алергічна реакція

Тахіфілаксія

Токсична дія

4180 / 6854
У студента медінституту, госпіталізованого в інфекційне відділення на 2-гу добу захворювання, припускають інфекційний мононуклеоз. Який результат лабораторного дослідження може підтвердити діагноз у цього студента в день госпіталізації?

Виявлення 4-разового наростання антитіл до вірусу Епштейна-Барр

Ізоляція (виділення) вірусу герпесу

Виявлення антитіл до цитомегаловірусу

Виявлення ІgМ-антитіл до вірусу простого герпесу

Виявлення ІgМ-антитіл до вірусу Епштейна-Барр

4181 / 6854
Для лікування кропив’янки з метою усунення сверблячого висипу на шкірі хворому призначений димедрол. Який механізм забезпечує його ефективність у цьому разі?

Конкурентна блокада Н1-рецепторів

Незалежний антагонізм з гістаміном

Інгібіція синтезу гістаміну

Прискорення руйнування гістаміну

Пригнічення вивільнення гістаміну

4182 / 6854
Юнак 25 років звернувся до лікаря зі скаргами на загальну слабкість, швидку втомлюваність, дратівливість, зниження працездатності, кровоточивість ясен. Недостатність якого вітаміну може мати місце у цьому разі?

Рибофлавін

Тіамін

Аскорбінова кислота

Ретинол

Фолієва кислота

4183 / 6854
Після накладання джгута у досліджуваного виявили точкові крововиливи. З порушенням функції яких клітин крові це пов’язано?

Тромбоцити

Моноцити

Лімфоцити

Нейтрофіли

Еритроцити

4184 / 6854
У пацієнта спостерігаються птоз (опущення повіки), розбіжна косоокість, порушення акомодації, розширення зіниць. Ядра якої пари черепних нервів уражені?

VI

IV

VII

III

V

4185 / 6854
Сечокам’яна хвороба ускладнилася виходом камінця з нирки. На якому рівні сечовода, найімовірніше, він може зупинитися?

У мисці

На межі черевної та тазової частин

На 5 см вище тазової частини

У середній черевній частині

На 2 см вище впадіння в сечовий міхур

4186 / 6854
У чоловіка 64 років спостерігаються симптоми різкого порушення процесів обміну речовин та енергії. Під час проведення комп’ютерної томографії в одній із ділянок головного мозку виявлена пухлина. Яка структура головного мозку, що відіграє значну роль у регуляції процесів обміну речовин, може бути уражена в цьому разі?

Ретикулярна формація

Гіпоталамус

Таламус

Чорна субстанція

Червоне ядро

4187 / 6854
Чоловік 40 років, м’ясник, номер від сепсису. На правій щоці його визначається конусоподібний, щільний, темно- червоний інфільтрат 6 см із чорною кірочкою в центрі. Права половина обличчя, шиї різко набряклі, щільні. Під час мікроскопічного дослідження в інфільтраті визначається вкрай гостре серозно- геморагічне запалення, у центрі інфільтрату спостерігається некроз епідермісу і підлеглих шарів. Який діагноз поставив патологоанатом?

Флегмона шиї

Туляремія

Сибірка

Чума

Фурункул

4188 / 6854
Хлопчик на другому році життя став часто хворіти на респіраторні захворювання, гноячкові ураження шкіри. Установлено, що в крові дитини практично відсутні імуноглобуліни всіх класів. Зниження функціональної активності якої клітинної популяції лежить в основі описаного синдрому?

Е. Нейтрофілів

Макрофагів

Т-лімфоцитів

NK-лімфоцитів

В-лимфоцитів

4189 / 6854
Моделюючи запалення на брижі жаби, спостерігали крайове стояння лейкоцитів та їх еміграцію крізь судинну стінку. Який із наведених факторів обумовлює цей процес?

Зменшення гідростатичного тиску в судинах

Зниження онкотичного тиску в судинах

Вплив хемотаксичних речовин

Збільшення онкотичного тиску в осередку запалення

Збільшення гідростатичного тиску в судинах

4190 / 6854
Дівчинка 11 років прийшла на прийом до лікаря із мамою, яка скаржиться на слабкість та набряклість обличчя її дитини протягом 3 днів. Мати стверджує, що до початку симптомів її дитина завжди була здоровою та активною. Під час фізикального обстеження виявлено генералізований набряк обличчя, якщо натискати на набряк нижніх кінцівок, залишається ямка, яка поступово згладжується. Під час збору анамнезу дівчинка зазначає пінистий вигляд сечі, але заперечує домішки крові у сечі, ніктурію або біль під час сечовиділення. Лабораторні дослідження виявили протеїнурію та мікрогематурію. Що з наведеного є найімовірнішою причиною змін у лабо-раторному аналізі сечі?

Підвищення гідростатичного тиску у клубочку

Підвищення проникності стінки клубочкового капіляру

Підвищення онкотичного тиску плазми крові

Підвищення гідростатичного тиску у капсулі Шумлянського-Боумена

4191 / 6854
Застосування еубіотика коліцину з лікувальною і профілактичною метою пов'язане з особливістю нспатогенних кишкових паличок синтезувати коліцин - речовину білкової природи, що пригнічує ріст патогенних видів мікроорганізмів. Яка структура кодує здатність бактеріальної клітини синтезувати коліцини?

Мeзосома

Плазміда

Рибосома

Ядро

Нуклеоїд

4192 / 6854
Хворому з неоперабельним раком легені, що супроводжується болем, який важко переносити, лікар призначив знеболювальний засіб. На цьому тлі у хворого виникла непрохідність кишечника. Який зі знеболювальних препаратів міг зумовити описане ускладнення?

Промедол

Морфін

Омнопон

Фентаніл

Анальгін

4193 / 6854
На прийомі у лікаря хворий повідомив, що самостійно застосовує протиалергійний препарат, який ефективно зменшує прояви алергії, проте викликає значну сонливість. Який препарат із групи блокаторів НІ-рецепторів застосовує хворий?

Ранітидин

Тавегіл

Кромолін-натрій

Димедрол

Лоратадин

4194 / 6854
Досить часто причиною набутих імунодефіцитів є інфекційне ураження організму, під час якого збудники розмножуються безпосередньо в клітинах імунної системи і руйнують їх. Виберіть серед наведених ті захворювання, за яких має місце вищезгадане:

Поліомієліт, гепатит А

Дизентерія, холера

Ку-гарячка, висипний тиф

Туберкульоз, мікобактеріоз

Інфекційний мононуклеоз, СНІД

4195 / 6854
Регуляція експресії генів здійснюється за допомогою різних механізмів. Назвіть ділянки ДНК, у разі індукції яких активується експресія гену:

Сайленсер

Енхансер

Термінатор

Атенюатор

Спейсср

4196 / 6854
Гемоглобін дорослої людини (НЬАІ) - білок-тетрамер, який складається з двох альфа- та двох бета-пептидних ланцюгів. Яку назву має така структура цього білка?

Третинна

Вторинна

Первинна

Четвертинна

4197 / 6854
У чоловіка ЗО років перед операцією визначили групову належність кро-ві. Кров резус-позитивна. Реакцію аглютинації еритроцитів не викликали стандартні сироватки груп Оа/3 (І), Ар (II), Ва (III). Досліджувана кров належить до групи:

Ва (III)

0аb (І)

АВ (IV)

Аb (II)

4198 / 6854
Недостатня продукція мінералокортикоїдів (Аддісонова хвороба) супроводжується м’язовою слабкістю, що зумовлена підвищеним виділенням із сечею іонів:

Натрію

Калію

Гідрогену

Кальцію

Магнію

4199 / 6854
У хворого після загострення хронічного калькульозного холециститу гостро розвинулася жовтяниця. Під час ЕКГ-обстеження звернено увагу, що на тлі правильного синусового ритму (ЧСС - 51/хв.) періодично з’являються екстрасистоли. Який механізм, найімовірніше, викликав порушення електричної активності серця?

Подразнення провідної системи токсинами, які не були знешкоджені в печінці

Подразнювальна дія жовчних кислот на синусовий вузол

Пошкоджуюча дія жовчних кислот на міокард

Пошкоджуюча дія жовчних кислот на синусовий вузол

Подразнення рецепторів блукаючого нерва жовчними кислотами

4200 / 6854
У пацієнта з аускультацією серця прослуховується патологічний піум у другому міжребер’ї праворуч від грудини. Ураження якого клапана можна припустити?

Мітрального

Аортального

Тристулкового

Легеневого

4201 / 6854
У новонародженої дитини спостерігається зниження інтенсивності смоктання, часте блювання, гіпотонія. У сечі та крові значно підвищена концентрація цитруліну. Який метаболічний процес порушений?

ЦТК

Орнітиновий цикл

Гліколіз

Глюконеогенез

Цикл Корі

4202 / 6854
У хворої дитини з підозрою на дифтерію було взято на дослідження ви-ділення ураженої слизової оболонки зіву. Приготовано і забарвлено мазок. Під час мікроскопії виявлено жовті палички з темно-синіми потовщеннями на кінцях. Який структурний елемент мікробної клітини визначається у виявлених мікроорганізмів?

Джгутики

Капсула

Зерна волютину

Спори

Плазміди

4203 / 6854
На розтині тіла померлої виявлено такі морфологічні прояви: стеноз ліво-го атріовентрикулярного отвору, недостатність мітрального клапана. Гістологічно в міокарді спостерігається вогнищевий кардіосклероз, наявність квітучих гранульом Ашоффа-Талалаєва. Який із наведених нижче діагнозів найімовірніший?

Системний червоний вовчак

Склеродермія

Ревматизм

Дерматоміозит

Вузликовий периартеріїт

4204 / 6854
Експериментальній тварині після попередньої сенсибілізації підшкірно введено дозу антигену. У місці ін’єкції розвинулось фібринозне запалення з альтерацією стінок судин, основної речовини та волокнистих структур сполучної тканини у вигляді мукоїдного та фібриної- дного набухання, фібриноїдного некрозу. Яка імунологічна реакція має місце?

Гранульоматоз

Реакція трансплантаційного імунітету

Гіперчутливість сповільненого типу

Гіперчутливість негайного типу

Нормергічна реакція

4205 / 6854
На електронній мікрофотографії ділянки нирки у стінці приносної та вино-сної артеріол визначаються клітини з великими секреторними гранулами в цитоплазмі. Визначте структурне утворення нирки, до складу якого входять ці клітини:

Юкстагломерулярний апарат

Дистальний відділ нефрона

Проксимальний відділ нефрона

Петля нефрона

Ниркове тільце

4206 / 6854
Для розвитку гарячкових станів характерним є зростання рівня білків 'го-строї фази” - церулоплазміну, фібриногену, С-реактивного протеїну. Укажіть можливий механізм цього явища:

Руйнівна дія підвищеної температури на клітини організму

Проліферативна дія ІЛ-2 на Т-лімфо- цити

Стимулювальний вплив ІЛ-1 на гепатоцити

Дегрануляція тканинних базофілів

4207 / 6854
Вивчаючи під електронним мікроскопом клітини підшлункової залози, було знайдено структури, які поділяють клітину на велику кількість комірок, каналів, цистерн та поєднані із плазмолемою. Укажіть ці органели:

Рибосоми

Комплекс Гольджі

Центросоми

Мітохондрїї

Ендоплазматична сітка

4208 / 6854
Хворий з хронічною серцево- судинною недостатністю приймав ди- гоксин. Після призначення додаткової терапії розвинулися явища інтоксикації серцевими глікозидами. Який препарат може викликати підсилення інтоксикації серцевими глікозидами?

Магнію хлорид

Розчин глюкози

Калію хлорид

Аспаркам

Кальцію хлорид

4209 / 6854
Надмірне споживання вуглеводів (600 г на добу), що перевищує енергетичні потреби у людини 28 років, буде супроводжуватися активацією:

Глюконеогенезу

Бета-окисленню жирних кислот

Гліколізу

Ліпогенезу

Ліполізу

4210 / 6854
Під час лабораторного дослідження дихальної функції крові встановлено, що має місце погіршення транспорту нею СО^. З дефіцитом якого ферменту це може бути пов’язано?

Карбоангідрази

Протеїнкінази

Аденілатциклази

2,3-дифосфогліцсрату

Фосфорилази

4211 / 6854
Під час ревізії черевної порожнини виявлено венозну кровотечу з печінково- дванадцятипалої зв’язки. Яку з вен пошкоджено?

Нижню брижову вену

Ворітну вену печінки

Селезінкову вену

Нижню порожнисту вену

Верхню брижову вену

4212 / 6854
Жінка 49 років тривалий час хворіла на хронічний гломерулонефрит, унаслідок чого настала смерть. На розтині встановлено, що нирки мають розміри 7x3x2,5 см, масу 65,0 г, щільні, дрібнозернисті. Також виявлено фібринозне запалення серозних і слизових оболонок, дистрофічні зміни паренхіматозних органів, набряк головного мозку. Яке ускладнення призвело до вказаних змін серозних оболонок і внутрішніх органів?

Тромбоцитопенія

ДВЗ-синдром

Сепсис

Анемія

Уремія

4213 / 6854
Хворому на стрептококову пневмонію призначено протимікробний засіб, що порушує побудову мікробної оболонки. Який це препарат?

Доксицикліну гідрохлорид

Азитроміцин

Еритроміцин

Гентаміцину сульфат

Бензилпеніциліну натрієва сіль

4214 / 6854
У препараті 10-дснного зародка людини видно 2 міхурці, що контактують між собою (амніотичний та жовтковий). Як називається структура, що лежить у місці їх контакту?

Позазародкова мезодерма

Дно амніотичного міхурця

Дах жовткового міхурця

Зародковий щиток

Амніотична ніжка

4215 / 6854
У 35-річного наркомана, який тривало хворів на фіброзно-кавернозний туберкульоз легень, під час патологоана- томічного дослідження встановлено, що нирки та селезінка збільшені в розмірах, щільної консистенції, на розрізі тканина із сірим відтінком та 'сальним” блиском. Мікроскопічно вичвлено в білій та червоній пульпі селезінки, у інтерстиції та мезангіумі клубочків нирок депозити Конго-рот-позитивних мас. Діагностуйте вид ураження внутрішніх органів:

Дифузний гіаліноз

Ідіопатичний амілоїдоз

Локальний пухлиноподібний амілоїдоз

Старечий амілоїдоз

Вторинний амілоїдоз

4216 / 6854
Під час лікування хронічного набрякового синдрому фуроссмідом у хворого виникло порушення катіонного складу плазми крові. Який засіб слід використовувати для його корегування?

Літію карбонат

Калію хлорид

Кальцію хлорид

Натрію гідрокарбонат

Натрію хлорид

4217 / 6854
У хворого 56 років із серцевою недостатністю спостерігається набряк стоп та гомілок, шкіра в місці набряку бліда і холодна. Яка провідна ланка патогенезу набряку у хворого?

Порушення лімфовідтоку

Зменшення онкотичного тиску в капілярах

Підвищення гідростатичного тиску ввенулах

Позитивний водний баланс

Підвищення проникності капілярів

4218 / 6854
Під час гістологічного дослідження біоптату печінки 67-річного чоловіка, який тривалий час хворіє на хронічну дифузну обструктивну емфізему, було знайдено такі морфологічні зміни: центральні вени дилатовані; у центрі часточок синусоїди гіперемовані, з ознаками капілярізації; частина гепатоцитів дистрофічно змінена; помірний периваску- лярний склероз; перипортально спостерігаються гепатоцити з ознаками жирової дистрофії, Діагностуйте вид ураження печінки:

Портальний цироз

'Мускатна” печінка

Жировий гепатоз

'Гусяча” печінка

'Кремнієва” печінка

4219 / 6854
Судово-медичний аналіз останків тіл царської сім’ї, ідентифікація останків тіла українського журналіста Георгія Гон- гадзе були здійснені за допомогою методу ДНК-діагностики - ланцюгової полі- меразної реакції, в основу якої покладено:

Аналіз нуклеотидного складу рРНК

Аналіз амінокислотного складу білків

Аналіз нуклеотидного складу тРНК

Аналіз нуклеотидного складу мРНК

Ампліфікацію генів

4220 / 6854
Під час реєстрації ЕКГ хворого з гіперфункцією щитоподібної залози зареєстровано збільшення частоти серцевих скорочень. Скорочення якого елемента ЕКГ про це свідчить?

Інтервалу Р-Q

Інтервалу Р-Т

Інтервалу R-R

Комплексу QRS

Сегмента Р-Q

4221 / 6854
Мікротравмування слизової оболонки порожнини рота виникають щодня під час вживання їжі, проте кровотеча швидко припиняється за допомогою:

Лізоциму

Гепарину

Муцину

Антигепаринового фактору

Тромбопластину

4222 / 6854
У хворого зіниця звужена і у разі зменшення освітлення не розширюється. У якому місці відбулося ураження центральної нервової системи?

Бічне колінчасте тіло проміжного мозку

Верхні горбики покришки середнього мозку

Бічний ріг сірої речовини спинного мозку на рівні С VIII - Тh І

Основа ніжки середнього мозку на рівні нижніх горбиків покришки

Покрив ніжки середнього мозку на рівні верхніх горбиків покришки

4223 / 6854
Хворий 38 років, який страждає на шизофренію, тривалий час перебував на лікуванні психотрошшми засобами. Він звернувся до лікаря зі скаргами на порушення координації рухів, тремор рук, сонливість. Яка група препаратів може викликати такий симптомокомплекс?

Адаптогени

Транквілізатори

Нейролептики

Антидепресанти

Психомоторні стимулятори

4224 / 6854
У хворого виявлено вивих кришталика та павукоподібні пальці. Який син-дром діагностує лікар, беручи до уваги ще й порушення розвитку сполучної тканини, форми кисті та стопи хворого?

Трисомїї X

Шерешсвського-Тернера

Дауна

Клайнфельтера

Марфана

4225 / 6854
У хворого з опіковою хворобою як ускладнення розвинувся ДВЗ-синдром. Яку стадію ДВЗ-синдрому можна припустити, якщо відомо, що час згортання крові хворого за Лі-Уайтом становить менше, ніж 3 хвилини?

Перехідну

Фібринолізу

Гіпокоагуляцїї

Термінальну

Гіпeркоагуляції

4226 / 6854
У хворого з двобічним ураженням надниркових залоз з’явилося темне ко-ричневе забарвлення шкірних покривів. Під час гістохімічного дослідження шкіри реакція Перлса негативна. Який пігмент зумовив зміну кольору шкіри?

Порфірин

Ліпофусцин

Білівердин

Меланін

Гемосидерин

4227 / 6854
Аміак особливо токсичний для ЦНС людини. Укажіть головний шлях його знешкодження в нервовій тканині:

Трансамінування

Синтез сечовини

Синтез глутаміну

Синтез солей амонію

Утворення парних сполук

4228 / 6854
На відміну від нервових клітин, які зазвичай не розмножуються, стовбурові клітини можуть відновлюватися багато разів. Процес багаторазового відновлення клітин називають:

Гіпертрофія

Диференціація

Проліферація

Апоптоз

Атрофія

4229 / 6854
Після хімічного опіку у хворого розвинувся стеноз стравоходу. Відбулося різке схуднення через утруднене вживання їжі. У крові виявлено еритроцити - 3,0 • 1012/л, Нb - 106 г/л, загальний білок -57 г/л. Який вид голодування у хворого?

Повне

Абсолютне

Білкове

Водне

Неповне

4230 / 6854
Реалізація загального адаптаційного синдрому здійснюється переважно через нейроендокринну систему. Якій із ланок цієї системи належить провідна роль у патогенезі реакції, що розвивається?

Гіпофізарно-адреногенітальна

Гіпофізарно-тиреоїдна

Гіпофізарно-адреналова

Гіпофізарно-інсулярна

Гіпофізарно-юкстагломерулярна

4231 / 6854
В обстежуваного відсутній колінний рефлекс. Укажіть рівень ушкодження спинного мозку:

VII - VIII грудні сегменти

III - IV поперекові сегменти

IX - X грудні сегменти

V - VII шийні сегменти

І - II поперекові сегменти

4232 / 6854
Чоловіка 45 років протягом останніх 3 років турбував сухий кашель, наростали задишка, легенева недостатність, швидка втрата ваги. На розтині встановлено легеневе серце. У легенях різко виражений фіброз із наявністю порожнин, що створюють картину 'медових стільників'. Гістологічно спостерігається інтерстиціальний фіброз з вираженою інфільтрацією строми лімфогістіоцита- ми з домішкою нейтрофілів. Установіть діагноз:

Хронічна бульозна емфізема

Пиловий пневмосклероз

Бронхіальна астма

Бронхоектатична хвороба

Післязапальний пневмосклероз

4233 / 6854
У гістопрепараті представлений орган, у власній пластинці слизової обо-лонки якого розташовані прості трубчасті залози, що складаються переважно з головних і парієтальних, а також слизових, шийкових ендокринних клітин. Укажіть вид залоз:

Власні залози шлунка

Кардіальні залози шлунка

Власні залози стравоходу

Пілоричні залози шлунка

Кардіальні затози стравоходу

4234 / 6854
У людини збільшена вентиляція легень унаслідок фізичного навантаження. Який з наведених показників зовнішнього дихання у неї значно більший, ніж у стані спокою?

Резервний об’єм вдиху

Життєва ємність легенів

Резервний об’єм видиху

Дихальний об’єм

Загальна ємність легенів

4235 / 6854
До стаціонару надійшов хворий з отруєнням грибами, серед яких випадково виявився мухомор. Крім промивання шлунка, активованого вугілля і сольових проносних усередину, а також інфузійної дезінтоксикаційної терапії, хворому призначили ін’єкції атропіну сульфату, унаслідок чого симптоми отруєння значно послабилися. Укажіть тип взаємодії мускарину (алкалоїду мухомора) і атропіну сульфату:

Опосередкований функціональний антагонізм

Фізико-хімічний антагонізм (антидоти-зм)

Непрямий функціональний антагонізм

Прямий функціональний однобічний антагонізм

Хімічний антагонізм

4236 / 6854
У пацієнта перфоративна виразка передньої стінки шлунка. У яке похідне очеревини потрапить вміст шлунка?

Печінкова сумка

Лівий брижовий синус

Правий брижовий синус

Передшлункова сумка

Чепцева сумка

4237 / 6854
Унаслідок токсичного ушкодження клітин печінки з порушенням білково- синтезуючої функції у пацієнта спостерігаються різко знижені кількість альбумінів у плазмі крові та онкотичний тиск. Яке явище буде наслідком цих змін?

Зменшення діурезу

Поява набряків

Зменшення ШОЕ

Зменшення густини крові

Збільшення темпу діурезу

4238 / 6854
Офтальмолог із діагностичною метою (розширення зіниць для огляду очного дна) використав 1% - й розчин мезатону. Мідріаз, викликаний препаратом, обумовлений:

Активацією а1-адренорецепторів

Активацією bі-адренорецепторів

Блокадою а1-адренорецеиторів

Активацією М-холінорецепторів

Активацією а2-адренорецепторів

4239 / 6854
Хворий 55 років обстежувався у ендокринолога через порушення ендокринної функції підшлункової залози, що виявляється у зменшенні гормона глюкагону в крові. Функція яких клітин цієї залози порушена в цьому разі?

А-клітини

В-клітини

D-клітини

D1-клітини

РР-клітини

4240 / 6854
У чоловіка, померлого від внутрішньої кровотечі (гемоперитонеум), у печінці субкапсулярно виявлено губчастий вузол темно-червоного кольору розмірами 15 х 10 см, добре відмежований від навколишньої тканини. Мікроскопічно виявлено, що тканина вузла складається з великих судинних тонкостінних порожнин, вистелених ендотеліальними клітинами та заповнених рідкою або згорнутою кров’ю. Установіть вид пухлини:

Лімфангіома

Капілярна гемангіома

Венозна гемангіома

Кавернозна гемангіома

Гсмангіопсрицитома

4241 / 6854
Аналіз крові виявив знижений вміст гемоглобіну. Яка функція крові порушиться?

Транспорт газів

Транспорт поживних речовин

Зсідання

Забезпечення імунітету

Транспорт гормонів

4242 / 6854
Пацієнт упродовж 15 років хворіє на бронхіальну астму. Які зміни лейкоцитарної формули може знайти лікар у цього пацієнта?

Лейкопенія

Базофілія

Еозинофілія

Зсув лейкоцитарної формули вліво

Лейкоцитоз

4243 / 6854
У хворого з підозрою на черевний тиф упродовж двох тижнів захворювання лабораторний діагноз не був встановлений. Який матеріал треба направити до лабораторії для бактеріологічного дослідження на третьому тижні?

Фекалії та сечу

Слиз із носу

Промивні води шлунка

Харкотиння

Слиз із зіву

4244 / 6854
Чоловік 40 років скаржиться на загальну слабкість, головний біль, кашель із виділенням мокротиння, задишку. Після клінічного огляду й обстеження поставлено діагноз ”пневмонія'. Який тип гіпоксії має місце у хворого?

Респіраторна

Тканинна

Гіпоксична

Циркуляторна

Гемічна

4245 / 6854
Після проведення туберкулінової проби (проба Манту) у дитини за 48 годин на місці введення туберкуліну утворилася папула діаментром до 10 мм. Який механізм гіперчутливості лежить в основі описаних змін?

Анафілаксія

Гранулематоз

Антитілозалежна цитотоксичність

Клітинна цитотоксичність

Імунокомплсксна цитотоксичність

4246 / 6854
Хворого доставили до клініки у коматозному стані. В анамнезі вказано цукровий діабет І типу впродовж 5 років. Об’єктивно: дихання шумне, глибоке, у видихуваному повітрі відчутно запах ацетону. Уміст глюкози у крові -18,2 ммоль/л, кетонових тіл - 100 мкмоль/л. Для якого ускладнення цього захворювання характерні такі розлади?

Гіперосмолярна кома

Гіперглікемічна кома

Кетоацидотична кома

Печінкова кома

Гіпоглікемічна кома

4247 / 6854
Хворому зі стрептококовою інфекцією ясен було призначено препарат, що містить у своїй структурі b-лактамне кільце. Який препарат із наведених належить до цієї групи?

Еритроміцин

Бензилпеніцилін

Рифампіцин

Левоміцетин

Стрептоміцину сульфат

4248 / 6854
Внутрішньоклітинне дослідження біопотенціалів ізольованої культури тканини показало, що потенціали дії, які виникають у клітинах, характеризуються наявністю плато тривалістю до 300 мс у фазі рeполяризації. Яку тканину досліджували?

Гладенький м’яз

Атипові кардіоміоцити

Нервове волокно

Скелетний м’яз

Скоротливий міокард

4249 / 6854
У хворого діагностовано бронхіальну астму. Які зміни показників вентиляції легень будуть спостерігатися?

Збільшення життєвої ємності легень

Зменшення об’єму форсованого видиху

Збільшення об’єму форсованого видиху

Зменшення залишкового об’єму легень

Збільшення резервуарного об’єму видиху

4250 / 6854
У хворого з гострим циститом під час дослідження сечі виявили лейкоцити й багато грамнегативних паличок. Під час посіву виросли колонії слизового характеру, які утворювали зелений, розчинний пігмент. Який мікроорганізм, імовірно, є причиною захворювання?

КІеbsіеllа рпеитопіае

Еsсhеrіhіа соli

Sаlтопеllа епtеrіtіdіs

Рsеиdотопаз аеrиgіпоsа

Рrоtеиs тіrаbіlіs

4251 / 6854
У дитини, яка часто хворіє на ангіни та фарингіти, спостерігається збільшення лімфовузлів і селезінки. Зовнішній вигляд характеризується пастозністю та блідістю, м’язова тканина роз-винена слабко. У крові спостерігається лімфоцитоз. Як називається такий вид діатезу?

Геморагічний

Ексудативно-катаральний

Лімфатико-гіпопластичний

Астенічний

Нервово-артритичний

4252 / 6854
Під час патоморфологічного дослідження жовчного міхура після холецистектомії виявлено, що розміри його збільшені, стінки потовщені, серозна оболонка тьмяна та повнокровна, у порожнині міхура містяться в’язкі жовто- зелені маси. Мікроскопічно в стінці міхура спостерігається дифузна інфільтрація сегментоядерними нейтрофілами. Яка форма холециститу найімовірніша?

Гострий флегмонозний

Хронічний

Гранульоматозний

Гострий катаральний

Гострий гангренозний

4253 / 6854
Унаслідок радіаційного випромінювання були ушкоджені стовбурові гемо- поетичні клітини. Утворення яких клітин сполучної тканини буде порушено?

Остеобласти

Макрофаги

Адипоцити

Меланоцити

Фібробласти

4254 / 6854
Гіповітаміноз С призводить до зменшення утворення органічного матриксу, порушення синтезу коллагену, тому що цей вітамін бере участь у процесах:

Гідроксилювання аргініну

Карбоксилювання проліну

Гідроксилювання проліну

Карбоксилювання лізину

4255 / 6854
Хвора 24 років потрапила до лікарні зі скаргами на головний біль, біль у поперековій ділянці, набряки на обличчі, загальну слабкість. Місяць тому перенесла ангіну. На час надходження: АТ - 180/110 мм рт.ст., у сечі вичвлено виражену протеінурію, мікрогематурію, лейко- цитурію. На яку форму гіпертензії страждає хвора?

Есенціальну

Первинну

Ендокринну

Гіпертонічну хворобу

Ниркову

4256 / 6854
У хворої похилого віку вже 4 рази траплявся перелом кісток верхньої кінцівки. З підвищенням місткості якої речовини пов’язане підвищення крихкості кісток у такому віці?

Неорганічних речовин

Органічних речовин

Міжклітинної рідини

Води

Сполучної речовини

4257 / 6854
У хворого 70 років перебіг атеросклерозу ускладнився тромбозом судин нижніх кінцівок, виникла гангрена пальців лівої стопи. Початок тромбоутворення, найімовірніше, пов’язаний з:

Перетворенням фібриногену в фібрин

Пeрeтворeнням протромбіну в тромбін

Адгезією тромбоцитів

Зниженням синтезу гепарину

Активацією протромбінази

4258 / 6854
Повільне наповнення шлунка чи сечового міхура в межах фізіологічної норми не викликає підвищення тиску в цих органах. Яка властивість гладеньких м’язів лежить в основі цього явища?

Пластичність

Автоматія

Збудливість

Скоротливість

Рефрактерність

4259 / 6854
Відомо, що в осіб із генетично обумовленою недостатністю глюкозо-6- фосфатдегідрогенази еритроцитів у відповідь на призначення деяких протималярійних препаратів може розвиватися гемоліз еритроцитів. Як називається цей прояв атипових реакцій на лікарські засоби?

Алергія

Толерантність

Сенсибілізація

Ідіосинкразія

Тахіфілаксія

4260 / 6854
Жінка 38 років скаржиться на постійну спрагу, часте сечовиділення, зниження апетиту, головний біль. Сеча безбарвна, прозора, слабокислої реакції, не містить глюкози. Добовий діурез до 12 л. Нестача якого гормона може бути причиною цього стану?

Передсердного натрійуретичного фактору

Вазопресину

Глюкагону

Інсуліну

Норадреналіну

4261 / 6854
Під час дослідження каріотипу п’ятирічного хлопчика виявлено 46 хромосом. Одна з хромосом 15-ї пари довша від звичайної, тому що до неї приєднана ділянка хромосоми з 21-ї пари. Укажіть вид мутації, що має місце в цього хлопчика:

Транслокація

Делеція

Дуплікація

Поліплоїдія

Інверсія

4262 / 6854
Хворий переніс повторний інтраму- ральний інфаркт міокарда. Після лікування та реабілітації був виписаний у задовільному стані під нагляд дільничого терапевта. За 2 роки загинув у автомобільній катастрофі. Який характер патологічного процесу в міокарді було вста-новлено на розтині?

Некроз

Дрібновогнищевий кардіосклероз

Гіперплазія

Атрофія

Великовогнищевий кардіосклероз

4263 / 6854
У експериментальних тварин із раціону виключили ліпоєву кислоту. Водночас у них спостерігалося пригнічення піруватдегідрогеназного комплексу. Чим є ліпоєва кислота для цього ферменту?

Коферментом

Субстратом

Продуктом

Інгібітором

Алостеричним регулятором

4264 / 6854
Унаслідок закупорки загальної жовчної протоки (установлено рентгенологічно) надходження жовчі до дванадцятипалої кишки припинилося. Слід очікувати на порушення:

Емульгування жирів

Гальмування слиновиділення

Секреції соляної кислоти в шлунку

Абсорбції білків

Гідролізу вуглеводів

4265 / 6854
Антибіотик олігоміцин інгібірує АТФ-синтазу. У якому процесі бере участь цей фермент?

У синтезі білків

У субстратному фосфорилюванні

У синтезі нуклеїнових кислот

У циклі трикарбонових кислот

В окисному фосфорилюванні

4266 / 6854
У жінки 62 років розвинулася катаракта (помутніння кришталику) на тлі цукрового діабету. Укажіть, який тип модифікації білків має місце у разі при діабетичної катаракти:

Фосфорилювання

Метилювання

АДФ-рибозилювання

Глікозилювання

Обмежений протеоліз

4267 / 6854
У жінки 50 років у яєчнику виявлено новоутворення округлої форми, діаметром 2,5 см. На розрізі в ньому є порожнина з прозорою світло-жовтою рідиною, внутрішня поверхня порожнини гладенька. Установіть макроскопічну форму пухлини:

Кіста

Вузол

Виразка

Інфільтрат

Вузол із некрозом у центрі

4268 / 6854
Після перенесеного менінгоенцефа- літу у хворої виявлені залишкові явища такі як ураження лицевого нерва з правого боку. Порушення якої групи м’язів буде спостерігатися за цієї патології?

Підпід’язикових м’язів

Надпід'язикових м’язів

Глибоких м’язів шиї

Жувальних м’язів

Мімічних м’язів

4269 / 6854
У жінки під час мейозу відбулося порушення розходження аутосом. Утворилася яйцеклітина із зайвою 18-ю хромосомою. Яйцеклітина запліднюється нормальним сперматозооном. У майбутньої дитини буде синдром:

Шерешевського-Тернера

Дауна

Клайнфсльтсра

Едвардса

Патау

4270 / 6854
Унаслідок травми мозку у жінки порушена функція епіфізу. Які з наведених функції будуть порушені у цієї жінки?

Частота серцевих скорочень

Менструальний цикл

Частота дихання

Цикл сон-неспання

Серцевий цикл

4271 / 6854
У жінки з ішемічною хворобою серця ЧСС на ЕКГ становить 230/хв., зубець Р деформований, шлуночкові комплекси без змін. Які порушення серцевого ритму в хворої?

Миготлива аритмія

Шлуночкова екстрасистола

Фібриляція шлуночків

Тріпотіння передсердь

Передсердна пароксизмальна тахікардія

4272 / 6854
Хворий 15 років надійшов до алергологічного відділення з діагнозом 'бронхіальна астма'. Надмірне утворення яких антитіл обумовлює розвиток основних клінічних симптомів?

ІgЕ

IgM

ІgА

ІgD

4273 / 6854
Хворий надійшов у реанімаційне відділення в стані гіпоксії, що виникла внаслідок аспірації блювотних мас. Об’єктивно: стан важкий, шкірні покриви вологі, бліді, з акроціанозом. Спостерігаються тахіпное, тахікардія, зниження АТ. Який із наведених симптомів гіпоксії, що розвивається гостро, належить до термінових захисно-пристосувальних реакцій організму?

Блідість шкірних покривів

Підвищення потовиділення

Зниження АТ

Розвиток акроціанозу

Тахікардія

4274 / 6854
Для окислення жирних кислот необхідна транспортна система, що включає аміноспирт, який транспортує жирні кислоти крізь мембрану мітохондрій. Назвіть цю сполуку:

Карнозин

Крсатинін

Кардіоліиін

Карнітин

Карбомоїлфосфат

4275 / 6854
Для усунення больового синдрому пацієнту з інфарктом міокарда лікар призначив анальгетичний препарат. Стан хворого поліпшився, але з часом з’явилися ейфорія, міоз. Лікар виявив пригнічення дихання. Який лікарський засіб був призначений?

Ібупрофен

Баралгін

Мелоксикам

Парацстамол

Морфіну гідрохлорид

4276 / 6854
На аутопсії тіла жінки, яка хворіла на хронічну дизентерію, під час гістологічного дослідження внутрішніх органів у стромі та паренхімі міокарда, нирок, у слизовій оболонці шлунка та в сполучній тканині легень виявлені аморфні відкладення фіолетового кольору, що дають позитивну реакцію за Коссом. Яке ускладнення розвинулося у хворої?

Амілоїдоз

Метаболічне звапніння

Гіаліноз

Метастатичне звапнення

Дистрофічне звапнення

4277 / 6854
Зі слизових оболонок і харкотиння хворого, який тривалий час приймав імунодепресанти, були виділені великі грампозитивні овальні клітини з брунькуванням, розташовані хаотично, та довгасті клітини, розташовані ланцюжками. Який збудник виділений?

Актиноміцети

Кандиди

Стрептококи

Стрсптобактерії

Ієрсинії

4278 / 6854
У 12-річного хлопчика в сечі виявлено високий вміст усіх амінокислот аліфатичного ряду. Водночас зафіксована найвища екскреція цистину та цистеїну. Крім того, УЗД нирок показало наявність у них каменів. Виберіть можливу патологію:

Хвороба Хартнупа

Фенілкетонурія

Сечокам’яна хвороба

Цистинурія

Алкаптонурія

4279 / 6854
У жінки з групою крові III (В), Rh (-) народилася дитина з групою крові II (А). У дитини діагностовано гемолітичну хворобу новонароджених, спричинену резус-конфліктом. Яка група крові та резус-фактор можливі у батька?

І (0), Rh (+)

II (А), Rh (+)

III (В), Rh (+)

І (0), Rh (-)

II (А), Rh (-)

4280 / 6854
Унаслідок травми відбувся розрив нюхових волокон, які виходять із порожнини носа. Крізь яку кістку проходять ці волокна?

Носову

Нижню носову раковину

Клиноподібну

Решітчасту

Верхню щелепу

4281 / 6854
Хворому 35 років для обстеження очного дна був призначений атропіну сульфат у формі очних крапель. Для відновлення акомодації йому закрапали пілокарпіну гідрохлорид, але це не дало бажаного ефекту. Що є причиною відсутності ефекту?

Звикання

Тахіфілаксія

Однобічний антагонізм

Двобічний антагонізм

Синергізм

4282 / 6854
Під дією декарбоксилаз утворюються біогенні аміни. Який біогенний амін запускає багатостадійний механізм регуляції секреції НСl у шлунку?

Серотонін

Глутамін

Гістамін

ГАМК

Дофамін

4283 / 6854
На спеціальному живильному середовищі після посіву гною, що виділяється з уретри, виросли ніжні блакитнуваті колонії. При мікроскопії препаратів з них виділені грамнегативні бобовидні диплококи. Збудником якої хвороби вони є?

Сифілісу

Меліоїдозу

Хламідіозу

Туляремії

Гонореї

4284 / 6854
Молекули зрілої іРНК у клітині є носієм генетичної інформації про послідовність з’єднання між собою певних амінокислот. Це означає, що в молекулах іРНК закодована:

Первинна структура білка

Первинна структура ліпідів

Вторинна структура вуглеводів

Первинна структура полінуклеотидів

Первинна структура вуглеводів

4285 / 6854
Під час споживання їжі, що містить білок глютен, розвивається целіакія (глютенова хвороба), яка характеризується дегенерацією кишкових ворсинок із втратою їх абсорбтивної функції, діареєю і стеатореєю, здуттям живота, втратою ваги та іншими позакишковими проявами. Глютен є білком:

Яєць

Пшениці

Рису

Суниць

Кукурудзи

4286 / 6854
Чоловік 48 років помер при явищах серцевої недостатності. Під час макроскопічного дослідження серця встановлено, що стулки мітрального клапану ущільнені, потовщені та помірно деформовані. Мікроскопічно виявлено пучки колагенових фібрил гомогенізовані, еозинофільні, оточені незначним ма- крофагальним інфільтратом, метахро- мазія не виявляється. У разі забарвлення пікрофуксином виявляються вогнища жовтого кольору. Діагностуйте вид ураження сполучної тканини:

Амілоїдоз

Склероз

Мукоїдне набухання

Гіаліноз

Фібриноїдне набухання

4287 / 6854
У дитини 15 років за 14 днів після перенесеної ангіни з’явилися набряки на обличчі зранку, підвищення артеріального тиску, сеча має вигляд 'м’ясних помиїв'. Імуногістохімічне дослідження біоптату нирки виявило відкладання імунних комплексів на базальних мембранах капілярів та у мезангії клубочків. Яке захворювання розвинулося у пацієнта?

Ліпоїдний нефроз

Гострий пієлонефрит

Гострий гломерулонефрит

Гострий інтерстиціальний нефрит

Некротичний нефроз

4288 / 6854
У хворого з підозрою на озену з носоглотки були виділені грамнегативні палички, які утворювали капсулу на поживному середовищі. Які мікроорганізми спричинили хворобу?

Сальмонели

Хламідїї

Клебсієли

Мікоплазми

Шигeли

4289 / 6854
Жінка літнього віку зазнала сильного стресу. У крові різко збільшилася концентрація адреналіну і норадреналіну. Які ферменти каталізують процес інактивації катехоламінів?

Тирозинази

Моноаміноксидази

Карбоксилази

Пептидази

Глікозидази

4290 / 6854
Під час макро- та мікроскопічних досліджень ділянки великогомілкової кістки та м'яких тканин довкола виявлено дифузне гнійне запалення, що захоплює кістковий мозок, гаверсові канали та периост, осередки некрозу. Яке захворювання можна припустити?

Паратиреоїдна остеодистрофія

Хронічний гематогений остеомієліт

Остеонекроз

Гострий гематогений остеомієліт

Хвороба Педжета

4291 / 6854
Під час аналізу крові виявлено фізіологічний лейкоцитоз. Причиною цього може бути те, що перед дослідженням людина:

Запальний процес

Палила тютюн

Не снідала

Поснідала

Випила води

4292 / 6854
У дитини 14-ти років був виявлений позитивний азотистий баланс. Що з наведеного може може бути причиною цього?

Ріст організму

Емоційне напруження

Голодування

Зниження вмісту білка у їжі

Значні фізичні навантаження

4293 / 6854
До клініки надійшла дитина 4-х років з ознаками тривалого білкового голодування: затримка росту, анемія, набряки, розумова відсталість. Причиною розвитку набряків у цієї дитини є зниження синтезу:

Альбумінів

Глікопротеїнів

Гемоглобіну

Ліпопротеїнів

Глобулінів

4294 / 6854
У чоловіка 58 років є ознаки атеросклеротичного ураження серцево-судинної системи. Збільшення якого з наведених показників біохімічного аналізу крові найхарактерніше для цього стану:

Рівня ЛПНЩ

Рівня ЛПВЩ

Хіломікронів

Активності сукципатдегідрогенази

Активності аланінамінотрансферази

4295 / 6854
Чоловік 38 років поступив у терапевтичне відділення з діагнозом: правосторонній ексудативний плеврит. Відкачана з плевральної порожнини грудної клітки рідина прозора, має відносну щільність 1.020; містить 55 г\л, білка альбуміно-глобуліновий коефіцієнт-1.6; загальна кількість клітин-2.8. в 1 мкл; pH-6.5.Який тип ексудату має місце у хворого?

Фібринозний

Гнійний

Серозний

Гнилісний

Геморагічний

4296 / 6854
Новонароджений, який з’явився на світ у домашніх умовах без кваліфікованої медичної допомоги і маги якого не ііроходила у період вагітності відповідних обстежень, на другу добу доставлений до лікарні з гострим гнійним кон’юнктивітом. Під час мікроскопії виділень з очей дитини виявлені грамнегативні диплококи, розташовані всередині лейкоцитів та поза клітинами. Який мікроорганізм є найімовірнішим збудником захворювання?

Chlamydia trachomatis

Pseudomonas aeruginosa

Neisseria gonorrhoae

Staphylococcus aureus

Corynebacterium diphtheriae

4297 / 6854
Під час мікроскопічного дослідження легені недоношеної дитини виявлено спадання стінок альвеол через відсутність сурфактанту. Укажіть, з порушенням розвитку яких клітин стінки альвеоли це пов’язано:

Альвеолоцитів І типу

Секреторних клітин

Альвеолярних макрофагів

Альвеолоцитів II типу

Фібробластів

4298 / 6854
У потерпілого колото-різана рана нижнього відділу задньої стінки пахвової ямки. Які м’язи пошкоджено?

Великий грудний м’яз

Дельтоподібний м’яз

Найширший м’яз спини

Підостний м’яз

Триголовий м’яз плеча

4299 / 6854
42-річний чоловік захворів гостро після переохолодження. Захворювання супроводжувалось серцево-дихальною недостатністю, від якої він помер. Під час розтину виявлено, що уся права легеня збільшена, гепатизована, на плеврі значні фібринозні накладення. На розрізі легеня сірого кольору, зерниста, з поверхні стікає каламутна рідина. Гістологічне дослідження виявило гостре запалення з наявністю у просвітах альвеол гнійно-фібринозного ексудату. Який із наведених діагнозів найімовірніший?

Осередкова пневмонія

Крупозна пневмонія

Інтерстиційна міжальвеолярна пневмонія

Ідіопатичний фіброзуючий альвеоліт

Стафілококова пневмонія

4300 / 6854
На ізольованому серці шляхом охолодження припиняють функціонування окремих структур. Яку структуру охолодили, якщо серце внаслідок цього спочатку припинило скорочення, а потім відновило їх із частотою у 2 рази меншою ніж вихідна?

Синоатріальний вузол

Атріовентрикулярний вузол

Волокна Пуркіньє

Пучок Гіса

Ніжки пучка Гіса

4301 / 6854
Жінці проведена операція з приводу позаматкової (трубної) вагітності. Гілки яких артерій повинен перев’язати хірург під час операції?

Маткової і нижньої міхурової

Маткової і яєчникової

Нижньої міхурової і яєчникової

Верхньої міхурової і яєчникової

Маткової і верхньої міхурової

4302 / 6854
У крові резус-негативної жінки під час вагітності виявлені специфічні білки, здатні руйнувати резус-позитивні еритроцити плода. Як називається цей захисний компонент організму матері?

Резус-фактор

Гормон

Антитіло

Сироватка

Фактор некрозу пухлини

4303 / 6854
Жінка 26 років через рік після важких пологів з кровотечею скаржиться на загальну слабкість, втрату маси тіла на 18 кг, відсутність менструацій. Об'єктивно: гіпоплазія молочних залоз. Діагностовано хвороба Сіммондса. Що являється основним механізмом втрати ваги у жінки?

Зниження функції статевих залоз

Гіпотиреоз

Зниження продукції гормонів аденогіпофізу

Зниження функції кіркового шару наднирників

Гіпопаратиреоз

4304 / 6854
Останніми роками застосовується метод геноіндикації збудників, що дає можливість виявити в досліджуваних зразках фрагменти нуклеїнових кислот патогенів. Виберіть із наведених реакцій ту, яка підходить для цього:

Реакція наростання титру фага

Радіоімунний аналіз

Реакція преципітації

Імуноферментний аналіз

Полімеразна ланцюгова реакція

4305 / 6854
Під час експерименту на жабі вивчали міотатичний рефлекс. Однак піл час розтягнення скелетного м’яза, його рефлекторне скорочення не відбулося. На порушення функції яких рецепторів слід звернути увагу?

Больових

Тактильних

Сухожильних органів Гольджі

Суглобових

М’язових веретен

4306 / 6854
У чоловіка, хворого на цукровий діабет, виникли значна спрага, дисфагія та порушення психічної діяльності. Який тип розладів водно-електроліпюго балансу характеризує поява цих ознак?

Дегідратація гіпоосмотична

Гідратація ізоосмотична

Гідратація гіпоосмотична

Дегідратація ізоосмотична

Дегідратація гіперосмотична

4307 / 6854
Пацієнту після видалення щитоподібної залози призначили препарат замісної терапії. Виберіть із наведених цей медикаментозний засіб:

Мерказоліл

L-тироксин

Калію йодид

Протирелш

Кортикотропін

4308 / 6854
На розтині чоловіка, який тривалий час хворів на пресенільне недоумство, виявлено атрофію кори головного мозку, стоншення переважно лобових, скроневих та потиличних часток. Мікроскопічно спостерігається: в корі атрофованих часток мозку виявляють старечі бляшки, пошкоджені нейрони, тільця Хірано. Який діагноз найімовірніший?

Розсіяний склероз

Церебральний атеросклероз

Хвороба Альцгеймера

Хвороба Паркінсона

Хвороба Кройцфельда-Якоба

4309 / 6854
Під час копрологічного дослідження встановлено, що кал знебарвлений, у ньому знайдено краплі нейтрального жиру. Найбільш імовірною причиною цього є порушення:

Надходження жовчі в кишечник

Секреції підшлункового соку

Процесів всмоктування в кишечнику

Секреції кишкового соку

Кислотності шлункового соку

4310 / 6854
Унаслідок гострої ниркової недостатності у чоловіка виникла олігурія. Яка добова кількість сечі відповідає симптому?

50-100 мл

1500-2000 мл

100-500 мл

1000-1500 мл

500-1000 мл

4311 / 6854
На аутопсії хворого, померлого від отруєння етиленгліколем, нирки дещо збільшені у розмірах, набряклі, їх капсула знімається дуже легко, кіркова речовина широка, блідо-сіра, мозкова речовина - темно-червона. Яка патологія нирок розвинулась у хворого?

Гострий тубуло-інтерстиціальний нефрит

Гострий пієлонефрит

Ліпоїдний нефроз

Некротичний нефроз

Гострий гломерулонефрит

4312 / 6854
Людина стоїть у кімнаті в легкому одязі. Іемпература повітря 14°С. Вікна і двері зачинені. Яким шляхом організм людини віддає найбільше тепла у таких умовах?

Конвекція

Теплопроведення

Перспірація

Випаровування

Теплорадіація

4313 / 6854
В епідермісі є клітини, що виконують захисну функцію та мають моноцитарний генез. Які цс клітини?

Кератиноцити зернистого шару

Кератиноцити базального шару

Меланоцита

Кератиноцити остистого шару

Клітини Лангерганса

4314 / 6854
Для лікування злоякісних пухлин призначають метотрексат - структурний аналог фолієвої кислота, який є конкурентним інгібітором дигідрофолатредуктази і тому гальмує синтез:

Моносахаридів

Глікогену

Жирних кислот

Гііцсрофосфатидів

Нуклеотидів ДНК

4315 / 6854
На заняттях із лікувальної фізкультури лікар-фізіотерапевт запропонував юнакам відхилитися назад і дістати долонями підлогу. Яка зв’язка запобігає надмірному розгинанню хребтового стовпа?

Надостьова

Жовта

Задня поздовжня

Міжпоперечпа

Передня поздовжня

4316 / 6854
Під час дослідження біоптату виявлена гранульома, що складається з лімфоцитів, плазматичних клітин, макрофагів із пінистою цитоплазмою (клітинами Мікуліча), багато гіалінових куль. Яке захворювання можна припустити?

Риносклерома

Лепра

Актиномікоз

Туберкульоз

Сифіліс

4317 / 6854
Жінці під час пологів перелили кров донора, який прибув із Анголи. За два тижні у рецишєнтки виникла пропасниця. Було запідозрено малярію. Яке лабораторне дослідження потрібно провести для підтвердження діагнозу?

Проведення серологічних досліджень

Вивчення мазка і товстої краплі крові для виявлення еритроцитарних стадій збудника

Вивчення пунктату лімфатичних вузлів

Визначення збудника методом посіву крові на живильне середовище

Вивчення лейкоцитарної формули

4318 / 6854
В умовах гострого експерименту кролику зробили перев’язку ниркової артерії. Унаслідок цього у тварини значно зріс рівень артеріального тиску, ще є результатом збільшення секреції.

Адреналіну

Реніну

Натріиуретичного гормону

Вазопресину

Норадреналіну

4319 / 6854
Жінка 40 років звернулася зі скаргами на неможливість розігнути стопу і пальці, що створює труднощі під час ходіння. Об'єктивно спостерігається: ступня звисає, дещо повернена всередину, пальці її зігнуті («кінська стопа»), чутливість втрачена на зовнішній поверхні гомілки та тильній поверхні стопи. Який нерв уражений?

Підшкірний

Великогомілковий

Стегновий

Сідничний

Спільний малогомілковий

4320 / 6854
Під час повторної постановки реакції аглютинації Відаля виявлено наростання титрів антитіл до О-антигенів S. typhi в сироватці пацієнта з 1:100 до 1:400. Як можна тлумачити отримані результати?

Є хронічним посієм черевнотифозних мікробів

Хворіє па черевний тиф

Є гострим носієм черевнотифозних мікробів

Раніше був щеплений проти черевного тифу

Раніше перехворів на черевний тиф

4321 / 6854
Аналіз сироватки крові пацієнта з гострим гепатитом показує підвищений рівень аланінамінотрансферази (АЛТ) та аспартатамінотрансферази (АСТ). Які зміни на клітинному рівні призвести до таких показників?

Пошкодження генетичного апарату клітин

Руйнування клітин

Порушення клітинних ферментних систем

Порушення міжклітинних взаємодій

Порушення енергопостачання клітин

4322 / 6854
У пацієнта з бронхіальною астмою за допомогою шкірних алергічних проб установлено сенсибілізацію алергеном тополиного пуху. Який фактор імунної системи відіграє вирішальну роль у розвитку цього імунопатологічного стану?

IgE

Сенсибілізовані Т-лімфоцити

IgG

IgM

IgD

4323 / 6854
Під час планового обстеження у вагітної взята кров з вени для постановки реакції Вассермана. Реакція виявилася позитивною. Позашлюбні статеві зв’язки вагітна та її чоловік заперечують. Що потрібно зробити, щоб підтвердити або спростувати діагностування сифілісу?

Поставити реакцію зв’язування комплементу

Повторити реакщю Вассермана

Зробити мазок з уретри

Поставити реакцію іммобілізації блідих трепонем

Поставити осадові реакції

4324 / 6854
У разі травматичного пошкодження верхніх кінцівок можливий розвиток дегенерації нервових волокон, яка супроводжується розпадом осьових циліндрів і мієліну. За допомогою яких нервових структур відбувається відновлення мієліну в процесі регенерації?

Нейролемоцитів (Шваннівських клітин)

Лізаксону

Псриневриту

Ендоневриту

Астроцитів

4325 / 6854
У чоловіка колаптоїдний стан через зниження тонусу периферичних судин. Який препарат найефективніший у цій ситуації?

Мезатон

Ізадрин

Празозин

Прозерин

Клофелін

4326 / 6854
Людина постійно живе високо в горах. Яку зміну показників крові можна виявити у неї?

Появу в крові еритробластів

Зниження Кількості рстикулоцитів

Зниження показників вмісту гемоглобіну

Зменшення колірного показника

Еритроцитоз

4327 / 6854
Людині, у якої напад бронхоспазму, треба зменшити вплив блукаючого нерва на гладеньку мускулатуру бронхів. Які мембранні циторецептори доцільно заблокувати для цього?

α-адренорецептори

α- та β-адренорецептори

М-холінорецептори

Н-холінорецептори

β-адренорецептори

4328 / 6854
Чоловік 35 років звернувся до лікарні зі скаргою на носові кровотечі. Лікар призначив пацієнту препарат, який є коагулянтом непрямого типу дії. Визначте цей препарат:

Фраксипарин

Вікасол

Тромбін

Кислота амінокапронова

Заліза сульфат

4329 / 6854
У чоловіка під час профілактичного обстеження на медіальній стінці лівої пахвової западини виявлено збільшений лімфовузол метастатичного походження. Укажіть найімовірнішу локалізацію первинної пухлини:

Щитоподібна залоза

Легеня

Шлунок

Молочна залоза

Піднижньощелепна слинна залоза

4330 / 6854
Чоловік 55 років прооперований із приводу гострого апендициту. На 5-ту добу, піднявшись із ліжка, відчув брак повітря. У нього розвинувся різкий ціаноз обличчя. Пацієнт втратив свідомість. Після безрезультатної реанімації констатована смерть. На розтині виявлена тромбоемболія легеневого стовбура. Що могло бути найімовірнішим джерелом тромбоемболії?

Кулястий тромб передсердя

Тромбоз брижових артерій

Тромбоз вен нижніх кінцівок

Тромбоз ворітної вени

Тромбоз у лівому шлуночку серця

4331 / 6854
У людей похилого віку часто спостерігається демінералізація кісток (зниженим вміст іонів кальцію). Причиною цього може бути знижена секреція:

Тироксину

Інсуліну

Тиреокальцитоніну

Альдостерону

Паратгормону

4332 / 6854
У жінки після операції видалення ракової пухлини молочної залози та регіонарних лімфовузлів розвинувся набряк руки, який був пов'язаний із лімфатичною недостатністю. Який це вид лімфатичної недостатності за механізмом виникнення?

Динамічна

Механічна

Акінетична

Спастична

Резорбційна

4333 / 6854
Дитина 3 років померла від гострої пневмонії на глі хронічної серцевої недостатності. Під час аутопсії виявлено: дефект міжпілуночкової перегородки, стеноз гирла легеневої артерії, гіпертрофія правого шлуночка серця, декстрапозиція аорти. Яка вада серця у дитини була встановлена під час аутопсії?

Синдром Марфана

Тетрада Фалло

Синдром Патау

Пентада Фалло

4334 / 6854
Чоловікові для введення лікувальної дози протиправцевої сироватки було зроблено пробу на чутливість, яка виявилась позитивною. Специфічну гіпосенсибілізацію хворому найправильніше провести введенням:

Малих доз протиправцевої сироватки

Дозволеної дози протиправцевої сироватки

Антигістамінних препаратів

Імунодепресантів

Глюкокортикоїдів

4335 / 6854
Артеріальна гіпертензія у жінки 44 років обумовлена наявністю феохромоцитоми - пухлини мозкового шару наднирників. Антигіпсртензивні засоби якої групи найдоцільніше буде призначити?

Антагоністи кальцію

Гангліоблокатори

Альфа-адреноблокатори

Симпатолітики

Бета-адреноблокатори

4336 / 6854
На ЕЕГ у потиличних відведеннях зареєстровано альфа-ритм. Яким є стан досліджуваного?

Спокій із розплющеними очима

Стрес

Стан наркозу

Спокій із заплющеними очима

Глибокий сон

4337 / 6854
44.Фермент оксидаза D-амінокислоі каталізує дезамінування лише D-амінокислот. Яка властивість ферментів тоді виявляється?

Абсолютна специфічність

Термолабільність

Відносна специфічність

Залежність від pH

Стереохімічна специфічність

4338 / 6854
Чоловікові, у якого є підозра на прогресуючу м’язову дистрофію, було зроблено аналіз сечі. Яка сполука в сечі підтверджує діагноз цього пацієнта?

Міоглобін

Колаген

Креатинін

Креатин

Порфирін

4339 / 6854
У чоловіка 50 років за місяць після перенесеного інфаркту міокарда розвинулася серцева недостатність. Невідповідність між навантаженням на серце та його здатністю виконувати роботу пов’язана в цьому разі з:

Змішаною формою серцевої недостатності

Підвищеним опором вигнанню крові в аорті

Перевантаженням серця

Підвищеним опором вигнанню крові в легеневому стовбурі

Ушкодженням міокарда

4340 / 6854
Пацієнтці, у якої наявні ротеїнурія гематурія, набряки і артеріальна гіпертензія, поставлено діагноз: гострий гломерулонефрит. Назвіть алергічні реакції за Кумбсом і Джеллом, які у 80% випадків є причиною гломерулонефриту

Стимулювального типу

Гіперчутливості сповільненого типу

Анафілактивні

Імунокомплексні

Цитотоксичні

4341 / 6854
Хворому на активну форму туберкульозу призначили ізоніазид. Який вітамінний препарат необхідно застосовувати для профілактики побічної дії ізоніазиду?

Токоферолу ацетат

Ретинолу ацетат

Піридоксину гідрохлорид

Рутин

Ціанокобаламін

4342 / 6854
Під час розтину тіла дівчинки 9 років у верхівці правої легені субплеврально було виявлено вогнище казеозного некрозу діаметром 15 мм, біфуркаційні лімфатичні вузли збільшені, містять дрібні вогнища некрозу коагуляційного типу Мікроскопічно спостерігається: у легеневому вогнищі та в лімфатичних вузлах навколо некротичних мас розташовані спітсліоїдиі клітини, лімфоцити та поодинокі багатоядерні гігантські клітини. Діагностуйте захворювання:

Первинний туберкульоз

Вторинний вогнищевий туберкульоз

Гематогенний туберкульоз із переважним ураженням легень

Вторинний фіброзно-вогнищевий туберкульоз

Гематогенний генералізований туберкульоз

4343 / 6854
Під час запальних процесів в органіці починається синтез білків «гостро. фази». Які речовини є стимуляторами їх синтезу?

Інтерлейкін-1

Інтерферони

Біогенні аміни

Ангіотензини

Імуноглобуліни

4344 / 6854
У хворого внаслідок зловживання алкоголем розвинулась жирова дистрофія печінки. Порушення якої ланки обміну ліпідів має місце в даному випадку?

Транспортування жирів

Обмін жирів у жировій клітковині

Всмоктування жирів

Проміжного обміну ліпідів

Бета - окислення ліпідів

4345 / 6854
У людини виявлено протозойне захворювання, унаслідок якого вражений головний мозок і спостерігається втрата зору. У крові знайдено одноклітинні півмісяцевоі форми із загостреним кінцем. Збудником цього захворювання є:

Амеба

Лейшманія

Лямблія

Токсоплазма

Трихомонада

4346 / 6854
У новонародженого хлопчика виявлена полідактилія, мікроцефалія, незарощення верхньої губи та верхнього піднебіння, а також гіпертрофія паренхіматозних органів. Вказані вади відповідають синдрому Патау. Яка найбільш ймовірна причина даної патології?

Трисомія 18-ої хромосоми

Нерозходження статевих хромосом

Трисомія 13-ої хромосоми

Часткова моносомія

Трисомія 21-ої хромосоми

4347 / 6854
Під час глікогенозу (хвороби Гірке) порушується перетворення глюкозо-6-фосфату на глюкозу, що призводить до накопичення глікогену в печінці та нирках. Дефіцит якого ферменту є причиною захворювання?

Глюкозо-6-фосфатаза

Фосфорилаза

Альдолаза

Гексокіназа

Глікогенсинтетаза

4348 / 6854
Жінка 40 років звернулась до лікаря зі скаргами на болі в дрібних суглобах ніг і рук. Суглоби збільшені, мають вигляд потовщених вузлів. У сироватці крові виявлено підвищений вміст уратів. Причиною є порушення обміну:

Вуглеводів

Амінокислот

Ліпідів

Пуринів

Піримідинів

4349 / 6854
Розділ фармакології, що вивчає всмоктування, розподіл та елімінацію ліків в організмі людини, називається:

Імунофармакологія

Фармакокінетика

Токсикологія

Фармакогенетика

Фармакодинаміка

4350 / 6854
3 хімічного виробництва до токсико логічного відділешія доставлено чоловіка з отруєнням ртуттю. Який препарат слід використати у цій ситуації?

Активоване вугілля

Налоксон

Ізонітрозин

Ентсросорбснт СКН

Унітіол

4351 / 6854
У чоловіка, хворого на гіпертонічну хворобу, спіронолактон викликав виражений терапевтичний ефект. Його терапевтична активність обумовлена ослабленням дії:

Альдостерону

Реніну

Ангіотензину II

Брадикініну

Ангіотензинперетворювального ферменту

4352 / 6854
Фенілкетонурія успадковується як аутосомно-рецесивна ознака. У здорових батьків народилася дитина, хвора на фенілкетонурію. Які генотипи батьків?

АА х Аа

аа х аа

Аа х аа

Аа х Аа

АА х АА

4353 / 6854
Хворого з явищами енцефалопатії госпіталізували до неврологічного стаціонару і виявили кореляцію між наростанням енцефалопатії і речовинами, що надходять із кишечнику до загального кровотоку. Які з'єднання, що утворюються в кишечнику, можуть викликати ендотоксемію?

Біотин

Бутират

Ацетоацетат

Орнітин

Індол

4354 / 6854
В організмі людини визначене порушення обміну мелатоніну. Це може бути пов’язано з нестачею амінокислоти, з якої мелатонін синтезується. Яка це амінокислота?

Гістидин

Глутамат

Аланін

Триптофан

ДОФА

4355 / 6854
Чоловіку 57 років, після обстеження був поставлений діагноз - В12 дефіцитна анемія, назначене лікування. Через 3 доби був зроблений контрольний аналіз крові. Що буде найбільш адекватним критерієм підвищення еритропоезу?

Зниження кольорового показника

Підвищення кількості тромбоцитів

Підвищення рівня гемоглобіну

Підвищення кількості лейкоцитів

Підвищення кількості ретикулоцитів

4356 / 6854
У разі зниження концентрації Na+ у плазмі крові в нирках посилюється його реабсорбція. Який основний механізм регуляції стимулює цей процес?

Натрійуретичний гормон

Ренін

Альдостерон

Парасимпатичні рефлекси

Симпатичні рефлекси

4357 / 6854
Хірург виявив у чоловіка болючість у правій пахвинній ділянці. Захворювання яких органів має місце?

Пряма кишка і сліпа кишка

Дуоденум, тонка кишка

Апендикс, сліпа кишка

Висхідна ободова кишка, права нирка

Підшлункова залоза і дуоденум

4358 / 6854
У хворої дитини з явищами гнійного керато-кон'юнктивіту лікар-офтальмолог запідозрив бленорею (гонорейний кон'юнктивіт). Якими методами лабораторної діагностики слід скористатися для підтвердження діагнозу?

Біологічним та алергічним

Серологічним та алергічним

Мікроскопічним та бактеріологічним

Мікроскопічним та серологічним

Біологічним та методом фагодіагностики

4359 / 6854
При вимірюванні артеріального тиску у чоловіка віком 56 років встановлено зростання діастолічного артеріального тиску до 100 мм рт. ст. Від якого із наведених факторів в основному залежить величина діастолічного артеріального тиску?

Об'єму циркулюючої крові

Периферичного опору судин

Швидкості кровотоку

Величини ударного об'єму лівого шлуночка

Величини кінцево-діастолічного об'єму лівого шлуночка

4360 / 6854
У чоловіка з клінічними ознаками імунодефіциту проведено імунологічні дослідження. Виявлено значне зниження кількості клітин, що утворюють розетки з еритроцитами барана. На основі даних аналізу можна зробити висновок, що у пацієнта наявне зниження кількості:

Компонентів системи комплементу

Клітин-ефекторів гуморального імунітету

В-лімфоцитів

Т-лімфоцитів

Натуральних кілерів (NK-клітини)

4361 / 6854
При мікроскопічному дослідженні біопсії нирки виявлено вогнища, в центрі яких знаходяться зернисті еозинофільні маси, оточені інфільтратом з лімфоцитів, епітеліоїдних клітин та поодиноких клітин Пирогова-Лангханса. Виберіть патологічний процес, що найбільш повно відповідає зазначеним змінам:

Коагуляційний некроз

Гранулематозне запалення

Альтеративне запалення

Проліферація та диференціювання макрофагів

Казеозний некроз

4362 / 6854
Чоловік 63 років звернувся до невропатолога зі скаргою на те, що протягом трьох місяців не може здійснювати столярні роботи, які потребують точності виконання, тому що права рука робить багато нецілеспрямованих рухів. Під час дослідження виявлено, що у хворого пошкоджена:

Gyrus precentralis

Gyrus postcentralis

Gyrus supramarginalis

Gyrus angularis .

Gyrus temporalis superior

4363 / 6854
У пацієнта під час обстеження зліва в V міжребірї на 1-2 см латеральншіе від середньоключичної Ліни під час аускультації краще прослуховується І іон, ніж П. Закриттям якого клапана цс обумовлено?

Лівим двостулковим клапаном

Двостулковим та тристулковим клапанами

Півмісяцсвим клапаном аорти

Півмісяцсвим клапаном легеневого стовбура

Правим трьохстулковим клапаном

4364 / 6854
На рентгенограмі газу видно всі три частини тазової кістки, відокремлені широкими проміжками, що відповідають невидимому на рентгенограмах хрящу. Про який вік суб'єкта можна говорити?

До 36 років

До 40 років

До 16 років

До 46 років

До 50 років

4365 / 6854
Подразнення слизової оболонки бронхів викликає кашльовий рефлекс. Де замикається рефлекторна дуга цього рефлексу?

Середній мозок

Передні роги спинного мозку

Бічні роги спинного мозку

Довгастий мозок

Уставні нейрони на рівні спинного мозку

4366 / 6854
Чоловіку, який страждає на гіпертонічну хворобу (AT - 200/110 мм pт. ст.), з-поміж препаратів комплексної терапії лікар призначив апаприлін (пропранолол). За 2 тижні після початку приймання цього препарату пацієнт став скаржитися на відчуття задухи, утруднення дихання. Поясніть можливу причину ускладнень і оберіть вашу тактику в цій ситуації:

Збудження М-холінорецспторів. Ппи- зїіачити атропіп

Алергічна реакція. Препарат скасувати, призначити антигістамінні засоби

Блокада бета-1-адренорецепторів. Призначити селективний бета-2-адрено- блокатор

Блокада бета-2-адренорецепторів. Призначити селективний бста-1-адрсно- блокатор

Міотропна бпонхоспастична дія Призначити еуфілін

4367 / 6854
Відповідно до сучасної концепції атерогенезу «Response to injury», атеросклероз є проявом хроні’пюго запалення в інтимі артерій. З якою стадією запалення пов’язане формування фіброзних бляшок у разі атеросклерозу?

Проліферація

Вторинна альтерація

Трансформація

Первинна альтерація

Ексудація

4368 / 6854
У людини трапляються хвороби, пов’язані із порушенням розщеплення і накопичення у клітинах глікогену, ліпідів та ін. Причиною виникнення цих спадкових хвороб є відсутність відповідних ферментів у:

Мітохондріях

Ендоплазматичній сітці

Ядрі

Мікротрубочках

Лізосомах

4369 / 6854
Для комплексної терапії туберкульозу було призначено препарат, що є антагоністом вітаміну Вб. Назвіть цей препарат:

Ізоніазид

Стрептоміцин

Етамбутол

Левофлоксацип

ПАСК

4370 / 6854
79.На одній зі стадій клітинного циклу хромосоми досягають полюсів клітини, Дссиіралізуються, навколо них формуються ядерні оболонки, відновлюється ядерце. У якій фазі мітозу перебуває клітина?

Прометафаза

Телофаза

Метафаза

Профаза

4371 / 6854
Під час мікроскопії мазка, приготованого з досліджуваного матеріалу від хворої дитини з підозрою на дифтерію і забарвленого за Неиссером, виявлено палички снітло-коричневого кольору з темно-синіми включеннями на кінцях. Який структурний елемент мікробної клітини виявлено?

Спори

Джгутики

Зерна волютину

Ядерна субстанція

Капсула

4372 / 6854
81. Судово-медичний експерт під час розтину тіла 20-річноі дівчини встановив, що смерть настала внаслідок отруєння ціанідами. Порушення якого процесу, найімовірніше, було причиною смерті дівчини?

Синтезу гемоглобіну

Тканинного дихання

Синтезу сечовини

Транспорту кисню гемоглобіном

Транспор ту водню за допомогою малат-аспартатного механізму

4373 / 6854
Під час перетворення глюкози в пентозному циклі утворюються фосфати різних моносахаридів. Яка із цих речовин може бути використана для синтезу нуклеїнових кислот?

Пентозо-5-фосфат

Седогептулозо-7-фосфат

Рибозо-5-фосфат

Рибулозо-5-фосфат

Еритрозо-4-фосфат

4374 / 6854
На розтині виявлено зменшені в розмірах нирки, вага 50,0 г, їх поверхня дрібнозерниста, кора рівномірно стоншена. Під час мікроскопічного дослі дження спостерігається: стінки артеріол значно потовщені внаслідок відкладен ня в них гомогенних безструктурних рожевого кольору мас, просвіт їх різко звужений, клубочки зменшені, склсрозовані, канальці атрофічні. Для якого захворювання характерні описані зміни?

Гострий некротичний нефроз

Гіпертонічна хвороба

Хронічний гломерулонефрит

Амілоїдоз нирок

Пієлонефрит зі зморщуванням нирок

4375 / 6854
Чоловіку з гострим гастроентероколітом після проведених бактеріологічних досліджень установлено діагноз холера. Який антибіотик є засобом вибору у разі цього захворювання?

Ампіцилін

Цефалексин

Пеніцилін

Оксацилін

Доксициклін

4376 / 6854
До дермато-венерологічного диспансера надійшла жінка ЗО років із діагнозом: первишшй сифіліс. Який засіб найдоцільніше їй призначити?

Тетрациклін

Бензилпеніцилін

Левоміцетин

Цефамізин

Феноксимстилпеніцилін

4377 / 6854
У дитячому садку планується проведення вакцинації проти кашлюку. Який із наведених нижче препаратів слід використати для цього?

Нормальний гамма-глобуліп

АДП анатоксин

Вакцина АКДП

Вакцина БЦЖ

Типоспецифічна сироватка

4378 / 6854
У дитини 3 років, яка страждає на квашіоркор, спостерігається порушення зроговіння епідермісу та збільшення його злущення, є жирова інфільтрація печінки. Який тип голодування спостерігається у цьому разі?

Жирове

Енергетичне

Білкове

Вуглеводне

Мінеральне

4379 / 6854
Під час огляду пацієнта лікар припустив синдром Іценка-Кушинга. Визначення якої речовини в крові пацієнта підтвердить припущення лікаря

Токоферолу

Адреналіну

17-кетостсроідш

Альдостерону

Кортизону

4380 / 6854
У крові пацієнта виявлено підвищення активності ЛДГ4, ЛДГ5, АлАТ, карбамоїлорнітинтрансферази. В якому органі можна передбачити розвиток патологічного процесу?

У скелетних м’язах

У нирках

У серцевому м’язі (можливий інфаркт міокарда)

У печінці (можливий гепатит)

У сполучній тканині

4381 / 6854
У чоловіка, який хворіє на верхній тип ожиріння, клінічно тривало спостерігалися артеріальна гіпертонія, гіперглікемія, глюкозурія. Смерть настала через крововилив у головний мозок. Під час патоморфологічного дослідження виявлені базофільна аденома гіпофіза і гіперплазія кори наднирникових залоз. Який діагноз є найправильнішим?

Акромегалія

Цукровий діабет

Адипозогенітальна дистрофія

Хвороба Іценка-Кушинга

Гіпофізарний нанізм

4382 / 6854
Учням 1-го класу поставлена проба Манту. З 35 учнів у 15 проба Мапту була негативною. Що треба зробити дітям із негативною пробою Манту?

Увести антитоксичну сироватку

Дослідити сироватку крові

Зробити повторну пробу

Увести вакцину БЦЖ

Увести антирабічну вакцину

4383 / 6854
В ядрі клітини з молекули незрілої і-РНК утворилася молекула зрілої і-РНК. яка має менший розмір, ніж незріла і-РНК. Сукупність етанів цього перетворення має назву:

Реплікація

Процесинг

Трансляція

Рекогніція

Термінація

4384 / 6854
У медичній практиці широкого застосування набули курареподібні речовини (міорелаксанти). Про яку побічну дію повинен пам’ятати лікар у разі їх застосування?

Тромбоутворення

Розслаблення дихальних м’язів

Зупинка серця

Напади судом

Розлади мозкового кровообігу

4385 / 6854
Цитохімічне дослідження виявило високий вміст гідролітичних ферментів у цитоплазмі. Про активність яких органел із наведених свідчить цей факт?

Лізосом

Ендоплазматичної сітки

Клітинного центру

Мітохондрій

Полісоми

4386 / 6854
Під час фіброгастродуодсноскопії лікарю необхідно оглянути великий сосочок 12-палої кишки. Який анатомічний утвір може бути орієнтиром для його виявлення?

Циркулярні складки 12-палої кишки

Цибулина 12-палої кишки

Дуоденальні залози

Печінково-дуоденальна зв’язка

Поздовжня складка 12-палої кишки

4387 / 6854
У чоловіка під час обстеження в клініці виявлено різке зниження показників гемоглобіну. Яка функція крові порушується?

Дихальна

Захисна

Трофічна

Імморальна

Гомеостатична

4388 / 6854
У жінки 32 років після перенесеного міокардиту виявлено під час електрокардіографічного дослідження порушення серцевого ритму (ритм не синусовий). Функції яких кардіоміоцитів порушені?

Пейсмекерних клітин

Перехідних провідних кардіоміоцитів

Провідних кардіоміоцитів ніжок пучка Гіса

Провідних кардіоміоцитів пучка Гіса

Скоротливих кардіоміоцитів

4389 / 6854
У дитини 7-ми років підвищилась температура тіла до 39oC, з’явилися нежить, кон’юнктивіт, кашель. На шкірі відзначались великоплямисті висипання, під час огляду порожнини рота - білуваті висипання на слизовій оболонці щік. Раптово з’явилось важке дихання і настала смерть при явищах асфіксії. Який найбільш імовірний діагноз?

Менінгококовий назофарингіт

Дифтерія

Грип

Кір

Скарлатина

4390 / 6854
Під час аналізу крові виявлено фізіологічний лейкоцитоз. Причиною цього може бути те, що перед дослідженням людина:

Палила тютюн

Випила води

Запальний процес

Не снідала

Поснідала

4391 / 6854
У хворого 30 років на електрокардіограмі відмічено зниження амплітуди зубця R. Що означає цей зубець на ЕКГ?

Реполяризацію шлуночків

Електричну діастолу серця

Поширення збудження від передсердь до шлуночків

Деполяризацію передсердь

Деполяризацію шлуночків

4392 / 6854
У пацієнта після травми виникли паралічі, розлади больової чутливості справа. Зліва паралічі відсутні, але порушена больова і температурна чутливість. Яка причина такого явища?

Одностороннє ураження спинного мозку з правої сторони

Пошкодження рухової зони кори головного мозку

Пошкодження мозочка

Одностороннє ураження спинного мозку з лівої сторони

Пошкодження стовбура мозку

4393 / 6854
У чоловіка, у якого яскраво виражена жовтушність шкіри, склер та слизових оболонок, сеча має колір темного пива, кал ахолічний. У крові виявлено підвищений вміст прямого білірубіну. У сечі - білірубін. Який тип жовтяниці у пацієнта?

Кон'югаційна

Екскреційна

Обтураційна

Паренхіматозна

Гемолітична

4394 / 6854
Показники аналізу крові 35-річного чоловіка: НЬ — 58 г/л, еритроцити — 1,3 • 10,2/л, колірний показник - 1,3, лейкоцити - 2,8 • 109/л, тромбоцити - 1,1* 109/л, ретикулоцити - 2%, ШОЕ - 35 мм/год. Визначаються полісегментовані нейтрофіли, а також тільця Жоллі, кільця Кебота. мегалоцити. Яка це анемія?

В12-фолієводефіцитна

Залізодефіцитна

Гіпопластична

Постгеморагічна

Гемолітична

4395 / 6854
Для лікування стрептодермії лікар призначив хворому мазь, що містить антибіотик групи тетрацикліну, і рекомендував обмежити перебування на сонці. Чим небезпечна інсоляція?

Зниженням активності тетрацикліну

Розвитком стійкості збудника до антибіотика

Підвищенням токсичності тетрацикліну

Генералізацією процесу

Розвитком фотосенсибілізації

4396 / 6854
У чоловіка гострий гломерулонефрит. Поява якої речовини в сечі свідчить про ушкодження базальної мембрани капілярів клубочків нирок у разі цієї патології?

Лейкоцитів

Ппокозн

Креатину

17-кетостероїдів

Білку

4397 / 6854
У дитини, яка страждає на геморагічний синдром, діагностована гемофілія. Вона зумовлена дефіцитом фактора:

VІІІ (антигемофільного глобуліну)

IX (Крістмаса)

XI (протромбопластину)

II (протромбіну)

XII (Хагемана)

4398 / 6854
Унітіол є антидотом і застосовується, зокрема, у разі отруєнь солями важких металів. Як називається такий тин взаємодії лікарських речовин?

Неконкурентний антагонізм

Хімічний антагонізм

Синергоантагонізм

Фізичний антагонізм

Фізіологічний антагонізм

4399 / 6854
Після введення місцевого анестетика у пацієнта розвинувся анафілактичний шок. Який механізм порушення кровообігу є провідним у цьому разі?

Зниження тонусу судин

Активація симнато-адреналової системи

Гіперволемія

Підвищення тонусу сулин

Зниження скоротливої функції серця

4400 / 6854
Для підвищення результатів спортсмену рекомендували застосовувати препарат, який містить карштин. Який процес найбільше активується карнітином?

Синтез кетонових тіл

Синтез ліпідів

Тканинне дихання

Транспорт жирних кислот у мітохондрії

Синтез стероїдних гормонів

4401 / 6854
Розтин тіла показує, що м’які мозкові оболонки померлої людини є повноцінними, потовщеннями, непрозорими та мають жовто-зелений колір. Який тип ексудативного запалення може характеризуватися такими змінами в м’яких мозкових оболонках?

Геморагічне

Катаральне

Серозне

Фібринозне

Гнійне

4402 / 6854
До клініки надійшла дитина 4 роківз ознаками тривалого білкового голодування: затримка росту, анемія, набряки, розумова відсталість. Виберіть причину розвитку набряків у дитини:

Зниження синтезу гемоглобіну

Зниження синтезу ліпопротеїиів

Зниження синтезу глікопротеїнів

Зниження синтезу альбумінів

Зниження синтезу глобулінів

4403 / 6854
товщі шкіри макроскопічно виявлено рухливу пухлину. Мікроскопічно спостерігаються хаотично розташовані пучки колагенових волокон із невеликою кількістю веретеноподібних клітин. Яка пухлина виявлена?

Ліпома

Гломус-ангіома

Лейоміома

Меланома

Фіброма

4404 / 6854
113. Під час обстеження у чоловіка похилого віку виявлено моторну афазію. Де локалізований осередок пошкодження головного мозку?

Центр Брока

Прецентральна звивина

Постцентральна звивина

Звивина Гешля

Кутова звивина

4405 / 6854
Чоловіку, після інфаркту міокарда, призначено ацетилсаліцилову кислоту, 75 мт щоденно. З якою метою призначено препарат?

Зменшення агрегації тромбоцитів

Зниження температури

Зменшення запалення

Розширсшія коронарних судин

Зменшення болю

4406 / 6854
Чоловік 25 років звернувся до приймального покою зі скаргами на нежить та головний біль протягом 4 днів. Після обстеження йому встановили діагноз: фронтит. Через який носовий хід інфекція потрапила до лобової пазухи?

Нижній

Середній

Носоглотковий

Верхній

Загальний

4407 / 6854
Злоякісна гіперхромна анемія – хвороба Бірмера – виникає внаслідок нестачі вітаміну В12. Який біоелемент входить до складу цього вітаміну?

Цинк

Магній

Молібден

Залізо

Кобальт

4408 / 6854
Хвора 13 років знаходиться на стаціонарному лікуванні в гематологічному відділенні обласної дитячої лікарні з діагнозом залізодефіцитна анемія. Який тип гіпоксії має місце у цієї хворої?

Змішана

Дихальна

Циркуляторна

Тканинна

Гемічна

4409 / 6854
При макро-мікроскопічному дослідженні ділянки великогомілкової кістки та м’яких тканин довкола виявлено дифузне гнійне запалення; що захоплює кістковий мозок, гаверсові канали та періост осередки некрозу Про яке захворювання слід думати?

Хвороба Педжета

Гострий гематогений остеомієліт

Остеонекроз

Хронічний гематогений остеомієліт

Паратиреоїдна остеодистрофія

4410 / 6854
Жінка 62 років скаржиться на частий біль у ділянці грудної клітки та хребта, переломи ребер. Лікар припустив мієломну хворобу (плазмоцитому). Який з перерахованих нижче лабораторних показників буде мати найбільше діагностичне значення?

Гiперальбумiнемiя

Гіпопротеїнемія

Парапротеїнемiя

Гіпогло6улінємія

Протєїнурія

4411 / 6854
Чоловік, хворий на бронхіальну астму, тривалий час приймає преднізолон. Який механізм дії препарату?

Гальмування активності фосфоліпази А2

Блокада натрієвих каналів

Пригнічення активності дигідрофолатредуктази

Блокада гістамінових рецепторів

Блокада лсйкотрієнових рецепторів

4412 / 6854
Тривале вживання великих доз аспірину викликає пригнічення синтезу простагландинів в результаті зниження активності фермента:

Пероксидази

Фосфодіестерази

Фосфоліпази А2

5-ліпоксигенази

Циклооксигенази

4413 / 6854
У дитини, що одужує після кору, развинулася пневмонія, викликана умовно-патогенними бактеріями. Яка найбільш імовірна форма цієї інфекції?

Вторинна інфекція

Реінфекція

Госпітальна інфекція

Персистуюча інфекція

Суперінфекція

4414 / 6854
Клінічне обстеження чоловіка дало можливість установити попередній діагноз: рак шлунка. У шлунковому соці виявлено молочну кислоту. Який тип катаболізму глюкози має місце в ракових клітинах?

Глюконеогенез

Анаеробний гліколіз

Глюкозо-аланіновий цикл

Пентозофосфатний цикл

Аеробний гліколіз

4415 / 6854
Чоловік 62 роки помер від хронічної ниркової недостатності. На секції виявлено: нирки зменшені, щільної консистенції, фіброзна капсула знімається важко, поверхня зерниста, коркова речовина витончена. Гістологічно - в частині клубочків проліферація ендотеліальних та мезангіальних клітин, потовщення мембран капілярів клубочків, деякі клубочки слерозовані та гіалізовані. В стромі нечисленні лімфоїдні інфільтрати. Який імовірний діагноз?

Хронічний гломерулонефрит

Артеріолосклеротичний нефросклероз (первинно зморщена нирка)

Пієлонефрит

Гострий екстракапілярний ексудативний гломерулонефрит

Атеросклеротично зморщена нирка

4416 / 6854
126. Чоловік не може розігнути коліно, у нього відсутня шкірна чутливість на передній поверхні стегна. Неврологічне обстеження виявило ураження нерва. Який із наведених нервів є ураженим?

Сідничного

Стегнового

Статево-стегнового

Заратульного

Верхнього сідничного

4417 / 6854
Із сироватки крові людини виділили п'ять ізоферментних форм лактатдегідрогенази і вивчили їх властивості. Яка властивість доводить, що виділені ізоферментні форми того самого ферменту?

Каталізують ту саму реакцію

Однакова тканинна локалізація

Однакова електрофоретична рухливість

Однакова молекулярна маса

Однакові фізико-хімі’нгі властивості

4418 / 6854
Пацієнту, у якого закритий перелом плечової кістки, накладено гіпсову пов’язку. Наступного дня з’явилися припухлість, синюшність і похолодніння кисті травмованої руки. Про який розлад периферичного кровообігу свідчать ці ознаки?

Венозна гіперемія

Емболія

Ішемія

Тромбоз

Артеріальна гіперемія

4419 / 6854
У пацієнта спостерігається збільшений тонус артеріол за нормальних показників роботи серця. Як це вплине на величину артеріального тиску?

Зросте переважно діастолічний

Зменшиться переважно діастолічний

Зросте переважно систолічний

Зменшиться переважно систолічний

Тиск не зміниться

4420 / 6854
130. У дитини, яка часто хворіє на ангіни та фарингіти, спостерігається збільшення лімфовузлів і селезінки. Зовнішній вигляд характеризується пастозністю та блідістю, м’язова тканина розвинена слабко. У крові виявлено лімфоцитоз. Як називаєгься такий вид діатезу?

Нервово-аріритичний

Геморагічний

Астенічний

Лімфатико-гіпопластичний

Ексудативно-катаральний

4421 / 6854
Під час експерименту треба оцінити рівень збудливості тканини. Для цього доцільно визначити величину:

Потенціалу спокою

Тривалості потенціалу дії

Порогу деполяризації

Амплітуди потенціалу дії

4422 / 6854
132. Під час мікроскопії нирки патоло- тЗанатом виявив, що приблизно у 80% клубочків спостерігається розмноження епітелію зовнішнього листка капсули Шумлянського, що формує «півмісяці». Він дійшов висновку що така картина відповідає:

Екстракапілярному ексудативному гломерулонефриту

Інтракапілярному проліферативному гломерулонефриту

Фібропластичному гломерулонефриту

Інтракапілярному ексудативному гломерулонефриту

Екстракапілярному проліферативному (швидкопрогресуючому) гломерулонефриту

4423 / 6854
Лікар-діетолог радить пацієнту під час лікування перніциозної анемії включити до раціону напівсиру печінку. Наявність якого вітаміну у цьому продукті стимулює процес кровотворення?

В2

В1

C

В12

Н

4424 / 6854
Під час обстеження у пацієнта спостерігається гіперглікемія, кетонурія, поліурія, гіперстенурія та глюкозурія. Яка форма порушення кислотно-лужної рівноваги спостерігається в цій ситуації?

Метаболічний алкалоз

Змішаний алкалоз

Метаболічний ацидоз

Газовий ацидоз

Негазовий алкалоз

4425 / 6854
У чоловіка, хворого на тимому (пухлину тимусу), спостерігаються ціаноз, розширення підшкірної венозної сітки і набряк м яких тканин обличчя ши верхньої половини тулуба і верхніх кініцнок. Який венозний стовбур непеснено пухлиною?

Передня яремна вена

Верхня порожниста вена

Внутрішня яремна вена_

Підключична вена

Зовнішня яремна вена

4426 / 6854
У гістологічному препараті паренхіма органа представлена часточками, які мають форму шестигранних призм і складаються з анастомозуючих пластинок, між якими лежать синусоїдні капіляри. що радіально сходяться до центральної вени. Який анатомічний орган має таку морфологічну будову?

Тймус

Печінка

Лімфатичний вузол

Підшлункова залоза

Селезінка

4427 / 6854
У клітину проник вірус грипу. Трансляція під час біосинтезу вірусного білка в клітині буде здійснюватися:

У ядрі

У лізосомах

На каналах гладенької ендоплазматичної сітки

На полірибосомах

У клітинному центрі

4428 / 6854
У чоловіка на 2-3-й де нь після операції ушивання кровоточивої виразки шлунка виявились ознаки атонії кишечника та затримка сечовиділення. Виберіть препарат для усунення ускладнення:

Гістамін

Атропін

Прозерин

Папаверин

Пілокарпін

4429 / 6854
Юнак 17 років страждає на фурункульоз, викликаний умовно-патогенним Staphylococcus epidermidis. Яке дослідження найдоцільніше провести, щоб правильно вибрати препарат для лікування цього хворого?

Визначити фаговар

Виявити фактори патогенності

Скласти антибіотикограму

Дослідити біохімічні властивості

Визначити антигенні властивості

4430 / 6854
У 30-ти річного чоловіка, який хворів на гостре респіраторне захворювання та загинув при явищах гострої легенево-серцевої недостатності, під час розтину знайдено фібринозно-геморагічне запалення в слизовій оболонці гортані та трахеї, деструктивний панбронхит, збільшені легені, яки мають пістрявий вигляд за рахунок абсцесів, крововиливів, некрозу. Який з перелічених діагнозів найбільш вірогідний?

Респіраторно-синцитіальна інфекція

Кір

Грип

Аденовірусна інфекція

Парагрип

4431 / 6854
У пацієнта 18 років при лабораторному обстеженні виявлено наявність глюкози в сечі при нормальній концентрації її в плазмі крові. Найвірогіднішою причиною цього є порушення:

Канальцсвої секреції

Клубочкової фільтрації

Секреції інсуліну

Секреції глюкокортикоїдів

Канальцсвої реабсорбції

4432 / 6854
Чоловік, хворий на ішемічну хворобу серця, не повідомив лікаря, що у нього трапляються напади бронхоспазму. Лікар призначив препарат, після приймання якого напади стенокардії порідшали, але почастішали напади бронхоспазму. У кажіть, який препарат був призначений:

Верапаміл

Анаприлін

Нітротросорбід

Атенолол

Дилтіазем

4433 / 6854
Під час мікроскопічного дослідження прямої кишки виявлено великі вогнища некрозу слизової оболонки, некротичні маси просякнуті фібрином, що утворює плівку. Слизова і підслизова оболонки на периферії ділянок некрозу повнокровні, набряклі, з крововиливами і лейкоцитарною інфільтрацією. Яке захворювання можна припустити?

Сальмонельоз

Холера

Амебіаз

Дизентерія

Черевний тиф

4434 / 6854
Молекула зрілої інформаційної РНК має меншу довжину, ніж відповідний ген молекули ДНК. Неінформативні послідовності нуклеотидів про-іРНК видаляються під час нроцесингу. Яку назву мають ці ділянки?

Екзони

Трапскриптони

Кластери

Мутони

Інтрони

4435 / 6854
У людини внаслідок травми мозку сталося вимкнення грудного дихання зі збереженням діафрагмального. У разі якої локалізації травми це може відбутися?

На рівні 1-го шийного сегмента спинного мозку

На рівні 8-го ірудного сегмента спинного мозку

На рівні варолієвого моста

Між шийними та грудними сегментами спинного мозку

На рівні ретикулярної формації стовбура

4436 / 6854
Жінка 25-ти років скаржиться на постійний біль у ділянці серця, задишку під час рухів, загальну слабкість. Об’єктивно: шкіра бліда та холодна, акроціаноз. Ps - 96/хв., АТ - 105/70 мм рт.ст. Межа серця зміщена на 2 см вліво. Перший тон над верхівкою серця послаблений, систолічний шум над верхівкою. Діагностовано недостатність мітрального клапана серця. Чим обумовлене порушення кровообігу?

Збільшенням об'єму судинного русла

Зниженням об’єму крові, що циркулює

Пошкодженням міокарда

Перевантаженням міокарда підвищеним опором відтоку крові

Перевантаженням міокарда збільшеним об’ємом крові

4437 / 6854
У жінки 28 років виявлена позаматкова вагітність, яка ускладнилася розривом маткової труби. У який простір очеревини можливе попадання крові?

Лівий брижовий синус

Прямокишково-матковий

Міхурово-матковий

Міжсигмоподібний синус

Правий брижовий синус

4438 / 6854
Потерпілий обробляв рослини розчином речовини з інсектицидною дією без індивідуальних засобів захисту. За деякий час у нього почалося сильне виділення слини, поту, сліз, біль у животі, пронос. Під час огляду виявлено міоз. Речовина, що викликала отруєння, належить до групи:

Органішіі сполуки хлору

Нітрати

Н-холіноміметики

Солі міді

Антихолінестеразні засоби

4439 / 6854
У жінки 45 років, яка тривалий час хворіє на бронхіальну астму, виник напад задухи. Який патогенетичний механізм має це явище?

Порушення рухомості грудної клітки

Спазм дрібних бронхів

Порушення перфузїї легеневої тканини

Втрата еластичності легеневої тканини

Зниження чутливості дихального центру

4440 / 6854
A patient has arterial hypertension. What long-acting drug from the group of calcium channel blockers should be prescribed?

Pyrroxanum

Octadine

Reserpine

Atenolol

Amlodipine

4441 / 6854
A drycleaner's worker has been found to have hepatic steatosis. This pathology can be caused by the disruption of synthesis of the following substance:

Phosphatidylcholine

Urea

Tristearin

Cholic acid

Phosphatidic acid

4442 / 6854
What condition may develop 15 30 minutes after re-administration of the antigen as a result of the increased level of antibodies, mainly IgE, that are adsorbed on the surface of target cells, namely tissue basophils (mast cells) and blood basophils?

Antibody-dependent cytotoxicity

Serum sickness

Delayed-type hypersensitivity

Immune complex hyperresponsiveness

Anaphylaxis

4443 / 6854
A patient underwent a course of treatment for atherosclerosis. Laboratory tests revealed an increase in the anti atherogenic lipoprotein fraction in the blood plasma. The treatment efficacy increase in: confirmed by the

VLDL

LDL

IDL

HDL

Chylomicrons

4444 / 6854
A patient has been found to have a marked dilatation of saphenous veins in the region of anterior abdominal wall around the navel. This is a symptom of pressure increase in the following vessel:

V. cava superior

V. portae hepatis

V. cava inferior

V. mesenterica superior

V. mesenterica inferior

4445 / 6854
An electron micrograph shows a cell-to-cell adhesion consisting, in each cell, of an attachment plaque The intercellular space is filled with electron-dense substance including transmembrane fibrillar Specify this adhesion: structures.

Synapse

Adherens junction

Tight junction

Nexus

Desmosome

4446 / 6854
A person with the fourth blood group (genotype IAIB) has in erythrocytes both antigen A controlled by allele IA and antigen В controlled by allele IB. This phenomenon is an example of the following gene interation:

Codominance

Polymery

Complementarity

Epistasls

Semidominance

4447 / 6854
A specimen shows an organ covered with the connective tissue capsule with trabeculae radiating inward the organ. There is also cortex containing some lymph nodules, and medullary cords made of lymphoid cells. What organ is under study?

Lymph node

Spleen

Red bone marrow

Tonsils

Thymus

4448 / 6854
One of the factors that cause obesity is the inhibition of fatty acids oxidation due to:

Lack of carbohydrates in the diet

Impaired phospholipid synthesis

Low level of carnitine

Excessive consumption of fatty foods

Choline deficiency

4449 / 6854
The resuscitation unit has admitted a patient in grave condition. It is known that he had mistakenly taken sodium fluoride which blocks cytochrome oxidase. What type of hypoxia developed in the patient?

Hemic

Respiratory

Cardiovascular

Tissue

Hypoxic

4450 / 6854
A 30-yearold patient has dyspnea fits, mostly at night. He has been diagnosed with bronchial asthma. What type of aller reaction according to the Gell-Coombs classification is most likely in this case?

Anaphylactic

Cytotoxic

Stimulating

Immune complex

Delayed-type hypersensitivity

4451 / 6854
Hepatitis В is diagnosed through laboratory tests that determine the presence of HBA-DNA in blood serum of the patient. What reference method is applied for this purpose?

Polymerase chain reaction

ELISA diagnostic method

Ligase chain reaction method

Hybridization signal amplification method

Hybridization method

4452 / 6854
A 37-year-old female patient complains of headache, vertigo, troubled sleep, numbness of limbs. For the last 6 years she has been working at the gas-discharge lamp producing factory in the lead processing shop. Blood test findings: low hemoglobin and RBC level, serum iron concentration exceeds the norm by several times. Specify the type of anemia: Minkowsky-Shauffard disease

Correct answer

Metaplastic anemia

Hypoplastic anemia

Iron-deficiency anemia

Iron refractory anemia

4453 / 6854
A patient complains that at the bare mention of the tragic events that once occurred in his life he experiences tachycardia, dyspnea and an abrupt rise of blood pressure. What structures of the CNS are responsible for these cardiorespiratory reactions in this patient?

Cerebral cortex

Lateral hypothalamic nuclei

Quadrigemina of mesencephalon

Cerebellum

Specific thalamic nuclei

4454 / 6854
A 49-year-old man complains of pain in his metatarsophalangeal joints and joint deformation. In blood hyperuricemy can be observed. X ray has revealed metatarsophalangeal joint space narrowing, erosion, periarticular calcification of the both joints, osteoporosis. Microscopy has revealed inflammatory granulomatous reaction surrounding necrotizing masses in the area of the first metatarsophalangeal joint. Choose the most likely diagnosis:

Rheumatoid arthritis

Pyrophosphate arthropathy

Hyperparathyroidism

Gout (podagra)

Urolithiasis

4455 / 6854
A young woman suddenly developed fever up to 39°C accompanied by a strong headache. Examination revealed marked nuchal rigidity. Spinal puncture was performed. Gram-stained of cerebrospinal fluid smear contained many neutrophils and Gram-positive diplococci. What bacteria could be the cause of this disease?

Neisseria meningitidis

Pseudomonas aeruginosa

Haemophilus influenza

Streptococcus pneumonia

Staphylococcus aureus

4456 / 6854
During ventricular systole, the cardiac muscle does not respond to additional stimulation because it is in the phase of:

Hyperexcitability

Relational refractoriness

Absolute refractoriness

Subnormal excitability

There is no correct answer

4457 / 6854
Histologic specimen of a kidney demonstrates cells closely adjoined to the renal corpuscle in the distal convoluted tubule. Their basement membrane is extremely thin and has no folds. These cells sense the changes in sodium content of urine and influence renin secretion occurring in juxtaglomerular cells. Name these cells:

Glomerular capillary endothelial cells

Macula densa cells

Mesangial cells

Podocytes

Juxtaglomerular cells

4458 / 6854
As a result of a continuous chronic encephalopathy, a patient has developed spontaneous motions and a disorder of torso muscle tone. These are the symptoms of the disorder of the following conduction tract:

Tractus corticospinalis

Tractus corticonuclearrs

Tractus rubrospinalis

Tractus spinothalamicus

Tractus tectospinalis

4459 / 6854
Work in a mine is known to cause inhalation of large amounts of coal dust. Inhaled coal dust can be detected in the following pulmonary cells:

Secretory epithelial cells

Alveolar macrophages

Capillary endothelial cells

Pericapillary cells

Respiratory epithelial cells

4460 / 6854
Inherited diseases, such as mucopolysaccharidoses, are manifested in metabolic disorders of connective tissue, bone and joint pathologies. The sign of this disease is the excessive urinary excretion of the following substance:

Lipids

Urea

Glucose

Amino acids

Glycosaminoglycans

4461 / 6854
A patient consulted a physician about chest pain, cough, fever. Roentgenography of lungs revealed eosinophilic infiltrates that were found to contain larvae What kind of helminthiasis are these presentations typical of?

Fascioliasis

Cysticercosis

Trichinosis

Echinococcosis

Ascariasts

4462 / 6854
A patient underwent surgical removal of a cavitary liver lesion 2 cm in diameter. It was revealed that the cavity wall was formed by dense fibrous connective tissue; the cavity contained muddy, thick, yellowish greenish fluid with an unpleasant odor Microscopically, the fluid consisted mainly of polymorphonuclear leukocytes. What pathological process are these morphological changes typical for?

Phlegmon

Acute abscess

Chronic abscess

Empyema

4463 / 6854
Due to the use of poor-quality measles vaccine for preventive vaccination, ej 1-year-old child developed an autoimmune renal injury. The urine was found to contain macromolecular proteins. What process of urine formation was disturbed?

Reabsorption and secretion

Secretion

Secretion and filtration

Filtration

Reabsorption

4464 / 6854
A 41-year-old male patient has a history of recurrent attacks of heartbeats (paroxysms), profuse sweating, headaches. Examination revealed hypertension, hyperglycemia, increased basal metabolic rate, and tachycardia. These clinical presentations are typical for the following adrenal pathology:

Hypofunction of the medulla

Hyperfunction of the adrenal cortex

Primary aldosteronism

Hyperfunction of the medulla

Hypofunction of the adrenal cortex

4465 / 6854
A 12-yearold child has a viral infection complicated by obstructive bronchitis. Bronchospasm can be eliminated by inhalations of a drug from the following pharmacological group:

N - c cholinomimetics

B2-agonists

Analeptics

M-anticholinergics

b2-adrenergic blockers

4466 / 6854
Prolonged treatment of hypothyroidism has caused general dystrophy, dental caries, tachycardia, tremor of extremities. What drug is the cause of these side effects?

L-thyroxin

Parathyreoidinum

Prednisolone

Humulin (Human insulin)

Thyrocalcitonin

4467 / 6854
During pathomorphological kidney investigation of a patient, who for a long time had been suffering from osteomyelitis and died from progressing renal failure, the following was revealed: deposits of homogeneous eosinophilic masses in glomerular mesangium, arterial and arteriolar walls, and stroma, which became red when stained with Congo red. What pathological process is this?

Amyloidosis

Calcinosis

Hyalinosis

Mucoid swelling

Carbohydrate degeneration

4468 / 6854
During cell division, DNA replication occurs by a signal from the cytoplasm, and a certain portion of the DNA helix unwinds and splits into two individual strains. What enzyme facilitates this process?

RNA polymerase

Ligase

DNA polymerase

Restrictase

Helicase

4469 / 6854
During appendectomy a patient had the a. appendicularis figated. This vessel branches from the following artery:

A. colica media

A. ileocolica

A. colica dextra

A. sigmoidea

A. mesenterica inferior

4470 / 6854
Хвора 36-ти років страждає на колагеноз. Збільшення вмісту якого метаболіту найбільш вірогідно буде встановлено у сечі?

Оксипроліну

Уробіліногену

Креатиніну

Індикану

Сечовини

4471 / 6854
Дівчина 19-ти років, хвора на цукровий діабет, чекає на донорську нирку. Яке ускладнення діабету є причиною хронічної ниркової недостатності?

Ретинопатія

Атеросклероз

Макроангіопатія

Мікроангіопатія

Невропатія

4472 / 6854
Під час розтину трупа чоловіка 48-ми років виявлено, що кістковий мозок пласких кісток, діафізів та епіфізів трубчастих кісток соковитий, сіро-червоний або сіро-жовтий гноєподібний (піоїдний кістковий мозок). Селезінка масою - 7 кг. На розрізі вона темно-червоного кольору, з ішемічними інфарктами. Всі лімфатичні вузли збільшені, м’які, сіро-червоного кольору. В печінці жирова дистрофія і лейкемічні інфільтрати. Який найбільш імовірний діагноз?

Гострий лімфоїдний лейкоз

Мієломна хвороба

Хронічний мієлоїдний лейкоз

Гострий мієлоїдний лейкоз

Лімфогранулематоз

4473 / 6854
Дослідників, які вивчають фізіологію серця, виявили, що надмірне розтягнення передсердь спричинює зниження реабсорбції натрію у дситальному звивистому канальці та підвищення швидкості клубочкової фільтрації. Що з наведенного є наймовірнішою причиною фізіологічних змін. виявлених дослідниками?

Ангіотензин

Натрійуретичний гормон

Альдостерон

Ренін

Вазопресин

4474 / 6854
У дитини 5 років відбулася інвазія гельмінтів, що призвело до сенсибілізації організму. Які показники лейкоцитарної формули підтвердять цей процес?

Зменшення базофілів

Збільшення нейтрофілів

Збільшення базофілів

Зменшення еозинофілів

Збільшення еозинофілів

4475 / 6854
Під час бактеріологічного дослідження гнійного матеріалу на МПЛ виросли великі безбарвні слизові колонії, які за 24 години з доступом сонячного світла утворили зелено-блакитний водорозчинний пігмент. Бактеріоскопія виявила грамнегативні палички. Чиста культура цього мікроорганізму має запах жасмину та стійкість до більшості антибіотиків. Культура якого мікроорганізму, ймовірно, виділена?

Proteus vulgaris

Brucella abortus

Yersinia pestis

Pseudomonas aureginosa

Klebsiella osaenae

4476 / 6854
Чоловік 50 років, у якого наявна хвороба Паркінсона, застосовує лікарський засіб, який утворюється в організмі з тирозину і є попередником дофаміну. Пацієнт зазначає зменшення тремтіння кінцівок, збільшення об’єму рухів, поліпшення уваги. Який препарат використовує пацієнт?

Натрію вальпроат

Скополамін

Діазепам

Леводопа

Циклодол

4477 / 6854
Жінка 28 років потрапила до інфекційної лікарні з приводу пожовтіння шкіри, склер, слизових оболонок. Лабораторно встановлено підвищення рівня прямого білірубіну в крові. У сечі виявлені уробі-ліноген і білірубін. Для якого з наведених захворювань характерні такі зміни?

Гемолітична жовтяниця

Паренхіматозна жовтяниця

Інфаркт нирки

Туберкульоз нирки

Механічна жовтяниця

4478 / 6854
У жінки, яка хворіє на ішемічну хворобу серця, розвинувся кардіосклероз, що супроводжується аритмією серця. Назвіть вид аритмії, за якого одночасно порушуються автоматизм і провідність:

Синусова аритмія

Миготлива аритмія

Внутрішньопередсердна блокада

Синоатріальна блокада

Передсердно-шлуночкова блокада

4479 / 6854
Під час розтину тіла чоловіка, померлого від гострої серцево-судинної недостатності, виявлені заповнена кров’ю порожнина серцевої сорочки та розплавлений серцевого м’яза бічної поверхні лівого шлуночка. Яка безпосередня причина смерті?

Кардіогснний шок

Фібриляція шлуночків

Гостра лівошлуночкова недостатність

Розрив хронічної аневризми серця

Гемотомпонада перикарда

4480 / 6854
Чоловікові, у якого підозра на системне захворювання сполучної тканини, зробили біопсію нирок і скелетного м'язу. Гістологічно в тканині нирки і м'язі виявлено поширений фібриноїдний некроз внутрішньої і середньої стінок дрібних артерій з їх нейтрофільною інфільтрацією. Виражена інфільтрація визначається в периваскулярній тканині та адвентиціальній оболонці судин, деякі артеріоли тромбовані. Укажіть найімовірніший діагноз:

Системний червоний вовчак

Гіпертонічна хвороба

Вузликовий периартеріїт

Облітеруючий ендартеріїт

Атеросклероз

4481 / 6854
В експериментальної тварини зроблено двобічну перерізку блукаючих нервів. Що станеться з її диханням?

Стане рідким і глибоким

Стане частим і поверхневим

Зупиниться у фазі видиху

Не зміниться

Зупиниться у фазі вдиху

4482 / 6854
У жінки, яка має клінічні ознаки імунодефіциту і незмінену кількість та функціональну активність Т- і В-лімфоцитів, під час обстеження виявлено дефект на молекулярному рівні, через який порушена функція антигенпрезент